Vous êtes sur la page 1sur 534

See discussions, stats, and author profiles for this publication at: https://www.researchgate.

net/publication/279712976

Cours de Mathématiques-Compléments D'Analyse et D'Algèbre

Book · November 2005


DOI: 10.13140/RG.2.1.4635.7606

CITATIONS READS

0 3,998

1 author:

Omran Kouba
Higher Institute for Applied Sciences and Technology
148 PUBLICATIONS   171 CITATIONS   

SEE PROFILE

Some of the authors of this publication are also working on these related projects:

Problems and Solutions View project

Combinatorial Identities View project

All content following this page was uploaded by Omran Kouba on 04 July 2015.

The user has requested enhancement of the downloaded file.


Cours de Mathématiques
Compléments
D’Analyse et D’Algèbre

OMRAN KOUBA

le 11 Octobre 2005
Introduction

Ce livre représente le cours de mathématiques enseigné depuis plusieures années en


troisième année de l’Institut Supérieur des Sciences Appliquées et de Technologie.

Des exercices résolus sont adjoints à chaque chapitre. Bien qu’il soient de difficulté
inégale, je n’ai pas jugé bon de les repérer par des lettres avertissant le lecteur de leur
difficulté croissante. En principe, les plus faciles sont en tête de chaque série.

La partie Algèbre constituée des trois premiers chapitres est généralement traitée
simultanément avec les trois premiers chapitres de la partie Analyse constituée des
dix chapitres suivants.

Enfin j’espère que cette version éléctronique de ce livre offre à nos étudiants un bon
outil de travail et un référence profitable.

Finalement, les exercices résolus sont publiés par l’auteur en 1995 dans les éditions
Marketing, sous le titre
“263 Exercices Résolus de Mathématiques En Spéciales”
Omran KOUBA
le 11 Octobre 2005
TABLE DES MATIÈRES

Algèbre
Dualité
I. Espace dual d’un espace vectoriel
II. Hyperplans
III. Transposition
IV. Dualité en dimension finie
Exercices
Solutions
Réduction des endomorphismes
I. Généralités
II. Endomorphismes diagonalisables
III. Endomorphismes trigonalisables
IV. Polynômes d’endomorphismes
Exercices
Solutions
Espaces préhilbertiens
I. Produit scalaire
II. Orthogonalité
III. Projections orthogonales
IV. Formes linéaires et adjoints
V. Applications linéaires orthogonales
VI. Le groupe orthogonal
VII. Reduction des endomorphismes symétriques
VIII. Exemples d’application
Exercices
Solutions

Analyse
Séries numériques
I. Limite inférieure et limite supérieure d’une suite numérique
II. Généralités
III. Séries à termes positifs
IV. Séries absolument convergentes, Séries semi-convergentes
V. Produit de deux séries
VI. Expressions asymptotiques liés aux séries numériques
Exercices
Solutions
iv

Intégrales généralisées
I. Généralités
II. Comparaison des convergence d’une série et d’une intégrale
III. Exemples d’application
Exercices
Solutions
Espaces vectoriels normés
I. Généralités
II. Voisinages, ouverts, fermés
III. Intérieur, adhérence et frontière d’une partie
IV. Limites et continuité
V. Les suites dans un espace vectoriel normé
VI. Les parties compactes dans un espace vectoriel normé
VII. Applications linéaires continues
VIII. Applications multilinéaires continues
IX. Les espaces vectoriels normés de dimension finie
Exercices
Solutions
Suites et séries de fonctions
I. Généralités
II. Continuité de la limite d’une suite d’applications
III. Intégrabilité et dérivabilité de la limite d’une suite d’applications
IV. séries de fonctions
V. Continuité, intégrabilité et dérivabilité de la somme
Exercices
Solutions
Intégrales dépendant d’un paramètre
Exercices
Solutions
Séries entières
I. Généralités
II. Propriétés de la somme d’une série entière
III. Fonctions développables en série entière
IV. Développement en série entière des fonctions usuelles
V. La fonction exponentielle complexe et sesapplications
Exercices
Solutions
v

Séries de Fourier
I. L’espace R2π
II. Coefficients et séries de Fourier
III. Propriétés des coefficients de Fourier
IV. Convergence ponctuelle des séries de Fourier
V. Convergence au sens de Cesàro des séries de Fourier
VI. Convergence en moyenne quadratique des séries de Fourier
VII. Applications
Exercices
Solutions
Équations différentielle linéaires
I. Généralités
II. La résolvante
III. Le wronskien
IV. Équations différentielles linéaires scalaires d’ordre n
V. Systèmes différentiels linéaires à coefficients constants
VI. Équations différentielles.“Prélude à la théorie générale”
Exercices
Solutions
Fonctions de plusieures variables
I. Continuité et limites
II. Différentiabilité
III. Dérivées partielles
IV. Inégalité des accroissements finis, et théoème de Taylor
V. Recherche des extremums d’une fonction numérique
VI. Théorème des fonctions implicites
VII. Formes différentielles du premier degré
Exercices
Solutions
Surfaces
I. Généralités
II. Surfaces cylindriques
III. Surfaces coniques
IV. Surfaces de révolution
V. Quadriques
VI. “Prélude” à la théorie générale des surfaces
Exercices
Solutions
DUALITÉ

I. Espace dual d’un espace vectoriel

Soit E un IK-espace vectoriel, supposé différent de {0}. On appelle forme linéaire


sur E, toute application linéaire de E dans IK. On note E ∗ l’espace vectoriel L(E, IK)
des formes linéaires sur E. Enfin, si f ∈ E ∗ et x ∈ E, on note hf, xi pour désigner f (x),
et on appelle h , i “crochet de dualité”.

Remarque : Une forme linéaire non nulle sur E est nécessairement surjective.

Définition :
♣ Pour x ∈ E, on définit

x⊥ = {y ∈ E ∗ : hy, xi = 0} ,

c’est un sous-espace vectoriel de E ∗ , que l’on appelle l’orthogonal de x dans E ∗ .


♣ Pour A, une partie non vide de E, on définit

A⊥ = {y ∈ E ∗ : ∀ x ∈ A, hy, xi = 0} ,
\
donc A⊥ = x⊥ , c’est aussi un sous-espace vectoriel de E ∗ , que l’on appelle
x∈A
l’orthogonal de A dans E ∗ .
♣ Pour y ∈ E ∗ , on définit de même

y 0 = {x ∈ E : hy, xi = 0} = Ker y,

c’est donc un sous-espace vectoriel de E, que l’on appelle le préorthogonal de y


dans E.
♣ Pour B, une partie non vide de E ∗ , on définit

B 0 = {x ∈ E : ∀ y ∈ B, hy, xi = 0} ,
\
donc B 0 = Ker y, c’est aussi un sous-espace vectoriel de E, que l’on appelle le
y∈B
préorthogonal de A dans E.
2 Dualité

♣ Enfin, Soient A une partie non vide de E, et B une partie non vide de E ∗ . On dit
que A et B sont orthogonales si, et seulement si,

∀ x ∈ A, ∀ y ∈ B, hy, xi = 0.

Remarque : Évidemment la forme linéaire nulle 0 et l’unique forme linéaire nulle sur
E, donc E ⊥ = {0}. Un résultat similaire affirme que (E ∗ )0 = {0}, la démonstration de
ceci lorsque E est de dimension infinie nécessite l’axiome du choix. Nous verrons une
démonstration élémentaire dans le cas où E est de dimension finie.

Proposition I.1. Soit E un IK-espace vectoriel, supposé différent de {0}.


1◦ . Soient A, B deux parties non vides de E telles que A ⊂ B alors B ⊥ ⊂ A⊥ .
2◦ . Soit A une partie non vide de E, alors A⊥ = ( Vect (A))⊥ et A ⊂ (A⊥ )0 .

La démonstration de ce résultat est simple et laissée en exercice au lecteur.


Voici une version “duale” de la proposition précédente.

Proposition I.2. Soit E un IK-espace vectoriel, supposé différent de {0}.


1◦ . Soient A, B deux parties non vides de E ∗ telles que A ⊂ B alors B 0 ⊂ A0 .
2◦ . Soit A une partie non vide de E ∗ , alors A0 = ( Vect (A))0 et A ⊂ (A0 )⊥ .

II. Hyperplans

Définition : Soit E un IK-espace vectoriel, supposé différent de {0}. Un sous-espace


vectoriel H de E est un hyperplan si, et seulement si, codim H = 1.

Théorème II.1. Soient E un IK-espace vectoriel, supposé différent de {0}, et H un


sous-espace vectoriel H de E. Les assertions suivantes sont équivalentes:
1◦ . H est un hyperplan.
2◦ . H est le noyau d’une forme linéaire non nulle.
3◦ . Il existe une droite D de E telle que E = H ⊕ D.
Transposition 3

Preuve : (1◦ =⇒ 2◦ ) Comme dim E/H = dim IK = 1, il existe un isomorphisme


θ : E/H −→ IK. Soit Q : E −→ E/H la surjection canonique, alors θ◦Q ∈ E ∗ et
Ker (θ◦Q) = H.
(2◦ =⇒ 3◦ ) Supposons que H = Ker y avec y ∈ E ∗ \ {0}. La forme linéaire y est
surjective, donc il existe b ∈ E tel que hy, bi = 1. On pose alors D = IK.b.
– Si x ∈ D ∩ H, alors x = λb et hy, xi = 0. Ce qui montre que λ = 0 et par
conséquent x = 0. On conclut que D ∩ H = {0}.
– Si x ∈ E alors
x = hy, xib + x − hy, xib .
| {z } | {z }
∈D ∈H

Alors E = H ⊕ D.
(3◦ =⇒ 1◦ ) Soit Q : E −→ E/H, la surjection canonique. Notons ϕ : D −→ E/H
la restriction de Q à D. (i.e. ϕ = Q|D ). Il est clair que Ker ϕ = D ∩ Ker Q = {0},
donc ϕ est injective. D’autre part, si x ∈ E, alors x = d + h avec d ∈ D et h ∈ H, et
par conséquent [x] = Q(x) = Q(d) = ϕ(d). Ce qui démontre que ϕ est surjective. On
conclut que ϕ est un isomorphisme, en particulier codim H = dim E/H = dim D = 1.

Proposition II.2. Soient E un IK-espace vectoriel, supposé différent de {0}, et H un


hyperplan de E. ALors H ⊥ est une droite vectoriel de E ∗ .

Preuve : Supposons que H = Ker y avec y ∈ E ∗ \{0}. La forme linéaire y est surjective,
donc il existe a ∈ E tel que hy, ai = 1. Clairement y ∈ H ⊥ donc IKy ⊂ H ⊥ .
Inversement, soit z ∈ H ⊥ . Pour tout x ∈ E nous avons

x = hy, xia + x − hy, xia


| {z }
∈H

et donc hz, xi = z(a)hy, xi. Il en résulte que z = z(a)y ∈ IKy. D’où H ⊥ = IKy.

Remarque : Si y ∈ E ∗ \ {0} et H = Ker y, alors l’équation hy, xi = 0 est dite


l’équation de l’hyperplan H. (Car hy, xi = 0 ⇐⇒ x ∈ H).

III. Transposition
Soient E et F deux IK-espaces vectoriels, et u ∈ L(E, F ), une application linéaire
de E dans F . On définit l’application t u : F ∗ −→ E ∗ par ∀ y ∈ F ∗ , t u(y) = y ◦u. Il est
4 Dualité

immédiat de vérifier que t u ∈ L(F ∗ , E ∗ ). L’application t u s’appelle la transposée de u.


Remarquons que t u est définie par

∀ x ∈ E, ∀ y ∈ F ∗ , ht u(y), xi = hy, u(x)i.

Voici quelques propriétés importantes de la transposition:


Propriétés :
♣ L’application u 7→ t u de L(E, F ) dans L(F ∗ , E ∗ ) est linéaire et injective. C’est
immédiat.
♣ Pour tout u ∈ L(E, F ) et v ∈ L(F, G), on a t (v ◦u) = t u◦t v.
En effet, pour tout x ∈ E et tout z ∈ G∗

ht (v ◦u)(z), xi =hz, v ◦u(x)i = hz, v(u(x))i


=ht v(z), u(x)i = ht u(t v(z)), xi = ht u◦t v(z), xi.

♣ Si IE (resp. IE ∗ ) est l’application identique de E (resp. de E ∗ ), alors t IE = IE ∗ .


Évident.
♣ Si u ∈ L(E, F ) est bijective, alors t u est bijective et (t u)−1 = t (u−1 ). En effet
comme u◦u−1 = IF et u−1 ◦u = IE on a t u◦t (u−1 ) = IE ∗ et t (u−1 )◦t u = IF ∗ . Donc
t
u est bijective et (t u)−1 = t (u−1 ).
♣ Soit u ∈ L(E, F ). ALors Ker (t u) = ( Im u)⊥ . En effet,

f ∈ Ker t u ⇐⇒ f ◦u = 0 ⇐⇒ ∀ x ∈ E, hf, u(x)i = 0


⇐⇒ ∀ y ∈ Im u, hf, yi = 0 ⇐⇒ f ∈ ( Im u)⊥ .

En particulier, si u est surjective, alors t u est injective. La réciproque est aussi


vraie.

IV. Dualité en dimension finie

Dans ce paragraphe E désigne un IK-espace vectoriel de dimension finie.


1◦ . L’espace dual E ∗ est de dimension finie et dim E ∗ = dim L(E, IK) = dim E.
2◦ . (Base duale) Soit E = (e1 , . . . , en ) une base de E. On définit les éléments
E ∗ = (e∗1 , . . . , e∗n ) de E ∗ par

1 si i=j
e∗i (ej ) = he∗i , ej i = δij =

0 si i 6= j
Dualité en dimension finie 5

E ∗ est une base de E ∗ dite la base duale de E. Nous avons


n
X
∀ f ∈ E∗, f = hf, ei i e∗i
i=1
Xn
∀x ∈ E , x= he∗i , xi ei
i=1

3◦ . Soit x ∈ E \ {0}, il existe f ∈ E ∗ tel que f (x) = 1. Il suffit de compléter {x}


en une base de E et de considérer ensuite la base duale.

Théorème IV.1. Soit E un IK-espace vectoriel de dimension finie. L’application

Ψ : E −→ E ∗∗ , x 7→ Ψ(x),

définie par hΨ(x), f i = hf, xi, pour tout (x, f ) ∈ E × E ∗ , est un isomprphisme.

Preuve : En effet, la linéarité de Ψ est immédiate. Le noyau de Ψ est (E ∗ )0 qui est


réduit à {0} d’après troisième point ci-dessus. Enfin dim E = dim E ∗∗ . On conclut que
Ψ est bien un isomorphisme.

Corollaire IV.2. Soient E un IK-espace vectoriel de dimension finie, et F =


(f1 , . . . , fn ) une base de E ∗ . Alors il existe une base E = (e1 , . . . , en ) de E dont la
base duale est F.

Preuve : Considérons dans E ∗∗ la base duale F ∗ = (f1∗ , . . . , fn∗ ) de F. Alors, pour tout
(i, j) ∈ {1, . . . , n}2 , nous avons hfj∗ , fi i = δij . On pose, pour 1 ≤ j ≤ n, ej = Ψ−1 (fj∗ ),
avec Ψ étant l’isomorphisme canonique de E sur E ∗∗ du théorème précédent. Il en
résulte que, pour tout (i, j) ∈ {1, . . . , n}2 ,

hfi , ej i = hΨ(ej ), fi i = hfj∗ , fi i = δij .

On conclut que E = (e1 , . . . , en ) est une base de E dont la base duale est F.

Théorème IV.3. Soit E un IK-espace vectoriel de dimension finie,


1◦ . Pour tout sous-espace vectoriel F de E on a

dim F + dim F ⊥ = dim E et F = (F ⊥ )0 .

2◦ . Pour tout sous-espace vectoriel G de E ∗ on a

dim G + dim G0 = dim E et G = (G0 )⊥ .


6 Dualité

Preuve : – Soit (e1 , . . . , ep ) une base de F , et complétons la en une base E = (e1 , . . . , en )


de E. Soit ensuite E ∗ = (e∗1 , . . . , e∗n ) la base duale de E. Pour tout f ∈ E ∗ on a
f ∈ F ⊥ ⇐⇒ ∀ i ∈ {1, . . . , p}, hf, ei i = 0
Xn
⇐⇒ f = hf, ei i e∗i
i=p+1

⇐⇒ f ∈ Vect (e∗p+1 , . . . , e∗n ).


Alors, F ⊥ = Vect (e∗p+1 , . . . , e∗n ) et dim F ⊥ = n − dim F .
– Soit E ∗ = (e∗1 , . . . , e∗n ) une base de E ∗ obtenue en complétant une base (e∗1 , . . . , e∗p )
de G. Considérons (e1 , . . . , en ) la base de E dont E ∗ est la base duale. Pour tout x ∈ E
on a
x ∈ G0 ⇐⇒ ∀ i ∈ {1, . . . , p}, he∗i , xi = 0
Xn
⇐⇒ x = he∗i , xi ei
i=p+1

⇐⇒ x ∈ Vect (ep+1 , . . . , en ).
Alors, G0 = Vect (ep+1 , . . . , en ) et dim G0 = n − dim G.
– Il est immédiat que F ⊂ (F ⊥ )0 et

dim(F ⊥ )0 = n − dim F ⊥ = n − (n − dim F ) = dim F

D’où F = (F ⊥ )0 , et on démontre de même G = (G0 )⊥ .

Théorème IV.4. Soient E et F deux IK-espaces vctoriels de dimension finie, et


u ∈ L(E, F ). Alors rg u = rg t u.
Preuve : En effet,

rg t u = dim Im t u = dim F ∗ − dim Ker t u (mais Ker t u = ( Im u)⊥ )


= dim F ∗ − dim( Im u)⊥ = dim Im u = rg u.

Théorème IV.5. Soient E et F deux IK-espaces vctoriels de dimension finie. On


considère E = (e1 , . . . , ep ) une base de E avec E ∗ = (e∗1 , . . . , e∗p ) la base duale
correspondante, et de même F = (f1 , . . . , fn ) une base de F avec F ∗ = (f1∗ , . . . , fn∗ )
la base duale correspondante. Si u ∈ L(E, F ). Alors t Mat(u, E, F) = Mat(t u, F ∗ , E ∗ ).

La preuve est tout simplement la relation ht u(fj∗ ), ei i = hfj∗ , u(ei )i.

Corollaire IV.6. Si M est une matrice n × p à coefficients dans le corps IK, alors
rg M = rg t M .
C’est immédiat.
Exercices 7

EXERCICES

Exercice .1 Soient E et F deux IK-espaces vectoriels, et soit u ∈ L(E, F ) une


application linéaire surjective, montrer que t u est injective.

Exercice .2 Soient E et F deux IK-espaces vectoriels de dimensions finies, et soit


u ∈ L(E, F ). Montrer: Ker (t u) = ( Im (u))⊥ , et Im (t u) = ( Ker (u))⊥ .

Exercice .3 Soit E un IK-espace vectoriel de dimension finie, et V1 , V2 (resp. W1 ,


W2 ) deux sous-espaces vectoriels de E (resp. de E ∗ ). Comparer

(V1 + V2 )⊥ et V1⊥ ∩ V2⊥ , (V1 ∩ V2 )⊥ et V1⊥ + V2⊥

(W1 + W2 )o et W1o ∩ W2o , (W1 ∩ W2 )o et W1o + W2o

Exercice .4 Soit E un ensemble non vide muni d’une loi interne notée +, et d’une
loi externe IK × E −→ E : (λ, x) 7→ λ>x. On note F l’ensemble des ϕ ∈ F(E, IK) telles
que

∀ (x, y) ∈ E 2 , ∀ λ ∈ IK ϕ(x + y) = ϕ(x) + ϕ(y) et ϕ(λ>x) = λϕ(x)

On suppose que

∀ (x, y) ∈ E 2 , x 6= y =⇒ ∃ ϕ ∈ F : ϕ(x) 6= ϕ(y)

Montrer que (E, +, >) est un IK-espace vectoriel.

Exercice .5 Soient E un IK-espace vectoriel, (f, g) ∈ E ∗2 ; on suppose que pour


tout x dans E on a f (x)g(x) = 0. Montrer que f = 0 ou g = 0.

Exercice .6 Soient E un IK-espace vectoriel, F, G deux sous-espaces supplémentaires


dans E, i : F −→ E l’injection canonique, p : E −→ G la projection sur G parallèlement
à F . Montrer que F ⊥ ⊕ G⊥ = E ∗ et Im(t p) = Ker(t i).
8 DUALITÉ

Exercice .7 Soit E un IK-espace vectoriel. Montrer les deux propriétés suivantes:


(Pk ) Soit {`1 , . . . , `k } une famille libre dans E ∗ . Alors il existe {x1 , . . . , xk } dans
E tel que pour tout i et tout j l’on ait `i (xj ) = δij ,( δij étant le symbole de
Kronicker).
(Qk ) Soit {`1 , . . . , `k } une famille libre dans E ∗ , et soit ` ∈ E ∗ telle que
\k k
X
Ker`i ⊂ Ker `. Alors il existe λ1 , . . . , λk dans IK tels que ` = λi `i .
i=1 i=1
En déduire, que pour toute famille {`, `1 , . . . , `k } de E ∗ , il y a équivalence entre
k
\
a: Ker`i ⊂ Ker`, et b : ` ∈ Vect ({`1 , . . . , `k }).
i=1

Exercice .8 Soit E un IK-espace vectoriel de dimension finie, et soit {`1 , . . . , `k }


une famille libre dans E ∗ . Montrer que
à k
!
\
dim Ker`i = dim E − k.
i=1

Exercice .9 Soient E et F des IK-espaces vectoriels de dimensions finies, V (resp.


W ) un sous espace de E (resp. F ). On considère l’espace

LV,W (E, F ) = {u ∈ L(E, F ) | V ⊂ Ker u, et Im u ⊂ W }

Montrer qu’il s’agit d’un sous-espace de L(E, F ) isomorphe à L(E/V, W ). En déduire


sa dimension.

Exercice .10 Soit E le sous-ensemble de Mn (IR) formé des matrices M = (aij )


vérifiant
n
X n
X
∃s ∈ IR : ∀ i, aij = s; ∀ j, aij = s.
j=1 i=1

et on désigne s par δ(M ).


1◦ . Montrer que E est un sous-espace vectoriel de Mn (IR), et que δ est une forme
linéaire sur E.
2◦ . Montrer que le produit de deux éléments de E est un élément de E.
3◦ . Soit J la matrice de Mn (IR) dont tous les coefficients valent 1. Montrer que

A ∈ E ⇐⇒ ∃ s : AJ = JA = sJ.

4◦ . Montrer que si A ∈ E est inversible alors A−1 ∈ E.


Exercices 9

5◦ . Montrer que E = Kerδ ⊕ IR.J, et calculer dim E.


6◦ . Soit Ee le sous-ensemble de Mn (IR) formé des matrices M = (aij ) vérifiant
n
X n
X
∃s ∈ IR : ∀ i, aij = s; ∀ j, aij = s;
j=1 i=1
n
X n
X
aii = s; ai,n+1−i = s.
i=1 i=1

Montrer que Ee est un sous espace vectoriel de Mn (IR). Calculer sa dimension.

Exercice .11 Soient E = IR2 [X] l’espace vectoriel des polynômes de degré inférieur
ou égal à 2, à coefficients réels, ϕ1 , ϕ2 , et ϕ3 les éléments de E ∗ définis par
Z 1
0
ϕ1 (P ) = P (1), ϕ2 (P ) = P (1), ϕ3 (P ) = P (t) dt.
0

Montrer que Φ = (ϕ1 , ϕ2 , ϕ3 ) est une base de E ∗ et déterminer une base de E dont Φ
est la base duale.

Exercice .12 Soient E = IR3 [X] l’espace vectoriel des polynômes de degré inférieur
ou égal à 3, à coefficients réels, ϕ1 , ϕ2 , ϕ3 , et ϕ4 les éléments de E ∗ définis par

ϕ1 (P ) = P (0), , ϕ2 (P ) = P (1), ϕ3 (P ) = P 0 (0), ϕ4 (P ) = P 0 (1).

Montrer que Φ = (ϕ1 , ϕ2 , ϕ3 , ϕ4 ) est une base de E ∗ et en déterminer une base de


E dont ZΦ est la base duale. Exprimer ensuite la forme linéaire ψ ∈ E ∗ définie par
1
ψ(P ) = P (t) dt sur la base précédente.
0

Exercice .13 Soient E = IRn [X] l’espace vectoriel des polynômes de degré inférieur
ou égal à n, à coefficients réels. Pour a ∈ IR, on pose

ϕa : E −→ IR : P 7→ P (a).

1◦ . Vérifier que pout tout a ∈ IR, ϕa ∈ E ∗ .


2◦ . a. Soient x0 , x1 , . . . , xn des élements deux à deux distincts de IR. Montrer que
(ϕx0 , ϕx1 , . . . , ϕxn ) est une base de E ∗ , et en déterminer la base duale.
b. Montrer qu’il existe une et une seule suite finie (λi )0≤i≤n ∈ IRn+1 telle que
Z 1 n
X
∀ P ∈ E, P (x) dx = λi P (xi ).
0 i=0
10 DUALITÉ

Exercice .14 Soit E = IRn [X] l’espace vectoriel des polynômes réels de degré
inférieur ou égal à n.
1◦ . On pose ek (X) = (X + a)k , Trouver la base duale {e∗k }0≤k≤n de la base {ek }0≤k≤n
R1
de E. Exprimer la forme linéaire ϕ : P 7→ P (t) dt sur cette base.
0
2◦ . On note ∆ l’opérateur de différence: ∆(P )(X) = P (X + 1) − P (X). Soit ϕk ∈ E ∗
défini par ϕk (P ) = ∆k (P )(0). Montrer que {ϕk }0≤k≤n est une base de E ∗ , et
trouver la base de E dont elle est la base duale.

Exercice .15 On définit la suite de polynômes (Pn )n≥0 de IR[X] par

1
P0 (X) = 1, P1 (X) = X, Pn (X) = X(X − n)n−1 pour n ≥ 2.
n!

1◦ . Vérifier que pour tout n ≥ 1, Pn0 (X) = Pn−1 (X − 1). En déduire que

∀ k ∈ IN, ∀ n ≥ k, Pn(k) (X) = Pn−k (X − k)

(Ici P (k) désigne la dérivée d’ordre k de P . On convient que P (0) = P ).


2◦ . Pour k ∈ IN, on définit la forme linéaire ϕk sur IR[X] par ϕk (P ) = P (k) (k). Calculer
ϕk (Pn ).
Dans la suite m désigne un entier supérieur à zéro donné. E = IRm [X] l’espace
vectoriel des polynômes de degré inférieur ou égal à m.
3◦ . Montrer que (P0 , P1 , . . . , Pm ) forme une base de E. Donner sa base Duale.
Xm

4 . Montrer que ∀ Q ∈ E, Q(X) = Q(k) (k)Pk (X).
k=0
m
X

5 . En déduire que ∀ a ∈ IR, Pm (X + a) = Pm−k (a)Pk (X).
k=0

Exercice .16
1◦ . Soient n ∈ IN∗ et T : Mn (IR) −→ (Mn (IR))∗ qui associe à chaque matrice
M ∈ Mn (IR), la forme linéaire TM (X) = Tr (M X) sur Mn (IR). Démontrer que T
est un isomorphisme d’espaces vectoriels.
2◦ . Soit ϕ ∈ (Mn (IR))∗ telle que ϕ(M N ) = ϕ(N M ) pour toutes les matrices M, N de
Mn (IR). On note M0 la matrice telle que ϕ = TM0 . Démontrer que

∀ M ∈ Mn (IR), M0 M = M M0 .

En déduire que ϕ = λTr, pour un certain λ ∈ IR.


3◦ . Trouver la dimension de vect {M N − N M | M, N ∈ Mn (IR)}.
Solutions 11

SOLUTIONS

Solution .1 Soit f ∈ Ker t u, Alors ∀ x ∈ E, 0 = ht u(f ), xi = hf, u(x)i. La forme


linéaire f s’annule donc sur l’image de u qui est F , car u est surjective, d’où f = 0.

Solution .2 Notons que

f ∈ Ker t u ⇐⇒ t u(f ) = 0
⇐⇒ ∀ x ∈ E, ht u(f ), xi = 0
⇐⇒ ∀ x ∈ E, hf, u(x)i = 0
⇐⇒ ∀ y ∈ Im u, hf, yi = 0
⇐⇒ f ∈ (Im u)⊥

donc Ker t u = (Im u)⊥ . D’autre part, si e = t u(f ) ∈ Im t u, et si x ∈ Ker u alors

he, xi = ht u(f ), xi = hf, u(x)i = hf, 0i = 0

il en résulte que Im t u ⊂ (Ker u)⊥ . Mais

dim Im t u = dim F ∗ − dim Ker t u


= dim F ∗ − dim(Im u)⊥
= dim F − (dim F − dim Im u)
= dim E − dim Ker u
= dim(Ker u)⊥

d’où Im t u = (Ker u)⊥ .

Solution .3 1◦ . Notons que, pour i ∈ {1, 2}, Vi ⊂ V1 + V2 , alors (V1 + V2 )⊥ est inclu
à la fois dans V1⊥ et dans V2⊥ , d’où (V1 + V2 )⊥ ⊂ V1⊥ ∩ V2⊥ . Inversement, toute forme
linéaire s’annulant sur V1 et sur V2 s’annule sur l’espace vectoriel engendré par V1 + V2 ,
d’où V1⊥ ∩ V2⊥ ⊂ (V1 + V2 )⊥ . On a donc

V1⊥ ∩ V2⊥ = (V1 + V2 )⊥ .

2◦ . Notons de même que, V1 ∩ V2 ⊂ Vi pour i ∈ {1, 2}. Alors V1⊥ et V2⊥ sont deux
sous-espaces vectoriels de (V1 ∩ V2 )⊥ , d’où V1⊥ + V2⊥ ⊂ (V1 ∩ V2 )⊥ . D’autre part, si
12 Dualité

n = dim E, on a

dim(V1⊥ + V2⊥ ) = dim V1⊥ + dim V2⊥ − dim(V1⊥ ∩ V2⊥ )


= dim V1⊥ + dim V2⊥ − dim(V1 + V2 )⊥
=n − dim V1 − dim V2 + dim(V1 + V2 )
=n − dim(V1 ∩ V2 )
= dim(V1 ∩ V2 )⊥

On conclut que V1⊥ + V2⊥ = (V1 ∩ V2 )⊥ .


3◦ . On démontre de même que

(W1 + W2 )o = W1o ∩ W2o , et (W1 ∩ W2 )o = W1o + W2o .

Solution .4 L’idée importante est de remarquer que, pour un couple (x, y) ∈ E ×E,

(∀ ϕ ∈ F, ϕ(x) = ϕ(y)) =⇒ x = y. (∗)

Soit (x, y) ∈ E × E, comme

∀ ϕ ∈ F, ϕ(x + y) = ϕ(x) + ϕ(y) = ϕ(y + x)

alors x + y = y + x. La loi + est commutative.


Pour x ∈ E on pose 0x = x + (−1)>x. Soit (x, y) ∈ E × E, comme

∀ ϕ ∈ F, ϕ(0x ) = ϕ(x) + ϕ((−1)>x) = ϕ(x) − ϕ(x) = 0 = ϕ(0y )

alors 0x = 0y . Donc 0x ne dépend pas de x, on note 0 cet élément de E, il vérifie


ϕ(0) = 0. On a, pour x ∈ E,

∀ ϕ ∈ F, ϕ(x + 0) = ϕ(x) + ϕ(0) = ϕ(x)

alors x + 0 = x. La loi + admet un élément neutre.


Pour x ∈ E, on note −x = (−1)>x. D’où

∀ ϕ ∈ F, ϕ(x + (−x)) = ϕ(x) + ϕ(−x) = 0 = ϕ(0)

alors x + (−x) = 0. On conclut que (E, +) est un groupe abelien.


En notant que toute ϕ ∈ F prend la même valeur en x et en 1>x, en (λ + µ)>x
et en λ>x + µ>x, en λ>(x + y) et en λ>x + λ>y, et enfin, en λ>(µ>x) et en (λµ)>x,
on montre que (E, +, >) est un IK-espace vectoriel.
Solutions 13

Solution .5 Posons Q(x) = f (x)g(x). La linéarité de f et de g montre que

Q(x + y) − Q(x) − Q(y)


∀ (x, y) ∈ E × E, f (x)g(y) = .
2
L’hypothèse montre alors, que ∀ (x, y) ∈ E × E, f (x)g(y) = 0. Si f 6= 0 il existe
x0 ∈ E tel que f (x0 ) 6= 0 et par conséquent, pour tout y ∈ E, on a g(y) = 0 ; c’est à
dire g = 0.

Solution .6 Si f ∈ F ⊥ ∩ G⊥ , alors f s’annule sur F et sur G donc elle s’annule sur


l’espace vectoriel engendré par F et G i.e. sur E ; alors f = 0. D’où F ⊥ ∩ G⊥ = 0.
Si h ∈ E ∗ , on pose g = h◦(I − p) et f = h◦p. alors f ∈ F ⊥ , g ∈ G⊥ et h = f + g.
On conclut que F ⊥ + G⊥ = E ∗ .
Notons que Ker (t i) = (Im (i))⊥ = F ⊥ . Si f ∈ Im (t p) alors il existe g ∈ G∗ tel
que f = g ◦p et par conséquent f ∈ F ⊥ . Inversement, si f ∈ F ⊥ , on pose g = f|G la
restriction de f à G. Pour tout x ∈ E, on a f (x) = f (x−p(x)+p(x)) = f (p(x)) = g ◦p(x).
D’où f = t p(g) i.e. f ∈ Im (t p). Alors F ⊥ = Im (t p).

Solution .7 Montrons d’abord P1 . Si `1 est un élément non nul de E ∗ alors il est


surjectif donc il existe x1 ∈ E tel que `1 (x1 ) = 1. D’où P1 .
Pk =⇒ Qk . En effet, soient {`1 , . . . , `k } une famille libre de E ∗ , et ` ∈ E ∗ tel
\k
que Ker `i ⊂ Ker `. D’après Pk on trouve {x1 , . . . , xk } une famille de E telle que
i=1
`i (xj ) = δij .
k
X
Soit x ∈ E. On pose y = x − `i (x)xi . On vérifie immédiatement que y ∈
i=1
k
\
Ker `i , et d’après l’hypothèse y ∈ Ker `, ce qui s’écrit
i=1
à k
! k
X X
0=` x− `i (x)xi = `(x) − `(xi )`i (x)
i=1 i=1

k
X
ce qui montre que ` = `(xi )`i . D’où Qk .
i=1
Qk =⇒ Pk+1 . En effet, soit {`1 , . . . , `k+1 } une famille libre de E ∗ .
Pour chaque j ∈ {1, . . . , k + 1} on a `j ∈
/ Vect ({`i , 1 ≤ i ≤ k + 1, i 6= j}). Alors
d’après Qk , il existe x
ej ∈ E tel que
\
x
ej ∈
/ Ker `j , et x
ej ∈ Ker `i .
1≤i≤k+1,i6=j
14 Dualité

On pose alors xj = x
ej /`j (e
xj ), et on vérifie immédiatement que `i (xj ) = δij pour tout i
et j dans {1, . . . , k + 1}.
Ceci démontre par récurrence sur k la vérité de Qk et de Pk .
D’autre part, notons que b. =⇒ a. est trivial. Inversement, soit A une partie de
{1, . . . , k} telle que (`i )i∈A soit libre. (On suppose que les (`i )1≤i≤k ne sont pas tous
nuls auquel cas le résultat est trivial). On a
k
\
Ker `i =( Vect ({`i : 1 ≤ i ≤ k}))o
i=1
=( Vect ({`i : i ∈ A}))o
\
= Ker `i .
i∈A

En utilisant la propriété Q, la proposition a. implique que

` ∈ Vect ({`i : i ∈ A}) = Vect ({`i : 1 ≤ i ≤ k})

Solution .8 Soit G = Vect ({`1 , . . . , `k }). On a


k
\
Go = Ker `i .
i=1

alors, dim Go = dim E − dim G ce qui donne le résultat.

Solution .9 La vérification que LV,W (E, F ) est un sous-espace vectoriel de L(E, F )


est facile. Notons i : W −→ F l’injection canonique, et q : E −→ E/V la surjection
canonique. Si u
e ∈ L(E/V, W ) alors il est immédiat que u = i◦u
e◦q appartient à
LV,W (E, F ). On considère alors l’application

ϕ : L(E/V, W ) −→ LV,W (E, F ) : u


e 7→ ϕ(e
u) = i◦u
e◦q,

il est clair que ϕ est une application linéaire injective. Montrons que ϕ est aussi
surjective. En effet soit u ∈ LV,W (E, F ). Pour [x] ∈ E/V on pose u
e([x]) = u(x) où
x est un représentant de la classe d’équivalence [x]. Comme u s’annule sur V alors la
définition de u
e([x]) ne dépend pas du choix du représentant de la classe [x]. On démontre
ensuite facilement que u
e ∈ L(E/V, W ) et que ϕ(e
u) = u. ϕ est, par conséquent, un
isomorphisme d’où:
Si W est un sous-espace vectoriel de F de dimension finie et si V est un sous-
espace vectoriel de E de codimension finie, alors LV,W (E, F ) est de dimension finie et
dim LV,W (E, F ) = dim W codim V .
Solutions 15

Solution .10 1◦ . Le fait que E soit un sous-espace vectoriel de Mn (IR), et le fait


que δ soit une forme linéaire sur E sont faciles et laissés au lecteur.
2◦ . Soient A = (aij ) et B = (bij ) deux éléments de E. On pose C = (cij ) = A.B.
Alors à n !  
n
X n
X X n
X Xn
cij = aik bkj = aik  bkj 
j=1 j=1 k=1 k=1 j=1
n
X
= aik δ(B) = δ(A) δ(B).
k=1

et de même à n ! à n !
n
X n
X X n
X X
cij = aik bkj = bkj aik
i=1 i=1 k=1 k=1 i=1
n
X
= akj δ(A) = δ(A) δ(B).
k=1

On conclut que

∀ (A, B) ∈ E 2 , AB ∈ E, et δ(AB) = δ(A)δ(B).

3◦ . La vérification est immédiate, et on a

∀ A ∈ E, AJ = JA = δ(A)J.

4◦ . Soit A ∈ E inversible. On a AJ = JA = δ(A)J, donc en multipliant par A−1


on trouve J = δ(A) A−1 J = δ(A) JA−1 en particulier δ(A) 6= 0 et alors
1
J = A−1 J = JA−1
δ(A)

On en déduit que A−1 ∈ E et que δ(A−1 ) = 1/δ(A).


5◦ . δ est une forme linéaire sur E et δ(J) = n 6= 0 alors E = Ker δ ⊕ IRJ.
Dans la suite nous identifions toute matrice A de Mn (IR) à l’endomorphisme de
IRn , qui à X associe AX, et on pose V = Im J, et W = Ker J.

Ker δ ={A ∈ Mn (IR) : AJ = JA = 0}


={A ∈ Mn (IR) : V ⊂ Ker A, et W ⊂ Im A}
D’après l’exercice précédent on a alors

dim Ker δ = dim W codim V = (n − dim Im J)2 = (n − 1)2 .

On conclut que dim E = n2 − 2n + 2.


16 Dualité

6◦ . On définit sur E les deux formes linéaires d1 et d2 , par


n
X n
X
∀ M = (aij ) ∈ E, d1 (M ) = aii , d2 (M ) = ai,n+1−i .
i=1 i=1

Il est clair que Ee est un sous-espace vectoriel de E. Notons

E0 = Ker (δ) ∩ Ker (d1 ) ∩ Ker (d2 ).

Il est immédiat de voir que Ee = E0 ⊕ IRJ.


On vérifie aussi facilement que pour n ∈ {1, 2}, le sous-espace vectoriel E0 est réduit
à {0} et par conséquent dim Ee = 1.
Supposons que n > 2. Nous allons voir que les trois formes linéaires δ, d1 et d2 de

E sont linéairement indépendantes.
Soit (a, b, c) ∈ IR3 tel que aδ + bd1 + cd2 = 0. Alors, en testant cette égalité sur les
matrices I, J, et H = (hij ), (avec hij = 1 si i + j = n + 1 et hij = 0 si i + j 6= n + 1),
on obtient le système linéaire
a + nb + εn c = 0
a + b + c = 0
a + εn b + nc = 0

avec εn = (1 − (−1)n )/2. Le déterminant de ce système vaut −εn − n(n − 2) qui est
différent de 0 pour n ≥ 3. Alors a = b = c = 0. Ceci démontre que {δ, d1 , d2 } est libre
dans E ∗ d’où dim E0 = dim E −3 = n2 −2n−1. Par conséquent, on arrive à la conclusion

dim Ee = max(1, n2 − 2n).

Solution .11 Un polynôme Q ∈ E qui vérifie ϕ1 (Q) = ϕ2 (Q) = 0 est de la forme


λ(X − 1) . Alors en prenant Q3 = 3(X − 1)2 on a ϕj (Q3 ) = δj3 .
2

Pour tout polynôme Q ∈ E on a Q − ϕ(Q)Q3 ∈ Ker ϕ3 . Alors, pour i ∈ {1, 2},


si Q ∈ Ker ϕi , on a Q − ϕ(Q)Q3 ∈ Ker ϕ3 ∩ Ker ϕi . Par Conséquent, Si l’on choisit
Q = 1 ∈ Ker ϕ2 , on a Q1 = 1 − Q3 ∈ Ker ϕ3 ∩ Ker ϕ2 , avec ϕ1 (Q1 ) = 1. Et si
1
l’on choisit Q = X − 1 ∈ Ker ϕ1 , on a Q2 = X − 1 − Q3 ∈ Ker ϕ3 ∩ Ker ϕ1 , avec
2
ϕ2 (Q2 ) = 1. On a donc trouvé trois polynômes {Q1 , Q2 , Q3 } de E tels que ϕj (Qi ) = δij .
1
Q1 = −3X 2 + 6X − 2, Q2 = (X − 1)(3X − 1), Q3 = 3(X − 1)2 .
2
Ceci démontre que {ϕ1 , ϕ2 , ϕ3 } est une base de E ∗ qui est la base duale de {Q1 , Q2 , Q3 }.
Solutions 17

Solution .12 Un polynôme Q1 ∈ E qui vérifie ϕj (Q1 ) = δj1 doit avoir 1 comme
racine double, donc il est de la forme (λX + µ)(X − 1)2 . Les deux conditions ϕ1 (Q1 ) = 1
et ϕ3 (Q1 ) = 0 impliquent que λ = 2 et µ = 1, donc si Q1 = (2X + 1)(X − 1)2 alors
ϕj (Q1 ) = δj1 .
Notons alors que Q2 (X) = Q1 (1 − X) = (3 − 2X)X 2 vérifie aussi ϕj (Q2 ) = δj2 .
Un polynôme Q3 ∈ E qui vérifie ϕj (Q3 ) = δj3 doit avoir 1 comme racine double et
0 comme racine simple, donc il est de la forme λX(X − 1)2 . La condition ϕ3 (Q3 ) = 1
implique que λ = 1, donc si Q3 = X(X − 1)2 alors ϕj (Q3 ) = δj3 .
Enfin, Q4 (X) = −Q3 (1 − X) = (X − 1)X 2 vérifie ϕj (Q4 ) = δj4 .
La famille {Q1 , Q2 , Q3 , Q4 } de E vérifie que ϕj (Qi ) = δij . Alors c’est une base de
E dont la base duale est {ϕ1 , ϕ2 , ϕ3 , ϕ4 }.
On sait alors que
3
X
ψ= ψ(Qi )ϕi ,
i=1

1 1
avec ψ(Q1 ) = ψ(Q2 ) = et ψ(Q3 ) = −ψ(Q4 ) = , d’où
2 12
Z 1
P (0) + P (1) P 0 (0) − P 0 (1)
∀ P ∈ E, P (t) dt = + .
0 2 12

Solution .13 1◦ . C’est immédiat.


2◦ . Notons, pour chaque i ∈ {0, 1, . . . , n},
Yn
X − xj
Pk (X) = .
j=0
xk − xj
j6=k

Il est clair que ϕxi (Pk ) = δik , ce qui démontre que (ϕx0 , ϕx1 , . . . , ϕxn ) est une base
de E ∗ qui est la base duale de la base (P0 , P1 , . . . , Pn ).
Z 1
◦ ∗
3 . La forme linéaire ψ ∈ E définie par ψ(P ) = P (t) dt s’exprime de manière
0
n
X
unique sur la base précédente sous la forme ψ = ψ(Pk )ϕxk , donc λk = ψ(Pk ).
k=0

Solution .14 1◦ . D’après la formule de Taylor on a


n
X n
X
1 (k) k 1 (k)
∀ P ∈ E, P (X) = P (−a)(X + a) = P (−a)ek (X).
k! k!
k=0 k=0

Il en résulte que la base duale (e∗k )0≤k≤n de la base (ek )0≤k≤n est donnée par
1
e∗k (P ) = P (k) (−a).
k!
18 Dualité

(1 + a)k+1 − ak+1
D’autre part, ϕ(ek ) = . D’où
k+1
n
X (1 + a)k+1 − ak+1
ϕ= e∗k .
k+1
k=0

2◦ . Posons
p−1
1 Y
e0 (X) = 1 et ep (X) = (X − j), pour p > 0.
p! j=0

On vérifie que ∆(ep+1 ) = ep si p ≥ 0, et que ∆(e0 ) = 0. Il en résulte que



 ep−k si p ≥ k
k
∆ (ep ) =

0 si p < k

Par conséquent ϕk (ep ) = ∆k (ep )(0) = δkp . Alors (ep )0≤p≤n est la base de E dont
la base duale est (ϕk )0≤k≤n .
Application: Soit P un polynôme de degré n. Alors,
n
X n
X ¡ ¢
P (X) = ∆k (P )(0) ek (X) = ∆k (P )(0) ek+1 (X + 1) − ek+1 (X) .
k=0 k=0

n
X
Donc si l’on pose Q(X) = ∆k (P )(0) ek+1 (X), on trouve P (X) = Q(X + 1) − Q(X)
k=0
et par conséquent,
m
X n
X
k+1
∀ m ∈ IN, P (j) = Q(m + 1) = ∆k (P )(0) Cm+1 .
j=0 k=0

Par exemple, si P (X) = X 2 + X + 1 alors ∆2 (P )(0) = 2, ∆(P )(0) = 2, et P (0) = 1.


D’où
m
X (m + 1)3 + 2(m + 1)
(j 2 + j + 1) = Cm+1
1 2
+ 2Cm+1 3
+ 2Cm+1 = .
j=0
3

Solution .15 1◦ . il est clair que, pour n ≥ 2, on a


1 ¡ ¢
Pn0 (X) = (X − n)n−1 + (n − 1)X(X − n)n−2
n!
1
= (X − 1)(X − n)n−2 = Pn−1 (X − 1)
(n − 1)!

Il en résulte par récurrence sur k que

∀ k ∈ IN, ∀ n ≥ k, Pn(k) (X) = Pn−k (X − k)


Solutions 19

2◦ . le degré de Pn est égal à n donc si k > n on a clairement ϕk (Pn ) = 0, et


si k ≤ n, on peut utiliser ce qui précède pour conclure que ϕk (Pn ) = Pn−k (0). Alors
ϕk (Pn ) = 0 si k 6= n et ϕk (Pn ) = 1 si k = n.
3◦ . On voit immédiatement que
∀ (k, `) ∈ {0, 1, . . . , m}2 , hϕk , P` i = δk `
donc (P0 , P1 , . . . , Pm ) est une base de E dont la base duale est (ϕ0 , ϕ1 , . . . , ϕm ).
4◦ . Alors,
m
X m
X
∀ Q ∈ E, Q(X) = hϕk , QiPk (X) = Q(k) (k)Pk (X)
k=0 k=0
qui est la relation demandée.
5◦ . En prenant Pm (X + a) pour Q dans la relation précédente, et en utilisant 1◦ .,
on trouve:
m
X m
X
(k)
Pm (X + a) = Pm (a + k)Pk (X) = Pm−k (a + k − k)Pk (X)
k=0 k=0
soit
m
X
Pm (X + a) = Pm−k (a)Pk (X)
k=0
qui est la relation demandée.

Solution .16 1◦ . Il est clair que T est une application linéaire entre deux espaces
de même dimension : dim Mn (IR) = dim(Mn (IR))∗ = n2 ,
il suffit donc de prouver que Ker T = {0}. Or, soit M = (ai j ) ∈ Ker T , alors
TM = 0, et en particulier TM (t M ) = 0. Mais
 
n
X n
X X
TM (t M ) = Tr (M · t M ) =  (ai j )2  = a2i j
i=1 j=1 1≤i,j≤n

Donc,
X
M = (ai j ) ∈ Ker T ⇐⇒ a2i j = 0 ⇐⇒ M = 0
1≤i,j≤n

On conclut que T est un isomprphisme.


2◦ . Notons d’abord que la trace Tr () vérifie la propriété Tr (AB) = Tr (BA) pour
tout (A, B) ∈ Mn (IR)2 . Soit M ∈ Mn (IR), On a

∀ N ∈ Mn (IR), Tr (M0 M N ) = Tr (M0 N M ) = Tr (M M0 N )

ce qui prouve que TM0 M −M M0 = 0, ou bien M0 M = M M0 .


20 Dualité

Soit E` k = (δi ` δj k )i j ∈ Mn (IR) où (`, k) ∈ IN2n . et supposons que M0 = (mi j )i j .


Alors

∀ k ∈ INn , M0 Ek 1 = Ek 1 M0 =⇒ ∀ (i, j, k) ∈ IN3n , δ1 j mi k = δi k m1 j

en particulier, pour j = 1, ∀ (i, k) ∈ IN2n , mi k = δi k m11 . C’est à dire, M0 = λI où


λ = m11 . on conclut que ϕ = λ Tr.
3◦ . Notons C = Vect {M N − N M : (M, N ) ∈ Mn (IR)2 }. On a démontré dans 2◦ .
que C ⊥ = IRTr. Donc dim C = n2 − 1. .

OKMRAN
OUBA
RÉDUCTION DES ENDOMORPHISMES

I. Généralités

Définition : Soient E un IK-espace vectoriel, supposé différent de {0}, et u ∈ L(E). On


appelle valeur propre de u tout λ ∈ IK tel que Ker (u − λIE ) 6= {0}. Si λ est une valeur
propre de u, alors le sous-espace vectoriel Eλ = Ker (u − λIE ) s’appelle sous-espace
propre de u associé à la valeur propre λ, et les éléments non nuls de Eλ s’appellent
vecteurs propres de u associé à la valeur propre λ. L’ensemble des valeurs propres d’un
endomorphisme u s’appelle le spectre de u et il est noté Sp (u).
Un endomorphisme u de E est dit diagonalisable si et seulement si, il existe une
base E = (ei )i∈I de E telle que, pour tout i ∈ I, u(ei ) ∈ IKei .

Théorème I.1. Soit u un endomorphisme de E, et (λi )1≤i≤n une famille finie de


n
X
valeurs propres deux à deux distinctes de u. Alors la somme Eλi des sous-espaces
i=1
propres associés est directe.

Preuve : Soit u un endomorphisme de E. Notons Pn la propriété que “Pour toute


famille finie (λi )1≤i≤n de n valeurs propres deux à deux distinctes de u, la somme
Xn
Eλi des sous-espaces propres associés est directe.”
i=1
On remarque que P1 et P2 sont trivialement vraies. Supposons qu’il existe un
entier k tel que Pk soit fausse, et définissons m le plus petit entier tel que Pm soit
fausse. Clairement, nous avons m > 2. Alors il existe une famille finie (λi )1≤i≤m de
Xm
valeurs propres deux à deux distinctes de u, telle que la somme Eλi ne soit pas
i=1
directe. Par conséquent, on peut trouver x1 , . . . , xm tels que
m
X
0= xi , et pour tout i, xi ∈ Eλi \ {0}.
i=1

(Les (xi )1≤i≤m sont tous non nuls, à cause de la minimalité de m). Il en résulte que
m
X m
X m
X
0= u(xi ) = λi xi et 0 = λm xi .
i=1 i=1 i=1
2 Réduction des endomprphismes

m−1
X m−1
X
soit (λm − λi )xi = 0. Mais la somme Eλi est directe donc pour tout 1 ≤ i < m,
i=1 i=1
on a (λm − λi )xi = 0. Puis λm = λi , pour tout 1 ≤ i ≤ m − 1, ce qui contredit
l’hypothèse. Cette contradiction montre que Pn est vraie pour tout n.

Dans la suite de ce chapitre E est un IK-espace vectoriel de dimension n ≥ 1.

Lemme I.2. Soit u ∈ L(E). Le polynôme det(Mat(u, E) − XIn ) ne dépend pas du choix
de la base E = (e1 , . . . , en ) de E.

Preuve : Soient E = (e1 , . . . , en ) et F = (f1 , . . . , fn )


u
deux bases de E. Le diagramme ci-contre permet (E,E) −−−→ (E,xE)
 
d’écrire M = P −1 M 0 P avec P = Mat(IE , E, F), IEy
IE

u
M = Mat(u, E), et M 0 = Mat(u, F). Il en résulte (E, F) −−−→ (E, F)
que M − XIn = P −1 (M 0 − XIn )P . Par conséquent,

det(M − XIn ) = det P −1 det(M 0 − XIn ) det P = det(M 0 − XIn ).

Définition : Soit u un endomorphisme de E. On appelle polynôme caractéristique de


u le polynôme
Xu (X) = det(Mat(u, E) − XIn ) ∈ IK[X],

où E est une base quelconque de E.

Proposition I.3. Soit u ∈ L(E). Les racines dans IK de Xu (X) sont les valeurs propres
de u.
C’est immédiat.

Définition : Soit u un endomorphisme de E. La multiplicité d’une valeur propre λ de


u est la multiplicité de λ comme racine du polynôme caractéristique Xu (X), et on la
note m(λ).

Pour B une matrice carrée d’ordre n, nous allons noter Cj (B) la j ième colonne de
B. Soit E la base canonique de IKn . En exploitant la n-linéarité du déterminant, nous
pouvons écrire

1 X
X 1 1
X
det(A0 + A1 ) = ··· det E (C1 (Aj1 ), C2 (Aj2 ), . . . , Cn (Ajn )). (1)
j1 =0 j2 =0 jn =0
Généralités 3

On peut reformuler ce résultat de la manière suivante. Si, pour J ⊂ {1, 2, . . . , n}, l’on
note AJ la matrice dont les colonnes sont définies par

 Cj (A1 ) si j ∈ J
Cj (AJ ) =

Cj (A0 ) si j ∈ /J

Alors (1) s’écrit,


X
det(A0 + A1 ) = det(AJ ). (2)
J⊂{1,...,n}

En utilisant la formule précédente pour calculer det(M − XIn ), (avec A0 = −XIn et


A1 = M ), nous obtenons
X
det(M − XIn ) = (−X)n + (−X)n−Card(J) det(MJ,J ). (3)
Ø6=J⊂{1,...,n}

où MJ,J est la matrice d’ordre Card(J) obtenue de M en supprimant les colonnes et
les lignes d’indices n’appartenant pas à J. D’où
n
X X
n
det(M − XIn ) = (−X) + (−X)n−k det(MJ,J ).
k=1 J⊂{1,...,n}
Card(J)=k

Nous avons démontré la proposition suivante:

Proposition I.4. Soit M une matrice carrée d’ordre n à coefficients dans IK. Alors
à n
!
X
det(M − XIn ) = (−1)n X n + (−1)k τk (M )X n−k
k=1

avec
X
τk (M ) = det(MJ,J ).
J⊂{1,...,n}
Card(J)=k

et MJ,J la matrice d’ordre Card(J) obtenue de M en supprimant les colonnes et les


lignes d’indices n’appartenant pas à J. En particulier, det(M − XIn ) est un polynôme
de degré n, de terme dominant (−1)n X n , de terme constant det M , et le terme de
degré n − 1 est (−1)n−1 Tr (M )X n−1 .

Corollaire I.5. Soit u un endomorphisme de E. Alors le polynôme caractéristique


Xu (X) de u est un polynôme de IK[X], de degré n, de terme dominant (−1)n X n , de
terme constant det u, et le terme de degré n − 1 est (−1)n−1 Tr (u)X n−1 . En particulier,
si IK = C
| alors, u admet au moins une valeur propre.
4 Réduction des endomprphismes

Théorème I.6. Soit u un endomorphisme de E.


1◦ . u et t u ont le même polynôme caractéristique.
2◦ . Si F est un sous-espace vectoriel de E, non réduit à {0}, et stable par u, et si v
est l’application induite par u sur F , (i.e. v : F −→ F : x 7→ u(x)), alors Xv (X)
divise Xu (X).
3◦ . Soient F et G deux sous-espaces vectoriels de E, non réduits à {0}, supplémentaires
et stables par u. Si v et w sont respectivement les applications induites par u sur
F et G, alors Xu (X) = Xv (X)Xw (X).

Preuve : 1◦ . Soit E une base de E et E ∗ la base duale. Alors si M = Mat (u, E) on a


t
M = Mat (t u, E ∗ ). Par conséquent,
Xu (X) = det(M − XIn ) = det(t M − XIn ) = Xt u (X).
2◦ . Soit E 0 = (e1 , . . . , er ) une base de F , on la complète en une base E = (e1 , . . . , en )
de E. La matrice de u dans cette base admet la décoposition par blocks suivante
· ¸
P R
M = Mat (u, E) =
0 Q
avec P = Mat (v, E 0 ). Alors
Xu (X) = det(M − XIn ) = det(P − XIr ) det(Q − XIn−r ) = Xv (X) det(Q − XIn−r ).
3◦ . Soit E1 = (e1 , . . . , er ) une base de F , et E2 = (er+1 , . . . , en ) une base de G,
alors E = (e1 , . . . , en ) est une base de E. La matrice de u dans cette base admet la
décoposition par blocks suivante
· ¸
P 0
M = Mat (u, E) =
0 Q
avec P = Mat (v, E1 ) et Q = Mat (w, E2 ). Alors
Xu (X) = det(M − XIn ) = det(P − XIr ) det(Q − XIn−r ) = Xv (X)Xw (X).

Corollaire I.7. Soient u un endomorphisme de E, et λ une valeur propre de u. La


dimension de l’espace propre de u associé à λ est plus petite ou égale à la multiplicité
de λ.
dim Ker (u − λI) ≤ m(λ).

Preuve : En effet, si F = Ker (u − λI) alors F est un sous-espace vectoriel de E, non


réduit à {0}, et stable par u, de plus si v est l’application induite par u sur F , alors
Xv (X) = (λ − X)dim F . D’après le théorème précédent (λ − X)dim F divise Xu (X), d’où
le résultat.
Endomorphismes diagonalisables 5

II. Endomorphismes diagonalisables

Théorème II.1. Soit u un endomorphisme de E. Les assertions suivantes sont


équivalentes:
1◦ . Il existe une base E de E dans laquelle la matrice de u est diagonale. (i.e. u est
diagonalisable).

2 . Le polynôme caractéristique de u est scindé sur IK et, pour toute valeur propre λ
de u, la multiplicité de λ est égale à la dimension du sous-espace propre associé,
m(λ) = dim Ker (u − λI).

Preuve : (1◦ =⇒ 2◦ ) Soit E = (e1 , . . . , en ) une base de E dans la quelle Mat (u, E) est
diagonale. Les éléments distincts de la diagonale de cette matrice sont (λ1 , λ2 , . . . , λp ) et
Yp
chaque λi est répété mi fois. Alors Xu (X) = (λi − X)mi . Ce qui montre que Xu (X)
i=1
est scindé et que pour chaque i, λi est une valeur propre de multiplicité mi .
Notons que mi = Card {j ∈ {1, . . . , n} : u(ej ) = λi ej }. Donc mi ≤ dim Eλi . Mais
l’inégalité inverse résulte du corollaire I.7, ce qui démontre 2◦ .
Yp
(2◦ =⇒ 1◦ ) D’après l’hypothèse Xu (X) = (λi − X)mi , avec mi = dim Eλi . Le
i=1
p
X
théorème I.1 montre que la somme Eλi est directe, de plus
i=1
p
M p
X p
X
dim Eλi = dim Eλi = mi = deg Xu (X) = n = dim E.
i=1 i=1 i=1
p
M p
[
Alors E = Eλi . Il suffit de prendre comme base de E une base de la forme E = Ei
i=1 i=1
avec Ei une base de Eλi = Ker (u − λi I).

Corollaire II.2. Soit u un endomorphisme de E. Si u admet n, (n = dim E), valeurs


propres distinctes, alors u est diagonalisable.

Passons au point de vu matriciel. Soit M ∈ Mn (IK), on dit que M est diagonalisable


si, et seulement si, l’endomorphisme

UM : IKn −→ IKn , X 7→ M X

est diagonalisable. Ce qui revient à dire que M est diagonalisable si, et seulement si, il
existe une matrice diagonale D et une matrice inversible P telles que M = P DP −1 . En
6 Réduction des endomprphismes

effet P = Mat (IIKn , V, E) où E est la base canonique de IKn , et V est la base de IKn
formée de vecteurs propres de UM .

Exemple : Étudions la matrice


 
−1 1 7 −3
1 1 −1 −3 7
M=  .
4 7 −3 −1 1
−3 7 1 −1

Calculons d’abord le polynôme caractéristique de M .


 
−1/4 − X 1/4 7/4 −3/4
 1/4 −1/4 − X −3/4 7/4 
XM (X) = det  .
7/4 −3/4 −1/4 − X 1/4
−3/4 7/4 1/4 −1/4 − X

En ajoutant la troisième ligne à la première et la quatrième ligne à deuxième on trouve:


 
3/2 − X −1/2 3/2 − X −1/2
 −1/2 3/2 − X −1/2 3/2 − X 
XM (X) = det  
7/4 −3/4 −1/4 − X 1/4
−3/4 7/4 1/4 −1/4 − X

Puis en soustrayant la première colonne de la troisième et la deuxième colonne de la


quatrième on trouve:
 
3/2 − X −1/2 0 0
 −1/2 3/2 − X 0 0 
XM (X) = det  
7/4 −3/4 −2 − X 1
−3/4 7/4 1 −2 − X
· ¸ · ¸
3/2 − X −1/2 −2 − X 1
= det det
−1/2 3/2 − X 1 −2 − X
3 1
=[( − X)2 − ] · [(2 + X)2 − 1]
2 4
=(1 − X)(2 − X)(1 + X)(3 + X).

Le spectre de M est alors {1, 2, −1, −3}. Le polynôme caractéristique de M admet 4


valeurs propres distinctes donc M est diagonalisable.
– Comme la somme des coefficients de chaque ligne de M vaut 1, alors V1 =
t
[ 1, 1, 1, 1 ] est un vecteur propre de M associé à la valeur propre 1, (M V1 = V1 ).
t
– Cherchons V2 = [ x, y, z, t ] vecteur propre de M associé à la valeur
propre 2. Un calcul simple montre que le système (M − 2I)V2 = 0 est équivalent à
x = −y = z = −t. D’où, V2 = t [ 1, −1, 1, −1 ].
Endomorphismes trigonalisables 7

– De même V−1 = t [ 1, 1, −1, −1 ] est le vecteur propre de M associé à la


t
valeur propre −1. Et V−3 = [ 1, −1, −1, 1 ] est le vecteur propre de M associé à
la valeur propre −3.
Alors, M = P DP −1 avec D = diag(1, 2, −1, −3) et
 
1 1 1 1
1 −1 1 −1 
P = .
1 1 −1 −1
1 −1 −1 1

1
On note que P −1 = P.
4

III. Endomorphismes trigonalisables

Définition : Soit u un endomorphisme de E. On dit que u est un endomorphisme


trigonalisable si, et seulement si, il existe une base de E de E telle que Mat (u, E) soit
trianglaire.
Remarquons que si la matrice Mat (u, (e1 , e2 , . . . , en )) est triangulaire supérieure
alors la matrice Mat (u, (en , en−1 , . . . , e1 )) est triangulaire inférieure.

Théorème III.1. Soit u un endomorphisme de E. u est trigonalisable si, et seulement


si, le polynôme caractéristique Xu (X) est scindé sur IK.

Preuve : • Si u est trigonalisable alors il existe une base E de E telle que


 
a11 a12 ... a1n
 0 a22 ... a2n 
Mat (u, E) = 
 ... .. .. .. 
. . . 
0 ··· 0 ann
n
Y
d’où Xu (X) = (aii − X) est un polynôme scindé dans IK.
i=1
• Inversement, nous allons démontrer par récurrence sur la dimension n de l’espace
E que tout endomorphisme de E ayant un polynôme caractéristique scindé sur IK est
trigonalisable.
C’est trivialement vrai si n = 1.
Supposons la propriété vraie pour tout espace vectoriel de dimension k < n. Soit u
un endomorpisme d’un IK-espace vectoriel E de dimension n, et supposons que Xu (X)
est scindé sur IK.
8 Réduction des endomprphismes

Soit λ ∈ IK une valeur propre de u, (qui existe car Xu (X) est scindé sur IK), et soit
x ∈ E \ {0} un vecteur propre de u associé à λ. (u(x) = λx).
Posons e1 = x et complètons e1 en une base E = (e1 , e2 , . . . , en ) de E. Enfin, notons
p la projection de E sur F = Vect (e2 , . . . , en ) parallèlement à IKe1 et s l’injection
canonique de F dans E, (i.e. s(x) = x).
n
X
Si M = (mij ) = Mat (u, E) alors, pour tout j ∈ {2, . . . , n}, p◦u◦s(ej ) = mij ej . Il en
i=2
résulte que  
λ m12 . . . . . . m1n
0 
Mat (u, E) = 
 ...

Mat (p◦u◦s, E ) 
0

0
0
avec E = (e2 , . . . , en ). L’application p◦u◦s est un endomorphisme de F avec dim F =
n − 1. D’autre part, Xu (X) = (λ − X)Xp◦u◦s (X) alors Xp◦u◦s (X) est scindé sur IK.
L’hypothèse de récurrence montre qu’il existe Ee = (e
e2 , . . . , een ) une base de F pour
e est triangulaire supérieure. Alors la matrice de u dans la base
laquelle Mat (p◦u◦s, E)
(e1 , ee2 , . . . , een ) est triangulaire supérieure.

Corollaire III.2. Tout endomorphisme d’un C-espace


| vectoriel est trigonalisable.

IV. Polynômes d’endomorphismes

Soit u un endomorphime de E. Si k ∈ IN, on pose

u0 = IE , et uk = u◦uk−1 pour k ≥ 1.

Théorème IV.1. Soit u un endomorphisme de E. L’application qui au polynôme


Xm m
X
k
P (X) = ak X ∈ IK[X] associe l’endomorphisme P (u) = ak uk ∈ L(E) est un
k=0 k=0
morphisme de la IK-algèbre IK[X] dans la IK-algèbre L(E).

Preuve : Il s’agit de vérifier que pour tout (λ, P, Q) ∈ IK × IK[X] × IK[X] Nous
avons P (u) + Q(u) = (P + Q)(u), λP (u) = (λP )(u), et P (u)◦Q(u) = (P Q)(u). C’est
une tache facile que nous laissons au lecteur.

Théorème IV.2. Soient u un endomorphisme de E, et (P, Q) ∈ IK[X] × IK[X] deux


polynômes premiers entre eux. Alors

Ker (P Q)(u) = Ker P (u) ⊕ Ker Q(u).


Polynômes d’endomorphismes 9

Preuve : D’aprèrs le théorème de Bezout, il existe (S, T ) ∈ IK[X] × IK[X] tel que
SP + T Q = 1 d’où
S(u)◦P (u) + T (u)◦Q(u) = IE (†)

• Soit x ∈ Ker P (u) ∩ Ker Q(u) alors d’après (†)

x = S(u)◦P (u)x + T (u)◦Q(u)x = 0.

• D’autre part, si x ∈ Ker (P Q)(u) on pose x1 = T (u)◦Q(u)x ∈ Ker P (u) et


x2 = S(u)◦P (u)x ∈ Ker Q(u), et la relation (†) montre que x = x1 + x2 . D’où le
résultat.

Corollaire IV.3. Soient u un endomorphisme de E, et (Pi )1≤i≤r des polynômes deux


r
Y
à deux premiers entre eux. Si P = Pi , alors
i=1

r
M
Ker P (u) = Ker Pi (u).
i=1

C’est une généralisation immédiate.

Corollaire IV.4. Soit u un endomorphisme de E. Alors u est diagonalisable si, et


seulement si, il existe un polynôme P scindé, ayant toutes ses racines simples dans IK,
et tel que P (u) = 0.

L
Preuve : En effet, supposons que u est diagonalisable. Alors E = Eλ , et donc
λ∈Sp(u)
Y
si P (X) = (X − λ) alors P (u) = 0 et P est scindé dans IK[X] et a toutes ses
λ∈Sp(u)
racines simples.
r
Y
Inversement, supposons que le polynôme P (X) = (X − µi ), avec µ1 , . . . , µr
i=1
distincts, annule u. Alors comme les polynômes {(X − µi )}1≤i≤r sont deux à deux
premiers entre eux, on a
r
M
E = Ker P (u) = Ker (u − µi IE ).
i=1

Si J = {i ∈ {1, . . . , r} : Ker (u − µi IE ) 6= {0}} alors, il suffit de prendre comme base


S
de E une base de la forme E = Ei avec Ei une base de Eµi = Ker (u − µi I). Dans
i∈J
cette base u admet une matrice diagonale.
10 Réduction des endomprphismes

Corollaire IV.5. Soit u un endomorphisme de E. Supposons qu’il existe un polynôme


r
Y
P (X) = (X − µi )mi avec (µ1 , . . . , µr ) distincts dans IK, tel que P (u) = 0, alors
i=1

r
M
E= Ker (u − µi IE )mi .
i=1

Car les polynômes {(X − µi )mi }1≤i≤r sont deux à deux premiers entre eux.

Théorème IV.6. (Cayley-Hamilton) Soit u un endomorphisme de E. Si Xu (X) est le


polynôme caractéristique de u, alors Xu (u) = 0.

Preuve : Soit x ∈ E\{0}. L’espace E est de dimension finie n, alors (x, u(x), . . . , un (x))
est une famille liée. On pose alors

p = p(x) = min{k : (x, u(x), . . . , uk (x)) est liée }

et
Eu (x) = Vect ({x, u(x), . . . , up−1 (x)}).

Le système (x, u(x), . . . , up−1 (x)) est libre à cause de la minimalité de p donc c’est
une base de Eu (x). Il en résulte, toujours à cause de la définition de p, que

p−1
X
p
u (x) = ak uk (x). (∗)
k=0

En particulier u(up−1 (x)) ∈ Eu (x) d’où u(Eu (x)) ⊂ Eu (x). Si, alors

v : Eu (x) −→ Eu (x) : y 7→ u(y),

on a Xu (X) = Q(X)Xv (X).


Mais, en prenant B = (x, u(x), . . . , up−1 (x)) comme base de Eu (x) nous calculons
 
−X 0 ··· ··· 0 a0
 1 −X 0 0 a1 
 
 .. .. .. .. .. 
 0 . . . . . 
Xv (X) = det(Mat(v, B) − XIp ) = det 
 .. .. .. .. .. .

 . . . . 0 . 
 . .. .. 
 .. . . −X ap−2 
0 ··· ··· 0 1 ap−1 − X
Polynômes d’endomorphismes 11

Alors, (voir l’exercice 1.b.)


à p−1
!
X
Xv (X) = (−1)p Xp − ak X k
k=0

En particulier, d’après (∗),


à p−1
!
X
Xv (u)(x) = (−1)p up (x) − ak uk (x) =0
k=0

Enfin, Xu (u)x = Q(u)Xv (u)(x) = 0. Nous avons alors démontré que Xu (u)(x) = 0 pour
tout x ∈ E. Ce qui démontre que Xu (u) = 0.
12 RÉDUCTION DES ENDOMORPHISMES

EXERCICES

Exercice .1 Calculer les polynômes caractéristiques des matrices M ∈ Mn ( C)


|

suivantes
a. M = [aij ] (n ≥ 2) avec aij = a si i > j, aij = b si i < j et aii = 0 pour tout i.
   
0 ··· ··· 0 a0 0 ··· ··· 0 an
 . ..   .. .. 
 1 .. . a1   . . an−1 
 .. ..   .. .. .. 
b. M =  . ..  (n ≥ 2), c. M =  
 0 .. . . .   . . . 
. . .   .. 
 .. .. .. 0 an−2   0 ··· ··· 0 . 
0 ··· 0 1 an−1 an an−1 ··· a2 a21
   
a2 ab ab b2 3 −5 2 −6
 ab a2 b2 ab   0 5 0 4
d. M =   e. M =  
ab b2 a2 ab −2 7 −1 11
b2 ab ab a2 0 −4 0 −3

f. M = Γ(A0 , A1 , . . . , An−1 ) = [akj ] avec akj = AR(j−k) où (A0 , A1 , . . . , An−1 ) ∈


| n et R : Z
C Z −→ {0, 1, . . . , n − 1} la fonction qui à chaque entier associe le reste positif
de sa division par n. (On pourrait utiliser la matrice Ω = [ωkj ] avec ωkj = ω (k−1)(j−1)
où ω = exp(2iπ/n)).
Exercice .2 Soit la matrice :
 
m 1 1
A=1 m 1
1 1 m

Déterminer les valeurs propres et les vecteurs propres de A, la diagonaliser ; calculer


Ap , p ∈ IN .
Exercice .3 Soit E = IR3 espace vectoriel euclidien muni d’une base orthonormée
directe B. Soient v ∈ E \ {0} fixé, et u ∈ L(E) définie par u(x) = v ∧ x. Donner les
valeurs propres et les vecteurs propres de u.

Exercice .4 Montrer que les matrices suivantes sont semblables :

   
1 2 3 4 1 1 0 0
0 1 2 3 0 1 1 0
A=  et B= .
0 0 1 2 0 0 1 1
0 0 0 1 0 0 0 1
Exercices 13

Exercice .5 Etudier la réduction des matrices non nulles suivantes :


A ∈ Mn ( C)
| telle que ∀ (j, i) ∈ {1, . . . .n}2 ai,j = ai .

1 si i+j =n+1
2
B ∈ Mn ( C) telle que
| ∀ (j, i) ∈ {1, . . . .n} bi,j =

0 si i + j 6= n + 1

 ci si i+j =n+1
2
C ∈ Mn ( C) telle que
| ∀ (j, i) ∈ {1, . . . .n} ci,j =

0 si i + j 6= n + 1
avec ck ∈ IR∗+ pour tout k ∈ {1, 2, . . . , n}.

1 si i = 1 ou j = 1 ou i = j
D ∈ Mn ( C)
| telle que ∀ (j, i) ∈ {1, . . . .n}2 di,j =

0 sinon

Exercice .6 Le corps de base est C.


| Diagonaliser, si possible, les matrices suivantes
et calculer leurs puissances k ièmes pour k ∈ IN∗ .
   
0 0 1 0 1 1 0 0
3 2 0 0  1 −2 1 0
a. M =   b. M =  
0 0 2 3 0 1 −2 1
0 1 0 0 0 0 11
2 −2 1 −1 0 2
c. M=  2 −3 2  d. M =  0 0 1
−1 2 0 0 −1 1

Exercice .7 Soit M une matrice carrée d’ordre n sur un corps IK. Supposons que
M s’écrit sous la forme · ¸
A B
M=
C D
où A ∈ Mp (IK), D ∈ Mq (IK), B ∈ Mp×q (IK), et C ∈ Mq×p (IK).(p + q = n).
1◦ . Démontrer que si A est inversible alors, det M = det A. det(D − C.A−1 .B).
2◦ . Démontrer que si D est inversible alors, det M = det D. det(A − B.D−1 .C).
3◦ . Soient 1 ≤ p ≤ n deux nombres entiers. Soient A ∈ Mn×p (IK), et B ∈ Mp×n (IK).
Montrer que, det(XIn + AB) = X n−p det(XIp + BA).
4◦ . Application:
a. Soient (X, Y ) ∈ (Mn×1 (IK))2 , On pose M = X.t X + Y.t Y . Calculer le
polynôme caratéristique de· M . Expliciter
¸ le cas M = [cos(j − i) θ]1≤i,j≤n .
0 In
b. Soit A ∈ Mn (IK), et B = ∈ M2n (IK). Montrer que
A 0

XB (X) = (−1)n XA (X 2 ).
14 RÉDUCTION DES ENDOMORPHISMES
· ¸
3 −4
Exercice .8 Soit la matrice A = . On considère Φ ∈ L(M2 ( C))
| défini par
2 −3

∀ M ∈ M2 ( C),
|
Φ(M ) = AM − M A

Déterminer le noyau de Φ et montrer que Φ est diagonalisable.

Exercice .9 Soit E l’espace vectoriel des fonctions continues de [0, 1] dans C.


|

On considère u ∈ L(E) qui associe, à f ∈ E, la fonction g : [0, 1] −→ C, | x 7→


R
1
min(x, t)f (t) dt. Déterminer les valeurs propres et vecteurs propres de u.
0
Exercice .10 Soient a, b distincts dans IR, et E = IR2n [X]. Donner les valeurs
propres et les vecteurs propres de l’endomorphisme u de E tel que

u(P )(X) = (X − a)(X − b)P 0 (X) − [2nX − n(a + b)]P (X) (P ∈ E).

Exercice .11 Soient a, b, c distincts dans IR, et E = IRn [X]. Discuter la diagonal-
isabilité de l’endomorphisme u de E tel que
d
u(P )(X) = ((aX + b)P (X)) + cP (X) (P ∈ E).
dX
Exercice .12 Soient M et A deux matrices carrées d’ordre n, à coefficients complexes
telles que AM = M A. On suppose que toutes les valeurs propres de M sont distinctes.
a. Montrer que tout vecteur propre de M est un vecteur propre de A.
b. Montrer que A est de la forme

A = α0 In + α1 M + · · · + αn−1 M n−1 avec αi ∈ C.


|

c. Résoudre B 2 = A où A est la matrice :


 
11 −5 5
A =  −5 3 −3 
5 −3 3

Exercice .13 Soit M ∈ Mn ( C).


| On suppose qu’il existe (µ, ν) ∈ | 2 et
C
2
(A, B) ∈ (Mn ( C))
| , tels que M k = µk A + ν k B pour k ∈ {1, 2, 3}. Discuter la
diagonalisabilité de M .
Exercice .14 Soit M ∈ Mn ( C)
| et (n ≥ 1). Prouver l’équivalence:

XM (X) = (−1)n X n ⇐⇒ (∀k ∈ IN∗ ) Tr (M k ) = 0 ⇐⇒ M est nilpotente.


Exercices 15

Exercice .15 On dit qu’une matrice A = (aij ) ∈ Mn (IR) est une matrice
stochastique si, et seulement si,
n
X
2
∀ (i, j) ∈ {1, . . . , n} , aij ∈ [0, 1], et ∀ i ∈ {1, . . . , n}, aij = 1.
j=1

Soit A = (aij ) ∈ Mn (IR) une matrice stochastique.


1◦ . Montrer que 1 est une valeur propre de A.
2◦ . Soit λ une valeur propre de A dans C.
| Montrer que | λ | ≤ 1.
3◦ . On suppose que aii > 0 pour tout i ∈ {1, . . . , n}. Montrer que les valeurs propres
complexes de A sont toutes situées dans un disque fermé de rayon strictement
inférieur à 1, tangent intérieurement en 1 au cercle trigonométrique.
Exercice .16 Soient E un espace vectoriel de dimension n ≥ 1 sur C,
| a et b deux
endomorphismes de E. On pose c = ab − ba, et on suppose que ca = ac et cb = bc.
1◦ . Montrer que c n’est pas inversible. (On pourrait montrer que pour tout
polynôme P ∈ C[X]
| on a P (a) b − b P (a) = P 0 (a) c et conclure).
2◦ . En déduire qu’il exist x un vecteur propre commun à a,b et c.
Exercice .17 Soit M une matrice d’ordre 2 à coefficients dans ZZ. On suppose qu’il

existe n ∈ IN tel que M n = I2 . Montrer que M 12 = I2 .
Exercice .18 Soient E un IK-espace vectoriel de dimension n ≥ 1, et f ∈ L(E).
1◦ . Soit x ∈ E \ {0}.
a. Montrer qu’il existe k ∈ IN∗ tel que (x, f (x), . . . , f k (x)) soit lié. On note p(x)
le plus petit entier strictement positif tel que (x, f (x), . . . , f p(x) (x)) soit lié, et
Ef (x) = Vect ({x, f (x), . . . , f p(x)−1 (x)}).
b. Montrer que Ef (x) est stable par f . On note gx : Ef (x) −→ Ef (x) : y 7→ f (y).
c. Prouver que Xgx divise Xf .
2◦ . En déduire que Xf (f ) = 0. Théorème de Cayley-Hamilton.
Exercice .19
n
X

1 . Soit P (X) = an−k X k un polynôme de degré n dans C[X].
| On suppose que
k=0
n
Y n
X
P (X) = (X − λi ), et on note Sk = λki pour k = 0, 1, . . . , n.
i=1 i=1
a. Montrer l’identité suivante:
n n
P 0 (X) X Sk 1 X λn+1 i
= + n+1 .
P (X) X k+1 X i=1
X − λ i
k=0

b. En déduire qu’au voisinage de l’infini on a,


n
Ãn−` !
X X 1
P 0 (x) = an−`−i Si x`−1 + O( ).
i=0
x
`=1
16 RÉDUCTION DES ENDOMORPHISMES

c. Conclure que

k
1X
a0 = 1, ak = − ak−i Si , 0 < k ≤ n.
k i=1

2◦ . Soit A ∈ Mn ( C).
| On définit les deux suites (Bk )1≤k≤n et (ak )1≤k≤n par
 
 B1 = A  Bk = A(Bk−1 + ak−1 I)
 
a1 = − Tr(B1 ) ak = − Tr(Bk )/k

Montrer que XA (X), le polynôme caractéristique de A, est égal à


à n−1
!
X
XA (X) = (−1)n Xn + an−k X k
k=1
Solutions 17

SOLUTIONS

Solution .1 a. Soit J la matrice d’ordre n dont tous les coefficients valent 1.


Considérons pour A ∈ Mn ( C)
| et t ∈ C
| la matrice A(t) = A + tJ. En soustrayant la

première ligne des lignes suivantes et ensuite la première colonne des colonnes suivantes,
on s’apperçoit que det A(t) est une fonction polynomiale de degré au plus égal à 1 en t.
Venons à notre problème et posons A = M −xIn . On a, d’après la remarque précédente,
det A(t) = det(A + tJ) = αt + β. Mais A(−a) est une matrice triangulaire supérieure
avec det A(−a) = (−1)n (x + a)n , et A(−b) est une matrice triangulaire inférieure avec
det A(−b) = (−1)n (x + b)n . Il en resulte que

(−1)n (x + a)n = β − αa, et (−1)n (x + b)n = β − αb.

D’où
b(x + a)n − a(x + b)n
β = det(M − xIn ) = (−1)n .
b−a
Il est clair que det(M −xIn ) est une fonction polynomiale en a et b donc le cas a = b
s’obtient du cas précédent en faisant tendre b vers a par exemple, d’où la conclusion:
 n n
 (−1)n b(X + a) − a(X + b) si a 6= b
XM (X) = b−a

(−1)n (X + a)n−1 (X − a(n − 1)) si a = b

b. Notons
 
−X 0 ··· ··· 0 a0
 1 −X 0 0 a1 
 
 .. .. .. .. .. 
 0 . . . . . 
∆n (a0 , a1 , . . . , an−1 )(X) = det 
 .. .. .. .. .. .

 . . . . 0 . 
 . .. .. 
 .. . . −X an−2 
0 ··· ··· 0 1 an−1 − X
| n
Nous allons démontrer par récurrence sur n que, pour tout (a0 , a1 , . . . , an−1 ) ∈ C
à n−1
!
X
∆n (a0 , a1 , . . . , an−1 )(X) = (−1)n X n − ak X k .
k=0

En effet, en développant le déterminant suivant la première ligne on trouve

∆n (a0 , a1 , . . . , an−1 )(X) = −X∆n−1 (a1 , . . . , an−1 )(X) − (−1)n a0 . (∗)


18 RÉDUCTION DES ENDOMORPHISMES

et · ¸
−X a0
∆2 (a0 , a1 )(X) = det = X 2 − a1 X − a0 .
1 a1 − X
Si le résultat est vrai pour n − 1, alors
à n−2
!
X
∆n−1 (a1 , . . . , an−1 )(X) = (−1)n−1 X n−1 − ak+1 X k
k=0

et (∗) montre que


à n−2
!
X
∆n (a0 , a1 , . . . , an−1 )(X) = (−1)n Xn − ak+1 X k+1 − (−1)n a0
k=0

ce qui démontre que


à n−1
!
X
XM (X) = (−1)n Xn − ak X k .
k=0

c. Notons
 
−X 0 ··· 0 an
 .. .. 
 0 −X . . an−1 
 .. .. .. .. 
∆n (a1 , a2 , . . . , an )(X) = det  . . .
 . 0 . 
 
0 ··· 0 −X a2
an an−1 ··· a2 a21 −X

| n
Nous allons démontrer par récurrence sur n que, pour tout (a1 , a2 , . . . , an ) ∈ C
à n
!
X
∆n (a1 , a2 , . . . , an )(X) = (−1)n X n−2 X 2 − a21 X − a2k .
k=2

En effet, en développant le déterminant suivant la première colonne on trouve


 
0 ··· 0 an
 .. .. 
n −X . . an−1 
∆n (a1 , a2 , . ., an ) = −X∆n−1 (a1 , . ., an−1 ) − (−1) an det  . .. .. 
 . . 
. 0 .
0 ··· −X a2

donc
∆n (a1 , a2 , . . . , an )(X) = −X∆n−1 (a1 , . . . , an−1 )(X) − (−X)n−2 a2n (∗)

et · ¸
−X a2
∆2 (a1 , a2 )(X) = det 2 = X 2 − a21 X − a22 .
a2 a1 − X
Solutions 19

Si le résultat est vrai pour n − 1, alors


à n−1
!
X
∆n−1 (a1 , . . . , an−1 )(X) = (−1)n−1 X n−3 X 2 − a21 X − a2k
k=2

et (∗) montre que


à n−1
!
X
∆n (a1 , a2 , . . . , an )(X) = (−1)n X n−2 X 2 − a12 X − a2k − (−X)n−2 a2n
k=2

ce qui démontre que


à n
!
X
XM (X) = (−1)n X n−2 X 2 − a21 X − a2k
k=2

d. Remarquons que, en prenant la somme de toutes les lignes à la prmière, dans le


déterminant suivant nous obtenons
 
a2 − X ab ab b2
2
 ab a −X b2 ab 
XM (X) = det  
ab b2 a2 − X ab
2
b ab ab a2 − X
 
(a + b) − X (a + b)2 − X
2
(a + b)2 − X (a + b)2 − X
 ab a2 − X b2 ab 
= det  
ab b2 2
a −X ab
b2 ab ab a2 − X
 
1 1 1 1
2
 ab a − X b2 ab 
=((a + b)2 − X) det  
ab b2 2
a − X ab
b2 ab ab a2 − X

on soustrait alors la première colonne des suivantes:


 
1 0 0 0
a2 − ab − X b2 − ab
 ab 0 
XM (X) =((a + b)2 − X) det  
abb2 − ab a2 − ab − X 0
b2ab − b2 ab − b2 a2 − b2 − X
· 2 ¸
2 2 2 a − ab − X b2 − ab
=((a + b) − X)(a − b − X) det
b2 − ab a2 − ab − X

On conclut que

XM (X) = ((a + b)2 − X) ((a − b)2 − X) (a2 − b2 − X)2


20 RÉDUCTION DES ENDOMORPHISMES

e. Remarquons que, en échangeant les lignes d’indices 2 et 3 et ensuite les colonnes


d’indices 2 et 3 dans le déterminant suivant nous obtenons
 
3 − X −5 2 −6
 0 5−X 0 4 
XM (X) = det  
−2 7 −1 − X 11
0 −4 0 −3 − X
 
3−X 2 −5 −6
 −2 −1 − X 7 11 
= det  
0 0 5−X 4
0 0 −4 −3 − X
· ¸ · ¸
3−X 2 5−X 4
= det det
−2 −1 − X −4 −3 − X

d’où, finalement,
XM = (X − 1)4 .

| n . On a
f. Soit (e1 , e2 , . . . , en ) la base canonique de C
n
X n
X
M (ek ) = AR(k−`) e` , et Ω(ej ) = ω (j−1)(k−1) ek .
`=1 k=1

Il en résulte que à n !
n
X X
M Ω(ej ) = ω (j−1)(k−1) AR(k−`) e`
k=1 `=1
n
à n !
X X
= ω (j−1)(k−1) AR(k−`) e`
`=1 k=1
Xn
= Λ(`, j)e`
`=1
n
X
avec Λ(`, j) = ω (j−1)(k−1) AR(k−`) . Mais
k=1

`−1
X n
X
(j−1)(k−1)
Λ(`, j) = ω An+k−` + ω (j−1)(k−1) Ak−`
k=1 k=`
n−1
X n−`
X
(j−1)(p−n+`−1)
= ω Ap + ω (j−1)(p+`−1) Ap
p=n+1−` p=0
n−1
X
= ω (j−1)(p+`−1) Ap
p=0
n−1
X
(j−1)(`−1)
=ω ω (j−1)p Ap = ω (j−1)(`−1) λj
p=0
Solutions 21

n−1
X
avec λj = ω (j−1)p Ap . D’où,
p=0

n
X
M Ω(ej ) = λj ω (j−1)(`−1) e` = λj Ω(ej ).
`=1

Si (A0 , A1 , . . . , An−1 ) = (0, 1, 0, . . . , 0) on a λj = ω j−1 et les nombres (ω j−1 )1≤j≤n


| n (étant vecteurs
sont distincts, donc les vecteurs (Ω(ej ))1≤j≤n forment une base de C
propres associés à des valeures propres distinctes de Γ(0, 1, 0, . . . , 0)). Il en résulte que
(Ω(ej ))1≤j≤n sont les vecteurs propres de Γ(A0 , A1 , . . . , An−1 ) et que les valeurs propres
correspondantes sont données par

n−1
X
λj = ω (j−1)p Ap j ∈ {1, 2, . . . , n}
p=0

et à !
n−1
Y n−1
X
XM = (−1)n X− ω jp Ap .
j=0 p=0

Solution .2 Posons  
0 1 1

A0 = 1 0 1
1 1 0
Nous avons
 
−X 1 1
X A0 = det  1 −X 1  = −X 3 + 3X + 2 = −(X + 1)2 (X − 2).
1 1 −X

Alors, A0 admet −1 comme valeur propre double et 2 comme valeur propre simple.
t
Si V = [x, y, z] est un vecteur propre associé à la valeur propre −1, alors
A0 V = −V , d’où x + y + z = 0. On conclut que l’espace propre de A0 associé à
−1 est engendré par   

1 1
U =  1, V =  −1 
−2 0
Enfin, l’espace propre de A0 associé à 2 est engendré par
 
1
W = 1
1
22 RÉDUCTION DES ENDOMORPHISMES

Notons que la matrice considérée est A = A0 + mI3 . Alors la base (U, V, W )


diagonalise aussi A et on a

AU = (m − 1)U, AV = (m − 1)V, AW = (m + 2)W.

Notons que (A − (m − 1)I3 )(A − (m + 2)I3 )Y = 0 pour Y ∈ {U, V, W } d’où P (A) = 0


avec P (X) = (X − (m − 1))(X − (m + 2)).
Le reste de la division euclidienne de X p par P (X) est un polynôme R(X) de degré
au plus égal à 1, donc R(X) = α(X − (m − 1)) + β(X − (m + 2)):

X p = P (X)Q(X) + α(X − (m − 1)) + β(X − (m + 2)).

D’où, en substituant X par m − 1 et par m + 2 on trouve


1 1
β = − (m − 1)p , α = (m + 2)p .
3 3
Alors, En notant que P (A) = 0,
1
Ap = ((m + 2)p (A − (m + 2)I3 ) − (m − 1)p (A − (m − 1)I3 ).
3

Solution .3 Si z est un vecteur propre de u associé à une valeur propre λ, alors


v ∧ z = λz, d’où les vecteurs z et λz sont orthogonaux et par conséquent λ = 0. D’autre
part, u(x) = 0 équivaut à x ∈ IR.v. On conclut que 0 est l’unique valeur propre de u et
que l’espace propre associé à cette valeur est IRv.

Solution .4 Notons que


 
0 0 0 8
 0 0 0 0
(A − I4 )3 =  .
0 0 0 0
0 0 0 0
On pose alors v4 = t [0, 0, 0, 1] tel que (A − I4 )3 v4 6= 0, et on définit vk = (A − I4 )4−k v4
pour k = 1, 2, 3. On a
       
8 12 4 0
0  4  3 0
v1 =   , v2 =   , v3 =   , v4 =  
0 0 2 0
0 0 0 1
et Av1 = v1 , Av2 = v2 + v1 , Av3 = v3 + v2 et Av4 = v4 + v3 .
Ce qui démontre que A et B sont semblables et que P −1 AP = B avec
 
8 12 4 0
0 4 3 0
P = .
0 0 2 0
0 0 0 1
Solutions 23

Solution .5 ♣ Étude de A.
 
a1 a1 ··· a1
 a2 a2 ··· a2 
A=
 ... .. ..  .
. . 
an an ··· an

Si a = t [a1 , a2 , . . . , an ] ∈ C
| n alors, pour tout X = t [x , x , . . . , x ] ∈
1 2 n
| n on a
C
n
X
AX = ( xk ) a. Distinguons deux cas:
k=1
n
X n
X
◦ n
1 . λ= ak 6= 0, On note H l’hyperplan de C | d’équation xk = 0. On a
k=1 k=1
AX = 0 pour tout vecteur X ∈ H. Soit (e2 , e3 , . . . , en ) une base de H, et notons e1 = a.
On a
A(e1 ) = λe1 , et A(ek ) = 0 pour k ∈ {2, 3, . . . , n}

donc A est diagonalisable dans ce cas.


n
X
2◦ . λ = ak = 0, alors Aa = 0 donc A2 = 0 et 0 est l’unique valeur propre de A.
k=1
On conclut que A n’est pas diagonalisable dans ce cas.
♣ Étude de B.  
0 ··· ··· 1
0 ··· 1 0
B=
 ... · ..  .
·· .
1 ··· ··· 0
| n . On a Be
Soit (e1 , e2 , . . . , en ) la base canonique de C k = en+1−k . Il en résulte que

B 2 = In , le polynôme P (X) = X 2 − 1 est scindé, a toutes ses racines simples, et est


annulé par B. On conclut que B est diagonalisable.
Plus précisément, si l’on pose vk = ek + en+1−k pour k ∈ {1, 2, . . . , E((n + 1)/2)}
et vk = ek − en+1−k pour k ∈ {E((n + 1)/2) + 1, . . . , n}, alors
 n+1

 vk si 1 ≤ k ≤ E( )
2
B(vk ) =

 −v si E( n + 1 ) < k ≤ n
k
2
Donc 1 est une valeur propre d’ordre E((n + 1)/2) de B et −1 est une valeur propre
d’ordre E(n/2) de B.
♣ Étude de C.  
0 ··· ··· c1
 0 ··· c2 0
C=
 ... · ..  .
·· . 
cn ··· ··· 0
24 RÉDUCTION DES ENDOMORPHISMES

| n . On a Ce
Soit (e1 , e2 , . . . , en ) la base canonique de C k = cn+1−k en+1−k . On
√ √
pose vk = ck ek + cn+1−k en+1−k pour k ∈ {1, 2, . . . , E((n + 1)/2)} et vk =
√ √
ck ek − cn+1−k en+1−k pour k ∈ {E((n + 1)/2) + 1, . . . , n}, alors
 n+1
 √
 ck cn+1−k vk si 1 ≤ k ≤ E( )
2
Cvk =

 − √c c n+1
k n+1−k vk si E( )<k≤n
2
La matrice C est par conséquent diagonalisable. Les vecteurs (vk )1≤n fournissent les
colonnes de la matrice de passage.
♣ Étude de D.  
1 1 1 ··· ··· 1
1 1 0 ··· ··· 0
 .. .. 
1 0 1 . .
 
D =  .. .. .. .. .. ..  .
. . . . . .
 .. 
 . 1 
1 0 0
1 0 ··· ··· 0 1
Notons M = D − In , la réduction de D se ramène à celle de M . Soit (e1 , e2 , . . . , en )
Xn
n
la base canonique de C . On a M e1 =
| ek et M ek = e1 pour k ≥ 2. Soit, alors,
k=2
n
X
1
u1 = e1 , u2 = ek , et vk = ek − u2 pour tout k ∈ {3, . . . , n}.
n−1
k=2
On a

M u1 = (n − 1)u2 , M u2 = u1 , et M vk = 0 pour k ∈ {3, . . . , n}.


√ √ √
Enfin, posons v1 = u1 + n − 1 u2 , et v2 = u1 − n − 1 u2 , et calculons M v1 = n − 1 v1

et M v2 = − n − 1 v2 .
Il en résulte que M (et par conséquent D) est diagonalisable, et les vecteurs (vk )1≤n
fournissent les colonnes de la matrice de passage.

Solution .6 a. Déterminons d’abord le polynôme caractéristique.


 
−X 1 0 0
 3 −X 2 0 
XM (X) = det(M − XI4 ) = det  
0 2 −X 3
0 0 1 −X
   
−X 2 0 3 2 0
= − X det  2 −X 3  
− det 0 −X 3 
0 1 −X 0 1 −X
= − X(−X 3 + 7X) − 3(X 2 − 3) = X 4 − 10X 2 + 9
Solutions 25

Les valeurs propres de M sont donc

λ1 = 1, λ2 = −1, λ3 = 3 et λ4 = −3.

Elles sont toutes simples, donc la matrice M est diagonalisable. Un calcul simple montre
que l’on peut prendre comme vecteurs propres associés à ces valeurs, les vecteurs
       
1 1 1 1
 1   −1  3  −3 
v1 =   , v2 =   , v3 =   et v4 =  ,
−1 −1 3 3
−1 1 1 −1
qui vérifient M vk = λk vk pour k ∈ {1, 2, 3, 4}
Le reste de la division euclidienne de X k par XM (X) est un polynôme Rk (X) de
degré au plus 3. Donc

Rk (X) = (X 2 − 1)(αk (X − 3) + βk (X + 3)) + (X 2 − 9)(γk (X − 1) + δk (X + 1)).

En substituant successivement X par 1, −1, 3 et −3 nous obtenons

(−3)k 3k (−1)k 1
αk = − , βk = , γk = , δk = − .
48 48 16 16
Mais X k = Q(X)XM (X) + Rk (X) donc M k = Rk (M ). Ce qui, tout calcul fait, donne
   
−1 1 −1 1 1 1 1 1
k k
(−3)  3 −3 3 −3  3 3 3 3 3
Mk = −  +  +
8 −3 3 −3 3 8 3 3 3 3
1 −1 1 −1 1 1 1 1
   
3 −1 −1 3 3 1 −1 −3
(−1)k  −3 1 1 −3  1  3 1 −1 −3 
 +  .
8 −3 −1 −1 −3 8 −3 −1 1 3
3 1 1 3 −3 −1 1 3

b. Déterminons d’abord le polynôme caractéristique.


 
1−X 1 0 0
 1 −2 − X 1 0 
XM (X) = det(M − XI4 ) = det  
0 1 −2 − X 1
0 0 1 1−X
   
−2 − X 1 0 1 1 0
=(1 − X) det  1 −2 − X 1  − det  0 −2 − X 1 
0 1 1−X 0 1 1−X
¡ 2
¢
=(1 − X) (2 + X) (1 − X) − (1 − X) + (2 + X) − (X − 1)(X + 2) + 1
2
= ((2 + X)(1 − X) + 1) − (1 − X)2
=(X 2 − 2)(X 2 + 2X − 4)
26 RÉDUCTION DES ENDOMORPHISMES

Les valeurs propres de M sont donc


√ √ √ √
λ1 = 2, λ2 = − 2, λ3 = 5 − 1 et λ4 = − 5 − 1.
Elles sont toutes simples, donc la matrice M est diagonalisable. Un calcul simple montre
que l’on peut prendre comme vecteurs propres associés à ces valeurs, les vecteurs
       
√ 1 √ −1 √1 1√
 2 − 1  2 + 1  5 − 2  −2 − √5 
v1 =  √  , v2 =  √  , v3 =  √  , v4 =  .
2−1 2+1 − 5+2 2+ 5
1 −1 −1 −1
qui vérifient M vk = λk vk pour k ∈ {1, 2, 3, 4}
Le reste de la division euclidienne de X k par XM (X) est un polynôme Rk (X) de
degré au plus 3. Donc
√ √ √ √
Rk = (X 2 −2)(αk (X+1− 5)+βk (X+1+ 5))+(X 2 +2X−4)(γk (X− 2)+δk (X+ 2)).
√ √ √ √
En substituant successivement X par 2, − 2, 5 − 1 et − 5 − 1, nous obtenons
√ √
2− 5 √ k 2+ 5 √
αk = (−1 − 5) , βk = − (−1 + 5)k ,
20√ √20
2− 2 √ k 2+ 2 √ k
γk = (− 2) , δk = ( 2) .
8 8
Mais X k = Q(X)XM (X) + Rk (X) donc M k = Rk (M ). Ce qui, tout calcul fait, donne
√ √ √ √
M k = ( 2)k A1 + (− 2)k A2 + ( 5 − 1)k A3 + (− 5 − 1)k A4
√ √
avec A1 = 2+8 2 (M 2 + 2M − 4I4 )(M + 2I4 ), c’est-à-dire :
√ √ 
2+1 √ 1 √ 1 2+1
1  1 √2 − 1 √2 − 1 1 
A1 = √  
4 2 √ 1 2−1 2−1 √ 1
2+1 1 1 2+1

2− 2

et A2 = 8 (M 2 + 2M − 4I4 )(M − 2I4 ), c’est-à-dire :
√ √ 
2 − 1 √ −1 √ −1 2−1
1  −1 √2 + 1 √2 + 1 −1 
A2 = √  
4 2 √ −1 2 + 1 2 + 1 √ −1
2−1 −1 −1 2−1
√ √
et A3 = − 5+28 5 (M 2 − 2I4 )(M + (1 + 5)I4 ), c’est-à-dire :
 √ √ 
5+2 √ 1 √−1 − 5−2
1  1 √5 − 2 −√ 5 + 2 −1 
A3 = √  
4 5 √−1 − 5+2 5−2 √ 1
− 5−2 −1 −1 5+2
√ √
et finalement A4 = −5+2 8
5
(M 2 − 2I4 )(M + (1 − 5)I4 ), c’est-à-dire :
√ √ 
5 − 2 √ −1 √ 1 2 − 5
1  −1 √5 + 2 −√ 5 − 2 1 
A4 = √  
4 5 −1√ − 5 − 2 5 + 2 √ −1
2− 5 1 −1 5−2
Solutions 27

c. Déterminons d’abord le polynôme caractéristique.


 
2 − X −2 1
XM (X) = det(M − XI3 ) = det  2 −3 − X 2 
−1 2 −X
=X(X + 3)(2 − X) + 4 + 4 − (X + 3) − 4X − 4(2 − X)
= − X 3 − X 2 + 5X − 3 = −(X − 1)2 (X + 3)

Les valeurs propres de M sont donc

λ1 = 1 de multiplicité 2 et λ2 = −3.

Un calcul simple montre que le rang de M − I3 est 1, donc l’espace propre associé à
1 est de dimension 2 ; c’est l’hyperplan d’équation x − 2y + z = 0 dont une base est
donnée par    
2 0
v1 = 1  ,
 v2 = 1 

0 2
Par contre, une base de l’espace propre associé à −3 est
 
1
v3 =  2  .
−1

La matrice M est donc diagonalisable et

M v1 = v1 , M v 2 = v2 , M v3 = −3v3 .

Notons que (M − I3 )(M + 3I3 )vk = 0 pour k ∈ {1, 2, 3}. Alors le polynôme
P (X) = (X − 1)(X + 3) est annulé par M . Le reste de la division euclidienne de
X k par P (X) est un polynôme Rk (X) de degré au plus 1. Donc

Rk (X) = αk (X − 1) + βk (X + 3)).

En substituant successivement X par 1 et −3, nous obtenons

(−3)k 1
αk = − , βk = .
4 4
Mais X k = Q(X)P (X) + Rk (X) donc M k = Rk (M ). Ce qui, tout calcul fait, donne
   
k 1 −2 1 5 −2 1
(−3)   1
k
M =− 2 −4 2 + 2 0 2.
4 4
−1 2 −1 −1 2 3
28 RÉDUCTION DES ENDOMORPHISMES

d. Déterminons d’abord le polynôme caractéristique.


 
−1 − X 0 2
XM (X) = det(M − XI3 ) = det  0 −X 1 
0 −1 1−X
=(−1 − X)(−X + X 2 + 1)
= − X 3 − 1 = −(X + 1)(X + j)(X + j 2 )

où {1, j, j 2 } sont les racines cubiques de 1.


Les valeurs propres de M sont donc

λ1 = −1, λ2 = −j, λ3 = −j 2 .

Les trois valeurs propres de M sont simples, donc la matrice M est diagonalisable.
Un calcul simple montre que l’on peut prendre comme vecteurs propres associés à ces
valeurs, les vecteurs
     
1 1−j 1 − j2
v1 =  0  , v2 =  1  et v3 =  1 
0 −j −j 2

qui vérifient M vk = λk vk pour k ∈ {1, 2, 3}.


On a M 3 = −I3 , alors

 (−1)m I3 si n = 3m
n
M = (−1)m M si n = 3m + 1

(−1)m M 2 si n = 3m + 2

Solution .7 1◦ . On a l’identité matricielle suivante


· ¸ · ¸ · ¸
A B A 0 Ip A−1 B
= .
C D C Iq 0 D − CA−1 B

ce qui permet d’écrire


det M = det A det(D − CA−1 B).

2◦ . On a aussi l’identité matricielle suivante


· ¸ · ¸ · ¸
A B Ip B A − BD−1 C 0
= .
C D 0 D D−1 C Iq

ce qui permet d’écrire


det M = det D det(A − BD−1 C).
Solutions 29

3◦ . Appliquons les résultats précédents à la matrice


· ¸
I A
M= n
B XIp

considérée à coefficients dans le corps de fraction IK(X). Nous obtenons

1
det M = det In det(XIp − BA) = det(XIp ) det(In − AB).
X

D’où, det(XIn − AB) = X n−p det(XIp − BA), ce qui est équivalent à

XAB (X) = (−X)n−p XBA (X).

Il suffit de remplacer A par −A pour avoir le résultat demandé.


4◦ .a. Posons B ∈ Mn×2 (IK) la matrice dont les deux colonnes sont X et Y :
B = [X, Y ]. Il est clair que M = B.t B.
Mais ·t ¸ · ¸
t
t X.X X.Y X2 X.Y
B.B = t t =
X.Y Y.Y X.Y Y2
n
X n
X n
X
2
avec la notation X = x2k , Y = 2
yk2 et X.Y = xk yk .
k=1 k=1 k=1
En utilisant XB.t B (Λ) = (−Λ)n−2 Xt B.B (Λ), on trouve

XM (Λ) = (−Λ)n−2 (Λ2 − (X 2 + Y 2 )Λ + X 2 Y 2 − (X.Y )2 ).

Posons   
cos θ sin θ
 cos 2θ   sin 2θ 
X=
 ...  ,
 Y =
 ...  .

cos nθ sin nθ
Il est immédiat que M = X.t X + Y.t Y . Pour achever les calculs il est nécessaire de
calculer X 2 , Y 2 et XY .

n
X n
2 2 1X
X = cos kθ = (1 + cos 2kθ)
2
k=1 k=1
n
2n − 1 1 X 2ikθ
= + e
4 4
k=−n

2n − 1 1 e2i(n+1)θ − e−2inθ
= +
4 4 e2iθ − 1
2n − 1 sin(2n + 1)θ
= +
4 4 sin θ
30 RÉDUCTION DES ENDOMORPHISMES

d’autre part, Y 2 + X 2 = n donc


2n + 1 sin(2n + 1)θ
Y2 = − .
4 4 sin θ
Enfin,
n
X n
1X
XY = sin kθ cos kθ = sin 2kθ
2
k=1 k=1
à n !
1 X
= Im e2ikθ
2
k=1
cos θ − cos(2n + 1)θ
=
4 sin θ
ce qui donne
µ ¶2
2 2 n2 1 2 sin nθ
X Y − (XY ) = − .
4 4 sin θ
On conclut, Ã µ ¶2 !
n2 1 sin nθ
XM (Λ) = (−Λ)n−2 Λ2 − nΛ + − .
4 4 sin θ
4◦ .b. Clairement,
· ¸
−XIn In
XB (X) = det
A −XIn
1
= det(−XIn ) det(−XIn + A) = det(X 2 In − A)
X
=(−1)n XA (X 2 )
D’où le résultat.

Solution .8 Clairement vect ({I2 , A}) ⊂ Ker Φ. Si E = (Eij )1≤i,j≤2 est la base
canonique de M2 ( C),
| alors
¸ · · ¸
4 0 −2 6
Φ(E11 ) = , Φ(E12 ) = ,
0 2 0 2
· ¸ · ¸
−4 0 0 −4
Φ(E21 ) = , Φ(E22 ) = .
−6 4 −2 0
Il en résulte que dim Im Φ ≥ 2. Comme 4 = dim Im Φ + dim Ker Φ, on en déduit que
dim Ker Φ = 2 et Ker Φ = vect ({I2 , A}). D’autre part
 
0 −2 −4 0
4 6 0 −4 
M = M at(Φ, E) =  .
2 0 −6 −2
0 2 4 0
D’où, XM (X) = X 2 (X 2 − 4). Ceci démontre que Φ est diagonalisable car la dimension
de l’espace propre associé à la valeur propre 0 est 2 et les deux autres valeurs propres
sont simples.
Solutions 31

Solution .9 Notons d’abord que 0 n’est pas valeur propre de u. En effet, u(f ) = 0
implique que Z Z
x 1
∀ x ∈ [0, 1] tf (t) dt + x f (t) dt = 0
0 x

d’où en dérivant Z 1
∀ x ∈ [0, 1] f (t) dt = 0
x

puis en dérivant une deuxième fois

∀ x ∈ [0, 1] f (x) = 0.

Alors, f = 0. On en déduit que 0 n’est pas valeur propre de u.


Soit λ ∈ C
| \ {0} et f ∈ E tels que u(f ) = λf . Alors

Z x Z 1
∀ x ∈ [0, 1] tf (t) dt + x f (t) dt = λf (x)
0 x

cette égalité montre que f est dérivable et que f (0) = 0. D’où en dérivant
Z 1
∀ x ∈ [0, 1] f (t) dt = λf 0 (x)
x

cette égalité montre que f 0 est aussi dérivable et que f 0 (1) = 0. D’où en dérivant

∀ x ∈ [0, 1] f (x) + λf 00 (x) = 0

| tel que λ = 1/ω 2 . Alors


Soit ω ∈ C

f (0) = 0, f 0 (1) = 0, f 00 + ω 2 f = 0.

Ceci implique que f est de la forme x 7→ A cos ωx + B sin ωx, puis la condition f (0) = 0
montre que A = 0, et la condition f 0 (1) = 0 montre que B cos ω = 0. Il en résulte
que si cos ω 6= 0, alors f = 0 et λ = 1/ω 2 n’est pas valeur propre de u. Par contre, si
cos ω = 0, (i.e. ω ∈ π/2 + πZZ), alors x 7→ f (x) = sin ωx est une solution non nulle de
u(f ) = (1/ω 2 )f . On conclut que l’ensemble des valeurs propres de u est
½ ¾
4
λk = 2 : k ∈ IN .
π (2k + 1)2

et l’espace propre associé à λk est engendré par la fonction fk qui est définie par
π
fk (x) = sin (2k + 1)x.
2
32 RÉDUCTION DES ENDOMORPHISMES

Solution .10 Supposons que u admet un polynôme P vecteur propre de u associé


à une valeur propre λ. En comparant les termes du plus haut degré dans l’égalité
u(P ) = λP , il arrive que deg P = 2n.
Soit α un zéro de multiplicité m de P dans C,
| (donc c’est un zéro de multiplicité
m − 1 de P 0 ). Mais l’égalité u(P ) = λP montre que α est un zéro de multiplicité
m de (X − a)(X − b)P 0 (X), donc α ∈ {a, b}. Il en résulte que P est de la forme
µ(x − a)k (x − b)2n−k .
Posons Pk = (x − a)k (x − b)2n−k , un calcul simple montre que

u(Pk ) = (n − k)(b − a)Pk = λk Pk .

Or (λ0 , λ1 , . . . , λ2n ) sont distincts et en nombre 2n + 1 égal à la dimension de E. On


conclut que u est diagonalisable, ses valeurs propres sont les (λk )0≤k≤2n et les vecteurs
propres correspondants sont les (Pk )0≤k≤2n .

Solution .11 Notons que u(X k ) = (a(k + 1) + c)X k + bkX k−1 , donc la matrice de u
dans la base canonique est triangulaire et les valeurs propres de u sont (λk )0≤k≤n avec
λk = a(k + 1) + c. Les valeurs propres de u sont toutes simples et en nombre n + 1 égal
à la dimension de l’espace, donc u est diagonalisable. De plus

u((aX + b)k ) = λk (aX + b)k

Ce qui donne les vecteurs propres de u.

Solution .12 a. Soit v un vecteur propre de M associé à la valeur propre λ. On a


M (Av) = A(M v) = A(λv) = λAv, alors Av appartient à l’espace propre Eλ associé à
la valeur propre λ de M qui est de dimension 1 (Eλ = Cv).
| On conclut que Av ∈ Cv,
|

et par conséquent il existe µ tel que Av = µv.


b. Soient λ1 , . . . , λn les valeurs propres distinctes de M et soit (v1 , . . . , vn ) la base
| n formée des vecteurs propres correspondants.
de C

M v k = λk v k , pour k ∈ {1, . . . , n}.

D’après a. il existe, pour tout k ∈ {1, . . . , n}, un nombre µk tel que Avk = µk vk .
Définissons,
n
Y X − λk
P` (X) =
λ` − λk
k=1
k6=`
Solutions 33
n
X
Et posons P (X) = µj Pj (X). Il est immédiat que P (λk ) = µk pour tout k ∈
j=1
{1, . . . , n}. Soit k ∈ {1, . . . , n}, P (M )vk = P (λk )vk = µk vk = Avk . Alors P (M ) = A,
avec P un polynôme de degré au plus n − 1. D’où le résultat.
c. Un calcul simple montre que XA (X) = −X(X − 1)(X − 16). A admet trois
valeurs propres simples 0, 1, et 16.
Si B est une matrice qui vérifie B 2 = A, alors BA = AB. Donc d’après ce qui
précède B = P (A) où P est un polynôme de degré au plus 2, de plus P (0), P (1) et
P (16) sont les valeurs propres de B. La relation B 2 = A montre alors que (P (0))2 = 0,
(P (1))2 = 1 et (P (16))2 = 16. On conclut qu’il existe (ε, ε0 ) ∈ {−1, 1}2 tels que

P (0) = 0, P (1) = ε, P (16) = 4ε0 .

Mais alors, P (X) = αX(X − 1) + βX(X − 16) avec α = ε0 /60, et β = −ε/15. On a


donc démontré que si B 2 = A, alors il existe (ε, ε0 ) ∈ {−1, 1}2 tels que

ε0 ε
B= A(A − I3 ) − A(A − 16I3 ).
60 15
Il est facile de vérifier que la réciproque est aussi vraie.

  
−1 −1 01 4 −2 2
ε 2ε 
B 2 = A ⇐⇒ B =  −1 −1 1 + −2 1 −1  , (ε, ε0 ) ∈ {−1, 1}2
3 3
1 1 −1 2 −1 1

Solution .13 D’après l’hypothèse on a

P (M ) = P (µ)A + P (ν)B

pour tout polynôme P ∈ C[X]


| tel que P (0) = 0 et deg P ≤ 3. En particulier si
Q(X) = X(X − µ)(X − ν), on a Q(M ) = 0.
Si {0, µ, ν} sont distincts, alors M est diagonalisable car elle annule un polynôme
scindé ayant toutes ses racines simples.
Si µ = 0 et ν 6= 0, alors M 2 = ν 2 B = νM . Donc M (M − νI) = 0 et M est
diagonalisable pour les mêmes raisons.
De même, si µ 6= 0 et ν = 0, on a M (M − µI) = 0, donc M est diagonalisable.
Enfin, si µ = ν 6= 0, on a M (M − µI) = 0, donc M est diagonalisable.
On conclut que dans tous les cas M est diagonalisable.
34 RÉDUCTION DES ENDOMORPHISMES

Solution .14 Le corps C


| est algébriquement clos, donc il existe une matrice
triangulaire supérieure N = (aij ) et une matrice inversible P telles que M = P N P −1 .
– Supposons que XM (X) = (−X)n . Alors pour tout i ∈ {1, . . . , n} on a aii = 0.
Il en résulte que, pour tout k ∈ IN∗ les éléments diagonaux de N k sont nuls, et par
conséquent, pour tout k ∈ IN∗ , Tr (N k ) = 0, mais M k = P N k P −1 donc pour tout
k ∈ IN∗ , Tr (M k ) = 0.
– Supposons que pour tout k ∈ IN∗ , Tr (M k ) = 0. Notons λi = aii . D’après
l’hypothèse on a
n
X

∀ k ∈ IN , λki = 0
i=1

il en résulte, que
n
X
∀ Q ∈ C[X],
|
λi Q(λi ) = 0
i=1

ce qui s’écrit,
p
X
∀ Q ∈ C[X],
|
mi µi Q(µi ) = 0 (∗)
i=1

avec {µ1 , . . . , µp } = {λ1 , . . . , λn }, les (µi )1≤i≤p sont distincts et mi = Card ({j : λj =
µi }).
En prenant pour Q dans (∗) le polynôme Qj qui vérifie Qj (µi ) = δij , on obtient que
µj = 0 pour tout j ∈ {1, . . . , p} ce qui démontre que p = 1 et que tous les (λi )1≤i≤n sont
nuls. On conclut que N est une matrice triangulaire supérieure dont tous les éléments
diagonaux sont nuls, alors N n = 0 et par conséquent M n = 0. La matrice M est donc
nilpotente.
–Supposons que la matrice M est nilpotente. Alors 0 est l’unique valeur propre de
M . D’où XM (X) = (−X)n .

Solution .15 1◦ . Soit e ∈ IRn le vecteur dont toutes les composantes valent 1. Alors
Ae = e, donc 1 est une valeur propre de A.
2◦ . Soit λ une valeur propre de A dans C
| et soit X = t [x , . . . , x ] ∈ C
1 n
| n un vecteur

propre associé à λ.
Posons | x` | = max {| xi | : 1 ≤ i ≤ n}. Comme X 6= 0 alors | x` | > 0. En
explicitant la ligne ` de l’égalité matricielle AX = λX on trouve

n
X
λx` = a`k xk .
k=1
Solutions 35

d’où
n
X n
X
| λ | . | x` | ≤ a`k | xk | ≤ a`k | x` | = | x` | .
k=1 k=1
On conclut que | λ | ≤ 1.
1
3◦ . Soit ω ∈]0, min aii [. Alors la matrice B = (A − ωIn ) est clairement
1≤i≤n 1−ω
λ−ω
stochastique. Si λ est une valeur propre de A dans C, | alors est une valeur
1−ω
propre de B qui est une matrice stochastique, donc d’après 2◦ on a
¯ ¯
¯λ−ω ¯
¯ ¯
¯ 1 − ω ¯ ≤ 1.
Ceci démontre que λ appartient au disque fermé de centre ω et de rayon 1 − ω.
D’où le résultat.

Solution .16 1◦ . Démontrons par récurrence sur k que kak−1 c = ak b − bak .


En effet, d’après l’hypothèse ceci est vrai pour k = 1. Supposons que kak−1 c =
ak b − bak alors,

kak c = ak+1 b − abak et ak c = cak = abak − bak+1

(On a utilisé que a et c commutent). En prenant la somme on trouve

(k + 1)ak = ak+1 b − bak+1 .


m
X
Si P (X) = λk X k alors
k=0
n
X n
X
k k
P (a) b − bP (a) = λk (a b − ba ) c = kλk ak−1 c = P 0 (a) c
k=0 k=1

Supposons que a 6= 0, (le cas contraire est trivial), et soit P un polynôme unitaire
de degré minimal tel que P (a) = 0, alors P 0 (a) 6= 0 et P 0 (a) c = 0 d’où c n’est pas
inversible.
2◦ . Notons E1 = Ker c, un sous-espace vectoriel de E, d’après 1◦ , E1 6= {0}.
Comme ac = ca et bc = cb alors a(E1 ) ⊂ E1 et b(E1 ) ⊂ E1 . Posons alors

a1 : E1 −→ E1 : x 7→ a(x) et b1 : E1 −→ E1 : x 7→ b(x).

Nous avons a1 b1 = b1 a1 . Soit E2 un sous-espace propre de a1 , alors la commutativité


de a1 et b1 montre que b1 (E2 ) ⊂ E2 . Nous pouvons alors considérer

b2 : E2 −→ E2 : x 7→ b(x)

et un sous-espace propre E3 de b2 . Clairement tout vecteur non nul de E3 est un vecteur


propre commun à a, b et c.
36 RÉDUCTION DES ENDOMORPHISMES

Solution .17 Notons t = Tr (M ) et d = det(M ). Clairement (t, d) ∈ ZZ2 et comme


1 = det(M n ) = dn alors d ∈ {−1, +1}.
Identifions matrices de M2 ( C)
| | 2 dans
et applications linéaires de C | 2 . Soit
C
ω = exp(2iπ/n), d’après le théorème de décomposition des noyaux on a
n−1
M
2
C =
|
Ker (M − ω k I2 ). (∗)
k=0

Distinguons deux cas:


– Il existe un seul entier p ∈ {0, 1, . . . , n − 1} tel que Ker (M − ω p I2 ) 6= {0}, alors
d’après (∗) on a M = ω p I, mais M est à coefficients dans ZZ, donc dans ce cas ω p ∈ ZZ ;
c’est à dire ω p ∈ {−1, +1} et A ∈ {−I2 , +I2 }.
– Il existe au moins deux entiers (p, r) ∈ {0, 1, . . . , n − 1}2 tels que

0≤p<r et (Ker (M − ω p I2 ) 6= {0} et Ker (M − ω r I2 ) 6= {0}).

En prenant en considération les dimensions des espaces intervenant dans (∗) on trouve
que dim Ker (M − ω p I2 ) = Ker (M − ω r I2 ) = 1, et

| 2
C = Ker (M − ω p I2 ) ⊕ (Ker (M − ω r I2 ).

Mais alors
XM (X) = X 2 − tX + d = (X − ω r )(X − ω p ).

¦ Si d = −1 alors ω r ω p = d = −1 d’où t = ω p − ω −p ∈ ZZ ∩ iIR = {0}. Par


conséquent XM (X) = X 2 − 1.
¦ Si d = +1 alors ω r ω p = d = +1 d’où p + r = n et par conséquent
2πp
t = ω p + ω −p = 2 cos( ) ∈ ZZ∩] − 2, 2[.
n
(Car t ∈ {2, −2} implique que ω p = ω −p ∈ {1, −1} et par conséquet p = 0, r = n
ou p = r = n/2, si n est pair, ce qui contredit 0 ≤ p < r < n). On conclut que
t ∈ {−1, 0, +1}. Finalement

XM (X) ∈ {X 2 + 1, X 2 − X + 1, X 2 + X + 1}

Nous avons démontré que si M est une matrice de GL2 (ZZ) telle qu’il existe n ∈ IN∗
avec M n = I2 alors P (M ) = 0, où

P (X) ∈ {X − 1, X + 1, X 2 − 1, X 2 + 1, X 2 − X + 1, X 2 + X + 1}

Tous ces polynômes sont diviseurs de X 12 − 1 d’où M 12 = I2 .


Solutions 37

Solution .18 1◦ .a. L’espace E est de dimension finie n, alors (x, f (x), . . . , f n (x))
est une famille liée. On pose alors

p = p(x) = min{k : (x, f (x), . . . , f k (x)) est liée }

et
Ef (x) = vect ({x, f (x), . . . , f p−1 (x)}).

1◦ .b. Le système (x, f (x), . . . , f p−1 (x)) est libre à cause de la minimalité de p donc
c’est une base de Ef (x). Il en résulte, toujours à cause de la définition de p, que

p−1
X
p
f (x) = ak f k (x). (∗)
k=0

En particulier f (f p−1 (x)) ∈ Ef (x) d’où f (Ef (x)) ⊂ Ef (x).


1◦ .c. C’est bien connu.
2◦ . En prenant B = (x, f (x), . . . , f p−1 (x)) comme base de Ef (x) nous calculons
 
−λ 0 ··· ··· 0 a0
 1 −λ 0 0 a1 
 
 .. .. . .. .. 
 0 . . .. . . 
Xgx (λ) = det(M at(gx , B) − λIp ) = det 
 .. .. .. . .. .

 . . . .. 0 . 
 . .. . 
 .. . .. −λ ap−2 
0 ··· ··· 0 1 ap−1 − λ

Alors, (voir l’exercice 1.b.)


à p−1
!
X
Xgx (λ) = (−1)p λp − a k λk
k=0

En particulier
à p−1
!
X
Xgx (f )(x) = (−1)p f p (x) − ak f k (x) =0
k=0

en utilisant(∗). Mais Xgx divise Xf alors Xf (f )(x) = 0.


Nous avons alors démontré que Xf (f )(x) = 0 pour tout x ∈ E. Ce qui démontre
que Xf (f ) = 0.
38 RÉDUCTION DES ENDOMORPHISMES

Solution .19 1◦ .a. Nous avons successivement


n n
P 0 (X) X 1 1 X 1
= =
P (X) i=1
X − λi X i=1 1 − λi /X
n
à n
!
1 X X λi k (λi /X)n+1
= ( ) +
X i=1 X 1 − λi /X
k=0
n
X n
Sk 1 X λn+1 i
= + n+1
X k+1 X i=1
X − λi
k=0

1◦ .b. Clairement,
à n
!
1 X 1
P 0 (x) = P (x) Sn−k xk + O( )
xn+1 x2
k=0

soit,  Ã
n n
!
1 X X 1
0
P (x) =  an−j x j
Sn−k x k
+ O( )
xn+1 j=0
x2
k=0

ou bien, en posant ∆ = {0, 1, . . . , n}2 ,


 
2n
X  X  r−n−1 1
P 0 (x) =  a S x + O( )
 n−j n−k  x
r=n+1 (j,k)∈∆
j+k=r
 
n
X X
  `−1 1
=  an−j Sn−k 
 x + O( )
x
`=1 (j,k)∈∆
j+k=`+n
 
n
X  X  `−1 1
=  an−j Si 
 x + O( )
x
`=1 (j,i)∈∆
j=`+i
n
Ãn−` !
X X 1
= an−i−` Si x`−1 + O( )
i=0
x
`=1

1◦ .c. Nous avons immédiatement,


n
X
0
P (x) = `an−` x`−1 .
`=1

ce qui démontre, en combinant avec les résultats précédents,


n
à n−`
!
X X 1
`an−` − an−i−` Si x`−1 = O( )
i=0
x
`=1
Solutions 39

n−`
X
On conclut que, pour ` ∈ {1, . . . , n − 1}, `an−` − an−` S0 = an−i−` Si ou bien
i=1
n−`
X
−(n − `) an−` = an−`−i Si
i=1
ce qui démontre que
k
1X
∀ k ∈ {1, . . . , n − 1} ak = − ak−i Si .
k i=1
Montrons que c’est aussi vrai pour k = n. En effet on a
n
X
P (λj ) = an−i λij = 0
i=0
pour j ∈ {1, . . . , n}, alors en prenant la somme de ces égalités, nous obtenons
Xn
an−i Si = 0 ce qui démontre que
i=0
n
1X
an = − an−i Si
n i=1
D’où les égalités cherchées.
2◦ . Démontrons par récurrence sur k que
k
X
Bk = ak−i Ai . (∗)
i=1
En effet si k = 1 c’est trivialement vérifiée. Supposons que cette relation est vraie pour
k < n alors
Xk k+1
X
Bk+1 = A(Bk + ak I) = A( ak−i Ai ) + ak A = ak+1−i Ai .
i=1 i=1
D’où (∗) pour tout k ∈ {1, . . . , n}. Supposons que
Yn n
X
n n
XA (X) = (−1) (X − λi ) = (−1) bn−k X k .
i=1 k=0
n
X
k
En trigonalisant A, nous trouvons immédiatement que Tr (A ) = λki = Sk . Nous
i=1
allons démontrer par récurrence sur k que ak = bk .
En effet, a0 = b0 = 1 et b1 = −Tr (A) = a1 . Supposons que aj = bj pour
j ∈ {0, 1, . . . , k − 1} alors
k k
1 1X i 1X
ak = − Tr (Bk ) = − ak−i Tr (A ) = − bk−i Si = bk .
k k i=1 k i=1
Ce qui démontre le résultat.
OKMRAN
OUBA
ESPACES PRÉHILBERTIENS

Dans ce chapitre IK désigne IR ou C


|

I. Produit scalaire

Définition : Soit E un IK-espace vectoriel. On appelle produit scalaire sur E, toute


application h·, ·i de E × E dans IK vérifiant:
♥ Pour tout x ∈ E l’application partielle E −→ IK : y 7→ hx, yi est linéaire.
♥ Pour tout (x, y) ∈ E × E, hx, yi = hy, xi.*
♥ Pour tout x ∈ E \ {0} hx, xi ∈ IR∗+ .
Un IK-espace vectoriel E muni d’un produit scalaire s’appelle un espace préhilbertien,
et on le note (E, h·, ·i) ou simplement E s’il n’y a pas d’ambiguı̈té. Un espace
préhilbertien réel de dimension finie s’appelle un espace euclidien, et un espace
préhilbertien complexe de dimension finie s’appelle un espace hermitien.

Remarque : Soient (x1 , x2 , y) ∈ E 3 , et (λ, µ) ∈ IK2 , alors

hλx1 + µx2 , yi =hy, λx1 + µx2 i


=λhy, x1 i + µhy, x2 i = λhx1 , yi + µhx2 , yi.

Il en résulte que, si IK = IR alors, pour tout y ∈ E, l’application partielle


E −→ IK : x 7→ hx, yi est linéaire, et h·, ·i est une forme bilinéaire symétrique définie
positive. Par contre, si IK = C
| alors, pour tout y ∈ E, l’application partielle
E −→ IK : x 7→ hx, yi est anti-linéaire, et h·, ·i est une forme sesquilinéaire hermitienne
définie positive.
Exemples :
♣ Si E est IRn alors le produit scalaire dit usuel sur E est défini par

n
X
hx, yi = xk yk ,
k=1

pour tout x = (x1 , . . . , xn ) et tout y = (y1 , . . . , yn ) de E.

* Evidemment, si IK = IR cette condition devient hx, yi = hy, xi.


2 Espaces préhilbertiens

| n alors le produit scalaire dit usuel sur E est défini par


♣ Si E est C
n
X
hx, yi = xk yk ,
k=1

pour tout x = (x1 , . . . , xn ) et tout y = (y1 , . . . , yn ) de E.


♣ Si E est le C-espace
| vectoriel des fonctions continues sur [0, 1] à valeurs complexes,
noté C([0, 1], C),
| alors nous pouvons définir un produit scalaire sur E en posant
Z 1
hf, gi = f (t) g(t) dt,
0

pour tout (f, g) ∈ E 2 . Plus généralement, si ω ∈ E est strictement positive sur un


sous-ensemble dense de [0, 1], alors
Z 1
hf, giω = f (t) g(t) ω(t) dt,
0

défini aussi un produit scalaire sur E.


♣ Si (E, h·, ·i) est un espace préhilbertien, et T ∈ L(E) un automorphisme de E, alors
on définit un deuxième produit scalaire sur E en posant hhx, yii = hT x, T yi, pour
tout (x, y) ∈ E 2 .

2
Définition : Soit (E, h·, ·i) un espace préhilbertien. Pour x ∈ E on note k x k = hx, xi.
2
L’application de E dans IR+ définie par x 7→ k x k s’appelle la forme quadratique
associée au produit scalaire. D’autre part, l’application de E dans IR+ définie par
p
x 7→ k x k = hx, xi s’appelle la norme* préhilbertienne associée au produit
scalaire.

Connaı̂ssant la forme quadratique on peut retrouver le produit scalaire en utilisant


ce qu’on applle les identités polaires:
– Lorsque E est un espace préhilbertien réel alors, pour tout (x, y) ∈ E 2 et tout
ε ∈ {−1, 1},
2 2 2
k x + εy k = k x k + k y k + 2εhx, yi.

D’où,

1³ 2 2 2
´ 1³
2 2
´
hx, yi = kx + yk − kxk − kyk = kx + yk − kx − yk .
2 4

* Cette appelation sera justifiée par la proposition I.2.


Produit scalaire 3

– Lorsque E est un espace préhilbertien complexe alors, pour tout (x, y) ∈ E 2 et


tout ε ∈ {−1, 1},
2 2 2
k x + εy k = k x k + k y k + 2ε Re hx, yi.

D’où,
1³ 2 2
´
Re hx, yi = kx + yk − kx − yk
4
1³ 2 2
´
Im hx, yi = Re hix, yi = k ix + y k − k ix − y k ,
4
soit,
3
1X k° °2
hx, yi = i ° ik x + y ° .
4
k=0

2iπ
Remarque : On peut, plus généralement, montrer que, si ω = exp avec (n ≥ 3),
n
alors pour tout (x, y) ∈ E 2

n−1
1 X k° °2
hx, yi = ω ° ωk x + y ° .
n
k=0

Proposition I.1. Soit (E, h·, ·i) un espace préhilbertien. Alors

∀ (x, y) ∈ E 2 , | hx, yi | ≤ k x k k y k . (Inégalité de Schwarz)

avec égalité si, et seulement si, x et y son liés.

Preuve : Soit (x, y) ∈ E 2 . Si y = 0, le résultat est immédiat. Supposons donc que


hy, xi
y 6= 0, et posons λ = 2 . Alors
kyk

2 2 2 2
k x − λy k = k x k + | λ | k y k − 2 Re hx, λyi
2
2 | hx, yi |
=kxk − 2 .
kyk

D’où, | hx, yi | ≤ k x k k y k avec égalité si, et seulement si, x = λy.

Proposition I.2. Soit (E, h·, ·i) un espace préhilbertien. Alors, x 7→ k x k est une norme
sur E.
4 Espaces préhilbertiens

Preuve : La seule propriété à vérifier est l’inégalité triangulaire. Soit (x, y) ∈ E 2 ,


d’après l’inégalité de Schwarz

| Re hx, yi | ≤ | hx, yi | ≤ k x k k y k ,

donc,

2 2 2 2 2
k x + y k = k x k + k y k + 2 Re hx, yi ≤ k x k + k y k + 2 k x k k y k = (k x k + k y k)2 .

D’où, k x + y k ≤ k x k+k y k, avec égalité si, et seulement si, Re hx, yi ≥ 0, Im hx, yi = 0,


et {x, y} liés. Par conséquent, il y a égalité dans l’inégalité triangulaire si, et seulement
si, k y k x = k y k x.
La proposition suivante est immédiate.

Proposition I.3. Soit (E, h·, ·i) un espace préhilbertien. Alors

2 2 2 2
∀ (x, y) ∈ E 2 , kx + yk + kx − yk = 2kxk + 2kyk .
(Identité du prallélogramme)

Remarque : L’identité du prallélogramme caractérise les espaces préhilbertiens, c’est


à dire si (E, k · k) est un IK-espace vectoriel normé qui vérifie

2 2 2 2
∀ (x, y) ∈ E 2 , kx + yk + kx − yk = 2kxk + 2kyk .

2
alors il existe un produit scalaire h·, ·i sur E tel que, pour tout x ∈ E, hx, xi = k x k .

Définition : Rappelons certaines définitions concernant les matrices. Si M ∈


Mn×p (IK) alors t M ∈ Mp×n (IK) est la matrice transposée de M , M ∈ Mn×p (IK)
est la matrice conjuguée de M dont les coefficients s’obtienent en prenant les conjugués
des coefficients de M , et M ∗ ∈ Mp×n (IK) est la transposée de la matrice conjuguée de
M , (i.e. M ∗ = t M ). Soit M ∈ Mn (IK).
– On dit que M est symétrique si t M = M .
– On dit que M est hermitienne ou autoadjointe si M ∗ = M .†
– On dit que M est positive si, et seulement si, M ∗ = M et, pour tout X ∈
Mn×1 (IK), X ∗ M X ∈ IR+ .‡
– On dit que M est définie positive si, et seulement si, M ∗ = M et, pour tout
X ∈ Mn×1 (IK) \ {0}, X ∗ M X ∈ IR∗+ .


Si IK = IR cette notion est identique à la précédente.

Si IK = C| alors la deuxième condition implique la première.
Produit scalaire 5

Définition : Soient (E, h·, ·i) un espace préhilbertien, et x1 , x2 , . . . , xm des vecteurs de


E. On appelle matrice de Gram de la suite (x1 , x2 , . . . , xm ), la matrice carrée d’ordre
m dont le coefficient de la iième ligne et la j ième colonne est hxi , xj i.

Proposition I.4. Soient (E, h·, ·i) un espace préhilbertien, et x1 , x2 , . . . , xm des


vecteurs de E. Alors Gram (x1 , . . . , xm ) est une matrice autoadjointe positive. De plus,
les trois conditions suivantes sont équivalentes:
1◦ . Gram (x1 , . . . , xm ) est définie positive.
2◦ . Gram (x1 , . . . , xm ) est inversible.
3◦ . La famille (x1 , . . . , xm ) est libre.
Preuve : Posons G = (gij ) = Gram (x1 , . . . , xm ). Comme gji = hxj , xi i = hxi , xj i =
gij , alors G = G∗ . D’autre part, soit Λ = t [λ1 , . . . , λm ] ∈ Mm×1 (IK). Un calcul simple
montre que ° m °2
°X °
° °
° λk xk ° = Λ∗ G Λ. (∗)
° °
k=1

Ceci démontre que G est positive. De plus, il existe Λ = t [λ1 , . . . , λm ] ∈ Mm×1 (IK)\{0}
Xn
tel que λk xk = 0 si, et seulement s’il existe Λ ∈ Mm×1 (IK) \ {0} tel que Λ∗ G Λ = 0.
k=1
Ceci démontre que (1◦ ⇐⇒ 3◦ ).
Démontrons que (1◦ =⇒ 2◦ ). En effet, si Λ ∈ Mm×1 (IK) vérifie GΛ = 0 alors
Λ∗ G Λ = 0 et par conséquent Λ = 0. L’endomorphisme canoniquement associé à G est
alors injectif et par conséquent il est bijectif.
t
Enfin, si la famille (x1 , . . . , xm ) est liée alors il existe Λ = [λ1 , . . . , λm ] ∈
m
X
Mm×1 (IK) \ {0} tel que λj xj = 0 et par conséquent
j=1

m
X m
X
∀ i ∈ {1, . . . , m}, λj hxi , xj i = λj gij = 0.
j=1 j=1

ce qui s’écrit GΛ = 0 et G n’est pas inversible. Donc (2◦ =⇒ 3◦ ).

Définition : Soient (E, h·, ·i) un espace préhilbertien de dimension n, et E =


(e1 , . . . , en ) une base de E. Alors Gram (E) s’appelle la matrice du produit scalaire
dans la base E, et pour tout X = t [x1 , . . . , xn ] ∈ Mn×1 (IK), et tout Y = t [y1 , . . . , yn ] ∈
Mn×1 (IK) on a
n
X n
X
h xk ek , yk ek i = X ∗ Gram (E) Y.
k=1 k=1
6 Espaces préhilbertiens

1
Exemple : Soit Hn = (aij ) la matrice de Hilbert d’ordre n définie par aij = .
i+j−1
Alors Hn est une matrice définie positive. En effet, si E = IRn−1 [X] muni du produit
scalaire Z 1
hP, Qi = P (t)Q(t) dt,
0

et si l’on considère la base canonique E = (1, X, X 2 , . . . , X n−1 ) de E, alors Hn =


Gram (E).

Proposition I.5. Soient (E, h·, ·i) un espace préhilbertien de dimension finie n, E1 =
(x1 , x2 , . . . , xn ) et E2 = (y1 , y2 , . . . , yn ) des bases de E. Alors

Gram (x1 , x2 , . . . , xn ) = P ∗ Gram (y1 , y2 , . . . , yn )P

où P = Mat (IE , E1 , E2 ).


n
X
Preuve : En effet, si P = (pij ) on a xj = pkj yk pour tout j ∈ {1, . . . , n}. Alors
k=1
X X
hxi , xj i = pki p`j hyk , y` i = (P ∗ )ik hyk , y` ip`j
1≤k,`≤n 1≤k,`≤n

=(P Gram (y1 , y2 , . . . , yn )P )ij .
Ce qui démontre le résultat.

Corollaire I.6. Soient (E, h·, ·i) un espace préhilbertien de dimension finie n, et
E = (x1 , x2 , . . . , xn ) une base de E. Alors det Gram (x1 , x2 , . . . , xn ) ne change pas si l’on
permute les vecteurs x1 , . . . , xn ou si l’on soustrait de l’un des vecteurs x1 , x2 , . . . , xn
une combinaison linéaire des autres.

Preuve : En effet, soit σ une permutation de {1, . . . , n}. Considérons la nouvelle base
F = (xσ(1) , . . . , xσ(n) ), et P = Mat (IE , E, F). On a det P = ε(σ), (la signature de σ),
2
donc det P ∗ det P = | det P | = 1. Mais, en utilisant la proposition précédente, on a

det Gram (E) = det P ∗ det Gram (F) det P = det Gram (F).
n
X
D’autre part, Considérons F = (e
x1 , x2 , . . . , xn ), avec x
e1 = x1 − λk xk . Alors
k=2
 
1 0 ··· ··· 0
 λ2 1 0 0
 . .. 
 .. .. 
P = Mat (IE , E, F) =  .. . . .
 . 
 .. 1 0
λn 0 ··· 0 1
Orthogonalité 7

Alors det P = 1 et par conséquent,

det Gram (x1 , x2 . . . , xn ) = det P ∗ det Gram (e


x1 , x2 , . . . , xn ) det P
= det Gram (e
x1 , x2 , . . . , xn ).

Ce qui démontre le résultat demandé.

II. Orthogonalité

Définition : — Soient (E, h·, ·i) un espace préhilbertien, et (x, y) ∈ E 2 . On dit que x
et y sont orthogonaux (et on écrit x⊥y) si, et seulement si, hx, yi = 0.
— Une famille (xα )α∈A de E est dite orthogonale si, et seulement si,

∀ (α, β) ∈ A × A, α 6= β =⇒ xα ⊥xβ .

— Une famille (xα )α∈A de E est dite orthonormale si, et seulement si, elle est
orthogonale et tous les xα sont de norme 1.

∀ (α, β) ∈ A × A, hxα , xβ i = δαβ .

— Soit B une partie non vide de E. On dit que x ∈ E est orthogonal à B, (et on
écrit x⊥B) si, et seulement si, ∀ y ∈ B, x⊥y. On note B ⊥ l’ensemble des x qui sont
orthogonaux à B.

Exemple Important : Si E désigne l’espace C([0, 2π], C)


| des fonctions continues sur
[0, 2π] muni du produit scalaire
Z 2π
1
∀ (f, g) ∈ E × E, hf, gi = f (t)g(t) dt.
2π 0

Alors la famille (ek )k∈ZZ , définie par ek (x) = eikx , est orthonormale.

Proposition II.1 Une famille orthogonale (xα )α∈A formée de vecteurs non nuls d’un
espace préhilbertien est libre.

Preuve : En effet, toute sous-famille finie (xα1 , . . . , xαm ) admet une matrice de Gram
Gram (xα1 , . . . , xαm ) inversible, car diagonale à coefficients diagonaux non nuls, donc
elle est libre.
8 Espaces préhilbertiens

Proposition II.2 Soient (E, h·, ·i) un espace préhilbertien, (α1 , . . . , αn ) une famille
libre dans E. Alors il existe une, et une seule, famille orthonormale (β1 , . . . , βn ) de E
telle que, pour tout k ∈ {1, . . . , n}, les deux conditions suivantes sont réalisées
1◦ . Vect (β1 , . . . , βk ) = Vect (α1 , . . . , αk ).
2◦ . hαk , βk i ∈ IR∗+ .

Preuve : Démontrons d’abord l’unicité. Supposons qu’il existe deux familles orthonor-
males (β1 , . . . , βn ) et (γ1 , . . . , γn ) telles que, pour tout k ∈ {1, . . . , n}, on ait

1◦ . Vect (β1 , . . . , βk ) = Vect (α1 , . . . , αk ) = Vect (γ1 , . . . , γk ),


2◦ . hαk , βk i ∈ IR∗+ , et hαk , γk i ∈ IR∗+ .

Soit k ∈ {1, . . . , n}. Comme βk ∈ Vect (γ1 , . . . , γk ), alors on peut écrire


k
X
βk = λj γ j
j=1

il en résulte que λj = hγj , βk i par orthonormalité de la suite (γ1 , . . . , γk ). Mais si j < k


alors γj ∈ Vect (β1 , . . . , βj ) et par conséquent λj = hγj , βk i = 0 car (β1 , . . . , βk ) est
orthogonale. On conclut que βk = λk γk . Mais hαk , βk i = λk hαk , γk i donc λk ∈ IR∗+ ,
Enfin la condition k γk k = k βk k = 1 montre que λk = 1 et par conséquent βk = γk .
Nous allons démontrer l’existence par récurrence sur n.
– Si n = 1 on prend β1 = α1 / k α1 k.
– Supposons le résultat vrai pour n. Soit (α1 , . . . , αn+1 ) une famille libre de E.
D’après l’hypothèse de récurrence il existe une famille orthonormale (β1 , . . . , βn ) de E
telle que, pour tout k ∈ {1, . . . , n}, les deux conditions suivantes sont réalisées
1◦ . Vect (β1 , . . . , βk ) = Vect (α1 , . . . , αk ).
2◦ . hαk , βk i ∈ IR∗+ .
On pose alors
n
X
βen+1 = αn+1 − hβk , αn+1 iβk .
k=1

il est facile de voir que βe°n+1 6=° 0 car la famille (α1 , . . . , αn+1 ) est libre, on peut
° °
alors poser βn+1 = βen+1 / ° βen+1 °. Aussi un calcul simple montre que βen+1 ⊥βk pour
° °2
° °
1 ≤ k ≤ n. Ceci implique que ° βen+1 ° = hαn+1 , βen+1 i. Alors hαn+1 , βn+1 i ∈ IR∗+ .
L’égalité
Vect (β1 , . . . , βn+1 ) = Vect (α1 , . . . , αn+1 )
est immédiate. Ce qui achève la démonstration par récurrence.
Orthogonalité 9

Remarque : Le procédé exposé dans le théorème précédent s’appelle orthonormalisa-


tion de Gram-Schmidt.

Le corollaire suivant est immédiat.

Corollaire II.3 Tout espace préhilbertien de dimension finie admet une base orthonor-
male.

Proposition II.4 Soient (E, h·, ·i) un espace préhilbertien de dimension finie n,
x1 , . . . , xm des vecteurs de E. Alors il existe une matrice A ∈ Mn×m (IK) telle que
Gram (x1 , . . . , xm ) = A∗ A.

Preuve : En effet, soit E = (e1 , . . . , en ) une base orthonormale de E. Alors


n
X
∀ i ∈ {1, . . . , m}, xi = hek , xi iek .
k=1

Alors, pour tout (i, j) ∈ {1, . . . , m}2 on a


n X
X n
hxi , xj i = hek , xi ihe` , xj ihek , e` i,
k=1 `=1

ou bien,
n
X
hxi , xj i = hek , xi ihek , xj i.
k=1

Donc, en définissant A = (aij ) ∈ Mn×m (IK) par aij = hei , xj i, on a


n
X
hxi , xj i = aki akj ,
k=1

ce qui prouve que Gram (x1 , . . . , xm ) = A∗ A.

Proposition II.5 Soient (E, h·, ·i) un espace préhilbertien de dimension finie n,
(x1 , . . . , xn ) une famille libre de vecteurs de E. Alors il existe une, et une seule, matrice
A ∈ Mn (IK) triangulaire supérieure à coefficients diagonaux strictement positifs, telle
que Gram (x1 , . . . , xn ) = A∗ A. De plus,
n
Y 2
det Gram (x1 , . . . , xn ) ≤ k xk k ,
k=1

avec égalité si, et seulement si, la famille (x1 , . . . , xn ) est orthogonale.


10 Espaces préhilbertiens

Preuve : Démontrons d’abord l’unicité. Supposons que A∗ A = B ∗ B avec A et B deux


matrices triangulaires supérieures à coefficients diagonaux strictement positifs. Alors
(B ∗ )−1 A∗ = BA−1 . Mais BA−1 est une matrice triangulaire supérieure à coefficients
diagonaux strictement positifs, et (B ∗ )−1 A∗ est une matrice triangulaire inférieure à
coefficients diagonaux strictement positifs. On conclut qu’il existe une matrice diagonale
D à coefficients diagonaux strictement positifs, telle que (B ∗ )−1 A∗ = BA−1 = D. Mais
alors BA−1 = D et AB −1 = D∗ = D et par conséquent D = D−1 , d’où D = I et
A = B.
En utilisant le procédé d’orthonormalisation de Gram-Schmidt, il existe une base
orthonormale E = (e1 , . . . , en ) de E telle que, pour tout k,

Vect (x1 , . . . , xk ) = Vect (e1 , . . . , ek ), et hek , xk i > 0.

Considérons la matrice A = (aij ) ∈ Mn (IK) défine par aij = hei , xj i. On a aii > 0
pour tout i, et aij = 0 si i > j (car dans ce cas ei ⊥ Vect (e1 , . . . , ej ) et xj ∈
Vect (e1 , . . . , ej )). Donc A est bien triangulaire supérieure à coefficients diagonaux
strictement positifs. Enfin, des calculs similaires à ceux de la proposition précédente
montrent que Gram (x1 , . . . , xn ) = A∗ A.
D’autre part,
n
Y n
Y n
Y
det A = akk = hek , xk i ≤ k ek k k xk k . (∗)
k=1 k=1 k=1

Donc,
n
Y
2 2
det Gram (x1 , . . . , xn ) = | det A | ≤ k xk k .
k=1

En regardant (∗) on voit qu’il y a égalité dans cette inégaité si, et seulement si,
hek , xk i = k ek k k xk k pour tout k, ce qui est équivalent à xk = k xk k ek pour tout
k. C’est à dire si, et seulement si, la famille (x1 , . . . , xn ) est orthogonale.

Corollaire II.6 Soient (F, h·, ·i) un espace préhilbertien, et (x1 , . . . , xm ) une famille
de vecteurs de F . Alors,
m
Y 2
det Gram (x1 , . . . , xm ) ≤ k xk k ,
k=1

avec égalité si, et seulement si, la famille (x1 , . . . , xm ) est orthogonale ou l’un de ses
vecteurs est nul.
Orthogonalité 11

Preuve : L’inégalité est immédiate si la famille (x1 , . . . , xm ) n’est pas libre et il y a


égalité si l’un des vecteurs x1 , . . . , xm est nul. Supposons que, la famille (x1 , . . . , xm )
est libre, il suffit dans ce cas d’appliquer la proposition précédente en prenant pour E
l’espace Vect (x1 , . . . , xm ).

Corollaire II.7 (Décomposition de Cholesky) Soit M ∈ Mn (IK) une matrice définie


positive. Alors il existe une, et une seule, matrice A ∈ Mn (IK) triangulaire supérieure
à coefficients diagonaux strictement positifs, telle que M = A∗ A.

Preuve : En effet, considérons sur E = Mn×1 (IK) le produit scalaire défini par
hX, Y i = X ∗ M Y , et E = (e1 , . . . , en ) la base canonique de E. Il est facile de voir
que M = Gram (e1 , . . . , en ), et nous pouvons appliquer la proposition II.5 pour obtenir
le résultat.

Remarque : La réciproque de la proposition précédente est vraie. En effet, si M =


A∗ A avec A inversible alors M est une matrice définie positive. Car d’une part,
M ∗ = A∗ (A∗ )∗ = A∗ A = M , et d’autre part, pour tout vecteur X ∈ Mn×1 (IK)
2
on a X ∗ M X = k AX k où k · k est la norme associée au produit scalaire usuelle sur
Mn×1 (IK) ∼
= IKn .

Corollaire II.8 (Inégalité de Hadamard) Soit A = (aij ) ∈ Mn (IK) une matrice


d’ordre n. Alors à !1/2
n
Y n
X 2
| det A | ≤ | aij | .
j=1 i=1

Preuve : On peut supposer que A est inversible. Considérons sur E = Mn×1 (IK)
le produit scalaire défini par hhX, Y ii = hAX, AY i où h·, ·i est le produit scalaire
canonique sur E ∼ = IKn , et E = (e1 , . . . , en ) la base canonique de E. Il est facile de
voir que A∗ A = Gram hh,ii (e1 , . . . , en ), et nous pouvons appliquer le corollaire II.6 pour
obtenir
n
Y
| det(A∗ A) | ≤ hhej , ej ii,
j=1

ce qui se traduit par

n q n
à n
!1/2
Y Y X 2
| det A | ≤ hAej , Aej i = | aij | .
j=1 j=1 i=1
12 Espaces préhilbertiens

III. Projections orthogonales

Théorème III.1. Soient F un sous-espace vectoriel d’un espace préhilbertien E, et


β ∈ E.
1◦ . Un élément α ∈ F est une meilleure approximation de β par un élément de F ,
(i.e. ∀ γ ∈ F, k β − α k ≤ k β − γ k ), si, et seulement si, β − α⊥F .
2◦ . Si une meilleure approximation de β par un élément de F existe alors elle est
unique.

3 . Si F est de dimension finie, et si E = (e1 , . . . , en ) est une base orthonormale
de F alors
n
X
α= hek , βiek
k=1

est la meilleure approximation de β par un élément de F .

Preuve : Nous allons effectuer la démonstration dans le cas IK = C,


| le cas réel étant
plus simple et laissé au lecteur.
1◦ . Soit γ ∈ F on a

2 2 2
k β − γ k = k β − α k + k α − γ k + 2 Re hβ − α, α − γi.

Alors, si β − α⊥F on a hβ − α, α − γi = 0 et par conséquent k β − γ k ≥ k β − α k. Donc


α est une meilleure approximation de β par un élément de F .
Inversement, si k β − γ k ≥ k β − α k pour tout γ ∈ F alors

2
∀ γ ∈ F, k α − γ k + 2 Re hβ − α, α − γi ≥ 0,

ou bien,
2
∀ δ ∈ F, k δ k + 2 Re hβ − α, δi ≥ 0,

ce qui implique,

2
∀ δ ∈ F, ∀ (x, θ) ∈ IR2 , x2 k δ k + 2x Re hβ − α, eiθ δi ≥ 0,

puis,
∀ δ ∈ F, ∀ θ ∈ IR, Re hβ − α, eiθ δi = 0,

et finalement ∀ δ ∈ F, hβ − α, δi = 0, c’est à dire β − α⊥F .


Projections orthogonales 13

2◦ . Si α1 et α2 sont deux meilleures approximations de β par des éléments de F


alors α1 − β⊥F et α2 − β⊥F , donc α1 − α2 ⊥F . Mais α1 − α2 est un élément de F ,
2
d’où, k α1 − α2 k = hα1 − α2 , α1 − α2 i = 0, et α1 = α2 .
3◦ . On vérifie immédiatement que si
n
X
α= hek , βiek ,
k=1

alors β − α⊥F .

Définition : Soient F un sous-espace vectoriel d’un espace préhilbertien E, et β ∈ E.


– S’il existe α ∈ F la meilleure approximation de β par un élément de F , alors α
s’appelle la projection orthogonale de β sur F .
– Si tout élément de E a une projection orthogonale sur F , alors l’application
PF : E −→ E qui à un élément de E associe sa projection orthogonale sur F s’appelle
la projection orthogonale de E sur F .

Corollaire III.2. Soient F un sous-espace vectoriel de dimension finie d’un espace


préhilbertien E. Alors la projecion orthogonale PF de E sur F existe et c’est une
projection linéaire qui admet F pour image et F ⊥ pour noyau. De plus I − PF est
la projecion orthogonale PF ⊥ de E sur F ⊥ .

Preuve : En effet, d’après la proposition précédente, si E = (e1 , . . . , en ) est une base


orthonormale de F alors
n
X
∀ x ∈ E, PF (x) = hek , xiek .
k=1

ce qui démontre la linéarité de PF . D’autre part, si x ∈ F alors x est la meilleure


approximation de x par un élément de F ! donc, pour tout x ∈ F , PF (x) = x et par
suite PF ◦PF = PF . Il est immédiat que Im PF = F . De l’autre côté x ∈ Ker PF si,
et seulement si, x est orthogonal aux éléments de la base E de F c’est à dire si, et
seulement si, x ∈ F ⊥ .
Finalement, si β ∈ E et α = β − PF (β), alors β − α⊥F ⊥ donc α = PF ⊥ (β). Par
suite PF ⊥ = I − PF .

Corollaire III.3. Soient F un sous-espace vectoriel de dimension finie d’un espace


préhilbertien E. Alors E = F ⊕ F ⊥ .
C’est immédiat.
14 Espaces préhilbertiens

Corollaire III.4. Soient E un espace préhilbertien, et (e1 , . . . , en ) un système or-


thonormale. Alors
n
X 2 2
∀ x ∈ E, | hek , xi | ≤ k x k , (Inégalité de Bessel)
k=1

avec égalité, si et seulement si, x ∈ Vect (e1 , . . . , en ).

Preuve : Notons F = Vect (e1 , . . . , en ), pour tout x ∈ E, on a

2 2 2
k x − PF (x) k + k PF (x) k = k x k .

2 2
Alors k PF (x) k ≤ k x k avec égalité si, et seulement si, PF (x) = x ⇐⇒ x ∈ F . Il
suffit alors de remarquer que
n
X
2 2
k PF (x) k = | hek , xi | .
k=1

Proposition III.5. Soient F un sous-espace vectoriel de dimension finie d’un espace


préhilbertien E, et E = (x1 , x2 , . . . , xn ) une base de F . Alors

det Gram (x, x1 , x2 , . . . , xn )


∀ x ∈ E, d2 (x, F ) = .
det Gram (x1 , x2 , . . . , xn )

Preuve : Soit x ∈ E, en utilisant le corollaire I.6 nous pouvons écrire

det Gram (x, x1 , x2 , . . . , xn ) = det Gram (x − PF (x), x1 , x2 , . . . , xn ).

Mais, x − PF (x)⊥xk pour tout k donc


 2

k x − PF (x) k 0 ············ 0
 0 
 
 .. 
 . 
Gram (x − PF (x), x1 , x2 , . . . , xn ) = 
 .. .

 . Gram (x1 , . . . , xn ) 
 .. 
 . 
0
On conclut que

2
det Gram (x, x1 , x2 , . . . , xn ) = k x − PF (x) k det Gram (x1 , x2 , . . . , xn ).

Ce qui donne le résultat car k x − PF (x) k = d(x, F ).


Projections orthogonales 15

Exemple : Soit E = IR[X] muni du produit scalaire défini par


Z 1
hP, Qi = P (t)Q(t) dt,
0
2 n−1
et F = Vect (1, X, X , . . . , X ). Cherchons d(X m , F ) pour m ≥ n. Si H =
Gram (1, X, . . . , X n−1 , X m ) alors
 1 1
... ... 1 1 
2 n m+1
 
 1 1 1 1 
 2 3 ... ... n+1 m+2 
 
 .. .. .. .. 
 
H =  .. .
..
.
..
.
.. 
 . 
 . . . . 
 1 1 1 1 
 n n+1 ... ... 2n−1 m+n 
 
1 1 1 1
m+1 m+2 ... ... m+n 2m+1
En soustrayant la dernière colonne des autres on trouve
 m m−1 m−n+1 1 
m+1 2(m+1) ... ... n(m+1) m+1
 
 m m−1 m−n+1 1 
 2(m+2) . . . . . . (n+1)(m+2) 
 3(m+2) m+2 
 .. .. .. .. 
 . . . . 
det H = det 
 .. .. .. ..


 . . . . 
 
 m m−1 m−n+1
. . . . . . (2n−1)(m+n) 1 
 n(m+n) (n+1)(m+n) m+n 
 
m m−1 m−n+1 1
(m+1)(2m+1) (m+2)(2m+1) . . . . . . (m+n)(2m+1) 2m+1
Ce qui permet d’écrire

 1 1
... ... 1
1
2 n
 
 1 1 1 
 2 3 ... ... n+1 1
 
(m!)2  .. .. .. .. 
 .
det H = det  .. .
..
.
.. .. 
(2m + 1)(m + n)! (m − n)!  . 
 . . . .
 1 1 1 
 n n+1 ... ... 2n−1 1
 
1 1 1
m+1 m+2 ... ... m+n 1
Puis, en soustrayant la dernière ligne des autres dans ce dernier déterminant on
obtient après arrangement
 1 1 
1 2 ... n
 
(m!)4 1 1 1 
 ... 
det H = det  2 3 n+1 
2 2
[(m − n)!] [(m + n)!] (2m + 1) . .. .. 
 .. . . 
1 1 1
n n+1 ... 2n−1
16 Espaces préhilbertiens

Ce qui démontre

det Gram (1, X, X 2 , . . . , X n−1 , X m ) (m!)4


= .
det Gram (1, X, X 2 , . . . , X n−1 ) [(m − n)!]2 [(m + n)!]2 (2m + 1)

Dóù,

(m!)2
d(X m , Vect (1, X, . . . , X n−1 )) = √ .
(m − n)! (m + n)! 2m + 1

IV. Formes linéaires et adjoints

Proposition IV.1. Soient (E, h·, ·i) un espace préhilbertien de dimension finie, f ∈ E ∗
une forme linéaire sur E. Alors il existe un, et un seul, β ∈ E tel que, pour tout x ∈ E,
f (x) = hβ, xi.

Preuve : Soit E = (e1 , . . . , en ) une base orthonormale de E.


Supposons qu’il existe β ∈ E tel que, pour tout x ∈ E, f (x) = hβ, xi. Alors

n
X n
X n
X
hβ, xi = f ( hek , xiek ) = hek , xif (ek ) = h f (ek )ek , xi.
k=1 k=1 k=1

n
X
Il en résulte que β = f (ek )ek , ce qui démontre l’unicité de β.
k=1
n
X
Inversement, posons β = f (ek )ek . Alors, f (ej ) = hβ, ej i pour tout j ∈
k=1
{1, . . . , n}, et par linéarité f (x) = hβ, xi pour tout x de E.

Corollaire IV.2. Soient (E, h·, ·i) un espace préhilbertien de dimension finie. On
munit E de la norme associée au produit scalaire, et E ∗ de la norme des applications
linéaires continues. Alors l’application

ψ : E −→ E ∗ : β 7→ fβ , où fβ (x) = hβ, xi

est une bijection anti-linéaire isométrique.


Formes linéaires et adjoints 17

Preuve : Le fait que ψ est bijective c’est l’énoncé de la proposition IV.1. De plus, pour
tout (β, α) ∈ E 2 , tout (λ, µ) ∈ IK2 , et tout x ∈ E on a

ψ(λβ + µα)(x) = hλβ + µα, xi = λhβ, xi + µhα, xi = (λψ(β) + µψ(α))(x).

Alors, pour tout (β, α) ∈ E 2 , et tout (λ, µ) ∈ IK2 , on a

ψ(λβ + µα) = λψ(β) + µψ(α).

On conclut que ψ est anti-linéaire. Enfin,

k ψ(β) k = k fβ k = sup{| hβ, xi | : k x k ≤ 1} = k β k .

La dernière égalité résulte de l’inégalité de Cauchy-Schwarz et du cas d’égalité dans


cette dernière.

Proposition IV.3. Soient (E, h·, ·iE ) et (H, h·, ·iH ) deux IK-espaces préhilbertiens de
dimensions finies, et soit u ∈ L(E, H). Alors il existe une, et une seule, application
linéaire u∗ ∈ L(H, E) telle que

∀ (x, y) ∈ E × H, hy, u(x)iH = hu∗ (y), xiE .

De plus, si l’on munit L(E, H) (resp. L(H, E), L(E, E) et L(H, H) ) de la norme des
applications linéaires continues alors
p p
∀ u ∈ L(E, H), k u k = k u∗ k = k u u∗ k = k u∗ u k.

Preuve : Fixons y ∈ H, l’application x 7→ hy, u(x)iH est une forme linéaire sur
E donc, d’après la proposition IV.1, il existe un élément unique u∗ (y) ∈ E tel que
∀ x ∈ E, hy, u(x)iH = hu∗ (y), xiE .
L’application de H dans E qui à y associe u∗ (y) est linéaire, en effet, pour tout
(y, z) ∈ H 2 , tout (λ, µ) ∈ IK2 , et tout x ∈ E on a

hu∗ (λy + µz), xiE =hλy + µz, u(x)iH


=λhy, u(x)iH + µhz, u(x)iH
=λhu∗ (y), xiE + µhu∗ (z), xiE
=hλu∗ (y) + µu∗ (z), xiE ,

d’où u∗ (λy + µz) = λu∗ (y) + µu∗ (z). On conclut que u∗ ∈ L(H, E) et on l’appelle
l’adjoint de u.
18 Espaces préhilbertiens

D’autre part, notons A = sup{| hy, u(x)iH | : k x kE ≤ 1, k y kH ≤ 1}. On a, en


utilisant l’inégalité de Cauchy-Schwarz et le cas d’égalité dans cette dernière,

A = sup{k u(x) kH : k x kE ≤ 1} = k u k .

Mais, aussi
A = sup{| hx, u∗ (y)iE | : k x kE ≤ 1, k y kH ≤ 1}
= sup{k u∗ (y) kE : k y kH ≤ 1} = k u∗ k .
Il en résulte que k u k = k u∗ k. D’autre part, pour tout x ∈ E, on a

2 2
k u(x) kH = hu(x), u(x)iH = hu∗ u(x), xiE ≤ k u∗ u(x) k k x kE ≤ k u∗ u k k x kE .
p 2
Par conséquent, k u k ≤ k u∗ u k, mais k u∗ u k ≤ k u∗ k k u k = k u k . Alors k u k =
p
k u∗ u k.
p
En appliquant ce qui précède à u∗ , on trouve k u∗ k = k u u∗ k. Ceci achève la
démonstration.

Proposition IV.4. Soient (E, h·, ·iE ) et (F, h·, ·iF ) deux IK-espaces préhilbertiens de
dimensions finies, E = (e1 , . . . , en ) une base orthonormale de E et F = (f1 , . . . , fm )
une base orthonormale de F . Si u ∈ L(E, F ). Alors

Mat (u∗ , F, E) = [ Mat (u, E, F)]∗ .

Preuve : En effet, si (aij ) = Mat (u, E, F) et (bij ) = Mat (u∗ , F, E) alors, aij =
hfi , u(ej )iF et bij = hei , u∗ (fj )iE et par conséquent,

bij = hu(ei ), fj iF = hfj , u(ei )iF = aji .

Proposition IV.5. Soient E, F et G des IK-espaces préhilbertiens de dimensions finies.


Alors
¦ Pour tout u et v de L(E, F ) on a (u + v)∗ = u∗ + v ∗ .
¦ Pour tout λ ∈ IK et tout u ∈ L(E, F ) on a (λu)∗ = λu∗ .
¦ Pour tout u ∈ L(E, F ) on a (u∗ )∗ = u.
¦ Pour tout u ∈ L(E, F ) et v ∈ L(F, G) on a (v ◦u)∗ = u∗ ◦v ∗ .
C’est immédiat.
Applications linéaires orthognales 19

Définition : Soient E un espace préhilbertien de dimension finie, et u ∈ L(E).


— On dit que u est autoadjoint si, et seulement si, u∗ = u. Si IK = IR, on dit dans
ce cas que u est symétrique et si IK = C
| qu’il est hermitien.

— On dit que u est autoadjoint positif si, et seulement si, u∗ = u et

∀ x ∈ E, hx, u(x)i ≥ 0.

— On dit que u est autoadjoint défini positif si, et seulement si, u∗ = u et

∀ x ∈ E \ {0}, hx, u(x)i > 0.

— Si IK = IR et u∗ = u−1 , on dit que u est orthogonal.


| et u∗ = u−1 , on dit que u est unitaire.
— Si IK = C

Exemple : Une projection orthogonale est autoadjoint. En effet, soit P une projection
orthogonale d’une espace préhilbertien E, pour tout (x, y) ∈ E 2 ,

hx, P (y)i =hx − P (x) + P (x), P (y)i = hP (x), P (y)i


=hP (x), P (y) − y + yi = hP (x), yi.

V. Applications linéaires orthognales

Dans cette section IK = IR

Définition : Soient (E, h·, ·iE ) et (H, h·, ·iH ) deux espaces préhilbertiens réels, et
u ∈ L(E, H). On dit que u conserve le produit scalaire si, et seulement si,

∀ (x, y) ∈ E × E, hu(x), u(y)iH = hx, yiE ,

ce qui, d’après les identités polaires, est équivalent à

∀ x ∈ E, k u(x) kH = k x kE .

On dit aussi que u est un isomorphisme isométrique si, et seulement si, u est une
bijection linéaire qui conserve le produit scalaire.

Remarque : Si u ∈ L(E, H) conserve le produit scalaire alors u est injectif.


20 Espaces préhilbertiens

Proposition V.1. Soient E et H deux espaces euclidiens de même dimension, et


u ∈ L(E, F ). Les assertions suivantes sont équivalentes:
1◦ . u conserve le produit scalaire.
2◦ . u est un isomorphisme isométrique.
3◦ . L’image par u de toute base orthonormale de E est une base orhtonormale de H.
4◦ . L’image par u d’une base orthonormale de E est une base orhtonormale de H.

La démonstration est très simple et directe, elle est laissée au lecteur.

Corollaire V.2. Deux espaces euclidiens sont isométriquement isomorphes si, et


seulement s’ils ont la même dimension.

Preuve : Soient E et H deux espaces euclidiens. S’ils sont isomorphes alors, ils sont
de même dimension. Inversement, supposons que E et H sont de même dimension, on
considère E = (e1 , . . . , en ) une base orthonormale de E, et H = (h1 , . . . , hn ) une base
orthonormale de H. L’application linéaire qui à ek associe hk pour tout k ∈ {1, . . . , n},
est un isomorphisme isométrique d’après la proposition V.1.

Définition : Soit E un espace préhilbertien réel. Un endomorphisme u ∈ L(E) est


dit orthogonal si, et seulement si u est un isomorphisme isométrique. L’ensemble des
endomorphismes orthogonaux muni de la composition des applications est un groupe
noté O(E) et dit le groupe orthogonal de E.

Remarque : Soit E un espace euclidien. Il est immédiat de voir qu’un endomorphisme


u de E appartient au groupe orthogonal O(E) si, et seulement si, u∗ u = IE . (Et dans
ce cas u u∗ = IE ).

Nous utiliserons dans la suite la notation O(n) pour désigner l’ensemble des
matrices A ∈ Mn (IR) qui vérifient t A A = In . C’est un sous-groupe du groupe des
matrices inversibles.
Soit E un espace euclidien de dimension n, et E = (e1 , . . . , en ) une base orthonor-
male de E. Alors l’application de O(E) dans O(n) qui à u associe Mat (u, E) est un
isomorphisme de groupes.

Proposition V.3. Soit M une matrice carrée réelle inversible d’ordre n, (i.e. M ∈
GLn (IR) ). Alors il existe un couple unique (O, T ) avec O une matrice orthogonale, et T
une matrice triangulaire supérieure à coefficients diagonaux positifs, tel que M = OT .
(Décomposition D’Iwasawa).
Le groupe orthogonal 21

Preuve : Posons G = t M M . G est une matrice définie positive. Donc, en utilisant le


corollaire II.7, il existe une matrice triangulaire supérieure T à coefficients diagonaux
strictement positifs telle que G = t T T .
Comme t M M = t T T alors M = t (T M −1 )T et t (T M −1 ) = M T −1 = (T M −1 )−1 .
On pose alors O = t (T M −1 ). On a t O = O−1 donc O ∈ O(n), et M = OT .
Montrons l’unicité de cette décomposition. En effet, si M = OT = O0 T 0 avec
O et O0 des matrices orthogonales et T , T 0 des matrices triangulaires supérieures à
coefficients diagonaux strictement positifs, alors t O O0 = T T 0−1 . Mais alors T T 0−1 est
à la fois une matrice triangulaire supérieure à coefficients diagonaux strictement positifs
et une matrice orthogonale. Ceci implique immédiatement que T T 0−1 = In donc T = T 0
puis O = O0 .

VI. Le groupe orthogonal

Dans cette section IK = IR

Soit E un espace euclidien, et O(E) le groupe orthogonal de E. Si u ∈ O(E)


alors u∗ u = IE et par conséquent det u ∈ {−1, 1}. L’application u 7→ det u est un
homormorphisme de groupes entre O(E) et ({−1, 1}, ·). Ce morphisme est surjectif dès
que dim E > 0. Le noyau de ce morphisme est noté habituellement SO(E) ou O+ (E).

O+ (E) = {u ∈ O(E) : det u = 1}.

Nous notons aussi

O− (E) = {u ∈ O(E) : det u = −1} = O(E) \ O+ (E).

O+ (E) est un sous-groupe de O(E) dont les éléments s’appellent des isométries positives
ou des rotations. Par contre les éléments de O− (E) s’appellent des isométries négatives
ou des retournements.

Exemple : Les symétries orthogonales.


Rappelons qu’un endomorphisme S d’un espace vectoriel E est une symétrie si,
et seulement si, S 2 = IE . Si S est une symétrie de E, on définit E + (S) = E + =
Ker (S −IE ) et E − (S) = E − = Ker (S +IE ). Il est immédiat de voir que E = E + ⊕E − ,
on dit que S est la symétrie par rapport à E + parallèlement à E − .
Si dim E − = 1 on dit que S est une reflexion, et si dim E − = 2 on dit que S est
un renversement.
22 Espaces préhilbertiens

Si E est un espace euclidien et si S ∈ O(E) est une symétrie alors S est dite une
symétrie orthogonale. Si S est une symétrie orthogonale telle que dim E − = 1 alors
elle est dite une réflexion orthogonale.
Enfin, une symétrie S est une symétrie orthogonale si, et seulement si, E + ⊥E − .
Nous laissons la vérification de cette propriété comme exercice au lecteur.

Proposition VI.1. Soient E un espace euclidien, et x ∈ E \ {0}. Alors la réflexion


orthogonale parallèlement à la droite engendrée par x est définie par

hy, xi
∀ y ∈ E, Tx (y) = y − 2 2 x.
kxk

La vérification est immédiate.

Proposition VI.2. Soient E un espace euclidien, et S une symétrie orthogonale. Alors,


Pour tout u ∈ O(E), l’application Se = u◦S ◦u−1 est une symétrie orthogonale. De plus
e = u(E + (S)) et E − (S)
E + (S) e = u(E − (S)).

La vérifivation est très simple et directe.

Théorème VI.3. Soient E un espace euclidien, et u ∈ O(E). On pose Ind(u) pour


désigner codim Ker (u − IE ). Alors u est la composée d’au plus Ind(u) réflexions
orthogonales † . Donc les réflexions orthogonales engendrent le groupe orthogonal.

Preuve : Nous raisonnons par récurrence sur Ind(u).


– Si Ind(u) = 0 alors u = IE .
– Soit k ∈ {1, 2, . . . , n}, et supposons que le résultat est démontré pour tout
v ∈ O(E) tel que Ind(v) ≤ k − 1. Considérons u ∈ O(E) tel que Ind(u) = k. On
pose F = Ker (u − IE ) et G = F ⊥ , on a dim G = k et il existe x ∈ G \ {0}. Soit
y = u(x), comme x ∈
/ F alors y 6= x. Mais k x k = k y k donc hx + y, x − yi = 0.
Soit T = Tx−y la réflexion orthogonale parallèlement à x−y. Alors T (x−y) = y −x
et T (x + y) = x + y ce qui donne T (y) = x. Il en résulte que T ◦u(x) = x.
D’autre part, si z ∈ F alors u(z) = z, puis

hz, yi = hz, u(x)i = hu∗ (z), xi = hz, xi = 0.

Alors x − y⊥F et par conséquent ∀ z ∈ F, T (z) = z.



On convient que IE est la composée de 0 réflexions.
Réduction des endomorphismes symértiques 23

L

On conclut que F IRx ⊂ Ker (T ◦u − IE ). Il en résulte que Ind(T ◦u) < Ind(u).
L’hypothèse de récurrence montre que T ◦u = T1 ◦T2 ◦ · · · ◦Tr avec r ≤ k − 1. D’où,
u = T ◦T1 ◦ · · · ◦Tr , et la démonstration s’achève car r + 1 ≤ k.
Orientation d’un espace euclidien :
Soit E un espace euclidien de dimension n > 0. On note BON (E) l’ensemble des
bases orthonormales E = (e1 , . . . , en ) de E. Si E = (e1 , . . . , en ) et E 0 = (e01 , . . . , e0n ) sont
deux éléments de BON (E), on peut considérer l’unique application orthogonale UE,E 0
définie par UE,E 0 (ei ) = e0i pour tout i ∈ {1, . . . , n}. Ce qui nous permet de définir sur
les éléments de BON (E) la relation binaire: E < E 0 ⇐⇒ UE,E 0 ∈ O+ (E).
< est une relation d’équivalence qui admet exactement deux classes d’équivalences.
En effet, soit E = (e1 , . . . , en ) ∈ BON (E) et posons E 0 = (−e1 , e2 , . . . , en ) c’est aussi
une base orthonormale. Nous allons démontrer que {[E]< , [E 0 ]< } est l’ensemble des
classes d’équivalence BON (E)/<. Si F ∈ BON (E) alors UE,F = UE 0 ,F UE,E 0 , donc
det UE,F = − det UE 0 ,F et par conséquent l’un de ces deux déterminants vaut +1 et par
suite F ∈ [E]< ou F ∈ [E 0 ]< .
Les deux classes d’équivalences de BON (E)/< sont dites des orientations de E.
Orienter E c’est distinguer l’une des deux orientations, ceci revient à choisir un élément
E ∈ BON (E) et à dire que toutes les bases de [E]< sont directes, les autres bases de
BON (E) sont indirectes ou retrogrades.

VII. Réduction des endomorphismes symértiques

Dans cette section E est un espace euclidien de dimension finie n ≥ 1.

Proposition VII.1. Soit u ∈ L(E) un endomorphisme symétrique.


1◦ . Si F est un sous-espace vectoriel de E stable par u, alors F ⊥ est aussi stable
par u.
2◦ . Si λ et µ sont deux valeurs propres distinctes de u, alors les sous-espaces propres
correspondant sont orthogonaux,i.e. Eλ ⊥Eµ .

3 . u admet une valeur propre réelle.

Preuve : 1◦ . Fixons x ∈ F ⊥ . Pour tout y ∈ F , on a hy, u(x)i = hu(y), xi = 0 (car


u(y) ∈ F ). On conclut que u(x) ∈ F ⊥ , et par suite u(F ⊥ ) ⊂ F ⊥ .
2◦ . Soient (x, y) ∈ Eλ × Eµ . On a

λhx, yi = hλx, yi = hu(x), yi = hx, u(y)i = hx, µyi = µhx, yi.


24 Espaces préhilbertiens

Par conséquent, (λ − µ)hx, yi = 0 et par suite hx, yi = 0. On conclut que Eλ ⊥Eµ .


3◦ . Considérons S = {x ∈ E : k x k = 1}, c’est une partie fermée et bornée de E
qui est de dimension finie, alors S est compacte. La continuité de l’application

Φ : S −→ IR, x 7→ hu(x), xi,

sur le compacte S, montre qu’il existe x


e ∈ S tel que

λ = max Φ(x) = Φ(e


x).
x∈S

D’après la définition de λ, pour tout y ∈ E et tout t ∈ IR, nous avons


2
hu(e
x + ty), x
e + tyi ≤ λ k x
e + ty k .

Ce qui est équivalent à dire que, pour tout y ∈ E et tout t ∈ IR ,


2
t2 (hu(y), yi − λ k y k ) + 2t hu(e
x) − λe
x, yi ≤ 0.

Il en résulte que, pour tout y ∈ E, hu(e


x) − λe
x, yi = 0. Ceci prouve que u(e
x) = λe
x et λ
est une valeur propre réelle de u.

Remarque : En notant que si u est un endomorphisme symétrique alors −u l’est aussi,


on voit que les deux nombres

m = min{hu(x), xi : k x k = 1}, et M = max{hu(x), xi : k x k = 1}.

sont des valeurs propres réelles de u.

Théorème VII.2.(Décomposition spéctrale) Soit u ∈ L(E) un endomorphisme


symétrique. Pour λ ∈ Sp (u) on note Pλ la projection orthogonale de E sur Eλ =
Ker (u − λIE ). Alors
1◦ . Pour tout (λ, µ) ∈ Sp (u) × Sp (u), si λ 6= µ alors Pλ ◦Pµ = 0.
X
2◦ . IE = Pλ .
λ∈Sp(u)
X

3 . u= λPλ .
λ∈Sp(u)
De plus, Si A est une partie finie non vide de IR, et (Qλ )λ∈A est une famille de
projections orthogonales non nulles de L(E), qui vérifie les trois assertions précédentes,
alors A = Sp (u) et Qλ = Pλ pour tout λ ∈ A.
Y X −µ
Enfin, pour λ ∈ Sp (u) on pose `λ (X) = ∈ IR[X], alors Pλ =
λ−µ
µ∈Sp(u)\{λ}
`λ (u).
Réduction des endomorphismes symértiques 25

Preuve : Montrons que (Pλ )λ∈Sp(u) vérifie les trois assertions.


Soit (λ, µ) ∈ Sp (u) × Sp (u) tel que λ 6= µ. D’après la proposition précédente,
Eλ ⊥Eµ et alors
Im Pµ = Eµ ⊂ Eλ⊥ = Ker Pλ .

Donc Pλ ◦Pµ = 0. Ce qui démontre 1◦ .


Soit

M
F = Eλ .
λ∈Sp(u)

F est un sous-espace stable par u, donc F ⊥ est aussi un sous-espace stable par u. Si
F 6= E, alors F ⊥ 6= {0} et l’application

v : F ⊥ −→ F ⊥ , x 7→ u(x)

est un endomorphisme symétrique de l’espace euclidien F ⊥ qui est de dimension non


e ∈ F ⊥ \{0} et λ ∈ IR
nulle. Alors v admet une valeur propre réelle. C’est à dire, il existe x
tels que u(e
x) = λe e ∈ (F ⊥ \{0})∩Eλ ⊂ (F ⊥ \{0})∩F = Ø.
x. Mais alors λ ∈ Sp (u). D’où x
Cette contradiction montre que F = E, et par conséquent

M
E= Eλ .
λ∈Sp(u)

X
Si x ∈ E alors il s’écrit d’une manière unique x = xλ avec xλ ∈ Eλ . Mais
λ∈Sp(u)
Pµ (xλ ) = 0 si λ 6= µ donc xλ = Pλ (x). On conclut que
X
∀ x ∈ E, x= Pλ (x).
λ∈Sp(u)

Ce qui démontre 2◦ .
En utilisant le fait que Eλ = Ker (u − λIE ) on trouve u◦Pλ = λPλ . Alors
 
X X X
u = u◦  Pλ  = u◦Pλ = λPλ .
λ∈Sp(u) λ∈Sp(u) λ∈Sp(u)

M
Posons Fλ = Im Qλ 6= {0}. Les deux assertions 1◦ et 2◦ montrent que E = Fλ .
λ∈A
Soit λ ∈ A, d’après 3◦ , pour tout x ∈ Fλ , on a u(x) = λx, et par conséquent
λ ∈ Sp (u) et Fλ ⊂ Eλ . Or
X X X
dim E = dim Fλ ≤ dim Eλ ≤ dim Eλ = dim E.
λ∈A λ∈A λ∈Sp(u)
26 Espaces préhilbertiens

Donc il y a égalité dans toutes les inégalités précédentes ; A = Sp (u) et Fλ = Eλ pour


tout λ ∈ A. Ceci démontre l’unicié de la décomposition spectrale.
Enfin, en utilisant les assertions 3◦ et 1◦ nous voyons immédiatement que
X
uk = λk Pλ .
λ∈Sp(u)

pour tout k ∈ IN∗ et c’est aussi vrai pour k = 0 d’après 2◦ . On conclut que
X
∀ S ∈ IR[X], S(u) = S(λ)Pλ .
λ∈Sp(u)

En particulier, pour tout µ ∈ Sp (u),


X
`µ (u) = `µ (λ)Pλ = Pµ .
λ∈Sp(u)

Ce qui achève la démonstration.

Corollaire VII.3. Soit u ∈ L(E) un endomorphisme symétrique. Alors, il existe une


base orthonormale E = (e1 , . . . , en ) telle que Mat (u, E) soit diagonale.

Preuve : Soit Eλ = Ker (u − λIE ), par conséquent


M
E= Eλ .
λ∈Sp(u)

On obtient la base orthonormale demandée, en choisissant, pour chaque λ ∈ Sp (u),


Eλ une base orthonormale de Eλ , et en prenant E = ∪Eλ .

Voici ensuite la traduction matricielle de ce corollaire:

Corollaire VII.4. Soit A ∈ Mn (IR) une matrice symétrique. Alors, il existe une
matrice orthogonale O ∈ O(n) et une matrice diagonale D ∈ Mn (IR), telles que
A = O D t O.

Proposition VII.5. Soit u ∈ L(E) un endomorphisme symétrique. u est positif (resp.


définie positif) si, et seulement si, Sp (u) ⊂ IR+ (resp. Sp (u) ⊂ IR∗+ ).

Il suffit de diagonaliser u dans une base orthonormale.


Exemples d’application 27

Proposition VII.6. Soient u et v deux endomorphismes de l’espace euclidien E. On


suppose que u symétrique défini positif et v symétrique. Alors, il existe E = (e1 , . . . , en )
une base de E, et (λ1 , . . . , λn ) ∈ IRn tels que
1◦ . ∀ (i, j) ∈ {1, . . . , n}2 , hu(ei ), ej i = δij .
2◦ . ∀ (i, j) ∈ {1, . . . , n}2 , hv(ei ), ej i = λi δij .

Preuve : Posons, pour (x, y) ∈ E × E, hhx, yii = hu(x), yi. Il est clair que hh·, ·ii est
un produit scalaire sur E. Si w = u−1 ◦v alors, pour (x, y) ∈ E × E, on a

hhw(x), yii = hv(x), yi = hx, v(y)i = hu−1 ◦u(x), v(y)i = hu(x), u−1 ◦v(y)i = hhx, w(y)ii.

On conclut que w est symétrique pour le produit scalaire hh·, ·ii. Alors il existe
E = (e1 , . . . , en ) une base orthonormale pour hh·, ·ii, qui diagonalise w. Donc, il existe,
pour chaque i ∈ {1, . . . , n}, un réel λi tel que w(ei ) = λi ei ce qui est équivalent à
v(ei ) = λi u(ei ).
Dire que E est orthonormale pour hh·, ·ii implique que

∀ (i, j) ∈ {1, . . . , n}2 , hu(ei ), ej i = δij .

Puis, comme ∀ i ∈ {1, . . . , n}, v(ei ) = λi u(ei ), on a

∀ (i, j) ∈ {1, . . . , n}2 , hv(ei ), ej i = λi δij .

Corollaire VII.7. Soient A et B deux matrices de Mn (IR). On suppose que A est


symétrique définie positive et B est symétrique. Alors, il existe T ∈ Mn (IR) inversible
et D ∈ Mn (IR) diagonale telles que

A = tT T et B = t T D T.

Preuve : On peut considérer A et B comme les matrices d’endomorphismes de l’espace


euclidien usuel IRn dans la base canonique C = (c1 , . . . , cn ). D’après le théorème
précédent, il existe E = (e1 , . . . , en ) une base de E, et (λ1 , . . . , λn ) ∈ IRn tels que,
pour tout (i, j) ∈ {1, . . . , n}2 on a hA(ei ), ej i = δij et hB(ei ), ej i = λi δij . On note T =
Mat (In , C, E) de telle manière que T −1 soit la matrice dont les colonnes sont les vecteurs
de la base E. Alors pour tout (i, j) ∈ {1, . . . , n}2 on a ht (T −1 )AT −1 (ci ), cj i = δij et
ht (T −1 )BT −1 (ci ), cj i = λi δij . Ce qui s’écrit t (T −1 )AT −1 = In et t (T −1 )BT −1 = D
avec D = Diag(λ1 , . . . , λn ). Ou bien, A = t T T et B = t T D T .
28 Espaces préhilbertiens

VIII. Exemples d’application

♣ Soient A = (aij ) et B = (bij ) deux matrices positives de Mn (IK). Alors la matrice


C = (cij ) ∈ Mn (IK) définie par cij = aij bij est aussi positive.
Supposons d’abord que A est B sont définies positives. D’après le corollaire II.7
il existe deux matrices S = (αij ) ∈ Mn (IK) et T = (βij ) ∈ Mn (IK) telles que
A = S ∗ S et B = T ∗ T . Ce qui implique que
X
cij = aij bij = αki αkj β`i β`j .
1≤k,`≤n

Par conséquent, pour tout X = t [x1 , . . . , xn ] ∈ Mn×1 (IK),


 
X X X
X ∗ CX = xi cij xj =  αki β`i xi αkj β`j xj 
1≤i,j≤n 1≤k,`≤n 1≤i,j≤n
¯ ¯2
X ¯Xn ¯
¯ ¯
= ¯ αki β`i xi ¯ ≥ 0.
¯ ¯
1≤k,`≤n i=1

Alors C est bien une matrice positive.


Supposons que A est B sont seulement positives, On note DA (resp. DB ) le diagonal
de A (resp. B), et nous posons, pour ε > 0, A e = A + εI et B e = B + εI. Il est
e et B
immédiat de voir que A e sont définies positives. Alors, d’après ce qui précède,
e = C + ε(DA + DB ) + ε2 I est positive. Mais ε est arbitraire donc en
la matrice C
faisant tendre ε vers 0 nous concluons que C est positive.
Comme application, considérons A = (aij ) une matrice positive de Mn (IK). En
utilisant ce qui précède nous voyons que, pour tout k ∈ IN∗ , la matrice Ak = (akij )
est positive. La somme de deux matrices
à m positives
! étant positive, nous voyons
X akij
em =
immédiatement que la matrice A est positive pour tout m. En
k!
k=0
faisant tendre m vers l’infini nous trouvons que la matrice  = (eaij ) est aussi
positive.

♣ Soit u un endomorphisme symétrique positif d’un espace euclidien E, et m un


entier supérieur au égal à 2. Alors il existe un unique endomorphisme symétrique
positif v ∈ L(E) tel que v m = u. De plus v est un polynôme en u.
Exemples d’application 29

Montrons d’abord l’unicité. Si v est un endomorphisme symétrique positif v ∈ L(E)


tel que v m = u, alors v admet la décomposition spectrale
X
v= λPλ .
λ∈Sp(v)

où Pλ est la projection orthogonale sur le sous-espace propre de v associé à λ. Mais


alors
X
u = vm = λm Pλ .
λ∈Sp(v)

D’où Sp (u) = {λm : λ ∈ Sp (v)} et par conséquent Sp (v) = { m µ : µ ∈ Sp (u)},

et Ker (v − m µIE ) = Ker (u − µIE ). On conclut que, si u admet la décomposition
spectrale
X
u= µPµ ,
µ∈Sp(u)

(où Pµ est la projection orthogonale sur le sous-espace propre de u associé à µ),


alors
X √
v= m
µPµ .
µ∈Sp(u)

Ceci démontre l’unicité.


X √
Pour l’existence il suffit de poser v = m
µPµ , où Pµ est la projection
µ∈Sp(u)
orthogonale sur le sous-espace propre de u associé à µ. On vérifie immédiatement
que v est symétrique positif tel que v m = u. Enfin, comme les Pµ sont des polynômes
en u alors v est aussi polynôme en u.
30 ESPACES PRÉHILBERTIENS

EXERCICES

Exercice .1 Soient (E, h., .i) un espace préhilbertien, (e1 , e2 , . . . , en ) des vecteurs
de E vérifiant
1◦ . ∀ i, k ei k = 1 ;
P
n
2 2
2◦ . ∀ x ∈ E, | hx, ei i | = k x k .
i=1
Montrer que (e1 , e2 , . . . , en ) est une base orthonormée de E.
Exercice .2 E désigne IR4 muni du produit scalaire usuel. Déterminer la projection
orthogonale de E sur le sous-espace F défini par
( 4 4
)
X X
F = (x1 , x2 , x3 , x4 ) : xi = ixi = 0 .
i=1 i=1

Exercice .3 Considérons (E, h., .i) un espace préhilbertien, et (e1 , . . . , en ) une base
orthonormée de E. Soit (x1 , . . . , xn ) une suite de n vecteurs de E qui vérifie l’inégalité
Xn
2
k ek − xk k < 1. Montrer que (x1 , . . . , xn ) est une base de E.
k=1
Exercice .4 Soient {e1 , . . . , en } des vecteurs d’un espace préhilbertien réel. On
suppose que pour tout i différent de j on a hei , ej i < 0, et qu’il existe x ∈ E tel que
pour tout i on a hx, ei i > 0. Montrer que (e1 , . . . , en ) est libre.
Exercice .5 Soit (E, h., .i) un espace préhilbertien. p une projection de E, (i.e.
2
p = p). Montrer que les propriétés suivantes sont équivalentes
a. p est une projection orthogonale.
b. p∗ = p.
c. Ker p ⊂ ( Im p)⊥ .
d. ∀ x ∈ E, k p(x) k ≤ k x k.
Exercice .6 Soit A une matrice symétrique positive d’ordre n. Montrer que

sup | aij | = sup aii .


1≤i,j≤n 1≤i≤n

Exercice .7 Soient A1 , A2 deux matrices hermitiennes. Montrer que

A1 A2 est hermitienne ⇐⇒ A1 A2 = A2 A1 .
· ¸
4 λ1 − λ2 λ3 + iλ4
Exercice .8 Soit (λ1 , λ2 , λ3 , λ4 ) ∈ IR , on note A = . Montrer
λ3 − iλ4 λ1 + λ2
que si λ1 > 0 et si det(A) = 1 alors A est hermitienne définie positive.
Exercices 31

Exercice .9 Soit O = (aij ) une matrice orthogonale d’ordre n. Montrer que


¯ ¯
¯ X ¯
X √ ¯ ¯
| aij | ≤ n n, ¯ aij ¯¯ ≤ n.
¯
1≤i,j≤n ¯ 1≤i,j≤n ¯

Exercice .10 Soient (E, h., .i) un espace euclidien, P une projection orthogonale de
E et (e1 , e2 , . . . , en ) une base orthonormale de E. Montrer que
n
X 2
k P (ek ) k = rg (P ).
k=1

Exercice .11 Soit (E, h., .i) un espace préhilbertien de dimension finie, T ∈ L(E)
Montrer que
1◦ . ( Im T )⊥ = Ker T ∗ .
2◦ . ( Ker T )⊥ = Im T ∗ .
Exercice .12 Soit E un espace euclidien.
1 . Soit u ∈ L(E). Montrer que u∗ .u est symétrique et que toutes ses valeurs propres

sont positives. On note λmin (u) (resp. λmax (u) ) la plus petite (resp. la plus grande)
des valeurs propres de u∗ .u.
2◦ . Montrer que si u ∈ L(E), on a

2 2 2
∀ x ∈ E, λmin (u) k x k ≤ k u(x) k ≤ λmax (u) k x k .

3◦ . Soient u, v ∈ L(E), Montrer que toute valeur propre ν de u◦v vérifie

2
λmin (u)λmin (v) ≤ | ν | ≤ λmax (u)λmax (v).

Exercice .13 Soit E un espace euclidien de dimension finie m, et soit u ∈ L(E) un


endomorphisme vérifiant u2 = −I.
1◦ . Montrer que m = 2n. (On pourrait considérer le déterminant).
2◦ . On note λ1 = k u k, (c’est la norme d’application linéaire sur E) et on considère
e1 ∈ E tel que k e1 k = 1 et k u(e1 ) k = λ1 , enfin on pose f1 = (1/λ1 )u(e1 ).
a. Calculer u(f1 ), et montrer que λ1 ≥ 1.
2 2
b. Montrer que pour tout t ∈ IR, on a k u(e1 + tf1 ) k ≤ λ21 k e1 + tf1 k . En
déduire que e1 ⊥f1 .
3◦ . Soit y ∈ E tel que y⊥e1 . Montrer que pour tout t ∈ IR, on a

2 2
k u(y + te1 ) k ≤ λ21 (k y k + t2 ).
32 ESPACES PRÉHILBERTIENS

En déduire que u(y)⊥f1 .



4 . Soit y ∈ E tel que y⊥f1 . Montrer que pour tout t ∈ IR, on a

2 2
k y k + t2 ≤ λ21 k u(y + tf1 ) k .

En déduire que u(y)⊥e1 .


5◦ . Soit G1 = vect ({e1 , f1 }) et G2 = G⊥
1 . Montrer que u(G2 ) = G2 .
6◦ . Montrer qu’il existe une base orthonormale {e1 , f1 , e2 , f2 , . . . , en , fn } de E et des
réels λ1 ≥ λ2 ≥ . . . ≥ λn ≥ 1 avec
1
∀ i ∈ {1, . . . , n} u(ei ) = λi fi , u(fi ) = − ei .
λi
Exercice .14 Soit E un espace euclidien de dimension finie, et soit u ∈ L(E) un
endomorphisme. On munit L(E) de la norme des applications linéaires continues sur E.
1◦ . Montrer que k u k = k u∗ k.
2◦ . On suppose que k u k ≤ 1.
a. Montrer que Ker (I − u) = Ker (I − u∗ ).
b. On pose F1 = Ker (I − u), et F2 = Im (I − u). Montrer que F2⊥ = F1 .
c. Pour k ≥ 1 on pose
p−1
1X k
up = u
p
k=0

Montrer que, pour tout x ∈ E, la suite {up (x)}p≥1 converge vers la projection
orthogonale de x sur F1 .
Exercice .15 Dans un espace euclidien orienté, de dimension 3, rapporté à une base
orthonormée directe, caractériser les endomorphismes de matrices
     
−2 6 −3 8 1 −4 2 1 2
1 1 1
A=  6 3 2  , B =  −4 4 −7  , C =  −2 2 1
7 9 3
−3 2 6 1 8 4 −1 −2 2

Exercice .16 Dans un espace euclidien orienté, de dimension 3, on désigne par R




la rotation d’axe dirigé par le vecteur unitaire K et d’angle θ et par ρ une rotation.
Montrer que ρ◦R◦ρ−1 est une rotation dont on précisera l’axe et l’angle.
Exercice .17 Soit E un espace euclidien orienté, de dimension 3, rapporté à une

→ −→ → −
base orthonormée directe B = ( i , j , k ). On désigne par R la rotation d’axe dirigé


par le vecteur unitaire ` et d’angle θ.
1◦ . En utilisant une méthode de changement de base, donner la matrice de R dans la

→ −
→ → − √
base B. Achever les calculs pour θ = π/3 et ` = ( i + j )/ 2.

− → −
→ −
2◦ . Montrer que ` ⊥− x =⇒ R(→ −
x ) = cos θ→

x + sin θ ` ∧ →
x.
Exercices 33

→ → −
→ →− →
3◦ . Montrer que R(−

x ) = cos θ−

x + sin θ ` ∧ −
x + (1 − cos θ) h ` , −
xi ` .
Exercice .18 Soit (a, b, c) ∈ IR3 . Montrer que
 
a b c
M =c a b
b c a

est une matrice de rotation si, et seulement si, a, b et c sont racines d’une équation

x3 − x2 + k = 0, où 0 ≤ k ≤ 4/27.

4 sin2 φ
En posant k = , déterminer explicitement les matrices M correspondantes, ainsi
27
que les axes et les angles des rotations qu’elles représentent.
Exercice .19 Soit A ∈ Mn (IR) une matrice symétrique. Montrer que s’il existe
k ∈ IN, k ≥ 2, tel que Ak = In , alors A2 = In .
Exercice .20 On note E l’espace vectoriel des polynômes à coefficients réels. On
munit E du produit scalaire
Z 1
hP, Qi = P (x)Q(x) dx.
−1

1 dn ¡ 2 ¢
Pour tout entier n on pose Pn (X) = (X − 1) n
. (Le nième polynôme de
2n n! dxn
Legendre).
On notera aussi En le sous espace de E formé de polynômes de degré inférieur ou
égal à n.
I
1◦ . Montrer que pour tout n, Pn est un polynôme de degré n. Quel est le coefficient
de X n dans Pn ?
2◦ . Montrer que P2n est pair et que P2n+1 est impair.
3◦ . Montrer que pour tout polynôme Q on a
Z 1
(−1)n
hPn , Qi = n (x2 − 1)n Q(n) (x) dx.
2 n! −1

(On pourrait effectuer des intégrations par parties successives).


4◦ . Montrer que pour tout n ≥ 1, Pn ⊥En−1 . En déduire que si m 6= n on a
hPn , Pm i = 0.
Z 1
22n+1 (n!)2

5 . Montrer que (1 − x2 )n dx = . En déduire k Pn k.
−1 (2n + 1)!
6◦ . Donner une base orthonormale de En .
34 ESPACES PRÉHILBERTIENS

II
On considère l’application
µ ¶
d 2 dP
µ : E −→ E : P →
7 µ(P ) = (X − 1) .
dX dX
1◦ . Montrer que µ est symetrique i.e. ∀ (P, Q) ∈ E × E, hµ(P ), Qi = hP, µ(Q)i.
2◦ . Montrer que µ(En ) ⊂ En . En déduire que si n 6= m, on a hPn , µ(Pm )i = 0.
3◦ . Montrer qu’il existe λn ∈ IR tel que µ(Pn ) = λn Pn . Déterminer λn .
4◦ . On pose Fn = {P : µ(P ) = n(n + 1)P }. Montrer que tout polynôme de Fn est soit
de degré n soit nul. En déduire que Fn est un sous-espace vectoriel de dimension
1. Donner une base de Fn .
Déduire que l’équation différentielle (x2 − 1)y 00 + 2xy 0 − n(n + 1)y = 0 admet une
solution polynomiale unique à un facteur multiplicatif près. Exprimer cette solution en
fonction de Pn .

III
1◦ . Montrer qu’il existe des constantes αn , βn , γn telles que

XPn = αn Pn+1 + βn Pn + γn Pn−1 . (1)

2◦ . Déterminer αn en comparant les termes de plus haut degré.


3◦ . Déterminer βn en utiliant la parité de Pn .
4◦ . Déterminer γn .
5◦ . En utilisant des techniques similaires, montrer
n(n + 1)
(1 − X 2 )Pn0 = (Pn−1 − Pn+1 ) = −n(XPn − Pn−1 ) = (n + 1)(XPn − Pn+1 )
2n + 1
(2)
puis,
1 ¡ 0 0
¢ 1
Pn = Pn+1 − Pn−1 = (P 0 − XPn0 ). (3)
2n + 1 n + 1 n+1
Que valent Pn (1) et Pn (−1) ?
6◦ . Utiliser la relation (1) pour montrer que
n
X Pn+1 (Y )Pn (X) − Pn+1 (X)Pn (Y )
(2k + 1)Pk (X)Pk (Y ) = (n + 1)
Y −X
k=0

puis
n
X ¡ 0 ¢
(2k + 1)Pk2 (X) = (n + 1) Pn+1 (X)Pn (X) − Pn+1 (X)Pn0 (X) .
k=0
Exercices 35

IV

1◦ . Supposons que Pn admet un zéro α ∈ C


| \ IR. Montrer qu’il existe un polynôme

2
Q de degré n − 2 tel que Pn (X) = (X 2 − (α + α)X + | α | )Q(X). En déduire une
contradiction en calculant hPn , Qi.
2◦ . Utiliser la méthode de 1◦ . pour montrer que
a. Pn n’a pas de zéro dans IR\] − 1, 1[.
b. Pn n’a pas de zéro multiple dans ] − 1, 1[.
3◦ . Que peut-on déduire concernant les zéros de Pn ?
4◦ . Écrire P0 , P1 , P2 et P3 . Déterminer les zéros de ces polynômes.
5◦ . Montrer que Pn et Pn+1 n’ont pas de racine commune et que Pn+1 /Pn est
strictement croissante sur tout inervalle où elle est définie. (On pourrait utiliser
III.6◦ ).
6◦ . En déduire que si {xk }1≤k≤n (resp. {yk }1≤k≤n+1 ) sont les racines de Pn (resp.
Pn+1 ) rangées dans l’ordre croissant, alors

−1 < y1 < x1 < y2 < . . . < xk < yk+1 < xk+1 < . . . < xn < yn+1 < 1

Dans cette partie, on note (x1 , x2 , . . . , xn ) les zéros de Pn dans l’ordre croissant.
1◦ .a. Montrer qu’il existe une unique suite de n réels (a1 , a2 , . . . , an ) telle que

Z 1 n
X
∀ P ∈ En−1 , P (x) dx = ak P (xk ). (4)
−1 k=1

1◦ .b. Montrer que (4) reste valable pour tout polynôme P ∈ E2n−1 . (On pourrait prendre
{P0 , P1 , . . . , Pn , XPn , X 2 Pn , . . . , X n−1 Pn } comme base de E2n−1 ). Montrer aussi
que ak > 0 pour tout k. Expliciter ak en utilisant III.6◦ .
2◦ .a. Soit f ∈ C 2n ([−1, 1]). Montrer qu’il existe un polynôme unique Pf de E2n−1 tel
que
∀ i ∈ {1, 2, . . . , n}, Pf (xi ) = f (xi ), Pf0 (xi ) = f 0 (xi ).

f (x) − Pf (x)
2◦ .b. Pour x ∈ [−1, 1] \ {x1 , x2 , . . . , xn }, on pose g(x) = . Montrer que g
Pn2 (x)
est prolongeable par continuité sur [−1, 1].
2◦ .c. On fixe x ∈ [−1, 1]\{x1 , x2 , . . . , xn }. Pour t ∈ [−1, 1], on pose ϕ(t) = f (t)−Pf (t)−
APn2 (t), où A est déterminé par ϕ(x) = 0. Montrer, en appliquant le théorème de
Rolle, qu’il existe ξ ∈ [−1, 1] tel que ϕ(2n) (ξ) = 0.
36 ESPACES PRÉHILBERTIENS

2◦ .d. En déduire que

22n (n!)4 ¯ ¯
¯ (2n) ¯
∀ x ∈ [−1, 1], | g(x) | ≤ sup ¯ f (t) ¯
[(2n)!]3 t∈[−1,1]

3◦ . Soit f : [−1, 1] −→ IR une fonction de classe C 2n . Montrer en utilisant 1◦ et 2◦ que


¯Z ¯
¯ 1 n
X ¯ 22n+1 (n!)4 ¯ ¯
¯ ¯ ¯ (2n) ¯
¯ f (x) dx − ak f (xk ) ¯ ≤ sup ¯ f (t) ¯.
¯ −1 ¯ [(2n)!]2 (2n + 1)! t∈[−1,1]
k=1

Expliciter le cas n = 3.
VI
On note F (X) = (X 2 − 1)n .
X2n
◦ 1 dk F (X) k
1 . Montrer que F (X + Y ) = Y .
0
k! dX k
2◦ . Montrer que pour tout x ∈ IR et tout r ∈ IR∗+ , on a
Z 2π
1 rn dn F (x)
F (x + reiθ )e−inθ dθ =
2π 0 n! dxn

3◦ . En faisant un choix convenable de r montrer que


Z 2π p
1
∀ x ∈ [−1, 1], Pn (x) = (x + i 1 − x2 sin θ)n dθ.
2π 0

4◦ . En déduire que sup {| Pn (x) | : x ∈ [−1, 1]} = 1.


5◦ . Montrer que pour tout P ∈ En , on a
n
X 2k + 1
P (x) = hP, Pk iPk (x).
2
k=0

En déduire que
µZ 1 ¶1/2
n+1 2
∀ P ∈ En , sup | P (x) | ≤ √ | P (t) | dt . (5)
x∈[−1,1] 2 −1

n
X
Que devient cette inégalité si P = (2k + 1)Pk ?
k=0
Solutions 37

SOLUTIONS

Solution .1 En remplaçant dans 2◦ , x par ej on trouve


n
X n
X
2 2 2
1 = k ej k = | hej , ei i | = 1 + | hej , ei i | .
i=1 i=1
i6=j

Ce qui démontre que hej , ei i = δij . La famille (e1 , e2 , . . . , en ) est donc orthonormale. Soit
X n
y ∈ E, notons x = y − hy, ej i ej clairement hx, ei i = 0 pour tout i ∈ {1, 2, . . . , n},
k=1
n
X

donc d’après 2 on a k x k = 0, d’où y = hy, ej i ej . La famille (e1 , e2 , . . . , en ) est donc
k=1
génératrice, c’est une base orthonormale de E.

Solution .2 Notons v1 = t [1, 1, 1, 1] et v2 = t [1, 2, 3, 4]. Clairement F =


¡ ¢⊥
vect ({v1 , v2 }) . En effectuant un procédé d’orthonormalisation on construit une base
orthonormale {w1 , w2 } de vect ({v1 , v2 }) à savoir
   
1 √ 3
1 1 1 5 2 1  1
w1 = v1 =   , w2 = (v1 − v2 ) = √  .
2 2 1 2 5 2 5 −1
1 −3

La projection orthogonale p : E −→ E de E sur vect ({w1 , w2 }) est donnée par

p(X) = hX, w1 i w1 + hX, w2 i w2 = (w1 .t w1 + w2 .t w2 ).X

La matrice de p dans la base canonique est donc


 
7 4 1 −2
1  4 3 2 1 
P = w1 .t w1 + w2 .t w2 =  
10 1 2 3 4
−2 1 4 7

La projection orthogonale de E sur F est q = I − p dont la matrice dans la base


canonique est
 
3 −4 −1 2
1  −4 7 −2 −1 
Q=I −P =  
10 −1 −2 7 −4
2 −1 −4 3
38 ESPACES PRÉHILBERTIENS
n
X
Solution .3 Considérons une combinaison linéaire nulle λk xk = 0. Alors,
k=1

n
X n
X
λk (ek − xk ) = λk ek
k=1 k=1

D’où,
° n ° Ã !1/2
°X ° X Xn
° ° 2
° λk ek ° = | λk | ≤ | λk | k ek − xk k
° °
k=1 k=1 k=1
à n !1/2 à n !1/2
X 2
X 2
≤ | λk | k ek − xk k .
k=1 k=1

Alors,
n
X n
X
2 2
(1 − k ek − xk k ) | λk | ≤ 0.
k=1 k=1

n
X 2
Enfin, l’hypothèse montre que | λk | = 0 et λ1 = λ2 = · · · = λn = 0. On conclut
k=1
que la famille (x1 , . . . , xn ) est libre, donc c’est une base car E est de dimension n.

n
X
Solution .4 Considérons une combinaison linéaire nulle λk ek = 0. Raisonnons
k=1
par l’absurde en supposant (λ1 , . . . , λn ) 6= 0, l’hypothèse ∀ i, hx, ei i > 0 montre que les
deux ensembles suivants sont disjoints et non vides,

A+ = {k : λk > 0}, A− = {k : λk < 0}

Notons, pour k ∈ A− , µk = −λk . On a

X X
y= λk e k = µk ek
k∈A+ k∈A−

D’où,
2
X X
kyk = λi µj hei , ej i < 0
i∈A+ j∈A−

Ce qui est contradictoire. D’où λ1 = λ2 = · · · = λn = 0.


Solutions 39

Solution .5 a. =⇒ b. Nous avons, pour tout (x, y) ∈ E × E,

hp(x), yi = hp(x), y − p(y) + p(y)i = hp(x), p(y)i = hp(x) − x + x, p(y)i = hx, p(y)i

Car z − p(z) est orthogonal à Im p.


b. =⇒ c. Soit x ∈ Ker p. Pour tout z = p(y) ∈ Im p nous avons

hx, zi = hx, p(y)i = hp(x), yi = 0,

donc x ∈ (Im p)⊥ .


c. =⇒ d. Soit x ∈ E, clairement x − p(x) ∈ Ker p et p(x) ∈ Im p, d’où, d’après le
théorème de Pythagore,
2 2 2
k x k = k x − p(x) k + k p(x) k

ce qui démontre le résultat.


d. =⇒ a. Soit y ∈ Imp, et x ∈ E. On définit u ∈ IK par hy, x − p(x)i =
| hy, x − p(x)i | u. Alors

∀ r ∈ IR, k y k ≤ k y + ru(x − p(x)) k

ou bien
∀ r ∈ IR, 0 ≤ r2 k x − p(x) k + 2r | hy, x − p(x)i |
ce qui démontre que hy, x − p(x)i = 0, c’est à dire que x − p(x)⊥ Im p pour tout x ∈ E,
et la projection p est une projection orthogonale.

Solution .6 Démontrons d’abord que


p p
∀ (x, y) ∈ E × E, | hAx, yi | ≤ hAx, xi hAy, yi. (∗)

En effet, soit (x, y) ∈ E × E, nous avons

∀ λ ∈ IR, hA(x + λy), x + λyi ≥ 0

ou bien
∀ λ ∈ IR, hAx, xi + 2λhAx, yi + λ2 hAy, yi ≥ 0.
Si hAy, yi = 0, alors l’inégalité précédente montre que hAx, yi = 0 et (∗) est vérifiée. Si
par contre hAy, yi =
6 0, alors on est en présence d’un trinôme de deuxième degré (en λ)
qui garde un signe positif donc il est de discriminant négatif ou nul ce qui donne (∗).
Notons m = max {aii : 1 ≤ i ≤ n}. Alors, pour tout (i, j) ∈ {1, . . . , n}2 ,
q q

| aij | = | hAej , ei i | ≤ hAei , ei i hAej , ej i = aii ajj ≤ m.

Ce qui démontre que max | aij | ≤ m d’où le résultat, car l’inégalité inverse est triviale.
1≤i,j≤n
40 ESPACES PRÉHILBERTIENS

Solution .7 En effet,

A1 A2 est hermitienne ⇐⇒ A1 A2 = (A1 A2 )∗ ⇐⇒ A1 A2 = A∗2 A∗1 = A2 A1 .

Solution .8 Notons que det(A) = λ21 − λ22 − λ23 − λ24 = 1. Alors λ1 > λ2 . Posons
· ¸
1 λ1 − λ2 λ3 + iλ4
B=√ .
λ1 − λ2 0 1

Et calculons
· ¸
∗ 1 (λ1 − λ2 )2 (λ3 + iλ4 )(λ1 − λ2 )
B B=
λ1 − λ2 (λ3 − iλ4 )(λ1 − λ2 ) λ23 + λ24 + 1

Mais, λ23 + λ24 + 1 = λ21 − λ22 , donc B ∗ B = A. La matrice B est inversible d’où le résultat.

Solution .9 Remarquons que hO(ej ), ei i = aij , où (e1 , . . . , en ) est la base canonique.
Alors
n
à n !1/2
X √ X 2 √ √
| aij | ≤ n | aij | = n k O(ej ) k = n.
i=1 i=1

En prenant la somme de ces inégalités pour j variant entre 1 et n, nous arrivons à

X √
| aij | ≤ n n.
1≤i,j≤n

D’autre part,
X
aij = hO(V ), V i
1≤i,j≤n

n
X
avec V = ek . Alors
k=1

¯ ¯
¯ ¯
¯ X ¯
¯ a ¯ = | hO(V ), V i | ≤ k O(V ) k k V k = k V k2 = n.
¯ ij ¯
¯ 1≤i,j≤n ¯
Solutions 41

Solution .10 Soit (f1 , f2 , . . . , fn ) une autre base orthonormale de E. Clairement,


 
n
X n
X Xn
2  2
k P (ek ) k = | hP (ek ), fj i | 
k=1 k=1 j=1
 
n
X Xn
 2
= | hek , P (fj )i | 
k=1 j=1
n n
à !
X X 2
= | hP (fj ), ek i |
j=1 k=1
Xn
2
= k P (fk ) k
k=1

En prenant pour la base (f1 , f2 , . . . , fn ) la base orthonormale obtenue en considérant


(f1 , f2 , . . . , fr ) comme base orthonormale de Im p et (fr , fr+1 , . . . , fn ) comme base
orthonormale de Ker p, nous obtenons
n
X r
X
2 2
k P (fk ) k = k fk k = r = rg (P ).
k=1 k=1

Ce qui démontre le résultat.

Solution .11 1◦ . Notons que

x ∈ Ker T ∗ ⇐⇒ T ∗ x = 0 ⇐⇒ ∀ y ∈ E, hy, T ∗ xi = 0
⇐⇒ ∀ y ∈ E, hT y, xi = 0
⇐⇒ ∀ z ∈ Im T, hz, xi = 0
⇐⇒ x ∈ ( Im T )⊥

Nous concluons que Ker T ∗ = ( Im T )⊥ .


2◦ . Si x ∈ Im T ∗ , alors x = T ∗ z pour un certain z ∈ E. Pour tout y ∈ Ker T
on a hy, xi = hy, T ∗ zi = hT y, zi = 0. Il en résulte que x ∈ ( Ker T )⊥ . D’où
l’inclusion Im T ∗ ⊂ ( Ker T )⊥ mais ces deux sous-espaces sont de même dimension
alors Im T ∗ = ( Ker T )⊥ .

Solution .12 1◦ . Clairement (u∗ u)∗ = u∗ u∗∗ = u∗ u, donc u∗ u est symétrique.


2
D’autre part, hu∗ u(x), xi = k u(x) k ≥ 0.
2
Si λ est une valeur propre de u∗ u et x un vecteur propre associé alors λ k x k =
hu∗ u(x), xi ≥ 0 donc λ ≥ 0.
42 ESPACES PRÉHILBERTIENS

2◦ . Soit (e1 , e2 , . . . , en ) une base orthonormale de E formée de vecteurs propres de


u∗ u, (i.e. u∗ u(ei ) = λi ei ). Soit x ∈ E,
n
X n
X n
X
∗ ∗
x= hx, ei iei , u u(x) = hx, ei iu u(ei ) = λi hx, ei iei .
i=1 i=1 i=1

Alors
n
X
2 2
k u(x) k = hu∗ u(x), xi = λi | hx, ei i | .
i=1

Mais, pour tout i, λmin (u) ≤ λi ≤ λmax , donc


n
X n
X
2 2 2 2
λmin (u) k x k ≤ λmin (u) | hx, ei i | ≤ λi | hx, ei i | = k u(x) k ,
i=1 i=1

et
n
X n
X
2 2 2 2
k u(x) k = λi | hx, ei i | ≤ λmax (u) | hx, ei i | ≤ λmax (u) k x k .
i=1 i=1

Ce qui démontre l’inégalité demandée.


3◦ . Pour x ∈ E, on a

2 2 2
λmin (u) k v(x) k ≤ k u(v(x)) k ≤ λmax (u) k v(x) k

En utilisant l’inégalité de 2◦ une deuxième fois on arrive à

2 2 2
λmin (u)λmin (v) k x k ≤ k u◦v(x) k ≤ λmax (u)λmax (v) k x k

Si ν est une valeur propre de u◦v et x un vecteur propre associé, alors

2 2 2 2
λmin (u)λmin (v) k x k ≤ | ν | k x k ≤ λmax (u)λmax (v) k x k

ce qui démontre
2
λmin (u)λmin (v) ≤ | ν | ≤ λmax (u)λmax (v).

Solution .13 1◦ . Clairement, (det u)2 = (−1)m det I alors (−1)m ∈ IR+ et par
conséquent m est pair. Par exemple m = 2n.
2◦ . On sait que λ1 = k u k = sup { k u(x) k : k x k = 1}. Comme la sphère unité
de E est compacte cette borne supérieure est atteinte, par exemple en e1 ∈ E. D’où
l’existence de e1 ∈ E tel que k e1 k = 1 et k u(e1 ) k = λ1 . Posons f1 = (1/λ1 )u(e1 ).
1 2 1
2◦ .a. Nous avons k f1 k = 1 et u(f1 ) = u (e1 ) = − e1 . D’après la définition
λ1 λ1
1
de λ1 , nous avons k u(f1 ) k ≤ λ1 donc ≤ λ1 ou bien 1 ≤ λ21 ce qui démontre que
λ1
λ1 ≥ 1.
Solutions 43

2◦ .b. Par définition de k u k on a,

2 2
∀ t ∈ IR, k u(e1 + tf1 ) k ≤ λ21 k e1 + tf1 k .

ou bien ° °2
° t °
∀ t ∈ IR, ° λ1 e1 − f1 ° ≤ λ21 k e1 + tf1 k2 .
° λ1 °
d’où en développant, et en utilisant k e1 k = k f1 k = 1,

1
∀ t ∈ IR, t2 (λ21 − ) + 4t he1 , f1 i ≥ 0
λ21

ce qui démontre que he1 , f1 i = 0 et par conséquent e1 ⊥f1 .


3◦ . Soit y ∈ E tel que y⊥e1 . On a

2 2
∀ t ∈ IR, k u(y + te1 ) k ≤ λ21 k y + te1 k

ou bien
2 2
∀ t ∈ IR, k u(y) + tλ1 f1 k ≤ λ21 (k y k + t2 )

d’où en développant,

2 2
∀ t ∈ IR, 2tλ1 hu(y), f1 i ≤ λ21 k y k − k u(y) k

ce qui démontre que hu(y), f1 i = 0 et par conséquent u(y)⊥f1 .


4◦ . Soit y ∈ E tel que y⊥f1 . On a
° 2 °
∀ t ∈ IR, ° u (y + tf1 ) °2 ≤ λ21 k u(y + tf1 ) k2

ou bien ° °2
° t °
∀ t ∈ IR, k y + tf1 k ≤
2
λ21 ° u(y) − e1 °
° λ1 °
d’où en développant,

2 2
∀ t ∈ IR, 2tλ1 hu(y), e1 i ≤ λ21 k u(y) k − k y k

ce qui démontre que hu(y), e1 i = 0 et par conséquent u(y)⊥e1 .


5◦ . En effet, si y ∈ G⊥ ◦ ◦
1 alors y⊥e1 et y⊥f1 donc, d’après 3 et 4 , u(y)⊥e1 et
u(y)⊥f1 , c’est à dire u(y) ∈ G⊥
1 . Nous avons démontré que u(G2 ) ⊂ G2 , en fait c’est
une égalité à cause de la dimension finie et du fait que u est inversible.
6◦ . Démontrons par récurrence la propriété Pn suivante:
44 ESPACES PRÉHILBERTIENS

 
 Pour tout endomorphisme u d’un espace euclidien de dimen- 

 


 sion 2n et vérifiant u2 = −I, il existe une base orthonormale 


 


 {e1 , f1 , e2 , f2 , . . . , en , fn } de E et des réels (λk )1≤k≤n vérifiant 


 


 

k u k = λ1 ≥ λ2 ≥ · · · ≥ λn ≥ 1 et
Pn :

 


 1 


 ∀ i ∈ {1, . . . , n}, u(ei ) = λi fi , u(fi ) = − ei 


 λi 


 


 

 

En effet, P1 résulte de 2◦ . Supposons que Pn−1 est vérifiée pour n ≥ 2.


Soit u un endomorphisme d’un espace euclidien de dimension 2n et vérifiant
L

u2 = −I. D’après ce qui précède E = G1 G2 avec u(G2 ) = G2 , G1 = vect({e1 , f1 }),
e1 ⊥f1 , k e1 k = k f1 k = 1 et enfin u(e1 ) = λ1 f1 et u(f1 ) = −(1/λ1 )e1 avec λ1 = k u k.
Utilisons l’hypothèse de récurrence pour

u
e : G2 −→ G2 : x 7→ u(x)

alors, il existe une base orthonormale {e2 , f2 , . . . , en , fn } de G2 et des réels (λk )2≤k≤n
vérifiant k u
e k = λ2 ≥ · · · ≥ λn ≥ 1 et
1
∀ i ∈ {2, . . . , n}, u(ei ) = λi fi , u(fi ) = − ei
λi
Il suffit de noter que λ2 = k u
e k ≤ k u k = λ1 pour conclure que Pn est vérifiée.

Solution .14 1◦ . Notons que, d’après l’inégalité de Schwarz, on a

∀ z ∈ E, k z k = sup {hz, xi : x ∈ B1 (E)}

où B1 (E) = {x ∈ E : k x k ≤ 1}. En utilisant cette remarque,


n o
k u k = sup k u(x) k : x ∈ B1 (E)
n o
= sup hu(x), yi : (x, y) ∈ B1 (E) × B1 (E)
n o
= sup hx, u∗ (y)i : (x, y) ∈ B1 (E) × B1 (E)
n o

= sup k u (y) k : y ∈ B1 (E) = k u∗ k .

2◦ .a. Soit x ∈ Ker(I − u) c’est à dire u(x) = x. Calculons


2 2 2
k u∗ (x) − x k = k u∗ (x) k + k x k − 2hx, u∗ (x)i
2 2
= k u∗ (x) k + k x k − 2hu(x), xi
2 2 2 2 2
= k u∗ (x) k − k x k ≤ k u∗ k k x k − k x k ≤ 0
Solutions 45

2
On conclut que k u∗ (x) − x k = 0 i.e. x ∈ Ker (I − u∗ ). Alors Ker(I − u) ⊂
Ker (I − u∗ ), l’inclusion inverse s’obtient en utilisant ce même résultat pour u∗ .
2◦ .b. Nous avons vu, dans l’exercice 11, que Ker T ∗ = ( Im T )⊥ , alors

F2⊥ = ( Im (I − u))⊥ = Ker (I − u)∗ = Ker (I − u∗ ) = Ker (I − u) = F1 .

2◦ .c. Notons que, si x1 ∈ F1 alors u(x1 ) = x1 et par conséquent up (x1 ) = x1 pour


tout p ∈ IN∗ . D’autre part, si x2 ∈ F2 alors il existe y ∈ E tel que x2 = (I − u)(y) et
par conséquent
1
∀ p ∈ IN∗ , up (x2 ) = (y − up (y))
p
p
Mais k up k ≤ k u k ≤ 1, alors
2
∀ p ∈ IN∗ , k up (x2 ) k ≤ kyk
p
D’où, lim up (x2 ) = 0.
p→∞
Soit x ∈ E, alors x = x1 +x2 avec x1 (resp. x2 ) la projection orthogonale de x sur F1
(resp. F2 ). Clairement, up (x) = x1 +up (x2 ) donc d’après ce qui précède lim up (x) = x1 .
p→∞

Solution .15 ¦ Notons que t A.A = I3 , t A = A et que det A = −1. Alors A est une
symétrie orthogonale et une isométrie négative. Un calcul simple montre que

AX = −X ⇐⇒ X ∈ IRv1 , et AX = X ⇐⇒ X ∈ IRv2 + IRv3 .

avec     
−3 0 5
v1 =  2  , v2 =  1  , v3 =  6  .
−1 2 −3
On conclut que A est la matrice de la réflexion orthogonale parallèlement à la droite
IRv1 .
¦ Remarquons que t B.B = I3 et que det B = 1. Donc B est une rotation. Un calcul
simple montre que si 

−3
1
v1 = √  1  ,
11 1
alors Bv1 = v1 et IRv1 est l’axe de la rotation B. Si θ est l’angle de cette rotation alors
cos θ = (Tr (B) − 1)/2 = 7/18. Considérons un vecteur unitaire v2 orthogonal à v1 et
calculons v3 = Bv2 , par exemple
   
0 5
1 1
v2 = √  −1  , v3 = − √  11 
2 1 9 2 4
46 ESPACES PRÉHILBERTIENS

Comme v2 ∧ v3 = sin θv1 , alors sin θ = −5 11/18. On conclut que B est la matrice de
la rotation dans le sens direct autour du vecteur v1 d’angle θ = −Arccos (7/18).
¦ Remarquons que t C.C = I3 et que det C = 1. Donc C est une rotation. Un calcul
simple montre que si  
−1
1
v1 = √  1  ,
3 −1

alors Cv1 = v1 et IRv1 est l’axe de la rotation C. Si θ est l’angle de cette rotation, alors
cos θ = (Tr (C) − 1)/2 = 1/2. Considérons un vecteur unitaire v2 orthogonal à v1 et
calculons v3 = Cv2 , par exemple
   
0 1
1 1
v2 = √  1  , v3 = √  1 
2 1 2 0

Comme v2 ∧ v3 = sin θv1 , alors sin θ = 3/2. On conclut que C est la matrice de la
rotation dans le sens direct autour du vecteur v1 d’angle θ = π/3.


→ →
− − → − →
Solution .16 Complétons K en une base orthonormée E = { I , J , K } de E. Alors
 
cos θ − sin θ 0
Mat (R, E) =  sin θ cos θ 0  .
0 0 1

→ →− − →
Comme ρ est une rotation, alors on obtient une base orthonormale Ee = { i , j , k } en

− −
→ − → −
→ −
→ →

prenant i = ρ( I ), j = ρ( J ) et k = ρ( K ). Posons S = ρ◦R◦ρ−1 . Clairement
 
cos θ − sin θ 0
e =  sin θ
Mat (S, E) cos θ 0  .
0 0 1

Il en résulte que S = ρ◦R◦ρ−1 est la rotation d’axe dirigé par le vecteur unitaire

→ −

k = ρ( K ) et d’angle θ.



Solution .17 1◦ . Soit S la rotation d’axe dirigé par le vecteur unitaire k et d’angle
θ. Alors  
cos θ − sin θ 0
Mat (S, B) =  sin θ cos θ 0  .
0 0 1

→ →
− −
→ →

Supposons que ` ∈ / IR k . Cherchons une rotation ρ qui vérifie ρ( k ) = ` =

→ −
→ →
− −
→ −

α i + β j + γ k . Evidemment, ρ( j ) doit être orthogonal à ρ( k ), par exemple

→ 1 −
→ →

ρ( j ) = p (−β i + α j ).
2
α +β 2
Solutions 47

− −
→ −

Et {ρ( i ), ρ( j ), ρ( k )} doit être une base orthonormale directe donc


→ →
− −
→ 1 −
→ −
→ −

ρ( i ) = ρ( j ) ∧ ρ( k ) = p (αγ i + βγ j − (α2 + β 2 ) k ).
2
α +β 2

p
En posant δ = α2 + β 2 , on trouve
 
αγ −β αδ
1
Mat (ρ, B) = βγ α βδ  .
δ
−δ 2 0 γδ

Mais d’après l’exercice précédent on sait que R = ρ◦S ◦t ρ, alors M = Mat (R, B) vérifie
   
αγ −β αδ cos θ − sin θ 0 αγ βγ −δ 2
1
M = 2  βγ α βδ   sin θ cos θ 0   −β α 0 .
δ 2
−δ 0 γδ 0 0 1 αδ βδ γδ
 
(1 − α2 ) cos θ + α2 αβ(1 − cos θ) − γ sin θ αγ(1 − cos θ) + β sin θ
M =  αβ(1 − cos θ) + γ sin θ (1 − β 2 ) cos θ + β 2 βγ(1 − cos θ) − α sin θ 
αγ(1 − cos θ) − β sin θ βγ(1 − cos θ) + α sin θ (1 − γ 2 ) cos θ + γ 2
ou bien,
   
α2 αβ αγ 0 −γ β
Mat (R, B) = cos θ I3 + (1 − cos θ)  αβ β2 
βγ + sin θ  γ 0 −α 
2
αγ βγ γ −β α 0

− −
→ −
→ −
→ → − √
ce résultat est aussi vrai si ` ∈ IR k . Ce qui donne pour θ = π/3 et ` = ( i + j )/ 2.
 √ 
3 1 √6
1
Mat (R, B) =  √1 √3 − 6  .
4
− 6 6 2

2◦ . et 3◦ . Notons que sur l’expression de Mat (R, B) déjà obtenue nous avons,


→ →− → −
→ →
R(−

x ) = cos θ−

x + (1 − cos θ)h ` , −
x i ` + sin θ ` ∧ −
x.

Solution .18 M est une matrice de rotation si, et seulement si, M.t M = I3 et
det M = 1 ce qui est équivalent aux conditions

ab + bc + ca = 0, a2 + b2 + c2 = 1, et a3 + b3 + c3 − 3abc = 1.

Posons σ1 = a + b + c, σ2 = ab + bc + ca, σ3 = abc et enfin Sk = ak + bk + ck pour


k = 1, 2, 3. Alors, nous avons les relations suivantes:

S2 − σ12 + 2σ2 = 0, S3 − σ1 S2 + σ2 S1 − 3σ3 = 0


48 ESPACES PRÉHILBERTIENS

Ce qui démontre que M est une matrice de rotation si, et seulement si, les réels a, b, c
vérifient σ1 = 1 et σ2 = 0. Ce qui est à son tour équivalent à dire que (a, b, c) sont
trois racines réelles (comptées avec leurs multiplicités) d’une équation de la forme
X 3 − X 2 + k = 0.
Or la fonction f (x) = x3 −x2 +k admet trois zéros réels si, et seulement si, f (0) ≥ 0
et f (2/3) ≤ 0. Ce qui est équivalent à l’inégalité 0 ≤ k ≤ 4/27.
Supposons la condition précédente vérifiée. La matrice M est une matrice de
rotation et on remarque que si  
1

− 1
K = √ 1,
3 1

− −
→ →

alors M ( K ) = K , et K dirige l’axe de la rotation M . Si θ est l’angle de la rotation M ,

→ −

alors cos θ = (3a−1)/2. Considérons un vecteur unitaire I orthogonal à K et calculons

→ −

J = M ( I ), par exemple
   
1 a−b

→ 1 →
− 1
I = √  −1  , J = √ c − a
2 0 2 b−c


→ − → −
→ 3
Comme I ∧ J = sin θ K alors sin θ = (c − b). On conclut que M est la rotation

→ 2
d’axe dirigé par K et d’angle θ déterminé par

3a − 1 3
cos θ = , sin θ = (c − b).
2 2
posons k = 4(sin φ)2 /27 avec φ ∈ [0, π/2], alors le fait que a soit l’une des racines de
x3 − x2 + 4
27 sin2 φ = 0 montre que cos θ = (3a − 1)/2 doit vérifier
µ ¶2 µ ¶
1 + 2 cos θ 2(1 − cos θ) 4
+ sin2 φ = 0
3 3 27
ce qui donne cos 3θ = cos 2φ ou bien
½ ¾
2φ 2(φ − π) 2(φ + π)
cos θ ∈ cos , cos , cos
2 2 2
et par conséquent pour chaque φ il y a en général six matrices possibles, ce sont
   
cos 2ψ/3 cos 2(ψ − π)/3 cos 2(ψ + π)/3 1 1 1
2 1
Mψ =  cos 2(ψ + π)/3 cos 2ψ/3 cos 2(ψ − π)/3  +  1 1 1 
3 3
cos 2(ψ − π)/3 cos 2(ψ + π)/3 cos 2ψ/3 1 1 1


avec ψ = φ+kπ, k ∈ {0, 1, 2, 3, 4, 5}. Et Mψ est la rotation d’axe dirigé par K et d’angle
θ = 2ψ/3.
Solutions 49

Solution .19 Soit (e1 , . . . , en ) une base orthonormale de IRn qui diagonalise A (i.e.
Aej = λj ej ). Comme Ak = In , alors λkj = 1 pour tout j ∈ {1, . . . , n}, mais λj ∈ IR ce
qui montre que λj ∈ {−1, +1} pour tout j ∈ {1, . . . , n}. Il en résulte que, pour tout
j ∈ {1, . . . , n}, on a λ2j = 1. D’où, pour tout j ∈ {1, . . . , n}, on a A2 ej = ej . Ce qui est
équivalent à A2 = In .

Solution .20
n
X
2 n

I.1 . Comme (X − 1) = Cnk (−1)n−k X 2k , alors un calcul simple montre
k=0

X (2k)!
Pn (X) = (−1)n−k X 2k−n (†)
n 2n (2k − n)! (n − k)! k!
2 ≤k≤n

Il en résulte que Pn est un polynôme de degré n et que le coefficient de X n dans Pn est


(2n)!
.
2n (n!)2
I.2◦ . En revenant à (†) on trouve que Pn (−X) = (−1)n Pn (X). Alors P2n est pair
et P2n+1 est impair.
I.3◦ . Posons, pour k ∈ {1, 2, . . . , n},
Z 1
k
¡ ¢(k)
Jk = (−1) (x2 − 1)n Q(n−k) (x) dx.
−1

Une intégration par parties montre que


h¡ ¢ i+1 Z 1 ¡ ¢(k−1)
k 2 n (k−1) (n−k)
(−1) Jk = (x − 1) Q (x) − (x2 − 1)n Q(n−k+1) (x) dx
−1 −1

Mais 1 (resp. −1) est une racine d’ordre n de (X 2 − 1)n alors 1 (resp. −1) est une
¡ ¢(k−1)
racine de (x2 − 1)n . Il en résulte que Jk = Jk−1 pour tout k ∈ {1, 2, . . . , n}, en
particulier Jn = J0 ce qui démontre
Z 1 Z 1
(−1)n
hPn , Qi = Pn (x) Q(x) dx = n (x2 − 1)n Q(n) (x) dx.
−1 2 n! −1

I.4◦ . En effet, si Q ∈ En−1 alors Q(n) = 0 donc, d’après le résultat de I.3◦ , on


a hPn , Qi = 0. On conclut que, pour tout n ≥ 1, Pn ⊥En−1 . Si n 6= m, alors ou bien
Pn ∈ Em−1 , ou bien Pm ∈ En−1 , et dans les deux cas nous trouvons hPn , Pm i = 0.
I.5◦ . Pour (p, q) ∈ IN2 , on pose
Z 1
I(p, q) = (1 − x)p (1 + x)q dx
−1
50 ESPACES PRÉHILBERTIENS

Une intégration par parties montre que, si q > 0,


· ¸+1 Z 1
(1 − x)p+1 q
I(p, q) = − (1 + x)q + (1 − x)p+1 (1 + x)q−1 dx
p+1 −1 p+1 −1
q
= I(p + 1, q − 1).
p+1

Il en résulte

I(p, q) I(p + 1, q − 1) I(p + q, 0) 2p+q+1


= = ··· = =
p! q! (p + 1)! (q − 1)! (p + q)! (p + q + 1)!

ou bien Z 1
p! q!
(1 − x)p (1 + x)q dx = 2p+q+1 ,
−1 (p + q + 1)!
en particulier
Z 1
(n!)2
(1 − x2 )n dx = 22n+1 .
−1 (2n + 1)!
Mais
Z 1 Z 1
2 1 2 n (2n)! 2
k Pn k = n (1 − x ) Pn(n) (x) dx = 2n (1 − x2 )n dx = .
2 n! −1 2 (n!)2 −1 2n + 1
r
2
D’où, k Pn k = .
2n + 1 r
◦ 2n + 1
I.6 . Si l’on note Qn = Pn . Alors (Q0 , Q1 , . . . , Qn ) est une base orthonor-
2
male de En .

II.1◦ . En effet,
Z 1 ¡ 2 ¢0
hµ(P ), Qi = (x − 1)P 0 (x) Q(x) dx
−1
· ¸+1 Z 1
2 0
= (x − 1)P (x)Q(x) + (1 − x2 ) P 0 (x) Q0 (x) dx
−1 −1
Z 1
= (1 − x2 ) P 0 (x) Q0 (x) dx
−1

La dernière expression est symétrique en P et Q, donc hµ(P ), Qi = hP, µ(Q)i.


II.2◦ . Il est immédiat que deg µ(P ) ≤ deg P , il en résulte que µ(En ) ⊂ En .
Si n > m, alors µ(Pm ) ∈ Em ⊂ En−1 et d’après I.4◦ nous avons hPn , µ(Pm )i = 0.
Si au contraire n < m, alors hPn , µ(Pm )i = hµ(Pn ), Pm i = 0. Ce qui démontre que
n 6= m =⇒ hPn , µ(Pm )i = 0.
Solutions 51

II.3◦ . Comme µ(Pn ) ∈ En , alors en utilisant le résultat de I.6◦ et II.2◦ nous arrivons

n
X 2k + 1 2n + 1
µ(Pn ) = hµ(Pn ), Pk iPk = hµ(Pn ), Pn iPn = λn Pn .
2 2
k=0

Pour déterminer λn il suffit de comparer les coefficients de X n dans la relation


µ(Pn ) = λn Pn . Nous trouvons λn = n(n + 1).
II.4◦ . Nous savons, d’après II.3◦ , que Pn ∈ Fn . Soit P ∈ Fn \ {0}, si deg P = k alors
en comparant les coefficients de X k dans l’égalité µ(P ) = n(n + 1)P , nous arrivons à
k(k + 1) = n(n + 1) ce qui est équivalent à (n − k)(n + k + 1) = 0 ou bien k = n. Les
éléments de Fn \ {0} sont tous de degré n.
Soit P ∈ Fn \ {0}, il existe bn ∈ IR tel que deg(P − bn Pn ) < n. Mais P − bn Pn ∈ Fn
donc P − bn Pn = 0. Ceci démontre que Fn = IR.Pn et dim Fn = 1.
Ce qui se traduit en disant que Pn est l’unique solution polynomiale (à un facteur
multiplicatif près) de l’équation différentielle (x2 − 1)y 00 + 2xy 0 − n(n + 1)y = 0.

III.1◦ . Le polynôme XPn appartient à En+1 donc en utilisant I.6◦ on a

n+1
X 2k + 1
XPn = hXPn , Pk i Pk .
2
k=1

Mais hXPn , Pk i = hPn , XPk i = 0 si k ∈ {1, 2, . . . , n − 2}. Alors XPn = αn Pn+1 +


βn Pn + γn Pn−1 avec αn = (n + 3/2)hXPn , Pn+1 i, βn = (n + 1/2)hXPn , Pn i et
γn = (n − 1/2)hXPn−1 , Pn i.
(2n)!
III.2◦ . Le coefficient de X n+1 dans XPn est et le coefficient de X n+1 dans
2n (n!)2
(2n + 2)!
αn Pn+1 + βn Pn + γn Pn−1 est αn . Alors
2n+1 [(n + 1)!]2

(2n + 2)! (2n)!


αn = n .
2n+1 [(n+ 1)!] 2 2 (n!)2

n+1
Ce qui donne αn = .
2n + 1
III.3◦ . En substituant X par −X dans l’égalité XPn = αn Pn+1 +βn Pn +γn Pn−1 et
en utilisant Pn (−X) = (−1)n Pn (X), nous obtenons XPn = αn Pn+1 − βn Pn + γn Pn−1 .
Ce qui démontre que βn = 0.
III.4◦ . Nous avons αn = (n + 3/2)hXPn , Pn+1 i et γn = (n − 1/2)hXPn−1 , Pn i, alors
2n − 1 n
γn = αn−1 . D’où γn = . Il en résulte
2n + 1 2n + 1

(n + 1)Pn+1 + nPn−1 = (2n + 1)XPn (1)


52 ESPACES PRÉHILBERTIENS

n+1
X
2 (n)

III.5 . Le polynôme (1 − X 2
)Pn0 appartient à En+1 donc (1 − X )Pn0 = δk P k .
k=0
Mais, une intégration par parties montre que,

h(1 − X 2 )Pn0 , Pk i = hPn , ((1 − X 2 )Pk )0 i

Il en résulte que si k ≤ n − 2, alors h(1 − X 2 )Pn0 , Pk i = 0. Par conséquent

(n) (n)
(1 − X 2 )Pn0 = δn+1 Pn+1 + δn(n) Pn + δn−1 Pn−1

(n)
En utilisant la parité nous trouvons δn = 0.
D’autre part, en comparant le coefficient de X n+1 dans les deux membres de
l’égalité, nous arrivons à

(2n)! (2n + 2)! (n)


−n n 2
= n+1 2
δn+1
2 (n!) 2 [(n + 1)!]

(n) n(n + 1)
ou bien, δn+1 = − . D’autre part, comme hPn+1 , Pn0 i = 0,
2n + 1
(n)
h(1 − X 2 )Pn0 , Pn0 i = δn−1 hPn−1 , Pn0 i.

Une intégration par parties montre que

2 2n(n + 1)
h(1 − X 2 )Pn0 , Pn0 i = hµ(Pn ), Pn i = n(n + 1) k Pn k = .
2n + 1
2
De l’autre côté Pn0 − (2n − 1)Pn−1 ∈ En−2 alors hPn−1 , Pn0 i = (2n − 1) k Pn−1 k = 2.
(n) n(n + 1)
On conclut que δn−1 = , ce qui démontre que
2n + 1

n(n + 1)
(1 − X 2 )Pn0 = (Pn−1 − Pn+1 ) . (2)
2n + 1

(2n + 1)XPn − nPn−1 (2n + 1)XPn − (n + 1)Pn+1


D’après (1), Pn+1 = et Pn−1 =
n+1 n
donc en remplaçant dans la formule précédente

(1 − X 2 )Pn0 = (n + 1)(XPn − Pn+1 ) = −n(XPn − Pn−1 ) (20 )

En dérivant les deux membres de (2) et (20 ) et en utilisant ((1 − x2 )Pn0 )0 = −n(n + 1)Pn
nous arrivons à

1 ¡ 0 0
¢ 1 0
Pn = Pn+1 − Pn−1 = (Pn+1 − XPn0 ). (3)
2n + 1 n+1
Solutions 53

En substituant X par 1 dans (20 ), nous arrivons à Pn+1 (1) = Pn (1) pour tout n,
donc Pn (1) = 1 et la parité de Pn montre que Pn (−1) = (−1)n .
III.6◦ . Posons

Pn+1 (Y )Pn (X) − Pn+1 (X)Pn (Y )


An (X, Y ) = (n + 1) ,
Y −X

nous obtenons A0 (X, Y ) = 1 = P0 (X)P0 (Y ), et si ∆n = An (X, Y ) − An−1 (X, Y ) alors

((n + 1)Pn+1 (Y ) + nPn−1 (Y ))Pn (X) − ((n + 1)Pn+1 (X) + nPn−1 (X))Pn (Y )
∆n =
Y −X
(2n + 1)Y Pn (Y )Pn (X) − (2n + 1)XPn (X)Pn (Y )
=
Y −X
=(2n + 1)Pn (X)Pn (Y ).

Il en résulte que
n
X n
X
An (X, Y ) = A0 (X, Y ) + ∆k = (2k + 1)Pk (X)Pk (Y )
k=1 k=0

qui représente l’identité cherchée. Cette identité s’écrit


n
X µ ¶
Pn+1 (y) − Pn+1 (x) Pn (y) − Pn (x)
(2k + 1)Pk (x)Pk (y) = (n + 1) Pn (x) − Pn+1 (x)
y−x y−x
k=0

d’où en faisant tendre y vers x,


n
X ¡ 0 ¢
(2k + 1)Pk2 (x) = (n + 1) Pn+1 (x)Pn (x) − Pn+1 (x)Pn0 (x) .
k=0

IV.1◦ . Comme Pn est à coefficients réels alors α est aussi racine de Pn et


2
(X − α)(X − α) divise Pn , donc Pn (X) = (X 2 − (α + α)X + | α | ) Q(X). Mais Q ∈ En−1
donc hPn , Qi = 0. Ceci implique
Z 1
2
(Q(x))2 (X 2 − (α + α)X + | α | ) dx = 0
−1

ce qui est contradictoire car la fonction intégrée est continue positive et non nulle sur
[−1, 1]. On conclut que les racines de Pn sont réelles.
IV.2◦ .a. Si Pn admet une racine α appartenent à IR\] − 1, 1[. Alors nous pouvons
écrire Pn (X) = (X − λ)Q(X), et comme avant
Z 1
0 = hPn , Qi = (Q(x))2 (x − λ) dx.
−1
54 ESPACES PRÉHILBERTIENS

Mais la fonction x 7→ (Q(x))2 (x − λ) est continue et garde un signe constant sur [−1, 1]
ce qui est aussi contradictoire. On conclut que les racines de Pn sont dans ] − 1, 1[.
IV.2◦ .b. Si Pn admet une racine multiple α appartenent à ] − 1, 1[. Alors Pn (X) =
Q(X)(X − λ)2 , et comme avant
Z 1
0 = hPn , Qi = (Q(x))2 (x − λ)2 dx.
−1

Mais la fonction x 7→ (Q(x))2 (x − λ)2 est continue et positive sur [−1, 1] ce qui est
contradictoire.
IV.3◦ . On conclut que les racines de Pn sont toutes simples et appartiennent à
] − 1, 1[.
IV.4◦ . Clairement,

1 3 2 1 5 3 3
P0 = 1, P1 = X − , P2 = X − , P3 =
X − X.
2 2 2 2 2
½ ¾ ( r r )
1 1 3 3
Les racines de P2 sont − √ , √ et les racines de P3 sont − , 0, .
3 3 5 5
IV.5◦ . En effet, si Pn et Pn+1 admettent une racine commune α, alors d’après III.6◦
nous obtenons
n
X
1≤ (2k + 1)Pk2 (α) = 0
k=0

ce qui est contradictoire. Il en résulte que Pn et Pn+1 n’ont pas de racine commune.
Soit I un intervalle contenu dans [−1, 1] \ {x1 , . . . , xn }, où {x1 , . . . , xn } sont les
Pn+1 (x)
racines de Pn . La deuxième relation de III.6◦ montre que la dérivée de x 7→
Pn (x)
est strictement positive sur I.
Pn+1 (x)
IV.6◦ . Pour k ∈ {1, 2, . . . , n − 1}, on pose Ik =]xk , xk+1 [. La fonction x 7→
Pn (x)
est strictement croissante sur Ik et admet des limites infinies aux bornes de l’intervalle
Pn+1 (x) Pn+1 (x)
Ik . Donc lim = −∞ et lim = +∞. Il en résulte qu’il existe une
x → xk Pn (x) x → xk+1 Pn (x)
> <

racine unique de Pn+1 dans Ik . Enfin si Pn+1 n’a pas de racines dans ] − 1, x1 [, alors
l’identité Pn+1 (−X) = (−1)n+1 Pn+1 (X) montre que Pn+1 n’a pas, non plus, de racines
dans ]xn , 1[, mais alors Pn+1 n’aurait que n − 1 racines ce qui contredit le résultat de
IV.3◦ . On conclut que Pn+1 admet au moins une racine dans chacun des intervalles
] − 1, x1 [ et ]xn , 1], mais il en admet (n − 1) dans ]x1 , xn [. Il en résulte que Pn+1 admet
exactement une racine dans chacun des intervalles ] − 1, x1 [, ]xk , xk+1 [, (1 ≤ k ≤ n − 1)
et ]xn , 1]. Ce qui est le résultat demandé.
Solutions 55

V.1◦ .a. Il est bien connu que les formes linéaires (ϕk )1≤k≤n définies sur En−1
par ϕk (P ) = P (xk ) constituent une base du duale de En−1 , (car les (x1 , . . . , xn ) sont
Z 1
distincts). Alors la forme linéaire ϕ définie sur En−1 par ϕk (P ) = P (x) dx s’exprime
−1
d’une manière unique comme combinaison linéaire des (ϕk )1≤k≤n . Alors il existe un seul
n
X
n
élément (a1 , a2 , . . . , an ) ∈ IR tel que ϕ = ak ϕk . Ce qui s’écrit
k=1

Z 1 n
X
∀ P ∈ En−1 , P (x) dx = ak P (xk ). (∗)
−1 k=1

V.1◦ .b. En effet, La forme linéaire ψ définie sur E par


Z 1 n
X
ψ(P ) = P (x) dx − ak P (xk )
−1 k=1

s’annule en (Pk )0≤k≤n−1 car ces polynômes appartiennent à En−1 . D’autre part, pour
k ∈ {0, 1, . . . , n − 1}, on a
n
X
k k
ψ(X Pn ) = hPn , X i − aj xkj Pn (xj ) = 0.
j=1

Alors ψ s’annule sur l’espace engendré par (P0 , P1 , . . . , Pn , XPn , . . . , X n−1 Pn ) qui est
E2n−1 . Posons
n
Y X − xk Pn (X)
Vj (X) = =
xj − xk (X − xj )Pn0 (xj )
k=1
k6=j

Nous avons Vj (xk ) = δjk et deg(Vj2 ) = 2n − 2. Alors ψ((Vj )2 ) = 0 et par conséquent


Z 1
2
aj = (Vj (x))2 dx = k Vj k > 0.
−1
En utilisant III.6◦ , nous obtenons
n
X Pn (X)
(2k + 1)Pk (xj )Pk (X) = −(n + 1)Pn+1 (xj )
X − xj
k=0

alors,
° °2 n
° Pn (X) ° X
(xj ) ° ° =
2 2 2
(n + 1) Pn+1 ° ° (2k + 1)2 Pk2 (xj ) k Pk (X) k ,
X − xj
k=0

soit
° °2 n
° Pn (X) ° X
(n + 1) 2 2
Pn+1 (xj ) °
°
° =2
° (2k + 1)Pk2 (xj ) = −2(n + 1)Pn+1 (xj )Pn0 (xj ),
X − xj
k=0
56 ESPACES PRÉHILBERTIENS

ce qui démontre que,


° °2 n
X
° Pn (X) ° −2
°
2
a j = k Vj k = ° ° =2 (2k + 1)Pk2 (xj ) = ,
0
(X − xj )Pn (xj ) ° (n + 1)Pn+1 (xj )Pn0 (xj )
k=0

V.2◦ .a. Considérons

Φ : E2n−1 −→ IR2n : P 7→ (P (x1 ), . . . , P (xn ), P 0 (x1 ), . . . , P 0 (xn )).

Clairement, Φ est une application linéaire de E2n−1 dans IR2n . Montrons que Φ est
injective. En effet, si P ∈ Ker Φ, alors P admet chaque xj comme racine double donc
P = λ(X − x1 )2 (X − x2 )2 · · · (X − xn )2 , ce qui implique que λ = 0 (sinon deg P = 2n
et P ∈
/ E2n−1 ; absurde), on conclut que P = 0. Par conséquent, Φ est injective. Mais
dim E2n−1 = dim IR2n = 2n, alors Φ est bijective. Il en résulte qu’il existe un polynôme
unique Pf ∈ E2n−1 tel que Φ(Pf ) = (f (x1 ), . . . , f (xn ), f 0 (x1 ), . . . , f 0 (xn )). D’où le
résultat.
V.2◦ .b. Remarquons que, pour k ∈ {1, 2, . . . , n},

1 00
f (x) = f (xk ) + f 0 (xk )(x − xk ) + f (xk )(x − xk )2 + o((x − xk )2 )
2
1 00
Pf (x) = Pf (xk ) + Pf0 (xk )(x − xk ) + Pf (xk )(x − xk )2 + o((x − xk )2 )
2
donc, pour x ∈ [−1, 1] \ {x1 , x2 , . . . , xn },
µ ¶2 µ ¶−2 µ ¶
x − xk f (x) − Pf (x) Pn (x) 1 00 00
g(x) = = (f (xk ) − Pf (xk )) + ε(x − xk )
Pn (x) (x − xk )2 x − xk 2

avec lim ε(t) = 0. Alors


t→0
f 00 (xk ) − Pf00 (xk )
lim g(x) = .
x→xk 2[Pn0 (xk )]2
Ceci démontre que g est prolongeable par continuité sur [−1, 1] en posant

f 00 (xk ) − Pf00 (xk )


g(xk ) = pour k ∈ {1, 2, . . . , n}.
2[Pn0 (xk )]2

V.2◦ .c. Démontrons d’abord le lemme suivant:


Lemme: Soit ψ : [a, b] −→ IR une fonction de classe C m . On suppose qu’il existe
une suite finie strictement croissante (zk )1≤k≤m+1 de [a, b] telle que, pour
1 ≤ k ≤ m + 1, ψ(zk ) = 0. Alors il existe ξ ∈]a, b[ tel que ψ (m) (ξ) = 0.
Démontrons ce lemme par récurrence sur m. Si m = 1 c’est le théorème de Rolle.
Supposons le lemme vrai pour m − 1, (m > 1). Soit ψ : [a, b] −→ IR une fonction de
Solutions 57

classe C m , telle qu’il existe une suite finie strictement croissante (zk )1≤k≤m+1 de ]a, b[
vérifiant ψ(zk ) = 0, pour 1 ≤ k ≤ m + 1. D’après le théorème de Rolle il existe, pour
chaque k ∈ {1, . . . , m} un réel tk ∈]xk , xk+1 [ tel que ψ 0 (tk ) = 0. En utilisant l’hypothèse
de récurrence pour la fonction ψ 0 et la suite (tk )1≤k≤m , nous trouvons ξ ∈]a, b[ tel que
(ψ 0 )(m−1) (ξ) = 0. Ce qui démontre le lemme pour m.
Soit x ∈ [−1, 1] \ {x1 , . . . , xn }. Notons (tk )1≤k≤n+1 la suite strictement croissante
de [−1, 1] déterminée par

{t1 , t2 , . . . , tn+1 } = {x, x1 , x2 , . . . , xn }.

D’après le théorème de Rolle, pour chaque k ∈ {1, . . . , n}, il existe zk ∈]tk , tk+1 [ tel que
ϕ0 (zk ) = 0. La fonction ϕ0 s’annule aux points (zk )1≤k≤n et aussi aux points (xk )1≤k≤n ,
donc elle s’annule en 2n points distincts de [−1, 1]. On conclut, d’après le lemme, qu’il
existe ξ ∈ [−1, 1] tel que (ϕ0 )(2n−1) (ξ) = 0, ce qui démontre le résultat.
V.2◦ .d. Il suffit d’interpréter le résultat précédent. La condition ϕ(x) = 0 équivaut
22n (n!)4 (2n)
à A = g(x), et la condition ϕ(2n) (ξ) = 0 équivaut à A = f (ξ). Alors, pour
[(2n)!]3
tout x ∈ [−1, 1] \ {x1 , . . . , xn }, on a

22n (n!)4 ¯ ¯
¯ ¯
| g(x) | ≤ sup ¯ f (2n) (t) ¯
[(2n)!]3 t∈[−1,1]

qui reste aussi vrai pour x ∈ {x1 , . . . , xn } à cause de la continuité de g sur [−1, 1].
V.3◦ . D’après V.6◦ .d, nous avons
à !
22n (n!)4 ¯ ¯
¯ (2n) ¯
∀ x ∈ [−1, 1], | f (x) − Pf (x) | ≤ sup ¯ f (t) ¯ Pn2 (x),
[(2n)!]3 t∈[−1,1]

alors
¯Z 1 Z 1 ¯ Ã 2n ¯
!
¯ Z 1
¯ ¯ 2 (n!)4 ¯ (2n) ¯
¯ f (x) dx − Pf (x) dx ¯¯ ≤ sup ¯ f (t) ¯ Pn2 (x) dx.
¯ [(2n)!]3
−1 −1 t∈[−1,1] −1

2
Mais k Pn k = 2/(2n + 1) et Pf ∈ E2n−1 donc en utilisant V.2◦ .b, Nous arrivons à
¯Z ¯
¯ 1 n
X ¯ 22n+1 (n!)4 ¯ ¯
¯ ¯ ¯ (2n) ¯
¯ f (x) dx − ak f (xk ) ¯ ≤ sup ¯ f (t) ¯ .
¯ −1 ¯ [(2n)!]2 (2n + 1)! t∈[−1,1]
k=1

Pour expliciter le cas n = 3 il nous faut déterminer (a1 , a2 , a3 ) qui vérifient


Z 1 Ã r ! Ãr !
3 3
∀ P ∈ E2 , P (t) dt = a1 P − + a2 P (0) + a3 P .
−1 5 5
58 ESPACES PRÉHILBERTIENS

En testant cette relation pour P ∈ {1, X, X 2 } nous arrivons à


2 3
a1 + a2 + a3 = 2, a1 − a3 = 0, − (a1 + a2 ) = 0
3 5
5 8
ce qui donne a1 = a3 = et a2 = .
9 9
Pour toute fonction f : [−1, 1] −→ IR de classe C 6 nous avons
¯Z Ã Ã r ! Ãr !! ¯
¯ 1 1 3 3 ¯ 1 ¯ ¯
¯ ¯ ¯ (6) ¯
¯ f (x) dx − 5f − + 8f (0) + 5f ¯ ≤ sup ¯ f (t) ¯.
¯ −1 9 5 5 ¯ 15750 t∈[1,1]

VI.1◦ . C’est la fonmule de Taylor pour les polynômes.


VI.2◦ . Clairement,
Z 2π 2n k k
X Z 2π
1 iθ −inθ r d F (x) 1 i(k−n)θ rn dn F (x)
F (x + re ) e dθ = e dθ =
2π 0 k! dxk 2π 0 n! dxn
k=0

VI.3◦ . En prenant r = 1 − x2 , (x ∈] − 1, 1[), nous obtenons
¡ ¢n p p
F (x + reiθ ) e−inθ = ((x + reiθ )2 − 1)e−inθ = ( 1 − x2 )n 2n (x + i 1 − x2 sin θ)n

ce qui démontre
Z 2π p
1
∀ x ∈] − 1, 1[, Pn (x) = (x + i 1 − x2 sin θ)n dθ.
2π 0

Cette formule reste clairement vraie pour x ∈ {−1, 1}.


¯ √ ¯2
VI.4◦ . Comme ¯ x + i 1 − x2 sin θ ¯ = x2 + (1 − x2 ) sin2 θ ≤ 1. Nous déduisons
que, pour tout x ∈ [−1, 1], | Pn (x) | ≤ 1. D’autre part, Pn (1) = 1 donc

sup { | Pn (x) | : x ∈ [−1, 1]} = 1.

VI.5◦ . D’après I.6◦ nous pouvons écrire


n
X 2k + 1
P (x) = hP, Pk iPk (x).
2
k=0

Pour x ∈ [−1, 1],


n n r ¯ ¯
X 2k + 1 X
2k + 1 ¯¯ Pk ¯¯
| P (x) | ≤ | hP, Pk i | = hP, i
2 2 ¯ k Pk k ¯
k=0 k=0
à n !1/2 à n µ ¶2 !1/2
X 2k + 1 X Pk
≤ hP, i
2 k Pk k
k=0 k=0
n+1
≤ √ kP k
2
Solutions 59

ce qui démontre l’inégalité (5) cherchée.


Notons que, pour tout x ∈ [−1, 1],
¯ n ¯
¯X ¯ X n
¯ ¯
| P (x) | = ¯ (2k + 1)Pk (x) ¯ ≤ (2k + 1) = (n + 1)2 = P (1)
¯ ¯
k=0 k=0
¯ ¯
¯X n ¯ √
¯ ¯
alors sup ¯ (2k + 1)Pk (x) ¯ = (n + 1)2 . D’autre part, k P k = 2 (n + 1). Il en
x∈[−1,1] ¯ k=0 ¯
Xn
résulte que le polynôme P = (2k + 1)Pk réalise l’égalité dans l’inégalité (5), ce qui
k=0
démontre
sup | P (x) |
x∈[−1,1] n+1
sup µZ 1 ¶1/2 = √2 .
P ∈En \{0} 2
| P (x) | dx
−1

OKMRAN
OUBA
SÉRIES NUMÉRIQUES

I. Limite inférieure et limite supérieure d’une suite numérique

Soit (xn )n∈IN une suite de réels, posons, pour tout n ∈ IN,

Xn = {xk : k ≥ n}, an = sup Xn , et bn = inf Xn .

L’inclusion Xn+1 ⊂ Xn , pour tout n, permet de voir que la suite (an )n∈IN est une suite
décroissante dans IR, elle converge alors vers un élément a ∈ IR qu’on appelle la limite
supérieure de (xn )n∈IN , et l’on note a = lim xn . De même, la suite (bn )n∈IN est une
n→∞
suite croissante dans IR, elle converge alors vers un élément b ∈ IR qu’on appelle la
limite inférieure de (xn )n∈IN , et l’on note b = lim xn .
n→∞
Notons que pour une suite donnée la limite supérieure et la limite inférieure existent
toujours dans IR, et elles peuvent ne pas être égales.

Exemple : Si xn = (−1)n alors, il est immédiat de vérifier que lim xn = −1 et


n→∞
lim xn = 1.
n→∞

Proposition I.1. Soit (xn )n∈IN une suite réelle. On pose ` = lim xn et L = lim xn .
n→∞ n→∞

1 . Si (xϕ(n) )n∈IN est une sous-suite de (xn )n∈IN qui tend vers a ∈ IR, alors
` ≤ a ≤ L.
2◦ . Il existe une sous-suite (xϕ(n) )n∈IN de (xn )n∈IN telle que lim xϕ(n) = `.
n→∞
3◦ . Il existe une sous-suite (xψ(n) )n∈IN de (xn )n∈IN telle que lim xψ(n) = L.
n→∞
Preuve : 1◦ . Rappelons la notation Xn = {xk : k ≥ n}, an = sup Xn , et bn = inf Xn .
Nous avons aϕ(n) ≥ xϕ(n) ≥ bϕ(n) , pour tout n. Alors en faisant tendre n vers
l’infini nous obtenons l’inégalité demandée.
2◦ . Supposons
½ que ` ∈ IR ∪ {+∞}. Nous définissons
¾ ϕ(0) = 0, et pour n ≥ 1,
1
l’ensemble k > ϕ(n − 1) : xk ≤ b1+ϕ(n−1) + n’est pas vide par définition de
n
b1+ϕ(n−1) , on pose alors
½ ¾
1
ϕ(n) = min k > ϕ(n − 1) : xk ≤ b1+ϕ(n−1) + .
n
2 Séries numériques

La suite d’entiers naturels (ϕ(n))n∈IN est strictement croissante, et de plus, pour tout
n ≥ 1, nous avons xϕ(n) ∈ X1+ϕ(n−1) , donc

1
b1+ϕ(n−1) ≤ xϕ(n) ≤ b1+ϕ(n−1) + .
n

Mais la suite (bn )n∈IN tend vers ` alors, lim xϕ(n) = `.


n→∞
D’autre part, si ` = −∞ alors bn = −∞ pour tout n. Nous définissons ϕ(0) = 0,
et pour n ≥ 1, l’ensemble {k > ϕ(n − 1) : xk ≤ −n} n’est pas vide par définition de
b1+ϕ(n−1) , on pose alors

ϕ(n) = min {k > ϕ(n − 1) : xk ≤ −n} .

La suite d’entiers naturels (ϕ(n))n∈IN est strictement croissante, et de plus, pour tout
n ≥ 1, nous avons xϕ(n) ≤ −n alors, lim xϕ(n) = −∞ = `.
n→∞
3◦ . C’est une démonstration similaire à la précédente. Nous la laissons en exercice
au lecteur.

Corollaire I.2. Soit (xn )n∈IN une suite réelle. Alors lim xn ≤ lim xn , avec égalité
n→∞ n→∞
si, et seulement si, la suite (xn )n∈IN tend vers une limite x ∈ IR.

En effet, La première inégalité résulte de 1◦ de la proposition I.1, et le cas d’égalité


résulte aussi de 2◦ et de 3◦ de la proposition I.1, car toutes les sous-suites de la suite
(xn )n∈IN tendent aussi vers x.

Remarque : Il est très facile de vérifier que pour une suite réelle (xn )n∈IN , nous avons

lim xn = − lim (−xn ) et lim xn = − lim (−xn ).


n→∞ n→∞ n→∞ n→∞

Cette remarque permet de déduire le 3◦ . de la proposition I.1 à partir du 2◦ .

Proposition I.3: Soient (xn )n∈IN et (yn )n∈IN deux suites réelles, telles que xn ≤ yn
pour tout n ≥ n0 . Alors

lim xn ≤ lim yn et lim xn ≤ lim yn


n→∞ n→∞ n→∞ n→∞

Preuve : c’est une application directe de la définition. La deuxième inégalité résulte


de la première en utilisant la précédente.
Généralités 3

Nous laissons la vérification des propriétés suivantes comme exercice au lecteur:


¦ Soient (xn )n∈IN une suite réelle et α ∈ IR,

lim xn < α =⇒ (∃n0 , n ≥ n0 =⇒ xn < α)


n→∞

¦ Soient (xn )n∈IN et (yn )n∈IN deux suites réelles. Si (xn )n∈IN tend vers x ∈ IR, alors

lim (xn + yn ) = x + lim yn .


n→∞ n→∞

¦ Soient (xn )n∈IN et (yn )n∈IN deux suites réelles à termes positifs. Si (xn )n∈IN tend
vers x ∈ IR et si la suite (yn )n∈IN est majorée, alors

lim xn yn = x lim yn .
n→∞ n→∞

¦ Soit (xn )n∈IN une suite réelle telle que lim (xn+1 − xn ) = 0. Alors toute valeur
· ¸ n→∞

x ∈ lim xn , lim xn est la limite d’une sous-suite de la suite (xn )n∈IN .


n→∞ n→∞

II. Généralités

Soit (xn )n∈IN une suite de IR ou de C.


| On définit la suite de ses sommes partielles
Xn
P
(Sn )n∈IN par Sn = xk . On dira que la série de terme général xn , (et on écrit xn ),
k=0
converge et admet S pour somme, si la suite (Sn )n∈IN converge vers S, dans ce cas on

X
écrit S = xn .
k=0

Or toute suite réelle ou complexe converge si, et seulement si, elle est de Cauchy.
Alors
X
xn converge ⇐⇒ (Sn )n∈IN est de Cauchy
¯ n+m ¯
¯X ¯
¯ ¯
⇐⇒ (∀ ε > 0), (∃ Nε ∈ IN), (n ≥ Nε , m ∈ IN =⇒ ¯ xk ¯ < ε)
¯ ¯
k=n

L’avantage de cette critère, dite de Cauchy, est qu’elle permette de prouver la conver-
gence de la série sans connaı̂tre à priori la somme de la série.
5
! Du critère précédent nous pouvons déduire qu’une condition nécessaire de convergence
P
de la série xn est que le terme général tende vers zéro. Cette condition n’est pas
√ √
suffisante comme le montre l’exemple de la série de terme général xn = n + 1 − n.
4 Séries numériques
P P
Proposition II.1. Soient un et vn deux séries numériques convergentes. Alors
pour tout λ ∈ C
| la série de terme général λu
n + vn est convergente.

La preuve de cette proposition est immédiate.

P P
Proposition II.2. Soient xn et an deux séries numériques. On suppose que pour
P P
tout n ∈ IN, | xn | ≤ an , et que an est convergente Alors la série xn est convergente.

Preuve : La preuve de cette proposition repose sur le critère de Cauchy et sur l’inégalité
immédiate ¯ n+m ¯ n+m
¯X ¯ X n+m
X
¯ ¯
¯ xk ¯ ≤ | xk | ≤ ak .
¯ ¯
k=n k=n k=n

Cette proposition montre l’importance de l’étude des séries à termes positifs.

III. Séries à termes positifs

P P
Proposition III.1. Soit xn une série à termes positifs. Alors xn converge si, et
seulement si, la suite de ses sommes partielles est majorée.

Preuve :
Car la suite des sommes partielles d’une série à termes positifs est croissante.

Exemples :
P
¦ Soit a ∈ IR+ , la série géométrique an converge si, et seulement si, a ∈ [0, 1[.
En effet, si a ≥ 1 la série diverge car son terme général ne tend pas vers zéro.
Xn
1 − an+1
Par contre, si a ∈ [0, 1[, alors Sn = ak = et donc (Sn )n∈IN tend vers
1−a
k=0
1
.
1−a
X 1
¦ Soit α ∈ IR, la série, dite de Riemann, converge si, et seulement si,

n≥1
α ∈]1, +∞[.
(α)
En effet, Notons Sn la somme partielle de cette série, si α > 1, nous avons

X ½ ¾
(α) (α) 1 1
S2k+1 − S2k = α
≤ 2k max α
: 2k < n ≤ 2k+1 < 2(1−α)k .
n n
2k <n≤2k+1
Séries à termes positifs 5

Alors,
n−1
X n−1
X ∞
X
(α) (α) (α) (α) (1−α) k 1
S 2n − S1 = S2k+1 − S 2k < (2 ) < (2(1−α) )k = .
1 − 21−α
k=0 k=0 k=0

(α)
Donc, si α > 1 la suite des sommes partielles (Sn )n≥1 est majorée, et la série
converge.
Supposons α = 1. Dans ce cas, une étude simple de la fonction x 7→ x − Log (1 + x)
montre qu’elle est positive sur IR+ , il en résulte que pour tout entier n ≥ 1, nous
1
avons ≥ Log (1 + n) − Log n. ALors
n
n
X
Sn(1) ≥ (Log (1 + k) − Log k) = Log (1 + n).
k=1

(1)
X1
La suite (Sn )n≥1 n’est alors pas majorée et la série , dite harmonique, est
n
divergente.
1 1
Enfin, si α < 1 alors pour tout entier n ≥ 1 nous avons α
≥ , et par conséquent
n n
(α) (1) (α)
Sn ≥ Sn . Il en résulte que la suite (Sn )n≥1 n’est pas majorée et la série est
divergente.

Proposition III.2. Soient (un )n∈IN et (vn )n∈IN deux suites de IR+ .
P P
1◦ . Si 0 ≤ un ≤ vn pour tout n ≥ n0 , et si vn converge alors un converge.
P P
2◦ . Si 0 ≤ un ≤ vn pour tout n ≥ n0 , et si un diverge alors vn diverge.
u n
3◦ . S’il existe deux réels strictement positifs a et b tels que, a ≤ ≤ b pour tout
P P vn
n ≥ n0 . Alors les deux séries un et vn sont de même nature.
un
4◦ . S’il existe un réel strictement positif ` tel que, lim = `. Alors les deux
P P n→∞ vn
séries un et vn sont de même nature.
u n P P
5◦ . Si lim = 0, et si vn converge alors un converge.
n→∞ vn
un P P
6◦ . Si lim = +∞, et si vn diverge alors un diverge.
n→∞ vn
La démonstration de cette proposition est simple et laissée en exercice au lecteur.

Exemples :
α
¦ Soit α > 0. Il est immédiat que lim n2 e−n = 0, donc la covergence de la série de
n→∞
X 1 X
−nα
Riemann implique la convergence de la série e .
n2 P
¦ Étudions la série an avec
p
3
p b
an = n3 + n2 + n + 1 − n2 + 1 + a + .
n
6 Séries numériques

En effet, un développement limité en (1/n) de an montre que

1 5 1 14 1
an = (a + ) + (b − ) + 2
+ O( 3 ).
3 18 n 81n n

1 P
Donc si a 6= − , la série an diverge car son terme général ne tend pas vers zéro. Si
3
1 5 5 P
a = − et b 6= , on a nan −−−→ (b − ) 6= 0. Donc la série an diverge car elle
3 18 X1
n→∞ 18
1 5 14
est de même nature que . Enfin, si (a, b) = (− , ). alors n2 an −−−→ 6= 0.
n 3 18 X
n→∞ 81
P 1
Donc la série an converge car elle est de même nature que .
n2

P
Proposition III.3. (Règle de Cauchy) Soit an une série à termes positifs. On pose

L = lim n an .
n→∞
P
¦ Si L < 1, alors an converge.
P
¦ Si L > 1, alors an diverge.
¦ Si L = 1, on ne peut pas conclure.


Preuve : ¦ Si L < 1, on prend µ ∈]L, 1[. Il existe alors n0 tel que n an ≤ µ, pour tout
P n
n ≥ n0 . Alors an ≤ µn pour tout n ≥ n0 . Mais µ est convergente car µ < 1, alors
P
an est aussi convergente. (Voir la proposition précédente).
P
¦ Si L > 1, alors la suite (an )n∈IN ne tend pas vers zéro, et la série an diverge.
(En effet, si an ≤ M , pour tout n ∈ IN, alors

√ √
n
1 < α = lim n
an ≤ lim M = 1).
n→∞ n→∞


¦ Enfin, si an = 1/nα alors lim n
an = 1 pour tout α ∈ IR, et pourtant la série
n→∞
ne converge que si α > 1.

P
Proposition III.4. (Règle de D’Alembert) Soit an une série à termes strictement
an+1 an+1
positifs. On pose ` = lim et L = lim .
n→∞ an
P
n→∞ an
¦ Si L < 1, alors an converge.
P
¦ Si ` > 1, alors an diverge.
¦ Si ` ≤ 1 ≤ L, on ne peut pas conclure.
Séries à termes positifs 7
an+1
Preuve : ¦ Si L < 1, on prend µ ∈]L, 1[. Il existe alors N tel que ≤ µ, pour
an
tout n ≥ N . Alors (µ−n an )n≥N est décroissante. Par conséquent, il existe A > 0 tel
P n P
que an ≤ Aµn , mais la série µ est convergente (car µ < 1), alors an est aussi
convergente. (Voir la proposition III.1).
an+1
¦ Si ` > 1, alors il existe N tel que ≥ 1, pour tout n ≥ N . Alors an ≥ aN
an P
pour tout n ≥ N , donc la suite (an )n∈IN ne tend pas vers zéro, et la série an diverge.
an+1
¦ Enfin, si an = 1/nα alors lim = 1 pour tout α ∈ IR, et pourtant la série
n→∞ an
ne converge que si α > 1.

La proposition suivante permet de comparer les règles de Cauchy et de D’Alembert.

Proposition III.5. Soit (an )n∈IN une suite à termes strictement positifs. Alors
an+1 √ √ an+1
lim ≤ lim n an ≤ lim n an ≤ lim .
n→∞ an n→∞ n→∞ n→∞ an

an+1
Preuve : Notons ` = lim . Supposons ` > 0, et soit λ ∈]0, `[. Il existe N tel que,
n→∞ an
an+1
pour tout n ≥ N , nous ovons ≥ λ. La suite (λ−n an )n≥N est donc croissante, d’où
an
∀ n ≥ N, an ≥ Aλn (avec A = λ−N aN ).
√ √
n
Il en résulte que lim n an ≥ λ lim A = λ. Comme λ est arbitraire dans ]0, `[, alors
n→∞ n→∞

on conclut que lim n an ≥ `. Ce résultat est trivialement vrai si ` = 0. D’où la première
n→∞
inégalité.
an+1
De même, notons L = lim . Supposons L < +∞, et soit λ ∈]L, +∞[. Il
an
n→∞
an+1
existe N tel que, pour tout n ≥ N , nous avons ≤ λ. La suite (λ−n an )n≥N est
an
donc decroissante, d’où

∀ n ≥ N, an ≤ Aλn (avec A = λ−N aN ).


√ √
n
Il en résulte que lim n an ≤ λ lim A = λ. Comme λ est arbitraire dans ]L, +∞[,
n→∞ √ n→∞
alors on conclut que lim n an ≤ L. Ce résultat est trivialement vrai si L = +∞. D’où
n→∞
la dernière inégalité.

Cette proposition montre que l’ensemble des séries pour lesquelles la règle de
D’Alembert permet de conclure leur convergence ou divergence est inclus (strictement
comme le montre l’exemple suivant) dans l’ensemble des séries pour lesquelles la règle
de Cauchy permet de conclure leur convergence ou divergence. On dit que la règle de
Cauchy est plus précise que la règle de D’Alembert.
8 Séries numériques
P
Exemple : Pour 0 < a < b < 1, on considère la série xn avec x2n = b2n et

x2n+1 = a2n+1 . Il est immédiat de vérifier que lim n xn = b < 1 donc la série converge
n→∞
xn+1 xn+1
d’après la règle de Cauchy. Mais lim = +∞ et lim = 0, donc la règle de
n→∞ xn n→∞ xn
D’Alembert ne permet pas de conclure pour cette série.

Le corollaire suivant est très utile.

an+1
Corollaire III.6. Soit (an )n∈IN une suite à termes strictement positifs. Si lim
n→∞ an

existe et vaut ` ∈ IR+ Alors lim n an existe et vaut ` aussi.
n→∞

n an+1 2(2n + 1) p
Exemple : Si an = C2n alors = n = 4.
−−−→ 4. Donc lim n C2n
an n + 1 n→∞ n→∞

IV. Séries absolument convergentes, Séries semi-convergentes

P
Soit an une série numérique (i.e. à termes réels ou complexes). On dira que
P P
an est absolument convergente si, et seulement si, la série à termes positifs | an |
P
est convergente. On dira aussi que an est semi-convergente si, et seulement si, elle
P
est convergente et la série à termes positifs | an | est divergente.
Remarquons que la proposition II.2 montre que toute série absolument convergente
est convergente. L’existence de séries semi-convergentes montre que la réciproque de
cette proposition est fausse.

X∞
(−1)n
Exemple : La série n’est pas absolument convergente. Mais
n=1
n

n
X n
X Z 1 Z 1
(−1)k k k−1 1 − (−x)n
Sn = = (−1) x dx = − dx
k 0 0 1+x
k=1 k=1

ce qui permet d’écrire que


¯ Z 1 ¯ Z 1 n Z 1
¯ dx ¯¯ x 1
¯ Sn + ≤ dx ≤ xn dx =
¯ 1+x ¯
0 0 1+x 0 n+1

X∞
1 (−1)n
ou bien | Sn + Log 2 | ≤ . Donc la série converge et admet −Log 2 pour
n+1 n=1
n
somme.
Convergence absolue et Semi-convergence 9

Le théorème suivant met en avant une technique importante et assez classique pour
l’étude des séries semi-convergentes, c’est ce qu’on appelle la transformation d’Abel, elle
joue pour les séries le rôle de l’intégration par parties pour les intégrales.
P
Théorème IV.1. Pour que la série an bn converge il suffit que les trois conditions
suivantes soient satisfaites:
n
X
¦ Les sommes partielles An = ak forment une suite bornée.
k=0
¦ La suite (bn )n∈IN tend vers zéro.
P
¦ La série | bn+1 − bn | est convergente.

Preuve : Notons A−1 = 0. Si 0 ≤ p ≤ q, nous pouvons écrire


q
X q
X q
X q
X
an bn = (An − An−1 )bn = An bn − An−1 bn
n=p n=p n=p n=p
q
X q−1
X
= An b n − An bn+1
n=p n=p−1
q
X
= An (bn − bn+1 ) + Aq bq+1 − Ap−1 bp
n=p

D’après l’hypothèse, il existe une constante M tel que, pour tout n, nous avons
| An | ≤ M . Il en résulte que, pour 0 ≤ p ≤ q,
¯ q ¯ Ã q !
¯X ¯ X
¯ ¯
¯ an bn ¯ ≤ M | bn − bn+1 | + | bq+1 | − | bp | (∗)
¯ ¯
n=p n=p

Soit ε > 0, la convergence de (bn )n vers zéro montre qu’il existe N1 tel que
ε
k ≥ N1 =⇒ | bk | ≤ ,
4M
P
d’autre part, la convergence de | bn+1 − bn | montre qu’il existe N2 tel que
q
X ε
N2 ≤ p ≤ q =⇒ | bn+1 − bn | ≤ ,
n=p
2M

Alors, en revenant à (∗),


¯ q ¯
¯X ¯ ³ ε ε ε ´
¯ ¯
max(N1 , N2 ) ≤ p ≤ q =⇒ ¯ an bn ¯ ≤ M + + = ε.
¯ n=p ¯ 2M 4M 4M
P
La suite des sommes partielles de la série an bn est alors de Cauchy donc elle est
convergente.
10 Séries numériques

Corollaire IV.2. Si (bn )n est une suite réelle décroissante vers zéro, et si (an )n est
Xn
une suite telle que les sommes partielles An = ak forment une suite bornée. Alors
P k=1
la série an bn est convergente.
C’est immédiat car la troisième condition du théorème est automatiquement vérifiée
dans ce cas.
P
Exemple : Soit (λn )n∈IN une suite décroissante vers zéro. Alors la série λn einx
converge pour tout x ∈]0, 2π[.
En Effet, pour tout n ∈ IN∗ et tout x ∈]0, 2π[,
n−1
X 1 − einx ei(n+1/2)x − e−ix/2
eipx = = .
p=0
1 − eix 2i sin(x/2)

donc ¯ ¯
¯ n−1 ¯
¯ X ipx ¯ 1
¯ e ¯≤ .
¯ ¯ sin(x/2)
p=0

On peut donc, utiliser le corollaire pour conclure. En particulier, pour tout α > 0 et
X∞
einx
tout x ∈]0, 2π[, la série α
est convergente. Elle est absolument convergente si
n=1
n
α > 1 et semi-convergente si α ∈]0, 1[.

5
! Les résultats de la proposition III.2 ne s’appliquent pas pour les séries qui ne sont pas
P P
à termes positifs. Considérons par exemple les deux séries an et bn avec
sin n sin n
an = √ , bn = √ .
n + sin n n + cos n
Un développement limité simple montre que
sin n cos 2n 1 1
an = √ + − + O( 3/2 ).
n 2n 2n n
P sin n cos 2n 1
Alors, la série αn avec αn = an − √ − + est absolument convegente, et
n 2n 2n
X sin n cos 2n
d’après l’exemple précédent les deux séries √ et sont semi-convergentes.
n 2n
X 1 X1
On conclut que la série (an + ) est convergente. Mais est divergente, alors
P 2n n
an est une série divergente.
D’autre part,
sin n sin 2n 1
bn = √ − + O( 3/2 ).
n 2n n
P
Une étude similaire montre que dans ce cas bn est une série convergente. Pourtant
an
−−−→ 1.
bn n→∞
Produit de deux séries 11

Une classe importante de séries numériques est celle des séries alternées. Une série
alternée est une série numérique dont le term général est de la forme (−1)n αn (ou
(−1)n+1 αn ), avec (αn )n≥0 une suite à termes positifs.
P
Proposition IV.3. Soit an une série alternée avec an = (−1)n αn , et (αn )n≥0 une
Xn
P
suite décroissante vers zéro. Alors la série an est convergente. De plus, si Sn = ak
k=0

X
et S = ak , Alors
k=0

∀ n ∈ IN, S2n+1 ≤ S ≤ S2n et | S − Sn | ≤ αn+1 .

Preuve : En effet, pour tout n ≥ 0, nous avons les inégalités suivantes:


S2n − S2n+1 =α2n+1 ≥ 0,
S2n+2 − S2n =α2n+2 − α2n+1 ≤ 0,
S2n+1 − S2n−1 =α2n − α2n+1 ≥ 0.
Donc la suite (S2n )n≥0 est décroissante, la suite (S2n+1 )n≥0 est croissante, et de plus
(S2n − S2n+1 )n≥0 tend vers zéro toute en restant positive. Il en résulte que les deux
suites sont adjacentes, et ont une limite commune S. De plus, (S2n+1 )n≥0 croit vers S,
et (S2n )n≥0 decroit vers S, ce qui démontre la première inégalité. D’autre part, les deux
inégalités S2n+1 ≤ S ≤ S2n et S2n+1 ≤ S ≤ S2n+2 montrent que | S − Sp | ≤ αp+1 pour
tout p. (En distinguant les cas p = 2n et p = 2n + 1).

V. Produit de deux séries

Soient A = (an )n≥0 et B = (bn )n≥0 deux suites numériques. On définit la suite
C = A ∗ B = (cn )n≥0 , dite produit de convolution de A et B, par
n
X X
∀ n ≥ 0, cn = ak bn−k = ai bj .
k=0 i+j=n

Théorème V.1. Soient A = (an )n≥0 et B = (bn )n≥0 deux suites numériques, on note
P
C = A∗B la suite produit de convolution de A et B. Si an est absolument convergente
P P
et bn est convergente. Alors cn est aussi convergente. De plus,

Ã∞ !à ∞ !
X X X
cn = an bn .
n=0 n=0 n=0
12 Séries numériques

Preuve : Fixons d’abord les notations suivante:


n
X ∞
X ∞
X
¯ ¯
SnB = bk , S B
= bn , M = sup ¯ SnB ¯ , S |A|
= | an | .
n≥0 n=0
k=0 k=0

Écrivons,

c0 = a0 b0
c1 = a0 b1 + a1 b0
c2 = a0 b2 + a1 b1 + a2 b0
.. .. .. .. ..
. . . . .
cn = a0 bn + a1 bn−1 + a2 bn−2 + · · · + an b0
n
X
ck = a0 SnB + a1 Sn−1
B B
+ a2 Sn−2 + · · · + an S0B
k=0
Nous voyons que, pour tout n ≥ 0,
n
X n
X n
X
B
ck − S ak = an−k (SkB − S B ). (∗)
k=0 k=0 k=0

Soit ε > 0, il existe un entier N tel que


¾ n+m
X
n≥N ¯ ¯ ε ε
=⇒ ¯ SnB − S B ¯ < et | ak | < .
m ∈ IN 2S | A | 4M
k=n

Soit alors n > 2N , en utilisant (∗) on a,


¯ ¯ ¯ ¯ ¯ ¯
¯X n n
X ¯ ¯X N ¯ ¯ X n ¯
¯ B ¯ ¯ B B ¯ ¯ B B ¯
¯ ck − S ak ¯ ≤ ¯ an−k (Sk − S ) ¯ + ¯ an−k (Sk − S ) ¯
¯ ¯ ¯ ¯ ¯ ¯
k=0 k=0 k=0 k=N +1
N
X n
X
ε
≤2M | an−k | + | an−k |
k=0
2S | A | k=N +1
n
X ∞
X
ε
≤2M | ak | + | ak |
k=n−N
2S | A | k=0
ε ε
≤2M + | A |
S| A | = ε
4M 2S
à n n
!
X X P
Il en résulte que lim ck − B ak = 0. D’où cn converge et
n→∞
k=0 k=0


à ∞
!Ã ∞
!
X X X
cn = an bn .
n=0 n=0 n=0

Remarquons que la condition de convergence absolue de l’une des deux séries est
indispensable comme le montre le l’exemple suivant.
Expressions asymptotiques liés aux séries numériques 13

(−1)n P
Exemple : Si A = (an )n≥1 avec an = √ , alors la série an est semi-convergente.
n
Par contre, si C = A ∗ A alors
n−1
X
n 1
cn = (−1) p .
k=1
k(n − k)

en particulier
n
1X 1 k
| cn | = p avec xk = .
n xk (1 − xk ) n
k=1
p 2(n − 1)
mais x(1 − x) ≤ 1/2 pour tout x ∈ [0, 1], donc | cn | ≥ . En particulier, (cn )n
P n
ne tend pas vers zéro, et la série cn diverge.

VI. Expressions asymptotiques liés aux séries numériques

Théorème VI.1. Soient (un )n≥0 et (vn )n≥0 deux suites numériques, avec un ≥ 0 pour
tout n.
P
¦ Si la série un converge, alors Ã∞ !
X∞ X
1◦ . vn = O(un ) =⇒ vk = O uk .
k=n

à k=n

!
X X
2◦ . vn = o(un ) =⇒ vk = o uk .
k=n k=n
X∞ ∞
X
3◦ . vn ∼ un =⇒ vk ∼ uk .
P k=n k=n
¦ Si la série un diverge, alors à n !
Xn X
1◦ . vn = O(un ) =⇒ vk = O uk .
k=1
n
à n
k=1 !
X X
2◦ . vn = o(un ) =⇒ vk = o uk .
k=1 k=1
Xn n
X
3◦ . vn ∼ un =⇒ vk ∼ uk .
k=1 k=1
P
Preuve : Supposons d’abord que un converge.

1 . L’hypothèse vn = O(un ) montre l’existence d’une constante A > 0 et d’un entier
N0 tels que | vn | ≤ Aun , pour tout n ≥ N0 . Ce qui démontre par inégalité triangulaire
que ¯ ¯
¯X∞ ¯ ∞
X
¯ ¯
∀ n ≥ N0 , ¯ vk ¯ ≤ A uk .
¯ ¯
k=n k=n
14 Séries numériques

2◦ . Soit ε > 0, l’hypothèse vn = o(un ) montre l’existence d’un entier Nε tel que
| vn | ≤ εun , pour tout n ≥ Nε . Ce qui démontre par inégalité triangulaire que
¯ ∞ ¯
¯X ¯ ∞
X
¯ ¯
∀ n ≥ Nε , ¯ vk ¯ ≤ ε uk .
¯ ¯
k=n k=n

3◦ . L’hypothèse vn ∼ un est équivalente à vn − un = o(un ). Le point précédent


permet de déduire que à !

X ∞
X ∞
X
vk − uk = o uk .
k=n k=n k=n

ce qui démontre le résultat.


P
Supposons ensuite que un diverge.

1 . D’après l’hypothèse, il existe une constante A > 0 et un entier N1 tels que
P
| vn | ≤ Aun , pour tout n ≥ N1 . La divergence de un montre l’existence
¯N ¯ d’un Nentier
XN ¯ ¯
¯X ¯ X
0 0 0

N0 ≥ N1 tel que uk > 0. On définit alors la constante B par ¯ vk ¯ = B uk .


¯ ¯
k=0 k=0 k=0
Pour tout n ≥ N0 , nous avons
¯ ¯ ¯N ¯ ¯ ¯
¯X n ¯ ¯X 0 ¯ ¯ X n ¯
¯ ¯ ¯ ¯ ¯ ¯
¯ vk ¯ ≤ ¯ vk ¯ + ¯ vk ¯
¯ ¯ ¯ ¯ ¯ ¯
k=0 k=0 k=N0 +1
N0
X n
X n
X
≤B uk + A uk ≤ max(A, B) uk .
k=0 k=N0 +1 k=0

n
à n
!
X X
d’où vk = O uk .
k=0 k=0
◦ ε
2 . Soit ε > 0, Il existe Nε tel que | vn | ≤ un , pour tout n ≥ Nε . D’où
2
¯ ¯
¯ X
n ¯ εX n
¯ ¯
∀ n ≥ Nε , ¯ vk ¯ ≤ uk .
¯ ¯ 2
k=Nε +1 k=0

P eε ≥ Nε tel que
La divergence de un montre l’existence de N
¯N ¯
¯X ε ¯ εX n
e ¯ ¯
∀ n ≥ Nε , ¯ vk ¯ ≤ uk .
¯ ¯ 2
k=0 k=0

Il en résulte que,
¯ ¯ ¯N ¯ ¯ ¯
¯Xn ¯ ¯X ε ¯ ¯ X n ¯ ε ε X
n n
X
eε , ¯ ¯ ¯ ¯ ¯ ¯
∀n ≥ N ¯ vk ¯ ≤ ¯ vk ¯ + ¯ vk ¯ ≤ ( + ) uk = ε uk .
¯ ¯ ¯ ¯ ¯ ¯ 2 2
k=0 k=0 k=Nε +1 k=0 k=0
Expressions asymptotiques liés aux séries numériques 15

Ce qui démontre le résultat.


3◦ . L’hypothèse vn ∼ un est équivalente à vn − un = o(un ). Le point précédent
permet de déduire que à !
n
X n
X n
X
vk − uk = o uk .
k=0 k=0 k=0

Ce qui démontre le résultat.

Exemples :

X 1
¦ Soit α > 1. Notons Rn(α) = . Il est immédiat de voir que

k=n
µ ¶
1 1 1 1
− ∼ .
α−1 nα−1 (n + 1)α−1 nα
X 1
Comme la série est convergente alors le théorème précédent montre que

1
Rn(α) ∼ .
(α − 1)nα−1
n
X 1
¦ Notons Hn = . Il est immédiat de voir que
k
k=1

1 1
Log (1 + n) − Log n = Log (1 + )∼ .
n n
X1
Comme la série est divergente alors le théorème précédent montre que
n
1 1
Hn ∼ Log n. Précisons un plus ce résultat. Notons γn = − Log (1 + ), il est
n n
X∞
1 P
facile de voir que γn ∼ 2 donc la série γn converge. On pose γ = γn , et
2n n=1
on appelle γ la constante d’Euler. Notons, d’autre part, que
n
X
γk = Hn − Log (n + 1).
k=1

D’où lim (Hn − Log (n + 1)) = γ. La convergence de Hn − Log (n + 1) vers γ est


n→∞
très lente car

X ∞
X 1 1
λn = γ − (Hn−1 − Log n) = γk ∼ ∼ .
2k 2 2n
k=n k=n

Ce qui voudrait dire qu’il faut calculer environs un million de termes pour obtenir
les premiers 6 ciffres significatifs de γ. Il faut donc “accélérer la convergnce” de
16 Séries numériques

cette suite pour pouvoir obtenir une valeur approchée de γ avec moins de termes
à calculer. Cherchons pour cela a et b qui rendent
a b
δn = γn − −
n(n + 1) n(n + 1)(n + 2)
1
un infiniment petit de l’ordre le plus élevé en . Or
n
1 1 1 1 1 1 1
δn = ( − a) 2 + (− + a − b) 3 + ( − a + 3b) 4 + O( 5 ).
2 n 3 n 4 n n
1 1 1
Nous prenons alors a = et b = , de sorte que δn ∼ 4 . En utilisant le théorème
2 6 4n
X∞
1
et l’exemple précédents nous obtenons δk ∼ .
12n3
k=n
En notant que
1 1 1 1 1 1
= − , = −
n(n + 1) n n+1 n(n + 1)(n + 2) 2n(n + 1) 2(n + 1)(n + 2)
on voit que
X∞ ∞
X 1 1
δk = γk − −
2n 12n(n + 1)
k=n k=n
1 1
=γ − (Hn−1 − Log n) − −
2n 12n(n + 1)
µ ¶
6n + 7
=γ − Hn−1 − Log n +
12n(n + 1)
D’où, µ ¶
e 6n + 7 1
λn = γ − Hn−1 − Log n + ∼ .
12n(n + 1) 12n3
6n + 7
Ce qui voudrait dire qu’avec le terme correcteur , il suffit de calculer une
12n(n + 1)
centaine de termes pour obtenir les premiers 6 ciffres significatifs de γ. Le tableau
suivant illustre l’idée d’accélération de convergence. Les chiffres souslignés sont des
chiffres exacts de γ.

1 6n + 7
n Hn−1 − Log n Hn−1 − Log n + Hn−1 − Log n +
2n 12n(n + 1)
10 0.5310729 0.57652752662 0.57715883975574201
102 0.5722570 0.57720749584 0.57721558489206429
103 0.5767160 0.57721558173 0.57721566481854024
104 0.5771656 0.57721566406 0.57721566490144959
5
10 0.5772106 0.57721566489 0.57721566490153273

Et voici γ à 10−40 près:


γ = 0.57721 56649 01532 86060 65120 90082 40243 10422 . . .!
Exercices 17

EXERCICES

P
Exercice .1 Etudier les séries vn de termes généraux:

µ ¶n3
¡ 1 ¢2n ¡ 2 ¢n Arc tg n
vn = 1+ − 1+ ; vn = ;
n+1 n + a2 Arc tg(n + 1)
a b b p p
vn = (cos + sin )n − eax (1 + ); vn = ( n2 + an + 2 − n2 + bn + 1)n ;
n n n
µ ¶ √
πn π 1 n2 + n
vn = tg − cos ; vn = tg + Log 2 ;
4n + 1 n n n −n
µ ¶n2 p p
Log n
vn = ; vn = n4 + 2n + 1 − n4 + kn;
Log (n + 1)
n n
1 X √ Y 1
vn = (Log p)2 ; vn = n! sin √ .
nα p=1 p=1
p

P
Exercice .2 Etudier de même les séries vn de termes généraux:

p p √
n2 + n
vn = Arctg 1 + 1/n − Arctg 1 − 1/n; vn = Log 2 tg (1/n2 );
n −n
¯ p ¯2/3 µ ¶n µ ¶
¯ ¯ n+1 3 3
vn = ¯ sin(π n4 + 1) ¯ ; vn = −e 1+ ;
n−2 2n
p3
p
vn = sin(π n3 + λnα ) (0 < α ≤ 2); vn = sin(π n2 + an + b);

(−1)n (−1)n
vn = ; vn = .
n + (−1)n+1 Log (n + (−1)n )

Exercice .3 Soit p un entier plus grand que 2, et (an )n∈IN une suite décroissante de
P P
réels positifs. Montrer que les séries an et bn où bn = pn apn sont de même nature.
X 1
Etudier .
n( Log n)β
n≥2

Exercice .4 Soit (un )n≥0 une suite décroissante à termes positifs telle que la série
P
un converge. Montrer que lim nun = 0.
n→+∞
18 SÉRIES NUMÉRIQUES

Exercice .5 Soient (an )n≥1 une suite de réels positifs tendant vers l’infini en
P xn
croissant, et (xn )n≥1 une suite de réels telle que la série converge. Montrer que
an
n
1 X
lim xk = 0.
n→+∞ an
k=1

Exercice .6 Soit f : [0, 1] −→ IR une application intégrable ; montrer que la série



X Z 1
n
(−1) xn f (x) dx
n=0 0
Z 1
f (x)
converge, et que sa somme est dx.
0 1+x
Exercice .7 Déterminer la nature de la suite de terme général vn , dans chacun des
cas suivants
n
X n
X 1 n1−α
vn = Log k − (n + 1/2)Log n + n; vn = − , α ∈]0, 1[;
kα 1−α
k=2 k=1
Xn Xn
1
vn = th k − Log ( ch n) ; vn = √ − Argsh n ;
k=1 k=1
k2 +1
Xn Xn
Log k (Log n)2 1
vn = − ; vn = − Log Log n .
k 2 kLog k
k=1 k=2

Exercice .8 1◦ . Soient f et g deux fonctions ayant un développement limité du


premier ordre en 0 de la forme f (x) = a + bx + o(x) ; g(x) = a + b0 x + o(x) où b 6= b0 .
Que peut-on dire du signe de (f − g)(x) pour x assez petit positif ?
vn+1 1
2◦ . Trouver le développement limité de au premier ordre en lorsque
vn n
1
vn = λ où λ 6= 0 pour n tendant vers l’infini.
n
3◦ . Soit (un )n∈IN une suite à termes strictement positifs. On pose
¡ un ¢ ¡ un ¢
L = lim n −1 , l = lim n −1
n→∞ un+1 n→∞ un+1
P P
Montrer que, si L < 1 alors la série un diverge, et que si l > 1 alors la série un
converge. Que ce passe-t-il si l ≤ 1 ≤ L ?

Exercice .9 Soit (an )n≥1 une suite de réels strictement positifs. On note vn =
an
.
(1 + a0 ) · · · (1 + an ) P
1◦ . Montrer que la série vn converge.
2◦ . Montrer que
+∞
X P
vn = 1 ⇐⇒ La série an diverge.
n=0
Exercices 19
P
Exercice .10 Soit un une série convergente à termes réels positifs.

1 . Montrer lim (u1 + 2u2 + · · · + nun )/n = 0.
n→∞
X 1
En déduire que la série (u1 + 2u2 + · · · + nun ) converge, et que
n(n + 1)
n≥1


X X∞
1
(u1 + 2u2 + · · · + nun ) = un .
n=1
n(n + 1) n=1

X (n!u1 u2 . . . un )1/n
2◦ . Montrer que la série converge, et que
n+1
n≥1

X∞ X∞
(n!u1 u2 . . . un )1/n
≤ un .
n=1
n + 1 n=1

X u1 + u2 + · · · + un
3◦ . Montrer que s’il existe n0 tel que un0 6= 0 alors diverge.
n
n≥1
X
◦ 1/n
4 . Montrer que (u1 u2 . . . un ) converge, et que
n≥1


X ∞
X
1/n
(u1 u2 . . . un ) ≤e un .
n=1 n=1

Exercice .11 Soit (un )n≥1 une suite à termes strictement positifs telle que la série
X 1 X n
converge. Montrer que la série converge, et que
un u1 + · · · + un
n≥1 n≥1


X X∞
n 1
≤2 .
n=1
u 1 + · · · + un n=1
u n

X ϕ(n)
Exercice .12 Montrer que si ϕ : IN∗ −→ IN∗ est injective, alors la série
n2
n≥1
diverge.

Exercice .13 Soient α ∈ IR∗+ , et (un )n≥1 la suite définie par

n
(−1)n+1 X
u1 ∈ IR∗+ , ∀n ≥ 1, un+1 = uk .
(n + 1)α
k=1
P
Etudier la nature de la série un .
20 SÉRIES NUMÉRIQUES
P
Exercice .14 Soit (un )n≥0 une suite de IR+ telle que la série un converge. On
X∞
suppose qu’il existe c > 0 tel que uk ≤ cun . Montrer que la suite (un ) est majorée
k=n+1
par une suite du type (b.an )n∈IN avec 0 < a < 1.

P
Exercice .15 Soit (an )n≥0 une suite de IR+ telle que la série a2n converge. On
X∞ Xn
2 An
note A = an et ∀ n, An = ak et αn = .
n=1
n
k=1
1◦ . Montrer que, pour tout n, αn2 − 2αn an ≤ (n − 1)αn−1
2
− nαn2 .
P 2
2◦ . En déduire que la série αn converge et que sa somme est ≤ 4A.

n
X
Exercice .16 Soit (an )n≥0 une suite de IR+ , et, pour tout n, Sn = ak . On
k=0
Sn P
suppose ∀ n ≥ 1, an ≤ 2
. Quelle est la nature de la série an .
n

Exercice .17 Montrer que la série de terme général an = Arc tg (n + a) − Arc tg n


est convergente. Si f (a) désigne sa somme, trouver un équivalent de f (a) au voisinage
de l’infini.
Exercice .18 Soit (un )n≥0 une suite à termes strictement positifs. On suppose qu’il
existe α > 1 tel que
µ ¶α
un+1 n
≤ .
un n+1

P 1.3.5 . . . (2n − 1) 1
Montrer que un converge. Appliquer le résultat à un = .
2.4.6 . . . (2n) 2n + 1

Exercice .19 Soit a un réel strictement positif. On considère la série de terme


général
n
Y
(n + 1)(n + 2) · · · (n + n) n+k
vn = n n
=
a n an
k=1

1◦ . Montrer qu’il existe λ ∈ IR+


∗ tel que
P
si a > λ la série vn converge.
P
si a < λ la série vn diverge.
2◦ . Dans cette question on suppose que a = λ.
X vn+1
—Montrer que la série Log converge.
vn
n≥1
—En déduire que la suite (vn )n≥1 ne tend pas vers 0. Conclure.
Exercices 21

Exercice .20 Soit f : IR+ −→ IR∗+ une fonction de classe C 1 . On suppose qu’il
f 0 (x)
existe un nombre µ ∈ IR∗ tel que lim = µ.
x→∞ f (x)
R∞
1◦ . Si f < +∞, montrer que
0


X Z ∞
µ
f (p) ∼ µ f (t) dt.
p=n
e −1 n

R∞
2◦ . Si f diverge, montrer que
0

n
X Z n+1
µ
f (p) ∼ µ f (t) dt.
p=0
e −1 0

Etudier le cas µ = 0.
22 SÉRIES NUMÉRIQUES

SOLUTIONS

P
Solution .1 1◦ . Étude de la série vn avec

¡ 1 ¢2n ¡ 2 ¢n
vn = 1 + − 1+ .
n+1 n + a2

On a,
¡ 1 ¢ ¡ 2¢ ¡ 1¢
Log 1 + =Log 1 + − Log 1 +
n+1 n n
¡2 2 ¢ ¡ 1 1 ¢ 1
= − 2 − − 2 + O( 3 )
n n n 2n n
¡1 3 ¢ 1
= − + O( 3 ).
n 2n2 n
alors
¡ 1 ¢2n 3 1
1+ = exp(2 − + O( 2 ))
n+1 n n
µ ¶
2 3 1
=e 1 − + O( 2 ) .
n n
De même,

¡ 2 ¢ ¡ 2 + a2 ¢ ¡ a2 ¢
Log 1 + =Log 1 + − Log 1 +
n + a2 n n
¡ 2 + a2 (2 + a )2 2¢ ¡ a2
a4 ¢ 1
= − 2
− − 2
+ O( 3 )
n 2n n 2n n
¡2 2(1 + a )2 ¢
1
= − 2
+ O( 3 ).
n n n

alors
¡ 2 ¢n 2(1 + a2 ) 1
1+ 2
= exp(2 − + O( 2 ))
n+a n n
µ 2

2 2(1 + a ) 1
=e 1 − + O( 2 ) .
n n
On conclut que, µ ¶
2 2a2 − 1 1
vn = e + O( 2 ) .
n n
P
d’où vn converge si, et seulement si, a2 = 1/2.
P
2◦ . Étude de la série vn avec

µ ¶n3
Arctg n
vn = .
Arctg (n + 1)
Solutions 23

π 1
Si x > 0, alors Arctg x = − Arctg . D’où
2 x
1 1
Arctg (n + 1) − Arctg n =Arctg − Arctg
n n+1
1 1 1
= − + O( 3 )
n n+1 n
1 1
= 2 + O( 3 )
n n
π 1 π 1
mais Arctg (n + 1) = − Arctg = + O( ). Alors
2 n+1 2 n

Arctg n 2 1
1− = 2
+ O( 3 ).
Arctg (n + 1) πn n

Il en résulte que µ ¶
Arctg n 2 1
Log =− 2
+ O( 3 ).
Arctg (n + 1) πn n
puis
µ ¶n2
Arctg n 2 1
= exp(− )(1 + O( )).
Arctg (n + 1) π n
√ P
On conclut que lim n vn = e−2/π < 1. La série vn est, par la règle de Cauchy,
n→∞
convergente.
P
3◦ . Étude de la série vn avec

a b b
vn = (cos + sin )n − eax (1 + ).
n n n

Notons que
a b b a2 1
cos + sin = 1 + − 2 + O( 3 ).
n n n 2n n
Pour n assez grand on peut, écrire
µ ¶
a b b a2 + b2 1
Log cos + sin = − 2
+ O( 3 )
n n n 2n n

soit µ ¶n µ ¶
a b a2 + b2 1
cos + sin = exp b − + O( 2 )
n n 2n n
µ 2 2

b a +b 1
=e 1 − + O( 2 ) .
2n n
Alors
(a2 + b2 )eb − 2beax 1
vn = eb − eax + + O( 2 ).
2n n
24 SÉRIES NUMÉRIQUES
P
On conclut que la série vn converge si, et seulement si, b = ax et a2 + b2 − 2b = 0.
Ce qui est équivalent à

2x 2x2
(a = 0, b = 2) ou (a = , b= ).
1 + x2 1 + x2
P
4◦ .Etude de la série vn avec
p p
vn = ( n2 + an + 2 − n2 + bn + 1)n .

Notons que
p µ ¶1/2
2
c d
n + cn + d =n 1 + +
n n2
µ ¶
1 c d 1 c d 2 1
=n 1 + ( + ) − ( + 2 ) + O( 3 )
2 n n2 8 n n n
c 4d − c2 1
=n + + + O( 2 ).
2 8n n
Alors,
p p a − b 4 + b2 − a2 1
n2 + an + 2 − n2 + bn + 1 = + + O( 2 ).
2 8n n
P
Si | a − b | > 2 alors (vn )n≥1 ne tend¯ pas vers
¯ 0 et alors vn diverge.
p ¯ a − b ¯ P
Si | a − b | < 2 alors lim n | vn | = ¯¯ ¯ < 1 et alors vn converge.
n→∞ 2 ¯
P
Si | a − b | = 2 alors, lim | vn | = e−b/2 6= 0, donc vn diverge. On conclut que
P n→∞
vn converge si, et seulement si, | a − b | < 2.
P
5◦ . Étude de la série vn avec
µ ¶
πn π
vn = tg − cos .
4n + 1 n
En effet, on a

µ ¶ π πn π πn π
πn sin
cos − cos sin sin
un = 1 − tg = 4 4n + 1 4 4n + 1 = 16n + 4
4n + 1 π πn π πn
cos cos cos cos
4 4n + 1 4 4n + 1
π π P
D’où, lim nun = . On conclut que lim nvn = − , et alors, vn diverge.
n→∞ 8 P n→∞ 8
6◦ .Etude de la série vn avec

1 n2 + n
vn = tg + Log 2 .
n n −n

n2 + n 1 1 1
Clairement, Log 2 ≥ 0 et tg ≥ . Alors, pour tout n ≥ 1, vn ≥ et la série
P n −n n n n
vn diverge.
Solutions 25
P
7◦ . Étude de la série vn avec
µ ¶n2
Log n
vn = .
Log (n + 1)
Notons que,
1 1 1
Log (n + 1) − Log n = Log (1 + ) = + O( 2 ).
n n n
D’où,
Log (n + 1) 1 1
=1+ + O( 2 )
Log n nLog n n Log n
puis µ ¶ √
√Log (n + 1) n 1
n n Log = + O( √ ).
Log n Log n nLog n
µ ¶
√ Log (n + 1)
Alors, lim n n Log = +∞. Il existe N ∈ IN tel que
n→∞ Log n
µ ¶
√ Log (n + 1)
∀ n ≥ N, n n Log ≥ 1.
Log n
Ce qui montre,

∀ n ≥ N, 0 ≤ vn ≤ e− n
.

Mais lim n2 e− n e ∈ IN tel que
= 0, alors il existe N
n→∞

e,
√ 1
∀n ≥ N e− n
≤ .
n2
On conclut que
e ), 1
∀ n ≥ max(N, N 0 ≤ vn ≤
n2
P
et la série vn converge.

P
8 . Étude de la série vn avec
p p
vn = n4 + 2n + 1 − n4 + kn.

On a,
1 k 1
vn =n2 (1 +3
) − n2 (1 + 3 ) + O( 2 )
n 2n n
2−k 1
= + O( 2 ).
2n n
P
La série vn converge si, et seulement si, k = 2.
P
9◦ . Étude de la série vn avec
n
1 X
vn = α (Log p)2 .
n p=1
26 SÉRIES NUMÉRIQUES

Notons que, pour β < 1, on a


n
X X 1 + nβ
(Log p)2 ≥ (Log p)2 ≥ (n − nβ − 1)(Log nβ )2 = β 2 n(Log n)2 (1 − ).
p=1
n
nβ ≤p≤n

On en déduit que, pour tout β < 1,

Xn
1
lim 2
(Log p)2 ≥ β 2 ,
n→∞ n(Log n)
p=1

ce qui montre que


Xn
1
lim 2
(Log p)2 ≥ 1.
n→∞ n(Log n)
p=1
n
X
De l’autre côté, on a évidemment (Log p)2 ≤ n(Log n)2 , donc
p=1

Xn
1
lim (Log p)2 ≤ 1.
n→∞ n(Log n)2
p=1

On conclut que
Xn
1
lim (Log p)2 = 1.
n→∞ n(Log n)2
p=1

nα−1 P X (Log n)2


Alors lim v n = 1. Les séries v n et sont de même nature, donc
n→∞ (Log n)2 nα−1
P
vn concerge si, et seulement si, α > 2.
P
10◦ . Étude de la série vn avec

√ n
Y 1
vn = n! sin √ .
p=1
p

Il est facile de vérifier que,

∀ x ∈ IR+ , sin x ≥ x − x3 .
√ 1 p−1
alors p sin √ ≥ , pour tout p ≥ 1. Il en résulte que
p p

√ n
Y Yn
1 p−1 1
n! sin √ ≥ = .
p=2
p p=2 p n

sin 1 P
D’où vn ≥ et la série vn diverge.
n
Solutions 27

P
Solution .2 1◦ . Étude de la série vn avec
p p
vn = Arctg 1 + 1/n − Arctg 1 − 1/n.

On a p p
1 + 1/n − 1 − 1/n 1 1
vn = Arctg p = + O( 2 ).
1 + 1 − 1/n2 2n n
P P1
Alors, vn diverge car elle est de la même nature que .
P n
2◦ . Étude de la série vn avec

n2 + n
vn = Log 2 tg (1/n2 ).
n −n
X 1
Clairement on a lim n2 vn = 0. Comme converge alors il en est de même de la
P n→∞ n2
série vn .
P
3◦ . Étude de la série vn avec
¯ p ¯2/3
¯ ¯
vn = ¯ sin(π n4 + 1) ¯ .

On a
p 1 1
n4 + 1 = n2 + 2
+ O( 6 ).
2n n
D’où ¯ ¯ ¯¯ ¯
¯ p ¯ ¯ π 1 ¯¯ 1
4
¯ sin(π n + 1) ¯ = ¯ sin( 2 + O( 6 )) ¯ = O( 2 ).
2n n n
1 P
On conclut que vn = O( 4/3
) et la série vn converge.
nP
4◦ . Étude de la série vn avec
µ ¶n µ ¶
n+1 3 3
vn = −e 1+ .
n−2 2n

On a, pour n ≥ 3,
n+1 1 2
Log =Log (1 + ) − Log (1 − )
n−2 n n
1 1 2 2 1 3 3 1
=( − 2 ) − (− − 2 ) + O( 3 ) = ( + 2 ) + O( 3 )
n 2n n n n n 2n n
puis µ ¶n µ ¶
n+1 3 1 3 3 1
= exp(3 + + O( 2 )) = e 1 + + O( 2 ) .
n−2 2n n 2n n
1 P
Alors vn = O( 2
), et la série vn converge.
n
28 SÉRIES NUMÉRIQUES
P
5◦ . Étude de la série vn avec
p
3
vn = sin(π n3 + λnα ) (0 < α ≤ 2).

Supposons d’abord que α ∈]0, 2[, et notons β = 3 − α ∈]1, 3[. Montrons d’abord que la

fonction définie sur ]0, 1[ par f (x) = λ 3 1 + λxβ est convexe dans un voisinage de 0. En
effet, un calcul simple montre que,

00 βxβ−2 λ2 3 1 + λxβ ¡ β
¢
f (x) = 3(β − 1) − λ(3 − β)x .
9(1 + λxβ )2
D’où, f est convexe sur ]0, 1/N ] pour un certain N ∈ IN∗ . Il en résulte que

λ 3 1 + λxβ − λ
x 7→ est croissante sur ]0, 1/N ].
x
ou bien, en composant avec x 7→ 1/x,
p
3
x 7→ λ x3 + λxα − λx est décroissante sur [N, +∞[.

3
Si λ > 0, alors la suite définie par un = n3 + λnα − n est décroissante à partir
de n = N . Mais vn = sin(πn + πun ) = (−1)n sin πun avec (sin πun )n≥N décroissante et
1
tendant vers 0, (car un = O( 2−α )). D’après la règle de convergence des séries alternées,
P n
on trouve que vn est convergente.

Si λ < 0, alors la suite définie par un = n − 3 n3 + λnα est décroissante à partir de
n = N . Mais vn = sin(πn − πun ) = (−1)n−1 sin πun avec (sin πun )n≥N décroissante et
P
tendant vers 0. D’après la règle de convergence des séries alternées on trouve que vn
est aussi convergente dans ce cas.
P
On conclut que si α ∈]0, 2[ alors vn converge pour toute valeur de λ.
Supposons que α = 2. Alors
p λ
n3 + λn2 =n(1 + )1/3
3

n
λ λ2 1
=n(1 + − 2 + O( 3 ))
3n 9n n
2
λ λ 1
=n + − + O( 2 )
3 9n n
D’où,
πλ πλ2 1
vn = (−1)n sin( − + O( 2 )).
3 9n n
P
On conclut que, si λ ∈
/ 3ZZ alors vn diverge car le terme général vn ne tend pas vers
0. Par contre, si λ = 3k pour k ∈ ZZ alors
πk 2 1
vn = (−1)n+k−1 + O( 2 ),
n n
Solutions 29
P
et la série vn converge.
On en déduit que
n X p o
3
(α, λ) ∈]0, 2] × IR : sin(π n3 + λnα ) converge = (]0, 2] × IR) ∪ ({2} × 3ZZ).
P
6◦ . Étude de la série vn avec
p
vn = sin(π n2 + an + b).

On a p a b
n2 + an + b = n(1 + + 2 )1/2
n n
a 4b − a2 1
=n+ + + O( 2 )
2 8n n
alors
aπ (4b − a2 )π 1
vn = (−1)n sin( + + O( 2 ))
2 8n n
P
donc si a ∈ / 2ZZ la série vn diverge car le terme général ne tend pas vers 0, et si
(b − k 2 )π 1 P
a = 2k ∈ 2ZZ alors vn = (−1)n+k + O( 2 ) et la série vn converge.
P 2n n
7◦ . Étude de la série vn avec

(−1)n
vn = .
n + (−1)n+1

(−1)n 1
Clairement, vn = + O( 2 ), donc la série est convergente.
n P n
8◦ . Étude de la série vn avec

(−1)n
vn = .
Log (n + (−1)n )

(−1)n 1
On a immédiatement, vn = + O( ), donc la série converge, car
Log n n(Log n)2
X (−1)n X 1
vérifie la règle de convergence des séries alternées et est con-
Log n n(Log n)2
vergente.(Voir l’exercice suivant).

Solution .3 Comme la suite (an )n≥1 est décroissante, alors pour tout entier k

pn ≤ k < pn+1 =⇒ apn ≥ ak ≥ apn+1 .

On en déduit que
X
(pn+1 − pn )apn ≥ ak ≥ (pn+1 − pn )apn+1 .
pn ≤k<pn+1
30 SÉRIES NUMÉRIQUES

Alors, pour tout n ≥ 0,

X p−1
(p − 1)bn ≥ ak ≥ bn+1
p
pn ≤k<pn+1

ce qui montre que, pour tout m > 0,

m
m−1
X pX −1 m
p−1 X
(p − 1) bn ≥ ak ≥ bn .
n=0
p n=1
k=1

P P
On conclut que les deux séries à termes positifs an et bn sont de la même nature.
P 1
Par exemple, la série de terme général an avec an = est de même
n(Log n)β
P 2n 1
nature que la série bn avec bn = n β
= β donc elle converge si, et
2 (nLog 2) n (Log 2)β
seulement si, β > 1.

2n
X P
Solution .4 Posons Bn = uk . La convergence de la série un montre que
k=n+1
lim Bn = 0. Mais la décroissance de la suite un montre aussi que
n→∞

0 ≤ 2nu2n ≤ 2Bn , et 0 ≤ (2n + 1)u2n+1 ≤ 2nu2n + u2n ≤ 2Bn + u2n .

Alors lim nun = 0.


n→∞
Notons que l’hypothèse de décroissance est essentielle comme le montre la série
P
convergente un de terme général un = 1/n si n est le carré d’un entier et un = 0
dans les autres cas.


X xk
Solution .5 Posons Sn = , d’après l’hypothèse lim Sn = 0.
ak n→∞
k=n
On a xn = (Sn − Sn+1 )an , et alors

n n n n
1 X 1 X 1 X 1 X
bn = xk = (Sk − Sk+1 ) ak = ak Sk − ak Sk+1
an an an an
k=1 k=1 k=1 k=1
n
X n+1
X n
X
1 1 1
= ak Sk − ak−1 Sk = Sk (ak − ak−1 ) − Sn+1
an an an
k=1 k=2 k=1

avec la convention a0 = 0.
Solutions 31

Soit M = sup{| Sk | : k ∈ IN∗ }. Alors pour 0 < N < n on a

N n
1 X 1 X
| bn | ≤ | Sn+1 | + | Sk | (ak − ak−1 ) + | Sk | (ak − ak−1 )
an an
k=1 k=N +1
N n
M X 1 X
≤ | Sn+1 | + (ak − ak−1 ) + sup | Sk | (ak − ak−1 )
an an k>N
k=1 k=N +1
M aN an − aN
≤ | Sn+1 | + + sup | Sk |
an an k>N
M aN
≤ | Sn+1 | + + sup | Sk |
an k>N
M aN
≤ + 2 sup | Sk |
an k>N

ε eε > Nε
Soit ε > 0, il existe un entier Nε tel que sup | Sk | ≤ . Il existe ensuite N
k>Nε 3
eε on ait M aN < ε . Alors pour tout n ≥ N
tel que pour tout n ≥ N eε on a | bn | < ε.
an 3
D’où lim bn = 0.
n→∞

Solution .6 On a
n−1
X 1 − (−x)n
(−x)k =
1+x
k=0

alors
Z 1 X n−1 Z 1 Z 1
f (x) xn f (x)
Rn = dx − (−1)k k
x f (x) dx = (−1) n
dx
0 1+x 0 0 1+x
k=0

Pour a ∈]0, 1[, on a

Z 1 Z 1
n
| Rn | ≤ a | f (x) | dx + | f (x) | dx,
0 a

Z 1
alors, pour tout a ∈]0, 1[ on a, lim | Rn | ≤ | f (x) | dx. D’où, en faisant tendre a
n→∞ a
vers 1, on trouve lim | Rn | = 0. Par conséquent
n→∞

Z 1 X ∞ Z 1
f (x)
dx = (−1)k xk f (x) dx.
0 1+x 0
k=0
32 SÉRIES NUMÉRIQUES

Solution .7 1◦ . Étude de la suite (vn ), avec

n
X
vn = Log k − (n + 1/2)Log n + n.
k=2

On a
1 1
vn − vn−1 =(n − )Log (1 − ) + 1
2 n
1 1 1 1 1
=(n − )(− − 2 + O( 3 )) + 1 = O( 2 )
2 n 2n n n
P
On conclut que la série (vn − vn−1 ) est convergente ce qui est équivalent à la
convergence de la suite (vn )n≥2 . Remarquons que ce résultat montre l’existance d’un
n!
λ ∈ IR∗+ tel que lim n −n √ = λ.
n→∞ n e n

2 . Étude de la suite (vn ), avec

n
X 1 n1−α
vn = − , α ∈]0, 1[
kα 1−α
k=1

On a µ ¶
1 1 1 1
vn − vn−1 = α + −
n (1 − α) (n − 1)α−1 (n)α−1
µ ¶
1 1 1 1−α
= α+ (1 − ) −1
n (1 − α)nα−1 n
µ ¶
1 1 1−α 1 1
= α+ α−1
− + O( 2 ) = O( 1+α )
n (1 − α)n n n n
P
On conclut que la série (vn − vn−1 ) est convergente ce qui est équivalent à la
convergence de la suite (vn )n≥2 .
3◦ . Étude de la suite (vn ), avec

n
X
vn = th k − Log ( ch n).
k=1

On a
X n
en
vn =Log ( )− (1 − th k)
ch n
k=1
n
X 2e−2k
2
=Log ( ) − ( )
1 + e−2n 1 + e−2k
k=1

X 2e−2k
Comme la série converge car son terme général est dominé par e−2k , alors
1 + e−2k
on conclut immédiatement que (vn )n≥1 est convergente.
Solutions 33

4◦ . Étude de la suite (vn ), avec


n
X 1
vn = √ − Argsh n.
k 2+1
k=1
p
1 n − 1 + (n − 1)2 + 1
vn − vn−1 = √ + Log √
n2 + 1 n + n2 + 1
à r ! à r !
1 1 2 2 1
=√ + Log 1 − + 1 − + 2 − Log 1 + 1 + 2
n2 + 1 n n n n
µ ¶ µ ¶
1 2 1 1
=√ + Log 2 − + O( 2 ) − Log 2 + O( 2 )
n2 + 1 n n n
µ ¶
1 1 1
=√ + Log 1 − + O( 2 )
2
n +1 n n
1 1 1 1
=√ − + O( 2 ) = O( 2 ).
n +1 n
2 n n
P
On conclut que la série (vn − vn−1 ) est convergente ce qui est équivalent à la
convergence de la suite (vn )n≥1 .
5◦ . Étude de la suite (vn ), avec
n
X Log k (Log n)2
vn = − .
k 2
k=1

Log n (Log n)2 (Log n − 1)2


vn − vn−1 = − +
n 2 2
Log n 1 1
= − Log (1 − )Log (n2 − n)
n 2 n
Log n 1 1 1
= + Log (1 − )Log n + (Log (1 − ))2
n n 2 n
Log n 1 1 1 Log n
= + Log (1 − )Log n + (Log (1 − ))2 = O( 2 ).
n P n 2 n n
On conclut que la série (vn − vn−1 ) est convergente ce qui est équivalent à la
convergence de la suite (vn )n≥1 .
6◦ . Étude de la suite (vn ), avec
n
X 1
vn = − Log Log n.
kLog k
k=2
1
vn − vn−1 = + Log Log (n − 1) − Log Log n
nLog n
1 Log (n − 1)
= + Log
nLog n Log n
µ ¶
1 Log (1 − 1/n)
= + Log 1 +
nLog n Log n
µ ¶
1 1 1 1
= + Log 1 − + O( 2 ) = O( 2 ).
nLog n nLog n n Log n n Log n
34 SÉRIES NUMÉRIQUES
P
On conclut que la série (vn − vn−1 ) est convergente ce qui est équivalent à la
convergence de la suite (vn )n≥1 .

(f − g)(x)
Solution .8 1◦ . Notons que, d’après l’hypothèse, on a lim = 1, donc il
>
x→0
x(b − b0 )
existe x0 > 0 tel que
(f − g)(x)
x ∈]0, x0 [ =⇒ >0
x(b − b0 )
alors, dans l’intervalle ]0, x0 [ la fonction x 7→ f (x) − g(x) a le signe de b − b0 .
2◦ . Clairement,
µ ¶−λ
vn+1 1 λ 1
= 1+ = 1 − + O( 2 ).
vn n n n

3◦ . Supposons Lµ < 1. On choisit


¶ β et λ tels que L < β < λ < 1.
un
Comme lim n − 1 < β, alors il existe n0 tel que
n→∞ un+1
µ ¶
un
n ≥ n0 =⇒ n −1 ≤β
un+1

ce qui montre que pour n ≥ n0 on a


µ ¶−1
un+1 β β 1
≥ 1+ =1− + O( 2 ).
un n n n

Si alors vn = n−λ on a d’après 2◦ ,


un+1 vn+1 λ−β 1
∀ n ≥ n0 , − ≥ + O( 2 )
un vn n n
Ce qui démontre l’existence de n1 ≥ n0 tel que
un+1 vn+1
∀ n ≥ n1 , − ≥0
un vn
µ ¶
un
La suite est croissante à partir de n1 . On conclut que
vn n≥1

∀ n ≥ n1 , u n ≥ M vn
un1 P P
où M = . Mais vn diverge (car λ < 1) donc un diverge aussi.
vn1
D’autre part, siµl > 1. On ¶choisit β et λ tels que l > β > λ > 1.
un
Comme lim n − 1 > β, alors il existe n0 tel que
n→∞ un+1
µ ¶
un
n ≥ n0 =⇒ n −1 ≥β
un+1
Solutions 35

ceci montre que pour n ≥ n0 on a


µ ¶−1
un+1 β β 1
≤ 1+ =1− + O( 2 ).
un n n n

Si alors vn = n−λ on a d’après 2◦ .


un+1 vn+1 λ−β 1
∀ n ≥ n0 , − ≤ + O( 2 )
un vn n n
Ce qui démontre l’existence de n1 ≥ n0 tel que
un+1 vn+1
∀ n ≥ n1 , − ≤0
un vn
µ ¶
un
Ceci prouve que la suite est décroissante à partir de n1 . On conclut que
vn n≥1

∀ n ≥ n1 , u n ≤ M vn
un1 P P
où M = . Mais vn converge (car λ > 1) donc un converge aussi.
vn1
1 P
Enfin, si un = on a l = L = 1 et la série un converge si β > 1 et
n(Log n)β
diverge si β ≤ 1.

Solution .9 1◦ . On pose
1
u−1 = 1, un =
(1 + a0 ) · · · (1 + an )

alors pour tout entier n ≥ 0 on a vn = un−1 − un . On en déduit que


n
X
∀ n ∈ IN, vk = 1 − un ≤ 1.
k=0
P
Il en résulte que vn est convergente.

X

2 . Il est clair d’après ce qui précède que vn = 1 est équivalent au fait que
n=0
lim un = 0. Mais
n→∞
à n
! n n
X Y 1 X
exp ak ≥ (1 + ak ) = ≥ ak .
un
k=0 k=0 k=0

On conclut que
X
lim un = 0 ⇐⇒ an diverge.
n→∞
36 SÉRIES NUMÉRIQUES

X

Solution .10 1 . Notons Sn = uk , on a clairement
k=n

n
X n
X n
X n+1
X n
X
kuk = k(Sk − Sk+1 ) = kSk − (k − 1)Sk = −nSn+1 + Sk .
k=1 k=1 k=1 k=1 k=1

D’où Ã !
n n
1X 1 X
Vn = kuk = Sk − Sn+1
n n
k=1 k=1

Soit ε > 0, alors il existe nε tel que, pour tout k ≥ nε , on a | Sk | < ε/3. Il en résulte
que, pour tout n ≥ nε ,
Ãn ! n
ε 1 Xε
n − nε ε 2ε 1Xε
2ε nε S1
| Vn | ≤ + Sk + ≤ + S1 ≤ +
3 n n 3 3 n 3 n
k=1 k=1

nε S1 ε
On en déduit qu’il existe n
eε ≥ nε tel que < , d’où
n
eε 3
∀n ≥ n
eε , | Vn | < ε

et lim Vn = 0.
n→∞
Remarquons que

X n
1 1 1
Vn = ( − ) kuk = Vn − Vn+1 + un+1 .
n+1 n n+1
k=1

D’où Ãm+1 !
m
X Xm X
1
Vn = V1 − Vm+1 + un+1 = un − Vm+1 .
n=1
n+1 n=1 n=1
m
X
P 1
Comme uk est convergente, on déduit de ce qui précède que Vn est
n=1
n+1
convergente et que
X∞ X∞
u1 + 2u2 + · · · + nun
= un .
n=1
n(n + 1) n=1

2◦ . En utilisant l’inégalité connue entre la moyenne arithmétique et la moyenne


géométrique, on déduit que
p
n u1 + 2u2 + · · · + nun
n!u1 u2 . . . un ≤
n
D’où √

X X∞
n
n!u1 u2 . . . un
≤ un .
n=1
n+1 n=1
Solutions 37

3◦ . C’est evident, car pour tout n ≥ n0 , on a


u1 + u2 + · · · + un un
≥ 0.
n n
4◦ . En étudiant la fonction x 7→ x − Log (1 + x) on démontre que
µ ¶n
1
∀ n ≥ 1, 1+ <e
n

ce qui montre que


n µ
Y ¶k
(n + 1)n 1
= 1+ < en .
n! k
k=1

n
n! 1
D’où > . En revenant au résultat de 2◦ on trouve
n+1 e

X ∞
X

n
u1 u2 . . . un ≤ e un .
n=1 n=1

Remarque: Notons λ0 la borne inférieure de l’ensemble des réels positifs M qui


P
vérifient, pour toute série à termes positifs convergente un ,

X ∞
X

n
u1 u2 . . . un ≤ M un . (∗)
n=1 n=1

On a démontré que λ0 ≤ e. On peut, en fait, démontrer que λ0 = e. Soit, en effet,


M > 0 vérifiant (∗), et N un entier provisoirement fixé. Considérons la suite (un )n≥1
définie par un = 1/n si n ≤ N et un = 0 si n > N . On a

XN XN
1 1
√ ≤M .
k
k! k
k=1 k=1

Mais, en étudiant la fonction x 7→ x − Log (1 + x) on démontre que


µ ¶−n
1
∀ n ≥ 1, 1− >e
n

ce qui montre que


(n + 1)n+1 Yµ
n+1
1
¶−k
= 1− > en .
n! n
k=2

1 e
D’où √
n
> √ . Alors
n! (n + 1) n n + 1
N
X +1 N
X
1 1
e ≤M ,
k=2
k.k 1/(k−1) k
k=1
38 SÉRIES NUMÉRIQUES

N
X N
X
1
ce qui s’écrit: eSN − RN ≤ M SN avec SN = , et RN = 1 + rk avec
k
k=1 k=2
1 Log k Log k
rk = (1 − exp(− )). Mais rk = O( 2 ) donc (RN )N ≥1 converge vers une limite
k k−1 k
RN
finie R, et de l’autre côté lim SN = +∞. En combinant ceci avec e − ≤ M on
N →∞ SN
trouve que e ≤ M . Ce qui démontre que λ0 = e.

Solution .11 Remarquons que, pour n ∈ IN∗ , et en utilisant l’inégalité de Schwarz

n n
à n
!1/2 Ã n
!1/2
n(n + 1) X X√ k X X k2
= k= uk √ ≤ uk
2 uk uk
k=1 k=1 k=1 k=1

n
1 X k2
Posons alors Bn = . On a
n2 uk
k=1

Xn
n 4n k2
n ≤ 2
X n (n + 1)2 uk
uk k=1

k=1
n
4n 2n + 1 X k 2
=
2n + 1 n2 (n + 1)2 uk
k=1
µ ¶X n
4n 1 1 k2
= −
2n + 1 n2 (n + 1)2 uk
k=1
µ ¶
4n 1
= Bn − Bn+1 +
2n + 1 un+1
µ ¶
1
≤2 Bn − Bn+1 + .
un+1

En prenant la somme des inégalités précédentes pour n variant entre 1 et m on trouve


m
X m+1
X 1
n
≤2
u + u2 + · · · + un
n=1 1
u
n=1 n

ce qui démontre le résultat en faisant tendre m vers l’infini.


Remarque: Notons λ−1 la borne inférieure de l’ensemble des réels positifs M qui
P
vérifient, pour toute série à termes positifs convergente un ,

X X∞
n 1
≤M . (∗)
u + u2 + · · · + un
n=1 1
u
n=1 n

On a démontré que λ−1 ≤ 2. On peut, en fait, démontrer que λ−1 = 2. Soit, en


effet, M > 0 vérifiant (∗), et N un entier provisoirement fixé. Considérons la suite
Solutions 39

(un )n≥1 définie par un = n si n ≤ N et un = 0 si n > N . On a

N
X X1 N
2
≤M ,
k+1 k
k=1 k=1

XN
1
ce qui s’écrit: 2SN − 1 ≤ M SN avec SN = , qui tend vers l’infini lorsque l’on fait
k
k=1
tendre N vers l’infini. on trouve alors que 2 ≤ M . Ce qui démontre que λ−1 = 2.
Xn
Solution .12 Notons A0 = 0 et An = ϕ(k) pour n ≥ 1. La remarque importante
k=1
n
X n(n + 1)
ici est que An ≥ k= . On peut alors écrire
2
k=1

Xm m m m−1
ϕ(n) X An − An−1 X An X An
2
= 2
= 2

n=1
n n=1
n n=1
n n=1
(n + 1)2
m−1
X 1
Am 1
= + ( − ) An
m2 n=1
n2 (n + 1)2

m−1
m+1 X 1 1 n(n + 1)
≥ + ( 2− 2
)
2m n=1
n (n + 1) 2
m−1
m+1 X 1 1
≥ + ( + )
2m n=1
2n 2(n + 1)
m−1
X 1 m−1
X m
X
1 1 1 1
= + + + = .
2 2m n=1 2n n=1 2(n + 1) n=1 n

X ϕ(n)
Il en résulte que diverge.
n2
n
X
Solution .13 Posons Sn = uk . Alors
k=1

(−1)n+1
Sn+1 = (1 + ) Sn
(n + 1)α

Comme S1 = u1 > 0 alors, par récurrence sur n, on démontre que ∀ n ≥ 1, Sn > 0.


On pose alors,
µ ¶
(−1)n+1 (−1)n+1
bn = Log Sn − Log Sn+1 + =ϕ
(n + 1)α (n + 1)α

avec ϕ(x) = x − Log (1 + x).


40 SÉRIES NUMÉRIQUES

1 P
Clairement, bn > 0 pour tout n ∈ IN∗ , et lim n2α bn = . Il en résulte que, bn
n→∞ 2
X (−1) n+1
converge si, et seulement si, α > 1/2. Mais, pour tout α ∈ IR∗+ , la série
(n + 1)α
n≥1
est convergente d’après la règle de convergence des séries alternées. On conclut que, si
P
α > 1/2 alors la série (Log Sn − Log Sn+1 ) converge, et si α ≤ 1/2 alors les sommes
P
partielles de la série (Log Sn − Log Sn+1 ) tendent vers l’infini, et donc (Sn )n≥1 tend
vers 0. Ceci démontre que (Sn )n≥1 converge pour tout α ∈ IR∗+ .

X
Solution .14 Posons Rn = uk . Alors on a Rn ≤ c(Rn−1 − Rn ) pour tout
k=n+1
n ≥ 1. On conclut que
c
∀ n ≥ 1, Rn ≤ Rn−1
1+c
ce qui montre µ ¶n
c
∀ n ≥ 0, Rn ≤ R0 .
1+c
Mais un ≤ Rn−1 , alors ∀ n ≥ 1, un ≤ an b, avec a = c/(c + 1) et b = R0 (1 + c)/c.

Solution .15 Remarquons simplement que nαn − (n − 1)αn−1 = an .


1◦ . Soit bn la différence entre les deux membres de l’inégalité demandée. Alors pour
tout n ≥ 1,
2
bn =((n − 1)αn−1 − nαn2 ) − (αn2 − 2αn an )
2
=(n − 1)αn−1 − (n + 1)αn2 + 2αn an
2
=(n − 1)αn−1 − (n + 1)αn2 + 2αn (nαn − (n − 1)αn−1 )
=(n − 1)(αn − αn−1 )2 ≥ 0

D’où le résultat.
2◦ . En effectuant la somme des inégalités précédentes pour n variant entre 1 et m
on trouve
m
X m
X
αn2 −2 2
αn an ≤ −mαm ≤ 0.
n=1 n=1

D’où, en utilisant l’inégalité de Schwarz, on trouve pour tout m ≥ 1,


m m
Ãm !1/2 Ã m !1/2
X X X X
αn2 ≤ 2 αn an ≤ αn2 a2n .
n=1 n=1 n=1 n=1

ce qui permet d’écrire


∞ µ
X ¶2 ∞
X
a1 + a2 + · · · an
≤4 a2n .
n=1
n n=1
Solutions 41

Remarque: Notons µ la borne inférieure de l’ensemble des réels positifs M qui


vérifient, pour toute suite à termes positifs (un )n≥1 ,
X∞ µ ¶2 X∞
a1 + a2 + · · · an
≤M a2n . (∗)
n=1
n n=1

On a démontré que µ ≤ 4. On peut, en fait, démontrer que µ = 4. Soit, en effet,


M > 0 vérifiant (∗), et N un entier provisoirement fixé. Considérons la suite (un )n≥1
√ √
définie par un = n − n − 1 si n ≤ N et un = 0 si n > N . On a
N
X N
X
1
≤M u2k .
k
k=1 k=1

XN
1 M 1
Mais, pour n > 1, un ≤ √ ce qui montre: SN ≤ (1 + SN ) avec SN = ,
2 n−1 4 k
k=1
qui tend vers l’infini lorsque l’on fait tendre N vers l’infini, on trouve alors que 4 ≤ M .
Ce qui démontre que µ = 4.
1
Solution .16 On a pour tout n ≥ 1, (1 − )Sn ≤ Sn−1 . Ce qui s’écrit
n2
Sn n Sn−1
∀ n > 1, ≤ .
n n+1n−1
Ceci permet de démontrer que
Sn 2
∀ n > 1, ≤ S1 .
n n+1
P
On conclut que ∀ n > 1, Sn ≤ 2S1 , et la série an converge.

Solution .17 Notons d’abord que


1 1 1 1
an = − + O( 3 ) = O( 2 ).
n n+a n n
P
La série an est par conséquent absolument convergente.
D’autre part,

X
f (m + 1) − f (m) = (Arctg (n + m + 1) − Arctg (n + m))
n=0
X∞
= (Arctg (n + 1) − Arctg n)
n=m
k
X
= lim (Arctg (n + 1) − Arctg n)
k→∞
n=m
= lim Arctg (k + 1) − Arctg m
k→∞
π 1
= − Arctg m = Arctg
2 m
42 SÉRIES NUMÉRIQUES

on en déduit
m−1
X
π 1
∀ m ≥ 1, f (m) = + Arctg .
2 k
k=1

En utilisant le fait que


m−1
X 1
∀ m ≥ 1, Log m = Log (1 + )
k
k=1

on trouve

π Xµ
m−1
1 1

∀ m ≥ 1, f (m) − Log m = + Arctg − Log (1 + ) .
2 k k
k=1

1 1 1
On pose uk = Arctg − Log (1 + ). Clairement, uk = O( 2 ) donc
k k k
∞ µ ¶
π X 1 1
lim (f (m) − Log m) = λ = + Arctg − Log (1 + ) . (†)
m→∞ 2 k k
k=1

y−x
Mais 0 < x < y =⇒ Arctg y − Arctg x ≤ , donc pour tout (x, y) ∈ IR∗+ , avec
x2
0<x<y

X 1 1
0 ≤ f (y) − f (x) ≤ Arctg − Arctg
n=0
n+x n+y
X∞
y−x 1
≤ 2 + (y − x)
x n=1
(n + x)2
à ∞
!
1 X 1 1
≤(y − x) + ( − )
x2 n=1 n − 1 + x n + x
µ ¶
1 1
≤(y − x) +
x2 x

On conclut que

1 + E(x)
∀ x ∈ [1, +∞[, 0 ≤ f (x) − f (E(x)) ≤
(E(x))2

On en déduit que lim [f (x) − f (E(x))] = 0. Or on a aussi lim [Log x − Log E(x)] = 0.
x→∞ x→∞
En combinant ceci avec (†) on trouve
∞ µ ¶
π X 1 1
lim f (x) − Log x = + Arctg − Log (1 + ) .
x→∞ 2 k k
k=1

Une valeur approchée de λ est 1.872436647263 à 10−12 près.


Solutions 43

Solution .18 D’après l’hypothèse, il est clair que la suite (nα un )n≥1 est décroissante,
donc
u1
∀ n ≥ 1, un ≤ α .
n
P −α P
Mais n converge (α > 1) alors un converge.
1.3.5 . . . (2n − 1) 1
Si un = . Alors
2.4.6 . . . (2n) 2n + 1

un+1 (2n + 1)2


=
un (2n + 3)(2n + 2)

Mais si n ≥ 14 on a
µ ¶4/3
(2n + 1)2 4 n
≤1− < .
(2n + 3)(2n + 2) 3n n+1
P
Alors un converge.

Solution .19 1◦ . Notons que


µ ¶−n
vn+1 2(2n + 1) 1
= 1+ .
vn a(n + 1) n

vn+1 4
D’où lim = . Il suffit de prendre λ = 4/e.
n→∞ vn ae

2 . On a
vn+1 1 1 1
Log = 1 − nLog (1 + ) + Log (1 + ) − Log (1 + )
vn n 2n n
vn+1 1 P
Donc Log = O( 2 ), et la série (Log vn+1 − Log vn ) converge, ce qui entraı̂ne
vn n P
la convergence de (Log vn )n≥1 vers un réel µ. Alors lim vn = eµ 6= 0, et la série vn
n→∞
diverge dans ce cas.

Solution .20 Pour simplifier on pose ϕ(x) = (ex − 1)/x, c’est une fonction
prolongeable en une fonction de classe C ∞ sur IR, avec ϕ(0) = 1, de plus ϕ(x) > 0
pour tout x ∈ IR.
Soient (α, β) ∈ IR2 , avec α < µ < β, alors d’après l’hypothèse il existe xαβ > 0 tel
que
f 0 (x)
∀ x ≥ xαβ , α≤ ≤ β.
f (x)
En intégrant entre p et x on trouve

f (x)
∀ (x, p) ∈ IR2 , x ≥ p ≥ xαβ , =⇒ α(x − p) ≤ Log ≤ β(x − p),
f (p)
44 SÉRIES NUMÉRIQUES

ou bien,

∀ (x, p) ∈ IR2 , x ≥ p ≥ xαβ , =⇒ eα(x−p) f (p) ≤ f (x) ≤ eβ(x−p) f (p).

En intégrant par rapport à x entre p et p + 1 on trouve


Z p+1
∀ p ∈ IN, p ≥ xαβ , =⇒ ϕ(α).f (p) ≤ f (x) dx ≤ ϕ(β) f (p). (†)
p

1◦ . Soit ε > 0, la continuité de ϕ en µ, montre l’existence de (α, β) ∈ IR2 tels que


α < µ < β et
ϕ(α) ≥ ϕ(µ) − ε et ϕ(β) ≤ ϕ(µ) + ε.

Pour ces valeurs de α et β on trouve Nε = 1 + E(xαβ ) tel que, d’après (†),


n
X Z ∞
∀ n ≥ Nε , ϕ(α) f (p) ≤ f (x) dx.
p=Nε 0

P
donc f (p) converge. En revenant à (†) on trouve
X∞ Z ∞ ∞
X
∀ n ≥ Nε , (ϕ(µ) − ε) f (p) ≤ f (x) dx ≤ (ϕ(µ) + ε) f (p)
p=n n p=n

Ce qui démontre que



X Z ∞
1
f (p) ∼ f (t) dt.
p=n
ϕ(µ) n

2◦ . Soit ε > 0, la continuité de ϕ en µ, montre l’existence de (α, β) ∈ IR2 tels que


α < µ < β et
ϕ(α) ≥ ϕ(µ) − ε et ϕ(β) ≤ ϕ(µ) + ε.

Pour ces valeurs de α et β on trouve Nε = 1 + E(xαβ ) tel que, d’après (†),


Z n+1 n
X
∀ n ≥ Nε , f (x) dx ≤ ϕ(β) f (p)
Nε p=Nε
P
donc f (p) diverge. En revenant à (†) on trouve
Xn Z n+1 n
X
∀ n ≥ Nε , (ϕ(µ) − ε) f (p) ≤ f (x) dx ≤ (ϕ(µ) + ε) f (p)
p=Nε Nε p=Nε


X Z Nε n
X
Si l’on pose Aε = f (p), Bε = f (t) dt et Sn = f (p), alors on trouve, pour
k=0 0 p=0
tout n ≥ Nε ,
Z n
Aε 1 Bε
(ϕ(µ) − ε)(1 − )≤ f (t) dt − ≤ (ϕ(µ) + ε).
Sn Sn 0 Sn
Solutions 45

Alors Z n Z n
f (t) dt f (t) dt
0 0
ϕ(µ) − ε ≤ lim n ≤ lim n ≤ ϕ(µ) + ε
n→∞ X n→∞ X
f (p) f (p)
p=0 p=0

D’où le résultat car ε est arbitraire.


Le cas µ = 0 est considéré dans la démonstration précédente.

OKMRAN
OUBA
INTÉGRALES GÉNÉRALISÉES

Dans ce chapitre IK désigne IR ou C


|

I. Généralités

Définition : Soient (a, b) ∈ IR × IR avec a < b et f : [a, b[−→ IK une application


Z b
continue. On dit que l’intégrale généralisée f (x) dx converge (ou existe) si, et
a
seulement si, la fonction
Z x
F : [a, b[−→ IK, x 7→ f (t) dt
a

admet une limite finie lorsque x tend vers b. Dans ce cas on écrit:
Z b Z x
f (x) dx = lim f (t) dt.
<
a x→b a

Z b
Si, d’autre part, l’intégrale généralisée f (x) dx ne converge pas, on dit qu’elle diverge.
a

Bien sûr, il y a une définition analogue pour ]a, b] (au lieu de [a, b[), (a, b) ∈ IR × IR
avec a < b.
Remarquons que, si (a, b) ∈ IR×IR avec a < b, et si f : [a, b[−→ IK et g : [a, b[−→ IK
Z b
sont deux applications continues telles que les intégrales généralisées f (x) dx et
Z b Z b a

g(x) dx convergent alors, pour tout λ ∈ IK, l’intégrale généralisée (λf + g)(x) dx
a a
converge et
Z b Z b Z b
(λf + g)(x) dx = λ f (x) dx + g(x) dx.
a a a

Exemples :
1
♠ Considérons, pour α ∈ IR, la fonction fα : [1, +∞[−→ IR, x 7→ α . L’intégrale
Z ∞ x
généralisée fα (x) dx converge si, et seulement si, α > 1. En effet,
1
 µ ¶
Z 
 1 1
x −1 si α 6= 1
Fα (x) = fα (t) dt = 1−α xα−1
1 

Log x si α = 1
2 Intégrales généralisées

D’où lim Fα (x) existe si, et seulement si, α − 1 > 0.


x→∞
1
♠ Considérons, pour α ∈ IR, la fonction gα :]0, 1] −→ IR, x 7→ α . L’intégrale
Z 1 x
généralisée gα (x) dx converge si, et seulement si, α < 1. En effet,
0

 µ ¶
Z 
 1 1
1 1− si α 6= 1
Gα (x) = gα (t) dt = 1−α xα−1
x 

−Log x si α=1

D’où lim Gα (x) existe si, et seulement si, α − 1 < 0.


>
x→0
Z 1
♠ L’intégrale généralisée Log x dx converge et vaut −1. En effet, pour x ∈]0, 1],
0

Z 1
F (x) = Log t dt = [xLog x − 1]1x = −1 − xLog x + x.
x

Donc lim F (x) = −1.


>
x→0

Proposition I.1. Soient (a, b) ∈ IR × IR avec a < b, et f : [a, b[−→ IK une application
Z b
continue et positive. L’intégrale généralisée f (x) dx converge si, et seulement si, il
a
existe une constante M telle que

Z x
∀ x ∈ [a, b[, f (t) dt ≤ M.
a

Z x
Preuve : C’est immédiat car la fonction x 7→ f (t) dt est croissante.
a

Proposition I.2. Soient (a, b) ∈ IR × IR avec a < b, et f : [a, b[−→ IK et g : [a, b[−→ IK
deux applications continues telles que 0 ≤ f ≤ g.
Z b Z b
– Si l’intégrale généralisée g(x) dx converge, alors f (x) dx converge.
a a
Z b Z b
– Si l’intégrale généralisée f (x) dx diverge, alors g(x) dx diverge.
a a

Preuve : C’est un corollaire immédiat de la proposition précédente.


Généralités 3

1
Exemple : Soit, pour (α, β) ∈ IR2 , la fonction hα,β : [e, +∞[−→ IR, x 7→ α .
Z ∞ x (Log x)β
L’intégrale hα,β (x) dx converge si, et seulement si, α > 1 ou (α = 1 et β > 1). En
1
effet, distinguons plusieurs cas:
¦ Si α > 1, on choisit γ ∈]1, α[. Alors lim xγ hα,β (x) = 0, et par conséquent il existe
x→∞
une constante c ≥ e telle que
1
∀ x ≥ c,
hα,β (x) ≤ γ .
x
Z ∞ Z ∞
dx
Mais converge car γ > 1, donc hα,β (x) dx converge. Il en résulte que,
c xγ Z c

si α > 1, alors hα,β (x) dx converge.
e
¦ Si α < 1, on choisit γ ∈]α, 1[. Alors lim xγ hα,β (x) = +∞, et par conséquent il
x→∞
existe une constante c ≥ e telle que
1
∀ x ≥ c, hα,β (x) ≥ .

Z ∞ Z ∞
dx
Mais diverge car γ < 1, donc hα,β (x) dx diverge. Il en résulte que, si
c xγZ c

α < 1, alors hα,β (x) dx diverge.
e
¦ Si α = 1, alors pour tout x ≥ e,
Z x Z Log x
du
Fβ (x) = h1,β (x) dx = .
e 1 uβ
Donc lim Fβ (x) existe si, et seulement si, β > 1.
x→∞

Un moyen très utile pour montrer la convergence d’une intégrale généralisée est le
critère de Cauchy qui s’énonce comme suit:
“Soit (a, b) ∈ IR × IR avec a < b, et f : [a, b[−→ IK une application continue. Alors,
Z b
l’intégrale généralisée f (x) dx converge si, et seulement si,
a
½ ¾ ¯Z v ¯
(u, v) ∈ IR2 ¯ ¯
(∀ ε > 0) (∃ c ∈ [a, b[) : =⇒ ¯¯ f (x) dx ¯¯ ≤ ε.”
c≤u<v<b u

Définition : Soient (a, b) ∈ IR × IR avec a < b et f : [a, b[−→ IK une application con-
Z b
tinue. On dit que l’intégrale généralisée f (x) dx est absolument convergente
Z b a

si, et seulement si, | f (x) | dx est convergente. Et dans ce cas


a
¯Z ¯ Z
¯ b ¯ b
¯ ¯
¯ f (x) dx ¯ ≤ | f (x) | dx.
¯ a ¯ a
4 Intégrales généralisées

Le critère de Cauchy permet de démontrer que toute intégrale généralisée absolu-


ment convergente est convregente. La réciproque est évidemment fausse comme nous le
verrons dans les exemples.

Proposition I.3. Soient (a, b) ∈ IR×IR avec a < b, et f : [a, b[−→ IK et g : [a, b[−→ IR+
Z b
deux applications continues telles que f ∼ g. Alors les intégrales généralisées f (x) dx
b− a
Z b
et g(x) dx sont de même nature.
a

Nous allons démontrer cette proposition lorsque IK = C,


| le cas IK = IR étant plus
facile.
Z b
Preuve : En effet, supposons g(x) dx convergente. Il existe une constante c ∈ [a, b[
a
telle que, pour tout x ∈ [c, b[, on a | f (x) − g(x) | ≤ g(x), en particulier, pour tout
x ∈ [c, b[, on a | f (x) | ≤ 2g(x). Il suffit alors d’utiliser la proposition précédente pour
Z b
voir que f (x) dx est absolument convergente, et donc convergente.
a Z b
Inversement, si g(x) dx est divergente, il existe une constante c ∈ [a, b[ telle que,
a
1
pour tout x ∈ [c, b[, on a | f (x) − g(x) | ≤ g(x), en particulier, pour tout x ∈ [c, b[, on
2
1 1
a Ref (x) − g(x) ≥ − g(x), ou bien Ref (x) ≥ g(x). Donc, en utilisant la proposition
Z 2 2 Z
b b
précédente, Ref (x) dx diverge et par conséquent, f (x) dx diverge.
a a

Définition : Soient (a, b) ∈ IR × IR avec a < b et f : [a, b[−→ IK une application


Z b
continue. On dit que l’intégrale généralisée f (x) dx est semi-convergente si,
a
et seulement si, elle est convergente mais non absolument convergente.
Z ∞
sin t
Exemple : Étudions l’intégrale généralisée dt.
0 t
En effet, une intégration par parties montre que, pour 0 < x < y,
Z y ¯y Z y
sin t cos t ¯¯ cos t
dt = − ¯ − dt.
x t t x x t2

Donc ¯Z y ¯ Z y
¯ sin t ¯¯ 1 1 dt 2
¯ dt ¯ ≤ + + = .
¯ t x y 2
x x t x
Si ε > 0, alors ¯Z y ¯
2 ¯ sin t ¯¯
¯
≤ x < y =⇒ ¯ dt ¯ ≤ ε.
ε x t
Comparaison des convergences d’une série et d’une intégrale 5
Z ∞
sin t
Le critère de Cauchy montre alors la convergence de l’intégrale généralisée dt.
0 t
Montrons que l’intégrale précédente n’est pas absolument convergente. En effet, si
k ∈ IN∗ ,
Z 4πk ¯ ¯ X Z 2π(r+1) ¯¯ sin t ¯¯
2k−1
¯ sin t ¯
¯ ¯ ¯ ¯
¯ t ¯ dt = ¯ t ¯ dt
2πk r=k 2πr

X Z 2π ¯¯ sin(t + 2πr) ¯¯
2k−1
= ¯ ¯
¯ t + 2πr ¯ dt
r=k 0
X Z 2π | sin t |
2k−1
= dt
t + 2πr
r=k 0
µZ 2π ¶ 2k−1
X 1
≥ | sin t | dt
0 2π(r + 1)
r=k
µZ 2π ¶ Z 2π
k 1 1
≥ | sin t | dt = | sin t | dt = .
0 4πk 4π 0 π

On a donc démontré que


Z 4πk ¯ ¯
¯ sin t ¯ 1
∀ k ∈ IN , ∗ ¯ ¯
¯ t ¯ dt ≥ π ,
2πk
Z ∞ ¯ ¯
¯ sin t ¯
ce qui démontre que l’intégrale généralisée ¯ ¯
¯ t ¯ dt est divergente car le critère
0
de Cauchy n’est pas vérifié.
Terminons ce paragraphe par la définition suivante des intégrales doublement
généralisées.
Définition : Soient (a, b) ∈ IR × IR avec a < b et f :]a, b[−→ IK une application
Z b
continue. On dit que l’intégrale généralisée f (x) dx converge (ou existe) si, et
a Z c
seulement si, il existe une constante c ∈]a, b[ telle que les deux intégrales f (x) dx et
Z b a

f (x) dx convergent, et si c’est le cas


c

Z b Z c Z b
f (x) dx = f (x) dx + f (x) dx.
a a c

II. Comparaison des convergences d’une série et d’une intégrale


6 Intégrales généralisées

Th
Z ∞éorème II.1. Soit f : [a, +∞[−→ C une fonction continue. L’intégrale généralisée
|

f (x) dx converge si, et seulement si, pour toute suite (xn )n≥0 de [a, +∞[ tendant
X Z xn+1
a

vers +∞, la série f (t) dt converge.


n≥0 xn

Z x
Preuve : Notons, pour x ∈ [a, +∞[, F (x) = f (t) dt. On a
a

Z ∞
f (x) dx converge ⇐⇒ lim F (x) existe,
a x→∞

 ∀ (xn )n≥0 de [a, +∞[ tendant vers +∞
⇐⇒
 lim F (x ) existe,
n
n→∞



 ∀ (xn )n≥0 de [a, +∞[ tendant vers +∞

⇐⇒ X Z xn+1

 f (t) dt converge.
 x n≥0 n

La dernière équivalence vient du fait que


Z x0 X Z xk+1
n−1
F (xn ) = f (t) dt + f (t) dt.
a k=0 xk

Théorème II.2. Soit f : [a, +∞[−→ C


| une fonction continue. On suppose qu’il existe

une suite croissante (un )n≥0 de [a, +∞[ tendant vers +∞, telle que

XZ un+1
¦ la série f (t) dt converge,
n≥0 un
Z un+1
¦ lim | f (t) | dt = 0
n→∞ un Z ∞
Alors, l’intégrale généralisée f (x) dx converge.
a
Preuve : Pour x ≥ u0 , on pose p(x) = max{m : um ≤ x}. Alors, pour tout x ≥ u0 ,

up(x) ≤ x < up(x)+1 .

La fonction x 7→ p(x) est croissante, non majorée donc lim p(x) = +∞. Comme
x→∞

Z x Z u0 p(x)−1 Z uk+1
X Z x
f (t) dt = f (t) dt + f (t) dt + f (t) dt.
a a k=0 uk up(x)
Comparaison des convergences d’une série et d’une intégrale 7

alors
¯  ¯
¯Z x Z u0 p(x)−1 Z uk+1 ¯ Z up(x)+1
¯ X ¯
¯ f (t) dt −  f (t) dt + f (t) dt ¯¯ ≤
 | f (t) | dt.
¯
¯ a a k=0 uk ¯ u p(x)

d’où le résultat, en faisant tendre x vers +∞.

Corollaire II.3. Soit f : [a, +∞[−→ IR+ une fonction continue. On suppose qu’il
existe une suite croissante (un )n≥0 de [a, +∞[ tendantZvers +∞, telle que la série
X Z un+1 ∞
f (t) dt converge. Alors, l’intégrale généralisée f (x) dx converge.
n≥0 un a

∞ Z
sin t
Exemple : Étudions l’intégrale généralisée dt avec α ∈ IR. En effet,
¯ ¯ π tα Z ∞
¯ sin t ¯ 1 dt
¯ ¯
Si α > 1, on a ¯ α ¯ ≤ α , pour tout t ∈ [π, +∞[. Comme converge, alors
tZ t π tα

sin t
l’intégrale généralisée dt est absolument convergente pour α > 1.
π tα
Si α ≤ 1, on pose
Z (n+1)π Z π
sin t sin t
an = α
dt = (−1)n dt
nπ t 0 (t + nπ)α

– Si α = −β ≤ 0 alors, pour tout n ≥ 1,


Z π Z π
β β
| an | = (t + nπ) sin t dt ≥ (nπ) sin t dt ≥ 2.
0 0
Z ∞
P sin t
d’où, an diverge et par conséquent, l’intégrale généralisée dt est divergente
π tα
pour α ≤ 0, (d’après le théorème II.1).
– Si α ∈]0, 1], on pose
Z (n+1)π ¯ ¯ Z π
¯ sin t ¯ sin t
n
bn = (−1) an = ¯ ¯ dt = dt.
¯ tα ¯ α
nπ 0 (t + nπ)

La suite (bn )n est clairement décroissante et, pour tout n ≥ 1,


2 2
α
≤ bn ≤ .
((n + 1)π) (nπ)α
P
On en déduit que lim bn = 0 et que an est convergente, (par la règle de convergence
n→∞
des
Z ∞séries alternées). On conclut, en utilisant le théorème II.3 que l’intégrale généralisée
sin t
dt est convergente pour α ∈]0, 1]. Cette intégrale n’est pas absolument
π tα P
convergente car bn est divergente, (d’après le théorème II.1).
8 Intégrales généralisées

Théorème II.4. Soit f : [a, +∞[−→ IR+ une fonction continue et décroissante. On
pose
n
X Z a+n+1
xn = f (a + k) − f (t) dt.
k=0 a
n+1
X Z a+n+1
yn = f (a + k) − f (t) dt.
k=0 a

Alors, pour tout n, on a xn ≤ xn+1 ≤ yn+1 ≤ yn . En particulier


Z ∞ les deux suites
X
(xn )n ≥ 0 et (yn )n ≥ 0 sont convergentes, et f (n + a) et f (t) dt sont de même
a
nature.

Preuve : Il suffit de démontrer seulement l’inégalité demandée. En utilisant la


décroissance de f on a, pour tout n ≥ 0,
Z a+n+2
xn+1 − xn = (f (a + n + 1) − f (t)) dt ≥ 0
a+n+1
Z a+n+2
yn+1 − yn = (f (a + n + 2) − f (t)) dt ≤ 0
a+n+1

yn − xn = f (a + n + 1) ≥ 0.
La suite (xn )n est donc croissante, majorée, et par conséquent convergente. De même,
la suite (yn )n est decroissante, minorée, et donc convergente. Remarquons que si
lim f (x) = 0 alors les deux suites (xn )n ≥ 0 et (yn )n ≥ 0 sont adjacentes.
x→∞

III. Exemples d’application

♠ Soit (an )n∈IN une suite croissante de IR∗+ . Alors la série


X∞ µ ¶
1 an−1
√ 1−
n=1
a n an

est convergente.
En effet, c’est une série à termes positifs donc il suffit de majorer la suite des
1 1
sommes partielles. Pour t ∈ [ak−1 , ak ] on a, √ ≤ √ , d’où
ak ak t t
Z ak
ak − ak−1 dt
∀ k ≥ 1, √ ≤ √ .
ak ak ak−1 t t

D’où, en prenant la somme pour k entre 1 et n,


Xn Z an
ak − ak−1 dt 2 2 2
√ ≤ √ =√ −√ ≤√ ,
ak ak a0 t t a0 an a0
k=1
Exemples d’application 9

donc la série considérée converge.


1
♠ Soit
Z ∞ f : [1, +∞[−→ C de classe C . On suppose queZ l’intégrale
| généralisée

f (x) dx est convergente, et que l’intégrale généralisée f 0 (x) dx est absolu-
1 P 1
ment convergente.Z x Alors la série f (n) est convergente.
Posons F (x) = f (t) dt. D’après l’hypothèse F admet une limite L quand x tend
1
vers l’infini.
Une intégration par parties montre que
Z k+1 ¯k+1 Z k+1
¯
F (k + 1) − F (k) = f (t) dt = (t − k − 1)f (t) ¯¯ − (t − k − 1)f 0 (t) dt.
k k k

Ce qui donne
Z k+1
f (k) = F (k + 1) − F (k) − (k + 1 − t)f 0 (t) dt.
k

ou bien,
n
X n
X Z k+1
f (k) = F (n + 1) − bk , avec bk = (k + 1 − t)f 0 (t) dt.
k=1 k=1 k

Z k+1
P
Mais, lim F (n + 1) = L et, pour tout k ≥ 1, | bk | ≤ | f 0 (t) | dt donc bk
n→∞
Z ∞ k

converge car l’intégrale généralisée f 0 (x) dx est absolument convergente. Il en


P 1
résulte que f (n) est convergente.
Comme application on peut considérer

sin x
f : [1, +∞[−→ IR, x 7→ .
x

En effet,
Z √
X Z √X
sin t sin u
dt = 2 du.
1 t 1 u
Z ∞ √ √
0 cos x sin x
Alors, f (t) dt converge. D’autre part, f (x) = √ − , alors, pour
1 Z ∞ 2x x x2
3
tout x ≥ 1, | f 0 (x) | ≤ √ . Il en résulte f 0 (t) dt est absolument convergente.
2x x
X sin √n
1

On conclut que la série est convergente. Plus généralement la série


X sin nα n
est convergente pour tout α ∈]0, 1].
n
10 INTÉGRALES GÉNÉRALISÉES

EXERCICES

Exercice .1 Étudier la nature des intégrales suivantes :

Z ∞ Z ∞ √ √
xα 3
x+1− 3
x
I1 = √ dx I2 = √ dx
1 1 + ex 1 x
Z ∞ p
I3 = (x + 1 − x2 + 2x + a) dx
1

Z ∞ Z ∞
1 1
I4 = dx I5 = dx
e xa (Log x)b ee xa (Log x)b (Log Log x)c
Z ∞ Z ∞
1 1 1
I6 = sin( ) dx I7 = sin x sin( ) dx
1 x x 1 x
Z ∞ Z ∞
I8 = Log sin(1/x) dx I9 = Log cos(1/x) dx
1 1

Z 1 Z ∞
(Log x)2 (sin x)2
I10 = √ dx I11 = dx
0 x(1 − x) 0 xα
Exercice .2 Calculer les intégrales suivantes, après avoir justifié leur existence :

Z π/2 Z ∞
I1 = cos x Log (tg x) dx I2 = xn e−x dx
0 0
Z π Z ∞
x e−ax − e−bx
I3 = Log (2 sin ) cos nx dx I4 = dx
0 2 0 x
Z π/2 Z π/2
I5 = Log (sin x) dx I6 = Log (cos x) dx
0 0
Z 1 Z ∞ µZ ∞ ¶
Log x sin t
I7 = √ dx I8 = dt dx
0 x(1 − x)3/2 0 x t
Exercice .3 Soit f : IR+ −→ IR une fonction uniformément continue sur IR+ , telle
R
+∞
que f (x) dx converge. Prouver que lim f (x) = 0.
0 x→+∞
Exercice .4 Soit f :IR+ −→ IR continue et bornée. Montrer que:
Z ∞
nf (x) π
lim 2 2
dx = f (0).
n→∞ 0 1+n x 2
Exercices 11

g 0 (x) µ
Exercice .5 Soit g : [0, +∞[−→ IR∗+ , de classe C 1 , telle que ∼ au voisinage
g(x) x
de +∞ avec µ 6∈ {−1, 0}.
R
+∞
a. On suppose µ > −1. Montrer que g(x) dx diverge et que
0

Z x
xg(x)
g(t)dt ∼ au voisinage de +∞.
0 µ+1

R
+∞
b. On suppose µ < −1. Montrer que g(x) dx converge et que
0

Z +∞
xg(x)
g(t)dt ∼ − au voisinage de +∞.
x µ+1

Exercice .6 Discuter suivant la valeur de (a, b) ∈ IR2 la convergence de l’intégrale


Z +∞
Log (1 + xa )
I(a, b) = dx.
0 xb

Exercice .7 Convergence et calcul de l’intégrale


Z 1
dx

3
.
0 x2 − x3

Exercice .8 Soit (α, β) ∈ IR2 , avec αβ 6= 0. Étudier la convergence de l’intégrale


Z 1
β
I(α, β) = | 1 − xα | dx.
0

Exercice .9 Soit (α, β) ∈ IR2 . Étudier la convergence de l’intégrale


Z +∞

I(α, β) = dx.
1 1 + xβ sin2 x

Exercice .10
R
+∞ 2
a. Montrer que I = e−x dx converge.
0
π/2
R 2
sin2 x
b. Montrer que lim λ e−λ dx = I .
λ→∞ 0
c. Étudier suivant les valeurs de (α, β) ∈ (IR∗+ )2 la convergence de l’intégrale

Z
+∞
α 2
xβ e−x sin x dx
0
12 INTÉGRALES GÉNÉRALISÉES

R1
Exercice .11 Soit f :]0, 1] −→ IR une fonction décroissante telle que f (x) dx
0
converge. Déterminer lim xf (x).
>
x→0
Exercice .12 Soient (m, n) ∈ (IN∗ )2 , on suppose que m < n.
1◦ . Convergence et calcul de
Z +∞
x2m−1
Fn,m = dx.
0 1 + x2n
π
(On trouvera Fn,m = ).
2n sin(πm/n)
2◦ . En effectuant un changement de variable convenable montrer que, pour α ∈
Q
/ ∩ ]0, 1[
Z +∞
xα−1 π
I(α) = dx = .
0 1+x sin πα
Calculer ensuite I(α) pour α ∈ ]0, 1[.
3◦ . Pour q ∈ IN∗ ,(m, n) ∈ (IN∗ )2 , avec m < n, on pose
Z ∞
tm−1
J(n, m, q) = dt.
0 (1 + tn )q
Trouver une relation simple entre J(n, m, q+1) et J(n, m, q). En déduire J(n, m, q).
Z ∞
2
Exercice .13 On se propose dans cet exercice de calculer l’intégrale I = e−x dx.
0
Considérons les intégrales suivantes:
Z π/2 Z 1 Z ∞
n dx
Wn = sin x dx, In = (1 − x2 )n dx, Jn =
0 0 0 (1 + x2 )n

1 . Calculer (In )n≥1 et (Jn )n≥1 en fonction de (Wn )n≥1 .
2 2 1
2◦ . Montrer que ∀ x ∈ [0, 1], 1 − x2 ≤ e−x et que ∀ x ∈ [0, +∞], e−x ≤ .
1 + x2
En déduire que
I
In ≤ √ ≤ J n .
n
3◦ . Trouver une relation entre Wn+2 et Wn , et montrer que pour tout n ≥ 1 on a
nWn Wn−1 = π/2.

4 . En déduire la valeur de I.
Exercice .14
I
n
X
1◦ . Soit t ∈ IR et k ∈ IN∗ . Calculer e−iπkt . Montrer que
k=−n
Z 1/2 Z 1
sin(2n + 1)πt sin(2n + 1)πt 1
dt = dt = .
0 sin πt 1/2 sin πt 2
Exercices 13

2◦ . Soit g une fonction continue sur un intervalle [a, b] de IR. Montrer que
Z b
lim g(t) sin mt dt = 0
m→∞ a

(On pourrait commencer par démontrer le résultat lorsque g est en escalier.)


3◦ . Soit f : [0, 1] −→ C
| une fonction continue, dérivable à droite en 0 et à gauche en

1. On pose
n Z
X 1
Sn = f (t) e−2iπkt dt.
k=−n 0

Montrer que Z 1
sin(2n + 1)πt
Sn = dt. f (t)
0 sin πt
¸ ¸
1 f (t) − f (0)
En prolongeant par continuité les deux fonctions 0, −→ C,|
t 7→ et
· · 2 sin πt
1 f (t) − f (1)
, 1 −→ C,|
t 7→ , calculer la limite de la suite (Sn )n∈IN∗ .
2 sin πt
4◦ . Soient f : IR −→ C | une fonction dérivable et p ∈ IN∗ . Montrer que

à n Z
!
1 1 X p
f (0) + f (1) + · · · + f (p − 1) + f (p) = lim f (t) e−2iπkt dt .
2 2 n→∞ 0
k=−n

II

1 . Montrer la convergence des integrales
Z +∞ Z +∞ Z +∞
2
2 2
α= cos t dt, β= sin t dt, γp = e2iπpt dt.
0 0 0

Exprimer γp en fonction de α , β et p.
2◦ . Montrer que, pour k ∈ ZZ,
Z 1 Z k/2
2iπp(x2 −kx) pk2 2
e dx = (−i) e2iπpu du.
0 (k−2)/2

à n Z
!
X 1
2iπp(x2 −kx)
En déduire lim e dx en fonction de α , β et p.
n→∞ 0
k=−n
3◦ . En applicant le résultat du I.4◦ . à la fonction f : t 7→ exp(2iπt2 /p), calculer la
p−1 2iπk2
X
somme de Gauss Gp = e p pour tout p ≥ 1, et déterminer explicitement les
k=0
valeurs des intégrales de Fresnel α et β.
14 INTÉGRALES GÉNÉRALISÉES

SOLUTIONS


Solution .1 1◦ . Notons que √ x
≤ xα e−x/2 ≤ M e−x/4 où M = sup xα e−x/4
1+e x≥1
qui est fini, soit par un calcul direct, soit en utilisant le fait suivant:
“ soit f : [a, +∞[−→ IR une fonction continue ayant une limite finie en +∞. Alors
f est bornée.” Z ∞
On conclut que I1 converge car e−x/4 dx converge.
1
2◦ . En utilisant le théorème des accroissements finis on a

3
√ 1
0≤ x+1− 3
x≤ √
3
3 x2

D’où √ √
x+1− 3x
3
1
0≤ √ ≤ 7/6 .
x 3x
Z ∞
On conclut que I2 converge car x−7/6 dx converge.
p1
3◦ . Notons que lim x(x + 1 − x2 + 2x + a) = (a − 1)/2. On conclut donc que si
x→∞ Z ∞
dx
a 6= 1, l’intégrale I3 diverge car elle est de même nature que , et si a = 1, elle
1 x
converge trivialement.
4◦ .– Si a > 1, alors on considère α ∈]1, a[, et M = sup xα−a (Log x)−b qui est fini
x≥e
car la fonction à maximiser est continue sur [e, +∞[ et tend vers 0 à l’infini. On a

M
∀ x ≥ e, x−a (Log x)−b ≤ .

et l’intégrale I4 converge si a > 1.


– Si a < 1, alors on considère α ∈]a, 1[. On a lim xα−a (Log x)−b = +∞. Donc il existe
x→∞
x0 ≥ e tel que, pour tout x ≥ x0 , on a xα−a (Log x)−b ≥ 1. Alors

1
∀ x ≥ x0 , x−a (Log x)−b ≥

et l’integrale I4 diverge si a < 1.


– Si a = 1 on a Z A Z LogA
dx dt
= .
e x(Log x)b 1 tb
Donc, dans le cas a = 1, I4 converge si, et seulement si, b > 1.
Conclusion, I4 converge si, et seulement si, (a > 1) ou (a = 1 et b > 1).
Solutions 15

5◦ . – Si a > 1, on considère α ∈]1, a[, et M = sup xα−a (Log x)−b (Log (Log x))−c
x≥e
qui est fini car la fonction à maximiser est continue sur [ee , +∞[ et tend vers 0 à l’infini.
On a
M
∀ x ≥ ee , x−a (Log x)−b (Log (Log x))−c ≤

et l’intégrale I5 converge si a > 1.
– Si a < 1, alors on considère α ∈]a, 1[. On a lim xα−a (Log x)−b (Log (Log x))−c = +∞
x→∞
donc il existe x0 ≥ ee tel que, pour tout x ≥ x0 , on a xα−a (Log x)−b (Log (Log x))−c ≥ 1.
Alors
1
∀ x ≥ x0 , x−a (Log x)−b (Log (Log x))−c ≥

et l’intégrale I5 diverge si a < 1.
– Si a = 1 on a
Z A Z LogA
dx dt
= .
e x(Log x) (Log (Log x))−c
b
e tb (Log t)c

Donc, dans le cas a = 1, I5 converge si, et seulement si, (b > 1) ou (b = 1 et c > 1).
Conclusion, l’intégrale I5 converge si, et seulement si, (a > 1) ou (a = 1 et b > 1)
ou (a = b = 1 et c > 1).
6◦ . Pour tout x ≥ 1 on a
1 1 1
0≤ sin( ) ≤ 2
x x x
Z ∞
dx
D’où l’intégrale I6 converge car elle est de même nature que .
1 x2
7◦ . On a pour tout x ≥ 1,
¯ ¯
¯ ¯
¯ sin( 1 ) − 1 ¯ ≤ 1
¯ x x ¯ x3
Z ∞ µ ¶ Z ∞
1 1 sin x
alors sin x sin( ) − dx est absolument convergente. Mais dx est
1 x x 1 x
semi-convergente. On en déduit que l’intégrale I7 est semi-convergente.

Z 8◦ . Pour tout x ≥ 1, on a sin(1/x) ≤ 1/x, alors Log x ≤ −Log sin(1/x). Mais



Log x dx diverge alors I8 diverge.
1
9◦ . On a lim x2 Log cos(1/x) = −1/2, alors l’intégrale I9 converge car elle est de
x→∞Z

dx
même nature que .
1 x2
(Log x)2
10◦ . La fonction x 7→ √ est prolongeable par continuité en 1, l’intégrale
x(1 − x)
2
2/3 (Log x)
est généralisée seulement du côté de la borne inférieure. Mais, lim x √ = 0,
>
x→0 x(1 − x)
16 INTÉGRALES GÉNÉRALISÉES

alors il existe x0 ∈]0, 1[ tel que

(Log x)2 1
∀ x ∈]0, x0 [, 0≤ √ ≤ 2/3 .
x(1 − x) x
Z 1
dx
On conclut que I10 est convergente car l’est.
0 x2/3 Z ∞
(sin x)2 (sin x)2
11◦ . La fonction x 7→ est positive, alors l’intégrale dx est de
P xα π xα
même nature que an avec
Z (n+1)π Z π
(sin x)2 (sin x)2
an = dx = dx.
nπ xα 0 (x + nπ)α

Clairement,
π π
α
≤ an ≤ α .
(n + 1) n
Z ∞
(sin x)2
On conclut que dx converge si, et seulement si, α > 1.
π xα Z π
2
α−2 (sin x) (sin x)2
D’autre part, nous avons lim x . α
= 1, alors α
dx est de même
> x 1 x
Z π x→0
1
nature que α−2
dx. Elle converge, par conséquent, si α − 2 < 1. On conclut que
1 x
l’intégrale I11 converge si, et seulement si, α ∈]1, 3[.

Solution .2 1◦ . Cherchons une primitive F de la fonction f (x) = cos xLog (tg x)


sur ]0, π/2[. Une intégration par parties montre
Z
dx
F (x) = sin xLog (tg x) −
cos x
Z
cos x
= sin xLog (tg x) − dx
1 − sin2 x
Z µ ¶
cos x cos x
= sin xLog (tg x) − + dx
1 − sin x 1 + sin x
1 1 + sin x
= sin xLog (tg x) − Log
2 1 − sin x
1 + sin x
= sin xLog (tg x) − Log
cos x

On calcule alors lim F (x) = 0 et lim F (x) = −Log 2. Il en résulte que l’intégrale
> <
x→0 x → π/2
Z π/2
cos xLog (tg x) dx converge et vaut −Log 2.
0 Z ∞

2 . Une récurrence simple sur n montre que xn e−x dx converge et vaut n!.
0
Solutions 17

Log (2 sin(x/2)) cos nx


3◦ . Le problème se manifeste en 0. Or lim = 1, alors I3 est
> Log x
Z π x → 0

de même nature que Log x dx qui est convergente. Posons


0
Z π
x
Kn = Log (2 sin ) cos nx dx.
0 2

On suppose n 6= 0. Une intégration par parties montre que


· ¸π Z π
x sin nx 1 cos(x/2) sin(nx)
Kn = Log (2 sin ) − dx
2 n 0 2n 0 sin(x/2)
Z π
1 cos(x/2) sin(nx)
=− dx
2n 0 sin(x/2)

Alors, K1 = −π/2 et
Z
1 π cos(x/2)(sin(n + 1)x − sin nx)
(n + 1)Kn+1 − nKn = dx
2 0 sin(x/2)
Z π µ ¶
cos(x/2) 1 x
= cos(n + )x sin dx
0 sin(x/2) 2 2
Z π
x 1
= cos cos(n + )x dx
0 2 2
Z π
1
= (cos(n + 1)x + cos nx) dx = 0
2 0
π
Il en résulte que Kn = − pour tout n ≥ 1. En ce qui concerne K0 , voir le calcul de
2n
I5 .
4◦ . On peut supposer 0 < a < b. On a
Z µ Z aµ Z bµ
e−ax − e−bx e−t e−t
dx = dt − dt
ε x aε t bε t
Zbε −t Z bµ
e e−t
= dt − dt
aε t aµ t

D’où ¯Z ¯ Z b Z b −µt
¯ µ
¯ e−ax − e−bx b ¯¯ 1 − e−εt e
¯ dx − Log ¯ ≤ dt + dt
ε x a a t a t
ou bien, en utilisant 1 − eεt ≤ εt et eµt ≤ eµa pour t ≥ a,
¯ Z µ −ax ¯
¯ e − e−bx b ¯¯ b
¯ dx − Log ¯ ≤ (b − a)ε + e−µa Log .
¯ x a a
ε

Z ∞
e−ax − e−bx
Ce qui démontre que dx est convergente et vaut Log (b/a).
0 x
18 INTÉGRALES GÉNÉRALISÉES

5◦ et 6◦ . La convergence de I5 résulte de 3◦ . En effectuant le changement de variable


π
x 7→ − x, on montre que I6 converge et que I5 = I6 . Mais
2
Z π/2
sin 2t
2I5 = I5 + I6 = Log dt
0 2
ce qui montre que
Z π
1 π π
2I5 = Log (sin t) dt − Log 2 = I5 − Log 2.
2 0 2 2
π
Alors, I5 = I6 = − .
2 x Z
◦ Log t
7 . Posons F (x) = √ dt pour x ∈]0, 1[. On effectue le changement
t (1 − t)3/2
1/2

de variable Arcsin ( x) = u.
Z √
Arcsin x
du
F (x) = 4 Log (sin u) .
π/4 (cos u)2

Une intégration par parties montre alors


· ¸Arcsin√x Z √
Arcsin x
F (x) = 4Log (sin u) tg u −4 du.
π/4 π/4

On conclut que
r
x √
F (x) = 2 Log x − 4Arcsin x + 2Log 2 + π.
1−x
Enfin, lim F (x) = π + 2Log 2 et lim F (x) = −π + 2Log 2. Il en résulte que l’intégrale
> <
x→0 x→1
Z 1
Log x
√ dx est donc convergente et vaut −2π.
0 x(1 − x)3/2 Z ∞
sin t sin x

8 . Posons G(x) = dt. On a G0 (x) = − .
x t x
Z x · ¸x Z x
G(t) dt = tG(t) + sin t dt = xG(x) + 1 − cos x
0 0 0

Or, une intégration par parties montre


· ¸∞ Z ∞ Z ∞
cos t cos t cos x cos t
G(x) = − − 2
dt = − dt.
t x x t x x t2
Z ∞
cos t
Alors xG(x) − cos x = −x dt. Une deuxième intégration par parties montre
x t2
que
· ¸∞ Z ∞ Z ∞
sin t sin t sin x sin t
xG(x) − cos x = −x 2
− 2x 3
dt = − 2x dt
t x x t x x t3
Solutions 19

Alors Z ∞
1 dt 2
| xG(x) − cos x | ≤ + 2x = .
x x t3 x
On conclut Z x µZ ∞ ¶
sin u 1
du dt = 1 − O( )
0 t u x
Z ∞ µZ ∞ ¶
sin u
Il en résulte que du dt converge et vaut 1.
0 t u

Solution .3 Nous utilisons le lemme utile suivant ¯Z ¯


1 ¯¯ b ¯
¯
Lemme: “Soit f une fonction continue sur [a, b] telle que ¯ f (t) dt ¯ ≤ λ.
b−a¯ a ¯
Alors il existe t0 ∈ [a, b] tel que | f (t0 ) | ≤ λ.”
En effet, si pour tout t ∈ [a, b] on a | f (t) | > λ, alors f garde un sign constant sur
[a, b] (car elle ne s’y annule pas). D’où, il existe σ ∈ {−1, +1} tel que

∀ t ∈ [a, b], σf (t) > λ.


Z b
σ
Il en résulte que f (t) dt > λ, ce qui contredit l’hypothèse. D’où le lemme.
b−a a
Soit ε > 0, la continuité uniforme de f montre l’existence de η > 0 tel que
ε
∀ (x, y) ∈ (IR+ )2 , | x − y | ≤ η =⇒ | f (x) − f (y) | < .
2
Z ∞
et la convergence de l’intégrale f (t) dt montre l’existence de xε tel que
0
¯Z y ¯
¯ ¯
∀ (x, y) ∈ (IR+ )2 , y > x > xε =⇒ ¯¯ f (t) dt ¯¯ < εη.
x

Soit x > xε + η = x0 on a ¯Z ¯
1 ¯¯ x+η ¯ ε
¯<
f (t) dt
2η ¯ x−η ¯ 2

donc d’après le lemme il exite tx ∈ [x − η, x + η] tel que | f (tx ) | ≤ ε/2. Alors


ε ε
| f (x) | ≤ | f (x) − f (tx ) | + | f (tx ) | < + = ε.
2 2
On a alors démontré que

∀ ε > 0, ∃x0 > 0, tel que x > x0 =⇒ | f (x) | < ε,

c’est à dire lim f (x) = 0.


x→∞
Notons que l’hypothèse de continuité uniforme est essentielle comme le montre
l’exemple f (x) = sin x2 .
20 INTÉGRALES GÉNÉRALISÉES

Solution .4 Notons M = sup | f (x) |. On a


x≥0

Z x ¯ ¯ Z nx
¯ nf (t) ¯ dt Mπ
¯ ¯ dt ≤ M = M Arctg nx ≤ ,
¯ 1 + n2 t2 ¯ 1+t 2 2
0 0
Z ∞
nf (x)
donc dx converge absolument.
0 1 + n2 x2 Z

n π
Notons aussi que 2 2
dx = , d’où
0 1+n x 2
Z ∞ Z ∞ Z ∞
nf (x) π n f (x/n) − f (0)
2 2
dx − f (0) = 2 2
(f (x) − f (0)) dx = dx
0 1+n x 2 0 1+n x 0 1 + x2

alors
¯Z ∞ ¯ Z α Z ∞
¯ nf (x) π ¯ | f (x/n) − f (0) | dx
¯ ¯
dx − f (0) ¯ ≤ dx + 2M dx
¯ 1+n x2 2 2 1+x 2 1 + x2
0 0 α
π 1
≤ sup | f (x) − f (0) | + 2M Arctg
2 0≤x≤α/n α

Soit alors ε > 0, il existe αε > 0 tel que 2M Arctg (1/α) < ε/2 et il existe ηε > 0 tel que
ε
0 < x < η =⇒ | f (x) − f (0) | < .
π
Alors ¯Z ∞ ¯
αε ¯ nf (x) π ¯ ε ε
∀n > , ¯ dx − f (0) ¯ < + = ε.
ηε ¯ 1 + n2 x2 2 ¯ 2 2
0

On conclut que Z ∞
nf (x) π
lim 2 2
dx = f (0).
n→∞ 0 1+n x 2

Solution .5 1◦ . Supposons µ > −1, et soit λ ∈] − 1, µ[. Il existe x0 tel que

g 0 (t) λ
∀ t ≥ x0 , >
g(t) t

En intégrant entre x0 et x on trouve

g(x) > M xλ
∀ x ≥ x0 ,
Z ∞ Z ∞
−λ λ
avec M = g(x0 )x0 . Mais x dx diverge, donc g(x) dx diverge.
1 0
2◦ . Supposons µ < −1, et soit λ ∈]µ, −1[. Il existe x0 tel que

g 0 (t) λ
∀ t ≥ x0 , <
g(t) t
Solutions 21

En intégrant entre x0 et x on trouve

g(x) < M xλ
∀ x ≥ x0 ,
Z ∞ Z ∞
−λ λ
avec M = g(x0 )x0 . Mais x dx converge, donc g(x) dx converge.
1 0
On pose h(x) = xg 0 (x)/g(x) d’après l’hypothèse lim h(x) = µ. Une intégration
x→∞
par parties sur [a, b], montre que
Z b Z b
0
xg (x) dx = bg(b) − ag(a) − g(x) dx
a a

Z b
(1 + h(x))g(x) dx = bg(b) − ag(a) (1)
a

a. En prenant a = 0 dans (1) on trouve


Z x
∀ x > 0, (1 + h(t))g(t) dt = xg(x)
0
Z x
D’où, en posant G(x) = g(t) dt,
0
¯ Z x ¯ Z x
¯ ¯
∀ x > 0, ¯ xg(x) − (1 + µ) g(t) dt ¯¯ ≤ | h(t) − µ | g(t) dt
¯
0 0

Soit ε > 0, il existe x0 > 0 tel que, pour tout x ≥ x0 , on a | h(x) − µ | ≤ ε/2. Ce qui
montre
Z x0
ε
∀ x > x0 , | xg(x) − (1 + µ)G(x) | ≤ | h(t) − µ | g(t) dt + G(x)
0 2

ou bien
¯ ¯ Z x0
¯ xg(x) ¯ 1 ε
∀ x > x0 , ¯ ¯
¯ G(x) − (1 + µ) ¯ ≤ G(x) | h(t) − µ | g(t) dt +
2
0

Mais lim G(x) = ∞, alors il existe x1 > x0 tel que


x→∞
Z x0
1 ε
∀ x > x1 , | h(t) − µ | g(t) dt <
G(x) 0 2

soit ¯ ¯
¯ xg(x) ¯ ε ε
∀ x > x1 , ¯ − (1 + µ) ¯ < + = ε,
¯ G(x) ¯ 2 2

ce qui démontre que Z x


xg(x)
g(t) dt ∼
0 ∞ µ+1
22 INTÉGRALES GÉNÉRALISÉES

b. Supposons µ < −1, et notons M = sup{1 + h(x) : x ∈ IR+ }, on a d’après (1),


¯Z ¯
¯ b ¯
¯ ¯
| bg(b) − ag(a) | ≤ M ¯ g(x) dx ¯
¯ a ¯

La convegence de l’intégrale implique l’existence de lim xg(x) = `. Si ` 6= 0 on aboutit


Z ∞ x→∞
dx
à une contradiction car diverge. Alors lim xg(x) = 0. En revenant à (1), et en
1 x x→∞
faisant tendre b vers l’infini, on trouve
Z ∞
∀ x > 0, (1 + h(x))g(x) dx = −xg(x) (2)
x

Soit ε > 0, il existe x0 > 0 tel que

∀ x > x0 , 1 + µ − ε ≤ 1 + h(x) ≤ 1 + µ + ε.

D’où
Z ∞ Z ∞ Z ∞
∀ x > x0 , (1 + µ − ε) g(t) dt ≤ (1 + h(t))g(t) dt ≤ (1 + µ + ε) g(t) dt
x x x

soit ¯ Z ∞ ¯ Z ∞
¯ ¯
∀ x > x0 , ¯ xg(x) + (1 + µ) ¯
g(t) dt ¯ ≤ ε g(t) dt
¯
x x

ce qui démontre que Z ∞


xg(x)
g(t) dt ∼ −
x ∞ µ+1

Z x
Log (1 + ta )
Solution .6 Posons F (x) = dt. Si a = 0, alors lim F (x) existe si,
1 tb x→∞
et seulement si, b > 1. Et lim F (x) existe si, et seulement si, b < 1. On conclut que
>
x→0
I(0, b) diverge pour tout b ∈ IR.
Z a
1 x Log (1 + t)
Si a > 0, alors F (x) = dt. On en déduit que lim F (x) existe si,
a 1 t1+(b−1)/a x→∞
b−1 b−1
et seulement si, + 1 > 1. Et lim F (x) existe si, et seulement si, < 1. On
a >
x→0
a
conclut que pour a > 0, I(a, b) converge si, et seulement si, 1 < b < 1 + a.
Z a
1 x Log (1 + t)
Si a < 0, alors F (x) = dt. On en déduit que lim F (x) existe si,
a 1 t1+(b−1)/a x→∞
Z 1
Log (1 + t) b−1
et seulement si, 1+(b−1)/a
dt converge, c’est à dire < 1. Et lim F (x) existe
0 t a >
x→0
Z ∞
Log (1 + t) b−1
si, et seulement si, dt converge, c’est à dire > 0. On conclut que
1 t1+(b−1)/a a
pour a < 0, I(a, b) converge si, et seulement si, 1 + a < b < 1.
Solutions 23

Enfin, l’intégrale I(a, b) définie par


b

Z ∞
Log (1 + xa )
I(a, b) = dx
0 xb
1
converge si, et seulement si a

I(a,b) converge
(1 < b < 1 + a) ou (1 + a < b < 1).
I(a,b) diverge

Z 1
dx
Solution .7 L’intégrale I = est clairement convergente. On
0 − x)1/3 x2/3 (1
effectue le changement de variable x = 1/(1 + t3 ), on obtient
Z ∞ Z ∞
3t 3
I= 3
dt = dt.
0 t +1 0 t3 +1

La deuxième égalité vient après le changement de variable t 7→ 1/t dans la première


intégrale. D’où
Z ∞ Z ∞
3 t+1 3 dt
I= 3
dt = .
2 0 t +1 2 0 t2 −t+1
2π 2t − 1
Enfin, on obtient I = √ , en effectuant le changement de variable θ = Arctg √ .
3 3

Solution .8 Si α > 0, on a
Z µ Z µα
α β 1
(1 − x ) dx = (1 − t)β t−1+1/α dt
ε α εα

R1
Donc, pour α > 0, l’intégrale I(α, β) converge si, et seulement si, 0
(1 − t)β t−1+1/α dt,
converge c’est à dire (β > −1) et (α < 1).
Si α < 0, on a
Z µ Z µα
α β 1
(1 − x ) dx = (1 − t)β t−1+1/α dt
ε α εα

R∞
Donc, pour α < 0, l’intégrale I(α, β) converge si, et seulement si, 1
(t − 1)β t−1+1/α dt
converge, c’est à dire (β > −1) et (βα > −1).
24 INTÉGRALES GÉNÉRALISÉES

Solution .9 La fonction considérée est positive. L’intégrale I(α, β) est de même


P
nature que la série an avec
Z (n+1)π Z π
xα (x + nπ)α
an = dx = dx
nπ 1 + xβ sin2 x β
0 1 + (x + nπ) sin x
2

Posons λ+ α α α − α α α + β β
n = π max(n , (n + 1) ), λn = π min(n , (n + 1) ) µn = π max(n , (n +
1)β ) et µ− β β β ± α α ± β β
n = π min(n , (n + 1) ). Clairement λn ∼ π n et µn ∼ π n .
D’autre part,
Z π Z π/2 Z π/2
du du dθ π
2 du = 2 2 du = 2 √ =√ .
0 1 + ` sin u 0 1 + ` sin u 0 1+` 1+`

Où l’on a effectué le changement de variable cotg u = 1 + ` tg θ. On conclut que
λ− λ+
p n ≤ a n ≤ p n .
1 + µ+
n 1 + µ−
n

P β
Si β > 0, alors an ∼ (πn)α−β/2 et an converge si, et seulement si, α − < −1.
√ P 2
Si β = 0, alors an ∼ (πn)α / 2 et an converge si, et seulement si, α < −1.
P
Si β < 0, alors an ∼ (πn)α et an converge si, et seulement si, α < −1.

Enfin, l’intégrale I(α, β) définie par β

Z ∞

I(α, β) = dx
1 1 + xβ sin2 x
2

converge si, et seulement si,


α
1 −1 0
α < −1 + max(0, β). I(α,β) converge
2
I(α,β) diverge

2
Solution
Z ∞ .10 a. Clairement e−x ≤ e−x pour tout x ≥ 1. D’où la convergence de
2
I= e−x dx.
0
b. Posons
Z π/2 Z π/2
−λ2 sin2 x 2
sin2 x
J(λ) = e dx et H(λ) = cos xe−λ dx.
0 0

Un changement de variable simple montre que


Z
1 λ −u2
H(λ) = e du.
λ 0
Solutions 25

Donc lim λH(λ) = I. D’autre part,


λ→∞

Z π/2
2
sin2 x
0 ≤ J(λ) − H(λ) = (1 − cos x) e−λ dx
0
Z π/2
x −λ2 sin2 x
=2 sin2 e dx
0 2
Z π/2
x2 −4λ2 x2 /π2
≤ e dx
0 2
Z λ
π3 2 −u2
= u e du
16λ3 0
Z ∞
π3 2
≤ 3
u2 e−u du
16λ 0

1
On conclut que λJ(λ) − λH(λ) = O( ). Il en résulte que
λ2

lim λJ(λ) = I.
λ→∞

c. Comme α > 0 et β > 0, le problème de convergence de l’intégrale


Z ∞
α
sin2 x
I(α, β) = xβ e−x dx
0

est du côté de la borne supérieure. La fonction intégrée étant positive alors l’intégrale
P
I(α, β) est de même nature que an avec
Z (n+1)π Z π
β −xα sin2 x α
sin2 x
an = x e dx = (x + nπ)β e−(x+nπ) dx.
nπ 0

On en déduit l’inégalité
Z π Z π
β (n+1)α π α sin2 x β α
π α sin2 x
(nπ) e dx ≤ an ≤ ((n + 1)π) en dx
0 0

p √
2(nπ)β J( (n + 1)α π α ) ≤ an ≤ 2((n + 1)π)β J( nα π α )
an P α
D’où lim = 2I. Alors an converge si, et seulement si, − β > 1.
n→∞ (nπ)β−α/2 2
Finalement, l’ensemble des (α, β) ∈ (IR∗+ )2 pour lesquels l’intégrale I(α, β) converge
est
{(α, β) ∈ (IR∗+ )2 : α > 2(β + 1)}
26 INTÉGRALES GÉNÉRALISÉES

Solution .11 Pour x ∈]0, 1], on a

x x
∀ t ∈ [ , x], f (x) ≤ f (t) ≤ f ( )
2 2

D’où, pour x ∈]0, 1], on a


Z x
x x x
f (x) ≤ f (t) dt ≤ f( )
2 x/2 2 2

soit, pour tout x ∈]0, 1/2],


Z 2x Z x
x x
f (t) dt ≤ xf (x) ≤ 2 f (t) dt ≤ f( )
x x/2 2 2
Z 1 Z x
La convergence de f (t) dt montre, alors, que lim f (t) dt = 0. D’où le résultat.
>
0 x→0 x/2

x2m−1
Solution .12 1◦ . Notons que 1 ≤ m < n montre que lim x2 = 0, donc
Z ∞ x→∞ 1 + x2n
dx
l’intégrale Fn,m converge car converge.
1 x2
En effectuant le changement de variable x 7→ 1/x dans l’intégrale Fn,m , on obtient
Z ∞ Z ∞ Z ∞
x2m−1 x2(n−m)−1 1 x2m−1 + x2(n−m)−1
Fn,m = dx = dx = dx.
0 1 + x2n 0 1+x 2n 2 0 1 + x2n

X 2m−1 + X 2(n−m)−1
Posons Gn,m (X) = , et cherchons la décomposition en éléments
1 + X 2n
simples de Gn,m (X).
2k + 1
Pour k un entier dans {0, 1, . . . , 2n − 1} on pose θk = π. Les pôles de
2n
Gn,m (X) sont {xk = exp(iθk ) : 0 ≤ k < 2n} et ils sont tous simples. D’où

2n−1
X λk
Gn,m (X) =
X − xk
k=0

avec
2(n−m)−1
x2m−1
k + xk i
λk = =− sin(2mθk ).
2nx2n−1
k
n
Alors
n−1
i X sin(2mθk ) sin(2mθ2n−1−k )
Gn,m (X) = − + .
n X − xk X − x2n−1−k
k=0
Solutions 27

Mais θ2n−1−k = 2π − θk et x2n−1−k = xk , par conséquent


n−1
i X sin(2mθk ) sin(2mθk )
Gn,m (X) = − −
n X − xk X − xk
k=0
n−1
X
2 sin(2mθk ) sin θk
=
n X 2 − 2 cos θk X + 1
k=0

n−1
1X
Il en résulte que Fn,m = sin(2mθk )J(θk ) avec
n
k=0
Z ∞
sin θ
J(θ) = dx,
0 x2 − 2 cos θ x + 1

pour θ ∈]0, π[. Mais le changement de variable x − cos θ = u sin θ montre que
Z ∞
du π
J(θ) = 2
= + Arctg (cotg θ) = π − θ.
−cotg θ u + 1 2

(Pour la dernière égalité on distinguera les cas θ ∈]0, π/2] et θ ∈]π/2, π[).
On conclut que
n−1
1X
Fn,m = (π − θk ) sin(2mθk ). (1)
n
k=0

D’autre part, si θ ∈]0, π[, on a


n−1
X e2inθ − 1 e2inθ − 1
ei(2k+1)θ =eiθ =
e2iθ − 1 2i sin θ
k=0
sin(2nθ) 1 − cos(2nθ)
= +i
2 sin θ 2 sin θ
D’où, en considérant les parties imaginaires, et en dérivant les parties réelles de cette
égalité:
n−1
X n−1
X
sin2 nθ cos θ sin 2nθ cos 2nθ
sin(2k + 1)θ = et (2k + 1) sin(2k + 1)θ = 2 −n .
sin θ 2 sin θ sin θ
k=0 k=0

En prenant θ = mπ/n nous trouvons


n−1
X n−1
X n
sin(2mθk ) = 0 et (2k + 1) sin(2mθk ) = − .
sin(mπ/n)
k=0 k=0

En revenant à (1) nous en déduisons


π
Fn,m = .
2n sin(mπ/n)
28 INTÉGRALES GÉNÉRALISÉES

2◦ . On effectue le changement de variable x = u1/2n dans Fn,m on trouve


Z ∞ m −1
1 un π
Fn,m = du =
2n 0 1+u 2n sin(mπ/n)
Alors, pour tout α ∈ Q
/ ∩]0, 1[,

Z ∞
uα−1 π
I(α) = du =
0 1+u sin(πα)
Remarquons que, pour tout α ∈]0, 1[ l’intégrale I(α) converge et
Z 1 α−1 Z ∞ α−1
x x
I(α) = dx + dx
0 1+x 1 1+x
Z 1 α−1 Z 0 1−α
x x −dx 1
= dx + , (x 7→ )
0 1+x 1 1+x x x
Z 1 α−1
x + x−α
= dx
0 1+x
On conclut que I(α) = I(1 − α). D’autre part, pour x fixé dans ]0, 1[, l’application
α 7→ xα−1 + x−α est décroissante sur ]0, 1/2[. Alors α 7→ I(α) est décroissante sur
]0, 1/2[.
Soit α ∈]0, 1/2] \ Q
/ . Il existe deux suites (r )
n n>0 et (sn )n>0 de ]0, 1/2[∩Q , telles
/

que (rn )n>0 soit croissante et tendant vers α et (sn )n>0 décroissante et tendant aussi
vers α. On a pour tout n > 0,
π π
= I(sn ) ≤ I(α) ≤ I(rn ) ≤ .
sin(πsn ) sin(πrn )
En faisant tendre n vers l’infini, on trouve I(α) = π/ sin(πα).
Si α ∈ [1/2, 1[\Q
/ , alors

π π
I(α) = I(1 − α) = = .
sin π(1 − α) sin πα
On a donc démontré que
Z ∞
xα−1 π
∀ α ∈]0, 1[, I(α) = dx = .
0 1+x sin πα

3 . La convergence de J(n, m, q) est immédiate.
Z ∞ m−1
t (1 + tn ) − tn−1
J(n, m, q) − J(n, m, q + 1) = dt
0 (1 + tn )q+1
Z ∞
tn+m−1
= dt
0 (1 + tn )q+1
Z ∞ m
t qntn−1
= dt
0 qn (1 + tn )q+1
· ¸∞ Z
tm m ∞ tm−1
= − + dt
qn(1 + tn )q 0 qn 0 (1 + tn )q
m
= J(n, m, q).
qn
Solutions 29

Alors, µ ¶
m
J(n, m, q + 1) = 1 − J(n, m, q).
qn
Mais, Z Z
∞ ∞
tm−1 u2m−1 π
J(n, m, 1) = dt = 2 du = .
0 1 + tn 0 1+u 2n n sin(mπ/n)
On conclut,
π Y³
q−1

J(n, m, q) = 1− .
n sin(mπ/n) kn
k=1

Le changement de variable tn = u montre, que pour α ∈]0, 1[∩Q


/ ,

Z Y³ α´
∞ q−1
tα−1 π
dt = 1 − .
0 (1 + t)q sin(πα) k
k=1

qui est valable pour α ∈]0, 1[ par un raisonnement semblable à celui de 2◦ .

Solution .13 1◦ . Le changement de variable Arccos x = θ dans l’intégrale In montre


que
Z π/2
In = sin2n+1 θ dθ = W2n+1 .
0

De même, le changement de variable Arctg (1/x) = θ dans l’intégrale Jn montre que


Z π/2
Jn = sin2n−2 θ dθ = W2n−2 .
0

2◦ . Il est bien connu que ∀ u ∈ IR, 1 + u ≤ eu . En prenant u = −x2 , on


2
obtient ∀ x ∈ [0, 1], 1 − x2 ≤ e−x , et en prenant u = x2 , on obtient ∀ x ∈
2 1
[0, +∞[, e−x ≤ . Alors,
1 + x2
2
∀ x ∈ [0, 1], (1 − x2 )n ≤ e−nx

d’où,
Z 1 Z 1 Z ∞
2 n −nx2 2 I
In = (1 − x ) dx ≤ e dx ≤ e−nx dx = √
0 0 0 n
et
2 1
∀ x ∈ [0, +∞[, e−nx ≤
(1 + x2 )n
d’où, Z Z
∞ ∞
I −nx2 dx
√ = e dx ≤ = Jn .
n 0 0 (1 + x2 )n
30 INTÉGRALES GÉNÉRALISÉES

On conclut que
I
In ≤ √ ≤ J n .
n
3◦ . Effectuons dans Wn+2 une intégration par parties

h iπ/2 Z π/2
2n+1
Wn+2 = − cos x sin x + (n + 1) cos2 x sinn x dx = (n + 1)(Wn − Wn+2 ).
0 0

D’où (n + 2)Wn+2 Wn+1 = (n + 1)Wn+1 Wn pour tout n ≥ 0. Ceci démontre que la suite
((n + 1)Wn+1 Wn )n≥0 est constante:

π
∀ n ≥ 0, (n + 1)Wn+1 Wn = W1 W0 = .
2

4◦ . Notons que la suite (Wn )n≥0 est décroissante donc, pour tout n ≥ 2,

2 I2
W2n+2 W2n+1 ≤ W2n+1 = In2 ≤ ≤ Jn2 = W2n−2
2
≤ W2n−2 W2n−3
n

ce qui donne, pour tout n ≥ 2,

π I2 π
≤ ≤
4(n + 1) n 4(n − 1)

ou bien
π n π n
≤ I2 ≤ .
4 n+1 4 n−1

π
Il en résulte que I = .
2
Solution .14 I.1◦ . En supposant que t n’est pas un entier pair on a
n
X eiπnt − e−iπ(n+1)t exp(i(n + 1/2)πt) − exp(−i(n + 1/2)πt)
e−iπkt = −iπt
=
1−e 2i sin(πt/2)
k=−n
sin(n + 1/2)πt
= .
sin(πt/2)

Il en résulte
Z Z n Z 1
1/2
sin(2n + 1)πt 1 1
sin(n + 1/2)πu 1 X 1
dt = du = e−iπku du = .
0 sin πt 2 0 sin(πu/2) 2 0 2
k=−n

En effectuant le changement de variable t = 1 − u dans l’intégrale précédente nous


obtenons Z Z
1/2 1
sin(2n + 1)πt sin(2n + 1)πt 1
dt = dt = .
0 sin πt 1/2 sin πt 2
Solutions 31

I.2◦ . Soit σ = (xk )0≤k≤n une subdivision de [a, b], (i.e. a = x0 < x1 < . . . < xn =
b), et soit
n−1
X
h= λk 1I[xk ,xk+1 [
k=0

où 1I[α,β[ (x) = 1 si x ∈ [α, β[ et 1I[α,β[ (x) = 0 si x ∈


/ [α, β[. Clairement,

Z b n−1
X Z xk+1
h(t) sin mt dt = λk sin mt dt
a k=0 xk
n−1
X cos mxk − cos mxk+1
= λk
m
k=0

donc ¯Z ¯
¯ b ¯ n−1
2 X
¯ ¯
¯ h(t) sin mt dt ¯ ≤ | λk | .
¯ a ¯ m
k=0
Z b
Il en résulte que, lim h(t) sin mt dt = 0, pour toute fonction en escalier h.
m→∞ a
Soit ε > 0. La continuité uniforme de g sur le compact [a, b] montre l’existence de
η > 0 tel que

ε
∀ (x, y) ∈ [a, b]2 , | x − y | < η =⇒ | g(x) − g(y) | < . (1)
2(b − a)

b−a
Soit alors Nε un entier tel que Nε > (b − a)/η, et soit xεk = a + k pour k ∈

{0, 1, . . . , Nε }. On pose

X
hε (x) = g(xεk )1I[xεk ,xεk+1 [ .
k=0

Il est immédiat d’après (1) que

ε
∀ x ∈ [a, b], | g(x) − hε (x) | ≤ .
2(b − a)

D’où, pour tout m ≥ 1,


¯Z Z b ¯ Z
¯ b ¯ b
ε
¯ ¯
¯ g(t) sin mt dt − hε (t) sin mt dt ¯ ≤ | g(t) − hε (t) | dt ≤ . (2)
¯ a a ¯ a 2

Mais la fonction hε est en escalier, donc il existe Mε tel que


¯Z ¯
¯ b ¯ ε
¯ ¯
∀ m ≥ Mε , ¯ hε (t) sin mt dt ¯ < (3)
¯ a ¯ 2
32 INTÉGRALES GÉNÉRALISÉES

D’où, en combinant (2) et (3), pour tout m ≥ Mε ,


¯Z ¯ ¯Z Z b ¯ ¯Z ¯
¯ b ¯ ¯ b ¯ ¯ b ¯
¯ ¯ ¯ ¯ ¯ ¯
¯ g(t) sin mt dt ¯ ≤ ¯ g(t) sin mt dt − hε (t) sin mt dt ¯ + ¯ hε (t) sin mt dt ¯ < ε
¯ a ¯ ¯ a a ¯ ¯ a ¯
Z b
Ce qui démontre, lim g(t) sin mt dt = 0.
m→∞ a
I.3◦ . En utilisant le résultat de I.1◦ ,
n Z 1 Z 1 Ã n !
X X
Sn = f (t) e−2iπkt dt = f (t) e−2iπkt dt
k=−n 0 0 k=−n
Z 1
sin(2n + 1)πt
= f (t) dt
0 sin πt
Z 1/2
sin(2n + 1)πt
= (f (t) − f (0)) dt+
0 sin πt
Z 1
sin(2n + 1)πt f (0) + f (1)
(f (t) − f (1)) dt +
1/2 sin πt 2
Z 1/2 Z 1
f (0) + f (1)
= g1 (t) sin(2n + 1)πt dt + g2 (t) sin(2n + 1)πt dt +
0 1/2 2
avec
f (t) − f (0) f (t) − f (1)
g1 (t) = , et g2 (t) = .
sin πt sin πt
Mais g1 (resp. g2 ) est prolongeable en une fonction continue sur [0, 1/2] (resp. [1/2, 1]).
Alors d’après I.2◦ on a
Z 1/2 Z 1
lim g1 (t) sin(2n + 1)πt dt = 0 et lim g2 (t) sin(2n + 1)πt dt = 0.
n→∞ 0 n→∞ 1/2

f (0) + f (1)
Alors lim Sn = .
n→∞ 2

I.4 . En appliquant le résultat précédent à t 7→ f (t + m), nous trouvons
Xn Z m+1
f (m) + f (m + 1)
lim f (t) e−2iπkt dt = .
n→∞ m 2
k=−n

Nous prenons ensuite la somme de ces égalités pour m variant entre 0 et p − 1.


Xn Z p
1 1
lim f (t) e−2iπkt dt = f (0) + f (1) + · · · + f (p − 1) + f (p).
n→∞ 0 2 2
k=−n

II.1◦ . Soit λ > 0. On a


Z y " #y Z y iλt2
iλt2
iλt 2 1 e 1 e
e dt = + dt
x t 2iλ 2iλ x t2
x
à Z y iλt2 !
iλy 2 2
1 e eiλx e
= − + dt
2iλ y x x t2
Solutions 33

D’où, si 0 < x < y,


¯Z y ¯ µ Z y ¶
¯ iλt2
¯ 1 1 1 1 1
¯ e dt ¯¯ ≤ + + dt =
¯ 2λ y x 2
x x t λx
Z ∞
2
Ce qui démontre la convergence de l’intégrale eiλt dt d’après le critère de Cauchy.
0
En prenant λ = 1 et λ = 2πp, on obtient la convergence des intégrales
Z +∞ Z +∞ Z +∞
2
2 2
α= cos t dt, β= sin t dt, γp = e2iπpt dt.
0 0 0

En effectuant le changement de variable u = 2πp t dans l’intégrale γp on trouve
Z ∞
1 2
γp = √ eiu du.
2πp 0

α + iβ
Alors, γp = √ .
2πp
II.2◦ . Clairement,
Z 1 Z 1
2iπ(x2 −kx) −iπpk2 /2 2
e dx =e e2iπp(x−k/2) dx
0 0
Z k/2
2 2
=(−i)pk e2iπpu du, (u = k/2 − x)
−1+k/2

n Z
X 1
2
Alors, Si l’on pose ∆n (p) = e2iπ(x −kx)
dx, on obtient,
k=−n 0

n
X Z k/2
pk2 2
∆n (p) = (−i) e2iπpu du
k=−n −1+k/2

X Z ` X Z `+1/2
2iπpu2 p 2
= e du + (−i) e2iπpu du
−n≤2`≤n `−1 −n≤2`+1≤n `−1/2

Z m Z m−1/2

 2 2

 e2iπpu du +(−i)p e2iπpu du si n = 2m

 −m−1 −m−1/2
=

 Z m Z m+1/2

 2iπpu2 2

 e du +(−i) p
e2iπpu du si n = 2m + 1
−m−1 −m−3/2

Il en résulte que
n Z r
X 1
2 2
lim e2iπ(x −kx)
dx = 2(1 + (−i)p ) γp = (1 + (−i)p ) (α + iβ).
n→∞ 0 πp
k=−n
34 INTÉGRALES GÉNÉRALISÉES

II.3◦ . Si f (t) = exp(2iπt2 /p), alors


n Z
X p n Z
X p
2
−2iπkt
f (t) e dt = e−2iπ(kt−t /p)
dt
k=−n 0 k=−n 0
n
X Z 1
2
=p e2iπp(u −ku)
du (t = pu)
k=−n 0

=p∆n (p)

Il en résulte d’après I.4◦ que


p
X f (p) − f (0)
p lim ∆n (p) = f (k) + = Gp .
n→∞ 2
k=0

On conclut que
p−1 r
X 2p
2iπk2 /p p
Gp = e = (1 + (−i) ) (α + iβ).
π
k=0

En prenant p = 1 on trouve
r
2
1 = (1 − i) (α + iβ).
π

Alors, Z Z r
∞ ∞
1 π
sin t2 dt = cos t2 dt =
0 0 2 2
et  √
 (1 + i) p si p ≡ 0 mod 4





 √
p−1
X √ (1 + i)(1 + (−i)p )  p si p ≡ 1 mod 4
2iπk2 /p
e = p =
2 

k=0 
 0 si p ≡ 2 mod 4



 √
i p si p ≡ 3 mod 4

OKMRAN
OUBA
ESPACES VECTORIELS NORMÉS

Dans ce chapitre IK désigne IR ou C


|

I. Généralités

Définition : Soit E un IK-espace vectoriel. Une norme sur E est une fonction
k · k : E −→ IR+ vérifiant:
ν1 . ∀ x ∈ E, k x k = 0 =⇒ x = 0,
ν2 . ∀ x ∈ E, ∀ λ ∈ IK, k λx k = | λ | k x k ,
ν3 . ∀ (x, y) ∈ E × E, kx + yk ≤ kxk + kyk.
Un un IK-espace vectoriel muni d’une norme s’appelle espace vectoriel normé.

Remarque : une application N : E −→ IR+ qui vérifie seulement ν2 et ν3 s’appelle


semi-norme.

Notons que la propriété ν3 implique

∀ (x, y) ∈ E × E, |kxk − kyk| ≤ kx − yk.

Par conséquent, l’appication x 7→ k x k est 1-Lipschitzienne.


Exemples :
♠ Le corps IR muni de la valeur absolue | · | est un espace vectoriel normé. De même
( C,
| | · |) est un espace vectoriel normé. Notons que sur IK toutes les normes sont
proportionnelles à | · | à cause de la propriété ν2 .
♠ Sur IKn on peut donner beaucoup d’exemples de normes, pour le moment nous en
donnerons les trois suivantes, définies pour x = (x1 , x2 , . . . , xn ) par:

n
à n
!1/2
X X 2
k x k1 = | xk | , k x k2 = | xk | , k x k∞ = max | xk | .
1≤k≤n
k=1 k=1

♠ Sur IK[X] on peut aussi donner beaucoup d’exemples de normes, En voici deux,
P
m
définies pour P (X) = ak X k par:
k=0

k P k∞ = max | ak | , N∞ (P ) = sup | P (t) | .


0≤k≤m t∈[0,1]
2 Espaces vectoriels normés

♠ Sur C([0, 1], IK), l’espace vectoriel des fonctions continues sur [0, 1], on peut par
exemple définir les normes
Z 1
k f k1 = | f (t) | dt, k f k∞ = sup | f (t) | où f ∈ C([0, 1], IK).
0 t∈[0,1]

♠ Sur C 1 ([0, 1], IK), l’espace vectoriel des fonctions continuement dérivables sur [0, 1],
on peut par exemple définir la norme

N (f ) = | f (0) | + sup | f 0 (t) | où f ∈ C 1 ([0, 1], IK).


t∈[0,1]

Sur un espace véctoriel normé (E, k · k) on peut définir beaucoup de notions


géométriques:
1◦ . Si (x, y) ∈ E × E on appelle k x − y k la distance entre x et y et on la note
d(x, y). Nous avons alors une application d : E × E −→ IR+ vérifiant
δ1 . ∀ (x, y) ∈ E × E, d(x, y) = 0 =⇒ x = y,
δ2 . ∀ (x, y) ∈ E × E, d(x, y) = d(y, x),
δ3 . ∀ (x, y, z) ∈ E × E × E, d(x, z) ≤ d(x, y) + d(y, z).
Cette application d s’appelle la distance sur E associée à la norme k · k.
2◦ . Si a ∈ E et r ∈ IR+ , alors les ensembles suivants
B(a, r) = {x ∈ E : d(x, a) < r} ,
B(a, r) = {x ∈ E : d(x, a) ≤ r} ,
S(a, r) = {x ∈ E : d(x, a) = r}
s’appellent respectivement, boule ouverte, boule fermée et sphère de center a et de rayon
r. La boule B(0, 1) s’appelle la boule unité ouverte de E, et la sphère S(0, 1) s’appelle
la sphère unité de E.
Par exemple, si B1 , B2 et B∞ sont les booles unités de IR2 lorsqu’on munit cet
espace des normes k · k1 , k · k2 et k · k∞ respectivement, alors
B∞ B2 B1

1 1 1

Et voici la boole unité B de (IR2 , N ), lorsque la norme N est donnée


par : B
µ¯ ¯ ¯ ¯ ¶
¯ y ¯¯ ¯¯ y ¯¯ 2
¯
N ((x, y)) = max ¯ x + √ ¯ , ¯ x − √ ¯ , √ | y | . 1
3 3 3
Généralités 3

3◦ . Une partie A de E est dite bornée si, et seulement si, il existe M ∈ IR+ tel
que A ⊂ B(0, M ).
Si A est une partie non vide de E, on appelle diamètre de A la borne supérieure
de d(x, y) lorsque x et y parcourent A.
+
diam (A) = sup {d(x, y) : (x, y) ∈ A × A} ∈ IR .

Proposition I.1. Soit A est une partie non vide de E, Alors


¦ diam (A) < +∞ ⇐⇒ A est bornée.
¦ Si A est symétrique, i.e. (x ∈ A =⇒ −x ∈ A), alors
diam (A) = 2 sup {k z k : z ∈ A} .
Preuve : ¦ Supposons diam (A) < +∞. Comme A 6= Ø on choisit a ∈ A, alors
A ⊂ B(0, M ) avec M = k a k + diam (A). Inversement, si A ⊂ B(0, M ) alors
diam (A) ≤ 2M < +∞.
¦ Supposons que A est symétrique. Pour tout (x, y) ∈ A2 on a

d(x, y) = k x − y k ≤ k x k + k y k ≤ 2 sup {k z k : z ∈ A} ,

donc diam (A) ≤ 2 sup {k z k : z ∈ A}. Inversement, pour tout z ∈ A

2 k z k = k z + z k = d(z, −z) ≤ diam (A),

ce qui démontre l’égalité. ½ ¾


2 x2 y2
Exemple : Soit (a, b) ∈ (IR∗+ )2 .
On considère A = (x, y) ∈ IR : 2 + 2 ≤ 1 . Si
a b
E2 = (IR2 , k · k2 ) alors, en utilisant la symétrie de A on a
½p ¾
2 2
x2 y2
diamE2 (A) =2 sup x +y : 2 + 2 ≤1
a b
np o
=2 sup a2 cos2 θ + b2 sin2 θ : θ ∈ IR
np o
=2 sup a2 t + b2 (1 − t) : t ∈ [0, 1] = 2 max(a, b).

Par contre, dans E1 = (IR2 , k · k1 ), on a


½ ¾ ½ ¾
x2 y2 x2 y2
diamE1 (A) =2 sup | x | + | y | : 2 + 2 ≤ 1 = sup | x | + | y | : 2 + 2 = 1
a b a b
=2 sup {a cos θ + b sin θ : θ ∈ [0, π/2]}
p p
=2 a2 + b2 sup {cos(θ − θ0 ) : θ ∈ [0, π/2]} = 2 a2 + b2 .

a b
avec cos θ0 = √ et sin θ0 = √ .
a2 + b2 a2 + b2
4 Espaces vectoriels normés

Enfin, dans E∞ = (IR2 , k · k∞ ), on a


½ ¾
x2 y2
diamE∞ (A) =2 sup max(| x | , | y |) : 2 + 2 ≤ 1
a b
=2 sup {max(a | cos θ | , b | sin θ |) : θ ∈ IR} = 2 max(a, b).
4◦ . Si A est une partie non vide de E, on appelle la distance d’un élément x ∈ E
à A la quantité
d(x, A) = inf{d(x, y) : y ∈ A}.
Remarquons que cette borne inférieure n’est pas un minimum en général. D’autre part,
la propriété ν3 implique

∀ (x, y) ∈ E × E, | d(x, A) − d(y, A) | ≤ k x − y k .

Par conséquent, l’appication x 7→ d(x, A) est 1-Lipschitzienne.


Exemple : Considérons l’espace vectoriel normé E = (C([0, 1], IR), k · k∞ ) et la partie
½ Z 1 ¾
A = f ∈ E : f (0) = 0, f (t) dt ≥ 1 .
0
Calculons d(0, A).
– Soit f ∈ A, on a
Z 1 Z 1 Z 1
1≤ f (t) dt ≤ | f (t) | dt ≤ k f k∞ dt = k f k∞ = d(0, f ).
0 0 0
D’où d(0, A) ≥ 1.
– Inversement, considérons la suite de fonctions (fn )n≥1 de E,
1
définie par 

 2n2 1
 x si x ∈ [0, ],
2n − 1 n
fn (x) =

 2n 1
 si x ∈ [ , 1].
2n − 1 n 0 1/n 1
Alors, pour tout n ≥ 1 on a fn ∈ A et par conséquent,
2n
d(0, A) ≤ d(0, fn ) = k fn k∞ = ,
2n − 1
d’où d(0, A) = 1.
– Montrons qu’il n’y a pas d’élément g ∈ A tel que d(0, g) = d(0, A) = 1. En effet,
supposons le contraire, alors
Z 1 Z 1 Z 1
1≤ g(t) dt ≤ | g(t) | dt ≤ k g k∞ dt = 1.
0 0 0
Donc, si h(t) = 1 − | g(t) | alors h est une fonction continue et positive qui vérifie
Z 1
h(t) dt = 0. On conclut que h = 0 sur [0, 1], en particulier 0 = h(0) = 1 − | g(0) | = 1.
0
Ce qui est absurde.
Généralités 5

De même, si A et B sont deux parties non vides de E, on appelle la distance de A


à B la quantité
d(A, B) = inf{d(x, y) : (x, y) ∈ A × B}.

5◦ . Soit A une partie de E. On dit que A est convexe si, et seulement si,

∀ (x, y) ∈ A × A, ∀ λ ∈]0, 1[, λx + (1 − λ)y ∈ A.

Ou, d’une façon imagée: “Un observateur situé en un point quelconque x de A peut voir
n’importe quel autre point y de A”.
Toute boule ouverte A = B(a, r) de E est une partie convexe. En effet, si
(x, y) ∈ A × A et si λ ∈]0, 1[ on a

k λx + (1 − λ)y − a k = k λ(x − a) + (1 − λ)(y − a) k


=λ k x − a k + (1 − λ) k y − a k < λr + (1 − λ)r = r.

Une partie A de E est dite étoilée si, et seulement si,

∃ x0 ∈ A, ∀ y ∈ A, ∀ λ ∈]0, 1[, λx0 + (1 − λ)y ∈ A.

Autrement dit: “Un observateur situé au point x0 de A peut voir n’importe quel autre
point y de A”.

Partie convexe Partie étoilée

6◦ . Soient k · k et ||| · ||| deux normes sur un IK-espace vectoriel E. On dit que k · k
et ||| · ||| sont équivalentes si, et seulement si, il existe deux constantes α et β de IR∗+
telles que
∀ x ∈ E, α k x k ≤ ||| x ||| ≤ β k x k .

Ce qui revient à dire que la boule unité de E muni de la norme k · k, contient et est
contenue dans des homothétiques de la boule unité de E muni de la norme ||| · |||.
6 Espaces vectoriels normés

Exemples :
♠ Sur IKn , les trois normes définies pour x = (x1 , x2 , . . . , xn ) par:

n
à n
!1/2
X X 2
k x k1 = | xk | , k x k2 = | xk | , k x k∞ = max | xk | ,
1≤k≤n
k=1 k=1

sont équivalentes car, nous avons

∀ x ∈ IKn , k x k∞ ≤ k x k2 ≤ k x k1 ≤ n k x k∞ .

♠ Sur E = C 1 ([0, 1]), on considère les trois normes définies pour f ∈ E par:

N1 (f ) = k f k∞ + k f 0 k∞ ,
N2 (f ) = | f (0) | + k f 0 k∞ ,
Z 1
N3 (f ) = | f (0) | + | f 0 (t) | dt.
0

Alors, N1 et N2 sont équivalentes, alors que N2 et N3 ne le sont pas. En effet, il est


facile de voir que N3 ≤ N2 ≤ N1 ≤ 2N2 . Par contre, la suite de fonctions (fn )n≥0
1
définie par fn (x) = permet de démontrer qu’il n’existe pas de α > 0 tel que
1 + nx
N2 ≤ αN3 .

II. Voisinages, ouverts, fermés

Dans cette section (E, k · k) désigne un espace vectoriel normé donné.

Définition : Soit a ∈ E, on dit qu’une partie V de E est un voisinage de a dans E


si, et seulement si, il existe ε > 0 tel que B(a, ε) ⊂ V . On note alors, V(a) l’ensemble
des voisinages de a dans E. On a les deux propriétés suivantes:
v1 . Si V ∈ V(a) alors (V ⊂ W =⇒ W ∈ V(a)).
v2 . Si (V, W ) ∈ V(a) × V(a) alors V ∩ W ∈ V(a).

Définition : Une partie A de E est dite ouverte si, et seulement si, A est un voisinage
de chacun de ses éléments: i.e.
A est un ouvert ⇐⇒ ∀ a ∈ A, A ∈ V(a)
⇐⇒ ∀ a ∈ A, ∃ ε > 0, B(a, ε) ⊂ A.
Voisinages, ouverts, fermés 7

Les propriétés suivantes sont évidentes


o1 . Les parties Ø et E sont ouvertes.
o2 . Toute réunion d’une famille de parties ouvertes est une partie ouverte.
o3 . Toute intersection d’une famille finie de parties ouvertes est une partie ouverte.

Exemples :
♠ Toute boule ouverte est une partie ouverte. En effet si A = B(a, r) avec r > 0 alors,
en utilisant l’inégalité triangulaire, nous avons

∀ x ∈ A, B(x, r − k x − a k) ⊂ A.

♠ Si E = (C([0, 1], IK), k · k∞ ), et A = {f ∈ E : f (0) 6= 0}, alors A est une partie


ouverte de E, car
1
∀ f ∈ A, B(f, | f (0) |) ⊂ A.
2

Définition : Une partie A de E est dite fermée si, et seulement si, la partie E \ A est
ouverte: i.e.
A est un fermé ⇐⇒ ∀ x ∈
/ A, ∃ ε > 0 : B(x, ε) ∩ A = Ø
⇐⇒ ∀ x ∈
/ A, ∃ V ∈ V(x) : V ∩ A = Ø.
Les propriétés suivantes sont évidentes
f1 . Les parties Ø et E sont fermées.
f2 . Toute intersection d’une famille de parties fermées est une partie fermée.
f3 . Toute réunion d’une famille finie de parties fermées est une partie fermée.

Exemples :
♠ Toute boule fermée est une partie fermée de E. Car

∀x ∈
/ B(a, r), B(x, k x − a k − r) ∩ B(a, r) = Ø.

♠ On peut aussi vérifier que toute boule fermée n’est pas une partie ouverte de E.

Définition : Soit A une partie non vide de E.


– Pour tout a ∈ A on appelle voisinage de a relativement à A, tout ensemble
de la forme V ∩ A où V ∈ V(a).
– On dit qu’une partie U de A est une partie ouverte relativement à A si, et
seulement si, il existe une partie ouverte O dans E telle que U = O ∩ A.
– On dit qu’une partie W de A est une partie fermée relativement à A si, et
seulement si, il existe une partie fermée F dans E telle que W = F ∩ A.
8 Espaces vectoriels normés

Remarque : Remarquons que remplacer la norme de E par une norme équivalente ne


change pas les voisnages d’un point. Il en sera donc de même pour toutes les notions
(ouvert, fermé,etc.) qui sont définies à partir de la notion de voisinage.

III. Intérieur, adhérence et frontière d’une partie

Dans cette section (E, k · k) désigne un espace vectoriel normé donné.

Définition : Soit A une partie de E. On appelle intérieur de A l’ensemble des points


0
de E qui admettent A pour voisinage, et on note cet ensemble A.

0
a ∈ A ⇐⇒ A ∈ V(a) ⇐⇒ ∃ ε > 0 : B(a, ε) ⊂ A.
0
En particulier, A ⊂ A.

Proposition III.1. Soit A une partie d’un espace vectoriel normé E, alors
0
1◦ . A est un ouvert de E.
0
2◦ . A est le plus grand ouvert de E inclus dans A.
0
3◦ . A est ouvert si, et seulement si A = A.
0 0 0
4◦ . (A) = A.

0
Preuve : 1◦ . Soit x ∈ A, alors il existe ε > 0 tel que B(x, ε) ⊂ A. Mais B(x, ε) est une
boule ouverte, c’est un voisinage de chacun de ses points. Par suite A est un voisinage
0 0
de tous les points de B(x, ε) et donc B(x, ε) ⊂ A. On conclut que A est un ouvert.
0
2◦ . On a vu que A est un ouvert inclus dans A. Soit O un ouvert inclus dans A,
alors
0
x ∈ O =⇒ O ∈ V(x) =⇒ A ∈ V(x) =⇒ x ∈ A.
0
Donc O ⊂ A. Ce qui démontre 2◦ .
3◦ . Si A est une partie ouverte alors elle est la plus grande partie ouverte incluse
0 0
dans A c’est à dire A = A d’après 2◦ . Inversement, si A = A alors A est ouvert d’après
1◦ .
4◦ . c’est une conséquence de 1◦ et de 3◦ .
Intérieur, adhérence et frontière d’une partie 9

0
Proposition III.2. Soit A une partie convexe d’un espace vectoriel normé E, alors A
est convexe.
0 0
Preuve : Soient (x, y) ∈ A × A, et λ ∈]0, 1[. Alors il existe ε > 0 tel que B(x, ε) ⊂ A et
B(y, ε) ⊂ A. On vérifie immédiatement que B(λx + (1 − λ)y, ε) ⊂ A et par conséquent,
0
λx + (1 − λ)y ∈ A.
Définition : Soient A une partie de E, et x ∈ E. On dit que x est un point adhérent
à A si, et seulement si, tout voisinage de x rencontre A, i.e.
∀ V ∈ V(x), V ∩ A 6= Ø
On appelle l’adhérence de A l’ensemble des points adhérents à A, et on note cet ensemble
A. En particulier, A ⊂ A.

Proposition III.3. Soit A une partie d’un espace vectoriel normé E, alors
1◦ . A est un fermé de E.
2◦ . A est le plus petit fermé de E contenant A.
3◦ . A est fermé si, et seulement si A = A.
4◦ . A = A.
Preuve : 1◦ . Soit x ∈ E \ A, alors il existe ε > 0 tel que B(x, ε) ∩ A = Ø. Si
B(x, ε) ∩ A 6= Ø, on trouve y ∈ B(x, ε) ∩ A. Mais B(x, ε) est un voisinage de y qui
doit alors rencontrer A car y ∈ A, d’où B(x, ε) ∩ A 6= Ø, ce qui est contradictoire. On
conclut que B(x, ε) ∩ A = Ø, i.e. B(x, ε) ⊂ E \ A. Finalement E \ A est un ouvert.
2◦ . On a vu que A est un fermé qui contient A. Soit F un fermé contenant A, si
x∈
/ F alors E \F est un voisinage ouvert de x qui ne rencontre pas A, et par conséquent
/ A. D’où E \ F ⊂ E \ A, ou bien A ⊂ F .
x∈
3◦ . Si A est une partie fermée alors elle est la plus petite partie fermée contenant
A c’est à dire A = A d’après 2◦ . Inversement, si A = A alors A est fermé d’après 1◦ .
4◦ . c’est une conséquence de 1◦ et de 3◦ .

Définition : Soit A une partie de E, on appelle frontière de A l’ensemble des éléments


0
de E qui sont adhérents à A mais pas à l’intérieur de A, i.e. Fr (A) = A \ A. Notons
0
que E \ A = E \ A, en effet,
0
x ∈ E \ A ⇐⇒ ∀ ε > 0, B(x, ε) 6⊂ A
⇐⇒ ∀ ε > 0, B(x, ε) ∩ (E \ A) 6= Ø
⇐⇒ x ∈ E \ A.
On conclut que Fr (A) = A ∩ E \ A.
10 Espaces vectoriels normés
0
Exemples : Nous laissons au lecteur le soin de vérifier que (B(x, r)) = B(x, r), et que
B(x, r) = B(x, r) si r > 0, et enfin que Fr (B(x, r)) = S(x, r).

Définition : Soient A, B deux parties de E. On dit que B est dense dans A si,
seulement si, A ⊂ B, i.e.

B est dense dans A ⇐⇒ ∀ x ∈ A, ∀ ε > 0, ∃ y ∈ B : k x − y k < ε.

Définition : Soient A une partie de E, et x ∈ A. On dit que x est un point isolé


dans A si, seulement si, {x} est un ouvert relativement à A, i.e.

x est un point isolé dans A ⇐⇒ ∃ ε > 0 : B(x, ε) ∩ A = {x}.

Remarque : Comme remplacer la norme de E par une norme équivalente ne change


pas les ouverts, fermés,etc.. Alors cela ne change pas non plus l’intérieur, l’adhérence,
la frontière d’une partie. Cette remarque est aussi valable pour toute notion faisant
intervenir les voisinages.

IV. Limites et continuité

Dans cette section (E, k · kE ) et (F, k · kF ) désignent deux espaces vectoriels normés
donnés.

Définition : Soient A une partie non vide de E, et f une application de A dans F .


On dit que f (x) admet une limite lorsque x tend vers a ∈ A tout en restant
dans A si, et seulement si, il existe ` ∈ F tel que

∀ V ∈ V(`), ∃ U ∈ V(a) : f (U ∩ A) ⊂ V (L)

ou bien,
¾
x∈A
∀ ε > 0, ∃η > 0 : =⇒ k f (x) − ` kF < ε (L0 )
k x − a kE < η
Si un tel élément ` existe alors il est unique† . On appelle ` la limite de f (x) lorsque
x −−−→ a, et on écrit ` = x→a
lim f (x).
x∈A x∈A


Car s’il existe deux éléments distincts ` et `0 vérifiant (L0 ), alors en prenant
1
ε = k ` − `0 kF > 0, on trouve η > 0 tel que, pour tout x ∈ A, k x − a kE < η
3
implique (k f (x) − ` kF < ε et k f (x) − `0 kF < ε. Mais a ∈ A alors il exist x ∈ A
tel que k x − a kE < η, d’où 3ε = k ` − `0 kF ≤ k f (x) − ` kF + k f (x) − `0 kF < 2ε ; une
absurdité.
Limites et continuité 11

Définition : Soit (xn )n∈IN une suite de E. On dit que la suite (xn )n∈IN est
convergente si, et seulement si, il existe ` ∈ E tel que
¾
n ∈ IN e
∀ V ∈ V(`), ∃ N ∈ IN : =⇒ xn ∈ V (L)
n≥N

ou bien, ¾
n ∈ IN e0 )
∀ ε > 0, ∃ N ∈ IN : =⇒ k xn − ` kE < ε (L
n≥N
De même, si un tel élément ` existe alors il est unique. On appelle ` la limite de (xn )n∈IN
lorsque n tend vers l’infini, et on écrit ` = lim xn .
n→∞

Proposition IV.1. Soient E, F , G trois espaces vectoriels normés, A ⊂ E, B ⊂ F


deux parties non vides, et f : A −→ F , g : B −→ G deux applications telles que
f (A) ⊂ B. Supposons que, pour un certain a ∈ A, la limite x→a
lim f (x) existe et vaut b,
x∈A

alors b ∈ B. Si de plus la limite lim g(x) existe, alors x→a


lim g ◦f (x) existe et
x→b
x∈B x∈A

lim g ◦f (x) = lim g(x).


x→a x→b
x∈A x∈B

Preuve : C’est immédiat, et laissée au lecteur.

Définition : Soient A une partie non vide de E, et f une application de A dans F .


On dit que f est continue en a ∈ A si, et seulement si,

∀ V ∈ V(f (a)), ∃ U ∈ V(a) : f (U ∩ A) ⊂ V

ou bien,
¾
x∈A
∀ ε > 0, ∃η > 0 : =⇒ k f (x) − f (a) kF < ε.
k x − a kE < η

f est dite continue sur A, si elle est continue en tout point a ∈ A.

Exemples :
♣ Si f : A −→ F est une application lipschitzienne, i.e.

∃ K > 0, ∀ (x, y) ∈ A × A, k f (x) − f (y) kF ≤ K k x − y kE ,

alors f est contiue sur A. Par exemple, les applications x 7→ k x k et x 7→ d(x, B)


(où B est une partie non vide de E) sont continues.
12 Espaces vectoriels normés

♣ Si E et F sont deux espaces vectoriels normés, on munit E × F de la norme


k (x, y) k = max(k x kE , k y kF ). Les deux applications suivantes sont continues

Φ :E × E −→ E : (x, y) 7→ x + y
Ψ :IK × E −→ E : (λ, y) 7→ λy

sont continues.
En effet, l’application Φ est lipschitzienne:

k Φ(x, y) − Φ(x0 , y0 ) kE = k x − x0 + y − y0 kE ≤ k x − x0 kE + k y − y0 kE
≤2 max(k x − x0 kE , k y − y0 kE ).

D’autre part, Fixons (λ0 , y0 ) ∈ IK × E. Pour tout (λ, y) ∈ IK × E, nous avons


Ψ(λ, y) − Ψ(λ0 , y0 ) = (λ − λ0 )(y − y0 ) + (λ − λ0 )y0 + λ0 (y − y0 ). Donc, si (λ, y) ∈
IK × E vérifie max(| λ − λ0 | , k y − y0 kE ) ≤ 1, on a

k Ψ(λ, y) − Ψ(λ0 , y0 ) k ≤ | λ − λ0 | k y − y0 kE + | λ − λ0 | k y0 kE + | λ0 | k y − y0 kE
≤(1 + | λ0 | + k y0 kE ) max(| λ − λ0 | , k y − y0 kE ).

ε
Alors, pour tout ε > 0, il existe η = tel que
1 + ε + | λ0 | + k y0 kE

max(| λ − λ0 | , k y − y0 kE ) < η =⇒ k Ψ(λ, y) − Ψ(λ0 , y0 ) k < ε.

La proposition suivante est une conséquence immédiate de la proposition IV.1.


Proposition IV.2. Soient E, F , G trois espaces vectoriels normés, A ⊂ E, B ⊂ F
deux parties non vides, et f : A −→ F , g : B −→ G deux applications telles
que f (A) ⊂ B. Supposons que f est continue en a ∈ A, et que g est continue en
b = f (a) ∈ B, alors g ◦f est continue en a.

Proposition IV.3. Soient E, F deux espaces vectoriels normés, A ⊂ E une partie


non vide, et f : A −→ F une application, alors les trois propriétés suivantes sont
équivalentes:
1◦ . f est continue sur A.
2◦ . L’image réciproque par f de tout ouvert de F est un ouvert relativement à A.
3◦ . L’image réciproque par f de tout fermé de F est un fermé relativement à A.
Les suites dans un espace vectoriel normé 13

Preuve : (1◦ =⇒ 2◦ ) En effet, soit O un ouvert de F . Pour tout a ∈ f −1 (O),


O est un voisinage de f (a) donc, par continuité de f en a, il existe ηa > 0 tel que
f (B(a, ηa ) ∩ A) ⊂ O ce qui implique que B(a, ηa ) ∩ A ⊂ f −1 (O). D’où

∀ a ∈ f −1 (O), ∃ ηa > 0 : B(a, ηa ) ∩ A ⊂ f −1 (O). (∗)


S
Posons alors W = B(a, ηa ). Clairement, W est un ouvert de E tel que
a∈f −1 (O)
f −1 (O) ⊂ W ∩ A. L’inclusion inverse est aussi vraie d’après (∗). Donc f −1 (O) = W ∩ A,
et f −1 (O) est un ouvert relativement à A.
(2◦ =⇒ 1◦ ) Soit a ∈ A. Pour tout ε > 0 l’ensemble f −1 (B(f (a), ε)) est un ouvert
relativement à A qui contient a, donc il existe η > 0 tel que

B(a, η) ∩ A ⊂ f −1 (B(f (a), ε)).

Ce qui se traduit par


¾
x∈A
∀ ε > 0, ∃η > 0 : =⇒ k f (x) − f (a) kF < ε.
k x − a kE < η
donc f est continue en a.
(3◦ ⇐⇒ 2◦ ) Car si f −1 (F \ V ) = W ∩ A alors f −1 (V ) = (E \ W ) ∩ A.
Voici une définition importante:
Définition : Soient A une partie non vide de E, et une application f : A −→ F . On
dit que f est uniformément continue sur A si, et seulement si,
¾
(x, y) ∈ A2
∀ ε > 0, ∃ η > 0 : =⇒ k f (x) − f (y) kF < ε.
k x − y kE < η
Notons que η ne dépend que de ε. Toute application lipschitzienne est uniformément
continue. La réciproque étant evidemment fausse.
Remarquons enfin, qu’une application uniformément continue f : A −→ F reste
uniformément continue si l’on remplace la norme de E ou celle de F par une norme
équivalente.

V. Les suites dans un espace vectoriel normé

Proposition V.1. Soit A une partie d’un espace vectoriel normé. Les deux assertions
suivantes sont équivalentes:
1◦ . a ∈ A.
2◦ . Il existe une suite (xn )n≥1 de A telle que lim xn = a.
n→∞
14 Espaces vectoriels normés

1
Preuve : Si a ∈ A alors pour tout n ≥ 1 l’intersection B(a, ) ∩ A n’est pas vide donc
n
1 ◦
il existe xn ∈ B(a, ) ∩ A. La suite (xn )n≥1 vérifie 2 .
n
Inversement, si V ∈ V(a) alors il existe, d’après 2◦ un entier n tel que xn ∈ V mais
xn ∈ A donc A ∩ V 6= Ø. D’où a ∈ A.

Corollaire V.2. Soit A une partie convexe d’un espace vectoriel normé E, alors A
est convexe.
Preuve : Soient (x, y) ∈ A × A, et λ ∈]0, 1[. Alors il existe deux suites (xn )n≥0 et
(yn )n≥0 de A telles que lim xn = x et lim yn = y. Mais alors λx + (1 − λ)y = lim zn
n→∞ n→∞ n→∞
avec zn = λxn + (1 − λ)yn ∈ A. D’où λx + (1 − λ)y ∈ A.

Proposition V.3. Soient E, F deux espaces vectoriels normés, A ⊂ E une partie non
vide, et f : A −→ F une application.
1◦ . Si a ∈ A, alors f admet une limite lorsque x −−−→ a si, et seulement si, pour
x∈A
toute suite (xn )n≥1 de A tendant vers a, la suite (f (xn ))n≥1 converge dans F .
De plus, dans ce cas x→a
lim f (x) = lim f (xn ) où (xn )n≥1 est une suite tendant
n→∞
x∈A
vers a.
2◦ . Si a ∈ A, alors f est continue en a si, et seulement si, pour toute suite (xn )n≥1
de A tendant vers a, la suite (f (xn ))n≥1 converge dans F , (Ou converge vers
f (a) ).

Preuve :
La démonstration est laissée en exercice au lecteur.

Définition : Une suite (xn )n≥1 d’un espace vectoriel normé E est dite de Cauchy si,
et seulement si,
¾
(n, m) ∈ IN × IN
∀ ε > 0, ∃ N ∈ IN : =⇒ k xn − xm k < ε.
n ≥ N et m ≥ N

ou bien
lim diam ({xk : k ≥ N }) = 0.
N →∞

Les propriétés suivantes sont immédiates:


– Toute suite de Cauchy dans un espace vectoriel normé est bornée.
– Toute suite convergente dans un espace vectoriel normé est de Cauchy.
Les suites dans un espace vectoriel normé 15

Définition : Soit E un espace vectoriel normé. On dit que E est complet si, et
seulement si, toute suite de Cauchy dans E est convergente. Un espace vectoriel normé
complet s’appelle un espace de Banach.

Exemples :
♣ Le corps IK muni de | · | est un espace vectoriel normé complet.
♣ L’espace vectoriel normé E = (IKp , k · k∞ ) est complet.
En effet, soit (xn )n≥1 une suite de Cauchy dans E. Le vecteur xn s’écrit
(1) (p)
xn = t [xn , . . . , xn ] ∈ IKp . Comme
¯ ¯
¯ (k) (k) ¯
∀ k ∈ {1, . . . , p}, ∀ (n, m) ∈ IN : ¯ xn − xm ¯ ≤ k xn − xm k∞ ,

(k)
alors, pour tout k ∈ {1, . . . , p}, la suite (xn )n≥1 est de Cauchy dans IK
(qui est complet), donc elle converge vers un élément x(k) ∈ IK. Notons
x = t [x(1) , . . . , x(p) ] ∈ IKp , ce qui précède montre que, pour tout
¯ ε > 0 et¯ tout
¯ (k) ¯
k ∈ {1, . . . , p}, il existe N (k) ∈ IN tel que n ≥ N (k) implique ¯ xn − x(k) ¯ < ε,
ou bien n ≥ max1≤k≤p N (k) implique k xn − x k∞ < ε. Par conséquent, la suite
(xn )n≥1 converge dans E vers x. L’espace E est alors complet.
♣ Soit E l’espace vectoriel normé IK[X] muni de la norme k · k1 définie par
Z 1
k P k1 = | P (t) | dt. Montrons que E n’est pas complet.
0
n
X
En effet, considérons la suite (Pn )n≥0 de E définie par Pn (X) = (−1)k X k . Pour
k=0
tout m > n et tout t ∈ [0, 1] on a
¯ m ¯
¯ X ¯ | 1 − (−t)m−n |
¯ ¯
| Pm (t) − Pn (t) | = ¯ (−1)k tk ¯ = tn+1 ≤ 2tn+1 .
¯ ¯ 1+t
k=n+1

donc
2
m > n =⇒ k Pm − Pn k1 ≤ .
n+2
Ce qui démontre que la suite (Pn )n≥0 est de Cauchy dans E.
Si E est complet, alors il existe S ∈ IK[X] tel que

k Pn − S k −−−→ 0
n→∞

Or (1 + X)Pn = 1 + (−1)n X n+1 , donc

(1 + X)S − 1 = (1 + X)(S − Pn ) + (−1)n X n+1 .


16 Espaces vectoriels normés

D’où
∀ t ∈ [0, 1], | (1 + t)S(t) − 1 | ≤ 2 | S(t) − Pn (t) | + tn+1 ,

par suite,
1
k (1 + X)S − 1 k1 ≤ 2 k Pn − S k1 + .
n+2
On conclut, en faisant tendre n vers l’infini, que (1 + X)S = 1. En comparant les
degrés des deux membres, on voit que ceci est une contradiction. La suite (Pn )n≥0
ne converge pas.

Définition : Soit (xn )n∈IN une suite d’un espace vectoriel normé E.
P
— On dit que la série xn converge et admet S ∈ E pour somme si, et
n
X
seulement si, la suite des sommes partielles (Sn )n∈IN (avec Sn = xk ) converge
k=0
vers S.
P P
— On dit la série xn converge normalement si, et seulement si, la série k xn k
converge.

Notons que si E est complet, alors toute série normalement convergente est con-
vergente. Inversement, si dans un espace vectoriel normé E, toute série normalement
convergente est convergente, alors l’espace E est complet. (cf. exercice 10).

VI. Les parties compactes dans un espace vectoriel normé

Définition : Une partie A d’un espace vectoriel normé E est dite compacte si, et
seulement si, de toute suite de A on peut extraire une sous-suite convergente vers un
élément de A.

Proposition VI.1. Soit A une partie compacte d’un espace vectoriel normé E. Alors
A est une partie fermée et bornée.

Preuve : – Soit x ∈ A, alors il existe une suite (xn )n≥1 de A qui converge vers x. Or
A est compacte donc, il existe une sous-suite (xϕ(n) )n≥1 de (xn )n≥1 qui converge vers
y ∈ A. Mais (xn )n≥1 converge vers x donc x = y ∈ A. On a démontré que A ⊂ A, d’où
A = A et A est fermé.
– Si A n’est pas bornée alors, pour tout n ≥ 1 il existe xn ∈ A tel que k xn k ≥ n.
La compacité montre qu’il existe une sous-suite (xϕ(n) )n≥1 de (xn )n≥1 qui converge vers
x ∈ A. Ceci est contradictoire car (xϕ(n) )n≥1 est une suite convergente et non bornée.
Les parties compactes dans un espace vectoriel normé 17

5
! La réciproque de la proposition précédente est en général fausse. Considérons le
contre-exemple suivant: Soit E l’espace vectoriel normé C([0, 2π], IR) des fonctions
continues sur [0, 2π] muni de la norme uniforme k · k∞ , (i.e. k f k∞ = sup | f (t) |),
x∈[0,2π]
et A = B(0, 1) la boule unité fermé de E. Nous savons que A est une partie fermée et
bornée de E.
Nous allons démontrer que A n’est pas compacte. En effet, considérons la suite
(fn )n≥1 de A telle que fn (x) = sin nx, si A est compacte alors, il existe ϕ : IN∗ −→ IN∗
strictement croissante telle que (fϕ(n) )n≥1 converge vers f ∈ A.
Pour tout n ≥ 1 et tout x ∈ [0, 2π],
¯ ¯
¯ fn (x)f (x) − f 2 (x) ¯ ≤ | f (x) | | fn (x) − f (x) | ≤ k fn − f k ,

Alors, pour tout n ≥ 1


¯Z 2π Z 2π ¯
¯ ¯ ° °
¯ sin(ϕ(n)x)f (x) dx − f (x) dx ¯¯ ≤ 2π ° fϕ(n) − f °∞ ,
2
¯
0 0

ce qui démontre que


Z 2π Z 2π
2
f (x) dx = lim sin(ϕ(n)x)f (x) dx = 0. (†)
0 n→∞ 0

La dernière égalité résulte du lemme de Riemann*.


D’autre part, pour tout n ≥ 1 et tout x ∈ [0, 2π],
¯ 2 ¯
¯ fn (x) − f 2 (x) ¯ ≤ | fn (x) + f (x) | | fn (x) − f (x) | ≤ 2 k fn − f k ,

donc ¯ ¯
¯ ¯
¯ Z 2π Z 2π ¯
¯ ¯ ° °
¯ 2
sin (ϕ(n)x) dx − f (x) dx ¯¯ ≤ 4π ° fϕ(n) − f °∞ ,
2
¯
¯ |0 {z } 0 ¯
¯ ¯

par suite, Z 2π
f 2 (x) dx = π. (‡)
0

Ce qui contredit (†), et démontre que la suite (fn )n≥1 n’admet aucune sous-suite
convergente. A n’est alors pas compacte.

Z b
* Soit g une fonction continue sur un intervalle [a, b] de IR alors lim g(t) sin mt dt = 0.
m→∞ a
(Voir l’exercice 14.2◦ du chapitre sur les intégrales généralisées).
18 Espaces vectoriels normés

Remarque : Nous rappelons cependant que dans IK toute partie fermée et bornée est
compacte, et nous verrons plus loin une généralisation.

Proposition VI.2. Soit A une partie compacte d’un espace vectoriel normé E. Alors
toute partie fermée B contenue dans A est compacte.

Preuve : En effet, soit (yn )n∈IN une suite de B. Comme B ⊂ A et A est compacte
alors il existe une sous-suite (yϕ(n) )n∈IN de (yn )n∈IN qui converge vers a ∈ A. D’après
la proposition V.1 a ∈ B, mais B est fermé donc a ∈ B.

Proposition VI.3. Soit (xn )n≥0 une suite d’un espace vectoriel normé E, qui converge
vers a. Alors l’ensemble A = {xn : n ≥ 0} ∪ {a} est une partie compacte de E.

Preuve : Considérons une suite (yn )n≥0 de A. Il y a l’alternative suivante:


– Soit, il existe ε0 > 0, et entier N ∈ IN tels que, pour tout n > N , on a
k yn − a k ≥ ε0 . On pose, alors ∆ = {x ∈ A : k x − a k ≥ ε0 }, on a
[
]N, +∞[∩IN ⊂ {n ∈ IN; yn = z}.
z∈∆

Mais ∆ est un ensemble fini, (car la suite (xn )n≥0 converge vers a), alors il existe z0 ∈ ∆
tel que l’ensemble {n ∈ IN; yn = z0 } soit infini. On définit alors ϕ : IN −→ IN par

ϕ(0) = min{k : yk = z0 }, et pour n ≥ 1 ϕ(n) = min{k > ϕ(n − 1) : yk = z0 }.

L’application ϕ est strictement croissante et telle que la sous-suite (yϕ(n) )n≥0 converge
vers z0 ∈ A.
– Soit, pour tout ε > 0 et tout N ∈ IN, il existe n > N tel que k yn − a k < ε. On
définit alors ϕ : IN −→ IN par

ϕ(0) = min{k : k yk − a k < 1},


ϕ(n) = min{k > ϕ(n − 1) : k yk − a k < 2−n } pour n ≥ 1.

L’application ϕ est strictement croissante, et la sous-suite (yϕ(n) )n≥0 converge vers


a ∈ A. Dans les deux cas, il existe une sous-suite (yϕ(n) )n≥0 de (yn )n≥0 qui converge
dans A.

Proposition VI.4. Soient E et F deux espaces vectoriels normés, et A ⊂ E, B ⊂ F


deux parties compactes. Alors A×B est une partie compacte de l’espace vectoriel normé
E × F muni de la norme définie par k (x, y) k = max(k x kE , k y kF ).
Les parties compactes dans un espace vectoriel normé 19

Preuve : Soit (xn )n∈IN une suite de A × B. Pour tout n ≥ 0, xn = (an , bn ). Mais
(an )n∈IN est une suite de A, qui est compacte, alors il existe ϕ : IN −→ IN, strictement
croissante, telle que (aϕ(n) )n∈IN converge vers a ∈ A. De même (bϕ(n) )n∈IN est une
suite de B, qui est compacte, alors il existe ψ : IN −→ IN, strictement croissante,
telle que (bϕ◦ψ(n) )n∈IN converge vers b ∈ B. Donc si θ = ϕ◦ψ alors la suite (bθ(n) )n∈IN
converge vers b ∈ B, et la suite (aθ(n) )n∈IN converge vers a ∈ A, (car c’est une sous-
suite de (aϕ(n) )n∈IN ). Ce qui se traduit par le fait que la sous-suite (xθ(n) )n∈IN tend vers
(a, b) ∈ A × B dans E × F .

Voici une généralisation immédiate.


Proposition VI.5. Soient E1 , . . . , Em des espaces vectoriels normés, on note F
l’espace vectoriel normé E1 × · · · × Em muni de la norme définie par

k (x1 , . . . , xm ) kF = max k xi kEi .


1≤i≤m

Si, pour tout i ∈ {1, . . . , m}, Ai est une partie compacte de Ei . Alors A = A1 × · · · × Am
est une partie compacte de F .

C’est une récurrence simple sur m.

Proposition VI.7. Soit E = (IKm , k · k∞ ) et A une partie de E. Alors A est compacte


si, et seulement si, A est une partie fermée et bornée.

Preuve : La proposition VI.1 montre que si A est compacte alors A est une partie
fermée bornée.
Inversement, Comme A est bornée alors il existe M > 0 tel que A ⊂ B(0, M ).
Mais, si ∆ = {x ∈ IK : | x | ≤ M } alors nous savons que ∆ est une partie compacte de
IK et la proposition précédente montre que ∆m est une partie compacte de E. Or, il est
clair que A ⊂ ∆m et A est fermée donc d’après la proposition VI.2, A est une partie
compacte.

Proposition VI.7.Soient A une partie compacte, non vide, d’un espace vectoriel normé
E, et f : A −→ F une application continue sur A à valeurs dans un espace vectoriel
normé F . Alors
1◦ . f (A) est une partie compacte.
2◦ . f est uniformément continue sur A.
20 Espaces vectoriels normés

Preuve : 1◦ . Soit (yn )n≥1 une suite de f (A), alors, pour tout n ≥ 1, il existe xn ∈ A tel
que f (xn ) = yn . Mais A est compacte donc on peut extraire une sous-suite (xϕ(n) )n≥1
de (xn )n≥1 qui converge vers x ∈ A. La continuité de f en x montre que la sous-suite
(yϕ(n) )n≥1 de (yn )n≥1 converge vers f (x) ∈ f (A). On conclut que f (A) est compacte.
2◦ . Supposons que f n’est pas uniformément continue sur A, alors

 1
 k xn − yn k ≤ (1)
∃ ε0 > 0, ∀ n ≥ 1, ∃ (xn , yn ) ∈ A × A avec n
 et

k f (xn ) − f (yn ) k ≥ ε0 (2)

Mais la compacité de A nous permet d’extraire une sous-suite (xϕ(n) )n≥1 de (xn )n≥1 qui
converge vers a ∈ A. L’inégalité (1) montre que (yϕ(n) )n≥1 converge aussi vers a ∈ A.
La continuité de f en a, montre qu’il existe η > 0 tel que

ε
(x ∈ A et k x − a k < η) =⇒ k f (x) − f (a) k < , (3)
3

et la convergence de suites (xϕ(n) )n≥1 et (yϕ(n) )n≥1 vers a, montre l’existence de N ∈ IN


° ° ° °
tel que, pour tout n ≥ N , ° xϕ(n) − a ° < η et ° yϕ(n) − a ° < η. Par suite, en utilisant
(3) et (2), pour n ≥ N on a

° ° ° ° ° ° 2ε0
ε0 ≤ ° f (xϕ(n) ) − f (yϕ(n) ) ° ≤ ° f (xϕ(n) ) − f (a) ° + ° f (yϕ(n) ) − f (a) ° ≤ .
3

Cette absurdité montre que f est uniformément continue sur A.

Propsition VI.8.Soient A une partie compacte, non vide, d’un espace vectoriel normé
E, et f : A −→ IR une application continue sur A. Alors f atteint ses bornes sur A.

Preuve : f (A) est une partie compacte de IR donc bornée. Soit

M = sup{f (x) : x ∈ A},

pour tout n ≥ 1, il existe xn ∈ A tel que

1
M− ≤ f (xn ) ≤ M. (∗)
n

De la suite (xn )n∈IN de A, on peut extraire une sous-suite (xϕ(n) )n≥0 qui converge vers
a ∈ A. La continuité de f en a et l’inégalité (∗) montrent que f (a) = M . Nous laissons
au lecteur la tâche de démontrer qu’il existe b ∈ A tel que f (b) = inf{f (x) : x ∈ A}.
Applications linéaires continues 21

VII. Applications linéaires continues

Théorème VII.1. Soient E et F deux espaces vectoriels normés et T : E −→ F une


application linéaire. Alors les assertions suivantes sont équivalentes:
1◦ . T est continue.
2◦ . T est continue en 0.
3◦ . Il existe M ∈ IR∗+ tel que ∀ x ∈ E, k T (x) kF ≤ M k x kE .

Preuve :
(1◦ =⇒ 2◦ ) C’est trivial.
(2◦ =⇒ 3◦ ) Pour ε = 1 il existe η > 0 tel que k z kE ≤ η =⇒ k T (z) kF ≤ 1. Soit
η
x ∈ E \ {0}, alors z = x vérifie k z kE ≤ η et par conséquent k T (z) kF ≤ 1, ce
k x kE
qui se traduit par
1
k T (x) kF ≤ k x kE .
η
(3◦ =⇒ 1◦ ) Car pour tout (x, y) ∈ E × E, on a

k T (x) − T (y) kF = k T (x − y) kF ≤ M k x − y kE

donc T est lipschitzienne, elle est même uniformément continue sur E.

Théorème VII.2. Soient E, F et G des IK-espaces vectoriels normés.


1◦ . Soit T : E −→ F une application linéaire continue. Alors l’ensemble

M = {M ≥ 0 : ∀ x ∈ E, k T (x) kF ≤ M k x kE }

est un intervalle fermé de la form [·, +∞[ contenu dans IR+ . On note k T k =
min M, alorst on a

k T k = sup {k T (x) kF : k x kE ≤ 1} .

2◦ . Si Lc (E, F ) désigne l’espace vectoriel des applications linéaires continues, alors


T 7→ k T k définie une norme sur Lc (E, F ). De plus, si T ∈ Lc (E, F ) et
S ∈ Lc (F, G), alors k S ◦T k ≤ k S k k T k.
3◦ . Si F est complet, alors (Lc (E, F ), k · k) est complet.
22 Espaces vectoriels normés

Preuve :
1◦ . Comme T est continue alors d’après le théorème précédent l’ensemble M est
une partie non vide de IR minorée par 0. Soit donc β = inf M. Il est clair que M
contient tous les réels m > β. Par conséquent
1
∀ n ≥ 1, ∀ x ∈ E, k T (x) kF ≤ (β + ) k x kE ,
n
et en faisant tendre n vers l’infini on obtient

∀ x ∈ E, k T (x) kF ≤ β k x kE ,

d’où, β ∈ M. Alors M = [β, +∞[. Notons

α = sup {k T (x) kF : k x kE ≤ 1} .
° °
° x °
– On a, pour tout x ∈ E \ {0}, ° T ( ° )° ≤ α. D’où, pour tout x ∈ E,
k x kE °F
k T (x) kF ≤ α k x kE . Par suite β ≤ α.
– On a k T (x) kF ≤ β k x kE , pour tout x ∈ E. En particulier, si k x kE ≤ 1 alors
k T (x) kF ≤ β d’où α ≤ β. Ce qui démontre l’égalité α = β.
2◦ . Les propriétés ν1 et ν2 sont immédiates. Démontrons ν3 . En effet, si T et S sont
deux applications linéaires continues sur E alors, pour tout x ∈ E avec k x kE ≤ 1, on a

k (T + S)((x) kF = k T (x) + S(x) kF ≤ k T (x) kF + k S(x) kF ≤ k T k + k S k .

D’où k T + S k = sup{k (T + S)(x) kF : k x kE ≤ 1} ≤ k T k + k S k.


D’autre part, si T ∈ Lc (E, F ) et S ∈ Lc (F, G), alors

∀ x ∈ E, k S ◦T (x) kG = k S(T (x)) kG ≤ k S k k T (x) kF ≤ k S k k T k k x kE .

Par conséquent, k S ◦T k ≤ k S k k T k.
3◦ . Considérons une suite de Cauchy (Tn )n≥0 de Lc (E, F ). Pour un x ∈ E fixé,
nous avons

∀ (n, m) ∈ IN × IN, k Tn (x) − Tm (x) kF ≤ k Tn − Tm k k x kE ,

donc (Tn (x))n≥0 est une suite de Cauchy de l’espace de Banach F , alors elle converge
vers un élément T (x) ∈ F .
L’application x 7→ T (x) est linéaire car, pour tout (x, y) ∈ E 2 et tout (α, β) ∈ IK2 ,

T (αx + βy) = lim Tn (αx + βy) = lim (αTn (x) + βTn (y))
n→∞ n→∞
=α lim Tn (x) + β lim Tn (y) = αT (x) + βT (y).
n→∞ n→∞
Applications linéaires continues 23

Ici nous avons utilisé la continuité des opérations algébriques dans F .


D’autre part, la suite (Tn )n≥0 est de Cauchy donc elle est bornée. On pose
λ = sup{k Tn k : n ≥ 0}. Nous avons

∀ n ≥ 0, ∀ x ∈ E, k Tn (x) kF ≤ k Tn k k x kE ≤ λ k x kE .

Il en résulte que ∀ x ∈ E, k T (x) kF ≤ λ k x kE , et par conséquent T ∈ Lc (E, F ).


Enfin, Soit ε > 0, il existe N ∈ IN tel que

∀ (n, m) ∈ IN2 , (n > N et m > N ) =⇒ k Tn − Tm k ≤ ε,

ou bien,

∀ x ∈ E, ∀ (n, m) ∈ IN2 , (n > N et m > N ) =⇒ k Tn (x) − Tm (x) kF ≤ ε k x kE .

En faisant tendre m vers +∞, on obtient

∀ x ∈ E, ∀ n > N, k Tn (x) − T (x) kF ≤ ε k x kE .

Nous avons donc démontré que,

∀ ε > 0, ∃ N ∈ IN, ∀ n > N, k Tn − T k ≤ ε.

C’est à dire que la suite (Tn )n≥0 converge vers T dans Lc (E, F ).

Corollaire VII.3. Soient E un espace vectoriel normé, complet, et u ∈ Lc (E) un


endomorphisme continue tel que k u k < 1. Alors IE −u est inversible, d’inverse continue
° ° 1
avec ° (IE − u)−1 ° ≤ .
1 − kuk

n
X
Preuve : Considérons, Sn = uk , pour tout (n, p) ∈ IN2 , on a,
k=0

° n+p °
° X ° n+p
X ° ° n+p
X kuk
n+1
° k° ° k° k
k Sn+p − Sn k = ° u °≤ u ≤ kuk ≤ .
° ° 1 − kuk
k=n+1 k=n+1 k=n+1

Il en résulte que (Sn )n≥0 est une suite de Cauchy dans l’espace de Banach Lc (E), donc
elle converge. On note

X
S = lim Sn = uk .
n→∞
k=0
24 Espaces vectoriels normés

Nous avons, pour tout n ∈ IN, (IE − u)◦Sn = IE − un+1 donc


(IE − u)S − IE = (IE − u)(S − Sn ) − un+1
Par conséquent,
n+1
k (IE − u)S − IE k ≤ k IE − u k k S − Sn k + k u k −−−→ 0.
n→∞
Alors, (IE − u)S = IE . On démontre de même que S(IE − u) = IE , ce qui prouve que
IE − u est inversible et admet S pour inverse. Enfin, pour tout n ≥ 0,
Xn
k 1
k Sn k ≤ kuk ≤
1 − kuk
k=0
° ° 1
d’où en faisant tendre n vers l’infini, ° (IE − u)−1 ° = k S k ≤ .
1 − kuk
Corollaire VII.4. Soient E un espace vectoriel normé et complet, et GLc (E) le groupe
des isomorphismes continues de E. (i.e. u ∈ GLc (E) si, et seulement si, u est linéaire,
inversible et u et u−1 sont continues). Alors GLc (E) est une partie ouverte de Lc (E).

Preuve : En effet, soit u0 ∈ GLc (E), et considérons u ∈ Lc (E) tel que k u − u0 k <
° °
1/ ° u−1
0
°. On a
° ° ° °
° IE − u−1 u ° ≤ ° u−1 ° k u0 − u k < 1,
0 0

donc IE − (IE − u−1 −1 −1


0 u) = u0 u ∈ GLc (E), et par conséquent, u0 (u0 u) = u ∈ GLc (E).
On conclut que
1
∀ u0 ∈ GLc (E), B(u0 , ° −1 ° ) ⊂ GLc (E).
°u °
0
Ce qui démontre que GLc (E) est un ouvert de Lc (E).

Exemples :
♣ Soit E l’espace vectoriel normé C([0, 1], IK) muni de la norme uniforme:
k f k∞ = sup | f (t) | .
t∈[0,1]

La forme linéaire T : E −→ IK définie par T (f ) = f (0) est continue. En effet,


∀ f ∈ E, | T (f ) | = | f (0) | ≤ k f k∞ .
avec égalité si f (t) = 1 pour tout t ∈ [0, 1]. On conclut que T est continue et que
k T k = 1.
♣ Soient F l’espace vectoriel normé C([0, 1], IK) muni de la norme
Z 1
k f k1 = | f (t) | dt.
0
La forme linéaire S : E −→ IK définie par S(f ) = f (0) n’est pas continue. En effet,
si fn (t) = (n + 1)(1 − t)n on a k fn k1 = 1 et S(fn ) = n + 1, pour tout n ≥ 0.
Applications multilinéaires continues 25

VIII. Applications multilinéaires continues

Théorème VIII.1. Soient E1 , E2 et F des espaces vectoriels normés et B une


application bilinéaire de E1 ×E2 dans F . Alors les assertions suivantes sont équivalentes:
1◦ . B est continue.
2◦ . B est continue en (0, 0).
3◦ . Il existe M ∈ IR∗+ tel que

∀ (x1 , x2 ) ∈ E1 × E2 , k B(x1 , x2 ) kF ≤ M k x1 kE1 k x2 kE2 .

Preuve :
(1◦ =⇒ 2◦ ) C’est immédiat.
(2◦ =⇒ 3◦ ) Pour ε = 1 il existe η > 0 tel que

max(k z1 kE1 , k z2 kE2 ) ≤ η =⇒ k B(z1 , z2 ) kF ≤ 1.

η η
Soient x1 ∈ E1 \ {0} et x2 ∈ E2 \ {0}, alors z1 = x1 et z2 = x2 vérifient
k x1 kE1 k x2 kE2
max(k z1 kE1 , k z2 kE2 ) ≤ η et par conséquent k B(z1 , z2 ) kF ≤ 1, ce qui se traduit par

1
k B(x1 , x2 kF ≤ k x1 kE1 k x2 kE2 .
η2

(3◦ =⇒ 1◦ ) Soit (α, β) ∈ E1 × E2 . Pour tout (x, y) ∈ E1 × E2 , on a

B(x, y) − B(α, β) = B(x − α, y − β) + B(α, y − β) + B(x − α, β)

donc, si max(k x − α kE1 , k y − β kE2 ) ≤ 1,

k B(x, y) − B(α, β) kF ≤M k x − α kE1 k y − β kE2 + M k α kE1 k y − β kE2 +


M k x − α kE1 k β kE2
≤M (1 + k α kE1 + k β kE2 ) max(k x − α kE1 , k y − β kE2 ).

Ce qui démontre que B est continue en (α, β).

Le théorème précédent admet la généralisation suivante:


26 Espaces vectoriels normés

Théorème VIII.2. Soient E1 , . . . , En et F des espaces vectoriels normés et B une


application n-linéaire de E1 × . . . × En dans F . Alors les assertions suivantes sont
équivalentes:
1◦ . B est continue.
2◦ . B est continue en (0, . . . , 0).
3◦ . Il existe M ∈ IR∗+ tel que
n
Y
∀ (x1 , . . . , xn ) ∈ E1 × . . . × En , k B(x1 , . . . , xn ) kF ≤ M k xk kEk .
k=1

IX. Les espaces vectoriels normés de dimension finie

Théorème IX.1. Soient E un espace vectoriel normé de dimension finie m, et


E = (e1 , . . . , em ) une base de E. On considère l’application
m
X
m
Φ : (IK , k · k∞ ) −→ E : (x1 , . . . , xm ) 7→ xk ek .
k=1

Alors Φ est un isomorphisme linéaire et Φ et Φ−1 sont continues.

Preuve : – La linéarité et la bijectivité de Φ sont evidentes.


– Pour tout x = (x1 , . . . , xm ) ∈ IKm on a

m
à m
!
X X
k Φ(x) kE ≤ | xk | k ek kE ≤ k ek kE k x k∞ .
k=1 k=1

Donc Φ est continue.


– D’autre part, l’ensemble S = {x ∈ IKm : k x k∞ = 1} est un fermé et borné de
(IKm , k · k∞ ) donc compact (proposition VI.7). La continuité de l’application

φ : S −→ IR : x 7→ k Φ(x) kE

et la compacité de S montrent que φ atteint sa borne inférieure sur S. Alors, il existe


x0 ∈ S tel que
µ = φ(x0 ) = min{k Φ(x) kE : x ∈ S}.

Evidemment, µ > 0 car si µ = 0 alors Φ(x0 ) = 0 donc x0 = 0 (puisque Φ est injective),


mais x0 = 0 est absurde car k x0 k∞ = 1.
Les espaces vectoriels normés de dimension finie 27
x
Soit x ∈ IKm \ {0}, alors ∈ S. Par conséquent,
k x k∞
° °
° x °
° Φ( ) ° ≥ µ.
° kxk °
∞ E

On conclut que
∀ x ∈ IKm , k Φ(x) kE ≥ µ k x k∞ .

Ce qui revient à écrire

° −1 ° 1
∀ x ∈ E, ° Φ (x) ° ≤ k x kE .
∞ µ

Ou bien que l’application Φ−1 est continue.

Corollaire IX.2. Soit E un espace vectoriel normé de dimension finie. Alors E est
complet, et les compactes de E sont les parties fermées et bornées.

Preuve : C’est une conséquence du théorème IX.1 et du fait que (IKm , k · k∞ ) vérifie
ces propriétés.

Théorème IX.3. Soient E et F deux espaces vectoriels normés avec E de dimension


finie m. Alors toutes les applications linéaires de E dans F sont continues.

Preuve : Soit u : E −→ F une application linéaire. Considérons E = (e1 , . . . , em ) une


base de E, et
m
X
Φ : (IKm , k · k∞ ) −→ E : (x1 , . . . , xm ) 7→ xk ek .
k=1

Posons enfin, v = u◦Φ. Pour tout x = (x1 , . . . , xm ) ∈ IKm on a

m
à m
!
X X
k v(x) kF ≤ | xk | k u(ek ) kF ≤ k u(ek ) kF k x k∞ .
k=1 k=1

Donc v est continue, et en utilisant le théorème précédent u = v ◦Φ−1 est aussi continue.
D’où le résultat.

Théorème IX.4. Toutes les normes sur un espace vectoriel de dimension finie sont
équivalentes.
28 Espaces vectoriels normés

Preuve : Soient k · k et ||| · ||| deux normes sur un espace vectoriel de dimension finie E.
Les deux applications linéaires
φ :(E, k · k) −→ (E, ||| · |||) : x 7→ x,
ψ :(E, ||| · |||) −→ (E, k · k) : x 7→ x.
sont continues d’après le théorème précédent. Ce qui s’écrit

∀ x ∈ E, ||| x ||| ≤ k φ k k x k et k x k ≤ k ψ k ||| x |||.

Donc les deux normes k · k et ||| · ||| sont équivalentes.

Corollaire IX.5. Tout sous-espace vectoriel de dimension finie F d’un espace vectoriel
normé E est fermé.
Preuve : Car F est complet !

Nous terminons ce chapitre par une caractérisation des espaces vectoriels normés
de dimension finie. Commençons par le lemme suivant:
Lemme IX.6. Soient E un espace vectoriel normé et F un sous-espace vectoriel fermé
de E différent de E. Alors
1
∀ ε > 0, ∃y ∈ E : (k y k = 1, et d(y, F ) ≥ .
1+ε

Preuve : Soit a ∈ E \ F . Comme F est fermé alors d = d(a, F ) > 0. Il existe b ∈ F tel
a−b
que k a − b k < (1 + ε)d. On pose y = alors, pour tout x ∈ F ,
ka − bk
¯¯ ¯¯
1 ¯¯ ¯¯ d d 1
ky − xk = ¯¯ a − b − x k a − b k ¯¯ ≥
k a − b k ¯¯ | {z } ¯¯ k a − b k > d(1 + ε) = 1 + ε .
∈F

Théorème IX.7. Soit E un espace vectoriel normé. Si la boule unité fermée B(0, 1) =
{x ∈ E : k x k ≤ 1} est compacte alors E est de dimension finie.
Preuve : Supposons que E est de dimension infinie. Alors nous allons construire une
suite (xn )n≥1 de E telle que pour tout n on a,
1
k xn k = 1, et d(xn , Vect(x1 , . . . , xn−1 )) ≥ . (∗)
2
En effet, prenons pour x1 un élément quelconque vérifiant k x1 k = 1. Ensuite, supposons
construits les éléments x1 , . . . , xn−1 . On définit F = Vect(x1 , . . . , xn−1 ), c’est un sous-
espace de dimension finie donc fermé dans E. Le lemme précédent s’applique (avec
ε = 1) et nous donne xn de norme 1 et tel que d(xn , F ) > 1/2.
Les espaces vectoriels normés de dimension finie 29

D’après (∗) nous avons une suite (xn )n≥1 de B(0, 1) qui vérifie k xn − xm k ≥ 1/2
pour tout n 6= m. En particulier aucune sous-suite de (xn )n≥1 ne converge, ce qui est
absurde car B(0, 1) est compacte. Cette contradiction montre que E est de dimension
finie.
30 ESPACES VECTORIELS NORMÉS

EXERCICES

Exercice .1 Soit E un e.v.n.


1◦ . Montrer que si x, y ∈ E \ {0} avec x 6= y alors
° °
max(k x k , k y k) ° x y °
kx − yk ≥ ¯ ¯ ° °
¯ k x k−k y k ¯ ° k x k k y k ° .

1 + ¯ k x−y k ¯

2◦ . En déduire que pour tous x, y ∈ E \ {0}


° °
1 ° x y °
k x − y k ≥ max(k x k , k y k) ° °
° kxk kyk °.

2
3◦ . Peut-on remplacer 1/2 par une constante plus grande ?

0
Exercice .2 Soient A, B deux parties d’un e.v.n E. Montrer que (A \ B)◦ = A \ B.

Exercice .3 Soient A un ouvert d’un e.v.n E, B une partie de E. Montrer que


A ∩ B = A ∩ B.

Exercice .4 Soient U, V deux ouverts d’un e.v.n E. Montrer que

U = V = E ⇐⇒ U ∩ V = E.

Exercice .5 Soit E un e.v.n, A et B deux parties de E.


0
1◦ . Comparer Fr (A), Fr (A), Fr (A).
2◦ . Montrer que Fr (A ∪ B) ⊂ Fr (A) ∪ Fr (B) ; a-t-on égalité en général ?

Exercice .6 Soit E un e.v.n, A une partie non vide de E et x ∈ E. Montrer que


d(x, A) = d(x, A) et d(x, A) = 0 ⇐⇒ x ∈ A.

Exercice .7 Soit E un e.v.n, F1 , F2 deux parties fermées de E. On suppose que


F1 ∩ F2 = Ø. Montrer qu’il existe deux ouverts O1 , O2 tels que F1 ⊂ O1 , F2 ⊂ O2 , et
O1 ∩ O2 = Ø.
Exercices 31

Exercice .8 Soit E un e.v.n sur le corps IK. Pour A et B deux parties de E et


λ ∈ IK on pose A + B = {a + b : (a, b) ∈ A × B} et λA = {λa : a ∈ A}. Montrer
0 0 0
A + B ⊂ A + B, A + B ⊂ (A + B)◦ , λA = λA et (λA)◦ = λA.

Exercice .9 Soit U un ouvert d’un e.v.n E. Montrer que


½ ¾
x+y
∀ (x, y) ∈ U × U, ∈U ⇐⇒ U est une partie convexe.
2

Exercice .10 Soit E un e.v.n.


1◦ . Soit (xn )n≥1 une suite d’éléments de E qui vérifie

1
∀ n ∈ IN, k xn+1 − xn k ≤ (∗)
2n

Montrer que cette suite est de Cauchy.


2◦ . Montrer que de toute suite de Cauchy de E on peut extraire une sous suite
vérifiant la propriété (∗).
3◦ . Montrer que E est complet si, et seulement si, toute suite de E vérifiant (∗) est
convergente.
4◦ . Montrer que E est complet si, et seulement si, toute série normalement
convergente est convergente.

Exercice .11 Soit (E, h , i) un espace préhilbertien de dimension finie, et soit C


une partie convexe fermée non vide de E. Montrer que pour tout x ∈ E il existe un
élément et un seul PC (x) ∈ C tel que k x − PC (x) k = d(x, C). (Indication: Montrer que
pour tout n il existe xn ∈ C tel que k x − xn k ≤ d(x, C) + 1/n. Montrer ensuite que
(xn )n≥1 est de Cauchy en utilisant l’identité du paraléllogramme et conclure.)

Exercice .12 Soit A une partie compacte d’un e.v.n E, et f : A → A une fonction
qui vérifie ∀ (x, y) ∈ A × A, x 6= y =⇒ k f (x) − f (y) k < k x − y k. Montrer qu’il
existe un unique point x ∈ A tel que f (x) = x. Montrer, de plus, que la suite récurrente
x0 ∈ A, xn+1 = f (xn ) converge vers le point fixe de f . Montrer, enfin, que la condition
de compacité est indispensable en considérant f : [1, +∞[−→ [1, +∞[: x 7→ x + 1/x.
32 ESPACES VECTORIELS NORMÉS

Exercice .13 Soit F une partie fermée non vide d’un espace de Banach E. Soit
k ∈]0, 1[ et ϕ : F −→ F une application k-contractante i.e.

∀ (x, y) ∈ F × F, k ϕ(x) − ϕ(y) k ≤ k k x − y k .

Pour x0 ∈ F , on considère la suite (xn )n≥0 définie par xn+1 = ϕ(xn ).


1◦ . Montrer que k xn+1 − xn k ≤ k n k x1 − x0 k pour n ≥ 0.
P
2◦ . Montrer que la série (xn+1 − xn ) converge.

3◦ . En déduire que la suite (xn )n≥0 converge vers une limite x ∈ F .

4◦ . Montrer que ϕ(x) = x et que si y ∈ F vérifie ϕ(y) = y alors y = x.


k
5◦ . Montrer k x − xn k ≤ k xn−1 − xn k.
1−k

Exercice .14 Soient E et F deux e.v.n, u : E −→ F une application linéaire.


Montrer que, si pour toute suite (xn )n≥0 de E tendant vers 0 la suite (u(xn ))n≥0 est
bornée dans F , alors u est continue.

Exercice .15 Soient E un e.v.n, f une forme linéaire non nulle sur E. Montrer

f est continue ⇐⇒ Ker f est fermé.

Exercice .16 Soient E un e.v.n. Montrer qu’il n’existe pas deux applications
linéaires continues u et v de E dans E telles que u◦v − v ◦u = id.

Exercice .17 Montrer que les formes linéaires suivantes sont continues et calculer
leurs normes:

Z 1
f (x)
T1 : C([−1, 1], IR) −→ IR : T1 (f ) = dx
−1 1 + x2
Z1
T2 : C([−1, 1], IR) −→ IR : T2 (f ) = sin(πx) f (x) dx
−1

L’espace C([−1, 1], IR) étant muni de la norme uniforme k f k∞ = sup | f (t) |.
t∈[−1,1]
Répondre à la même question lorsqu’on munit l’espace C([−1, 1], IR) de la norme
R1
k f k1 = | f (t) | dt.
−1
Exercices 33

Exercice .18 Soit ϕ : [0, 1] −→ [0, 1] définie par ϕ(x) = x2 . On considère


l’application
Ψ : C([0, 1], IR) −→ C([0, 1], IR) : f 7→ f ◦ϕ.

1◦ . Montrer que si l’on munit C([0, 1], IR) de la norme uniforme k . k∞ , alors Ψ
est continue, calculer sa norme.
2◦ . Montrer que si l’on munit C([0, 1], IR) de la norme k . k1 , alors Ψ n’est pas
continue.

Exercice .19 Soit E le sous espace vectoriel de C([0, 1], IR) formé des fonctions f
R1
telles que f (t) dt = 0 muni de la norme uniforme k . k∞ .
0
Montrer que pour tout f ∈ E, il existe un élément et un seul g = T (f ) ∈ E tel
que g 0 = f . Montrer que l’application T : E −→ E : f 7→ T (f ) est linéaire continue, et
calculer sa norme.

Exercice .20 Soit Pn l’espace vectoriel des polynômes de degré inférieur ou égal à
n, à coefficients réels.
1◦ . Montrer qu’il existe deux constantes α et β dans IR∗+ telles que pour tout
Pn
P (X) = ak X k ∈ Pn on ait
k=0

X
β sup | P (t) | ≤ | ak | ≤ α sup | P (t) |
t∈[0,1] 0≤k≤n t∈[0,1]

2◦ . Donner un exemple de fonction bornée et continuement dérivable sur IR telle


que f 0 ne soit pas bornée sur IR.
3◦ . Soit f ∈ C n+1 (IR). On suppose que f et f (n+1) sont bornées sur IR.
Montrer, en utilisant une formule de Taylor que
¯ ¯
¯Xn
f (k) (y) k ¯¯
¯
∃M ∈ IR∗+ ; ∀ y ∈ IR, ∀ x ∈ [0, 1], ¯ x ¯≤M
¯ k! ¯
k=0

En déduire que les dérivées f (k) (1 ≤ k ≤ n) de f sont bornées.

Exercice .21 Soient E un e.v.n, f une forme linéaire continue non nulle sur E.
Montrer
| f (x) |
∀ x ∈ E, d(x, Ker f ) = .
kf k
34 ESPACES VECTORIELS NORMÉS

Exercice .22 Soit E = C([0, 1], IR) l’algèbre des fonctions continues de [0, 1] dans
IR, munie de la norme uniforme k . k∞ . Montrer que tout isomorphisme d’algèbre
Φ : E −→ E est isométrique, i.e. ∀ f ∈ E, k Φ(f ) k∞ = k f k∞ .

Exercice .23 | × IN∗ , on suppose que


Soit (λ1 , m1 ), . . . , (λp , mp ) une suite finie de C
Xp
λ1 , . . . λp sont distincts, et on note n = mk .
k=1
1◦ . Montrer qu’il existe une, et une seule famille de polynômes (Pij ), 1 ≤ i ≤ p ;
(k)
0 ≤ j < mi de degrés inférieurs ou égals à n − 1, telle que Pij (λ` ) = δi` .δkj pour
1 ≤ ` ≤ p ; 0 ≤ k < m` .
2◦ . Exprimer en utilisant la famille {Pij } le reste de la division euclidienne de X m
p
Y
par P = (λk − X)mk .
k=1
3◦ . Soit A ∈ Mn ( C)
| telle que χA (X) = P (X). Montrer que

p m
X X i −1
∀m ∈ IN ∗
A m
= m(m − 1) · · · (m − j + 1)λm−j
i Pij (A).
i=1 j=0

4◦ . Soit A ∈ Mn ( C).
| On appelle rayon spectral de A le nombre

ρ(A) = max{| λ | : λ est une valeur propre de A}.

montrer l’équivalence

lim Am = 0 ⇐⇒ ρ(A) < 1.


m→∞

Exercice .24 On considère un espace vectoriel réel E. Une fonction p : E −→ IR


est dite sous-linéaire si, et seulement si,

∀ (x, y) ∈ E × E, p(x + y) ≤ p(x) + p(y)


∀ (λ, x) ∈ IR+ × E, p(λx) = λp(x)

1◦ . Montrer que si p est une fonction sous-linéaire de E dans IR, on a: p(0E ) = 0


et ∀ x ∈ E, −p(−x) ≤ p(x).

2 . On suppose que E est de dimension finie n. On donne p : E −→ IR sous-linéaire,
un sous-espace vectoriel F0 de E, et une forme linéaire f0 sur F0 majorée sur F0 par p.
On se propose de démontrer la proposition suivante:
Il existe f ∈ E ∗ qui prolonge f0 , et qui est majorée par p sur E.
Exercices 35

2◦ .a. Soit F l’ensemble des couples (g, G), où G est un sous-espace vectoriel de E
contenant F0 , et où g ∈ G∗ prolonge f0 et est majorée par p sur G. Vérifier qu’il existe
dans F un élément (g, G) tel que dim G soit maximum.
2◦ .b. Soit (g, G) ∈ F tel que dim G < n. Montrer que dim G n’est pas maximum
parmi les entiers dim H, où (h, H) ∈ F. Pour cela, soit z ∈ E \ G ; verifier que

A = sup (−p(−y − z) − g(y)) ≤ inf (p(x + z) − g(x)) = B.


y∈G x∈G

Choisir C ∈ [A, B]. Poser H = G ⊕ IRz. Pour x + λz ∈ H poser h(x + λz) = g(x) + λC
et montrer que (h, H) ∈ F.
2◦ .c. conclure.
3◦ . Enoncer le résultat démontré lorsque p(.) est une norme sur E.

Exercice .25 Soit E = C([0, 1], IR) l’espace vectoriel des fonctions continues de [0, 1]
dans IR. On définit sur E les trois applications :
Z 1 µZ 1 ¶1/2
2
N1 : f 7→ | f (t) | dt; N2 : f 7→ f (t) dt ; N∞ : f 7→ sup | f (t) | .
0 0 t∈[0,1]

1◦ . Montrer que ce sont des normes sur E.


2◦ . On considère les applications fn définies comme suit (pour n ∈ IN∗ )

 2nt si t ∈ [0, 1/(2n)]
fn (t) = 2 − 2nt si t ∈ [1/(2n), 1/n]

0 si t ∈ [1/n, 1]

En étudiant les suites (fn ) et ( nfn ), montrer que les normes N1 , N2 , et N∞ sont deux
à deux non équivalentes.
3◦ . On considère les applications gn définies comme suit (pour n ≥ 2)

1 pour t ∈ [0, 1/2]
gn (t) = 1 + n/2 − nt pour t ∈ [1/2, 1/2 + 1/n]

0 pour t ∈ [1/2 + 1/n, 1]
Montrer que (gn ) est une suite de Cauchy dans E muni de la norme N1 , et qu’elle ne
converge pas.
4◦ . On munit E de la norme N2 . Soit c ∈ [0, 1]. Montrer que l’application f 7→ f (c)
de E dans IR n’est pas continue.
36 ESPACES VECTORIELS NORMÉS

SOLUTIONS

Solution .1 1◦ . Si l’inégalité demandée n’est pas vérifiée, alors il exite x et y dans


E \ {0} tels que x 6= y, k x k ≥ k y k et
° °
° x y °
kx − yk + kxk − kyk <kxk° −
° kxk kyk °
°
° °
° kxk °
°
<°x − y°
kyk °
° °
° k x k °
<kx − yk + °
° y − y °
kyk °
¯ ¯
¯ k x k ¯¯
¯
<kx − yk + kyk¯1 −
kyk ¯
<kx − yk + kxk − kyk

ce qui est absurde. Cette contradiction démontre l’inégalité.


2◦ . Si x et y sont deux éléments distincts de E, alors | k x k − k y k | ≤ k x − y k
donc
|kxk − kyk|
1+ ≤2
kx − yk
ce qui démontre
° °
1 ° x y °
k x − y k ≥ max(k x k , k y k) °
° − °.
2 kxk kyk °

3◦ . En effet, 1/2 est la meilleure constante possible comme le montre l’exemple


suivant: E désigne l’espace vectoriel normé IR2 muni de la norme k (a, b) k = | a | + | b |.
X = (n, 0) et Y = (0, 1/n). Si
° °
° X Y °
k X − Y k ≥ c max(k X k , k Y k) ° °
° kX k kY k °,

1 1
alors n + ≥ 2cn ou bien 2c ≤ 1 + 2 . Ceci démontre que c ≤ 1/2 car n est arbitraire.
n n

0 0
Solution .2 Notons que A ⊂ A et E \ B ⊂ E \ B, alors A \ B est une partie ouverte
de A \ B, et par conséquent elle est contenue dans la plus grande partie ouverte de
0
A \ B, à savoir (A \ B)0. D’où A \ B ⊂ (A \ B)0. Inversement, d’une part (A \ B) ⊂ A
0
donc (A \ B)0 ⊂ A, et d’autre part comme (A \ B)0 est ouvert, alors (A \ B)0 ∩ B = Ø
Solutions 37

(car s’il existe un élément y dans l’intersection alors (A \ B)0 est un voisinage de y qui
doit rencontrer B car y ∈ B, c’est à dire Ø 6= (A \ B)0 ∩ B ⊂ (A \ B) ∩ B ce qui est
0
absurde). On conclut que A \ B = (A \ B)0.

Solution .3 Clairement, A ∩ B ⊂ A ∩ B alors A ∩ B ⊂ A ∩ B.


Inversement, soit x ∈ A ∩ B, si V est un voisinage de x alors V ∩ A est aussi
un voisinage de x (car A est un ouvert contenant x), et par conséquent ce voisinage
rencontre B (car x ∈ B), c’est à dire V ∩ (A ∩ B) 6= Ø. Ceci démontre que x ∈ A ∩ B.
On conclut que A ∩ B = A ∩ B.

Solution .4 Comme U ∩ V ⊂ U ⊂ E et U ∩ V ⊂ V ⊂ E, alors la condition


U ∩ V = E implique que U = V = E.
Inversement, en utilisant l’exercice 3. et le fait que U soit ouvert nous avons

U ∩ V = U ∩ V = U ∩ E = U = E.

0 0 0 0 0
Solution .5 1◦ . Comme A ⊂ A alors, A ⊂ A. Il en résulte que A \ A ⊂ A \ A, ou
0
bien Fr (A) ⊂ Fr (A). Avec inclusion qui peut être stricte comme le montre l’exemple
A=Q
/ dans IR.

0 0 0 0
D’autre part, comme A ⊂ A alors A ⊂ A. Il en résulte que A \ A ⊂ A \ A, ou
bien Fr (A) ⊂ Fr (A). Avec inclusion qui peut être stricte comme le montre l’exemple
A=Q
/ dans IR.

0
Enfin, Fr (A) et Fr (A) ne sont pas comparables en général, car si A est ouvert on
0 0
a Fr (A) ⊂ Fr (A), et si A est fermé alors Fr (A) ⊂ Fr (A), d’après ce qui précède.
0 0
2◦ . Notons que A ∪ B = A ∪ B et A ∪ B ⊂ (A ∪ B)0. alors

0 0 0 0
Fr (A ∪ B) = A ∪ B \ (A ∪ B)0 ⊂ A ∪ B \ (A ∪ B) ⊂ (A \ A) ∪ (B \ B) = Fr (A) ∪ Fr (B).

Si A =]0, 1] et B = [1, 2[ dans IR, alors Fr (A ∪ B) 6= Fr (A) ∪ Fr (B).

Solution .6 Comme A ⊂ A alors, pour tout x ∈ E, d(x, A) ≥ d(x, A). Inversement,


soit x ∈ E et soit ε > 0 alors, d’après la définition de d(x, A), il existe x
e ∈ A tel que
d(x, x
e) ≤ d(x, A) + ε/2. Mais la boule de centre x
e et de rayon ε/2 rencontre A, il en
résulte qu’il existe y ∈ A tel que d(e
x, y) ≤ ε/2. On conclut,

d(x, A) ≤ d(x, y) ≤ d(x, x x, y) ≤ d(x, A) + ε.


e) + d(e
38 ESPACES VECTORIELS NORMÉS

ε > 0 est arbitraire, donc d(x, A) ≤ d(x, A). Ceci démontre que

∀ x ∈ E, d(x, A) = d(x, A).

Si x ∈ A, alors 0 = d(x, A) = d(x, A). Inversement, si d(x, A) = 0, alors pour tout


n ≥ 1 il existe xn ∈ A tel que d(x, xn ) ≤ 1/n. x est, par conséquent, la limite d’une
suite d’éléments de A donc x ∈ A.

Solution .7 D’après l’exercice précédent, d(x, F1 ) + d(x, F2 ) > 0 pour tout x ∈ E


car F1 ∩ F2 = Ø.
On considère alors la fonction continue

d(x, F1 )
g : E −→ IR : x 7→ .
d(x, F1 ) + d(x, F2 )

Clairement, g −1 ({0}) = F1 et g −1 ({1}) = F2 . On pose alors, O1 = g −1 (] − ∞, 1/2[)


et O2 = g −1 (]1/2, +∞[). Les deux parties O1 et O2 sont ouvertes comme images
réciproques d’ouverts, disjointes, et F1 ⊂ O1 et F2 ⊂ O2 .

Solution .8 – Si x ∈ A + B, alors il existe a ∈ A et b ∈ B tels que x = a + b.


Il en résulte qu’il existe une suite (an )n∈IN dans A et une suite (bn )n∈IN dans B telles
que a = lim an et b = lim bn . Mais alors x = lim an + bn ∈ A + B. Par conséquent
n→∞ n→∞ n→∞
A + B ⊂ A + B.
– Si c ∈ E fixé, alors Tc : E −→ E : x 7→ x − c est clairement continue, si, par
conséquent, O est un ouvert alors c + O = Tc−1 (O) est aussi un ouvert. Il en résulte que
si C est une partie de E et si O est un ouvert alors
[
C +O = (c + O) est un ouvert.
c∈C

0 0 0 0
On conclut, que A + B est un ouvert contenu dans A + B, d’où A + B ⊂ (A + B)0.
– Enfin, si λ ∈ IK \ {0} alors hλ : E −→ E : x 7→ λx est un homéomorphisme. Il
0
s’en déduit que λA = λA et (λA)0 = λA.

Solution .9 Il est immédiat que si U est convexe alors

x+y
∀ (x, y) ∈ U × U, ∈ U.
2

Il s’agit donc de démontrer la réciproque.


Solutions 39

Soient (x, y) ∈ U × U . Notons Hn la propriété

k k
∀ k ∈ {0, 1, 2, . . . , 2n }, x + (1 − ) y ∈ U.
2n 2n

Remarquons que l’hypothèse est équivalente à H1 . Supposons que Hn est vraie. Alors
pour tout k ∈ {0, 1, . . . , 2n } on a

2k 2k k k
x + (1 − ) y = x + (1 − ) y ∈ U.
2n+1 2n+1 2n 2n

Et pour tout k ∈ {0, 1, . . . , 2n − 1} on a


µ ¶ µ ¶
2k + 1 2k + 1 1 k k 1 k+1 k+1
n+1
x + (1 − n+1 ) y = x + (1 − n ) y + x + (1 − n ) y ∈ U
2 2 2 2n 2 2 2n 2

où l’on a utilisé Hn et H1 . Il en résulte que Hn+1 est vraie. D’où

k k
∀ (x, y) ∈ U × U, ∀ n ∈ IN, ∀ k ∈ {0, 1, 2, . . . , 2n }, x + (1 − ) y ∈ U. (∗)
2n 2n

Soient (x, y) ∈ U × U . Comme U est un ouvert, alors il existe ε > 0 tel


que B(x, ε) ⊂ U et B(y, ε) ⊂ U . Choisissons n tel que k x − y k /2n· < ε. Soit ·
n k k+1
λ ∈]0, 1[, alors il existe un unique k ∈ {0, 1, . . . , 2 − 1} tel que λ ∈ n , n .
2 2
k
On note z = (λ − n ) (x − y). Clairement k z k < ε, alors x + z ∈ B(x, ε) ⊂ U et
2
y + z ∈ B(y, ε) ⊂ U . Il en résulte d’après (∗) que

k k
λ x + (1 − λ) y = n
(x + z) + (1 − n ) (y + z) ∈ U.
2 2

Ce qui démontre que U est convexe.

Solution .10 1◦ . En effet, soit ε > 0 il existe Nε tel que pout tout n ≥ Nε on a
1
n−1
< ε. Pour tout (m, n) ∈ IN2 on a
2
m−1
X ∞
X 1 1
m > n ≥ Nε =⇒ k xm − xn k ≤ k xk+1 − xk k ≤ k
= n−1 < ε.
2 2
k=n k=n

La suite (xn )n∈IN est donc de Cauchy.


2◦ . Soit (xn )n∈IN une suite de Cauchy dans E. Posons Xn = {xk : k ≥ n} et
δn = diam (Xn ). Il est immédiat de vérifier que

(xn )n∈IN est une suite de Cauchy ⇐⇒ lim δn = 0.


n→∞
40 ESPACES VECTORIELS NORMÉS

On pose alors,
ϕ(0) = min {n : δn ≤ 1}
1
ϕ(n) = min {k > ϕ(n − 1) : δk ≤ } pour n ≥ 1.
2n
Clairement, ϕ : IN −→ IN est strictement croissante, et comme ϕ(n + 1) > ϕ(n) alors
xϕ(n+1) ∈ Xϕ(n) et par conséquent
° °
° xϕ(n+1) − xϕ(n) ° ≤ δϕ(n) ≤ 1 .
2n
La sous-suite (xϕ(n) )n∈IN vérifie (∗).
3◦ . Supposons d’abord que E est complet. Si (xn )n∈IN est une suite vérifiant
(∗), alors elle est de Cauchy d’après 1◦ , et la complétude de E montre alors que
(xn )n∈IN converge. Inversement, Soit (xn )n∈IN une suite de Cauchy dans E. On peut
en extraire,d’après 2◦ , une sous-suite (xϕ(n) )n∈IN qui vérifie (∗) et par conséquent qui
converge. Mais une suite de Cauchy qui admet une sous-suite convergente est elle même
convergente. D’où l’équivalence demandée.
4◦ . Si E est complet alors la suite des sommes partielles de toute série normalement
convergente est de Cauchy donc elle converge. Inversement, si (xn )n∈IN est une suite
P
vérifiant (∗) alors la série (xn+1 − xn ) est une série normalement convergente dont la
convergence implique celle de la suite (xn )n∈IN . On conclut que E est complet d’après
3◦ .

Solution .11 Montrons d’abord l’unicité. Supposons qu’il existe un couple (y, z) ∈
C 2 tel que k x − y k = k x − z k = d(x, C). Alors d’après l’identité du parallèlogramme,
y+z
et notant que ∈ C par convexité,
2
2 2 2 2
k y − z k =2 k x − y k + 2 k x − z k − k (x − y) + (x − z) k
° °2
° y+z °
2 °
=4d (x, C) − 4 ° x − ° ≤ 4d2 (x, C) − 4d2 (x, C) = 0,
2 °
d’où y = z.
Posons pour simplifier δ = d(x, C). Considérons, pour tout n ≥ 1, un élément xn
de C tel que k x − xn k ≤ δ + 1/n. Alors en utilisant l’identité du parallèlogramme et
xn + xm
en notant que, par convexité, ∈ C, nous obtenons
2
2 2 2 2
k xn − xm k =2 k x − xn k + 2 k x − xm k − k (x − xn ) + (x − xm ) k
° °2
1 2 1 2 ° x n + xm °
=2(δ + ) + 2(δ + ) − 4 ° ° x − °
°
n m 2
µ ¶
1 2 1 2 2 1 1
≤2(δ + ) + 2(δ + ) − 4δ ≤ 2(2δ + 1) + .
n m n m
Solutions 41

Cette inégalité démontre que (xn )n∈IN∗ est une suite de Cauchy dans un espace vectoriel
normé complet (car de dimension finie), elle converge alors vers un élément PC (x).
Comme C est fermé alors PC (x) ∈ C, et l’inégalité
1
∀ n ∈ IN∗ , d(x, C) ≤ k x − xn k ≤ d(x, C) +
n
montre que k x − PC (x) k = d(x, C).

Solution .12 Montrons d’abord l’unicité du point fixe, c’est à dire que si le couple
2
(x, y) ∈ A vérifie f (x) = x et f (y) = y alors x = y. En effet,

x 6= y =⇒ k x − y k = k f (x) − f (y) k < k x − y k

ce qui est contradictoire.


La fonction ϕ : A −→ IR : t 7→ k t − f (t) k est continue comme composée de
fonctions continues. Et A est une partie compacte donc il existe x ∈ A tel que

ϕ(x) = min {ϕ(t) : t ∈ A}

Si ϕ(x) 6= 0 alors x 6= f (x) et par conséquent ϕ(f (x)) = k f (x) − f (f (x) k <
k x − f (x) k = ϕ(x), ce qui contredit la minimalité de ϕ(x). Il en résulte que ϕ(x) = 0
ou bien f (x) = x. On conclut que la fonction f admet un point fixe unique x ∈ A.
Posons δn = k x − xn k. La suite (δn )n∈IN est décroissante car

δn+1 = k x − xn+1 k = k f (x) − f (xn ) k ≤ k x − xn k ≤ δn .

Il en résulte qu’il existe ` ∈ IR+ tel que lim δn = `. Il reste à démontrer que
n→∞
` = 0. En effet, la compacité de A montre qu’il existe une suite extraite (xϕ(n) )n∈IN
de la suite (xn )n∈IN qui converge vers y ∈ A. Il résulte de lim xϕ(n) = y que
n→∞
lim δϕ(n) = k x − y k = `, et de la continuité de f que lim f (xϕ(n) ) = f (y) et puis
n→∞ n→∞
lim δϕ(n)+1 = k x − f (y) k = `. On conclut que k f (x) − f (y) k = ` = k x − y k ce qui
n→∞
implique que x = y et ` = 0, c’est à dire lim xn = x.
n→∞
La condition de compacité est indispensable. En fait si l’on considère
1
f : [1, +∞[−→ [1, +∞[: x 7→ x +
x
on voit que l’équation f (x) = x n’a pas de solution, bien que, pour tout (x, y) ∈
[1, +∞[×[1, +∞[,
µ ¶
1
x 6= y =⇒ | f (x) − f (y) | = | x − y | 1 − < |x − y|.
xy
42 ESPACES VECTORIELS NORMÉS

Solution .13 1◦ . C’est une récurrence immédiate.


P
2◦ . D’après 1◦ , la série (xn+1 − xn ) est normalement convergente, donc elle est
convergente car E est complet.
n−1
X

3 . Comme xn = x0 + (xk+1 − xk ), alors 2◦ montre que (xn )n≥0 converge vers
k=0
x dans E. Mais xn ∈ F , pour tout n ∈ IN, et F est fermé donc x ∈ F .
4◦ . ϕ est continue car elle est lipschitzienne. L’égalité ϕ(xn ) = xn+1 , valable pour
tout n, montre alors que ϕ(x) = x.
Si ϕ(y) = y alors k x − y k = k f (x) − f (y) k ≤ k k x − y kd’où (1 − k) k x − y k ≤ 0
ce qui démontre que x = y.
5◦ . En effet,

k x − xn k ≤ k x − xn+1 k + k xn+1 − xn k
≤ k f (x) − f (xn ) k + k f (xn ) − f (xn−1 ) k
≤k k x − xn k + k k xn − xn−1 k

k
ce qui donne k x − xn k ≤ k xn − xn−1 k.
1−k

Solution .14 En effet, si u n’est pas continue, alors pour tout n ≥ 1 il existe yn ∈ E
tel que k yn k = 1 et k u(yn ) k ≥ n2 . On pose alors xn = yn /n, il s’en suit que (yn )n≥1
tend vers 0 et (k u(yn ) k)n≥1 tend vers ∞.

Solution .15 Clairement si f est continue alors Ker f = f −1 ({0}) est fermé.
Inversement, supposons H = Ker f fermé. Comme f est non nulle alors il existe
a ∈ E tel que f (a) = 1. L’ensemble a + H est un fermé qui ne contient pas 0 donc
y
il existe ε > 0 tel que B(0, ε) ∩ (a + H) = Ø. Soit y ∈ E \ H alors − a ∈ H et
f (y)
y y
par conséquent ∈ a + H. Il en résulte que ∈
/ B(0, ε), ce qui se traduit par
f (y) f (y)
k y k ≥ ε | f (y) |. On en déduit que
1
∀ y ∈ E, | f (y) | ≤ kyk
ε
c’est à dire que f est continue.

Solution .16 Supposons que de telles applications u et v existent. Montrons par


récurrence, que pour tout n ∈ IN∗ , l’on a un ◦v − v ◦un = nun−1 . En effet, c’est vrai par
hypothèse si n = 1. Si c’est vrai pour n alors,

un+1 ◦v − u◦v ◦un = nun , et u◦v = I + v ◦u


Solutions 43

d’où un+1 ◦v − v ◦un+1 = (n + 1)un . Il en résulte que pour tout n


° ° ° °
(n + 1) k un k ≤ ° un+1 ◦v − v ◦un+1 ° ≤ 2 k v k ° un+1 ° ≤ 2 k u k k v k k un k

ou bien, (n + 1 − 2 k u k k v k) k un k ≤ 0. En choisissant n + 1 > 2 k u k k v k, on trouve


k un k = 0 c’est à dire que l’ensemble {n ∈ IN∗ : un = 0} n’est pas vide. Soit
k = min {n ∈ IN∗ : un = 0}. Si k > 1, alors la relation kuk−1 = uk ◦v − v ◦uk = 0
montre que uk−1 = 0 ce qui contredit la minimalité de k. Alors k = 1 et par conséquent
u = 0 mais dans ce cas on a I = u◦v − v ◦u = 0 ce qui est absurde sauf si dim E = 0
ceci représente le cas trivial.

Solution .17 Notons E∞ l’espace vectoriel normé C([−1, 1], IR) muni de la norme
uniforme.
– Il est clair que, pour tout f ∈ C([−1, 1], IR),
Z 1 Z 1
| f (x) | 1 π
| T1 (f ) | ≤ dx ≤ k f k∞ dx = k f k∞ .
−1 1 + x2 −1 1+x 2 2

D’où T1 : E∞ → IR est continue et k T1 : E∞ → IR k ≤ π/2. D’autre part, si f = 1I la


fonction constante égale à 1 sur [−1, 1], alors

π
= T1 (1I) ≤ k T1 : E∞ → IR k k 1I k∞ = k T1 : E∞ → IR k .
2

Soit k T1 : E∞ → IR k = π/2.
– Il est aussi clair que, pour tout f ∈ C([−1, 1], IR),
Z 1 Z 1
4
| T2 (f ) | ≤ | f (x) | | sin πx | dx ≤ k f k∞ | sin πx | dx = k f k∞ .
−1 −1 π

D’où T2 : E∞ → IR est continue et k T2 : E∞ → IR k ≤ 4/π. D’autre part, pour n ∈ IN∗ ,


on considère fn ∈ E∞ , définie par

1
fn
 1

 1 si x ∈ [ , 1]

 n


1 1 −1
fn (x) = nx si x ∈ [− , ]

 n n 1/n 1



 1
−1 si x ∈ [−1, − ]
n

−1
44 ESPACES VECTORIELS NORMÉS

Alors
Z 1 Z 1 Z 1/n
4
− T2 (fn ) = 2 sin πt dt − 2 fn (t) sin πt dt = 2 (1 − nt) sin πt dt
π 0 0 0

D’où
4 2
0≤ − T2 (fn ) ≤
π n
car ∀ t ∈ [0, 1/n], 0 ≤ (1 − nt) sin πt ≤ 1. Il en résulte que pour tout n ∈ IN∗ ,

4 2 4
− ≤ T2 (fn ) ≤ k T2 : E∞ → IR k k fn k∞ = k T2 : E∞ → IR k ≤ .
π n π
4
On conclut que k T2 : E∞ → IR k = .
π
Notons E1 l’espace vectoriel normé C([−1, 1], IR) muni de la norme k k1 .
– Il est clair que, pour tout f ∈ C([−1, 1], IR),
Z 1 Z 1
| f (x) |
| T1 (f ) | ≤ dx ≤ | f (x) | dx = k f k1 .
−1 1 + x2 −1

D’où T1 : E1 → IR est continue et k T1 : E1 → IR k ≤ 1. D’autre part, pour n ∈ IN∗ , on


considère fn ∈ E1 , définie par

n
 1

 0 si x∈[ , 1]

 n


1 1
fn (x) = n − n2 | x | si x ∈ [− , ] fn

 n n



 1
0 si x ∈ [−1, − ]
n

−1 1/n 1
Alors
Z 1/n Z 1
2 1 (1 − t)t2
k fn k1 − T1 (fn ) = 2 (n − n x)(1 − ) dx = 2 dt
0 1 + x2 0 n2 + t2

(1 − t)t2 (1 − t)t2
D’où, en utilisant, pour t ∈ [0, 1], la majoration ≤ ,
n2 + t 2 n2

1
0 ≤ 1 − T1 (fn ) ≤
6n2

Il en résulte que pour tout n ∈ IN∗ ,

1
1− ≤ T1 (fn ) ≤ k T1 : E1 → IR k k fn k1 = k T1 : E1 → IR k ≤ 1.
6n2
Solutions 45

On conclut que k T1 : E1 → IR k = 1.
– Enfin, il est immédiat que, pour tout f ∈ C([−1, 1], IR),
Z 1 Z 1
| T2 (f ) | ≤ | f (x) | | sin πx | dx ≤ | f (x) | dx = k f k1 .
−1 −1

D’où T2 : E1 → IR est continue et k T2 : E1 → IR k ≤ 1. D’autre part, pour n ∈ IN∗ , on


considère fn ∈ E1 , définie par

 n
 1 1

 0 si x ∈ [ + , 1]

 2 n

 ¯ ¯ ¯ ¯
¯ 1 ¯¯ ¯ ¯
2 ¯
fn (x) = n − n ¯ x − ¯ si ¯x − 1 ¯ ≤ 1 fn

 2 ¯ 2¯ n



 1 1
 0 si x ∈ [−1, − ]
2 n 1
−1 2 1
1 1 1 1
2−n 2+n

Alors Z 1/2+1/n
k fn k1 − T2 (fn ) =2 (n − n2 | x − 1/2 |)(1 − sin πx) dx
1/2−1/n
Z 1
πu
= (1 − | u |) (1 − cos ) du
−1 n
Où l’on a effectué le changement de variable n(x − 1/2) = u. En utilisant, la majoration
0 ≤ 1 − cos(πu/n) ≤ π 2 u2 /2n2 , nous obtenons

π2
0 ≤ 1 − T2 (fn ) ≤
12n2
Il en résulte que pour tout n ∈ IN∗ ,

π2
1− ≤ T2 (fn ) ≤ k T2 : E1 → IR k k fn k1 = k T2 : E1 → IR k ≤ 1.
12n2
On conclut que k T2 : E1 → IR k = 1.

Solution .18 1◦ . Notons E∞ l’espace vectoriel normé C([0, 1], IR) muni de la norme
uniforme. Comme x 7→ x2 est une bijection de [0, 1] alors il est immédiat que, pour tout
f ∈ C([0, 1], IR),
¯ ¯
k Ψ(f ) k∞ = sup ¯ f (x2 ) ¯ = sup | f (x) | = k f k∞ .
x∈[0,1] x∈[0,1]

ceci démontre que Ψ : E∞ → E∞ est une application isométrique donc elle est continue
et k Ψ : E∞ → E∞ k = 1.
46 ESPACES VECTORIELS NORMÉS

2◦ . Notons E1 l’espace vectoriel normé C([0, 1], IR) muni de la norme k k1 .


Considérons pour n ∈ IN∗ , la fonction fn ∈ E1 définie par
2n

 1

 0 si x ∈ [ , 1]
n fn
fn (x) =

 2n − 2n2 x si 1
x ∈ [0, ]
n

1
n 1

Alors k fn k1 = 1 et
Z √
1/ n
4√
k Ψ(fn ) k1 = (2n − 2n2 x2 ) dx = n.
0 3

k Ψ(f ) k1
Ce qui démontre que sup = ∞ et Ψ : E1 → E1 n’est pas continue.
f ∈E1 \{0} k f k1

Solution .19 Montrons d’abord l’unicité de g. En effet s’il existe g ∈ E telle que
g 0 = f , alors il existe une constante c telle que
Z x
∀ x ∈ [0, 1], g(x) = c + f (t) dt.
0

Mais g ∈ E implique que


Z 1 Z 1 µZ x ¶
0= g(x) dx = c + f (t) dt dx
0 0 0
· Z x ¸1 Z 1
=c + x f (t) dt − xf (x) dx
0 0 0
Z 1
=c − xf (x) dx
0

Où l’on a effectué une intégration par parties et utilisé le fait que f ∈ E. Alors,
Z x Z 1
∀ x ∈ [0, 1], g(x) = f (t) dt + tf (t) dt
0 0

Ceci démontre l’unicité de g.


Pour démontrer l’existence, il suffit de poser pour f ∈ E,
Z x Z 1
T (f )(x) = f (t) dt + tf (t) dt (∗)
0 0
Solutions 47

Il est immédiat que T (f ) est de classe C 1 et que (T (f ))0 = f . D’autre part, un calcul
simple, (similaire à celui qui a servi pour déterminer c), montre que T (f ) ∈ E.
La formule (∗) permet immédiatement de voir que T est linéaire. L’idée gagnante
est de remarquer que l’on peut écrire (∗) sous la forme
Z x Z 1 Z 1
1
T (f )(x) = f (t) dt + tf (t) dt − (x + ) f (t) dt
0 0 2 0
ou bien Z x Z 1
1 1
T (f )(x) = (t − x + )f (t) dt + (t − x − )f (t) dt
0 2 x 2
soit finalement,
Z −x+1/2 Z 1/2
1 1
T (f )(x) = uf (u + x + ) du + uf (u + x − ) du (∗∗)
−1/2 2 −x+1/2 2
De cette relation on obtient immédiatement
ÃZ Z !
−x+1/2 1/2
∀ x ∈ [0, 1], | T (f )(x) | ≤ | u | du + | u | du k f k∞
−1/2 −x+1/2
ÃZ !
1/2
1
≤ | u | du k f k∞ = k f k∞
−1/2 4
1
Il en résulte que k T (f ) k∞ ≤k f k∞ pour tout f ∈ E. L’application linéaire T : E →
4
E est par conséquent continue et de norme k T : E → E k ≤ 1/4.
Considérons pour tout entier n ≥ 2, la fonction fn ∈ E définie par
1

 1 1

 1 si x ∈ [0, − ] fn

 2 n

 1 1
2+n
1 1 1 1 1
fn (x) = n( − x) si x∈[ − , + ]

 2 2 n 2 n 1 1
2−n 1



 1 1
−1 si x∈[ + , 1]
2 n

−1

Alors fn ∈ E et k fn k∞ = 1. D’autre part, un calcul simple montre que


1 1 1 1
T (fn )( ) = − + 2.
2 4 2n 3n
Alors, pour tout n ≥ 2,
1 1 1 1
− ≤ T (fn )( ) ≤ k T (fn ) k∞ ≤ k T : E → E k k fn k∞ ≤ k T : E → E k ≤ .
4 2n 2 4
Il en résulte que k T : E → E k = 1/4.
48 ESPACES VECTORIELS NORMÉS

Solution .20 1◦ . Considérons sur Pn les deux normes


n
X n
X
k
N1 (P ) = sup | P (t) | , N2 ( ak X ) = | ak | .
t∈[0,1] k=0 k=0

Comme Pn est de dimension finie alors les deux normes N1 et N2 sont équivalentes et
par conséquent il existe (α, β) ∈ (IR∗+ )2 tels que βN1 ≤ N2 ≤ αN1 . Ce qui démontre le
résultat.
2◦ . Il suffit de considérer f (x) = sin x2 . ¯ ¯
◦ ¯ (n+1) ¯
3 . Posons M0 = sup | f (x) | et Mn+1 = sup ¯ f (x) ¯. D’après la formule de
x∈IR x∈IR
2
Taylor-Lagrange on a, pour tout (y, x) ∈ IR ,
¯ ¯ ¯ n+1 ¯
¯ n
X f (k) (y) k ¯¯ ¯x ¯ ¯ ¯
¯ ¯ (n+1) ¯
¯ f (y + x) − x ¯≤ sup ¯ f (y + tx) ¯ ,
¯ k! ¯ (n + 1)! t∈[0,1]
k=0

donc pour tout y ∈ IR et tout x ∈ [0, 1],


¯ n ¯
¯ X f (k) (y) ¯ Mn+1
¯ ¯
¯ xk ¯ ≤ M0 + .
¯ k! ¯ (n + 1)!
k=0

d’où le résultat avec M = M0 + Mn+1 /(n + 1)!.


n
X f (k) (y)

En utilisant le résultat de 1 appliqué au polynôme P = xk (y étant
k!
k=0
provisoirement fixé) on obtient

1 ¯¯ (k) ¯
n
X ¯
∀ y ∈ IR, ¯ f (y) ¯ ≤ αM.
k!
k=0

Ce qui démontre que pour tout k, (1 ≤ k ≤ n) la dérivée f (k) est bornée sur IR.

Solution .21 Soit ε > 0, il existe a ∈ E tel que k a k = 1 et (1 + ε) | f (a) | ≥ k f k


f (x)
(d’après la définition de k f k). Soit x ∈ E alors x − a ∈ Ker f et par conséquent
f (a)
° ° ¯ ¯
° f (x) ° ¯ f (x) ¯ | f (x) |
d(x, Ker f ) ≤ ° ° ¯ ¯
° x − (x − f (a) a) ° = ¯ f (a) ¯ ≤ (1 + ε) k f k

Nous avons donc démontré


| f (x) |
∀ x ∈ E, ∀ ε > 0, d(x, Ker f ) ≤ (1 + ε) .
kf k

Alors
| f (x) |
∀ x ∈ E, d(x, Ker f ) ≤ .
kf k
Solutions 49

Inversement, soit x ∈ E. Alors

∀ y ∈ Ker f, | f (x) | = | f (x − y) | ≤ k f k k x − y k .

Ce qui permet d’écrire

| f (x) |
≤ inf {k x − y k : y ∈ Ker f } = d(x, Ker f ).
kf k

D’où le résultat.

Solution .22 Soit 1I la fonction constante égale à 1. Comme 1I2 = 1I alors


(Φ(1I))2 = Φ(1I), donc pour tout x ∈ [0, 1] on a Φ(1I)(x) ∈ {0, 1}. Mais Φ(1I) est continue
donc vérifie le théorème des valeurs intermédiaires et par conséquent Φ(1I) = 0 ou
Φ(1I) = 1I, mais Φ est injective d’où Φ(1I) 6= 0. On conclut que Φ(1I) = 1I.
D’autre part, Soit f ∈ E telle que f ≥ 0. Alors il existe une fonction g ∈ E telle
que f = g 2 , et par conséquent Φ(f ) = (Φ(g))2 ≥ 0. Il en résulte que

∀ (f, g) ∈ E × E, f ≥ g =⇒ Φ(f ) ≥ Φ(g). (∗)

Soit f ∈ E, alors − k f k∞ 1I ≤ f ≤ k f k∞ 1I. Il en résulte d’après (*) que

− k f k∞ Φ(1I) ≤ Φ(f ) ≤ k f k∞ Φ(1I)

En utilisant, Φ(1I) = 1I, on obtient

∀ x ∈ [0, 1], − k f k∞ ≤ Φ(f )(x) ≤ k f k∞

ce qui démontre que k Φ(f ) k∞ ≤ k f k∞ , pour tout f ∈ E.


° °
Mais Φ−1 est aussi un isomorphisme d’algèbre de E, alors ° Φ−1 (f ) °∞ ≤ k f k∞ ,
pour tout f ∈ E. En utilisant cette inégalité pour Φ(f ) au lieu de f , on obtient k f k∞ ≤
k Φ(f ) k∞ pour tout f ∈ E. Ce qui démontre que ∀ f ∈ E, k Φ(f ) k∞ = k f k∞ .

Solution .23 Notons ∆ = {(i, j) : 1 ≤ i ≤ p, 0 ≤ j < mi }. D’après l’hypothèse


Card (∆) = n.
1◦ . Soient C
| ∆ l’espace vectoriel des familles de C
| indexées par ∆, et E
n−1 l’espace

vectoriel des polynômes complexes de degré ne dépassant pas n − 1. Notons

| ∆
ϕ : En−1 −→ C : P 7→ (P (k) (λ` ))(`,k)∈∆ .

Clairement ϕ est linéaire. De plus si Q ∈ Ker ϕ alors, pour tout k ∈ {1, 2, . . . , p}, λk est
une racine d’ordre mk de Q ; c’est à dire (λk −X)mk divise Q pour tout k ∈ {1, 2, . . . , p}.
50 ESPACES VECTORIELS NORMÉS
p
Y
Comme λ1 , . . . , λp sont distincts, alors Q est divisible par P = (λk − X)mk . Mais
k=1
deg P = n et deg Q < n. Il en résulte que Q = 0 et par conséquent ϕ est injective.
| ∆ . D’où ϕ est un isomorphisme
D’autre part dim En−1 = n = Card (∆) = dim C
d’espaces vectoriels.
La famille (Pij )(i,j)∈∆ est en fait l’image de la base canonique de | ∆ par
C
l’isomorphisme ϕ−1 . Ce qui démontre l’existence et l’unicité de cette famille.
2◦ . Le reste Rm de la division euclidienne de X m par P est un élément de En−1 et
donc s’exprime sur la base (Pij )(i,j)∈∆ de En−1 . On peut alors écrire
p m
X Xi −1
m (m)
X = Qm (X)P (X) + Rm (X) = Qm (X)P (X) + µij Pij . (∗)
i=1 j=0

En utilisant le fait que λk est une racine d’ordre mk de Qm (X)P (X) nous obtenons,
pour 0 ≤ ` ≤ mk ,
p m
X X i −1
(m) (`) (m)
m(m − 1) · · · (m − ` + 1)λm−`
k = µij Pij (λk ) = µk` .
i=1 j=0

Ce qui démontre que


p m
X Xi −1

Rm = m(m − 1) · · · (m − j + 1)λm−j
i Pij .
i=1 j=0

3◦ . En utilisant (∗) et le fait que XA (A) = 0 nous arrivons à Am = Rm (A) ou bien,


p m
X X i −1

∀ m ∈ IN A m
= m(m − 1) · · · (m − j + 1)λm−j
i Pij (A).
i=1 j=0

4◦ . Munissons C
| n d’une norme k k, et notons ||| ||| la norme d’applications
| n, k
linéaires de ( C | n, k
k) dans ( C k). Nous identifions matrices de Mn ( C),
| et
| n.
applications linéaires sur C
Supposons d’abord que ρ(A) ≥ 1, alors il existe λ une valeur propres de A telle que
| λ | ≥ 1. Soit x un vecteur propre associé à λ. Comme Ax = λx alors Am x = λm x et

k x k ≤ k λm x k ≤ k Am x k = ||| Am ||| k x k

donc ||| Am ||| ≥ 1 pour tout m, et la suite (Am )m∈IN ne tend pas vers 0.
Inversement, supposons que ρ(A) < 1, alors d’après 3◦ , pour tout m > n, nous
avons ||| Am ||| ≤ Kmn (ρ(A))m−n , avec
p m
X X i −1
n−j
K= | λi | ||| Pij (A) |||.
i=1 j=0
Solutions 51

Il en résulte que lim Am = 0.


m→∞
Remarque: En utilisant la relation de 3◦ nous pouvons écrire, pour t ∈ C,
|

X∞ p mi −1
tm m X X
exp(tA) = A = tj eλi t Pij (A) = Et (A).
m=0
m! i=1 j=0

Avec Et est l’unique polynôme de degré inférieur ou égal à n − 1 déterminé par les
(k)
conditions Et (λ` ) = tk eλ` t pour tout (`, k) ∈ ∆.

Solution .24 1◦ . En prenant λ = 0, on obtient p(0E ) = p(λx) = λp(x) = 0. Si


x ∈ E on a 0 = p(x + (−x)) ≤ p(x) + p(−x) d’où le résultat.
2◦ .a. Notons que (f0 , F0 ) ∈ F danc F n’est pas vide. Soit Λ = {dim F : (f, F ) ∈
F}. Alors Λ est une partie non vide de IN contenue dans {0, 1, . . . , dim E}. Par
conséquent Λ contient un plus grand élément, c’est à dire il existe (g, G) ∈ F tel que
dim G = max Λ.
2◦ .b. Supposons que dim G < n. Alors il existe z ∈ E \ G. Pour tout (x, y) ∈ G × G,

g(y) − g(x) = g(y − x) ≤ p(y − x) = p(y + z − y − x) ≤ p(y + z) + p(−y − z).

Il en résulte que

∀ (x, y) ∈ G × G, (−p(−y − z) − g(y)) ≤ (p(x + z) − g(x)).

Ou bien,
A = sup (−p(−y − z) − g(y)) ≤ inf (p(x + z) − g(x)) = B.
y∈G x∈G

Soit C ∈ [A, B]. Notons H = G ⊕ IRz, comme z ∈


/ G alors dim H = 1 + dim G.
Pour x + λz ∈ H, posons h(x + λz) = g(x) + λC. Par construction h est une forme
linéaire sur H,(i.e. h ∈ H ∗ ). Montrons que h est majorée sur H par p. En effet, soit
λ > 0, l’inégalité C ≤ B montre que

x x
∀ x ∈ G, C ≤ p( + z) − g( )
λ λ

ce qui démontre

∀ x ∈ G, h(x + λz) = g(x) + λC ≤ p(x + λz).

Supposons maintenant que λ = −µ < 0, l’inégalité C ≥ A montre que

x x
∀ x ∈ G, C ≥ −p( − z) + g( ),
µ µ
52 ESPACES VECTORIELS NORMÉS

ce qui démontre

∀ x ∈ G, h(x + λz) = g(x) − µC ≤ p(x − µz) = p(x + λz).

Il en résulte que ∀ t ∈ H, h(t) ≤ p(t). On conclut que (h, H) ∈ F.


2◦ .c. On conclut que max Λ = n, car nous avons démontré en 2◦ .b que pour tout
(g, G) ∈ F si dim G < n, alors dim G < max Λ. Ceci démontre le résultat énoncé en 2◦ .
3◦ . Dire qu’une forme linéaire g ∈ G∗ est majorée par k k sur G est équivalent à
dire que la norme de g en tant qu’application linéaire est majorée par 1, (i.e. k g k ≤ 1).
On conclut que l’assertion suivante est vraie:“ Soient E un espace vectoriel normé de
dimension finie, et F0 un sous espace vectoriel de E. Toute forme linéaire f0 sur F0
admet un prolongement f à E de même norme.”

Solution .25 1◦ . Résultat immédiat et laissé au lecteur. On remarque en particulier,


d’après l’inégalité de Schwarz que N1 ≤ N2 ≤ N∞ .
2◦ . Il suffit d’après l’inégalité précédente de démontrer que N∞ et N2 ne sont pas
équivalentes ainsi que N1 et N2 ne sont pas équivalentes. Or si
 · ¸
 1

 2nt si t ∈ 0,



 · 2n ¸
1 1
fn (t) = 2 − 2nt si t ∈ ,

 · 2n ¸n



 1
0 si t ∈ ,1
n
1 1
alors N1 (fn ) = , N2 (fn ) = √ et N∞ (fn ) = 1. La suite (fn )n≥1 tend vers 0 dans
2n 3n
(E, N2 ) et ne tend pas vers 0 dans (E, N∞ ), donc les deux normes N∞ et N2 ne sont

pas équivalentes. D’autre part, La suite ( nfn )n≥1 tend vers 0 dans (E, N1 ) et ne tend
pas vers 0 dans (E, N2 ) donc les deux normes N1 et N2 ne sont pas équivalentes.
3◦ . Il est immédiat de voir que
1 1 1
m > n ≥ 2 =⇒ N1 (gm − gn ) = − ≤ .
2n 2m 2n
La suite (gn )n≥2 est, par conséquent, de Cauchy dans (E, N1 ). Supposons que cette
suite converge vers g ∈ E. Pour tout n ≥ 2
Z 1/2 Z 1
| 1 − g(t) | dt + | g(t) | dt ≤ N1 (gn − g).
0 1/2+1/n

Donc en faisant tendre n vers l’infini il arrive


Z 1/2 Z 1
| 1 − g(t) | dt + | g(t) | dt = 0.
0 1/2
Solutions 53

Mais ceci contredit la continuité de g en 1/2. La suite (gn )n≥2 n’est donc pas conver-
gente.
4◦ . Posons,
r pour tout n ∈ IN∗ , hn (x) = max(0, 1 − n | x − c |). On vérifie que
2
N2 (hn ) ≤ , donc (hn )n≥1 tend vers 0 dans (E, N2 ). Mais pour tout n ≥ 1 on a
n
hn (c) = 1. On conclut que l’application f 7→ f (c) de E dans IR n’est pas continue.

OKMRAN
OUBA
SUITES ET SÉRIES DE FONCTIONS

Dans ce chapitre IK désigne IR ou C


|

I. Généralités

Définition : Soient A un ensemble non vide, et F un espace vectoriel normé.


— On dit qu’une suite d’applications (fn )n∈IN de A dans F converge simplement
vers une application f : A −→ F si, et seulement si, pour tout x ∈ A, la suite
(fn (x))n∈IN de F converge vers f (x). On appelle f la limite simple de la suite
(fn )n∈IN .
— On dit qu’une suite de fonctions (fn )n∈IN de A dans F converge uniformément
vers une application f : A −→ F si, et seulement si, la suite de terme général
µn = sup k fn (x) − f (x) kF de IR converge vers 0. On appelle f la limite uniforme
x∈A
de la suite (fn )n∈IN .

Remarque : Il est immédiat de voir que si une suite de fonctions converge uniformément
vers une certaine application, alors elle converge simplement vers cette même applica-
tion.

Exemple : Étudions la convergence simple et uniforme de la suite de fonctions


(fn )n∈IN∗ :
fn : IR+ −→ IR : t 7→ nα te−nt . (α ∈ IR+ )

Il est immédiat de voir que cette suite converge simplement vers l’application iden-
tiquement nulle notée 0. Si la suite (fn )n∈IN∗ converge uniformément alors d’après
la remarque précédente sa limite uniforme ne peut être que 0. Calculons alors
µn = sup | fn (x) |. Or le tableau de variation suivant
x∈IR+

x 0 1/n +∞
fn0 (x) + 0 −
fn (x) 0 % _ & 0
1
montre que µn = fn ( ) = nα−1 e−1 . Alors, si α < 1 on a lim µn = 0, et la suite de
n n→∞
fonctions (fn )n∈IN∗ converge uniformément vers 0. Par contre, si α ≥ 1 la suite (fn )n∈IN∗
ne converge pas uniformément.
2 Suites et séries de fonctions

Remarque : Soit (fn )n∈IN une suite de fonctions de A dans F qui converge simplement
vers une application f : A −→ F . Si l’on trouve une suite (ξn )n∈IN de A telle que
(k fn (ξn ) − f (ξn ) k)n∈IN ne tend pas vers 0 alors la suite de fonctions (fn )n∈IN ne
converge pas uniformément.

Exemple : Étudions la convergence simple et uniforme de la suite de fonctions


(fn )n∈IN∗ :
nt
fn : IR∗+ −→ IR : t 7→ .
1 + nt
Il est immédiat de voir que cette¯ suite converge¯ simplement vers l’application identique-
¯ 1 1 ¯ 1
ment égale à 1 notée 1I. Mais ¯¯ fn ( ) − 1I( ) ¯¯ = pour tout n donc la suite (fn )nIN∗
n n 2
ne converge pas uniformément.

Définition : Soient A une partie non vide d’un espace vectoriel normé E, et F
un espace vectoriel normé. On dit qu’une suite de fonctions (fn )n∈IN de A dans F
converge uniformément sur tout compact vers une application f : A −→ F si, et
seulement si, pour toute partie compacte non vide K ⊂ A, la suite de terme général
µn (K) = sup k fn (x) − f (x) kF de IR converge vers 0.
x∈K
Notons que la convergence uniforme implique la convergence uniforme sur tout
compact qui à son tour implique la convergence simple. La réciproque est évidemment
fausse.

Exemple : Étudions la convergence de la suite de fonctions (fn )n∈IN∗ :



0 si x=0
fn : IR −→ IR : x 7→
 x2 sin 1 si x 6= 0
nx

Cette suite converge uniformément sur tout compact vers la fonction identiquement
nulle. En effet, soit K un compact de IR. Il existe M > 0 tel que K ⊂ [−M, M ].
¯ ¯
¯ 1 ¯¯ x2 M
sup | fn (x) | ≤ sup 2¯
x ¯ sin ≤ sup = .
x∈K 0<| x |≤M nx ¯ 0<| x |≤M | nx | n

Donc, lim sup | fn (x) | = 0. Par contre, la suite (fn )n∈IN∗ ne converge pas uni-
n→∞ x∈K
formément sur IR car lim | fn (n) | = 1.
n→∞
Continuité de la limite d’une suite de fonctions 3

Définition : Soient A un ensemble non vide et F un espace vectoriel normé. On dit


qu’une suite de fonctions (fn )n∈IN de A dans F est uniformément de Cauchy si, et
seulement si,
½ ¾
2 n ≥ Nε
∀ ε > 0, ∃Nε ∈ IN, ∀(n, m) ∈ IN , =⇒ sup k fn (x) − fm (x) kF ≤ ε.
m ≥ Nε x∈A

Soient A un ensemble non vide, F un espace vectoriel normé, et (fn )n∈IN une suite
de fonctions de A dans F . Les propriétés suivantes sont immédiates:
— Si la suite (fn )n∈IN converge uniformément alors elle est uniformément de Cauchy.
— Si la suite (fn )n∈IN est uniformément de Cauchy, et si elle converge simplement
alors elle converge uniformément.
— Si F est complet, alors toute suite uniformément de Cauchy est uniformément
convergente.

II. Continuité de la limite d’une suite de fonctions

Dans cette section E et F sont deux espaces vectoriels normés.


Théorème II.1. Soit A une partie non vide de E, et (fn )n∈IN une suite de fonctions
de A dans F qui sont continues en a ∈ A. Si la suite (fn )n∈IN converge uniformément
vers une application f : A −→ F , alors f est continue en a.

Preuve : Soit ε > 0. La convergence uniforme de la suite (fn )n∈IN montre qu’il existe
m ∈ IN tel que
ε
sup k fm (x) − f (x) k ≤ . (1)
x∈A 3
Mais fm est continue en a, donc il existe un voisinage V ∈ V(a) de a tel que
ε
∀ x ∈ V ∩ A, k fm (x) − fm (a) k ≤ . (2)
3
D’où, en utilisant (1) et (2), pour tout x ∈ V ∩ A,

k f (x) − f (a) k ≤ k f (x) − fm (x) k + k fm (x) − fm (a) k + k fm (a) − f (a) k ≤ ε.

Ce qui démontre la continuité de f en a.

Corollaire II.2. Soit A une partie non vide de E, et (fn )n∈IN une suite de fonctions
continues de A dans F qui converge uniformément vers une application f : A −→ F ,
alors f est continue sur A.
4 Suites et séries de fonctions

Théorème II.3. Soit A une partie non vide de E, et (fn )n∈IN une suite de fonctions
continues de A dans F qui converge uniformément sur tout compact vers une application
f : A −→ F , alors f est continue sur A.

Preuve : En effet, soient a ∈ A, et (xn )n∈IN une suite de A qui converge vers vers a.
L’ensemble K = {xn : n ∈ IN} ∪ {a} est une partie compacte contenue dans A. Si
gn = fn |K (resp. g = f|K ) est la restriction de fn (resp. f ) à K, alors par hypothèse
(gn )n∈IN est une suite de fonctions de K dans F qui sont continues en a ∈ K et qui
converge uniformément sur K vers g. Donc, d’après le théorème II.1, g est continue en a.
En particulier, lim g(xn ) = g(a). Nous avons démontré que pour toute suite (xn )n∈IN
n→∞
A qui converge vers a la suite (f (xn ))n∈IN converge vers f (a). Il en résulte que f est
continue en a qui est un point arbitraire de A.

Théorème II.4. Soit A une partie non vide de E, et (fn )n∈IN une suite de fonctions
continues de A dans F qui converge uniformément vers une application f : A −→ F .
Alors pour toute suite (ξn )n∈IN de A qui converge vers ξ ∈ A on a lim fn (ξn ) = f (ξ).
n→∞

Preuve : En effet, soit (ξn )n∈IN une suite de A qui converge vers ξ ∈ A. Pour tout
n ∈ IN,

k fn (ξn ) − f (ξ) k ≤ k fn (ξn ) − f (ξn ) k + k f (ξn ) − f (ξ) k


≤ sup k fn (x) − f (x) k + k f (ξn ) − f (ξ) k
x∈A

La convergence uniforme de la suite (fn )n∈IN et la continuité de f en ξ ∈ A prouvent


alors que lim fn (ξn ) = f (ξ).
n→∞

Théorème II.5. Notons E = (C([a, b]), k · k∞ ) l’espace vectoriel des fonctions continues
sur l’intervalle compact [a, b], (a < b), à valeurs dans IK, muni de la norme uniforme.
Alors E est complet.

Preuve : Soit (fn )n∈IN une suite de Cauchy dans E, alors c’est une suite uniformément
de Cauchy de fonctions continues de [a, b] dans IK (qui est complet). Donc, cette suite
converge uniformément vers une application f : [a, b] −→ IK. Le corollaire II.2 montre
que f ∈ E. Ce qui donne le résultat.
Intégrabilité et dérivabilité de la limite d’une suite de fonctions 5

III. Intégrabilité et dérivabilité de la limite d’une suite de fonctions

Dans cette section [a, b] désign un intervalle compact de IR non réduit à un point.

Théorème III.1. Soit (fn )n∈IN une suite de fonctions continues de [a, b] à valeurs dans
IK. On note Z x
Fn : [a, b] −→ IK : x 7→ fn (t) dt.
a
Si la suite (fn )n∈IN converge uniformément vers une application f : [a, b] −→ IK, alors
la suite (Fn )n∈IN converge aussi uniformément vers l’application
Z x
F : [a, b] −→ IK : x 7→ f (t) dt.
a
Z b Z b
En particulier, lim fn (t) dt = lim fn (t) dt.
n→∞ a a n→∞

Preuve: En effet, pour tout x ∈ [a, b],


Z x
| Fn (x) − F (x) | ≤ | fn (t) − f (t) | dt ≤ (b − a) sup | fn (t) − f (t) | .
a t∈[a,b]

Alors,
sup | Fn (x) − F (x) | ≤ (b − a) sup | fn (t) − f (t) | .
x∈[a,b] t∈[a,b]

Cette inégalité démontre que (Fn )n∈IN converge uniformément vers F . En particulier,
il y a convergence au point x = b.
Z b Z b
Pour avoir lim fn (t) dt = lim fn (t) dt il ne suffit pas que la suite (fn )n∈IN
n→∞ a a n→∞
converge simplement. Voici un exemple:

Exemple : Considérons la suite (fn )n∈IN∗ de fonctions de


[0, 1] dans IR définie par 2n

fn (x) = max(2n(1 − | 2nx − 1 |), 0).

fn
Il est immédiat de voir que, d’une part, cette suite con-
verge simplement vers l’application identiquement
Z 1 nulle
sur [0, 1], et d’autre part, pour tout n ≥ 1, fn (t) dt = 1. 0 1
n
1
2n 1
0
6 Suites et séries de fonctions

Pour la dérivabilité les choses sont plus délicates, car la convergence uniforme d’une
suite de fonctions de classe C 1 n’antraı̂ne même pas la convergence simple de la suite
1 − cos nx
des dérivées. Par exemple, si, pour x ∈ [0, 2π], fn (x) = alors cette suite
n
converge uniformément vers la fonction identiquement nulle sur [0, 2π] et pourtant la
suite (fn0 )n∈IN∗ ne converge pas simplement.
Mais on a le théorème suivant qui est un corollaire du théorème III.1.

Théorème III.2. Soit (fn )n∈IN une suite de fonctions de classe C 1 sur [a, b] à
valeurs dans IK. Si la suite (fn0 )n∈IN converge uniformément vers une application
g : [a, b] −→ IK, et si, pour un certain x0 ∈ [a, b], la suite (fn (x0 ))n∈IN converge
vers y0 ∈ IK alors, la suite (fn )n∈IN converge uniformément vers une application
f : [a, b] −→ IK de classe C 1 et telle que f 0 = g .

Preuve: Pour tout x ∈ [a, b], nous avons


Z x
fn (x) = fn (x0 ) + fn0 (t) dt.
x0

Comme la suite de fonctions continues (fn0 )n∈IN converge uniformément vers g, alors le
théotème III.1 montre que la suite (fn )n∈IN converge uniformément vers l’application
Z x
f : [a, b] −→ IR : x 7→ y0 + g(t) dt.
x0

Il est immédiat alors de voir que f 0 = g.

Application: Le théorème suivant est un résultat moins connu et plus difficil que le
théorème qu’on obtient en remplaçant “convexe” par “croissante”.cf. Exercice 11.
Théorème III.3. Soit (fn )n∈IN une suite de de fonctions de [a, b] dans IR qui converge
simplement vers une application continue f : [a, b] −→ IR. On suppose que fn est
convexe pour tout n, alors la convergence de (fn )n∈IN vers f est uniforme.

Preuve : Nous utiliserons le lemme suivant:


Lemme : Soit f : [α, β] −→ IR une application convexe. On pose M = max(f (α), f (β))
α+β
et m = f ( ). Alors ∀ x ∈ [α, β], 2m − M ≤ f (x) ≤ M.
2
Commençons par Démontrer le lemme. Supposons d’abord que α = 0 et β = 1.
Pour x ∈ [0, 1], la convexité montre que :

f (x) = f (x.1 + (1 − x).0) ≤ xf (1) + (1 − x)f (0) ≤ xM + (1 − x)M = M.

Ce qui prouve la majoration.


Séries de fonctions 7

Démontrons alors la minoration:


1 1 1 − 2x
– Si x ∈ [0, 1/2] on a = x+ 1, et par convexité
2 2(1 − x) 2(1 − x)

1 1 1 − 2x 1 1 − 2x
m = f( ) ≤ f (x) + f (1) ≤ f (x) + M.
2 2(1 − x) 2(1 − x) 2(1 − x) 2(1 − x)

ce qui prouve que 2m − M ≤ 2m − M + 2x(M − m) ≤ f (x).


1 1 2x − 1
– Si x ∈ [1/2, 1] on a = x+ 0, et par convexité
2 2x 2x
1 1 2x − 1 1 2x − 1
m = f( ) ≤ f (x) + f (0) ≤ f (x) + M.
2 2x 2x 2x 2x

ce qui prouve aussi que 2m − M ≤ M − 2x(M − m) ≤ f (x). Le cas général résulte en


considérant g(t) = f (α + t(β − α)).
Venons à la démonstration du théorème. Soit ε > 0. La continuité uniforme de f
sur [a, b] montre qu’il existe η > 0 tel que

ε
∀ (x, y) ∈ [a, b]2 , | x − y | < η =⇒ | f (x) − f (y) | < . (†)
6

b−a
Soit m ∈ IN∗ tel que b − a < mη. On pose xk = a + k, (k = 0, 1, . . . , 2m). Il existe,
2m
à cause de la convergence simple, un entier Nε tel que

ε
∀ k ∈ {0, 1, . . . , 2m}, ∀ n ≥ Nε , | fn (xk ) − f (xk ) | < . (‡)
6

Soient x ∈ [a, b] et n ≥ Nε . Il existe k ∈ {0, 1, . . . , m − 1} tel que x ∈ [x2k , x2k+2 ].


D’une part,

ε ε
fn (x) ≤ max(fn (x2k ), fn (x2k+2 )) ≤ max(f (x2k ), f (x2k+2 )) + ≤ f (x) + .
|{z} 6 |{z} 3
par ‡ par †

Et d’autre part, en utilisant l’inégalité précédente ainsi que † et ‡, on a

ε ε
fn (x) ≥ 2fn (x2k+1 ) − max(fn (x2k ), fn (x2k+2 )) ≥ 2f (x2k+1 ) − − f (x) − ≥ f (x) − ε.
3 3

Nous avons démontré que

∀ n ≥ Nε , sup | fn (x) − f (x) | ≤ ε.


x∈[a,b]

Ce qui prouve la convergence uniforme.


8 Suites et séries de fonctions

IV. Séries de fonctions

Définition : Soient A une partie non vide de IK, et (fn )n∈IN une suite de fonctions de
A dans IK.
P
— On dit que la série fn converge simplement (resp. uniformément ; uniformément
n
X
sur tout compact) si, et seulement si, la suite de fonctions (Sn )n∈IN avec Sn = fk
k=0
converge simplement (resp. uniformément ; uniformément sur tout compact).
P
L’application limite s’appelle la somme de la série de fonctions fn et elle est
X∞
notée fn .
n=0 P
— On dit que la série fn est uniformément de Cauchy si, et seulement si, la suite
d’applications (Sn )n∈IN est uniformément de Cauchy. C’est à dire
¯ n+m ¯
¯X ¯
¯ ¯
∀ ε > 0, ∃ Nε ∈ IN, ∀(n, m) ∈ IN2 , n ≥ Nε =⇒ sup ¯ fk (x) ¯ ≤ ε.
x∈A ¯ ¯
k=n

Notons qu’une série d’applications à valeurs dans IK est uniformément convergente


si, et seulment si, elle est uniformément de Cauchy.
Notons aussi que si une série d’applications à valeurs dans IK est uniformément
convergente alors son terme général tend uniformément vers 0.

Définition : Soient A une partie non vide de IK, et (fn )n∈IN une suite d’applications
P
de A dans IK. On dit que la série fn converge normalement si, et seulement si, la
P
série µn (avec µn = sup | fn (x) |) converge.
x∈A

Remarque : Soient A une partie non vide de IK, et (fn )n∈IN une suite d’applications
de A dans IK. Nous avons les trois implications suivantes:
X X
fn { converge normalement } =⇒ fn { converge uniformément }
X X ½ converge uniformément ¾
fn { converge uniformément } =⇒ fn
sur tout compact
X ½ ¾ X
converge uniformément
fn =⇒ fn { converge simplement }
sur tout compact

Bien entendue, la réciproque de chacune de ces implications est fausse comme nous le
constaterons dans les exercices.
Continuité, dérivabilité, intégrabilité de la somme 9

Théorème IV.1. Soient A une partie non vide de IK, (hn )n∈IN et (gn )n∈IN deux suites
d’applications de A dans IK. On suppose que
1◦ . Pour tout x ∈ A, la suite (gn (x))n∈IN est une suite décroissante de réels.
2◦ . La suite (gn )n∈IN converge uniformément
¯ nvers 0. ¯
¯X ¯
¯ ¯
3◦ . Il existe M ∈ IR+ tel que ∀ n ∈ IN, sup ¯ hk (x) ¯ ≤ M .
x∈A ¯ ¯
P k=0
Alors la série d’application fn (avec fn = gn hn ) converge uniformément sur A.
n
X
Preuve : Notons Hn (x) = hk (x). D’après la transformation d’Abel, pour tout
k=0
(n, p) ∈ IN2 et tout x ∈ A, on a

n+p
X n+p
X
fk (x) = (gk (x) − gk+1 (x)) Hk (x) − gn+1 (x)Hn (x) + gn+p+1 (x)Hn+p (x).
n+1 n+1

D’où, pour tout (n, p) ∈ IN2 et tout x ∈ A, on a


¯ n+p ¯
¯X ¯
¯ ¯
¯ fk (x) ¯ ≤ M (gn+1 (x) − gn+p+1 (x) + gn+1 (x) + gn+p+1 (x)) = 2M gn+1 (x).
¯ ¯
n+1

D’où, en utilisant 1◦ et 3◦ , pour tout (n, p) ∈ IN2 ,


¯ n+p ¯
¯X ¯
¯ ¯
sup ¯ fk (x) ¯ ≤ 2M sup gn+1 (x).
x∈A ¯n+1
¯ x∈A

P
En utilisant 2◦ , on voit que la série fn est uniformément de Cauchy donc elle est
uniformément convergente.

Application : Soit (cn )n∈IN une suite réelle décroissante vers 0. Alors la série
P
d’applications cn einx de IR dans C
| converge uniformément sur tout intervalle
[2kπ + α, 2(k + 1)π − α] (avec α ∈]0, π[, et k ∈ ZZ). Il suffit de prendre dans le théorème
précédent gn (x) ≡ cn et hn (x) = einx . On utilise ensuite la majoration immédiate
(¯ n ¯ )
¯X ¯ 1
¯ ¯
sup ¯ eipx ¯ : x ∈ [2kπ + α, 2(k + 1)π − α] ≤ .
¯ ¯ sin(α/2)
p=0

V. Continuité, dérivabilité, intégrabilité de la somme d’une série


10 Suites et séries de fonctions

Les théorèmes suivants sont des traductions au langage des séries d’applications
des théorèmes correspondants pour les suites d’applications.

Théorème V.1. Soient A une partie non vide de IK, et (fn )n∈IN une suite
P
d’applications continues en a ∈ A à valeurs dans IK. Si la série fn converge uni-
X∞
formément alors la fonction somme fn est continue en a ∈ A.
n=0

Théorème V.2. Soient A une partie non vide de IK, et (fn )n∈IN une suite
P
d’applications continues sur A à valeurs dans IK. Si la série fn converge uni-

X
formément sur tout compact, alors la fonction somme fn est continue sur A.
n=0

Théorème V.3. Soient I un intervalle non réduit à un point de IR, et (fn )n∈IN une
P 0
suite d’applications de classe C 1 sur I à valeurs dans IK. Si la série fn converge
P P
uniformément sur I et s’il existe x0 ∈ I tel que fn (x0 ) converge, alors la série fn

X
converge uniformément sur I, la fonction somme fn est de classe C 1 sur I et vérifie
n=0
à ∞
!0 ∞
X X
fn = fn0 .
n=0 n=0

Théorème V.4. Soient I = [a, b] un intervalle compact non réduit à un point de IR,
P
et (fn )n∈IN une suite d’applications continues sur I à valeurs dans IK. Si la série fn
Z b ÃX∞
!
X∞
ÃZ
b
!
converge uniformément sur I, alors fn (t) dt = fn (t) dt .
a n=0 n=0 a
Exercices 11

EXERCICES

Exercice .1 Dans chacun des cas suivants, étudier la convergence simple sur
l’intervalle [0, 1] de la suite de fonctions (fn )n∈IN∗ . La fonction f limite de la suite
est-elle continue sur [0, 1] ?
Sur quels sous-intervalles de [0, 1] y a-t-il convergence uniforme ?

 2n  n
 x Log x si x ∈]0, 1]  nx Log x si x ∈]0, 1]
a. fn (x) = b. fn (x) =
 
0 si x=0 0 si x=0
 2
 sin nx

si x ∈]0, 1] n+1 n
c. fn (x) = n sin x d. fn (x) = 4n (x2 − x2 )


0 si x = 0

Exercice .2 Reprendre l’exercice précédent pour la suite de fonctions (fn )n∈IN∗


définie par
2n x
fn (x) =
1 + n2n x2
Z 1 Z 1
Calculer ensuite fn (t) dt et f (t) dt.
0 0
Exercice .3 Soit f une fonction continue sur [0, 1], on pose gn (x) = f (E(nx)/n), où
E(x) désigne la partie entière de x. Montrer que (gn )n≥1 converge uniformément vers
f sur [0, 1].

Exercice .4 F désigne l’espace vectoriel des fonctions polynomiales à coefficients


réels de degré inférieur ou égal à m, et a1 , a2 , . . . , am+1 des éléments distincts de [0, 1].
Soit (Pn )n∈IN une suite de F telle que (Pn (ai ))n∈IN converge pour tout i ∈ {1, . . . , m+1}.
Montrer que (Pn )n∈IN converge uniformément sur [0, 1] vers P ∈ F .

Exercice .5 Soit (fn )n∈IN une suite de fonctions définies sur [0, 1] à valeurs dans
IR. On suppose que, pour tout a ∈]0, 1], (fn )n converge uniformément sur [a, 1] vers 0
et que:
∃M ∈ IR, ∀ n ∈ IN, | fn (x) | ≤ M

Montrer que la suite (gn )n∈IN où gn (x) = xfn (x) tend vers 0 uniformément sur
[0, 1].
12 Suites et séries de fonctions
n
X (−1)k−1 xk
Exercice .6 Soit, pour x ∈ [0, 1] et n ∈ IN, fn (x) = − Log (1 + x).
k
k=1
1◦ . Démontrer que fn converge uniformément vers 0 sur [0, 1].
Xn µ ¶k
◦ (−1)k−1 n
2 . Démontrer que lim = Log 2.
n→∞ k n+1
k=1

Exercice .7 Soit une suite de polynômes à coefficients réels (Pn )n∈IN . On suppose
qu’elle converge uniformément sur IR vers une fonction f . Montrer que f est un
polynôme et que :

∃n0 , ∀n, n ≥ n0 =⇒ Pn − f = cn ; cn ∈ IR.

Exercice .8
1◦ . On pose, pour tout n ∈ IN∗ ,
 x
 ex − (1 + )n si x > −n
fn (x) = n
 x
e si x ≤ −n

Montrer que la restriction à IR+ de fn est une fonction croissante et en déduire


que, pour tout a > 0, la suite (fn )n∈IN converge uniformément sur l’intervalle [0, a] vers
0.
2◦ . Montrer que la restriction à IR− de fn est une fonction positive et atteint son
maximum en un point xn tel que lim xn = −2.
n→+∞
En déduire que (fn )n∈IN converge uniformément sur IR− .
Exercice .9 Soit f une application continue non nulle de IR+ dans IR+ vérifiant :

f (0) = 0, lim f (x) = 0.


x→∞

On pose : ∀x ∈ IR+ , fn (x) = f (nx), et gn (x) = f (x/n).


1◦ . Montrer que la suite (fn )n∈IN converge simplement et non uniformément sur IR+
vers 0. De même, que la suite (gn )n∈IN converge simplement et non uniformément
sur IR+ vers 0.
2◦ . Montrer qu’il existe M ∈ IR tel que: ∀x ∈ IR+ , | f (x) | ≤ M .
3◦ . Vérifier qu’alors : ∀x ∈ IR+

fn (x)gn (x) ≤ M inf(f (nx), f (x/n)).

En déduire que la suite (fn gn )n∈IN converge uniformément sur IR+ vers 0.
Exercices 13

Exercice .10 Théorème de Dini. Soit K un compact dans un e.v.n E. On considère


(fn )n∈IN une suite croissante d’applications continues de K dans IR. On suppose de
plus qu’elle converge simplement vers une fonction continue f . Montrer que (fn )n∈IN
converge uniformément vers f sur K.
Application : Etudier la suite (Pn )n∈IN de fonctions, définies sur [0, 1] à valeurs
dans IR, définie par
1
∀ t ∈ [0, 1], P0 (t) = t, Pn+1 (t) = Pn (t) + (t − Pn2 (t)) (n ≥ 1).
2
Exercice .11 On considère (fn )n∈IN une suite d’applications croissantes de [a, b]
dans IR qui converge simplement vers une application continue f . Montrer qu’elle
converge uniformément vers f sur [a, b].
Exercice .12 Théorème de Weierstrass. Pour n ∈ IN et k ∈ {0, 1, . . . , n} on pose
Bnk (X) = Cnk X k (1 − X)n−k .
Xn n
X
1◦ . Calculer ekt Bnk (x). En déduire des expressions simples de k p Bnk (x) pour
k=0 k=0
p = 0, 1 et 2. ¯ ¯2
Xn
¯k ¯

2 . Calculer ¯ − x ¯ Bnk (x). En déduire que si x ∈ [0, 1], δ > 0 et si l’on pose
¯n ¯
k=0 ¯ ¯
Aδ,x = {k : 0 ≤ k ≤ n et ¯ nk − x ¯ > δ} alors
X 1
Bnk (x) ≤
4nδ 2
k∈Aδ,x

3◦ . Soit f une fonction continue sur [0, 1], on pose

ω(f, δ) = sup {| f (x) − f (y) | : (x, y) ∈ [0, 1]2 et | x − y | ≤ δ}.

a. Montrer que lim ω(f, δ) = 0.


δ→0
n
X k
b. On pose Bn (f )(x) = f ( )Bnk (x). Montrer que
n
k=0

k f k∞
| Bn (f )(x) − f (x) | ≤ ω(f, δ) +
2nδ 2
où l’on a posé k f k∞ = sup {| f (t) | : t ∈ [0, 1]}.
c. Conclure que {Bn (f )}n≥1 converge uniformément vers f sur [0, 1].
4◦ . Soit f une fonction continue sur [a, b] et ε > 0. Montrer qu’il existe un polynôme
Pf,ε tel que
sup | f (x) − Pf,ε (x) | < ε.
x∈[a,b]
Z 1

5 . Soit f ∈ C[0, 1]. On suppose que ∀ n ∈ IN, tn f (t) dt = 0. Montrer que f = 0.
0
14 Suites et séries de fonctions

Exercice .13
1◦ . Soit f : IR −→ IR une fonction continue et 2π-périodique. Montrer que f est
uniformément continue, et que si l’on pose

ω(f, δ) = sup {| f (x) − f (y) | : (x, y) ∈ IR2 et | x − y | ≤ δ},

alors lim ω(f, δ) = 0.


δ→0
2◦ . Pour x ∈ IR, et n ∈ IN∗ on pose

n
X n−1
ikx 1X
D0 (x) = 1, Dn (x) = e , Kn (x) = Dk (x).
n
k=−n k=0

a. Montrer que, pour tout n ∈ IN∗ et tout x ∈ IR,

n µ
X ¶ µ ¶2
|k| ikx 1 sin(nx/2)
Kn (x) = 1− e = .
n n sin(x/2)
k=−n

b. Montrer que, pour tout n ∈ IN∗ et tout x ∈ IR, Kn (x) ≥ 0.


Z π
∗ 1
c. Montrer que, pour tout n ∈ IN , Kn (x) dx = 1.
2π −π
d. Montrer que, pour tout n ∈ IN∗ et tout δ ∈]0, π[,

1
∀ x ∈ [−π, π] \ [−δ, δ], Kn (x) ≤ 2 .
n sin (δ/2)

3◦ . Soit f : IR −→ IR une fonction continue et 2π-périodique. On pose,


Z π
1
cn (f ) = f (x) e−inx dx n ∈ ZZ
2π −π
X n µ ¶
|k|
σn (f )(x) = 1− ck (f ) eikx n ∈ IN∗ .
n
k=−n

Montrer successivement les relations


Z π suivantes
1
i. | σn (f )(0) − f (0) | ≤ | f (x) − f (0) | Kn (x) dx.
2π −π
2 k f k∞
ii. | σn (f )(0) − f (0) | ≤ sup | f (t) − f (0) | + ,
t∈[−δ,δ] n sin2 (δ/2)
où k f k∞ désigne sup {| f (t) | : t ∈ IR}.
Exercices 15

4◦ . Soit g : IR −→ IR une fonction continue et 2π-périodique, et soit x ∈ IR. On pose


fx (t) = g(t + x). Montrer que σn (fx )(0) = σn (g)(x). En déduire que

2 k g k∞
| σn (g)(x) − g(x) | ≤ ω(g, δ) + .
n sin2 (δ/2)

Montrer ensuite que {σn (g)}n≥1 converge uniformément vers g sur IR.
5◦ . Soit g : IR −→ IR une fonction continue et 2π-périodique, telle que ∀ n ∈ ZZ,
cn (g) = 0. Montrer que g = 0.
6◦ . Soit g : IR −→ IR la fonction 2π-périodique qui coı̈ncide avec x 7→ x(2π − x) sur
n
X
[0, 2π]. On note Sn (g)(x) = ck (g) eikx , n ∈ IN∗ . Montrer que Sn (g) converge
k=−n
uniformément vers g sur IR. En déduire

X∞
cos nx 2π 2 − 6πx + 3x2
∀ x ∈ [0, 2π], = .
n=1
n2 12
P
Exercice .14 Étudier les séries d’applications fn suivantes (convergence simple,
uniforme, normale etc.):

a. fn : IR −→ IR, fn (x) = xn n ∈ IN∗

(−1)n
b. fn : IR −→ IR, fn (x) = n ∈ IN∗
x2 + n
x
c. fn : IR+ −→ IR, fn (x) = n ∈ IN∗
(1 + nx)(1 + (n + 1)x)
nx2
d. fn : IR+ −→ IR, fn (x) = n ∈ IN∗
n3 + x 2
nx
e. fn : IR+ −→ IR, fn (x) = n ∈ IN∗
1 + n3 x 2
n+x
f. fn : IR+ −→ IR, fn (x) = n ∈ IN∗
n3 + x 2
xn
g. fn : IR+ −→ IR, fn (x) = n ∈ IN
1 + nx2n
Exercice .15 Étudier la convergence simple sur IR de la série de terme général un :

a(a − 1)(a − 2)...(a − n + 1) n


∀ x ∈ IR un (x) = x a ∈ IR
n!
Exercice .16 Pour n ∈ IN, on note fn : IR −→ IR, fn (x) = th (x + n) − th n.

P
1 . Étudier la série fn .
X∞
2◦ . On note S la somme de la série fn . Montrer que S est croissante et continue.
n=0
3◦ . Montrer: ∀ x ∈ IR, S(x + 1) = S(x) + 1 − th x.
16 Suites et séries de fonctions

Exercice .17 Donner le domaine de définition et un équivalent au voisinage de 0+


de

X √
f (x) = e−x n
.
n=0

Exercice .18 Étudier le domaine de définition, la continuité et donner un équivalent


au voisinage de 0+ et de +∞ de

X 1
f (x) = .
n=1
n + n2 x


X Z ∞
1
Exercice .19 Étude de f (x) = , convergence et calcul de f (x) dx.
n=1
n + n2 x2 0
X tn
Exercice .20 Étudier la série de fonctions sin nt. Montrer que la somme
n
n≥1
X X
de cette série est dérivable sur ] − 1, 1[. Calculer les sommes fn (t) et fn0 (t) où
n≥1 n≥1
tn
fn (t) = sin nt.
n
Exercice .21 Justifier
∞ Z
X 1 Z π
n sin t
t sin(πt) dt = dt.
n=0 0 0 t

Exercice .22 Existence et calcul, pour x > 0, de


X∞ µ Z ¶
1 x n
(Log t) dt .
n=1
n! 1

X cos nx
Exercice .23 Étudier la convergence de la série de fonctions √ , et la
n≥0
n+x
continuité de la somme sur ]0, 2π[.
Z 1 X 1
Log x
Exercice .24 Montrer que dx = .
0 x−1 n2
n≥1
Exercice .25 Soient les séries de terme général un et vn :

1 −Log n
∀ x ∈ IR+ , ∀ n ∈ IN∗ un (x) = ; vn (x) =
nx nx
1◦ . Montrer que, pour tout a > 1, ces deux séries convergent uniformément sur [a, +∞[.
X
En déduire que f = un est de classe C 1 sur ]1, +∞[. Préciser sa dérivée .
n≥1
2◦ . Soit la série de terme général wn :

(−1)n−1
∀ x ∈ IR+ , ∀ n ∈ IN∗ , wn (x) = .
nx
Exercices 17
X
Donner le domaine de convergence simple de cette série ; montrer que g = wn
n≥1
est de classe C 1 sur ]0, +∞[. Montrer ensuite que

1
∀ x > 1, g(x) = (1 − )f (x).
2x−1

Exercice .26 Soit la série de terme général un :

2x
∀ x ∈ IR ∀ n ∈ IN∗ un (x) =
n2 + x2
1◦ . Montrer que cette série converge uniformément sur tout [a, b]. On note S la somme
de cette série.

2 . Justifier la convergence uniforme sur [a, b] des séries de termes généraux respectifs
vn et wn :

x2 2(n2 − x2 )
∀ n ∈ IN∗ vn (x) = Log (1 + ), wn (x) =
n2 (n2 + x2 )2

En déduire que S est de classe C 1 sur IR et strictement croissante sur [−1, 1].
3◦ . Montrer, pour n ≥ 0, que

 
"µ ¶2n+1 µ ¶2n+1 # n
1 X X Y  X2 
1+ − 1− =X 1 + .
2 2n + 1 2n + 1 2 2 πk
k=1 (2n + 1) tg
2n + 1

4◦ . En déduire que, pour tout x dans IR,


n µ
Y ¶
x2
sh x = lim x 1+ 2 2
n→+∞ π k
k=1

5◦ . Donner une expression simple de S, et montrer qu’elle est croissante sur IR.

Exercice .27 Justifier, pour a ∈ IR, l’égalité


Z ∞ X∞
sin ax a
x
dx = .
0 e −1 n=1
n + a2
2

Exercice .28 Justifier l’égalité


Z 1 X∞
Log xLog (1 − x) 1
dx = 3
.
0 x n=1
n
18 Suites et séries de fonctions

Exercice .29 Si x ∈ IR, E(x) désigne la partie entière de x. On pose, pour x ∈ IR,
¯ ¯
¯ 1 ¯
∆(x) = ¯ x − E(x − ) − 1 ¯¯ .
¯
2

1◦ . Étude de ∆:
a. Montrer que ∆ définit une fonction périodique de IR dans IR de période 1.
b. Calculer lim ∆(x), et lim ∆(x). Que peut-on en déduire ?
> <
x → 1/2 x → 1/2
c. Exprimer simplement ∆(x) pour x ∈ [0, 1],(on distinguera les cas x ∈ [0, 1/2]
et x ∈ [1/2, 1]). Tracer le graphe de ∆.

P
n−1
2◦ . Pour x ∈ IR, on pose fn (x) = 2−k ∆(2k x).
k=0
a. Montrer que, pour tout x ∈ IR, la suite {fn (x)}n converge vers une limite que
l’on note dans la suite f (x).
b. Montrer que la fonction f est continue, bornée, périodique de période 1, et
vérifie
1
∀ x ∈ IR, f (x) − f (2x) = ∆(x).
2

1
3◦ . Soit h : IR → IR, une fonction bornée telle que, ∀ x ∈ IR, h(x) − h(2x) = ∆(x).
2
h(2m x)
Montrer que ∀ m ≥ 1, ∀ x ∈ IR, fm (x) = h(x) − . En déduire que f =
2m
h.

4◦ . Calcul de la borne supérieure de f :


1
a. Soit δ(x) = ∆(x) + ∆(2x). Tracer le graphe de δ.
2
m−1
X
b. Montrer que ∀ x ∈ IR, f2m (x) = 2−2k δ(22k x). En déduire la majoration
k=0
sup f (x) ≤ 2/3.
IR
c. Montrer que f2m (1/3) = (2/3)(1 − 4−m ), (On pourra commencer par calculer
le reste de la division de 22k par 3). En déduire la valeur de sup f (x).
IR

5◦ . Pour x ∈ IR on pose ²1 (x) = E(2x) − 2E(x), et ²n (x) = ²1 (2n−1 x).


a. Montrer que ²1 est une fonction 1-périodique qui prend seulement les valeurs
0 et 1.
X∞
²n (x)
b. Montrer que pour tout x ∈ IR, on a x = E(x) + n
.
n=1
2
Exercices 19

c. Pour x ∈ IR on pose

à n
!
X X ²n (x)
g(x) = 2+ (−1)²k (x) .
n=1
2n
k=1

1
Montrer que g est une fonction bornée qui vérifie g(x) − g(2x) = ∆(x). En
2
déduire que g = f .
d. Fixons x ∈ IR, pour simplifier on va noter ²n au lieu de ²n (x). On pose

m
X m+n+1
X
²k 1 1
xm = , ym = xm + m , ym,n = xm + .
2k 2 2k
k=1 k=m+1

de telle manière que ym = lim ym,n .


n→∞
Calculer f (ym,n ) − f (xm ), puis f (ym ) − f (xm ). En déduire que
m
f (ym ) − f (xm ) X
= (−1)²k (x) .
ym − xm
k=1

puis que f n’est nulle part dérivable.


20 Suites et séries de fonctions

SOLUTIONS

Solution .1 a. Étudions les variations de fn , comme fn0 (x) = x2n−1 (1 + 2nLog x),
alors nous avons le tableau de variations suivant:

x 0 e−1/2n 1
fn0 (x) − 0 +
1
fn (x) 0 & − 2en % 0
1
Il en résulte que sup | fn (x) | = . D’où la suite (fn )n≥1 converge uniformément
x∈[0,1] 2en
vers 0.
b. Il est immédiat que la suite (fn )n≥1 converge simplement vers 0.
Comme fn (e−1/n ) = −1/e, pour tout n ≥ 1, alors la suite (fn )n≥1 ne converge pas
uniformément sur [0, 1].
Pour tout a ∈ [0, 1[ on a

nan−1
sup | fn (x) | ≤ ,
x∈[0,a] e

car sup | xLog x | = 1/e. Il en résulte que la suite (fn )n≥1 converge uniformément vers
x∈]0,1]
0 sur tout intervalle [0, a] avec a ∈ [0, 1[.
c. Pour tout x ∈]0, 1] nous avons
1
| fn (x) | ≤ ,
n sin x
alors la suite (fn )n≥1 converge simplement vers 0 sur [0, 1].
1 sin2 1 1
Comme fn ( ) = , pour tout n ≥ 1, alors la suite fn ( ) ne tend pas vers
n n sin(1/n) n
0 et la suite (fn )n≥1 ne converge pas uniformément sur [0, 1].
Pour tout a ∈]0, 1] on a
1
sup | fn (x) | ≤ .
x∈[a,1] n sin a

Il en résulte que la suite (fn )n≥1 converge uniformément vers 0 sur tout intervalle [a, 1]
avec a ∈]0, 1].
d. Notons que fn (1) = 0 pour tout n. Soit a ∈ [0, 1[ alors, pour tout n,
n
sup | fn (x) | ≤ 4n a2 .
x∈[0,a]
Solutions 21

Mais lim n2 an = 0. Il en résulte que la suite (fn )n≥1 converge uniformément vers 0
n→∞
sur tout intervalle [0, a] avec a ∈ [0, 1[.
La convergence de cette suite n’est pas uniforme sur [0, 1] car, pour tout n,
n
fn (21/2 ) = −4n−1 .

Solution .2 Notons que fn (0) = 0 pour tout n. Soit a ∈]0, 1] alors, pour tout n,

1
sup | fn (x) | ≤ .
x∈[a,1] na

Il en résulte que la suite (fn )n≥1 converge uniformément vers 0 sur tout intervalle [a, 1]
avec a ∈]0, 1].
La suite (fn )n≥1 converge simplement vers 0. La convergence étant uniforme sur
tout intervalle [a, 1] avec a ∈]0, 1].
D’autre part,
Z 1 Z 1
2n x Log (1 + n2n )
fn (t) dt = dx = .
0 0 1 + n2n x2 2n
Z 1 √
Alors, lim fn (t) dt = Log 2 6= 0. Il en résulte que la convergence de la suite
n→∞ 0
(fn )n≥1 n’est pas uniforme sur [0, 1].

Solution .3 Soit ε > 0, la continuité uniforme de f sur [0, 1] montre qu’il existe
η > 0 tel que pour tout (x, y) ∈ [0, 1]2

| x − y | ≤ η =⇒ | f (x) − f (y) | < ε.

Choisissons n ≥ 1/η, et x ∈ [0, 1[ alors 0 ≤ nx − E(nx) ≤ 1 et par conséquent,

E(nx) 1
0≤x− ≤ ≤η
n n

D’où,
¯ ¯
¯ ¯
¯ f (x) − f ( E(nx) ) ¯ ≤ ε.
¯ n ¯

Nous avons donc démontré que, pour tout n ≥ 1/η, sup | f (x) − gn (x) | ≤ ε. D’où le
x∈[0,1]
résultat.
22 Suites et séries de fonctions

Solution .4 Notons E l’espace vectoriel normé F muni de la norme uniforme sur


[0, 1] (i.e. k P k∞ = sup | P (t) |), et G l’espace vectoriel normé IRm+1 muni de la norme
t∈[0,1]
N∞ ((x1 , . . . , xm+1 )) = max | xi |.
1≤i≤m+1
Considérons
ϕ : E → G : P 7→ (P1 (a1 ), . . . , P (am+1 )).

L’application ϕ est un isomorphisme linéaire entre E et G. (Car (a1 , . . . , am+1 ) sont


distincts et dim E = dim G).
D’après l’hypothèse, la suite (ϕ(Pn ))n≥1 converge dans G vers un élément Λ ∈ G.
Alors la suite (Pn )n≥1 converge dans E vers ϕ−1 (Λ) (Car ϕ−1 est continue). D’où le
résultat.

Solution .5 Soit ε > 0, considérons aε = min(1, ε/M ). La convergence uniforme de


la suite (fn )n≥1 sur [aε , 1], montre qu’il existe Nε tel que

n ≥ Nε =⇒ ∀ x ∈ [aε , 1], | fn (x) | ≤ ε.

Soit n ≥ Nε et x ∈ [0, 1].


– Si x ∈ [0, aε [ alors | gn (x) | ≤ M aε ≤ ε.
– Si x ∈ [aε , 1] alors | gn (x) | ≤ | fn (x) | ≤ ε.
Alors,

n ≥ Nε =⇒ sup | gn (x) | ≤ ε.
x∈[0,1]

D’où la suite (gn )n≥1 converge uniformément vers 0.

(−x)n
Solution .6 En effet, un calcul simple montre que fn0 (x) =− , pour tout
1+x
x ∈ [0, 1].
1◦ . Il en résulte que, pour tout x ∈ [0, 1],
¯Z x ¯ Z 1
¯ (−1)n+1 tn ¯¯ 1
| fn (x) | = ¯¯ dt ¯ ≤ tn dt =
0 1+t 0 n+1

1
ou bien, sup | fn (x) | ≤ . La suite (fn )n≥1 converge uniformément vers 0 sur
x∈[0,1] n+1
[0, 1].
2◦ . la suite de terme général xn = n/(n + 1) est une suite de [0, 1] qui tend vers 1.
Alors (fn (xn ))n≥1 converge vers 0. Ce qui donne le résultat.
Solutions 23

Solution .7 Remarquons d’abord que les seules fonctions polynomiales bornées sur
IR sont les constantes.
La suite (Pn )n≥0 est uniformément convergente sur IR, alors la suite (Qn )n≥0 définie
par Qn (X) = Pn (X) − Pn (0) est aussi uniformément convergente sur IR, elle est donc
uniformément de Cauchy. Il existe n0 tel que

n ≥ n0 =⇒ sup | Qn (x) − Qn0 (x) | ≤ 1.


x∈IR

On conclut que le polynôme Qn − Qn0 est constant pour tout n ≥ n0 , mais ce polynôme
est nul en 0, donc ∀ n ≥ n0 , Qn = Qn0 . D’autre part, (Qn )n≥0 converge vers f − f (0)
lorsque n tend vers l’infini, d’où f − f (0) = Qn0 et ∀ n ≥ n0 , Qn = f − f (0). Ce qui se
traduit par ∀ n ≥ n0 , Pn − f = Pn (0) − f (0) = cn .

x
Solution .8 1◦ . Posons, pour x ∈] − n, +∞[, hn (x) = x − (n − 1)Log (1 + ).
n
0 x+1
Un calcul simple donne hn (x) = . D’où le tableau de variations suivant:
x+n

x −n −1 0 +∞
h0n (x) − 0 + + (∗)
hn (x) +∞ & ^ % 0 % +∞

Le fait que hn (x) ≥ 0 sur IR+ , montre que


³ x ´n−1
x
∀ x ≥ 0, e ≥ 1 + ,
n
ce qui implique ∀ x ≥ 0, fn0 (x) ≥ 0. La fonction fn est donc croissante sur IR+ , et elle
est positive sur cet ensemble car fn (0) = 0.
Nous en déduisons que sup | fn (x) | = fn (a), mais lim fn (a) = 0. Alors, pour
x∈[0,a] n→∞

tout a ∈ IR+ , la suite (fn )n≥1 converge uniformément sur [0, a] vers 0.
2◦ . Le tableau de variations (∗) montre qu’il existe un unique xn ∈] − n, −1[ tel
que hn (xn ) = 0.
– Si x ∈] − n, xn ], alors hn (x) ≥ 0 et par conséquent fn0 (x) ≥ 0.
– Si x ∈]xn , 0], alors hn (x) ≤ 0 et par conséquent fn0 (x) ≤ 0.

Il en résulte que fn admet sur IR− le tableau de variations suivant:

x −∞ xn 0
fn0 (x) + 0 −
fn (x) 0 % _ & 0
24 Suites et séries de fonctions

On conclut que fn est positive sur IR− et que sup | fn (x) | = fn (xn ) ; où xn est
x∈IR−
l’unique racine de hn dans l’intervalle ] − n, −1[.
Soit ε ∈]0, 1[, on pose
µ ¶
Log (1 − ε)
Nε = 1 + E .
ε + Log (1 − ε)

(Remarquons que Log (1 − ε) < 0 et ε + Log (1 − ε) < 0).

Log (1 − ε)
n ≥ Nε =⇒ n >
ε + Log (1 − ε)
=⇒ n(ε + Log (1 − ε)) < Log (1 − ε)
=⇒ − nε − (n − 1)Log (1 − ε) > 0
=⇒ hn (−εn) > 0.

Mais −εn ∈] − n, 0[ et le tableau de variations (∗) de hn montre que la condition


xn
hn (−εn) > 0 implique −εn ≤ xn ≤ 0. Il en résulte que lim = 0.
n→∞ n
Posons εn = xn /n. L’équation hn (xn ) = 0 et la condition xn 6= 0 montrent que

εn Log (1 + εn )
xn = −
εn − Log (1 + εn )

un développemnet limité simple montre alors que xn = −2 + O(εn ) et par conséquent


lim xn = −2.
n→∞
Enfin, hn (xn ) = 0 implique que exn = (1 − εn )n−1 et par conséquent fn (xn ) =
−εn exn . On conclut que

xn xn
lim sup | fn (x) | = lim − e = 0.
n→∞ x∈IR− n→∞ n

La suite (fn )n∈IN∗ converge uniformément vers 0 sur IR− .

Solution .9 1◦ . La convergence simple des deux suites (fn )n≥1 et (gn )n≥1 vers 0
est immédiate.
La fonction f n’est pas nulle, alors il existe x0 ∈ IR∗+ tel que f (x0 ) 6= 0. Mais alors
les deux suites (fn (x0 /n))n≥1 et (gn (nx0 ))n≥1 ne tendent pas vers 0, et la convergence
des deux suites (fn )n≥1 et (gn )n≥1 vers 0 n’est pas uniforme.
x
2◦ . Soit h : [0, 1[−→ IR+ : x 7→ . On pose g = f ◦h, g est une fonction continue
1−x
sur [0, 1[ qui est prolongeable par continuité à [0, 1], donc g est bornée par M sur [0, 1[.
Ceci démontre ∀ x ∈ IR+ , | f (x) | ≤ M .
Solutions 25

3◦ . En utilisant le fait que f est à valeurs positives majorées par M , nous trouvons
immédiatement,

∀ x ∈ IR+ , fn (x)gn (x) ≤ M min(f (nx), f (x/n)).

Soit ε > 0, il existe a > 0 et A > 0 tels que


ε
0 ≤ x < a =⇒ f (x) <
M
ε
x > A =⇒ f (x) <
M
p
Soit n > A/a, alors an > A/n. Pour tout x ∈ IR+ , nous avons deux alternatives:
x
– Soit x < an et par conséquent < a, et alors
n

fn (x)gn (x) ≤ M f (x/n) < ε.

– Soit x ≥ an > A/n et alors nx > A, d’où

fn (x)gn (x) ≤ M f (nx) < ε,

ce qui démontre que,


r
A
∀n > , sup | fn (x)gn (x) | < ε.
a x≥0

La suite (fn gn )n∈IN∗ converge uniformément sur IR+ vers 0.

Solution .10 Soit ε > 0. Posons Fnε = {x ∈ K : f (x) − fn (x) ≥ ε}. La fonction
f − fn étant continue, alors Fnε est une partie fermée. D’autre part, la croissance de la
ε
suite montre que, pour tout n, Fn+1 ⊂ Fnε . Nous avons alors deux alternatives:
– Pour tout n, l’ensemble Fnε n’est pas vide. Choisissons alors pour chaque n ∈ IN un
élément xn ∈ Fnε . L’ensemble K est compact alors il existe une application strictement
croissante ϕ de IN dans IN telle que la suite extraite (xϕ(n) )n∈IN converge vers un
ε ε
e ∈ K. Remarquons que, pour tout m > n, on a Fϕ(m)
élément x ⊂ Fϕ(n) ⊂ Fnε , d’où
∀ m > n, xϕ(m) ∈ Fnε . En faisant tendre m vers l’infini et en utilisant le fait que Fnε
est fermé, on conclut que xe ∈ Fnε . Comme n est arbitraire nous arrivons à la conclusion
\
x
e∈ Fnε . Ce qui se traduit par
n≥0

∀ n ∈ IN, ε ≤ f (e
x) − fn (e
x),
26 Suites et séries de fonctions

ceci est en contradiction avec la convergence de (fn (e


x))n∈IN vers f (e
x). On est donc dans
la seconde alternative:
– Il existe Nε tel que FNε ε = Ø. Alors, en utilisant la croissance de la suite (fn )n≥0
∀ n ≥ Nε , ∀ x ∈ K, f (x) − ε < fNε (x) ≤ fn (x) ≤ f (x)
ou bien, ∀ n ≥ Nε , sup | f (x) − fn (x) | < ε. D’où la suite (fn )n≥0 converge uni-
x∈K
formément vers f .
Application: Considérons la suite (Pn )n≥0 de fonctions polynomiales sur [0, 1]:
1
P0 (t) = t, Pn+1 (t) = Pn (t) + (t − Pn2 (t)).
2 √
Montrons par récurrence sur n que ∀ t ∈ [0, 1], 0 ≤ Pn (t) ≤ t. En effet, c’est vrai si
n = 0. À t fixé, la fonction ϕt définie par x 7→ ϕt (x) = x + 21 (t − x2 ) est croissante sur
[0, 1] et par conséquent,
√ √
0 ≤ Pn (t) ≤ t =⇒ ϕt (0) ≤ ϕt (Pn (t)) ≤ ϕt ( t)
t √
=⇒ 0 ≤ ≤ Pn+1 (t) ≤ t.
2
On conclut que,

∀ n ≥ 0, ∀ t ∈ [0, 1], 0 ≤ Pn (t) ≤ Pn+1 (t) ≤ t.
Pour tout t ∈ [0, 1], la suite (Pn (t))n≥0 converge (car croissante et majorée) vers
√ √
h(t) ∈ [0, t] qui vérifie h2 (t) = t donc h(t) = t. On est en présence d’une suite
croissante (Pn )n≥0 de fonctions continues sur le compact [0, 1] qui converge simplement
vers la fonction continue h. Alors elle converge uniformément vers h.
Solution .11 Soit ε > 0. La continuité uniforme de f sur [a, b] montre qu’il existe
η > 0 tel que pour tout (x, y) ∈ [a, b]2
ε
| x − y | ≤ η =⇒ | f (x) − f (y) | < .
2
b−a k(b − a)
Choisissons, p tel que ≤ η, et posons xk = a + pour k ∈ {0, 1, . . . , p}. La
p p
convergence simple de la suite (fn )n∈IN aux points {x0 , . . . , xp }, montre qu’il existe Nε
tel que
ε
∀ n > Nε , ∀ j ∈ {0, 1, . . . , p}, | f (xj ) − fn (xj ) | ≤
.
2
Soient n > Nε , et x ∈ [a, b]. Il existe k ∈ {0, 1, . . . , p − 1} tel que xk ≤ x ≤ xk+1 ,
et alors
ε ε
f (x) − fn (x) ≤f (xk+1 ) − fn (xk ) = f (xk+1 ) − f (xk ) + f (xk ) − fn (xk ) ≤ + =ε
2 2
f (x) − fn (x) ≥f (xk ) − fn (xk+1 ) = f (xk ) − f (xk+1 ) + f (xk+1 ) − fn (xk+1 ) ≥ −ε
Il en résulte que ∀ n > Nε , sup | f (x) − fn (x) | ≤ ε, et la suite converge uni-
x∈[a,b]
formément vers f .
Solutions 27

Solution .12 1◦ . Notons que


n
X n
X
kt
e Bnk (x) = Cnk (xet )k (1 − x)n−k = (1 − x + xet )n
k=0 k=0

n
X
d’où, ekt Bnk (x) = (1 − x + xet )n (1)
k=0
en dérivant une et deux fois par rapport à t les deux membres de cette égalité, nous
arrivons à
n
X
kekt Bnk (x) = nxet (1 − x + xet )n−1 (2)
k=0
n
X
k 2 ekt Bnk (x) = nxet (1 − x + xet )n−2 (1 − x + nxet ) (3)
k=0

En substituant t par 0 dans ces égalités nous obtenons


n
X n
X n
X
Bnk (x) = 1, kBnk (x) = nx, k 2 Bnk (x) = nx + (n2 − n)x2 (4)
k=0 k=0 k=0

2◦ . En effet, En utilisant (4),

Xn ¯ ¯2
Xn
¯k ¯
¯ − x ¯ Bnk (x) = 1 (k 2 − 2nxk + n2 x2 )Bnk (x)
¯n ¯ n2
k=0 k=0
à n n n
!
1 X 2 k X X
= 2 k Bn (x) − 2nx kBnk (x) + n2 x2 Bnk (x)
n
k=0 k=0 k=0
2
1 x−x 1
= 2
(nx + (n2 − n)x2 − 2n2 x2 + n2 x2 ) = ≤
n n 4n
d’où,
n ¯
X
¯2
X ¯¯ k
¯2
X
1 ¯k ¯ k ¯
≥ ¯ − x ¯ Bn (x) ≥ ¯ − x ¯ Bnk (x) ≥ δ 2 Bnk (x)
4n ¯ n ¯ ¯ n ¯
k=0 k∈Aδ,x k∈Aδ,x

nous obtenons alors


X 1
Bnk (x) ≤ (5)
4δ 2 n
k∈Aδ,x

3◦ .a. Soit ε > 0. La continuité uniforme de f sur l’intervalle compact [0, 1] montre
qu’il existe η > 0 tel que

∀ (x, y) ∈ [0, 1]2 , | x − y | ≤ η =⇒ | f (x) − f (y) | ≤ ε

ce qui démontre que ∀ δ ≤ η, ω(f, δ) ≤ ε. On conclut que lim ω(f, δ) = 0.


δ→0
28 Suites et séries de fonctions

3◦ .b. En partageant la somme en deux, nous avons


¯ n ¯
¯X k X n ¯
¯ k k ¯
| Bn (f )(x) − f (x) | = ¯ f ( )Bn (x) − f (x) Bn (x) ¯
¯ n ¯
k=0 k=0
n ¯ ¯
X ¯ k ¯
≤ ¯ f ( ) − f (x) ¯ Bnk (x)
¯ n ¯
k=0
X ¯¯ k
¯
X ¯¯ k
¯
¯ k ¯ k
≤ ¯ f ( ) − f (x) ¯ B (x) + ¯ f ( ) − f (x) ¯ Bn (x)
¯ n ¯ n ¯ n ¯
k∈Aδ,x k∈A
/ δ,x
X X
≤ 2 k f k∞ Bnk (x) + ω(f, δ)Bnk (x)
k∈Aδ,x k∈A
/ δ,x

X n
X
≤2 k f k∞ Bnk (x) + ω(f, δ) Bnk (x)
k∈Aδ,x k=0

k f k∞
≤ + ω(f, δ)
2nδ 2
On conclut que,
k f k∞
k Bn (f ) − f k∞ ≤ + ω(f, δ). (6)
2nδ 2
3◦ .c. Soit ε > 0, il existe δ > 0 tel que ω(f, δ) ≤ ε/2. Pour δ ainsi fixé nous
k f k∞
choisissons N ≥ alors, en utilisant (6),
εδ 2
ε ε
n ≥ N =⇒ k Bn (f ) − f k∞ ≤ + = ε.
2 2
Ceci démontre que lim k Bn (f ) − f k∞ = 0, qui est le résultat demandé.
n→∞
4◦ . Soit g la fonction continue sur [0, 1] définie par g(t) = f (t b + (1 − t)a). D’après
3◦ .c il existe une fonction polynomiale Q telle que k g − Q k∞ < ε. Il suffit alors, de
X −a
poser Pf,ε (X) = Q( ), et on vérifie immédiatement que
b−a
sup | f (x) − Pf,ε (x) | < ε.
x∈[a,b]
Z 1

5 . Remarquons que d’après l’hypothèse nous avons, f (t)Bn (f )(t) dt = 0 pour
0
tout n. D’où, Z Z µ ¶
1 1
2
0≤ (f (t)) dt = f (t) f (t) − Bn (f )(t) dt
0 0
Z 1
≤ | f (t) | | f (t) − Bn (f )(t) | dt
0
≤ k f k∞ k f (−Bn (f ) k∞
Z 1
En faisant tendre n vers l’infini, nous obtenons (f (t))2 dt = 0, et la continuité de f
0
montre alors que f = 0.
Solutions 29

Solution .13 1◦ . Soit ε > 0. La continuité uniforme de f sur l’intervalle compact


[−π, 3π] montre qu’il existe ηε ∈]0, π[ tel que

∀ (x, y) ∈ [−π, 3π]2 , | x − y | < ηε =⇒ | f (x) − f (y) | < ε (1)

Soient (x, y) ∈ IR2 tel que | x − y | < ηε . On pose n = E(x/2π), X = x − 2πn et


Y = y − 2πn. Il est immédiat que X ∈ [0, 2π[. D’autre part,

| y − x | ≤ ηε =⇒ − π ≤ y − X − 2πn ≤ π
=⇒ X − π ≤ Y ≤ X + π =⇒ Y ∈ [−π, 3π]

Alors (X, Y ) ∈ [−π, 3π]2 et | X − Y | = | x − y | < ηε , il en résulte d’après (1) que

| f (x) − f (y) | = | f (X) − f (Y ) | < ε.

Nous avons donc démontré que f est uniformément continue sur IR. En procédant
comme dans la question 3◦ .a de l’exercice précédent nous arrivons à lim ω(f, δ) = 0.
δ→0
Xn µ ¶
e n (x) = | k |
2◦ .a. Posons K 1− eikx . Alors, pour m ≥ 1,
n
k=−n

m
X m
X
e m+1 (x) − mK
(m + 1)K e m (x) = ikx
(m + 1 − | k |) e − (m − | k |) eikx
k=−m k=−m
Xm
= eikx = Dm (x)
k=−m

e 0 (x) = D0 (x) = 1 alors, en prenant la somme des égalités précédentes pour m


Mais K
variant entre 1 et n − 1 nous obtenons,
n−1
X
e n (x) = D0 (x) +
nK Dm (x) = nKn (x). (2)
m=1

ce qui démontre la première égalité.


D’autre part,
n
X ei(n+1)x − e−inx sin(n + 1/2)x cos nx − cos(n + 1)x
Dn (x) = eikx = ix
= = .
e −1 sin(x/2) 1 − cos x
k=−n

La relation (n + 1)Kn+1 (x) − nKn (x) = Dn (x), démontre que, pour tout n ≥ 1,

cos(n + 1)x cos nx


(n + 1)Kn+1 (x) + = nKn (x) +
1 − cos x 1 − cos x
30 Suites et séries de fonctions

ce qui permet d’écrire


cos nx cos x 1
nKn (x) + = K1 (x) + = .
1 − cos x 1 − cos x 1 − cos x
Enfin,
µ ¶ µ ¶2
1 1 − cos nx 1 sin(nx/2)
Kn (x) = = . (3)
n 1 − cos x n sin(x/2)
2◦ .b. Il est immédiat d’après l’égalité (3) que Kn (x) ≥ 0 pour tout x ∈ IR et tout
n ∈ IN∗ . Z π

2 .c. En utilisant le fait que eikx dx = 0 si k 6= 0, et l’égalité (2) nous obtenons
−π
immédiatement que Z π
∗ 1
∀ n ∈ IN , Kn (x) dx = 1. (4)
2π −π

2◦ .d. Il est immédiat que

∀ x ∈ [−π, π] \ [−δ, δ], sin2 (x/2) ≥ sin2 (δ/2)

alors, en utilisant (3),


1
∀ x ∈ [−π, π] \ [−δ, δ], Kn (x) ≤ 2 (5)
n sin (δ/2)

3◦ . Remorquons que
n µ ¶Z π Z π
1 X |k| ikx 1
σn (f )(0) = 1− f (x)e dx = Kn (x)f (x) dx.
2π n −π 2π −π
k=−n

D’où,
¯Z π Z π ¯
1¯ ¯
| σn (f )(0) − f (0) | = ¯ Kn (x)f (x) dx − f (0) Kn (x) dx ¯¯
2π¯
¯ Z−π −π
¯
1¯ π ¯
= ¯ K (x)(f (x) − f (0)) dx ¯
2π¯ n ¯
−π
Z π
1
≤ Kn (x) | f (x) − f (0) | dx
2π −π
Z
1
≤ Kn (x) | f (x) − f (0) | dx+
2π | x |≤δ
Z
1
Kn (x) | f (x) − f (0) | dx
2π δ<| x |≤π
Z π Z
dx 2 k f k∞
≤ sup | f (x) − f (0) | Kn (x) + Kn (x) dx
x∈[−δ,δ] −π 2π 2π δ<| x |

2 k f k∞
≤ sup | f (x) − f (0) | +
x∈[−δ,δ] n sin2 (δ/2)
Solutions 31

4◦ . Notons que cn (fx ) = cn (g) einx et par conséquent σn (fx )(0) = σn (g)(x).
L’inégalité de 3◦ s’écrit alors

2 k g k∞
| σn (g)(x) − g(x) | ≤ sup | g(y) − g(x) | +
| y−x |≤δ n sin2 (δ/2)

ce qui implique
2 k g k∞
k σn (g) − g k∞ ≤ ω(g, δ) + .
n sin2 (δ/2)
Soit ε > 0, il existe δ > 0 tel que ω(g, δ) ≤ ε/2. Pour δ ainsi fixé nous choisissons
4 k g k∞
N≥ alors,
ε sin2 (δ/2)

ε ε
n ≥ N =⇒ k σn (g) − g k∞ ≤ + = ε.
2 2

Ceci démontre que lim k σn (g) − g k∞ = 0, qui est le résultat demandé.


n→∞
5◦ . Remarquons que d’après l’hypothèse nous avons, σn (g) = 0 pour tout n. D’où,
en faisant tendre n vers l’infini, nous obtenons g = 0.
6◦ . Un calcul immédiat montre que cn (g) = −2/n2 si n ∈ ZZ\{0}, et c0 (g) = 2π 2 /3.
Puis
−4
∀ n ≥ 1, Sn (g)(x) − Sn−1 (g)(x) = cos(nx).
n2
P
On conclut que la série k Sn (g) − Sn−1 (g) k∞ est convergente. L’espace C2π des
fonctions continues 2π-périodiques sur IR muni de la norme uniforme est complet, alors
la suite (Sn (g))n∈IN est uniformémant convergente sur IR vers une fonction continue
2π-périodique h.
Mais cm (h) = lim cm (Sn (g)) = cm (g) pour tout m ∈ ZZ. Il en résulte que ∀ m ∈
n→∞
ZZ, cm (g − h) = 0, d’où g = h.
Ce qui précède démontre que (Sn (g))n∈IN est uniformémant convergente sur IR vers
g. Mais
Xm
2π 2 cos nx
Sm (g)(x) = −4 .
3 n=1
n2

Alors,
X∞
cos nx 2π 2 − 6πx + 3x2
∀ x ∈ [0, 2π], 2
= .
n=1
n 12

En prenant x = 0 et x = π, nous obtenons



X ∞
X ∞
X
1 π2 (−1)n−1 π2 1 π2
= , = , et = .
n2 6 n2 12 (2n + 1)2 8
k=1 k=1 k=1
32 Suites et séries de fonctions
X
Solution .14 a. Étude de fn avec, fn : IR −→ IR, fn (x) = xn .
n≥1P
Il est immédiat que la série fn (x) diverge si | x | ≥ 1 car son terme général ne
P n
tend pas vers 0. Par contre si a ∈ [0, 1[, alors sup | fn (x) | = an et la série a
x∈[−a,a]
P
converge. Il en résulte que fn converge normalement sur tout compact contenu dans
P
] − 1, 1[, donc en particulier elle converge simplement sur ] − 1, 1[. La série fn ne
converge pas uniformément sur ] − 1, 1[ car dans le cas contraire le terme général fn
convergerait uniformément sur ] − 1, 1[ vers 0 ce qui est manifestement absurde.

X (−1)n
b. Étude de fn avec, fn : IR −→ IR, fn (x) = .
x2 + n
n≥1
En utilisant le critère de convergence des séries alternées nous voyons que la série
X
fn (x) converge pour tout x ∈ IR. D’autre part, si l’on note S la somme de cette
n≥1
série de fonctions, alors
¯ ¯
¯ n
X ¯ 1 1
¯ ¯
∀ n ∈ IN∗ , ∀ x ∈ IR, ¯ S(x) − f k (x) ¯≤ 2 ≤ .
¯ ¯ x +n+1 n+1
k=1

¯ ¯
¯ n
X ¯ X
¯ ¯
Il en résulte que lim sup ¯ S(x) − fk (x) ¯ = 0, i.e. la série fn converge uni-
n→∞ x∈IR ¯ ¯
k=1 n≥1
formément sur IR. Evidemment elle ne converge pas normalement.
X
c. Étude de fn avec,
n≥1

x 1 1
fn : IR+ −→ IR, fn (x) = = −
(1 + nx)(1 + (n + 1)x) 1 + nx 1 + (n + 1)x
n
X X
1 1
Il est immédiat que Sn (x) = fk (x) = − , donc fn con-
1 + x 1 + (n + 1)x
k=1 n≥1
verge simplement vers la fonction

0
 si x=0
S : IR+ −→ IR, S(x) = 1

 si x > 0
1+x

Cette fonction n’étant pas continue alors la convergence n’est pas uniforme sur IR+ .
1 X
Par contre, soit a > 0 alors, sup | fn (x) | ≤ donc la série fn
x∈[a,+∞[ an(n + 1)
n≥1
converge normalement sur tout intervalle de la forme [a, +∞[ avec a > 0.
Solutions 33

X nx2
d. Étude de fn avec, fn : IR+ −→ IR, fn (x) = .
n3 + x2
n≥1
Soit A > 0, clairement nous avons la majoration
A2
sup | fn (x) | ≤ .
x∈[0,A] n2
X
On conclut que la série fn converge normalement sur tout compact de IR+ . D’autre
n≥1
part, cette série ne converge pas uniformément sur IR+ car son terme général ne tend
pas uniformémant sur IR+ vers 0, (En effet lim fn (n) = 1).
n→∞
X nx
e. Étude de fn avec, fn : IR+ −→ IR, fn (x) = .
1 + n3 x 2
n≥1
1
Notons que la série converge pour x = 0, et que si x > 0 alors 0 ≤ fn (x) ≤
n2 x
donc la série converge simplement sur IR+ . De plus, l’inégalité précédente montre que
X
fn converge normalement sur [a, +∞[ pour tout a > 0.
n≥1
1
La convergence de cette série n’est pas uniforme sur IR+ , car si xn = √ ,
n n
¯ ¯ √
¯ X
2n ¯ X2n 2n
X nxn n
¯ ¯
sup ¯ fk (x) ¯ ≥ fk (xn ) ≥ ≥ .
x∈IR+ ¯ ¯ 3
1 + 8n xn 9
k=n+1 k=n+1 k=n+1

X n+x
f. Étude de fn avec, fn : IR+ −→ IR, fn (x) = .
n3 + x2
n≥1
Un calcul simple montre que
p
sup | fn (x) | = fn (xn ), avec xn = n3 + n2 − n.
IR+

2 X
Donc, sup | fn (x) | ≤ √ . La série fn converge normalement sur IR+ .
x∈IR+ n n
n≥1
X xn
g. Étude de fn avec, fn : IR+ −→ IR, fn (x) = .
1 + nx2n
n≥0
X
Soit a ∈ [0, 1[, alors sup | fn (x) | ≤ an et la série fn converge normalement
x∈[0,a] n≥0
sur [0, a] pour tout a ∈ [0, 1[.
X
Soit a ∈]1, +∞[, alors sup | fn (x) | ≤ a−n et la série fn converge normale-
x∈[a,+∞] n≥0
ment sur [a, +∞[ pour tout a ∈]1, +∞[.
X
Enfin, fn (1) diverge. Donc la série converge normalement sur tout compact de
n≥0
IR+ \ {1}.
34 Suites et séries de fonctions

Solution .15 Distinguons plusieurs cas:


P
1. a ∈ IN. Dans ce cas un (x) = 0 pour tout n > a et la série un (x) converge.
2. a ∈/ IN. Alors ici, nous distinguons aussi plusieurs cas:
P
2.1. | x | 6= 1. Alors la règle de D’Alembert montre que un (x) converge si | x | < 1
P
et que un (x) diverge si | x | > 1.
un+1 (1) a−n a+1
2.2. x = 1. Alors = = − 1.
un (1) n +P1 n+1
2.2.1. Si a + 1 > 0. La série un (1) est alternée à partir d’un certain rang
n0 = E(a) + 1. De plus, en utilisant le fait que 1 − x ≤ e−x ,
µ ¶
| un+1 (1) | a+1 a+1
∀ n ≥ n0 , =1− ≤ exp −
| un (1) | n+1 n+1
d’où, Ã !
n
X 1
∀ n ≥ n0 , | un (1) | ≤ | un0 (1) | exp −(a + 1)
k
k=n0 +1
P
alors lim | un (1) | = 0, et la série un (1) converge d’après le critère de convergence
n→∞
des séries alternées.
| un+1 (1) | P
2.2.2. Si a + 1 ≤ 0. Alors ≥ 1, et la série un (1) diverge car son terme
| un (1) |
général ne tend pas vers zéro.
un+1 (−1) n−a a+1
2.3. x = −1. Alors = =1− .
un (−1) n+1 n+1
Les termes de la série considérée gardent un sign constant à partir d’un certain
rang.
2.3.1. Si a > 0. On choisit α ∈]1, 1 + a[ et on pose vn = n−α . Un calcul simple
vn+1 | un+1 (−1) |
montre qu’à partir d’un certain n0 nous avons ≥ et par conséquent
P P vn | un (−1) |
la série un (−1) converge car vn convege.
2.3.2. Si a < 0. On choisit α ∈]1 + a, 1[ et on pose vn = n−α . Un calcul simple
vn+1 | un+1 (−1) |
montre qu’à partir d’un certain n0 nous avons ≤ et par conséquent
P P vn | un (−1) |
la série un (−1) diverge car vn divege.
Résumons,
P
si a ∈ IN, un (x) converge pour tout x ∈ IR
P
si a ∈ IR+ \ IN, un (x) converge si, et seulement si x ∈ [−1, 1]
P
si a ∈] − 1, 0[, un (x) converge si, et seulement si x ∈] − 1, 1]
P
si a ∈] − ∞, −1[, un (x) converge si, et seulement si x ∈] − 1, 1[
Solutions 35

Solution .16 1◦ . La fonction fn est croissante sur IR alors, pour tout a ∈ IR+,
sup | fn (x) | = max( th n − th (n − a), 1 − th n) ≤ 1 − th (n − a) ≤ ea−n .
x≥−a
P
On conclut que la série fn converge uniformément sur tout intervalle [b, +∞[ avec
b ∈ IR.
2◦ . La convergence simple de la série vers S et le fait que chaque fn est croissante
sur IR montrent que S est croissante sur IR. D’autre part, la continuité de chaque fn ,
P
et la convergence uniforme sur tout compact de fn montrent la continuité de S sur
IR.
3◦ . En effet,

X
S(x + 1) − S(x) = ( th (x + n + 1) − th (x + n))
n=0
m−1
X
= lim ( th (x + n + 1) − th (x + n))
m→∞
n=0
= lim ( th (x + m) − th x) = 1 − th x
m→∞
ce qui démontre le résultat.

Solution .17 Notons que si x ≤ 0 alors (e−x n )n≥0 ne tend pas vers 0, et la série
P −x√n √
e diverge. Par contre, si x > 0 alors lim n2 e−x n = 0 et par conséquent la
n→∞
P −x√n
série e converge. On conclut que nous définissons une fonction f sur IR∗+ par

X √
f (x) = e−x n .
n=0 √
En utilisant la décroissance, pour x > 0, de la fonction t 7→ ex t
, nous obtenons
√ √ √
x n−1 x t x n
∀ n > 0, ∀ t ∈ [n − 1, n], e ≥e ≥e
d’où, Z
√ n √ √
x n−1
∀ n > 0, e ≥ ex t
dt ≥ ex n
n−1
ce qui implique,
m−1
X √
Z m √ m
X √
x n x t
∀ m > 0, e ≥ e dt ≥ ex n

n=0 0 n=0
et en faisant tendre m vers l’infini
Z ∞ √
f (x) ≥ e−x t
dt ≥ f (x) − 1.
0
Z ∞ √
Z ∞
−x t 2
Mais, e dt = 2 ue−xu du = . Alors ∀ x ∈ IR∗+ , 2 ≤ x2 f (x) ≤ 2 + x2 .
0 0 x2
2
On conclut que, f (x) ∼+ 2 .
0 x
36 Suites et séries de fonctions

1
Solution .18 Posons fn (x) = . L’ensemble de définition commun à toutes
n + n2 x
les fonctions (fn )n≥1 diminué de {0} (qui est un point de divergence évident) est
 
[¸ 1 1
·
D = IR \ K0 =] − ∞, −1[∪  − ,−  ∪] 0, +∞[.
k k+1
k≥1

1 P
avec K0 = {− : k ≥ 1} ∪ {0}. Montrons que la série fn converge uniformément sur
k
tout compact K contenu dans D. En effet, comme K ∩ K0 = Ø alors
½¯ ¯ ¾
¯ 1 ¯¯
¯ ∗
d(K, K0 ) = inf d(x, K0 ) = inf ¯ x + ¯ : (x, n) ∈ K × IN > 0
x∈K n

1 1 P
et par conséquent sup | fn (x) | ≤2
et la série fn converge uniformément
x∈K n d(K, K0 )
X∞
sur K. On conclut que la fonction f définie sur D par f (x) = fn (x) est continue sur
n=1
D.

– Au voisinage de +∞.
X∞
1 π2
Notons que 2
= . Alors, pour tout x ∈ IR∗+ ,
n=1
n 6

X∞ µ ¶ X∞
π2 x 1 1
xf (x) − = 2
− 2 =− 2
.
6 n=1
n+n x n n=1
n (1 + nx)
¯ ¯ ∞
¯
¯ π 2 ¯
¯ 1X 1 M π2 1
et par conséquent, ¯ xf (x) − ¯ ≤ 3
= . Ce qui s’écrit f (x) = + O( 2 )
6 x n=1 n x 6x x
au voisinage de l’infini.

– Au voisinage de 0+ .
1
En utilisant la décroissance, pour x > 0, de la fonction t 7→ , nous obtenons
t + t2 x
1 1 1
∀ n > 1, ∀ t ∈ [n − 1, n], 2
≥ 2

(n − 1) + (n − 1) x t+t x n + n2 x

d’où, Z n
1 dt 1
∀ n > 1, ≥ ≥
(n − 1) + (n − 1)2 x n−1
2
t+t x n + n2 x
ce qui implique,
m−1
X Z m m
X
1 dt 1
∀ m > 1, ≥ ≥
n=1
n + n2 x 1 t + t x n=2 n + n2 x
2
Solutions 37

et en faisant tendre m vers l’infini,


Z ∞
dt 1
f (x) ≥ 2
≥ f (x) − .
1 t+t x 1+x
Z ∞
dt 1
Mais, 2
= Log (1 + ). Alors,
1 t+t x x
1 1 1
∀ x ∈ IR∗+ , Log (1 + ) ≤ f (x) ≤ Log (1 + ) + .
x x 1+x
1
On conclut que, f (x) ∼+ Log .
0 x
1 P
Solution .19 Posons fn (x) =
2 2
. Clairement, la série fn (0) diverge. Par
n+n x
1 P
contre si a > 0 alors sup | fn (x) | ≤ , et la série fn converge normalement sur
| x |≥a an2
tout ensemble de la forme ]−∞, −a]∪[a, +∞[ avec a > 0. Par conséquent f est continue
sur IR∗ .
P
Pour x > 0, la série fn converge normalement sur le segment joignant 1 à x, et
alors Z √
x ∞ Z
X x X∞ Z x n
dt du
f (t) dt = = √
1 n=1 1 n + n2 t 2 n=1

n n n(1 + u2 )
X∞
1 ¡ √ √ ¢
√ Arctg (x n) − Arctg ( n)
=
n=1
n n
X∞ µ ¶
1 π √ 1
= √ − Arctg n − Arctg √ .
n=1
n n 2 x n
X Arctg (x√n)
∞ X∞ √
Arctg (x/ n)
Posons alors G(x) = √ et H(x) = √ . Il est immédiat que
n=1
n n n=1
n n
les séries définissant G et H sont normalement convergentes sur IR, car la fonction Arctg
prend ses valeurs dans ] − π/2, π/2[. Il en résulte que G et H sont continues sur IR. Or
nous avons
Z ∞
x
πX 1 1
f (t) dt = G(x) − G(1) = √ − G(1) − H( ).
1 2 n=1 n n x
D’où,
Z x
lim f (t) dt =G(0) − G(1) = −G(1)
>
x→0 1
Z ∞ ∞
πX 1
x
πX 1
lim f (t) dt = √ − G(1) − H(0) = √ − G(1)
x→∞ 1 2 n=1 n n 2 n=1 n n
Z ∞ ∞
πX 1
ce qui démontre que l’intégrale f (t) dt converge et vaut √ .
0 2 n=1 n n
38 Suites et séries de fonctions

tn P
Solution .20 Posons fn (t) =
sin nt. Si | t | > 1 et la série fn (t) converge, alors
n
lim sin nt = 0, il en résulte que lim | cos nt | = 1 puis que
n→∞ n→∞

| sin(n + 1)t − sin nt cos t |


| sin t | = lim = 0.
n→∞ | cos nt |
P
On conclut que si | t | > 1 et la série fn (t) converge, alors t ∈ πZZ \ {0}. Inversement,
P
pour tout t ∈ πZZ \ {0} la série fn (t) converge et elle est de somme nulle.
P
Si t ∈ {−1, 1} alors d’après la règle d’Abel nous savons que fn (t) converge. Enfin,
n P
si | t | < 1 alors | fn (t) | ≤ | t | et la fn (t) converge absolument. Nous concluons que
P
la série fn converge simplement sur [−1, 1] ∪ πZZ et diverge sur le complémentaire de
cet ensemble.
Notons que pour tout n ≥ 1 la fonction fn est de classe C 1 sur IR. Si a ∈ [0, 1[,
alors
¯ ¯
sup | fn0 (t) | = sup ¯ tn−1 sin nt + tn cos nt ¯ ≤ 2an−1 .
t∈[−a,a] t∈[−a,a]
P
La série fn0 converge uniformément sur tout intervalle [−a, a] avec a ∈ [0, 1[.

X
Il en résulte que la fonction f définie par f (t) = fn (t) est de classe C 1 avec
n=1

X
f 0 (t) = fn0 (t) sur tout intervalle [−a, a], (a ∈ [0, 1[). On conclut que f est de classe
n=1

X
0
1
C avec f (t) = fn0 (t) sur ] − 1, 1[.
n=1
Fixons t ∈] − 1, 1[ et x ∈ IR. Un calcul simple montre que

m
à m
! Ã m−1
!
X X X
tn−1 sin nx =Im tn−1 einx = Im eix (teix )n
n=1 n=1 n=0
µ ix m

ix 1 − (te )
=Im e .
1 − teix

Il en résulte, en faisant tendre m vers l’infini, que



X µ ¶
n−1 eix sin x
t sin nx = Im = .
n=1
1 − teix 1 − 2t cos x + t2
P
Fixons x ∈ IR. Pour y ∈] − 1, 1[, la série tn−1 sin nx converge normalement (en t) sur
le segment joignant 0 à y, on peut alors intégrer, entre 0 et y, terme à terme:

X∞ Z y
yn sin x y sin x
sin nx = 2
dt = Arctg .
n=1
n 0 1 − 2t cos x + t 1 − y cos x
Solutions 39

Nous arrivons, en prenant y = x = t ∈] − 1, 1[, à

X∞ n
t t sin t
∀ t ∈] − 1, 1[, f (t) = sin nt = Arctg
n=1
n 1 − t cos t

sin t + t cos t − t2
puis en dérivant f 0 (t) = .
1 − 2t cos t + t2

Solution .21 En utilisant la somme d’une suite géométrique nous arrivons à


m−1
X sin πt
∀ t ∈ [0, 1], tn sin πt = (1 − tm )
n=0
1−t

sin πt
(Notons que t 7→ est prolongeable par continuité sur [0, 1] et par conséquent elle
1−t ¯ ¯
¯ sin πt ¯
est bornée sur [0, 1]. On pose M = sup ¯¯ ¯). On conclut que
¯
t∈[0,1] 1 − t

¯ m−1 Z Z 1 ¯ Z ¯ ¯
¯X 1 sin πt ¯ 1¯
sin πt ¯¯ m M
¯ n ¯ ¯
¯ t sin πt dt − dt ¯ ≤ ¯ ¯ t dt ≤ .
¯ 0 1−t ¯ 1−t m+1
n=0 0 0

Il suffit alors de faire tendre m vers l’infini pour obtenir


∞ Z
X 1 Z 1 Z π
n sin πt sin u
t sin πt dt = dt = du.
n=0 0 0 1−t 0 u

(Log t)n P
Solution .22 Posons fn (t) = . Pour tout x > 0, la série fn converge
n!

X
normalement sur le segment joignant 1 à x, et fn (t) = t − 1. On peut alors intégrer
n=1
sur le segment joignant 1 à x terme à terme:

X∞ Z
1 x (x − 1)2
∀ x > 0, (Log t)n dt = .
n=1
n! 1 2

Solution .23 Remarquons que si, x ∈ IR \ 2πZZ,


n
X 1 sin(n + 1/2)x
Dn (x) = cos kx = + .
2 2 sin(x/2)
k=0

1
Pour α ∈]0, π[, on pose Iα = [α, 2π − α]. On a sup | Dn (x) | ≤ = λα .
x∈Iα sin(α/2)
40 Suites et séries de fonctions

D’autre part, si m > n,


m
X Xm Xm m−1
X
cos kx Dk (x) − Dk−1 (x) Dk (x) D (x)
√ = √ = √ − √ k
k=n+1
k + x k=n+1 k+x k=n+1
k + x k=n k + 1 + x
Xm µ ¶
1 1 Dm (x) Dn (x)
= Dk (x) √ −√ +√ −√
k=n+1
k+x k+1+x m+1+x n+1+x

On conclut que si x ∈ Iα ,
¯ m ¯ µ ¶
¯ X cos kx ¯ Xm
λα λα λα λα
¯ ¯
¯ √ ¯≤ √ −√ +√ +√
¯ k + x ¯ k + x k + 1 + x m + 1 + x n+1+x
k=n+1 k=n+1
µ ¶
1 1 1 1
≤λα √ −√ +√ +√
n+1+x m+1+x m+1+x n+1+x
2λα 2λα
=√ ≤√
n+1+x n+1
Il arrive que, pour tout α ∈]0, π[
¯ m ¯
¯ X cos kx ¯ 2λα
¯ ¯
m > n > 1 =⇒ sup ¯ √ ¯≤ √ .
x∈Iα ¯ k+x¯ n+1
k=n+1

X∞
cos kx
Il en résulte que la série √ converge uniformément sur tout compact contenu
n=0
k + x
dans ]0, 2π[. La somme est, par conséquent, continue sur ]0, 2π[.

Solution .24 Pour n ≥ 1 nous considérons



 0 si x=0
fn (x) =
 n
x Log x si x ∈]0, 1]
Il est immédiat que, pour tout n ≥ 1, fn est une fonction continue sur [0, 1]. De plus
X∞
−1/n 1
sup | fn (x) | = −fn (e )= . Alors la série fn est uniformément convergente
x∈[0,1] en n=1
sur [0, 1]. Notons f la somme de cette série, elle est continue sur [0, 1] à cause de la
x Log x
convergence uniforme. On voit immédiatement que si x ∈]0, 1[, alors f (x) = .
1−x
On peut, toujours en utilisant la convergence uniforme, écrire,
Z 1 ∞ Z 1
X
f (x) dx = fn (x) dx.
0 n=1 0
Z 1
1
Une intégration par parties simple montre que tn (Log t) dt = − , d’où
0 (n + 1)2
Z 1 X∞
1
f (x) dx = − 2
.
0 n=2
n
Solutions 41

Log x
Enfin, f (x) = −Log x − pour tout x ∈]0, 1[. La fonction f est continue sur [0, 1]
Z 1 x−1 Z 1
Log x
et l’intégrale Log x dx est convergente et égale à −1, alors l’intégrale dx
0 0 x−1
Z 1 Z 1
converge et vaut − f (x) dx − Log x dx. On conclut que
0 0
Z 1 X∞
Log x 1 π2
dx = =
0 x−1 n=1
n2 6

Solution .25 1◦ . Remarquons que

sup | un (x) | = un (a), sup | vn (x) | = −vn (a).


x≥a x≥a
P P
Il en résulte que un et vn convergent uniformément sur [a, +∞[. En notant que
X
un est de classe C 1 sur IR∗+ et que u0n (x) = vn (x), nous déduisons que f = un est
n≥1
X
1 0
de classe C et de dérivée f = vn sur [a, +∞[, pour tout a > 1. Il en résulte que ce
n≥1
même résultat est valable sur ]1, +∞[.
P
2◦ . pour tout x > 0, la série wn (x) est convergente d’après le critère de
convergence des séries alternées, de plus
¯ ¯
¯ Xn ¯ 1
¯ ¯
∀ x > 0, ¯ g(x) − wk (x) ¯ ≤ .
¯ ¯ (n + 1)x
k=1
P
ce qui démontre que la série wn converge uniformément sur tout intervalle de la forme
[ε, +∞[ avec ε > 0. La fonction g est alors continue sur ]0, +∞[.
X∞
Log n
En utilisant le même argument nous démontrons que (−1)n x converge
n=1
n
Log n
uniformément sur tout intervalle de la forme [ε, +∞[ avec ε > 0, (Notons que n 7→
nx
1/ε
est décroissante à partir de nε = 1 + E(e ) et ceci pour tout x ≥ ε). Alors g est de
X∞
0 Log n
1
classe C sur ]0, +∞[ avec g (x) = (−1)n x .
n=1
n
Enfin, si x > 1, nous avons
X∞ X∞
1 1
g(x) = x

n=0
(2n + 1) n=1
(2n)x

X X 1 ∞
1
f (x) = +
n=0
(2n + 1)x n=1 (2n)x
1
alors, f (x) − g(x) = f (x). Ce qui démontre,
2x−1
∀ x > 1, g(x) = (1 − 21−x ) f (x).
42 Suites et séries de fonctions

Solution .26 1◦ . Nous voyons immédiatement que


¯ ¯
¯ 2x ¯ max(| a | , | b |)

∀ n ∈ IN , sup | un (x) | = sup ¯¯ 2 ¯≤
¯ .
x∈[a,b] n + x2
x∈[a,b] n2

X
Alors, la série un définissant S est uniformément convegente sur tout intervalle
n=1
compact [a, b] de IR.
2◦ . En utilisant l’inégalité simple Log (1 + t) ≤ t, valable pour t ≥ 0, nous avons
x2 max(a2 , b2 )
∀ n ∈ IN∗ , sup | vn (x) | = sup Log (1 + ) ≤ .
x∈[a,b] x∈[a,b] n2 n2

X
Alors, la série vn est uniformément convegente sur tout intervalle compact [a, b] de
n=1
2 ¯ 2 ¯
IR. D’autre part, | wn (x) | ≤ , car ¯ n − x2 ¯ ≤ n2 + x2 , d’où
n2 + x2
2
∀ n ∈ IN∗ , sup | wn (x) | ≤ 2 .
x∈IR n

X
Alors, la série wn est uniformément convegente sur IR.
n=1
Remarquons que un est de classe C 1 sur IR et que u0n = wn . La convergence
P
uniforme IR de la série wn montre alors que S est de classe C 1 sur IR, et que
X∞
S0 = wn . De plus ∀ n ∈ IN∗ , ∀ x ∈ [−1, 1], wn (x) ≥ 0, avec inégalité stricte si
n=1
n > 1. Alors S est strictement croissante sur [−1,
Z x 1].
Finalement, en remarquant que vn (x) = un (t) dt et en utilisant la convergence
0
Z x X∞
P
uniforme de un sur tout compact, nous obtenons S(t) dt = vn (x) = H(x).
0 n=1
3◦ . Considérons, pour n ≥ 0,
"µ ¶2n+1 µ ¶2n+1 #
1 X X
Pn (X) = 1+ − 1− .
2 2n + 1 2n + 1
Un calcul simple montre que les racines de Pn sont
½ ¾
πk
−i(2n + 1)tg : k = 0, 1, . . . , 2n .
2n + 1
1
Comme le coefficient de X 2n+1 de Pn est , alors
2n + 1
2n µ ¶
X Y πk
Pn (X) = X + i(2n + 1)tg
2n + 1 2n + 1
k=1
n µ ¶
X Y 2 2 2 πk
= X + (2n + 1) tg
2n + 1 2n + 1
k=1
Solutions 43
n
Y
1 πk
Mais le coefficient de X dans Pn est 1, alors (2n + 1)2 tg 2 = 1. D’où,
2n + 1 2n + 1
k=1
 
n
Y   X2
Pn (X) = X 1 + .
2 2
πk
k=1 (2n + 1) tg
2n + 1
³ x ´m

4 . En utilisant que ∀ x ∈ IR, lim 1 + = ex , nous obtenons
m→∞ m

ex − e−x
lim Pn (x) = = sh x
n→∞ 2

Or pour tout x ∈ [0, π/2[, x ≤ tg x, alors pour tout k ∈ {0, 1, . . . , n} nous avons
πk
l’inégalité π 2 k 2 ≤ (2n + 1)2 tg 2 . D’où
2n + 1

x2 x2
∀ x ∈ IR, ∀ k ∈ {0, 1, . . . , n}, 1+ ≤1+
πk π2 k2
(2n + 1)2 tg 2
2n + 1
ce qui démontre,
n µ
Y ¶
∗ 1 x2
∀ n ∈ IN , ∀ x ∈ IR, Pn (x) ≤ 1+ 2 2 . (1)
x π k
k=1

D’autre part, pour tout x ∈ IR, la condition m > n implique


   
n
Y m
x2  Y x2  1
1 + πk  ≤  1+ = P (x)
πk  x m
k=1 2
(2m + 1) tg 2 k=1 2
(2m + 1) tg 2
2m + 1 2m + 1
donc, en faisant tendre m vers l’infini, nous obtenons
n µ
Y ¶
∗ x2 sh x
∀ n ∈ IN , ∀ x ∈ IR, 1+ 2 2 ≤ . (2)
π k x
k=1

Alors, en combinant (1) et (2),


n µ
Y ¶
∗ 1 x2 sh x
∀ n ∈ IN , ∀ x ∈ IR, Pn (x) ≤ 1+ 2 2 ≤ .
x π k x
k=1

Il suffit alors de faire tendre n vers l’infini pour obtenir


n µ
Y ¶
x2
∀ x ∈ IR, sh x = lim x 1+ 2 2 .
n→+∞ π k
k=1
44 Suites et séries de fonctions

5◦ . Ce qui précède montre


X n µ ¶ n
X
sh πx x2
∀ x ∈ IR, Log ( ) = lim Log 1+ 2 = lim vk (x) = H(x),
πx n→∞ k n→∞
k=1 k=1

π 1
et par conséquent, pour x ∈ IR nous avons S(x) = H 0 (x) = − .
µ ¶ th (πx) x
2 2
1 π x
Un calcul simple montre que S 0 (x) = 2 1 − . Mais, on voit immédiatement
x sh 2 (πx)
que t ≥ 0 =⇒ t ≤ sh t et par conséquent, S 0 (x) > 0 pour tout x ∈ IR. La fontion S
est alors strictement croissante sur IR.
Z ∞
sin ax
Solution .27 Il est facile de voir que l’intégrale dx converge.
0 ex − 1
Pour tout x > 0, et tout m ∈ IN∗ , nous avons,
X m
1 −nx e−mx
− e = .
ex − 1 n=1 ex − 1

Alors, pour tout m ∈ IN∗ ,


¯Z ¯ Z ¯ ¯
¯ ∞ sin ax X m Z ∞ ¯ ∞¯
¯ ¯ sin ax ¯ −mx
− e −nx
sin ax dx ≤ ¯ ¯
¯
¯ 0 ex − 1
¯
¯ ¯ ex − 1 ¯ e dx
0 n=1 0

sin ax
Mais x 7→ est prolongeable par continuité en 0, et tend vers 0 à l’infini donc elle
ex − 1 ¯ ¯
¯ sin ax ¯
est bornée. Posons K = sup ¯¯ x ¯. Nous concluons que pour tout m ∈ IN∗ ,
¯
x>0 e − 1
¯Z m Z ∞
¯ Z ∞
¯ ∞ sin ax X ¯ K
¯ −nx ¯ −mx
¯ − e sin ax dx ¯ ≤ K e dx = .
¯ 0 ex − 1 0 ¯ 0 m
n=1

Ce qui permet d’écrire,


Z ∞ ∞ Z ∞
X
sin ax
x
= e−nx sin ax dx.
0 e − 1 n=1 0

Enfin,
Z ∞ µZ ∞ ¶ · −(n−ia)x ¸∞
−nx −(n−ia)x e
e sin ax dx =Im e dx = Im −
0 0 n − ia 0
µ ¶
n + ia a
=Im = .
n2 + a2 n2 + a2
D’où, en utilisant le résultat de l’exercice précédent
Z ∞ ∞
X
sin ax a π 1
x
= 2 2
= − .
0 e − 1 n=1 n + a 2 th (πa) 2a
Solutions 45

Log xLog (1 − x)
Solution .28 Notons d’abord que la fonction x 7→ est positive sur
x
]0, 1[ et qu’elle est prolongeable par continuité en 1 et équivalente à x 7→ −Log x au
Z 1
Log xLog (1 − x)
voisinage de 0. Il en résulte que l’intégrale dx est convergente.
0 x
Soient x ∈]0, 1[, et n ∈ IN∗ nous avons,

m
X 1 xm+1
xn − =− .
n=0
1−x 1−x

D’où
Xm Z x m+1
xn+1 t
+ Log (1 − x) = − dt
n=0
n + 1 0 1 − t

ou bien,
¯ m ¯
¯ X xn+1 ¯ 1 xm+2
¯ ¯
¯ + Log (1 − x) ¯ ≤
¯ n+1 ¯ m+21−x
n=0

ce qui démontre enfin,


¯ ¯
¯X m
x Log x Log (1 − x)Log x ¯¯
n
xm xLog x K
¯
¯ + ¯≤ ≤ xm
¯ n+1 x ¯ m+2 x−1 m+2
n=0

¯ ¯
¯ xLog x ¯
avec K = sup ¯¯ ¯. Ce qui permet d’écrire, en intégrant entre 0 est 1,
x∈]0,1[ x−1 ¯

¯ Z 1 Z 1 ¯
¯X m
1 Log (1 − x)Log x ¯ K
¯ ¯
∀ m ∈ IN∗ , ¯ n
x Log x dx + dx ¯ ≤ .
¯ n+1 0 0 x ¯ (m + 2)(m + 1)
n=0

Z 1
1
Mais, une intégration par parties montre que xn Log x dx = − , donc
0 (n + 1)2

¯Z ¯
¯ 1 Log (1 − x)Log x m+1
X 1 ¯ K
¯ ¯
∀ m ∈ IN∗ , ¯ dx − ¯≤
¯ 0 x n3 ¯ (m + 2)(m + 1)
n=1

ce qui démontre que


Z 1 X∞
Log (1 − x)Log x 1
dx = 3
.
0 x n=1
n
46 Suites et séries de fonctions

Solution .29 1◦ .a. En utilisant le fait immédiat: ∀ x ∈ IR, E(x + 1) = E(x) + 1 on


voit que la fonction ∆ est 1-périodique sur IR.
1
 2

 x si x ∈ [0, 1/2[ ∆

∆(x) =

1 − x si x ∈ [1/2, 1[
0 1 1
2

Il en résulte que lim ∆(x) = lim ∆(x) = 1/2, et que lim ∆(x) = lim ∆(x) = 0.
< > < >
x → 1/2 x → 1/2 x→1 x→0
La fonction ∆ est alors continue sur IR.
2◦ .a. Remarquons que

m−1
X 1
m > n > 0 =⇒ sup | fm (x) − fn (x) | ≤ 2−k−1 ≤
x∈IR 2n
k=n

car ∀ x ∈ IR, 0 ≤ ∆(x) ≤ 1/2.


On conclut que la suite (fn )n≥1 est uniformément de Cauchy sur IR, elle est donc
uniformément convergente vers une fonction f : IR −→ IR.
2◦ .b. La convergence uniforme de la suite (fn )n≥1 vers f et la continuité de fn
sur IR pour tout n ∈ IN∗ montrent que f est continue sur IR. D’autre part, fn est
1-périodique pour tout n, alors f est aussi 1-périodique. Enfin,

n−1
X
∀ n ≥ 1, ∀ x ∈ IR, 0 ≤ fn (x) ≤ 2−k−1 ≤ 1.
k=0

Il en résulte que ∀ x ∈ IR, 0 ≤ f (x) ≤ 1.


De l’autre côté, pour tout x ∈ IR,

X ∞ X ∞
1
f (x) − f (2x) = 2−k ∆(2k x) − 2−(k+1) ∆(2k+1 x)
2
k=0 k=0
X∞ X∞
= 2−k ∆(2k x) − 2−k ∆(2k x) = ∆(x).
k=0 k=1

3◦ . D’après l’hypothèse, nous avons

∀ x ∈ IR, ∀ k ∈ IN, 2−k h(2k x) − 2−(k+1) h(2k+1 x) = 2−k ∆(2k x).

D’où, en prenant la somme de ces égalités pour k allant de 0 jusqu’à m−1, nous arrivons

∀ x ∈ IR, ∀ m ∈ IN∗ , h(x) − 2−m h(2m x) = fm (x).


Solutions 47

Comme h est bornée sur IR, alors en faisant tendre m vers l’infini dans l’égalité
précédente nous obtenons ∀ x ∈ IR, f (x) = h(x).
4◦ .a. Il est clair que δ est 1-périodique et que
 1
 2x si x ∈ [0, 1/4[ 2

 δ

δ(x) = 1/2 si x ∈ [1/4, 3/4[




2 − 2x si x ∈ [1/4, 1] 0 1
4
1
4
1

4◦ .b. En effet, si x ∈ IR, nous avons


m−1
X m−1
X
f2m (x) = 2−2k ∆(22k x) + 2−2k−1 ∆(22k+1 x) = 2−2k δ(22k x).
k=0 k=0

Mais la fonction δ est majorée par 1/2 donc


m−1 ∞
1 X −2k 1X 1 2
∀ x ∈ IR, f2m (x) ≤ 2 ≤ k
= .
2 2 4 3
k=0 k=0

4◦ .c. Notons que 22k = (1 + 3)k = 1 + 3`k alors,

22k 1 1 1
δ( ) = δ( + `k ) = δ( ) =
3 3 3 2
Il en résulte que
m−1
X m−1
1 −2k 22k 1 X 1 2
f2m ( ) = 2 δ( )= k
= (1 − 4−m ).
3 3 2 4 3
k=0 k=0

2
Par conséquent f (1/3) = 2/3 et sup f (x) = .
x∈IR 3
5◦ .a. En utilisant le fait que E(1 + x) = 1 + E(x) pour tout x, nous réalisons
immédiatement que ²1 est une fonction 1-périodique. De plus,

 0 si x ∈ [0, 1/2[
²1 (x) =

1 si x ∈ [1/2, 0[

ce qui démontre le résultat.


5◦ .b. En effet, fixons x ∈ IR,

Xm m µ ¶
²n (x) X E(2n x) E(2n−1 x)
= −
n=1
2n n=1
2n 2n−1
Xm m−1
E(2n x) X E(2n x) E(2m x)
= n
− n
= m
− E(x)
n=1
2 n=0
2 2
48 Suites et séries de fonctions

Mais 2m x − 1 ≤ E(2m x) ≤ 2m x, donc x − 2−m ≤ 2−m E(2m x) ≤ x et par conséquent


X∞
−m m ²n (x)
lim 2 E(2 x) = x ce qui démontre que x = E(x) + .
x→∞
n=1
2n
5◦ .c. Notons que pour tout x ∈ IR, la série définissant g est normalement
convergente et que
X∞
2+n
| g(x) | ≤ n
= M.
n=1
2
Alors g est une fonction bornée sur IR. D’autre part, comme ²n (2x) = ²n+1 (x), alors

à n
! ∞
à n
!
1 X X ² (x) X X ²n (x)
(−1)²k+1 (x) (−1)²k (x)
n+1
g(2x) = 2+ n+1
= 2+ n
.
2 n=1
2 n=2
2
k=1 k=2

Il en résulte que

1 ³ ´ ² (x) X ²n (x)

g(x) − g(2x) = 2 + (−1)²1 (x) (−1)²1 (x) n
1
+
2 2 n=2
2
X∞
²n (x)
=²1 (x) + (−1)²1 (x)
n=1
2n

=²1 (x) + (−1)²1 (x) (x − E(x)).


1 1
La fonction x 7→ g(x) − g(2x) est alors 1-périodique et vérifie, g(x) − g(2x) = x pour
2 2
1
x ∈ [0, 1/2[ et g(x) − g(2x) = 1 − x si x ∈ [1/2, 1[. Nous en déduisons l’égalité
2
1
g(x) − g(2x) = ∆(x) pour tout x ∈ IR. En utilisant le résultat de 3◦ . Il arrive que
2
g = f.
5◦ .d. En utilisant ce qui précède
m+n+1
à m
!
X X X̀ 1
f (ym,n ) − f (xm ) = 2+ (−1)²k + (−1)
2`
`=m+1 k=1 k=m+1
m+n+1
à m
!
X X 1
= 2+ (−1)²k + m − `
2`
`=m+1 k=1
n+1
à m
!
X X 1
= 2+ ²
(−1) k − `
2`+m
`=1 k=1
à m
! n+1 n+1
1 X X 1 1 X `
= m 2+ (−1) ²k − m .
2 2` 2 2`
k=1 `=1 `=1

Mais f est continue, alors lim f (ym,n ) = f (ym ), donc


n→∞
à m
! ∞ ∞
1 X X 1 1 X `
f (ym ) − f (xm ) = m 2+ (−1)²k − m
2 2` 2 2`
k=1 `=1 `=1
Solutions 49

X∞ X∞ X∞ X∞ ∞ ∞
1 ` ` ` `+1 X ` X 1
Enfin, `
= 1 et `
= ( `−1 − ` ) = `
− `
=1+ = 2, ce
2 2 2 2 2 2 2`
`=1 `=1 `=1 `=0 `=1 `=1
qui démontre que
à m
! m
1 X 2 1 X
f (ym ) − f (xm ) = m 2+ (−1)²k − m = m (−1)²k
2 2 2
k=1 k=1

et par conséquent,
m
f (ym ) − f (xm ) X
= (−1)²k (x) . (†)
ym − xm
k=1

Supposons f dérivable en un point x ∈ [0, 1[. Comme lim xm = lim ym = x.


m→∞ m→∞
Alors
f (x) − f (xm ) f (ym ) − f (x)
lim = f 0 (x), lim = f 0 (x).
m→∞ x − xm m→∞ ym − x
Donc pour ε > 0, il existe m0 tel que

 | f (x) − f (xm ) − f 0 (x)(x − xm ) | ≤ ε(x − xm )
m ≥ m0 =⇒

| f (ym ) − f (x) − f 0 (x)(ym − x) | ≤ ε(ym − x)

d’où, pour tout m ≥ m0

| f (ym ) − f (xm ) − f 0 (x)(ym − xm ) | ≤ ε(ym − xm ).

Nous avons donc démontré que si f est dérivable en x, alors

f (ym ) − f (xm )
lim = f 0 (x)
m→∞ ym − xm

m−1
X
ou bien, en utilisant (†), lim (−1)²k (x) = f 0 (x). En particulier si f est dérivable
m→∞
k=1
P
en x la série (−1)²n (x) converge ce qui est absurde car le terme général de cette série
ne tend pas vers zéro, f n’est donc pas dérivable en x. En utilisant la 1-périodicité, nous
concluons que f n’est nulle part dérivable.

OKMRAN
OUBA
50 Intégrales dépendant d’un paramètre

INTÉGRALES DÉPENDANT D’UN PARAMÈTRE

Dans ce chapitre IK désigne IR ou C


|

Nous utiliserons les notations générales suivantes: I est un


intervalle de IR non réduit à un point, (a, b) est un couple de
IR2 avec a < b, et f une application de [a, b] × I dans IK.

Le lemme suivant est très utile dans la suite:

Lemme 1. Si f est continue sur [a, b] × I, et x0 ∈ I alors

∀ ε > 0, ∃ η > 0, ∀ x ∈ I, | x − x0 | < η =⇒ sup | f (t, x) − f (t, x0 ) | < ε.


t∈[a,b]

Preuve : Nous raisonnons par l’absurde. Si ce n’est pas vrai alors



 1
 | xn − x0 | <
∃ ε0 > 0, ∀ n ∈ IN∗ , ∃ (tn , xn ) ∈ [a, b] × I, n
 et

| f (tn , xn ) − f (tn , x0 ) | ≥ ε0 .

Mais [a, b] est un intervalle compact, donc il existe ϕ : IN∗ −→ IN∗ strictement croissante
telle que la suite (tϕ(n) )n∈IN∗ converge vers t0 ∈ [a, b]. D’autre part, la suite (xϕ(n) )n∈IN∗
converge vers x0 par construction. La continuité de f au point (t0 , x0 ) montre que

f (tϕ(n) , xϕ(n) ) −−−→ f (t0 , x0 ), et f (tϕ(n) , x0 ) −−−→ f (t0 , x0 ).


n→∞ n→∞

D’où, lim (f (tϕ(n) , xϕ(n) ) − f (tϕ(n) , x0 )) = 0 ce qui est en contradiction avec le fait
n→∞ ¯ ¯
que, pour tout n ∈ IN, ¯ f (tϕ(n) , xϕ(n) ) − f (tϕ(n) , x0 ) ¯ ≥ ε0 . D’où le résultat.

Théorème 2. Si f est continue sur [a, b] × I, alors l’application


Z b
F : I −→ IK : x 7→ F (x) = f (t, x) dt
a

est continue sur I.


Solutions 51

Preuve : Soit x0 ∈ I. Nous allons démontrer la continuité de F en x0 .


Soit ε > 0, d’après le lemme 1, il existe η > 0 tel que, pour tout x ∈ I qui vérifie
| x − x0 | < η, on a
ε
∀ t ∈ [a, b], | f (t, x) − f (t, x0 ) | ≤ ,
b−a
d’où, en intégrant par rapport à t sur [a, b],
¯Z ¯ Z
¯ b ¯ b
¯ ¯
| F (x) − F (x0 ) | = ¯ (f (t, x) − f (t, x0 )) dt ¯ ≤ | f (t, x) − f (t, x0 ) | dt ≤ ε.
¯ a ¯ a

Théorème 3. Supposons que f est continue sur [a, b] × I et qu’elle est dérivable par
∂f
rapport à x avec aussi continue sur [a, b] × I. Alors l’application
∂x
Z b
F : I −→ IK : x 7→ F (x) = f (t, x) dt
a
Z b
∂f
est de classe C 1 sur I avec F 0 (x) = (t, x) dt pour tout x ∈ I.
a ∂x

Preuve : Soit x0 ∈ I. Nous allons démontrer que F est dérivable en x0 .


Soit ε > 0, d’après le lemme 1, il existe η > 0 tel que, pour tout x ∈ I qui vérifie
| x − x0 | < η, on a
¯ ¯
¯ ∂f ∂f ¯
∀ t ∈ [a, b], ¯ (t, x) − (t, x ) ¯≤ ε .
¯ ∂x ∂x
0 ¯ b−a

Or, pour tout x ∈ I∩]x0 − η, x0 + η[, le théorème des accroissements finis montre
l’existence de ξx entre x0 et x tel que,
f (t, x) − f (t, x0 ) ∂f
= (t, ξx ),
x − x0 ∂x
En combinant ces deux résultats, pour tout x ∈ I∩]x0 − η, x0 + η[, on a
¯ ¯
¯ f (t, x) − f (t, x0 ) ∂f ¯ ε
∀ t ∈ [a, b], ¯ − (t, x0 ) ¯¯ ≤ .
¯ x − x0 ∂x b−a
D’où, en intégrant par rapport à t sur [a, b], pour tout x ∈ I∩]x0 − η, x0 + η[,
¯ ¯
¯ F (x) − F (x ) Z b ∂f ¯
¯ 0 ¯
¯ − (t, x0 ) dt ¯ ≤ ε.
¯ x − x0 a ∂x ¯
Z b
0 ∂f
Ce qui démontre que F est dérivable en x0 ∈ I avec F (x0 ) = (t, x0 ) dt. Le
a ∂x
théorème 2 montre alors que F 0 est continue sur I.
52 Intégrales dépendant d’un paramètre

Théorème 4. Si I = [c, d] et f est continue sur ∆ = [a, b] × [c, d], alors


Z ÃZ b
!
d Z ÃZ !
d b
f (t, x) dx dt = f (t, x) dt dx.
a c c a

Preuve : D’abord, comme f est une application continue sur le compact ∆, elle est
bornée, on pose alors M = sup | f |.

Définissons, pour (y, x) ∈ ∆,
Z y
H(y, x) = f (t, x) dt.
a

Nous allons prouver que H est cotinue sur ∆. En effet, soit (y0 , x0 ) ∈ ∆, et ε > 0.
Dáprès le lemme 1, il existe η > 0 tel que, pour tout x ∈ [c, d] qui vérifie | x − x0 | < η,
on a
ε
∀ t ∈ [a, b], | f (t, x) − f (t, x0 ) | ≤ .
2(b − a)
ε
Par conséquent, pour tout (y, x) ∈ ∆ qui vérifie | x − x0 | < η et | y − y0 | < ,
(2M + 1)
on a
¯Z y0 Z y ¯
¯ ¯
| H(y, x) − H(y0 , x0 ) | = ¯¯ [f (t, x) − f (t, x0 )] dt + f (t, x) dt ¯¯
a y0
≤(b − a) sup | f (t, x) − f (t, x0 ) | + M | y − y0 | ≤ ε.
t∈[a,b]

Ce qui prouve la continuité de H sur [a, b] × [c, d].


∂H
D’autre part, = f est aussi continue sur [a, b] × [c, d]. Donc en utilisant le
∂y
théorème 3. l’application
Z d
G : [a, b] −→ IK : y 7→ G(y) = H(y, x) dx
c
Z d
1 0
est de classe C avec G (y) = f (y, x) dx. Alors
c

Z Z ÃZ !
b b d
G(b) = G(b) − G(a) = G0 (y) dy = f (y, x) dx dy.
a a c

et Z Z ÃZ !
d d b
G(b) = H(b, x) dx = f (t, x) dt dx.
c c a

Ce qui démontre le résultat.


Exercices 53

EXERCICES

Z 1
x
Exercice .1 Soient f (t, x) = Log (1 + t ), et F (x) = f (t, x) dt. Montrer que
0
F (x) est définie pour tout x ∈ IR+ , et que F est continue sur IR∗+ . Peut-on affirmer que
F 0 existe sur x ∈ IR∗+ ?
Exercice .2 Soient a et b deux réels tels que a < b, et g une fonction continue
Z b
sur [a, b] à valeurs dans IR. On pose f (t, x) = cos(xt)g(t) et F (x) = f (t, x) dt.
a
Montrer que F ∈ C ∞ (IR, IR), et trouver F (n) (x), pour n ∈ IN∗ . Enfin, montrer que
lim F (x) = 0.
x→∞
Exercice .3
1
1◦ . Trouver les valeurs de x telles que la fonction f (t, x) = est intégrable
Z π 1 + x cos t
sur [0, π]. Calculer alors f (t, x) dt.
Z 0π
cos t
2◦ . En déduire la valeur de dt.
0 (1 + x cos t)2
Z π
◦ (cos t)k
3 . Calculer 3
dt, pour k ∈ {0, 1, 2}. Trouver ensuite les valeurs de
0 (1 + x cos t)
1
(a, b) ∈]0, +∞[×IR pour lesquelles la fonction définie par ϕ(t) = est
Z π a + b cos t
(cos t)k
intégrable sur [0, π]. Calculer, alors, dt, pour k ∈ {0, 1, 2}.
cos t)3
0 (a + b Z
1
dt
Exercice .4 Étudier les variations de F (x) = p . Donner un
0 (1 − t2 )(x2 − t2 )
équivalent de F (x) au voisinage de 1+ et au voisinage de l’infini.
Exercice
Z ∞ .5 Soit f une fonction définie sur IR+ continue et décroissante telle que
f (t) dt converge. Pour n ∈ IN∗ on pose un (x) = f (nx).
0 P
1◦ . Montrer que, pour tout x > 0, la série un (x) est à termes positifs, convergente
et que
Z ∞ ∞
X Z ∞
f (tx) dt ≤ un (x) ≤ f (tx) dt.
1 n=1 0


2 . Montrer que la somme S de la série de terme général un est continue sur IR∗+ .
Z ∞
3◦ . Montrer que lim xS(x) = f (u) du. Dans quel cas la somme de la série de terme
x→0 0
général vn , où vn (x) = xun (x) est-elle continue
Z ∞ −txsur IR+ ?
e
Exercice .6 Soient p ∈]1, +∞[, et g(x) = dt. Montrer que cette intégrale
0 1 + tp
est convergente sur IR+ . Montrer que g est de classe C ∞ sur IR∗+ .
54 INTÉGRALES DÉPENDANT D’UN PARAMÈTRE
Z ∞
2
Exercice .7 Soit f (x) = e−t cos(2xt) dt. Montrer que f est de classe C 1 sur IR.
0 Z ∞ √
0 −t2 π
Montrer que f (x)+2xf (x) = 0 et en déduire f . ( On admettra que e dt = ).
Z ∞ µ ¶ 0 2
2 x2
Exercice .8 Soit, pour x ≥ 0, F (x) = exp −t − 2 dt. Montrer que F est
0 t
continue, bornée sur IR+ et de classe C 1 sur IR∗+ . Montrer que F 0 = −2F , en déduire
F.
Exercice .9 Soit f : IR+ −→ IR une fonction continue. On suppose qu’il existe a et
b dans IR∗+ et m ∈ IN tels que

∀ t ∈ IR+ , | f (t) | ≤ atm + b


Z ∞
On pose, pour x > 0, g(x) = f (t)e−tx dt.
0
1◦ . Montrer que cette intégrale est convergente sur IR∗+ .
2◦ . Montrer que g est de classe C ∞ sur IR∗+ . Calculer g (n) , pour n ∈ IN∗ .
3◦ . Montrer que lim g(x) = lim g 0 (x) = 0.
x→∞ x→∞
4◦ . On considère  1 − cos t
 si t>0
f (t) = t2

1/2 si t=0
Caluler g 00 et en déduire g pourZx > 0. Montrer que g est continue sur IR+ . Calculer

sin t
g(0), puis donner la valeur de dt.
0 t
Exercice .10 Pour t ∈ [0, π/2[ et x ∈ [0, π/2], on pose
Z π/2
Log (1 + cos x cos t)
f (t, x) = , et F (x) = f (t, x) dt.
cos t 0

1◦ . Montrer que F est continue et dérivable sur [0, π/2].


2◦ . Calculer F 0 (x) pour x ∈ [0, π/2].
3◦ . En déduire une expression simple de F (x) pour x ∈ [0, π/2].
4◦ . Utiliser ce qui précède pour évaluer l’intégrale suivante:
Z 1
Log (1 + u)
√ du.
0 u 1 − u2
Z ∞
dt
Exercice .11 On pose .
0 tx (t+ 1)
1◦ . Étudier la définition de f .
2◦ . Donner un équivalent de f en 0.
3◦ . Montrer que f admet une symétrie.
Exercices 55

4◦ . Montrer que f est continue sur son ensemble de définition.


5◦ . Calculer la borne inférieure de f .
Exercice .12 Étudier l’existence et la dérivabilité de
Z ∞
Log (x2 + t2 )
x 7→ f (x) = dt.
0 1 + t2
Trouver une expression simple de f (x).
Exercice .13 Soit g la fonction définie par la relation
Z ∞
cos(ax)
g(a) = dx. (1)
0 1+x
1◦ . Démontrer que la fonction g est définie pour toute valeur de a différente de 0 et
qu’elle est une fonction paire.
Dans la suite, le réel a sera supposé strictement positif.
2◦ . Soient n ∈ IN∗ , gn la fonction définie par
Z n
cos(ax)
gn (a) = dx.
0 1+x
Démontrer que la fonction gn est continue ; démontrer que lorsque a vérifie
l’inégalité a ≥ a0 , où a0 est un réel donné strictement positif, la suite des fonctions
gn converge uniformément vers g.
3◦ . Démontrer les relations suivantes
Z ∞
cos(u)
g(a) = du. (2)
0 a+u
Z ∞ Z ∞
cos(v) sin(v)
g(a) = cos a dv + sin a dv. (3)
a v a v
Justifier la convergence des deux intégrales figurant dans la relation (3).

4 . En déduire que l’expression g(a) + (Log a) cos a admet une limite finie lorsque a
tend vers 0 (par valeurs positives). Déterminer un infiniment grand équivalent à
g(a) en 0.
5◦ . Démontrer Zque l’expression a | g(a) | est bornée lorsque a croit indéfiniment.

L’intégrale g 2 (a) da est-elle convergente ?
0
Exercice .14 Soient a > b > 0, pour x ∈ IR on pose
Z ∞ −at
e − e−bt
I(x) = cos xt dt
0 t
Montrer que I(x) est bien définie pour tout x ∈ IR. Calculer I(x). (On pourrait
dériver sous le signe somme).
56 INTÉGRALES DÉPENDANT D’UN PARAMÈTRE
Z π/2
Exercice .15 On pose, pour x ≥ −1, G(x) = Log (1 + x sin2 t) dt.
0
1◦ . Montrer que G est continue sur [−1, +∞[ et dérivable sur ] − 1, +∞[.
2◦ . Calculer G0 (x) pour x ∈] − 1, +∞[.
3◦ . En déduire la valeur de G(x), (x ≥ −1).

Exercice .16 Pour x ∈ IR+ , on pose


Z ∞ 2
e−xt
f (x) = dt
0 1 + t2

1◦ . Démontrer que f est définie et continue sur IR+ , et que lim f (x) = 0.
x→∞
2◦ . Démontrer que f est dérivable sur IR∗+ , et que l’on a
Z +∞
0 C 2
f (x) − f (x) + √ = 0, C= e−u du
x 0

3◦ . Résoudre cette équation et en déduire la valeur de C.


Solutions 57

SOLUTIONS

Solution .1 Précisons d’abord que f (t, 0) = Log 2 pour tout t ∈ [0, 1] et que si x > 0,
alors f (t, x) = Log (1 + tx ). La fonction t 7→ f (t, x) est alors continue sur [0, 1] pour
Z 1
tout x ∈ IR+ . Alors F (x) = f (t, x) dt est définie pour tout x ∈ IR+ . Le changement
0
de variable t = e−u permet d’écrire que
Z ∞
∀ x ∈ IR+ , F (x) = Log (1 + e−xu ) e−u du.
0

Fixons u ∈ IR+ , et posons gu (x) = Log (1 + e−xu ). La fonction gu est de classe C ∞


sur IR, de plus, d’après le théorème des accroissements finis, nous avons

| gu (x) − gu (y) | ≤ | x − y | sup | gu0 (x + θ(y − x)) | ≤ u | x − y | .


θ∈[0,1]

Ce qui permet d’écrire


Z ∞
2
∀ (x, y) ∈ (IR+ ) , | F (x) − F (y) | ≤ | x − y | ue−u du = | x − y |
0
Alors F est continue sur IR+ car elle est lipschitzienne.
Une intégration par parties montre que
Z ∞
e−u
F (x) = Log 2 − x du
0 1 + eux
ou bien si x > 0, Z ∞
e−u/x
F (x) = Log 2 − du.
0 1 + eu
Z ∞ −xu
e
Considérons, pour x > 0, G(x) = du. Clairement F (x) = Log 2 + G(1/x).
0 1 + eu
Montrons que G est dérivable sur IR∗+ ce qui demontrera la même propriété pour
F . Or
Z ∞ Z ∞
u e−ux e−ux −uh
G(x + h) − G(x) + h du = (e − 1 + uh) du.
0 1 + eu 0 1 + eu
y2 | y |
Mais nous avons l’inégalité simple suivante | ey − 1 − y | ≤ e qui est un cas
2
particulier (n = 2) de la formule de Taylor-Lagrange. Alors, si | h | ≤ 1/2,
¯ Z ∞ ¯ Z Z
¯ u e−ux ¯ h2 ∞ eu/2 h2 ∞ −u/2
¯ G(x + h) − G(x) + h ¯
du ¯ ≤ du ≤ e du = h2 .
¯ 1+e u 2 0 1+e u 2 0
0
Z ∞
G(x + h) − G(x) u e−ux
Ce qui démontre que lim = du. G est alors dérivable sur
h→0 h 0 1 + eu
IR∗+ donc F l’est aussi.
58 INTÉGRALES DÉPENDANT D’UN PARAMÈTRE

Solution .2 Montrons par récurrence sur n ≥ 1, la propriété Hn suivante:


Hn : “ F est de classe C n sur IR et que

Z b
(n)
F (x) = tn cos(xt + nπ/2) g(t) dt
a

pour tout x ∈ IR.”


Il est immédiat que la fonction gn : [a, b]×IR −→ IR : (t, x) 7→ tn cos(xt+nπ/2) g(t)
est continue pour tout n ≥ 0. Ce qui démontre à la fois que H1 est vraie et que
Hn =⇒ Hn+1 . D’où le résultat.
En utilisant la densité des fonctions polynomiales dans l’espace des fonctions
continues sur [a, b] pour la norme uniforme, nous savons qu’il existe une suite de
fonctions polynomiales (Pn )n≥1 telle que lim sup | g(t) − Pn (t) | = 0.
n→∞ t∈[a,b]

Soit ε > 0, il existe N ∈ IN tel que

ε
sup | g(t) − PN (t) | ≤ (1)
t∈[a,b] 2(b − a)

Mais, pour x > 0,

Z b ¯t=b Z b
sin(xt) ¯ 1
GN (x) = cos(xt) PN (t) dt = PN (t)¯¯ − sin(xt) PN0 (t) dt
a x t=a x a

donc
à Z !
b
1
| GN (x) | ≤ | PN (a) | + | PN (b) | + | PN0 (t) | dt .
x a

Par conséquent, il existe x0 tel que

ε
x ≥ x0 =⇒ | GN (x) | ≤ . (2)
2

En combinant (1) et (2), nous obtenons

Z b
x ≥ x0 =⇒ | F (x) | ≤ | cos(xt) (g(t) − PN (t)) | dt + | GN (x) | ≤ ε.
a

Alors, lim F (x) = 0.


x→∞
Solutions 59

1
Solution .3 1◦ . L’ensemble des réels x pour lesquels la fonction t 7→ est
1 + x cos t
1
bornée est ] − 1, 1[. Inversement, pour tout x ∈] − 1, 1[ la fonction t 7→ est
1 + x cos t
continue sur [0, π] donc elle est intégrable sur [0, π]. Ãr !
1+x
En effectuant le changement de variable t = 2Arctg u nous obtenons,
1−x
pour x ∈] − 1, 1[,
Z π Z ∞
dt 2 du π
F (x) = =√ 2
=√ . (1)
0 1 + x cos t 1 − x2 0 1 + u 1 − x2
2◦ . Considérons les trois applications

K :[0, π]×] − 1, 1[−→ [−1, 1]×] − 1, 1[, K(t, x) = (cos t, x)


L :[−1, 1]×] − 1, 1[−→] − 1, 1[, L(t, x) = tx
M :] − 1, 1[−→ IR, M (u) = 1/(1 + u)

Il est immédiat que K, L et M sont continues alors f = M ◦L◦K est continue sur
∂f ∂f
[0, π]×] − 1, 1[. D’autre part (t, x) = − cos t f 2 (t, x) donc est aussi continue sur
∂x ∂x
[0, π]×]−1, 1[. Le théorème de dérivation sous le signe intégrale permet alors de conclure
que Z π
0 cos t
∀ x ∈] − 1, 1[, F (x) = − dt.
0 (1 + x cos t)2
D’où, Z π
cos t −πx
∀ x ∈] − 1, 1[, 2
dt = . (2)
0 (1 + x cos t) (1 − x2 )3/2
∂2f ∂f ∂2f
De même, (t, x) = 2 cos t (t, x)f (t, x) donc est aussi continue sur
∂x2 ∂x ∂x2
l’ensemble [0, π]×] − 1, 1[ et ainsi le même théorème démontre que
Z π
00 2 cos2 t
∀ x ∈] − 1, 1[, F (x) = − 3
dt.
0 (1 + x cos t)

D’où, Z π
cos2 t π(x2 + 1/2)
∀ x ∈] − 1, 1[, dt = . (3)
0 (1 + x cos t)3 (1 − x2 )5/2
Enfin, de (1), (2) et (3) nous obtenons
Z π
1 π(x2 + 2)
∀ x ∈] − 1, 1[, 3
dt = ,
0 (1 + x cos t) 2(1 − x2 )5/2
Z π
cos t 3πx
3
dt = − ,
0 (1 + x cos t) 2(1 − x2 )5/2
Z π
cos2 t π(2x2 + 1)
3
dt = .
0 (1 + x cos t) 2(1 − x2 )5/2
60 INTÉGRALES DÉPENDANT D’UN PARAMÈTRE

1 b
3◦ . Remarquons que ϕ(t) = f (t, ). Alors ϕ est intégrable sur [0, π] si, et
a a
seulement si, | b | < a. En utilisant les résultats précédents nous obtenons, pour tout
(a, b) ∈]0, +∞[×IR, avec | b | < a,
Z π
1 π(b2 + 2a2 )
dt = ,
(a + b cos t)3 2(a2 − b2 )5/2
Z0 π
cos t 3πab
3
dt = − ,
0 (a + b cos t) 2(a2 − b2 )5/2
Z π
cos2 t π(2b2 + a2 )
dt = .
0 (a + b cos t)3 2(a2 − b2 )5/2

Solution .4 Notons que si | x | > 1, alors l’intégrale généralisée


Z 1
dt
p
0 (1 − t2 )(x2 − t2 )

Kx
converge car la fonction intégrée est équivalente, au voisinage de 1, à √ avec Kx
1−t
une constante dépendant de x.
Effectuons alors le changement de variable t = cos θ. Nous obtenons,
Z π/2

∀ x ∈ IR \ [−1, 1], F (x) = √ .
0 x2 − cos2 θ

La fonction L : [0, π/2] × (IR \ [−1, 1]) −→ IR∗+ : (x, θ) 7→ x2 − cos2 θ, est évidemment
continue donc sa composée avec la fonction racine carrée est aussi continue. Il en résulte
que F est continue sur IR \ [−1, 1]. D’autre part F est paire et strictement décroissante
sur ]1, +∞[.
Pour achever l’étude des variations de F nous allons chercher des équivalents de F
au voisinage 1+ et de +∞.
– Au voisinage de 1+ .
Z π/2
cos θ
Posons, pour x > 1, J(x) = √ dθ. Le changement de variable
√ 0 x2 − cos2 θ
θ = Arcsin (u x2 − 1) montre que
Z √ µ ¶
1/ x2 −1
du 1 x+1
J(x) = √ = Log .
0 1+u 2 2 x−1

Il est, d’autre part, facile de voir que pour x > 1,


Z π/2 r
x − cos θ
xF (x) − J(x) = dθ.
0 x + cos θ
Solutions 61
r
x − cos θ
Mais la fonction g : [0, π/2] × [1, +∞[−→ IR : (θ, x) 7→ , est continue, il en
x + cos θ
résulte que x 7→ xF (x) − J(x) est prolongeable par continuité en 1+ . Par conséquent,
Z π/2 r Z π/2
¡ ¢ 1 − cos θ θ
lim xF (x) − J(x) = dθ = tg dθ = Log 2.
x→+∞ 0 1 + cos θ 0 2
1 3
On conclut que F (x) = − Log (x − 1) + Log 2 + ε(x − 1), avec lim ε(x − 1) = 0.
2 2 >
x→1
– Au voisinage de +∞.

On a, pour x ≥ 5,
Z π/2 µ ¶
π x
0 ≤ xF (x) − = √ − 1 dθ
2 0 x2 − cos2 θ
Z π/2
cos2 θ π
= √ √ dθ ≤ .
0 x2 − cos2 θ(x + x2 − cos2 θ) 8x
π 1
Nous concluons qu’au voisinage de l’infini F (x) = + O( 2 ).
2x x

Solution .5 1◦ . Soient x > 0, et n ∈ IN∗ . La décroissance de la fonction f montre


que n ≤ t ≤ n + 1 =⇒ f ((n + 1)x) ≤ f (tx) ≤ f (nx), d’où,
Z n+1
un+1 (x) ≤ f (tx) dt ≤ un (x) (1)
n
Z ∞ Z ∞
La convergence de f (u) du entraı̂ne la convegence de f (tx) dt et puis celle de
Z 0 0
X n+1 P
la série f (tx) dt. Il en résulte d’après (1) que un (x) converge et que
n
Z ∞ ∞
X Z ∞
f (tx) dt ≤ un (x) ≤ f (tx) dt. (2)
1 n=1 0

2◦ . D’après l’inégalité (1), nous avons, pour x > 0,


X∞ Z ∞ Z
1 ∞
uk (x) ≤ f (tx)dt = f (u) du.
n x nx
k=n+1

Alors, si a > 0, ¯ ∞ ¯
¯ X ¯ 1Z ∞
¯ ¯
sup ¯ uk (x) ¯ ≤ f (u) du.
x≥a ¯ ¯ a 0
k=n+1
P
Il en résulte que la série un converge uniformément sur tout intervalle [a, +∞[ avec
X∞
a > 0. Mais pour tout n la fonction un est continue, on conclut que S = est continue
n=1
sur IR∗+ .
62 INTÉGRALES DÉPENDANT D’UN PARAMÈTRE

3◦ . La relation (2) montre que


Z ∞ Z 1
∀ x > 0, f (u) du ≤ xS(x) ≤ f (u) du.
x 0

Z ∞
D’où, en faisant tendre x vers 0 nous obtenons lim xS(x) = f (t) dt. Il est immédiat
>
x→0 0

X
que H(x) = vn (x) = xS(x). Si H est continue sur IR+ , alors
n=1

Z ∞
0 = H(0) = lim xS(x) = f (t) dt
>
x→0 0

Z ∞
Mais f est une fonction continue positive sur IR+ , donc f (t) dt = 0 entraı̂ne que
0
f = 0. On conclut que H est continue si, et seulement si, f = 0.

Solution .6 Montrons plus généralement le lemme suivant:


Lemme: Soit f : [0, +∞[−→ IR telle qu’il existe M > 0 et m Z∈ IN, vérifiant

| f (t) | ≤ M tm pour tout t ≥ 0. Alors la fonction F (x) = f (t) e−tx dt
Z ∞ 0
∗ 0 −tx
est dérivable sur IR+ avec F (x) = − tf (t) e dt.
0 Z ∞
Pour tout x > 0 nous avons | f (t) e−tx m −tx
| ≤ M t e . Mais tm e−tx dt converge
0
m!
(et vaut ). On conclut que l’intégrale définissant F (x) est convergente pour tout
xm+1
x > 0. Z ∞
Il en résulte que G(x) = tf (t) e−tx dt est aussi convergente pour tout x > 0.
0
Soient x > 0 et h ∈] − x/2, x/2[. Il est immédiat que
¯Z ∞ ¯
¯ ¡ ¢ ¯
| F (x + h) − F (x) + hG(x) | = ¯¯ f (t)e−xt e−ht − 1 + ht dt ¯¯
0
Z ∞
¯ ¯
≤ | f (t) | e−tx ¯ e−ht − 1 + ht ¯ dt
0
Z
M h2 ∞ m+2 −t(x−| h |)
≤ t e dt
2 0
Z
M h2 ∞ m+2 −tx/2
≤ t e dt
2 0

F (x + h) − F (x)
On conclut que lim = −G(x) pour tout x > 0, ce qui démontre le
h→0 h
y2 | y |
lemme. (Nous avons utilisé l’inégalité classique | ey − 1 − y | ≤ e ).
2
Solutions 63
Z ∞
1
Revenons à notre exercice. Nous posons f0 (t) = . Comme f0 (t) dt con-
Z ∞ 1 + tp 0

verge, alors l’intégrale g(x) = f0 (t) e−tx dt converge pour tout x ∈ IR+ .
0 Z ∞
(n)
n ∗
La propriété: “ g est de classe C sur IR+ et g (x) = (−1)n tn f0 (t) e−tx dt”,
0
se démontre alors immédiatement par récurrence en utilisant le lemme.
Z ∞
2
Solution .7 Remarquons que la convergence de l’intégrale tm e−t dt pour tout
0
entier m, entraı̂ne celle des intégrales :
Z ∞ Z ∞
−t2 2
f (x) = e cos(2xt) dt et g(x) = te−t sin(2xt) dt
0 0

Pour tout (x, h) ∈ IR2 , nous avons


Z ∞
2
f (x + h) − f (x) + 2hg(x) = (cos(2xt + 2ht) − cos(2xt) + 2ht sin(2xt)) e−t dt.
0

La formule de Taylor-Lagrange montre que, pour tout (u, v) ∈ IR2 ,

v2
| cos(u + v) − cos u + v sin u | ≤
2

Alors, pour tout (x, h) ∈ IR2 , nous avons


Z ∞ √
2 2 −t2 π 2
| f (x + h) − f (x) + 2hg(x) | ≤ 2h t e dt = h
0 2

Ce qui démontre que f est dérivable (donc en particulier continue) sur IR et que pour
tout x ∈ IR, f 0 (x) = −2g(x).
Une intégration par parties montre que, pour x ∈ IR,
Z ∞ Z ∞
−t2 0 2
−2g(x) = (e ) sin(2xt) dt = −2x e−t cos(2xt) dt = −2xf (x).
0 0

Il en résulte que ∀ x ∈ IR, f 0 (x) + 2xf (x) = 0, et que f est de classe C 1 sur√IR.
³ 2 ´0 2 π
Par conséquent ∀ x ∈ IR, ex f (x) = 0, ou bien, ∀ x ∈ IR, ex f (x) = f (0) = .
2
Finalement,
Z ∞ √
−t2 π −x2
∀ x ∈ IR, f (x) = e cos(2xt) dt = e .
0 2
64 INTÉGRALES DÉPENDANT D’UN PARAMÈTRE
µ ¶ Z n
2 x2
Solution .8 Posons, pour n ≥ 1 et x ≥ 0, Fn (x) = exp −t − 2 dt. La
1/n t
µ 2

x
fonction fn définie par (t, x) 7→ fn (t, x) = exp −t2 − 2 est continue sur [1/n, n] ×
t
IR+ alors, pour tout n ≥ 1, la fonction Fn est continue sur IR+ . D’autre part, Pour tout
x ≥ 0,

Z 1/n µ ¶ Z ∞ µ ¶
x2 2 2 x2
0 ≤ F (x) − Fn (x) = exp −t − 2 dt + exp −t − 2 dt
0 t n t
Z ∞
1 2
≤ + e−t dt.
n n

Il en résulte que la suite de fonctions (Fn )n≥1 converge uniformément sur IR+ vers F .
Donc F est continue sur IR+ . D’autre part, il est facile de voir que

Z ∞ √
−t2 π
∀ x ∈ IR+ , 0 ≤ F (x) ≤ e dt = .
0 2

∂fn 2x ∂fn
De l’autre côté, il est immédiat que (t, x) = − 2 fn (t, x) et par conséquent
∂x t ∂x
est continue sur [1/n, n] × IR+ , alors Fn est de classe C 1 sur IR+ et pour x ∈ IR∗+ ,

Z n µ ¶ Z nx µ ¶
−2x x2 x2
Fn0 (x) = 2
exp −t − 2 dt = −2 2
exp −u − 2 du.
1/n t2 t x/n u

(où pour la deuxième égalité nous avons effectué le changement de variable t = x/u).
Soient (a, b) ∈ (IR∗+ )2 tel que a < b. Il est facile de voir que, pour tout x ∈ [a, b],

Z x/n µ ¶ Z ∞ µ ¶
x2 x2
0 ≤ 2F (x) + Fn0 (x) = 2
2 exp −t − 2 dt + 2
2 exp −t − 2 dt
0 t nx t
Z ∞
2b 2
≤ +2 e−t dt.
n na

Donc (Fn0 )n≥1 converge uniformément sur tout intervalle compact de IR∗+ vers la fonction
−2F . On conclut que F est de classe C 1 sur IR∗+ et que F 0 = −2F sur cet ensemble.
On en déduit que F (x) = F (0)e−2x pour tout x ∈ IR+ . D’où

Z ∞ µ ¶ √
x2 2 π −2| x |
∀ x ∈ IR, exp −t − 2 dt = e .
0 t 2
Solutions 65
Z ∞
Solution .9 ◦
1 . En effet, (b + atm ) e−tx dt converge pour tout x ∈ IR∗+ , alors
0
l’intégrale définissant g(x) est absolument convergente, et g est définie sur IR∗+ .
2◦ . Démontrons par récurrence sur n ≥ 1 la propriété IPn suivante: “ Pour toute
fonction continue f : IR+ −→ IR pour laquelle il existe (a, b) ∈ Z(IR∗+ )2 et m ∈ IN tels

que ∀ t ≥ 0, | f (t) | ≤ atm + b, la fonction g définie par g(x) = f (t) e−tx dt est de
Z ∞ 0
n ∗ (n) n n −tx
classe C sur IR+ et que g (x) = (−1) t f (t) e dt ”.
0
Considérons le cas n = 1. Soit f une fonction comme dans l’énoncé. Alors t 7→ tf (t)
vérifie les mêmes hypothèses
Z ∞ que f (mais peut-être avec a, b et m différents). Donc
l’intégrale R(x) = tf (t) e−tx dt converge pour tout x ∈ IR∗+ .
0
Soit x > 0 et h ∈] − x/2, x/2[. Alors
Z ∞ ¯ −th ¯
| g(x + h) − g(x) + hR(x) | ≤ ¯e − 1 + ht ¯ | f (t) | e−tx dt
0
2Z ∞
h
≤ (atm+2 + bt2 ) e−t(x−| h |) dt
2 0
Z
h2 ∞ m+2
≤ (at + bt2 ) e−tx/2) dt
2 0

g(x + h) − g(x)
On conclut que lim = −R(x). g est alors dérivable sur IR∗+ . Ceci
h→0 h
démontre en particulier que g est continue sur IR∗+ . En appliquant ce qui précède à
t 7→ tf (t) au lieu de f nous voyons immédiatement que R est continue sur IR∗+ . Ce qui
démontre IP1 .
Supposons¯ IPn ¯vraie. La fonction fe définie par fe(t) = (−t)n f (t) est continue sur
¯ ¯
IR+ et vérifie ¯ fe(t) ¯ ≤ (a + b)(tn+m + 1), donc d’après IP1 appliquée à la fonction fe
nous obtenons que g (n) est de classe C 1 sur IR∗+ et vérifie
Z ∞ Z ∞
g (n+1)
(x) = − e
tf (t) e −tx
dt = (−1)n+1
tn+1 f (t) e−tx dt.
0 0

Ce qui démontre IPn+1 . D’où le Zrésultat.



k!
3◦ . Il est facile de voir que tk e−tx dt = pour tout k ∈ IN. Alors
0 xk+1
Z ∞
m! a b
| g(x) | ≤ (atm + b) e−tx dt = m+1
+
0 x x

et par conséquent lim g(x) = 0. De même nous démontrons aussi que lim g 0 (x) = 0.
x→∞ x→∞
4◦ . La continuité de f est immédiate. La formule de Taylor-Lagrange montre que

t2
∀ t ∈ IR, | cos t − 1 | ≤ .
2
66 INTÉGRALES DÉPENDANT D’UN PARAMÈTRE

Alors, pour tout t ≥ 0 nous avons 0 ≤ f (t) ≤ 1/2. Z ∞


1 − cos t −tx
Il en résulte que la fonction g définie sur IR+ par g(x) = e dt vérifie
0 t2
les propriétés démontrées dans les questions 2◦ et 3◦ . Un calcul simple montre que, pour
tout x > 0, Z ∞
00 1 x
g (x) = (1 − cos t) e−tx dt = − .
0 x 1 + x2
En utilisant, 2◦ , nous obtenons, pour tout x > 0,

1 1
g 0 (x) = − Log (1 + 2 )
2 x

puis que, pour tout x > 0,

π x 1
g(x) = − Arctg x − Log (1 + 2 ).
2 2 x

1 − cos t 1 2
Comme 0 ≤ 2
≤ min( , 2 ) pour tout t, alors l’intégrale définissant g(0) est
t 2 t
convergente et si A ∈]0, ∞[ et x > 0, nous avons
Z ∞
1 − cos t
0 ≤ g(0) − g(x) = 2
(1 − e−tx ) dt
0 t
Z Z ∞
1 A 2 A2 2
≤ tx dt + 2
dt ≤ x +
2 0 A t 4 A
µ ¶1/3
4
En prenant A = nous obtenons
x
r
x
∀ x > 0, 0 < g(0) − g(x) ≤ 3 3 .
4

π
Il en résulte que lim g(x) = g(0). Ce qui entraı̂ne g(0) = .
>
x→0 2
Une intégration par parties montre que
Z x ¯x Z x
sin t 1 − cos t ¯¯ 1 − cos t
dt = ¯ + dt.
0 t t 0 0 t2

D’où, en faisant tendre x vers l’infini


Z ∞ Z ∞
sin t 1 − cos t π
dt = 2
dt = g(0) = .
0 t 0 t 2
Solutions 67

Log (1 + u)
Solution .10 1◦ . Posons h(u) = pour u > 0 et h(0) = 1. La fonction h
u
est continue sur IR+ .
D’autre part, la fonction (t, x) 7→ cos t cos x est continue sur [0, π/2] × [0, π/2] à
valeurs dans IR+ . Il en résulte que la fonction (t, x) 7→ cos x h(cos t cos x) = f (t, x) est
continue sur [0, π/2] × [0, π/2]. Ce qui permet de déduire la continuité de F sur [0, π/2].
∂f − sin x
Un calcul simple montre que (t, x) = qui est évidemment con-
∂x 1 + cos x cos t
tinue sur [0, π/2] × [0, π/2]. Il en résulte que F est de classe C 1 sur [0, π/2] avec
Z π/2
0 sin x
F (x) = − dt.
0 1 + cos x cos t
2◦ . Le changement de variable t = 2Arctg u permet d’écrire
Z 1 Z 1
0 −2 sin x − sin x
F (x) = 2 2
du = 2 du.
0 1 + u + (1 − u ) cos x
2 2
0 cos (x/2) + u sin (x/2)

cos(x/2)
On effectue ensuite le changement de variable u = tg ϕ ce qui entraı̂ne, pour
sin(x/2)
x ∈ [0, π/2],
Z x/2
0
F (x) = − 2 dϕ = −x.
0
3◦ . Pour x ∈ [0, π/2], nous avons
π π2 x2
F (x) = F (x) − F ( ) = − .
2 8 2
4◦ . En prenant x = 0, et en effectuant le changement de variable u = cos t dans
F (0) nous obtenons,
Z 1
Log (1 + u) π2
√ du = .
0 u 1 − u2 8

1
Solution .11 1◦ . Posons g(t, x) = x pour t > 0 et x ∈ IR. Il est immédiat
t (t + 1)
Z 1 Z ∞
dt dt
que x
converge si x < 1, et que x
converge si x > 0. Alors
0 t (t
Z + 1) 1 t (t + 1)

l’intégrale g(t, x) dt converge si, et seulement si, x ∈]0, 1[. L’ensemble de définition
0
de f est alors ]0, 1[. Z ∞
◦ dt 1
2 . Remarquons que = . Alors
1 t1+x x
Z 1 Z ∞ µ ¶
1 dt dt 1 1
f (x) − = − −
x 0 tx (1 + t) 1 tx t 1+t
Z 1 Z ∞
dt dt
= −
0 tx (1 + t) 1 tx+1 (1 + t)
Z 1 Z 1 x
dt t dt
= x
− .
0 t (1 + t) 0 1+t
68 INTÉGRALES DÉPENDANT D’UN PARAMÈTRE

On conclut que pour tout x ∈]0, 1[,

Z 1 Z 1
1 dt dt 1
0 ≤ f (x) − ≤ x
≤ x
= .
x 0 t (1 + t) 0 t 1−x

1
Alors lim xf (x) = 1 et f (x) ∼ .
> 0 x
x→0

3 . Le changement de variable t = 1/u nous montre que f (x) = f (1 − x) pour
tout x ∈]0, 1[. Alors le graphe de f est symétrique par rapport à la droite d’équation
x = 1/2.
Z n
dt

4 . Posons fn (x) = x
, pour n ∈ IN∗ et x ∈]0, 1[.
1/n t (1 + t)
dt
Il est immédiat que (t, x) 7→ x est continue sur [1/n, n]×]0, 1[. Alors, pour
t (1 + t)
tout n ∈ IN∗ , la fonction fn est continue sur ]0, 1[.
D’autre part, soit ε ∈]0, 1/2[. Pour tout x ∈ [ε, 1 − ε], nous avons

Z Z ∞
1/n
dt dt
0 ≤ f (x) − fn (x) = x
+ x
0 t (1 + t) n t (1 + t)
Z 1/n Z ∞
dt dt
≤ x
+ x+1
0 t n t
1 1 1 1 2
≤ + ≤ .
1 − x n1−x x nx εnε

Il en résulte que la suite fn converge uniformément vers f sur [ε, 1 − ε], la fonction f
est alors continue sur [ε, 1 − ε] pour tout ε ∈]0, 1/2[. On conclut que f est continue sur
]0, 1[.
5◦ . Remarquons que

Z 1 µ ¶ Z ∞
1 1 dt du
f (x) = x
+ 1−x = (exu + e(1−x)u )
0 t t 1+t 0 1 + eu

Une étude simple montre que la fonction x 7→ exu + e(1−x)u est décroissante sur
]0, 1/2[ pour tout u > 0. Alors f est décroissante sur ]0, 1/2], et en utilisant 3◦ la
fonction f est croissante sur [1/2, 1[. On conclut que

1
inf f (x) = f ( ) = π.
x∈]0,1[ 2
Solutions 69

Solution .12 Pour tout x ∈ IR, on définit la fonction

Log (t2 + x2 )
ϕx :]0, +∞[−→ IR, t 7→ ϕx (t) =
t2 + 1

Elle est continue, donc localement intégrable. Si x 6= 0, alors lim ϕx (t) = Log (x2 ),
>
t→0
par contre si x = 0 alors au voisinage de 0 nous avons ϕx (t) ∼ 2Log t. D’autre part,
0+
2Log t
au voisinage de +∞, ϕx (t) ∼ . Cette étude aux bornes donne l’existence de
+∞ t2
l’intégrale, Z ∞
Log (t2 + x2 )
f (x) = dt.
0 t2 + 1
f est évidemment paire.
Introduisons la suite de fonctions
Z n
Log (t2 + x2 )
fn :]0, +∞[−→ IR, x 7→ fn (x) = dt,
0 t2 + 1

et
Log (t2 + x2 )
hn : [0, n]×]0, +∞[−→ IR, (t, x) 7→ hn (t, x) = .
t2 + 1
∂hn 2x ∂hn
Il est immédiat que (t, x) = 2 , donc h n et sont continues sur
∂x (t + 1)(t2 + x2 ) ∂x
[0, n]×]0, +∞[, ce qui prouve que fn est de classe C 1 sur ]0, +∞[ avec
Z n
2x
fn0 (x) = dt.
0 (t2 + 1)(t2 + x2 )
Z ∞
2x
Introduisons l’intégrale convergente g(x) = dt. Il est immédiat que
0 (t2 + 1)(t2 + x2 )
Z ∞
2x
g(x) − fn0 (x) = dt
n (t2 + 1)(t2 + x2 )

d’où, en utilisant 2xt ≤ x2 + t2 ,


Z ∞ Z ∞
dt dt 1
0 ≤ g(x) − fn0 (x) ≤ 2
dt ≤ 3
= 2.
n t(t + 1) n t 2n

Ceci démontre que la suite (fn0 )n≥1 converge uniformément sur ]0, +∞[ vers g et que f
est de classe C 1 sur IR∗+ avec f 0 = g. µ ¶
2x 2x 1 1
Pour x 6= 1, nous avons 2 = 2 − , d’où
(t + 1)(t2 + x2 ) x − 1 t2 + 1 t2 + x2
Z µ ¶
0 2x ∞
1 1 2x ³ π π´ π
f (x) = 2 − 2 dt = − = .
x −1 0 t + 1 t + x2
2 2
x −1 2 2x x+1
70 INTÉGRALES DÉPENDANT D’UN PARAMÈTRE

La continuité de f 0 en 1 nous permet d’écrire, pour tout x > 0,


π
f 0 (x) = , et f (x) = πLog (1 + x) + c
1+x
avec c une constante. Z ∞
Log t2
Nous avons vu l’existence de f (0) = dt. D’où, pour x > 0,
0 1 + t2
Z ∞
Log (x2 + t2 ) − Log t2
0 ≤ f (x) − f (0) = dt
0 1 + t2
Z 1 Z ∞
Log (x2 + t2 ) − Log t2 Log (1 + x2 /t2 )
= dt + dt
0 1 + t2 1 1 + t2
Z 1 Z ∞
2 2 2 2 dt
≤ (Log (x + t ) − Log t ) dt + x
0 1 t (1 + t2 )
2
Z 1
x2
≤ −2+ Log (x2 + t2 ) dt
3 0
Une intégration par parties démontre que
Z 1
Log (x2 + t2 ) dt = Log (1 + x2 ) − 2 + 2xArctg (1/x)
0
donc
x2 1 4
∀ x > 0, 0 ≤ f (x) − f (0) ≤+ Log (1 + x2 ) + 2xArctg ≤ πx + x2 .
3 x 3
Il en résulte que lim f (x) = f (0) et f est continue sur IR, (à cause de la parité de f ).
>
x→0
Enfin, le changement de variable t = 1/u dans f (0) nous montre que
Z ∞ Z ∞
Log t Log u
f (0) = 2 2
dt = − du = −f (0).
0 1+t 0 1 + u2
et par conséquent lim f (x) = f (0) = 0. Ceci démontre que c = 0 et que f (x) =
>
x→0
πLog (1 + x) pour tout x ≥ 0, puis f (x) = πLog (1 + | x |) pour tout x ∈ IR.
Remarque: Le changement de variable t = tg u permet décrire
Z π/2
∀ x ∈ IR, Log (x2 + tg 2 u) du = πLog (1 + | x |).
0
Z π/2
En particulier, si x = 1 nous obtenons Log (cos2 u) du = −πLog 2. Ce qui
0
démontre, en prenant la somme
Z π/2
1 + |x|
∀ x ∈ IR, Log (x2 cos2 u + sin2 u) du = πLog ( ).
0 2

Z π/2
|a| + |b|
2
∀ (a, b) ∈ IR \ {(0, 0)}, Log (a2 cos2 u + b2 sin2 u) du = πLog ( ).
0 2
Solutions 71

Solution .13 1◦ . Notons qu’une intégration par parties immédiate montre que pour
a 6= 0, ¯t
Z t Z
cos ax sin ax ¯¯ 1 t sin ax
dx = + dx
0 1+x a(1 + x) ¯0 a 0 (1 + x)2
Z
sin at 1 t sin ax
= + dx.
a(1 + t) a 0 (1 + x)2
Z ∞
sin at
Mais l’intǵrale dt est absolument convergente. Il en résulte que l’intégrale
Z ∞ 0 (1 + t)2
cos ax
dx converge et que, pour tout a ∈ IR∗ ,
0 1 + x
Z ∞ Z ∞
cos ax 1 sin ax
g(a) = dx = dx. (10 )
0 1+x a 0 (1 + x)2

Le fait que la fonction g est paire est immédiat à partir de celle de la fonction x 7→ cos x.
cos ax
2◦ . La fonction (x, a) 7→ est évidemment continue sur [0, n] × IR. Alors la
1+x
fonction gn est continue sur IR. D’autre part, une intégration par parties démontre que
Z ∞ Z ∞
cos ax sin an 1 sin ax
g(a) − gn (a) = dx = − + dx
n 1+x a(1 + n) a n (1 + x)2

donc Z ∞
1 1 1 2
| g(a) − gn (a) | ≤ + dx =
a(1 + n) a n (1 + x)2 a(1 + n)
ce qui entraı̂ne
2
sup | g(a) − gn (a) | ≤ .
a≥a0 a0 (1 + n)
Il en résulte que (gn )n≥1 converge uniformément vers g sur tout intervalle de la forme
[a0 , +∞[, avec a0 > 0. Par conséquent g est continue sur ]0, +∞[ et donc sur IR∗ à cause
de sa parité.
3◦ . Comme a > 0, alors la relation (2) résulte de (1) après le changement de variable
ax = u. Le changement de variable v = a + u permet alors d’écrire,
Z ∞ Z ∞ µ ¶
cos(v − a) cos v sin v
g(a) = dv = cos a + sin a dv.
a v a v v

Mais Z ¯Y Z Y iv
Y
eiv eiv ¯¯ e
dv = ¯ + 2
dv
X v iv X X iv

donc, ¯Z ¯
¯ Y eiv ¯ Z Y
¯ ¯ 1 1 dv 2
0 < X < Y =⇒ ¯ dv ¯ ≤ + + = .
¯ X v ¯ X Y X v
2 X
72 INTÉGRALES DÉPENDANT D’UN PARAMÈTRE
Z ∞
eiv
Il en résulte, d’après le critère de Cauchy, que l’intégrale dv est convergente, il
a v
va de même de ses parties réelles et imaginaires. Nous pouvons alors écrire

Z ∞ Z ∞
cos v sin v
g(a) = cos a dv + sin a dv. (3)
a v a v

Remarquons que, d’après (3)

Z ∞ Z a Z ∞
cos v cos v sin v
g(a) = cos a dv − cos a dv + sin a dv
v v v
Z1 ∞ 1
Z a a
Z a Z ∞
cos v 1 − cos v dv sin v
= cos a dv + cos a dv − cos a + sin a dv
1 v 1 v 1 v a v

alors,

Z ∞ Z a Z ∞
cos v 1 − cos v sin v
g(a) + cos a Log a = cos a dv + cos a dv + sin a dv
1 v 1 v a v

1 − cos v sin v
Mais les fonctions v 7→ et v 7→ sont prolongeables par continuité en zéro,
Z 1 v Z ∞v
1 − cos v sin v
donc les intégrales dv et dv sont convergentes. Il en résulte que
0 v 0 v

Z ∞ Z 1
cos v 1 − cos v
lim g(a) + cos a Log a = dv − dv = `.
a→0
>
1 v 0 v

Comme lim (1 − cos a)Log a = 0, alors lim g(a) + Log a = `. Ce qui permet de
> >
a→0 a→0
démontrer que g(a) ∼+ −Log a.
0
5◦ . En utilisant (10 ), nous avons

Z ∞
dx
∀ a > 0, a | g(a) | ≤ = 1.
0 (1 + x)2

Alors ∀ a > 0, a | g(a) | ≤ 1.


Enfin, au voisinage de 0+ , g 2 (a) est équivalente à Log 2 a et au voisinage
Z ∞ de +∞,
1
g 2 (a) est majorée par 2 . Ce qui nous permet d’affirmer que l’intégrale g 2 (a) da
a 0
Z 1 Z ∞
2 da
est convergente car Log a da et le sont.
0 1 a2
Solutions 73

Solution .14 Remarquons d’abord que l’intégrale définissant I(x) est absolument
convergente pour tout x ∈ IR.
Pour n ∈ IN∗ , nous posons
Z n
e−at − ebt
g(t, x) = cos tx, In (x) = g(t, x) dt.
t 0

∂g
Il est immédiat que g et sont continues sur [0, n] × IR. Alors x 7→ In (x) est de classe
∂x
C 1 sur IR. Z ∞
D’autre part, si c > 0, l’intégrale hc (x) = e−ct sin(xt) dt est absolument
0
convergente et un calcul simple démontre que
µZ ∞ ¶
(−c+ix)t x
hc (x) = Im e dt = .
0 x2 + c2

Il en résulte que,
Z ∞
hb (x) − ha (x) − In0 (x) = (e−bt − e−at ) sin(xt) dt
n

d’où, pour tout x ∈ IR,


Z ∞ Z ∞
e−nb
| hb (x) − ha (x) − In0 (x) | ≤ (e −bt at
− e ) dt ≤ e−bt dt = .
n n b

On conclut que la suite (In0 )n≥1 converge uniformément sur IR vers la fonction définie
par x 7→ hb (x) − ha (x). Évidemment la suite (In )n≥1 converge vers I, alors I est de
classe C 1 sur IR et I 0 = hb − ha . Ce qui permet de démontrer qu’il existe une constante
k telle que
1 x2 + b2
∀ x ∈ IR, I(x) = k + Log 2 .
2 x + a2
De l’autre côté,
Z R Z aR Z bR
e−at − e−bt e−t e−t
dt = dt − dt
r t ar t br t
Z ar −t Z aR
e e−t
=− dt + dt
br t bR t
Z ar Z ar Z aR
dt 1 − e−t e−t
=− + dt + dt.
br t br t bR t

Donc
¯Z ¯ Z ∞
¯ R e−at − e−bt b ¯ 1 e−bR
¯ ¯
¯ dt − Log ¯ ≤ r(b − a) + e−t dt = r(b − a) +
¯ r t a¯ bR bR bR
74 INTÉGRALES DÉPENDANT D’UN PARAMÈTRE

1 − e−t
où nous avons utilisé l’inégalité ≤ 1 qui est valable pour t > 0.
t
b
En faisant tendre r vers 0 et R vers +∞ nous obtenons I(0) = Log , et alors
a
k = 0, ce qui démontre

1 x2 + b2
∀ x ∈ IR, I(x) = Log 2 .
2 x + a2

Solution .15 1◦ . Considérons

g : [0, π/2]×] − 1, +∞[−→ IR : (t, x) 7→ Log (1 + x sin2 t) dt.

∂g
Il est immédiat de voir que g et sont continues sur [0, π/2]×] − 1, +∞[. Alors
∂x
G est de classe C 1 sur ] − 1, +∞[.
L’intégrale définissant G(−1) est convergente. (Résultat connu et simple laissé au
lecteur). Montrons que G est continue en −1. Soit −1 < x < 0,
Z π/2 µ ¶
1 + x sin2 t
G(x) − G(−1) = Log dt
0 cos2 t
Z π/2
= Log (1 + (x + 1)tg 2 t) dt
0

alors, pour x ∈] − 1, 0[, et a ∈]0, π/2[,


Z a Z π/2
2
0 ≤ G(x) − G(−1) ≤ Log (1 + (x + 1)tg t) dt + Log (1 + tg 2 t) dt
0 a
Z π/2
≤(x + 1)atg 2 a − Log (cos2 t) dt
a

Soit ε > 0, la convergence de l’intégrale G(−1) montre qu’il existe a ∈]0, π/2[ tel que
Z π/2
ε
0≤− Log (cos2 t) dt ≤ , on peut ensuite choisir η > 0 pour que 0 < 1 + x < η
a 2
2 ε
entraı̂ne (x + 1)atg a < , et par conséquent 0 ≤ G(x) − G(−1) ≤ ε. Ceci démontre
2
que G est continue au point −1.
2◦ . Nous avons vu que, pour x ∈] − 1, +∞[\{0},
Z π/2
0 sin2 t
G (x) = dt
0 1 + x sin2 t
Z ∞ Z µ ¶
u2 1 ∞ 1 1
= du = − du
0 (1 + u2 )((1 + x)u2 + 1) x 0 1 + u2 1 + (1 + x)u2
µ ¶
1 π π π
= − √ = √
x 2 2 x+1 2(x + 1 + x + 1)
Solutions 75

La continuité de G0 en 0 montre que pour tout x ≥ −1,

π
G0 (x) = √
2(x + 1 + x + 1)

3◦ . Un calcul simple de primitive et le fait que G(0) = 0 montrent que, pour tout
x > −1, √
1+ x+1
G(x) = πLog .
2
La continuité de G en −1 montre que la formule précédente pour G reste valable pour
x = −1.
2
◦ e−xt 1
Solution .16 1 . Pour x ∈ IR+ nous avons la majoration 2
≤ , donc
1+t 1 + t2
l’intégrale
Z ∞ 2
e−xt
f (x) = dt
0 1 + t2
est convergente, et f est définie sur IR+ .
Soient (x, y) ∈ (IR+ )2 avec x 6= y, et soit a > 0, nous avons
¯ ¯
Z ¯ −xt2 −yt2 ¯ Z
a ¯e −e ¯ ∞
dt
| f (x) − f (y) | ≤ dt +
0 1+ t2 a t2
Z a
t2 1
≤|x − y| 2
dt +
0 1+t a
1
≤a | x − y | +
a
p
En choisissant a = 1/ | x − y | nous obtenons
p
∀ (x, y) ∈ (IR+ )2 , | f (x) − f (y) | ≤ 2 | x − y |.


Alors f est continue sur IR+ . D’autre part le changement de variable t x = u permet
d’écrire Z 2 Z
∞ ∞
1 e−u 1 2
0 ≤ f (x) = √ 2
du ≤ √ e−u du
x 0 1 + u /x x 0

ce qui démontre que lim f (x) = 0.


x→+∞
Z ∞ 2 −xt2
◦ t e
2 . Pour x > 0 l’intégrale g(x) = dt est convergente, et nous avons,
0 1 + t2
pour x > 0 et h ∈ [−x/2, x/2],
Z ∞ 2
e−xt 2
f (x + h) − f (x) + hg(x) = 2
(e−ht − 1 + ht2 ) dt
0 1+t
76 INTÉGRALES DÉPENDANT D’UN PARAMÈTRE

ce qui permet d’écrire


Z
e−xt ¯¯ −ht2 ¯
∞ 2

| f (x + h) − f (x) + hg(x) | ≤ ¯ e − 1 + ht ¯ dt
0 1 + t2

y2 | y |
et la majortion, qui nous est devenue familière, | ey − 1 − y | ≤ e nous permet de
2
déduire Z ∞ 4 −xt2 /2
h2 t e
| f (x + h) − f (x) + hg(x) | ≤ dt.
2 0 1 + t2
En divisant par h et en faisant tendre h vers 0 nous voyons que f est dérivable en x et
que f 0 (x) = −g(x). Enfin, pour x > 0,
Z ∞ 2 −xt2 Z ∞ Z ∞ 2
t e −xt2 e−xt C
g(x) = dt = e dt − dt = √ − f (x).
0 1 + t2 0 0 1+t 2 x
Z ∞
2
avec C = e−u du. D’où le résultat.
0
3◦ . pour tout x > 0 nous avons
Z ∞
−x e−t
f (x) e =C √ dt.
x t

(Il suffit de dériver, et de considérer la limite à l’infini pour le vérifier). Nous utilisons
ensuite la continuité de f en 0 pour déduire que
Z ∞
π e−t
= f (0) = C √ dt = 2C 2 ,
2 0 t

π
d’où C = .
2

OKMRAN
OUBA
SÉRIES ENTIÈRES

I. Généralités

Définition : On appelle série entière de la variable complexe toute série


P
d’applications fn où fn : C
| −→ C
| : z 7→ an z n , avec an ∈ C,
| et on la note par
P
abus an z n .

On appelle le nombre an le coefficient d’ordre n, et a0 le terme constant.

Il est facile de vérifier que l’ensemble des séries entières de la variable complexe
forme un C-espace
| vectoriel pour l’addition des séries et la multiplication par un scalaire.
De plus cet espace vectoriel est stable par le produit des séries.

P
Lemme I.1. (d’Abel) Soient an z n une série entière et z0 ∈ C | tel que la suite
P
(an z0n )n∈IN soit bornée. Alors pour tout z ∈ D(0, | z0 |)† , la série an z n est absolument
convergente.

Preuve : En effet, par hypothèse, il existe M ∈ IR∗+ tel que ∀ n ∈ IN, | an z0n | ≤ M . Si
z0 = 0 il y a rien à démontrer. Par contre si z0 6= 0 alors, pour tout z ∈ D(0, | z0 |) on a

¯ ¯n
¯ z ¯
∀ n ∈ IN, | an z | ≤ M ¯¯
n ¯ .
¯
z0

P
Par conséquent, an z n converge car | z/z0 | < 1.

P P
Corollaire I.2. Soient an z n une série entière et z0 ∈ C | tel que an z0n
P
soit convergente. Alors pour tout z ∈ D(0, | z0 |), la série an z n est absolument
convergente.


D(a, r) est le disque ouvert de centre a et de rayon r.
2 Séries entières
P
Théorème et Définition I.3. À toute série entière an z n , on peut associer un, et
un seul, R ∈ IR tel que:
P
1◦ . Pour tout z ∈ C
| vérifiant | z | < R, la série an z n est absolument convergente.
P
2◦ . Pour tout z ∈ C
| vérifiant | z | > R, la série an z n est divergente.
P
On appelle R le rayon de convergence de la série an z n .

Preuve : Soit A l’ensemble des réels positifs r tels que la suite (an rn )n∈IN soit bornée.
Cet ensemble n’est pas vide, (car contient 0), donc il admet dans IR+ une borne
supérieure R.
– Soit z ∈ C| tel que | z | < R. D’aprés la définition de R, il existe r ∈ A tel
P
| z | < r ≤ R. Comme (an rn )n∈IN est bornée, alors le lemme d’Abel montre que an z n
est absolument convergente.
– Par contre, si z ∈ C| tel que | z | > R alors, d’aprés la définition de R, la suite

n P
(an | z | )n∈IN n’est pas bornée, an z n est divergente.

P
Théorème I.4. Le rayon de convergence R de la série entière an z n vérifie †

1 p
= lim n | an |
R n→∞

Preuve : Notons que


p p
lim n
| an z n | = | z | lim n
| an |.
n→∞ n→∞

La règle de Cauchy (cf. Séries numéiques proposition III.3) permet de conclure.

Remarque : Rappelons que, si an 6= 0 à partir d’un certain rang et si


¯ ¯
¯ an ¯
¯ ¯ −−→ ρ ∈ IR+ ,
¯ an+1 ¯ −
n→∞

P
alors ρ est le rayon de convergence de la série entière an z n , (cf. Séries numéiques
proposition III.5).

† 1 1
Avec la convention = 0 et = ∞.
∞ 0
Généralités 3

Exercice : Déterminer le rayon de convergence des séries entières suivantes:

X∞ ∞
X X∞ ∞
X
zn n n n! n n
, n z , z , z2
n=0
n! n=1 n=1
nn n=0


X ∞ µ
X ¶n2
n n n (−1)n
(3 + (−1) ) z , 1+ zn.
n=0 n=1
n

P
Définition : Soit an z n une série entière de rayon de convergence R. Le disque ouvert
de centre 0 et de rayon R dans le plan complexe s’appelle le disque de convergence
P
de la série entière an z n . Le cercle de centre 0 et de rayon R porte le nom de cercle
de convergence. Il est important de noter qu’une série entière peut diverger en tout
point de son cercle de convergence !.

P P
Théorème I.5. Soient an z n , et bn z n deux séries entières de rayons de convergence
respectifs α et β,
P
1◦ . Le rayon de convergence γ de la série entière somme cn z n , (cn = an + bn )
vérifie γ ≥ min(α, β) avec égalité si α 6= β.
P X
2◦ . Le rayon de convergence δ de la série entière produit dn z n , (dn = ak b` )
k+`=n
vérifie δ ≥ min(α, β).

P P
Preuve : 1◦ . Pour tout z ∈ C | tel que les deux séries an z n et bn z n convergent, la
P
série cn z n converge. Donc γ ≥ min(α, β). Si, par exemple, α > β, alors on a, d’une
part γ ≥ β et d’autre part, β ≥ min(γ, α) = γ (car bn = cn − an ). On conclut que
γ = min(α, β).
Remarquons que si α = β alors on peut avoir γ > α, comme le montre l’exemple
des deux séries entières correspondant à an = 1 + 2n et bn = 1 − 2n , où α = β = 1/2 et
γ = 1.
P P
2◦ . Pour tout z ∈ C | tel que les deux séries an z n et bn z n convergent
P
absolument, la série dn z n converge. Donc γ ≥ min(α, β). Notons aussi que l’on peut
X∞
avoir δ > min(α, β), comme le montre l’exemple des deux séries entières z n et 1 − z,
n=0
où α = 1, β = ∞ et δ = ∞.
4 Séries entières
P
Définition : Soit an z n une série entière. On appelle sa série entière dérivée la
P
série entière (n + 1)an+1 z n .

P
Théorème I.6. Soit an z n une série entière de rayon de convergence R. Alors le
rayon de convergence R0 de sa série entière dérivée est égal à R.

n+1 n
Preuve : Notons d’abord que | an+1 | | z | ≤ | z | (n + 1) | an+1 | | z | . Donc si
0 < | z | < R0 alors | z | ≤ R, ce qui démontre que R0 ≤ R.
D’autre part, soit ε > 0, on a l’inégalité immédiate ε(n + 1) ≤ (1 + ε)n+1 , ce qui
permet d’écrire, pour tout z 6= 0,

n 1
(n + 1) | an+1 | | z | ≤ | an+1 | ((1 + ε) | z |)n+1 .
ε|z|

Donc si 0 < (1 + ε) | z | < R alors | z | ≤ R0 , ce qui démontre que R ≤ (1 + ε) R0 . Mais ε


est arbitraire, d’où le résultat.

II. Propriétés de la somme d’une série entière

P
Proposition II.1. Soit an z n une série entière de rayon de convergence R non nul.
P
Alors la série d’applications an z n converge normalement, (donc uniformément), sur
tout disque fermé D(0, r) avec r ∈]0, R[.

Preuve : En effet, nous avons

sup | an z n | = | an | rn ,
z∈D(0,r)

P
et | an | rn converge car r < R.

P
Théorème II.2. Soit an z n une série entière de rayon de convergence R non nul.
X∞
L’application somme z 7→ an z n est continue sur le disque de convergence D(0, R).
n=0

Preuve : Car c’est une série d’application continue qui converge uniformément sur
tout compact du disque de convergence.
Propriétés de la somme d’une série entière 5

Définition : Soit Ω un ouvert non vide du plan complexe, et f une application de Ω


dans C.
| On dit que f est dérivable au point z0 ∈ Ω si, et seulement si, la limite
f (z) − f (z0 )
lim
z→z 0
z6=z0
z − z0
existe. On note f 0 (z0 ) la valeur de cette limile lorsqu’elle existe.

P
Théorème II.3. Soit an z n une série entière de rayon de convergence R non nul.
X∞
L’application somme z 7→ S(z) = an z n est dérivable en tout point du disque de
n=0
convergence. De plus,

X
∀ z ∈ D(0, R), S 0 (z) = (n + 1)an+1 z n .
n=0

Preuve : Soit ω ∈ D(0, R). Choisissons r ∈] | ω | , R[. Pour tout z ∈ D(0, r), on a
∞ ∞
Ãn−1 !
S(z) − S(ω) X z n − ω n X X
= an = an z k ω n−1−k ,
z−ω n=1
z−ω n=1 k=0

X
donc, si Q(ω) = nan ω n−1 , on a
n=1

Ãn−1 ! ∞
S(z) − S(ω) X X X
k n−1−k n−1
− Q(ω) = an (z ω −ω ) = fn (z).
z−ω n=2 n=2
k=1
n−1
X
avec, fn (z) = an (z k ω n−1−k − ω n−1 ). Mais
k=1

∀ z ∈ D(0, r), | fn (z) | ≤ 2n | an | rn−1 .


P
La série entière dérivée admet R pour rayon de convergence, donc n | an | rn−1
P
converge. Alors la série fn converge normalement sur D(0, r), et chaque fonction

X
z 7→ fn (z) est continue et vaut 0 en ω ∈ D(0, r). Il en résulte que lim fn (z) = 0. Ce
z→ω
n=2
qui prouve que
S(z) − S(ω)
lim = Q(ω)
z→ω
z6=ω
z−ω
pour tout ω ∈ D(0, R). Ceci démontre que S est dérivable en tout point de D(0, R) et
que

X
∀ z ∈ D(0, R), S 0 (z) = (n + 1) an+1 z n .
n=0
6 Séries entières
P
Corollaire II.4. Soit an z n une série entière de rayon de convergence R non nul.
X∞
L’application somme z 7→ S(z) = an z n est infiniment dérivable sur le disque de
n=0
convergence. De plus,
X∞
(p) (n + p)!
∀ z ∈ D(0, R), S (z) = an+p z n .
n=0
n!

1 (p)
En particulier ∀ p ∈ IN, ap = S (0).
p!
P
Corollaire II.5. Soit an z n une série entière de rayon de convergence R non nul.
X∞
L’application somme t 7→ S(t) = an tn est infiniment dérivable sur ]−R, R[. De plus,
n=0

X∞
(p) (n + p)!
∀ t ∈] − R, R[, S (t) = an+p tn .
n=0
n!

P
Théorème II.6. Soit an tn une série entière de la variable réelle de rayon de
X∞
convergence R non nul. L’application somme t 7→ S(t) = an tn admet pour unique
n=0
X∞
an−1 n
primitive s’annulant en 0 la somme de la série entière, t qui est définie sur
n=1
n
] − R, R[.

P
Théorème II.7. (d’Abel) Soit an z n une série entière de rayon de convergence 1.

X
On suppose que la série an converge et admet S pour somme. Alors
n=0


X
lim
x→1
an xn = S.
x∈]0,1[ n=0


X n
X
n
Preuve : Posons, pour x ∈]0, 1[, f (x) = an x et, pour n ∈ IN, Sn = ak . Comme
n=0 k=0
la suite (Sn )n∈IN converge vers S alors elle est bornée, soit M > sup | Sn |.
P n∈IN
Pour tout x ∈]0, 1[, la série Sn xn converge et on a

X ∞
X ∞
X ∞
X
n n n
f (x) = S0 + (Sn − Sn−1 )x = Sn x − x Sn x = (1 − x) Sn xn .
n=1 n=0 n=0 n=0
Fonctions développables en série entière 7

X
Mais, pour tout x ∈]0, 1[, S = (1 − x) Sxn . Donc, pour tout x ∈]0, 1[,
n=0


X
f (x) − S = (1 − x) (Sn − S)xn .
n=0

Ce qui donne la majoration suivante, pour tout x ∈]0, 1[ et tout N ∈ IN∗ ,


X
| f (x) − S | ≤(1 − x) | Sn − S | x n
n=0
N
X −1 µ ¶ ∞
X
k
≤2M (1 − x) x + sup | Sn − S | (1 − x) xk
n≥N
k=0 k=N
N
≤2M (1 − x ) + sup | Sn − S | .
n≥N


Soit ε > 0. La convergence de (Sn )n∈IN
r vers S montre l’existence de N = Nε ∈ IN
ε ε
tel que sup | Sn − S | ≤ . On pose x0 = N 1 − de telle manière que x ∈ [x0 , 1[
n≥N 2 4M + ε
ε
implique 2M (1 − xN ) ≤ . D’où
2

x ∈ [x0 , 1[ =⇒ | f (x) − S | ≤ ε.

Ce qui démontre que lim f (x) = S.


<
x→1

P
Corollaire II.8. Soit an z n une série entière de rayon de convergence R. On

X
suppose que la série an z0n (avec | z0 | = R) converge. Alors
n=0


X ∞
X
lim
x→1
an z0n xn = an z0n .
x∈]0,1[ n=0 n=0

III. Fonctions développables en série entière


8 Séries entières

Définition : On dit qu’une fonction f définie sur une partie ouverte Ω de C


| à valeurs

dans C | est développable en série entière au voisinage de z ∈ Ω si, et seulement si,


0
P
il existe r > 0, tel que D(z0 , r) ⊂ Ω et une série entière an z n de rayon de convergence
supérieur ou égal à r tels que

X
∀ z ∈ C,
|
z ∈ D(z0 , r) =⇒ f (z) = an (z − z0 )n .
n=0

Remarquons que dans ce cas f est infiniment dérivable au voisinage de z0 (dans


1
C)
| et que, pour tout n ∈ IN, an = f (n) (z0 ).
n!

Définition : Soit f une application infiniment dérivable au voisinage d’un point z0 ∈ C


|

à valeurs dans C.
| On appelle série de Taylor de f au voisinage de z0 , la série

X f (z0 )
(n)
(z − z0 )n .
n=0
n!

Remarques : — Si une application admet un développement en série entière au


voisinage de z0 , alors ce développement est unique. (Car ce développement est la série
de Taylor de f au voisinage de z0 ).
— Si deux applications f et g sont développables en série entière sur un disque D
de centre z0 , et si ∀ n ∈ IN, f (n) (z0 ) = g (n) (z0 ), alors f = g dans un voisinage de z0 .

1
Proposition III.1. Pour tout p ∈ IN∗ et a ∈ C
| \ {0}, la fonction z 7→ admet,
(z − a)p
au voisinage de 0, le développement en série entière (de rayon de convergence égal à
| a |), suivant:
X∞ p−1
1 (−1)p Cn+p−1
∀ z ∈ D(0, | a |), p
= n+p
zn.
(z − a) n=0
a

Preuve : C’est une récurrence immédiate sur p.

Corollaire III.2. Toute fonction rationnelle f , n’admettant pas 0 pour pôle, est
développable en série entière au voisinage de 0. La fonction f coı̈ncide avec la somme de
son développement en série entière en tout point du disque D(0, r), avec r la distance
de 0 à l’ensemble des pôles de f , et le rayon de convergence du développement est
précisement r.
Fonctions développables en série entière 9

Preuve : Il suffit de décomposer la fraction rationnelle en éléments simples et


d’appliquer à chacun d’eux la proposition précédente.
Il est immédiat que le développement obtenu coı̈ncide avec f sur D(0, r). Si ce
développement a un rayon de convergence R > r, alors en considérant un pôle a de f
qui vérifie r = | a | on voit que la somme de la série entière (qui coı̈ncide avec f sur
D(0, | a |)) est bornée au voisinage de a. ce qui est une contradiction.

Théorème III.3. Soit f une application définie sur un voisinage de 0 à valeurs dans
C.
| Les deux propriétés suivantes sont équivalentes:
1◦ . f est développable en série entière au voisinage de 0.
2◦ . Il existe (r, M, K) ∈ (IR∗+ )3 tel que f soit infiniment dérivable sur un ouvert
contenant D(0, r) et
¯ ¯
¯ (n) ¯
∀ n ∈ IN, ∀ z ∈ D(0, r), ¯ f (z) ¯ ≤ M n!K n .


X
Preuve : 1◦ ⇒ 2◦ . Soit f (z) = an z n le développement en série entière de f au
n=0
voisinage de 0. Supposons que ce développemnt converge sur D(0, ρ), (ρ > 0). On pose
f = sup (| an | ρn ).
M
n∈IN
Soit r ∈]0, ρ[. f est infiniment dérivable sur l’ouvert D(0, ρ) qui contient D(0, r).
X∞
(p) (n + p)!
D’autre part, comme f (z) = an+p z n , alors pour tout z ∈ D(0, r) et tout
n=0
n!
p ∈ IN on a
¯ ¯ X ∞ ¯ ¯n
¯ (p) ¯ (n + p)! ¯ ¯
n¯ z¯
¯ f (z) ¯ ≤ | an+p | ρ ¯ ¯
n=0
n! ρ
∞ ¯ ¯
fX
M (n + p)! ¯¯ r ¯¯
n
≤ p ¯ρ ¯
ρ n=0 n!
f
M p! f µ 1 ¶p
ρM
= p = p! .
ρ (1 − r/ρ)p+1 ρ−r ρ−r
fρ/(ρ − r) et K = 1/(ρ − r).
Ce qui démontre le résultat demandé, avec M = M
2◦ ⇒ 1◦ . Soit z ∈ D(0, r). Posons pour t ∈ [0, 1] et n ∈ IN,
n
X (1 − t)k
ϕ(t) = z k f (k) (tz).
k!
k=0
10 Séries entières

(1 − t)n n+1 (n+1)


Un calcul simple montre que ϕ0 (t) = z f (tz). La relation
n!
Z 1
ϕ(1) − ϕ(0) = ϕ0 (t) dt
0

montre que, pour tout z ∈ D(0, r), et tout n ∈ IN,


n
X Z 1
f (k) (0) k n+1 (1 − t)n (n+1)
Rn (z) = f (z) − z =z f (tz) dt.
k! 0 n!
k=0

En utilisant les majorations du 2◦ on obtient, pour tout z ∈ D(0, r), et tout n ∈ IN,
Z 1
n+1 n+1
| Rn (z) | ≤ | z | MK (n + 1) (1 − t)n dt = M (K | z |)n+1 .
0

On conclut que, si ρ = min(r, 1/K) alors ∀ z ∈ D(0, ρ), limn→∞ Rn (z) = 0 et par
conséquent

X f (k) (0)
∀ z ∈ D(0, ρ), f (z) = zk .
k!
k=0

Remarque : On dit qu’une fonction f définie sur une partie ouverte J de IR à valeurs
dans C
| est développable en série entière de la variable réelle au voisinage de

t0 ∈ J si, et seulement si, il existe r > 0, tel que ]t0 − r, t0 + r[⊂ J et une série entière
P
an z n de rayon de convergence supérieur ou égal à r tels que

X
∀ t ∈ IR, | t − t0 | < r =⇒ f (t) = an (t − t0 )n .
n=0

Notons que dans ce cas f est infiniment dérivable au voisinage de t0 (dans IR) et que,
1
pour tout n ∈ IN, an = f (n) (t0 ).
n!

Exemple : Considérons la fonction


½ 2
e−1/t si t 6= 0
f : IR −→ IR : t 7→
0 si t = 0
2
Une récurrence immédiate montre que, pour tout t 6= 0, t 7→ e1/t f (n) (t) est une fonction
1
polynomiale en et par conséquent
t
∀ n ∈ IN, lim f (n) (t) = 0.
t→0

La fonction f est alors de classe C ∞ sur IR et f (n) (0) = 0, pour tout n ∈ IN. f n’est
pas développable en série entière au voisinage de 0. Car si f était développable en série
entière au voisinage de 0, nous aurions f ≡ 0 dans un voisinage de 0 ce qui est absurde.
Développement en série entière des fonctions usuelles 11

IV. Développement en série entière des fonctions usuelles

♣ Soit α ∈ IR \ IN, et considérons fα (t) = (1 + t)α pour t ∈] − 1, +∞[.


On a, pour tout k ∈ IN∗ et tout t ∈] − 1, +∞[,

fα(k) (t) = α(α − 1) · · · (α − k + 1)(1 + t)α−k .

La formule de Taylor avec reste intégrale s’écrit, pour t ∈] − 1, ∞[ et n ∈ IN∗ ,


n
X α(α − 1) · · · (α − k + 1)
fα (t) − 1 − tk = Rn (t, α).
k!
k=1

avec
Z 1 µ ¶n
n+1 α(α − 1) · · · (α − n) 1−u
Rn (t, α) = t (1 + tu)α−1 du.
n! 0 1 + tu

L’application u 7→ (1 − u)/(1 + tu) est décroissante sur [0, 1], ce qui permet d’écrire

1−u
∀ u ∈ [0, 1], 0≤ ≤ 1.
1 + tu

D’où la majoration suivante


Z 1
n+1 | α(α − 1) · · · (α − n) |
| Rn (t, α) | ≤ | t | (1 + tu)α−1 du = An (t, α).
n! 0
P
Si | t | < 1 alors la régle de d’Alembert montre que la série An (t, α) est
convergente, donc son terme général tend vers zéro. On a donc démontré que
lim Rn (t, α) = 0 pour tout t ∈] − 1, 1[. Alors
n→∞

X∞
α α(α − 1) · · · (α − n + 1) n
∀ t ∈] − 1, 1[, (1 + t) = 1 + t . (α ∈ IR \ IN)
n=1
n!

le rayon de convergence de la série est 1 comme le montre un calcul immédiat.


♣ Si f (t) = Log (1 + t) pour t ∈] − 1, +∞[ alors f est la primitive qui s’annule en 0
1
de t 7→ . Ce qui donne le développement en série entière de f
1+t
X∞
(−1)n−1 n
∀ t ∈] − 1, 1[, Log (1 + t) = t .
n=1
n
12 Séries entières

Le rayon de convergence de la série précédente est 1.


1
♣ Si f (t) = Arctg t pour t ∈ IR alors f est la primitive qui s’annule en 0 de t 7→ .
1 + t2
Ce qui donne le développement en série entière de f

X∞
(−1)n 2n+1
∀ t ∈] − 1, 1[, Arctg t = t ,
n=0
2n + 1

qui a 1 pour rayon de convergence. De même



1 1 + t X t2n+1
∀ t ∈] − 1, 1[, Log = .
2 1 − t n=0 2n + 1

X∞ n
C2n t2n+1
∀ t ∈] − 1, 1[, Arcsin t = .
n=0
22n 2n + 1

p ∞
X n
n C2n t2n+1
∀ t ∈] − 1, 1[, Log (t + 1+ t2 ) = (−1) 2n .
n=0
2 2n + 1

V. La fonction exponentielle complexe et ses applications

X∞
zn
Définition : La série a un rayon de convergence infini. On appelle la valeur de
n=0
n!
sa somme en z, l’exponentielle de z qui sera noté exp(z) ou ez .

Théorème V.1.
1◦ . Pour tout (z1 , z2 ) ∈ C | 2 , ez1 +z2 = ez1 ez2 .

2◦ . Pour tout z ∈ C, | ez 6= 0.
3◦ . Pour tout z ∈ C, | (ez )0 = ez .
4◦ . La restriction de exp à IR est une fonction positive strictement croissante qui
tend vers 0 en −∞ et vers +∞ en +∞.
5 . Il existe un réel positif $ tel que e$i/2 = i et tel que

ez = 1 ⇐⇒ z ∈ 2$iZZ.

6◦ . exp est une application 2$i-périodique.


7◦ . L’application t 7→ eit est une surjection de IR sur le cercle unité U de C:
|

U = {z ∈ C
|
: | z | = 1}.

8◦ . L’image de l’application exp est C


| \ {0}.
La fonction exponentielle complexe et ses applications 13

Preuve : 1◦ . En effet, ez1 et ez2 sont les sommes des séries absolument convergentes
X∞ X∞
z1 z1n z2 z2n
e = et e = . Alors, d’après le théorème sur le produit de séries, ez1 ez2
n=0
n! n=0
n!

X
est la somme de la série dn , avec
n=0

n
X z1k z2n−k (z1 + z2 )n
dn = = .
k! (n − k)! n!
k=0

Donc, ez1 ez2 = ez1 +z2 .

2◦ . En effet, ez e−z = e0 = 1. Donc ez 6= 0.

3◦ . Le théorème II.3 permet de voir immédiatement que exp0 = exp.


X∞
xn
4◦ . On voit immédiatement sur l’expression ex = que exp est strictement
n=0
n!
croissante et positive sur IR+ et tend vers +∞ en +∞. La relation ex = 1/e−x permet
de voir que exp est aussi strictement croissante et positive sur IR− , et qu’elle tend vers

0 en −∞.

5◦ . En remarquant que exp(z) = exp(z), on voit que

∀ t ∈ IR, eit = e−it .

¯ ¯2
Par conséquent, ∀ t ∈ IR, ¯ eit ¯ = 1.

Pour t ∈ IR, on pose cos t = Re(eit ) et sin t = Im(eit ), de telle manière que

∀ t ∈ IR, eit = cos t + i sin t.

En dérivant, on obtient

∀ t ∈ IR, ieit = cos0 t + i sin0 t = − sin t + i cos t.

Donc cos0 = − sin et sin0 = cos.


eit + e−it
L’expression cos t = montre que
2

X∞
(−1)n 2n
∀ t ∈ IR, cos t = t .
n=0
(2n)!
14 Séries entières

X 22n
En particulier, cos 2 = (−1)n an avec an = . Mais, si n ≥ 1 on a
n=0
(2n)!

an+1 2
= <1
an (n + 1)(2n + 1)

donc l’encadrement de la somme d’une série alternée montre que

22 24 1
cos 2 ≤ 1 − + = − < 0.
2! 4! 3

D’autre part, cos 0 = 1. La continuité de cos montre que cette fonction s’annule au
moins une fois dans l’intervalle ]0, 2[. Posons

$ = 2 min{t ∈]0, 2[: cos t = 0}.

$ $ $
Mais 1 = cos2 + sin2 , alors sin ∈ {−1, 1}. La fonction cos est strictement
2 2 2
positive sur ]0, $/2[, donc la fonction sin est strictement croissante sur cet intervalle.

Comme cette dernière est nulle en 0 alors elle est positive en $/2. On conclut que
$
sin = 1. Par conséquent ei$/2 = i.
2
Il résulte de ce qui précède que e2i$n = 1 pour tout n ∈ ZZ.
Inversement, soit (x, y) ∈ IR2 tel que z = x + iy vérifie ez = 1. On a 1 = | ez | = ex ,

alors x = 0. Car la restriction de exp à IR est strictement croissante. Posons en suite

n = E(y/2$) alors y − 2$n = t ∈ [0, 2$[ et eit = ez e−2$in = 1.


Posons alors u = cos(t/4) et v = sin(t/4), nous avons u ∈ IR∗+ et v ∈ IR+ car
t $
∈ [0, [. La relation 1 = eit = (u + iv)4 montre que
4 2

1 = u4 − 6u2 v 2 + v 4 + 4iuv(u2 − v 2 ) = 2(u2 − v 2 )2 − 1 + 4iuv(u2 − v 2 ).

(pour la deuxième égalité nous avons utilisé le fait que u2 +v 2 = 1). La partie imaginaire

de l’égalité précédente est nulle, ce qui implique immédiatement que v = sin(t/4) = 0.

Enfin, sin est strictement croissante sur [0, $/2[ donc sin(t/4) = 0 implique t = 0.

Conclusion, z = 2$in ∈ 2$iZZ.


6◦ . c’est immédiat.

7◦ . Soit ω ∈ U , alors ω = u + iv avec u2 + v 2 = 1. Distinguons plusieurs cas:


La fonction exponentielle complexe et ses applications 15

• u ≥ 0 et v ≥ 0. Comme u ∈ [0, 1] le théorème des valeurs intermédiaires


montre qu’il existe t ∈ [0, $/2] tel que cos t = u. Évidemment, sin t ≥ 0 et

vérifie sin2 t = 1 − cos2 t = 1 − u2 = v 2 , donc sin t = v et eit = ω.


• u < 0 et v ≥ 0. Alors −iω est dans le cas précèdent, donc il existe t ∈ [0, $/2]

tel que −iω = eit . D’où ω = ei(t+$/2) .


• v < 0. Alors −ω est dans l’un des deux cas précèdents, donc il existe t ∈ [0, $]

tel que −ω = eit . D’où ω = ei(t+$) .


8◦ . c’est immédiat d’après ce qui précède.

Remarque : Le nombre $, qui est évidemment unique, est noté habituellement π.

Définition : Les applications de C


| dans C
| définies par:

X∞
eiz + e−iz (−1)n 2n
cos(z) = = z ,
2 n=0
(2n)!
X∞
eiz − e−iz (−1)n 2n+1
sin(z) = = z ,
2i n=0
(2n + 1)!
X∞
ez + e−z 1
ch (z) = = z 2n ,
2 n=0
(2n)!
X∞
ez − e−z 1
sh (z) = = z 2n+1
2 n=0
(2n + 1)!

sont respectivement appelées cosinus, sinus, cosinus hyperbolique et sinus hyper-

bolique.

Remarque : Notons que ch (iz) = cos z et que sh (iz) = i sin z.

Exercice : Soit z = x+iy. Calculer en fonction de x et y, les parties réelles, imaginaires,

et les modules de cos z, sin z, ch z et de sh z.


16 Séries entières

EXERCICES

Exercice .1 Déterminer le rayon de convergence des séries entières suivantes:


X ch n X µ ¶
n 1
a. z b. Arccos 1 − 2 z n
sh 2 n n
n≥1 n≥1

X n Log n X p
n
c. z d. sin(π n2 + 1)z n
n2 + 1
n≥1 n≥1
X ³ √ n´ n X zn
e. sin π(2 + 3) z f. √ √
n≥1 n≥1
n 2 − E(n 2)
X na(na + 1) · · · (na + n − 1) X (2n)! n2n
g. z n , (a ∈ IR∗+ ) h. zn
n! 2n n! (3n)!
n≥1 n≥1

Exercice .2 Déterminer le rayon de convergence et la somme des séries entières

suivantes:
X X
a. (−1)n+1 nx2n+1 b. (n2 + 1)2n+1 xn
n≥0 n≥0
X (−1) n X n2 + n + 1
c. xn d. xn
(2n + 1)(2n + 3) n
n≥0 n≥1
Xµ 1 1
¶ X x3n
e. 1 + + ··· + xn f.
2 n (3n)!
n≥1 n≥0
X (−1)n X sin nθ
g. xn h. xn
n(n + 1) n!
n≥1 n≥0

Exercice .3 Une série entière peut-elle converger uniformément sur C| ?

P P
Exercice .4 Soient an z n et bn z n deux séries entières de rayons de convergence
P
respectifs R et R0 . Montrer que la série entière an bn z n a un rayon de convergence

R00 ≥ R0 R. Donner un exemple d’inégalité stricte, et une condition suffisante d’égalité.


P
Exercice .5 Soit an z n une série entière telle que pour tout n ∈ IN, an 6= 0. On
¯ ¯
¯ an+k ¯
suppose qu’il existe un entier k ∈ IN tel que lim ¯¯
∗ ¯ = ` ∈ [0, +∞]. Quel est le
n→∞ an ¯
P
rayon de convergence de an z n ?
P
Exercice .6 Soit a z n une série entière telle que pour tout n ∈ IN, an 6= 0.
¯ n ¯ ¯ ¯
¯ a2n+1 ¯ ¯ a2n+2 ¯
On suppose que lim ¯¯ ¯ = `1 ∈ [0, +∞] et lim ¯ ¯ = `2 ∈ [0, +∞], avec
n→∞ a2n ¯ n→∞ ¯ a2n+1 ¯
P
(`1 , `2 ) 6∈ {(0, ∞), (∞, 0)}. Quel est le rayon de convergence de an z n ?
Exercices 17

P
n
Exercice .7 Soit (an )n≥0 une suite de nombres complexes. On note An = ak .
k=0
X an X An
1◦ . Prouver que les séries entières z n et z n ont le même rayon de
n! n!
convergence.
P
2◦ . Supposons que la série an z n admet 1 pour rayon de convergence. Montrer que
P
An z n admet aussi 1 pour rayon de convergence. Donner un contre-exemple dans
P
le cas où le rayon de convergence de an z n est différent de 1.

Exercice .8 Soient f et g deux fonctions continues sur [a, b] à valeurs dans IR∗+ .
Z b
n P
On pose an = (f (t)) g(t) dt, et M = sup f (t). Montrer que la série an z n a un
a t∈[a,b]
rayon de convergence R = 1/M .

Exercice .9 On considère la suite (an )n≥0 définie par

| 2
a0 = 0, a1 = 1 et an = αan−1 + βan−2 . (α, β) ∈ C .

P
1◦ . Démontrer que le rayon de convergence R de la série entière an z n est strictement

positif.

X z

2 . Montrer que an z n = pour | z | < R.
n=0
1 − αz − βz 2
3◦ . En déduire la valeur de R.

4◦ . Application: On suppose que (α, β) = (1, 1). Déterminer an .

Exercice .10 Soient (un )n≥0 et (vn )n≥0 les deux suites définies par

u0 = v0 = 1, un+1 = un + 2vn , vn+1 = un + vn .

X un
Trouver le rayon de convergence et expliciter les sommes des séries tn et
X vn n!
tn .
n!
Exercice .11 On considère la suite (an )n≥0 définie par

a0 = 1, a1 = 1, a2 = 0 et an+3 = 6an+2 − 11an+1 + 6an .


X
Déterminer le rayon de convergence et la somme de la série entière an xn .
n=0
18 Séries entières

Exercice .12 Soit (an )n≥1 une suite réelle convergente vers a > 0.
X∞
◦ an n
1 . Montrer que z a un rayon de convergence R = 1. On note la somme f (z).
n=1
n
2◦ . On considère la restriction de f à ] − 1, 1[. Montrer que f (t) ∼− −aLog (1 − t).
1
Exercice .13 Soient (an )n≥1 et (bn )n≥1 deux suites réelles positives telles que les

X ∞
X
deux séries f (x) = an xn , et g(x) = bn xn convergent pour 0 < x < 1, et divergent
n=0 n=0
pour x = 1. Montrer que si an ∼ cbn alors f (x) ∼− cg(x).
∞ 1
Exercice .14 Trouver le développement en série entière de la fonction

F (x) = Log (1 + x + x2 ).

Exercice .15 Trouver le développement en série entière de la fonction


µ ¶
1−x π π
F (x) = Arctg tg α , α ∈] − , [.
1+x 2 2

Exercice .16 Trouver le développement en série entière de la fonction


Z x
x2 2
F (x) = e e−t dt.
0

Exercice .17 Trouver le développement en série entière des deux fonctions

1 p
f (x) = , g(x) = Log 1 − 2x ch a + x2 .
1 − 2x ch a + x2

Exercice .18 Trouver le développement en série entière de la fonction

³ p ´k
f (x) = x + 1 + x2 , k ∈ IR∗ .

(On pourrait former une équation différentielle du second ordre vérifiée par f ).

Exercice .19 Trouver le développement en série entière de la fonction

f (x) = (Arcsin x)2 .

(On pourrait former une équation différentielle du second ordre vérifiée par f ).
Exercices 19

Exercice .20

1◦ . Étudier la convergence simple et uniforme de la suite de fonctions (fn )n≥1 avec

∀ t ∈ IR, fn (t) = (1 − at)(1 − a2 t) · · · (1 − an t); (a ∈] − 1, 1[).

2◦ . On pose f (t) = lim fn (t) ; Montrer que f est l’unique fonction de IR dans IR,
n→∞

continue en 0 et vérifiant f (0) = 1, ∀ t ∈ IR, f (t) = (1 − at)f (at).


3◦ . Développer f en série entière.
Exercice .21 Donner un développement en série entière de
Z π/2
F (x) = Log (1 + x sin2 t) dt.
0

Exercice .22 Soient p un entier strictement positif fixé, et an le nombre des couples
1
(x, y) ∈ IN2 tels que x + py = n, (n ∈ IN). Développer la fonction t 7→
(1 − t)(1 − tp )
en série entière et en déduire an .
X ∞ µ ¶n2 n
1 x
Exercice .23 Montrer que 1+ a un rayon de convergence infini, et
n=1
n n!
1
que si f (x) est sa somme alors f (x) ∼ √ eex .
∞ e
Exercice .24 Trouver le dévloppement en série entière de la fonction
Z 2π
f (z) = ez cos t dt.
0
20 Séries entières

SOLUTIONS

P ch n
Solution .1 a. Le rayon de convergence de la série entière an z n avec an = .
sh 2 n

Remarquons que lim en an = 2 et par conséquent lim n an = 1/e et le rayon de
n→∞ n→∞
P
convergence de la série entière an z n est e.
µ ¶
P n 1
b. Le rayon de convergence de la série entière an z avec an = Arccos 1 − 2 .
n
Remarquons que
1
an = 2Arcsin √
2n
√ an+1
et par conséquent lim nan = 2 d’où lim = 1 et le rayon de convergence de la
n→∞ n→∞ an
P
série entière an z n est 1.

P n nLog n
c. Le rayon de convergence de la série entière an z avec an = .
n2 + 1
an+1
Il est immédiat que lim = 1 et par conséquent, le rayon de convergence de la
n→∞ an
P
série entière an z n est 1.
P p
d. Le rayon de convergence de la série entière an z n avec an = sin(π n2 + 1).

Il est immédiat que


p π
π n2 + 1 = πn + √
n2 + 1 + n
¯ ¯
π ¯ an+1 ¯
par conséquent, lim n | an | = d’où lim ¯¯ ¯ = 1 et le rayon de convergence de
n→∞ 2 n→∞ an ¯
P
la série entière an z n est 1.
P ³ √ n´
n
e. Le rayon de convergence de la série entière an z avec an = sin π(2 + 3) .
√ n √ n
Notons que bn = (2 + 3) + (2 − 3) est un nombre entier. D’où

³ √ ´ ³ √ ´ ³ √ ´
sin π(2 + 3)n = sin πbn − π(2 − 3)n = −(−1)bn sin π(2 − 3)n .
¯ ¯
| an | ¯ an+1 ¯ √
Par conséquent, lim √ = π d’où lim ¯¯ ¯ = 2 − 3 et le rayon de con-
¯
n→∞ (2 − 3)n n→∞ an
P 1 √
vergence de la série entière an z n est √ = 2 + 3.
2− 3
P 1
f. Le rayon de convergence de la série entière an z n avec an = √ √ .
n 2 − E(n 2)
Solutions 21

Notons que 2n2 − (E(n 2))2 est un nombre entier strictement positif, donc supérieur
ou égal à 1. D’où
√ √
an ≤ n 2 + E(n 2) ≤ 3n.
1 √
D’auter part, an ≥ √ par conséquent, lim n an = 1, d’où le rayon de convergence
P 2n n
n→∞

de la série entière an z est 1.


P
g. Le rayon de convergence de la série entière an z n avec

na(na + 1) · · · (na + n − 1)
an = , (a ∈ IR∗+ ).
n!

Un calcul simple montre que

a Y ³ na ´
n
an = +1 .
a+1 k
k=1

Alors
√ 1 a
n
1X ³ na ´
Log n
an = Log + Log +1 .
n a+1 n k
k=1

La fonction f (x) = Log (1 + a/x) est décroissante sur ]0, 1] donc, pour tout k ∈
{1, 2, . . . , n},
k k+1 k+1 k
t∈[ , [, =⇒ f ( ) ≤ f (t) ≤ f ( )
n n n n
d’où, pour tout k ∈ {1, 2, . . . , n},
Z (k+1)/n
1 k+1 1 k
f( )≤ f (t) dt ≤ f ( ).
n n k/n n n

Il en résulte que
n Z n−1
1X k 1
1X k
f( ) ≤ f (t) dt ≤ f ( ).
n n 1/n n n
k=2 k=1
ou bien,
Z n Z 1
1
1 1X k 1 1
f (t) dt + f (1) ≤ f( ) ≤ f( ) + f (t) dt (1)
1/n n n n n n 1/n
k=1
Z 1
Mais une intégration par parties montre que l’intégrale f (t) dt converge et qu’elle
0
(a + 1)a+1
vaut Log . Alors en faisant tendre n vers l’infini dans (1) nous trouvons
aa
n
1X k (a + 1)a+1
lim f ( ) = Log ,
n→∞ n n aa
k=1
22 Séries entières

√ (a + 1)a+1
ce qui donne lim n an = , et le rayon de convergence de la série entière
n→∞
a
aa
P a
an z n est .
(a + 1)a+1
P (2n)! n2n
h. Le rayon de convergence de la série entière an z n avec an = n .
2 n! (3n)!
Notons que
µ ¶2n
an+1 1 (2n + 1)(n + 1) 1
= 1+ ,
an 3 (3n + 1)(3n + 2) n
an+1 2e2
par conséquent, lim = , d’où le rayon de convergence de la série entière
n→∞ an 27
P 27 −2
an z n est e .
2

X
Solution .2 a. Notons S(x) = (−1)n+1 nx2n+1 . Il est immédiat que le rayon de
n=0
convergence de cette série entière est 1.
Remarquons que

X
3
S(x) = x n(−x2 )n−1 .
n=0
Mais, il est immédiat que

X 1
g(z) = (−z)n = , |z| < 1
n=0
1+z

et

X 1
−g 0 (z) = − n(−z)n−1 = , | z | < 1.
n=0
(1 + z)2
Ce qui démontre que

X x3
S(x) = (−1)n+1 nx2n+1 = , | x | < 1.
n=0
(1 + x2 )2

X
b. Notons S(x) = (n2 + 1)2n+1 xn . Il est immédiat que le rayon de convergence
n=0
de cette série entière est 1/2.
Remarquons que n2 + 1 = (n + 2)(n + 1) − 3(n + 1) + 2 d’où, si | z | < 1,

X ∞
X ∞
X ∞
X
(n2 + 1)z 2 = (n + 2)(n + 1)z n − 3 (n + 1)z n + 2 zn
n=0 n=0 n=0 n=0

alors, Ã !00 Ã !0

X ∞
X ∞
X ∞
X
2 2 n n
(n + 1)z = z −3 z +2 zn
n=0 n=0 n=0 n=0
Solutions 23

ce qui donne, pour | z | < 1,



X µ ¶00 µ ¶0
2 2 1 1 2
(n + 1)z = −3 +
n=0
1−z 1−z 1−z
soit

X 1 − z + 2z 2
(n2 + 1)z 2 = , | z | < 1.
n=0
(1 − z)3
Ce qui démontre que

X 2 − 4x + 16x2 1
S(x) = (n2 + 1)2n+1 xn = , |x| < .
n=0
(1 − 2x)3 2

X (−1)n
c. Notons S(x) = xn . Il est immédiat que le rayon de conver-
n=0
(2n + 1)(2n + 3)
gence de cette série entière est 1.
µ ¶
1 1 1 1
Remarquons que = − , et que si | x | < 1,
(2n + 1)(2n + 3) 2 2n + 1 2n + 3
X∞ ∞
X
(−1)n 2n+1 1
Arctg x = x , et Argth x = x2n+1 .
n=0
2n + 1 n=0
2n + 1

Il en résulte que si | x | < 1,



X ∞ ∞
(−1)n 2n+1 1 X (−1)n 2n+1 1 X (−1)n 2n+1
x = x − x
n=0
(2n + 1)(2n + 3) 2 n=0
(2n + 1) 2 n=0
(2n + 3)
∞ ∞
1 X (−1)n 2n+1 1 X (−1)n+1 2n+3
= x + 2 x
2 n=0 (2n + 1) 2x n=0 (2n + 3)
1 1
= Arctg x + 2 (Arctg x − x)
2 2x
2
(1 + x )Arctg x − x
=
2x2
Il en résulte que, si x ∈]0, 1[,

X √
(−1)n n (1 + x)Arctg x 1
x = √ −
n=0
(2n + 1)(2n + 3) 2x x 2x
et

X ∞ ∞
1 1X 1 1X 1
x2n+1 = x2n+1 − x2n+1
n=0
(2n + 1)(2n + 3) 2 n=0
(2n + 1) 2 n=0
(2n + 3)
∞ ∞
1X 1 1 X 1
= x2n+1 − 2 x2n+3
2 n=0 (2n + 1) 2x n=0 (2n + 3)
1 1
= Argth x − 2 (Argth x − x)
2 2x
2
(x − 1)Argth x + x
=
2x2
24 Séries entières

d’où, si x ∈] − 1, 0[,


X √
(−1)n n (1 + x)Argth −x 1
x = √ − .
n=0
(2n + 1)(2n + 3) 2x −x 2x

Ce qui donne
 (1 + x)Arctg √x 1

 √ − si x ∈]0, 1[

 2x x 2x


∞ 
1
X (−1)n
xn = 3 si x=0
(2n + 1)(2n + 3) 
 √
n=0 


 (1 + x)Argth −x 1

 √ − si x ∈] − 1, 0[
2x −x 2x

X∞
n2 + n + 1 n
d. Notons S(x) = x . Il est immédiat que le rayon de convergence
n=1
n
de cette série entière est 1.

Pour x ∈] − 1, 1[,

X X∞
n xn
S(x) = (n + 1)x +
n=1 n=1
n
̰ !
X
= xn+1 − Log (1 − x)
n=1
2x − x2
= − Log (1 − x)
(1 − x)2
X∞ µ ¶
1 1
e. Notons S(x) = 1 + + ··· + xn . Il est immédiat que le rayon de
n=1
2 n
convergence de cette série entière est 1 car les coefficients sont compris entre 1 et n.

Pour x ∈] − 1, 1[, nous avons


X X∞
n 1 xn
x = , = −Log (1 − x).
n=0
1−x n=1
n

En prenant le produit de ces deux séries entières nous obtenons, pour x ∈] − 1, 1[,

X∞ µ ¶
1 1 Log (1 − x)
S(x) = 1 + + ··· + xn =
n=1
2 n x−1

X∞
x3n
f. Notons S(x) = . Il est immédiat que le rayon de convergence de cette
n=0
(3n)!
série entière est +∞.
Solutions 25

Soit j = e2iπ/3 , de telle manière que les racines d’ordre 3 de 1 soient {1, j, j 2 }. Il est
facile de vérifier que 1 + j n + j 2n vaut 0 si n n’est pas divisible par 3, et vaut 3 si n est

divisible par 3. Il en résulte que

1³ z ´ X ∞
2 z 3n
∀ z ∈ C,
|
e + ejz + ej z =
3 n=0
(3n)!

ou bien,
à √ ! ∞
1 3 X z 3n
∀ z ∈ C,
|
ez + 2e−z/2 cos( z) = .
3 2 n=0
(3n)!

X∞
(−1)n n
g. Notons S(x) = x . Il est immédiat que le rayon de convergence de
n=1
(n + 1)n
cette série entière est 1.

Pour x ∈] − 1, 1[, nous avons

X∞ ∞
(−1)n n X (−1)n n
S(x) = x − x
n=1
n n=1
n + 1
X∞ ∞
(−1)n n 1 X (−1)n+1 n+1
= x + x
n=1
n x n=1
n + 1
(1 + x)Log (1 + x)
=1 −
x

X∞
sin nθ n
h. Notons S(x) = x . Il est immédiat que le rayon de convergence de
n=0
n!
cette série entière est +∞.

D’autre part, pour tout x

X∞
(xeiθ )n
= exp(xeiθ ) = ex cos θ (cos(x sin θ) + i sin(x sin θ))
n=0
n!

ce qui démontre que, pour tout x,

X∞
sin nθ n
S(x) = x = ex cos θ sin(x sin θ).
n=0
n!

.
26 Séries entières

Solution .3 Rappelons le fait simple qu’une fonction polynomiale bornée sur C | est

P
nécessairement constante. Si an z n est une série entière uniformément convergente

sur C,
| alors il existe un entier N > 0 tel que
¯ n ¯
¯X ¯
¯ k¯
∀ n > N, sup ¯ ak z ¯ ≤ 1,
| ¯
z∈ C ¯
k=N

il en résulte que an = 0 pour tout n ≥ N et la série entière est une fonction polynomiale.
Inversement, toute fonction polynomiale à coefficients complexes est une série entière
uniformément convergente sur C.
|

Solution .4 En effet, soit γ ∈]0, RR0 [ alors il existe α ∈]0, R[ et β ∈]0, R0 [ tels que
γ = αβ.

On conclut que

sup | an | αn < +∞, sup | bn | β n < +∞.


n∈IN n∈IN

Alors sup | an bn | γ n < +∞ et γ ≤ R00 . Comme ceci est vrai pour tout γ ∈]0, RR0 [ nous
n∈IN
concluons que RR0 ≤ R00 .

L’inégalité précédente peut être stricte comme la montre l’exemple des deux séries
P 2n P 2n+1
entières z et z .
p p
D’autre part, si les limites lim n | an | = ` et lim n | bn | = `0 existent, et vérifient
n→∞ n→∞
0 0 00
{`, ` } 6= {0, +∞}, alors RR = R .

P
Solution .5 Notons R le rayon de convergence de la série entière an z n . Posons,
¯ p ¯
¯b ¯
pour p ∈ {0, 1, . . . , k −1}, bpn = akn+p . D’après l’hypothèse nous avons lim ¯¯ n+1 ¯
p ¯ = `,
n→∞ bn
P p n
donc le rayon de convergence de la série bn z est 1/`, et le rayon de convergence de
P p kn+p √
la série bn z est 1/ k `. Mais nous avons

R = sup{r ≥ 0 : la suite (| an | rn )n est bornée}


1

k
= sup{r ≥ 0 : la suite (| bpn | rkn+p )n est bornée}
`
Solutions 27

Et,
\
{r ≥ 0 : (| an | rn )n est bornée} = {r ≥ 0 : (| bpn | rn )n est bornée}
0≤p<k
\ h √
k
i
= 0, 1/ `
0≤p<k
h √ i
k
= 0, 1/ `
P √
Donc, le rayon de convergence de la série entière an z n est 1/ k `.

Solution .6 En remarquant que, pour tout n,


¯ ¯ ¯ ¯¯ ¯ ¯ ¯ ¯ ¯¯ ¯
¯ a2n+2 ¯ ¯ a2n+2 ¯ ¯ a2n+1 ¯ ¯ ¯ ¯ ¯¯ ¯
¯ ¯=¯ ¯¯ ¯ et ¯ a2n+1 ¯ = ¯ a2n+1 ¯ ¯ a2n ¯
¯ a2n ¯ ¯ a2n+1 ¯ ¯ a2n ¯ ¯ a2n−1 ¯ ¯ a2n ¯ ¯ a2n−1 ¯
¯ ¯
¯ an+2 ¯
Nous obtenons lim ¯ ¯ ¯ = `1 `2 et en utilisant l’exercice précédent nous démontrons
n→∞ an ¯
P √
immédiatement que le rayon de convergence de la série entière an z n est 1/ `1 `2 .
P (−1)n n
La série entière n z montre que la condition (`1 , `2 ) ∈ / {(0, ∞), (∞, 0)} est
indispensable pour avoir le résultat.

Solution .7 1◦ . Notons r et R respectivement, les rayons de convergence des séries


X an X An
entières z n et zn.
n! n!
µ ¶
| An | n
Si β ∈]0, R[, alors le suite β est bornée donc majorée par une constante M .
n! n
Il en résulte que,

| an | n | An − An−1 | n | An | n | An−1 | n
β = β ≤ β + β ≤ (1 + β/n)M
n! n! n! n!
µ ¶
| an | n
et la suite β est bornée, donc β ≤ r. Mais β est arbitraire dans l’intervalle
n! n
]0, R[ ce qui démontre que R ≤ r.
n! bn+1 n+1
Inversement, posons bn = n , comme = , alors pour tout n ≥ E(β) nous
β bn β
avons bn+1 > bn et pour tout n < E(β) nous avons bn+1 ≤ bn . Il en résulte que
n
X X n
X
n ≥ β =⇒ bk = bk + bk ≤ E(β) + (n − E(β))bn−1 + bn
k=0 k<E(β) k=E(β)

donc
n
1 X βn
n ≥ β =⇒ bk ≤ 1 + E(β) + β ≤ 1 + β(1 + eβ ) = Mβ
bn n!
k=0
28 Séries entières
µ ¶
| an | n
Soit β ∈]0, r[, alors le suite β est bornée donc majorée par une constante M .
n! n
Il en résulte que
n n
| An | n 1 X 1 X
∀ n ≥ E(β), β ≤ | ak | ≤ M bk ≤ M.Mβ .
n! bn bn
k=0 k=0

µ ¶
| An | n
La suite β est, par conséquent, bornée, donc β ≤ R. Mais β est arbitraire
n! n
dans l’intervalle ]0, r[ ce qui démontre que r ≤ R, et enfin nous concluons que r = R.
X
2◦ . Notons R le rayon de convergence de la série entière An z n . Les deux séries
P P n P
entières an z n et z ont 1 pour rayon de convergence alors la série produit An z n
P
admet un rayon de convergence R ≥ 1. Inversement, la série entière an z n est la série
P
produit des deux séries entières 1 − z et An z n qui ont des rayons de convergence ∞
et R respectivement, donc 1 ≥ min(R, ∞) = R. Alors R = 1.
P
L’exemple de la série an z n avec an = 1/n! montre que le résultat précédent ne
P
subsiste pas si le rayon de convergence de an z n n’est pas 1.

Solution .8 Notons d’abord que


Z b
n
an ≤ M g(t) dt,
a


alors lim n
an ≤ M .
n→∞
D’autre part, soit ρ ∈]0, 1[, il existe, à cause de la continuité de f et la définition de M ,

un intervalle ouvert I ⊂ [a, b] tel que ∀ t ∈ I, f (t) ≥ ρM . Alors, pour tout n,


Z Z
n
0 < (ρM ) g(t) dt ≤ f n (t)g(t) dt ≤ an
I I

√ √
d’où, ρM ≤ lim an . Mais ρ ∈]0, 1[ est arbitraire donc M = lim
n n
an . Le rayon de
n→∞ n→∞
P n
convergence de la série entière an z est 1/M .

Solution .9 1◦ . Posons ρ = max(1, | α | + | β |) et Sn = max(| an | , | an−1 |). Il est


immédiat que S1 ≤ ρ et que, d’après la relation de récurrence,

| an | ≤ max(| α | , | β |)Sn−1 , et | an−1 | ≤ Sn−1


Solutions 29

ce qui démontre que Sn ≤ ρSn−1 pour tout n ≥ 2. Il en résulte, par une récurrence
immédiate, que | an | ≤ Sn ≤ ρn pour tout n ∈ IN. Le rayon de convergence de la série
P
entière an z n est R ≥ 1/ρ > 0.
2◦ . Si | z | < R, nous avons
X∞ ∞
X ∞
X ∞
X
2 n n n+1
(1 − αz − βz ) an z = an z − αan z − βan z n+2
n=0 n=0 n=0 n=0
X∞ X∞ X∞
= an z n − αan−1 z n − βan−2 z n
n=1 n=1 n=2

X
=z + (an − αan−1 − βan−2 )z n = z
n=2
ce qui démontre le résultat demandé.

3◦ . Si β = 0 alors il est immédiat que R = 1/ | α |. Supposons que β 6= 0. Notons


alors ∆ une racine carrée de α2 + 4β. Les racines dans C
| de l’équation 1 − αz − βz 2 = 0

sont
−α + ∆ −α − ∆
λ= et µ = .
2β 2β

X z −z
L’égalité an z n = 2
= montre que R ≤ min(| λ | , | µ |).
n=0
1 − αz − βz β(λ − z)(µ − z)
Pour démontrer l’inégalité inverse distinguons deux cas:

Si λ = µ alors, pour | z | < | λ |,



−z −z 2 X −n n
= = nλ z
β(λ − z)(µ − z) β(λ − z)2 α n=0
2n P
donc an = n
pour tout n et la série entière an z n admet | λ | pour rayon de
αλ
convergence dans ce cas.

Si λ 6= µ alors, pour | z | < min(| λ | , | µ |),


µ ¶ ∞
−z 1 λ µ 1 X −n
= − = (λ − µ−n ) z n .
β(λ − z)(µ − z) (λ − µ)β λ − z µ−z ∆ n=0
1 −n P
Alors, an = (λ − µ−n ), et le rayon de convergence de la série an z n est R =

min(| λ | , | µ |).

4◦ . Si α = β = 1 nous avons d’après ce qui précède, pour tout n ∈ IN,


Ãà √ !n à √ !n !
1 1+ 5 1− 5
an = √ − .
5 2 2
30 Séries entières

Solution .10 Notons An = max(| un | , | vn |). On a A0 = 1 et An+1 ≤ 3An pour tout


X∞
n un n
n. Donc pour tout n nous avons An ≤ 3 . Il en résulte que les deux séries t et
n=0
n!
X∞
vn n
t ont un rayon de convergence infini, on note f et g respectivement les sommes
n=0
n!
de ces deux séries.
En utilisant les relations de récurrence, nous avons
X∞
vn+1 n
∀ t ∈ IR, f (t) + g(t) = t = g 0 (t)
n=0
n!
X∞
un+1 n
f (t) + 2g(t) = t = f 0 (t)
n=0
n!

Il en résulte que g 0 + g = f 0 = (g 0 − g)0 = g 00 − g 0 ou bien g 00 − 2g 0 − g = 0. Cette équation

avec les conditions g(0) = 1 et g 0 (0) = 2 montrent que

1 ³ √ t(1+√2) √ t(1−√2) ´
∀ t ∈ IR, g(t) = √ (1 + 2)e − (1 − 2)e
2 2

et la relation f = g 0 − g implique

1³ √ t(1+√2) √ t(1−√2) ´
∀ t ∈ IR, f (t) = (1 + 2)e + (1 − 2)e .
2

Un calcul simple nous permet de déduire, pour tout n,


√ √ √ √
(1 + 2)n+1 + (1 − 2)n+1 (1 + 2)n+1 − (1 − 2)n+1
un = et vn = √ .
2 2 2

Solution .11 Notons An = max(| an | , | an+1 | , | an+2 |). Il est immédiat que A0 = 1

et que pour tout n nous avons An+1 ≤ 23An . Donc, pour tout n, An ≤ (23)n . Le rayon
P
de convergence R de la série entière an z n est alors strictement positif (R ≥ 1/23).

X
Notons alors f (z) = an z n pour | z | < R.
n=0
Pour tout z vérifiant | z | < R nous avons

X ∞
X ∞
X ∞
X ∞
X
n n+3 n+2 2 n+1 3
an z = an+3 z = 6z an+2 z − 11z an+1 z + 6z an z n
n=3 n=0 n=0 n=0 n=0

ce qui démontre que

| z | < R =⇒ (1 − 6z + 11z 2 − 6z 3 )f (z) = 1 − 5z + 5z 2 .


Solutions 31

L’égalité précédente s’écrit aussi

| z | < R =⇒ (1 − z)(1 − 2z)(1 − 3z)f (z) = 1 − 5z + 5z 2 .

Comme 1 − 5z + 5z 2 ne s’annule pas en 1,1/2 ou 1/3 alors l’égalité précédente montre

que R ≤ 1/3.
D’autre part la fonction

1 − 5z + 5z 2 1 1 1
f (z) = = + −
(1 − z)(1 − 2z)(1 − 3z) 2(1 − z) 1 − 2z 2(1 − 3z)

est développable en série entière au voisinage de 0 et la série obtenue admet 1/3 pour
P
rayon de convregence. On conclut que le rayon de convergence de la série entière an z n
est 1/3 et que
1 − 3n
∀ n ∈ IN, an = + 2n .
2

Solution .12 1◦ . Il existe un entier n0 tel que, pour tout n ≥ n0 , nous avons
a an 3a X zn
≤ ≤ . Mais la série entière admet 1 pour rayon de convergence, donc
2n n 2n nX
an n
le rayon de convergence de la série entière z est 1.
n
2◦ . Soit t ∈]0, 1[, on a

X∞
an − a n
f (t) + aLog (1 − t) = t .
n=1
n

Soit ε > 0, il existe N tel que, pour tout n ≥ N , | an − a | ≤ ε/2. Il en résulte que
N
X ∞
ε X tn
∀ t ∈]0, 1[, | f (t) + aLog (1 − t) | ≤ | an − a | +
n=1
2 n
n=N +1
N
X ∞
ε X tn
≤ | an − a | +
n=1
2 n=1 n
N
X ε
= | an − a | + | Log (1 − t) |
n=1
2

ou bien,
N
X
¯ ¯ | an − a |
¯ f (t) ¯ ε
∀ t ∈]0, 1[, ¯ ¯ n=1
¯ Log (1 − t) + a ¯ ≤ | Log (1 − t) | + 2 .
32 Séries entières

Il existe alors un t0 ∈]0, 1[ tel que

¯ ¯
¯ f (t) ¯ ε ε
t ∈]t0 , 1[ =⇒ ¯¯ + a ¯¯ ≤ + = ε.
Log (1 − t) 2 2

Ceci démontre que f (t) ∼− −aLog (1 − t).


1

ε
Solution .13 Soit ε > 0, il existe N tel que | an − cbn | ≤ bn pour tout n ≥ N .
2
Alors, pour tout x ∈]0, 1[, nous avons

¯ ¯ N −1
¯X ∞ ¯ X ∞
ε X
¯ n¯
| f (x) − cg(x) | = ¯ (an − cbn )x ¯ ≤ | an − cbn | + bn xn
¯ ¯ 2
n=0 n=0 n=N
N
X −1
ε
≤ | an − cbn | + g(x)
n=0
2

ou bien, pour tout x ∈]0, 1[, nous avons

N
X −1

¯ ¯ | an − cbn |
¯ f (x) ¯ ε
¯ − c ¯≤ n=0
+ .
¯ g(x) ¯ g(x) 2

Mais lim g(x) = +∞, donc il existe x0 ∈]0, 1[ tel que


<
x→1

¯ ¯
¯ f (x) ¯ ε ε
∀ x ∈]x0 , 1[, ¯ − c ¯ ≤ + = ε.
¯ g(x) ¯ 2 2

Alors, f (x) ∼− cg(x).


1

Solution .14 Pour x ∈] − 1, 1[ nous pouvons écrire

X∞ ∞ ∞
3 xn X x3n X
F (x) = Log (1 − x ) − Log (1 − x) = − = an xn
n=1
n n=1
n n=1

1 2
avec an = si n 6= 0 mod (3) et an = − si n = 0 mod (3).
n n
Solutions 33

Solution .15 Un calcul simple montre que, pour tout x ∈] − 1, 1[, nous avons
µ ¶
0 − sin 2α 1 e−2iα e2iα
F (x) = = −
1 + 2x cos 2α + x2 2i 1 + xe−2iα 1 + xe2iα

Il en résulte que pour tout x ∈] − 1, 1[,


Ã∞ ∞
!
1 X X
F 0 (x) = e−2i(n+1)α (−x)n − e2i(n+1)α (−x)n
2i n=0 n=0

X
= (−1)n+1 sin(2(n + 1) α)xn .
n=0

On conclut, en intégrant,
µ ¶ X∞
1−x sin 2nα n
F (x) = Arctg tg α = α + (−1)n x , | x | < 1.
1+x n=1
n

Solution .16 Remarquons que F est l’unique solution du problème différentiel

y 0 (x) = 2xy(x) + 1 et y(0) = 0. IP



X
Si S(x) = an xn est une solution développable en série entière de rayon de conver-
n=0
gence R > 0 du problème IP, alors

X ∞
X
n
a0 = 0, et (n + 1)an+1 x = 1 + 2an−1 xn
n=0 n=1

ce qui implique

2an−1
a0 = 0, a1 = 1, et an+1 = , pour n ≥ 1
n+1

ou bien,
22n n!
∀ n ≥ 0, a2n = 0, a2n+1 = .
(2n + 1)!

X
22n n!
Il est immédiat que le rayon de convergence de la série entière xn
n=0
(2n + 1)!
définissant S(x) est infini et la fonction S est, par construction, est une solution du

problème IP. L’unicité montre alors que F = S sur IR et donc


Z x X∞
x2 −t2 22n n!
∀ x ∈ IR, e e dt = xn .
0 n=0
(2n + 1)!
34 Séries entières

Solution .17 Supposons d’abord α 6= 0. Pour tout x vérifiant | x | < e−| α | ,


µ ¶
1 1 eα e−α
f (x) = = −
(1 − xeα )(1 − xe−α ) 2 sh α 1 − xeα 1 − xe−α

1 X (n+1)α
= (e − e−(n+1)α ) xn
2 sh α n=0
X∞
sh (n + 1)α n
= x
n=0
sh α

sh (n + 1)α
qui est aussi valable pour α = 0 si l’on pose = n + 1 en α = 0.
sh α
D’autre part, pour x ∈] − e−| α | , e−| α | [,

x − ch α
g 0 (x) = = xf (x) − ch α f (x)
1 − 2x ch α + x2

d’où, si x ∈] − e−| α | , e−| α | [,

X∞ ∞
0 sh nα n X ch α sh (n + 1)α n
g (x) = x − x
n=1
sh α n=0
sh α
X∞
sh nα − ch α sh (n + 1)α n
= − ch α + x
n=1
sh α

X ∞
X
n
= − ch α − ch (n + 1)α x = − ch (n + 1)α xn .
n=1 n=0

Ce qui démontre que

p X∞
ch nα n
g(x) = Log 1 − 2x ch α + x2 = x pour | x | < e−| α | .
n=1
n

Solution .18 Notons que

p µ ¶
0 2 k−1 x k
f (x) = k(x + 1 + x ) 1+ √ =√ f (x)
1+x2 1 + x2

puis
−kx k2
f 00 (x) = f (x) + f 0 (x).
(1 + x2 )3/2 1 + x2
Il en résulte que f est l’unique solution du problème différentiel

(1 + x2 )y 00 (x) + xy 0 (x) − k 2 y(x) = 0, y(0) = 1, y 0 (0) = k. IP


Solutions 35
P
Supposons qu’il existe une série entière an xn de rayon de convergence non nul, dont
la somme S soit solution du problème IP. Alors dans l’intervalle de convergence nous

avons


X ∞
X ∞
X ∞
X
n n n 2
(n + 2)(n + 1) an+2 x + n(n − 1)an x + nan x − k an xn = 0
n=0 n=0 n=0 n=0

ou bien

X ¡ ¢
(n + 2)(n + 1) an+2 + an (n2 − k 2 ) xn = 0.
n=0

On conclut que

k 2 − n2
a0 = 1, a1 = k, an+2 = an , pour n ≥ 0.
(n + 2)(n + 1)

Une récurrence simple montre qu’alors, pour n ≥ 1,

n−1 n−1
1 Y 2 k Y
a2n = (k − 4p2 ), a2n+1 = (k 2 − (2p + 1)2 ).
(2n)! p=0 (2n + 1)! p=0

P
Pour les (an )n ainsi définis, la série entière an xn admet 1 pour rayon de convergence

et la fonction S définie sur ] − 1, 1[ par

n−1
Y n−1
Y
(k 2 − 4p2 ) k (k 2 − (2p + 1)2 )

X ∞
X
p=0 p=0
S(x) = 1 + kx + x2n + x2n+1
n=1
(2n)! n=1
(2n + 1)!

est une solution du problème IP, donc d’après l’unicité de cette solution nous avons

S = f sur ] − 1, 1[, soit pour x ∈] − 1, 1[,

n−1
Y n−1
Y
2 2
(k − 4p ) k (k 2 − (2p + 1)2 )
p ∞
X ∞
X
p=0 p=0
(x + 1 + x2 )k = 1 + kx + x2n + x2n+1 .
n=1
(2n)! n=1
(2n + 1)!
36 Séries entières

Solution .19 Si f (x) = (Arcsin x)2 alors pour tout x ∈] − 1, 1[


2 2x 2
f 0 (x) = √ Arcsin x, f 00 (x) = Arcsin x + .
1 − x2 2
(1 − x ) 3/2 1 − x2
Alors f est l’unique solution du problème différentiel IP

(1 − x2 )y 00 (x) − xf 0 (x) − 2 = 0, y(0) = 0, y 0 (0) = 0 IP


P
Supposons qu’il existe une série entière an xn de rayon de convergence non nul, dont
la somme S soit solution du problème IP. Alors dans l’intervalle de convergence nous
avons

X ∞
X ∞
X
n n
(n + 2)(n + 1) an+2 x − n(n − 1)an x − nan xn − 2 = 0
n=0 n=0 n=0

ou bien

X ¡ ¢
2a2 − 2 + (n + 2)(n + 1) an+2 − n2 an xn = 0.
n=1
On conclut que
n2
a0 = 0, a1 = 0, a2 = 1, an+2 = an , pour n ≥ 1.
(n + 2)(n + 1)
Une récurrence simple montre qu’alors, pour n ≥ 1,
22n−1
a0 = 0, a2n−1 = 0, a2n = n .
n2 C2n
P
Pour les (an )n ainsi définis, la série entière an xn admet 1 pour rayon de convergence

et la fonction S définie sur ] − 1, 1[ par


X∞
22n−1 2n
S(x) = 2C n
x
n=1
n 2n

est une solution du problème IP donc d’après l’unicité de cette solution nous avons

S = f donc, pour x ∈] − 1, 1[,


X∞
2 22n−1 2n
(Arcsin x) = 2C n
x .
n=1
n 2n

Ce qui permet de démontrer



X ∞
X X∞
1 π2 1 2π 1 2π 1
2C n
= , n = √ , n = √ +
n=1
n 2n 18 n=1
nC2n 3 3 C
n=1 2n
9 3 3
Solutions 37

N
Solution .20 1◦ . Soit A > 0. Il existe N ∈ IN tel que | a | A < 1/2, alors

¯ k ¯ 1
∀ k > N, ∀ t ∈ [−A, A], ¯ a t ¯ < | a |k−N .
2

Or, notons que si x ∈] − 1/2, 1/2[ alors | Log (1 + x) | ≤ 2 | x |. Car


Z x Z 1
Log (1 + x) 1 dt du
= = ≤2
x x 0 1+t 0 1 + xu

Il en résulte

¯ ¯ ¯ ¯
∀ k > N, ∀ t ∈ [−A, A], ¯ Log (1 − ak t) ¯ ≤ 2 ¯ ak t ¯ < | a |k−N .

X
La série Log (1 − ak t) converge donc normalement sur [−A, A]. Notons hN la
k>N
somme de cette série qui est une fonction continue sur [−A, A]. Posons GN (t) =
fN (t) exp(hN (t)) pour t ∈ [−A, A] et M = sup | GN (t) |.
t∈[−A,A]
Soit ε > 0 il existe η > 0 tel que (eη − 1)M ≤ ε. Mais
¯ ¯
¯ X n ¯
¯ k ¯
lim sup ¯ Log (1 − a t) − hN (t) ¯ = 0
n→∞ t∈[−A,A] ¯ ¯
k=N +1

donc il existe n0 > N tel que

fn (t)
∀ n ≥ n0 , ∀ t ∈ [−A, A], −η ≤ Log − hN (t) ≤ η
fN (t)

soit,
fn (t)
∀ n ≥ n0 , ∀ t ∈ [−A, A], −η ≤ Log − hN (t) ≤ η
fN (t)
ou bien,
fn (t)
∀ n ≥ n0 , ∀ t ∈ [−A, A], e−η − 1 ≤ − 1 ≤ eη − 1
GN (t)
∀ n ≥ n0 , ∀ t ∈ [−A, A], | fn (t) − GN (t) | ≤ (eη − 1) | GN (t) | ≤ ε.

Ce qui démontre que (fn )n converge uniformément sur tout intervalle [−A, A] de IR. La
limite f est une fonction continue sur IR.
2◦ . Il est immédiat que, fn (t) = (1 − at)fn−1 (at) pour tout n ≥ 2 et tout t. Alors

en passant à la limite nous obtenons ∀ t ∈ IR, f (t) = (1 − at)f (at). Inversement, soit g
38 Séries entières

une fonction continue en 0 telle que g(0) = 1 et ∀ t ∈ IR, g(t) = (1 − at)g(at). Alors par
récurrence sur n nous démontrons que pour tout n ≥ 1 et tout t ∈ IR, g(t) = fn (t)g(an t).

Mais lim g(an t) = g(0) = 1, alors en faisant tendre n vers l’infini nous obtenons g = f .
n→∞
n
Y
◦ ak n
3 . Considérons, pour n ≥ 1, bn = (−1) et b0 = 1. Il est immédiat que
1 − ak
k=1
bn+1 P
lim = 0, donc le rayon de convergence de la série entière bn tn est infini. On
n→∞ bn

X
pose alors pour t ∈ IR, h(t) = bn tn .
n=0
Il est facile de voir que h est continue en 0 et que h(0) = 1, de plus

X ∞
X
n n
(1 − at)h(at) = bn a t − bn an+1 tn+1
n=0 n=0

X ∞
X
n+1 n+1
=1 + bn+1 a t − bn an+1 tn+1
n=0 n=0
X∞
=1 + (bn+1 − bn )an+1 tn+1
n=0
X∞
=1 + bn+1 tn+1 = h(t).
n=0

(Où l’on a utilisé (bn+1 − bn )an+1 = bn+1 ). Il en résulte que h = f d’après 2◦ . On

conclut que à !

Y ∞
X n
Y
n ak
∀ t ∈ IR, (1 − a t) = 1 + tn .
n=1
ak − 1
k=1 k=1
En particulier,
∞ ∞
à n
!
Y X Y ak
∀ a ∈] − 1, 1[, (1 + an ) = 1 + .
n=1
1 − ak
k=1 k=1

X xn sin2n t
Solution .21 Soit x ∈] − 1, 1[. La série (−1)n−1 converge normalement
n=1
n
par rapport à t, et sa somme vaut Log (1 + x sin2 t). En intégrant terme à terme
Z π/2 ∞
ÃZ !
X (−1)n−1 π/2
2 2n
Log (1 + x sin t) dt = sin t dt xn .
0 n=1
n 0
Z π/2
π
Mais un calcul simple montre que sin2n t dt = 2n+1 C2n n
. Donc
0 2
Z π/2 X∞
2 (−1)n−1 n n
∀ x ∈] − 1, 1[, Log (1 + x sin t) dt = π C x .
0 n=1
n22n+1 2n
Solutions 39

Solution .22 Remarquons d’abord que

¡ ¢
an = Card {(x, y) ∈ IN2 : x + py = n} ∈ [1, n].

P
Alors le rayon de convergence de la série entière an tn est 1.

D’autre part, si | t | < 1,


à ∞
!Ã ∞
! ∞
1 X X X
k `p
= t t = an tn .
(1 − t)(1 − tp ) n=0
k=0 `=0

Mais, pour t ∈] − 1, 1[,

1 1 + t + . . . + tp−1
=
(1 − t)(1 − tp ) (1 − tp )2

X
p−1
=(1 + t + . . . + t ) (n + 1)tnp
n=0

X
= (n + 1)(tnp + tnp+1 + . . . + tnp+p−1 )
n=0
X∞ µ µ ¶¶
n
= 1+E tn
n=0
p
µ ¶
n
Alors an = 1 + E .
p

Solution .23 Remarquons que pour tout n ≥ 1,

1 1 1 1 1 1
− 2 ≤ Log (1 + ) ≤ − 2 + 3
n 2n n n 2n 3n

d’où,
1 1 1 1
≤ n2 Log (1 + ) ≤ n − +
n−
2 n 2 3n
µ ¶n 2
en 1 en en
√ ≤ 1+ ≤ √ + √ (e1/3n − 1).
e n e e
e
Mais le théorème des accroissements finis permet de démontrer que e1/3n − 1 ≤ ,
1+n
donc le rayon de convergence de la série entière définissant f est infini, et pour tout
x > 0,
∞ ∞ ∞
1 X (ex)n 1 X (ex)n 1 X (ex)n+1
√ ≤ f (x) ≤ √ +√
e n=1 n! e n=1 n! e x n=1 (n + 1)!
40 Séries entières

ce qui permet d’écrire, pour x > 0,


µ ¶
eex − 1 1 eex
√ ≤ f (x) ≤ 1+ √ .
e x e

√ 1
Il en résulte que lim e e−ex f (x) = 1, ou bien f (x) ∼ √ eex .
x→∞ ∞ e

X∞
cosn t n
Solution .24 Fixons z ∈ C, la série | z converge normalement par rapport
n=0
n!
à t et sa somme vaut ez cos t . On peut alors intégrer terme à terme:
Z 2π X∞ µZ 2π ¶
z cos t 1
e dt = cos t dt z n .
n
0 n=0
n! 0

Z 2π
2π n
Mais, si In = cosn t dt, alors I2n+1 = 0, et I2n = C . Par conséquent
0 22n 2n
Z 2π ∞
X
1 z 2n
∀ z ∈ C,
|
ez cos t dt = 2n (n!)2
.
2π 0 n=0
2

OKMRAN
OUBA
SÉRIES DE FOURIER

I. L’espace R2π

Définition : Soient [a, b] un intervalle compact de IR, et f une application de [a, b]


dans C
| . On dit que l’application f est réglée si, et seulement si, pour tout ε > 0 il

existe une application ϕε : [a, b] −→ C


| en escalier telle que sup | f (x) − ϕε (x) | < ε.
x∈[a,b]
Ce qui est équivalent à dire que f est limite uniforme sur [a, b] d’une suite d’applications
en escalier.

Définition : On note R2π l’ensemble des applications f : IR −→ C


| telles que, d’une

part, f admet 2π pour période, i.e. f est 2π-périodique, et d’autre part, la restriction
de f à [0, 2π] est réglée.
Il est facile de vérifier que R2π est un sous-espace vectoriel de l’espace vectoriel des
applications bornées sur IR et localement intégrables. Nous admettrons le résultat
suivant qui donne une caractérisation des éléments de R2π .
Théorème I.1 Soit f : IR −→ C.
| Alors f ∈ R2π si, et seulement si, f est 2π-périodique
et f admet une limite à gauche et une limite à droite en tout point de IR.

Si f ∈ R2π et x ∈ IR, nous utiliserons les notations

f (x− ) = lim f (t), et f (x+ ) = lim f (t).


< >
t→x t→x
Remarques :
♠ Dans ce chapitre nous ne considérons que les fonctions 2π-périodiques, étant entendu
que si une fonction f est T -périodique on se ramène au cas des fonctions 2π-périodique
T
en étudiant g(x) = f ( 2π x).
♠ L’espace R2π contient l’espace vectoriel C2π des fonctions continues et 2π-périodiques,
et contient aussi l’espace vectoriel CM2π des fonctions continues par morceaux et 2π-
périodiques. Z a+2π
♠ Si f ∈ R2π alors l’intégrale f (t) dt ne dépand pas de a ∈ IR et on la note
Z a

f (t) dt.
TT
2 Séries de Fourier

Si f et g sont deux fonctions de R2π , alors on pose


Z p
1
hf, gi = f (t)g(t) dt, et k f k2 = hf, f i.
2π TT

L’application h·, ·i est une forme sesquilinéaire positive sur R2π mais elle n’est pas
un produit scalaire sur R2π , car hf, f i = 0 n’implique pas que f = 0. (En effet,
hf, f i = 0 implique que f est nulle en tout point ou elle est continue donc partout
sauf eventuellement en un ensemble au plus dénombrable de points). On dit alors que
h·, ·i (resp. k · k2 ) est un semi-produit scalaire (resp. semi-norme) sur R2π .
Remarquons que la restriction de h·, ·i à C2π ×C2π est un produit scalaire et la restriction
de k · k2 à C2π est une norme.

Nous utiliserons aussi sur R2π , la norme

k f k∞ = sup | f (t) | = sup | f (t) | .


x∈IR x∈[0,2π]

Soient f et g deux éléments de R2π . Pour x ∈ IR, l’application t 7→ f (t)g(x − t) est


réglée, donc localement intégrable, et 2π-périodique (i.e. élément de R2π ). On pose alors
Z
1
f ∗ g(x) = f (t)g(x − t) dt.
2π TT

Définition : Si (f, g) ∈ R22π alors l’application 2π-périodique f ∗ g qui à x associe


f ∗ g(x), s’appelle le produit de convolution de f et g.

Le changement de variable t 7→ x − u dans l’intégrale définissant f ∗ g(x) montre que le


produit de convolution est commutatif, i.e. f ∗ g = g ∗ f .

Théorème I.2 : Soient f et g deux éléments de R2π , alors f ∗ g est une application
continue.
(f, g) ∈ R2π × R2π =⇒ f ∗ g ∈ C2π .
Coefficients et séries de Fourier 3

Preuve : Comme g est réglée, 2π-périodique alors, il existe une suite d’applications
2π-périodiques (gn )n∈IN , telle que, pour tout n, gn est en escalier sur [0, 2π] et
lim k g − gn k∞ = 0.
n→∞
Posons hn = f ∗ gn . Alors, pour tout x ∈ IR,
Z
1 1
| f ∗ g(x) − hn (x) | ≤ | f (x − t) | | g(t) − gn (t) | dt ≤ k f k∞ k g − gn k∞ .
2π TT 2π

Donc, la suite (hn )n∈IN converge uniformément vers f ∗ g.


Montrons que pour tout n la fonction hn est continue. Chaque fonction gn est une
combinaison linéaire de fonctions indicatrices d’intervalles, de la forme λ = 1I[a,b[ ∗ .
Alors il suffit, pour voir que hn est continue, de prouver que f ∗ λ est continue. Mais
Z b Z x−a
1 1
f ∗ λ(x) = f (x − t) dt = f (u) du.
2π a 2π x−b

Donc, pour tout (x, y) ∈ IR2 ,


ÃZ Z !
x−a y−b
1
f ∗ λ(x) − f ∗ λ(y) = f (u) du + f (u) du
2π y−a x−b

puis,
1
| f ∗ λ(x) − f ∗ λ(y) | ≤ k f k∞ | x − y | .
π
On conclut que f ∗ λ est continue, et par conséquent, que hn est continue sur IR.
L’application f ∗ g est une limite uniforme sur IR d’une suite d’applications continues,
elle est donc continue sur IR.

II. Coefficients et séries de Fourier

Définition : Pour k ∈ ZZ et x ∈ IR, on pose ek (x) = eikx . On appelle polynôme


trigonométrique toute combinaison linéaire des éléments de {ek : k ∈ ZZ}. On note
P = Vect (ek : k ∈ ZZ), et pour n ∈ IN, Pn = Vect (ek : | k | ≤ n). Les espaces P, et
Pn sont des sous-espaces vectoriels de R2π . Rappelons que la famille (ek )k∈ZZ est une
famille orthonormale de R2π .


c’est à dire λ(x) = 1 si x ∈ [a, b[ et λ(x) = 0 sinon.
4 Séries de Fourier

Soit f ∈ R2π . Considérons Tf l’endomorphisme de R2π défini par Tf (g) = f ∗ g. Pour


k ∈ ZZ et x ∈ IR, on a
Z
1
Tf (ek )(x) =f ∗ ek (x) = f (t)eik(x−t) dt
2π TT
µZ ¶
1 −ikt
= f (t)e dt ek (x) = hek , f i ek (x).
2π TT

Ce qui permet de poser la définition suivante :

Définition : Si f ∈ R2π , nous posons


Z
1
∀ n ∈ ZZ, Cn (f ) = f (t) e−int dt = hen , f i.
2π TT

Cn (f ) s’appelle le coefficient de Fourier exponentiel d’ordre n de f . La famille


(Cn (f ))n∈ZZ s’appelle le spectre de Tf † .

Remarque : Nous avons vu que

∀ f ∈ R2π , ∀ n ∈ ZZ, f ∗ en = Cn (f ) en .

Définition : Si f ∈ R2π , nous posons


Z
1
∀ n ∈ IN , an (f ) = f (t) cos nt dt
π TT
Z
1
∀ n ∈ IN∗ , bn (f ) = f (t) sin nt dt
π TT

an (f ) (resp. bn (f )) s’appelle le coefficient de Fourier en cosinus (resp. en


sinus) d’ordre n de f . Les deux familles (an (f ))n∈IN et (bn (f ))n∈IN∗ s’appellent les
coefficients de Fourier trigonométriques de f .

Proposition II.1 : Soit f ∈ R2π . Pour tout n ∈ IN, on a

an (f ) = Cn (f ) + C−n (f ), et bn (f ) = i(Cn (f ) − C−n (f )).

Convention : Soit (λn )n∈ZZ une famille d’un espace vectoriel normé. La notation
X ∞
X
λn désigne la série λ0 + (λn + λ−n ).
n∈ZZ n=1


On dit aussi le spectre de f par abus de langage.
Propriétés des coefficients de Fourier 5

Définition : Soit f ∈ R2π . On appelle série de Fourier de f , la série d’applications


X
S(f ) = Cn (f )en
n∈ZZ
ou bien,

a0 (f ) X
S(f )(x) = + an (f ) cos nx + bn (f ) sin nx.
2 n=1

III. Propriétés des coefficients de Fourier

Proposition III.1. Les propriétés suivantes sont vérifiées.


1◦ . Pour tout (f, g) ∈ R2π × R2π , (λ, µ) ∈ C
| 2 et n ∈ ZZ, on a

Cn (λf + µg) = λCn (f ) + µCn (g).

2◦ . Si τ ∈ IR et fτ ∈ R2π est définie par fτ (x) = f (x−τ ), alors pour tout f ∈ R2π ,
τ ∈ IR et n ∈ ZZ, on a Cn (fτ ) = Cn (f )e−inτ .
3◦ . Pour tout f ∈ R2π et (n, m) ∈ ZZ2 , on a Cn (em · f ) = Cn−m (f ).
4◦ . Pour tout (f, g) ∈ R2π × R2π et n ∈ ZZ, on a Cn (f ∗ g) = Cn (f ).Cn (g).

La preuve, sauf pour 4◦ , est immédiate, elle est laissée au lecteur. Pour 4◦ , c’est
facile si f et g sont continues. Le cas général s’obtient par densité.

Proposition III.2. (Inégalité de Bessel) Soit f ∈ R2π . Alors


X Z
2 1 2
| Cn (f ) | ≤ | f (t) | dt.
2π TT
n∈ZZ

Preuve : Notons
n
X n
X
Sn (f ) = Ck (f )ek = hek , f iek .
k=−n k=−n
Il est facile de voir que hf − Sn (f ), ek i = 0 pour tout k ∈ {−n, 1 − n, . . . , n}, donc
hf − Sn (f ), Sn (f )i = 0. Alors
2 2 2 2
k Sn (f ) k2 ≤ k Sn (f ) k2 + k f − Sn (f ) k2 = k f k2 .
n
X
2 2
Mais k Sn (f ) k2 = | Ck (f ) | . D’où
k=−n
n
X Z
2 1 2
∀ n ∈ IN, | Ck (f ) | ≤ | f (t) | dt.
2π TT
k=−n

Ce qui donne le résultat.


6 Séries de Fourier

Corollaire III.3. Soit f ∈ R2π . Alors


∞ Z
1 2 1X 2 2 1 2
| a0 (f ) | + (| an (f ) | + | bn (f ) | ) ≤ | f (t) | dt.
4 2 n=1 2π TT

Corollaire III.4. Soit f ∈ R2π . Alors,

lim Cn (f ) = lim C−n (f ) = lim an (f ) = lim bn (f ) = 0.


n→∞ n→∞ n→∞ n→∞

Proposition III.5. Soient (f, g) ∈ R2π × R2π , et h = f ∗ g. Alors la série de Fourier


de h converge normalement. Plus précisement,
X
| Cn (f ∗ g) | ≤ k f k2 k g k2 .
n∈ZZ

Preuve : On peut supposer k f k2 6= 0. Soit λ ∈ IR∗+ . Pour tout n ∈ ZZ, nous avons
1
Cn (h) = Cn (f )Cn (g) = Cn (λf )Cn ( g). Alors†
λ
à ¯ ¯2 !
1 ¯ 1 ¯
| Cn (λf ) | + ¯¯ Cn ( g) ¯¯ .
2
∀ n ∈ ZZ, | Cn (h) | ≤
2 λ

En prenant la somme,
n
X µ ¶
1 2 2 1 2
∀ n ∈ ZZ, | Ck (h) | ≤ λ k f k2 + 2 k g k2 .
2 λ
k=−n

Choisissons λ2 qui rend minimum le second membre de l’inégalité précédente, c’est à


dire λ2 = k g k2 / k f k2 . Alors

n
X
∀ n ∈ ZZ, | Ck (h) | ≤ k f k2 k g k2 .
k=−n

Ce qui démontre le résultat.


En utilisant, ab ≤ (a2 + b2 )/2.
Convergence ponctuelle des séries de Fourier 7

Proposition III.6. Soit f une fonction 2π-périodique, dérivable telle que f 0 ∈ R2π .
Alors
1
∀ n ∈ ZZ \ {0}, Cn (f ) = Cn (f 0 ).
in

Preuve : C’est une intégration par parties. En effet,


Z 2π
0 1
Cn (f ) = f 0 (t) e−int dt
2π 0
¯2π Z
1 ¯
−int ¯ in 2π
= f (t) e ¯ + 2π f (t) e−int dt
2π 0 0
f (2π) − f (0)
= + inCn (f ) = inCn (f ).

Corollaire III.7. µ Soit


¶ f une fonction 2π-périodique, k-fois continuement dérivable.
1
Alors Cn (f ) = o .
nk

Preuve : Car, d’après la proposition précédente, une récurrence immédiate permet


¯ ¯
de démontrer que nk | Cn (f ) | = ¯ Cn (f (k) ) ¯. Ce qui prouve le résultat.

IV. Convergence ponctuelle des séries de Fourier

Le lemme suivant est essentiel pour la suite :

Lemme IV.1. (de Riemann) Soit g : [a, b] −→ C


| une application réglée. Alors

Z b
lim g(t) sin(λt) dt = 0.
λ−→∞ a

Preuve : En effet, le résultat est vrai pour toute fonction g indicatrice d’intervalle,
comme le montre un calcul direct dans ce cas. Donc, le résultat reste vrai pour toute
fonction g en escalier sur [a, b].
Venons au cas général, soit g une fonction réglée sur [a, b]. Pour ε > 0, il existe hε
en escalier sur [a, b] telle que

ε
sup | g(x) − hε (x) | < .
x∈[a,b] 2(b − a)
8 Séries de Fourier

Pour hε ainsi choisie, il existe λε tel que


¯Z ¯
¯ b ¯ ε
¯ ¯
λ ≥ λε =⇒ ¯ hε (t) sin(λt) dt ¯ ≤ .
¯ a ¯ 2

D’où, si λ ≥ λε ,
¯Z ¯ ¯Z ¯ ¯Z ¯
¯ b ¯ ¯ b ¯ ¯ b ¯
¯ ¯ ¯ ¯ ¯ ¯
¯ g(t) sin(λt) dt ¯ ≤ ¯ h (t) sin(λt) dt ¯ + ¯ (g(t) − hε (t)) sin(λt) dt ¯
¯ a ¯ ¯ a ε ¯ ¯ a ¯
¯Z ¯
¯ b ¯
¯ ¯
≤¯ hε (t) sin(λt) dt ¯ + (b − a) sup | g(x) − hε (x) | ≤ ε.
¯ a ¯ x∈[a,b]

Ce qui prouve le résultat.

Soit f ∈ R2π . Nous avons vu que


n n
à n
!
X X X
Sn (f ) = Ck (f )ek = f ∗ ek = f ∗ ek . (1)
k=−n k=−n k=−n

n
X
On appelle noyau de Dirichlet l’élément Dn = ek de R2π .
k=−n
Un calcul simple montre que, pour n ∈ IN et x ∈ IR \ 2πZZ,
n
X e−inx − ei(n+1)x
Dn (x) = eikx = ,
1 − eix
k=−n

donc
sin((n + 1/2)x)
∀ n ∈ IN, ∀ x ∈ IR \ 2πZZ, Dn (x) = . (2)
sin(x/2)
La relation (1) s’écrit alors
Z π
1
Sn (f )(x) = Dn (t) f (x − t) dt.
2π −π

Compte tenu de la parité de Dn , l’intégrale précédente se transforme en


Z π Z π
1 1
Sn (f ) = Dn (t) f (x − t) dt + Dn (t) f (x + t) dt.
2π 0 2π 0
Z π
1 1
Puis, en utilisant le fait que Dn (t) dt = , on obtient
2π 0 2
Z π
1
Sn (f )(x) − ` = Dn (t) [f (x − t) + f (x + t) − 2`] dt.
2π 0
Convergence ponctuelle des séries de Fourier 9

Le lemme IV.1 montre que pour tout δ ∈]0, π] on a


Z π
lim Dn (t) [f (x − t) + f (x + t) − 2`] dt = 0.
n→∞ δ

Alors, on a l’équivalence suinvante :

lim Sn (f )(x) = `
n→∞

⇓ (3)
Z δ
∃ δ ∈]0, π], lim Dn (t) [f (x − t) + f (x + t) − 2`] dt = 0
n→∞ 0

1 2
Mais, l’application t 7→ − sur ]0, π] est prolongeable par continuité en 0, donc
sin(t/2) t
le lemme IV.1 montre que, pour tout δ ∈]0, π], on a
Z δ µ ¶
1 2
sin((n + 1/2)t) − [f (x − t) + f (x + t) − 2`] dt −−−→ 0.
0 sin(t/2) t n→∞

De (3) et de ce qui précède, on arrive à la conclusion

lim Sn (f )(x) = `
n→∞

⇓ (4)
Z δ
f (x − t) + f (x + t) − 2` 2n + 1
∃ δ ∈]0, π], lim sin( t) dt = 0
n→∞ 0 t 2

Théorème IV.2. (Théorème de Dirichlet) Soit f ∈ R2π , et x ∈ IR. Si les deux


applications 
 f (x− ) si t=x
fg :] − ∞, x] −→ C :fg (t) = |

f (t) si t<x

 f (x+ ) si t=x
fd : [x, +∞[−→ C :fd (t) = |

f (t) si t>x
sont dérivables en x, alors la série de Fourier S(f )(x) de f en x, converge et admet
f (x+ ) + f (x− )
pour somme .i.e.
2

f (x+ ) + f (x− )
lim Sn (f )(x) = .
n→∞ 2
10 Séries de Fourier

Preuve : Nous utilisons l’équivalence (4), avec ` = (f (x+ ) + f (x− ))/2 et δ ∈]0, π]. En
effet,
f (x + t) + f (x − t) − 2`
lim = fd0 (x) − fg0 (x).
>
t→0
t

f (x + t) + f (x − t) − 2`
donc t 7→ est prolongeable en une fonction réglée sur [0, δ], et
t
le lemme de Riemann s’applique.

Corollaire IV.3. Soit f ∈ C2π qui admet une dérivée à gauche et une dérivée à droite
en tout point. Alors la série de Fourier S(f ) de f converge simplement vers f .
Convergence au sens de Cesàro des séries de Fourier 11

V. Convergence au sens de Cesàro des séries de Fourier

n−1
1X
Lemme V.1. Pour x ∈ IR, et n ∈ IN∗ on pose Kn (x) = Dk (x), où Dn est le noyau
n
k=0
de Dirichlet.
a. Pour tout n ∈ IN∗ et tout x ∈ IR,
n µ
X ¶ µ ¶2
|k| ikx 1 sin(nx/2)
Kn (x) = 1− e = .
n n sin(x/2)
k=−n

b. Pour tout n ∈ IN∗ et tout


Z πx ∈ IR, Kn (x) ≥ 0.
1
c. Pour tout n ∈ IN∗ , Kn (x) dx = 1.
2π −π
d. Pour tout n ∈ IN∗ et tout δ ∈]0, π[,
1
∀ x ∈ [−π, π] \ [−δ, δ], Kn (x) ≤ 2 .
n sin (δ/2)

n µ
X ¶
e n (x) = | k |
Preuve : a. Posons K 1− eikx . Alors, pour m ≥ 1,
n
k=−n

m
X m
X
e m+1 (x) − mK
(m + 1)K e m (x) = ikx
(m + 1 − | k |) e − (m − | k |) eikx
k=−m k=−m
Xm
= eikx = Dm (x)
k=−m

e 0 (x) = D0 (x) = 1 alors, en prenant la somme des égalités précédentes pour m


Mais K
variant entre 1 et n − 1 nous obtenons,
n−1
X
e n (x) = D0 (x) +
nK Dm (x) = nKn (x).
m=1

ce qui démontre la première égalité.


D’autre part,

sin(n + 1/2)x cos nx − cos(n + 1)x


Dn (x) = = .
sin(x/2) 1 − cos x

La relation (n + 1)Kn+1 (x) − nKn (x) = Dn (x), démontre que, pour tout n ≥ 1,

cos(n + 1)x cos nx


(n + 1)Kn+1 (x) + = nKn (x) +
1 − cos x 1 − cos x
12 Séries de Fourier

ce qui permet d’écrire


cos nx cos x 1
nKn (x) + = K1 (x) + = .
1 − cos x 1 − cos x 1 − cos x
Enfin,
µ ¶ µ ¶2
1 1 − cos nx 1 sin(nx/2)
Kn (x) = = .
n 1 − cos x n sin(x/2)
b. Il est immédiat d’après l’égalité précédente que Kn (x) ≥ 0 pour tout x ∈ IR et
tout n ∈ IN∗ . Z π
c. En utilisant le fait que eikx dx = 0 si k 6= 0, et la première égalité de a. nous
−π
obtenons immédiatement que
Z π
∗ 1
∀ n ∈ IN , Kn (x) dx = 1.
2π −π

d. Il est immédiat que

∀ x ∈ [−π, π] \ [−δ, δ], sin2 (x/2) ≥ sin2 (δ/2)

alors, en utilisant l’égalité de a., on a


1
∀ x ∈ [−π, π] \ [−δ, δ], Kn (x) ≤ 2
n sin (δ/2)
Ce qui achève la preuve du lemme.

Remarque : La fonction Kn considérée dans le lemme précédent s’appelle Le noyau


de Fejér.

Soit f ∈ R2π . Nous avons vu que la suite des sommes partielles (Sn (f ))n∈IN∗ de la
série de Fourier de f peut ne pas converger en général. Mais qu’en est il pour la suite
n−1
1X
des moyennes de Cesàro (σn (f ))n∈IN∗ avec σn (f ) = Sk (f ) ?
n
k=0
En effet,
n−1 n−1
1X 1X
σn (f ) = Sk (f ) = f ∗ D k = f ∗ Kn .
n n
k=0 k=0
Ce qui s’écrit, en utilisant a. du lemme V.1,
n µ
X ¶
|k|
σn (f )(x) = 1− Ck (f ) eikx . (†)
n
k=−n

D’autre part, en utilisant la parité de Kn et la propriété c. du lemme V.1, on obtient


Z π
1
σn (f )(x) − ` = [f (x + t) + f (x − t) − 2`]Kn (t) dt. (‡)
2π 0
Convergence au sens de Cesàro des séries de Fourier 13

Théorème V.2. Soit f ∈ R2π , (Sn (f ))n∈IN∗ la suite des sommes partielles de la
série de Fourier de f , et (σn (f ))n∈IN∗ la suite de ses moyennes de Cesàro, i.e.
n−1
1X
σn (f ) = Sk (f ). Alors
n
k=0

f (x+ ) + f (x− )
∀ x ∈ IR, lim σn (f )(x) = .
n→∞ 2

f (x+ ) + f (x− )
Preuve : Soit x ∈ IR. En prenant ` = dans (‡) on obtient
2
Z π
1
σn (f )(x) − ` = [f (x + t) − f (x+ ) + f (x − t) − f (x− )]Kn (t) dt.
2π 0

Donc, pour tout δ ∈]0, π],


Z δ
1 ¯ ¯ ¯ ¯
| σn (f )(x) − ` | ≤ [¯ f (x + t) − f (x+ ) ¯ + ¯ f (x − t) − f (x− ) ¯]Kn (t) dt
2π 0
Z π
1
+ 4 k f k∞ Kn (t) dt,
2π δ

ou bien,

¯ ¯ ¯ ¯ 2 k f k∞
| σn (f )(x) − ` | ≤ sup [¯ f (x + t) − f (x+ ) ¯ + ¯ f (x − t) − f (x− ) ¯] + . ([)
0<t<δ n sin2 (δ/2)

Soit ε > 0, il existe δ ∈]0, π[ tel que


¯ ¯ ¯ ¯
t ∈]0, δ[ =⇒ ¯ f (x + t) − f (x+ ) ¯ + ¯ f (x − t) − f (x− ) ¯ ≤ ε.

Pour δ ainsi choisi, il existe n0 tel que

2 k f k∞ ε
n ≥ n0 =⇒ 2 ≤ .
n sin (δ/2) 2

Alors, d’après ([),


n ≥ n0 =⇒ | σn (f )(x) − ` | ≤ ε.
f (x+ ) + f (x− )
Ce qui prouve que σn (f )(x) −−−→ .
n→∞ 2

Corollaire V.3. Soit f ∈ R2π . Si la série de Fourier de f au point x ∈ IR converge,


f (x+ ) + f (x− )
alors sa somme vaut .
2
14 Séries de Fourier

Preuve : Car si ` est la somme de S(f )(x) alors la suite des sommes par-
tielles (Sn (f )(x))n∈IN converge vers ` et le lemme de Cesàro montre qu’alors la suite
(σn (f )(x))n∈IN converge aussi vers `. Le théorème précédent permet de conclure que
f (x+ ) + f (x− )
`= .
2
Corollaire V.4. Soit f ∈ C2π . Si la série de Fourier de f au point x ∈ IR converge,
alors sa somme vaut f (x).

Preuve : C’est immédiat.

Corollaire V.5. Soit f ∈ C2π . Si ∀ n ∈ ZZ, Cn (f ) = 0, alors f = 0.

Preuve : En effet, pour tout n ∈ IN∗ ,


Xn µ ¶
|k|
σn (f ) = 1− Ck (f ) ek = 0,
n
k=−n

et, d’après le théorème V.2, la suite (σn (f ))n∈IN∗ converge simplement vers f donc
f = 0.

Remarque : Ce corollaire exprime l’injectivité de l’application linéaire qui à une


fonction continue 2π-périodique associe son spectre, i.e. la famille de ses coefficients de
Fourier. Cette remarque est souvent utilisée sous la forme suivante : “Si deux fonctions
continues et 2π-périodiques ont le même spectre alors elles coı̈ncident”.
Plus généralement, la même preuve montre que “Si deux fonctions de R2π ont le
même spectre alors elles coı̈ncident en tout point ou elles sont continues”.

Théorème V.6. Soit f ∈ C2π , (Sn (f ))n∈IN∗ la suite des sommes partielles de la série
de Fourier de f , et (σn (f ))n∈IN∗ la suite de ses moyennes de Cesàro. Alors la suite
(σn (f ))n∈IN∗ converge uniformément vers f .

Preuve : En utilisant le lemme V.1.c et le fait que σn (f ) = f ∗ Kn on obtient,


Z π
∗ 1
∀ n ∈ IN , ∀ x ∈ IR, σn (f )(x) − f (x) = [f (x − t) − f (x)]Kn (t) dt.
2π −π

Donc, pour x ∈ IR, n ∈ IN∗ et δ ∈]0, π[,


Z δ
1
| σn (f )(x) − f (x) | ≤ | f (x − t) − f (x) |Kn (t) dt
2π −δ
Z
1
+ | f (x − t) − f (x) | Kn (t) dt
2π δ≤| t |≤π
Convergence en moyenne quadratique des séries de Fourier 15

d’où,

| σn (f )(x) − f (x) | ≤ sup | f (x − t) − f (x) | + 2 k f k∞ sup Kn (t),


| t |≤δ δ≤| t |≤π

2 k f k∞
| σn (f )(x) − f (x) | ≤ sup | f (x − t) − f (x) | + .
| t |≤δ n sin2 (δ/2)
Soit ε > 0, la continuité uniforme de f sur IR montre qu’il existe δ > 0 tel que

ε
∀ x ∈ IR, ∀ t ∈] − δ, δ[, | f (x − t) − f (x) | ≤ ,
2

et pour δ ainsi choisi, il existe n0 tel que

2 k f k∞ ε
n ≥ n0 =⇒ 2 ≤ .
n sin (δ/2) 2

Alors,
∀ n ≥ n0 , ∀ x ∈ IR, | σn (f )(x) − f (x) | ≤ ε

Ce qui démontre la convergence uniforme de (σn (f ))n∈IN∗ vers f .

Remarque : Le théorème précédent exprime la densité de l’ensemble des polynômes


trigonométriques P dans l’espace des fonctions continues 2π-périodiques muni de la
norme uniforme.

VI. Convergence en moyenne quadratique des séries de Fourier

Proposition VI.1. Soient (f, g) ∈ R2π × R2π . Alors


X
∀ x ∈ IR, f ∗ g(x) = Cn (f )Cn (g) einx .
n∈ZZ

Preuve : Posons, pour n ∈ ZZ, λn = Cn (f )Cn (g) = Cn (f ∗ g). Nous avons démontré
X X
dans la proposition III.5, que | λn | est convergente. Donc la série λn en converge
Xn∈ZZ n∈ZZ
normalement. Notons h = λn en . Les deux applications h et f ∗ g sont des éléments
n∈ZZ
de C2π qui ont le même spectre, alors elles coı̈ncident. Ce qui démontre le résultat.
16 Séries de Fourier

Corollaire VI.2. (Égalité de Bessel-Parseval) Soient (f, g) ∈ R2π × R2π . Alors


Z X
1
f (t) g(t) dt = Cn (f )Cn (g).
2π TT n∈ZZ

et Z
1 2
X 2
| f (t) | dt = | Cn (f ) | .
2π TT n∈ZZ

Preuve : La deuxième égalité s’obtient de la première en prenant g = f .


Démontrons la première. Soit (f, g) ∈ R2π × R2π Z . On pose, pour tout t ∈ IR,
1
h(t) = f (−t), alors Cn (h) = Cn (f ), et h ∗ g(0) = f (t)g(t) dt. Mais, d’après la
2π TT
proposition VI.1,
X
h ∗ g(0) = Cn (h)Cn (g).
n∈ZZ

Ce qui est l’égalité demandée.

Corollaire VI.2. Soit f ∈ R2π et (Sn (f ))n∈IN la suite des sommes partielle de la
série de Fourier de f . Alors
Z
1 2
| f (t) − Sn (f )(t) | dt −−−→ 0.
2π TT n→∞

On dit que la série de Fourier de f converge vers f en moyenne quadratique.

Preuve : En effet,
Z n
X
2 2 2 1 2 2
kf − Sn (f ) k2 = kf k2 − k Sn (f ) k2 = | f (t) | dt − | Ck (f ) | .
2π TT k=−n

2
Donc, d’après le corollaire précédent, lim k f − Sn (f ) k2 = 0.
n→∞
Applications 17

VII. Applications

♣ L’inégalité isopérimétrique
Théorème VII.1. Soit γ une courbe plane, simple, fermé, de classe C 1 , de
longueure ` et entourant une surface d’aire S. Alors 4πS ≤ `2 avec égalité si,
et seulement si, γ est un cercle.
Preuve : Supposons ` = 2π. Nous identifions le plan avec le corps des nombres
complexes C.
|

Soit f : [0, 2π] −→ C


| la paramétrisation normale de γ. On peut prolonger f

en une fonction 2π-périodique de classe C 1 sur IR. On pose x(t) = Re(f (t)) et
y(t) = Im(f (t)).
La fonction f admet un développement en série de Fourier
X
f (t) = x(t) + iy(t) = cn eint .
n∈ZZ

La paramétrisation f de γ est normale, alors x02 (t)+y 02 (t) = 1 pour tout t ∈ IR.
La longueur de γ est
Z 2π Z 2π
£ 02 ¤ 2
2π = x (t) + y 02 (t) dt = | f 0 (t) | dt.
0 0

X 2
Mais Cn (f 0 ) = incn , donc 2π = 2π n2 | cn | , soit
n∈ZZ

X 2
1= n2 | cn | . (1)
n∈ZZ

D’autre part, nous avons


Z 2π µZ 2π ¶
1 0 1 0 0
S= [x(t)y (t) − y(t)x (t)] dt = Im f (t)f (t) dt
2 0 2 0
à !
X
0
=π Im(hf, f i) = π Im cn incn
n∈ZZ
X 2
=π n | cn | .
n∈ZZ

Ce qui donne
X 2
S=π n | cn | . (2)
n∈ZZ
18 Séries de Fourier

De (1) et (2) nous obtenons,


X 2
π−S =π (n2 − n) | cn | ≥ 0
n∈ZZ

avec égalité si, et seulement si, cn = 0 pour tout n ∈ ZZ \ {0, 1}, c’est à dire
si, et seulement si, ∀ t ∈ IR, f (t) = c0 + c1 eit qui est la paramétrisation d’un
cercle. Nous avons démontré le résultat lorsque ` = 2π, le cas général s’obtient
par homothétie.
♣ L’inégalité de Writinger
Théorème VII.2. Soit [a, b] un intervalle compact non réduit à un point, et
E = C01 ([a, b]) l’espace vectoriel des fonctions continuement dérivables sur [a, b],
s’annulant en a et en b. Alors
Z b Z b
2 (b − a)2 2
| f (t) | dt ≤ | f 0 (t) | dt.
a π2 a

La constante (b − a)2 π −2 ne peut pas être améliorée.


Preuve : Supposons d’abord que a = 0 et b = π. On définit fe ∈ C2π
1
par

 f (t) si t ∈ [0, π]
∀ t ∈ [−π, π], fe(t) =

−f (−t) si t ∈ [−π, 0]
X
Si la série de Fourier de fe est S(fe) = Cn en alors, la série de Fourier de fe0
n∈ZZ
X
est S(fe0 ) = inCn en . En remarquant que C0 = 0 car fe est impaire, et en
n∈ZZ
utilisant deux fois l’égalité de Bessel-Parseval, on obtient
Z π ¯ ¯ X X Z π ¯ ¯
1 ¯ e ¯2 2 2 12 ¯ e0 ¯2
¯ f (t) ¯ dt = | Cn | ≤ n | Cn | = ¯ f (t) ¯ dt
2π −π 2π −π
n∈ZZ\{0} n∈ZZ\{0}

¯ ¯2 ¯ ¯2
¯ ¯ ¯ ¯
Enfin, en utilisant la parité des applications t 7→ ¯ fe(t) ¯ et t 7→ ¯ fe0 (t) ¯ , on
trouve Z π Z π
2 2
| f (t) | dt ≤ | f 0 (t) | dt.
0 0

Avec égalité si, et seulement si, Cn = 0 pour tout n ∈ ZZ \ {1, −1}. C’est à dire
si, et seulement si, il existe λ ∈ IK tel que ∀ t ∈ [0, π], f (t) = λ sin t.
Venons au cas général, et considérons f ∈ E. On définit

b−a
g : [0, π] −→ IK, g(x) = f (a + x).
π
Applications 19

Alors g est de classe C 1 sur [0, π] et s’annule en 0 et π. Donc d’après le cas


particulier Z Z
π π
2 2
| g(t) | dt ≤ | g 0 (t) | dt.
0 0

Ce qui est équivalent à


Z b Z b
(b − a)2
2 2
| f (t) | dt ≤ | f 0 (t) | dt.
a π2 a

Avec égalité si, et seulement si, la fonction g est proportionnelle à la fonction


sinus sur [0, π]. Ce qui est q́uivalent à l’existence d’un λ ∈ IK tel que
µ ¶
t−a
∀ t ∈ [0, π], f (t) = λ sin π .
b−a

♣ Polynômes de Bernoulli
Il est immédiat de voir que les trois conditions suivantes définissent par
récurrence une, et une seule, suite de fonctions polynomiales (Bn )n∈IN .

¦∀ x ∈ IR, B0 (x) = 1. (1)

¦∀ n ∈ IN∗ , ∀ x ∈ IR, Bn0 (x) = nBn−1 . (2)


Z 2π

¦∀ n ∈ IN , Bn (x) dx = 0. (3)
0

En effet, supposons connue Bn−1 . La deuxième condition montre l’existence


d’une constante λn telle que
Z x
Bn (x) = λn + n Bn−1 (t) dt.
0

Et la troisième condition permet de déterminer λn comme suit


Z 2π µZ x ¶
0 =2πλn + n Bn−1 (t) dt dx
0 0
· Z x ¸2π Z 2π
=2πλn + n (x − 2π) Bn−1 (t) dt −n (x − 2π)Bn−1 (x) dx.
0 0 0

donc Z 2π
x
λn = n ( − 1)Bn−1 (x) dx.
0 2π
Finalement,
Z 2π Z x
t
Bn (x) = n ( − 1)Bn−1 (t) dt + n Bn−1 (t) dt. (4)
0 2π 0
20 Séries de Fourier

Par exemple, on a

B1 (x) =x − π.
2π 2
B2 (x) =x2 − 2πx + . (5)
3
B3 (x) =x3 − 3πx2 + 2π 2 x.

Remarquons que, pour n ≥ 1,


Z 2π Z 2π
0
Bn+1 (2π) − Bn+1 (0) = Bn+1 (t) dt = (n + 1) Bn (t) dt = 0.
0 0

Donc
∀ n ≥ 2, Bn (0) = Bn (2π). (6)

en qui est de
Pour tout n ≥ 1 nous considérons la fonction 2π-périodique B
classe C 1 par morceaux et qui coı̈ncide avec Bn sur [0, 2π[.
e1 . En effet, C0 (B
Déterminons la série de Fourier de B e1 ) = 0 et pour k 6= 0
nous avons
Z 2π
1
Ck (f ) = (x − π)e−ikx dx
2π 0
· ¸2π Z 2π
(π − x)e−ikx 1 i
= + e−ikx dx = .
2πik 0 2πik 0 k

Le thórème de Dirichlet montre alors

X ∞
X sin kx
ieikx
∀ x ∈]0, 2π[, x−π = = −2 . (7)
k k
k∈ZZ\{0} k=1

en pour n ≥ 2. En
Déterminons plus généralement la série de Fourier de B
en ) = 0 et pour k 6= 0,
effet, d’après (3) nous avons C0 (B
Z 2π
e 1
Ck (Bn ) = Bn (x)e−ikx dx
2π 0
· ¸2π Z 2π
Bn (x)e−ikx n n en−1 ).
= − + Bn−1 (x)e−ikx dx = Ck (B
2πik 0 2πik 0 ik

Donc une récurrence immédiate montre

∀ n ≥ 1, ∀ k ∈ ZZ \ {0}, en ) = − n! .
Ck (B (8)
(ik)n
Applications 21

en et la convergence normale de la série de Fourier de B


La continuité de B en ,
pour n ≥ 2, montrent que

X eikx
∀ x ∈ [0, 2π], Bn (x) = −n! .
(ik)n
k∈ZZ\{0}

Donc, pour tout n ≥ 1 et tout x ∈ [0, 2π],


X cos kx (−1)n+1
= B2n (x) (9)
k 2n 2(2n)!
k=1
X∞
sin kx (−1)n+1
= B2n+1 (x) (10)
k 2n+1 2(2n + 1)!
k=1

Bn (0) 1
Posons bn = n
pour tout n ∈ IN. Nous avons b0 = 1, b1 = − . La
(2π) 2
relation (2) montre que, pour tout 0 ≤ k ≤ n, on a

n!
Bn(k) (0) = (2π)n−k bn−k . (11)
(n − k)!

Alors, La formule de Taylor pour les polynômes et le fait que le degré de Bn+1
est n + 1 montrent que

n+1
X (k)
Bn+1 (0) k
Bn+1 (t) = t .
k!
k=0

En utilisant, pour n ≥ 0, le fait que Bn+1 (0) = Bn+1 (2π) et (11) on obtient

n+1
X
n k
(2π) bn = Bn+1 (2π) = Cn+1 bn+1−k (2π)n .
k=0

Ce qui s’écrit, pour tout n ≥ 1,

n
X
k
0= Cn+1 bk . (12)
k=0

Ou bien,
n−1
1 X k
∀ n ≥ 1, bn = − Cn+1 bk . (13)
n+1
k=0

C’est une relation de récurrence qui, avec b0 = 1, permet de déterminer la


suite de nombres rationnels (bn )n∈IN qui s’appelle la suite des nombres
22 Séries de Fourier

de Bernoulli. Il est evident d’après la formule de Taylor et la définition des


nombres de Bernoulli que
n
X
∀ n ∈ IN, Bn (x) = Cnk bn−k (2π)n−k xk . (14)
k=0

Enfin, notons que la relation (10) montre que b2n+1 = 0 pour tout n ≥ 1, et la
relation (9) montre que

X 1 b2n
∀ n ≥ 1, 2n
= (−1)n+1 (2π)2n
k 2(2n)!
k=1

Voici quelques exemples numériques :

n 2 4 6 8 10
1 1 1 1 5
bn − −
6 30 42 30 66

X 1 π2 π4 π6 π8 π 10
kn 6 90 945 9450 93555
k=1
Exercices 23

EXERCICES

Exercice .1 Calculer la série de Fourier de la fonction paire, 2π-périodique f définie


sur [0, π] par : 
1
 si 0 ≤ x ≤ π/2
f (x) =
 2(π − x)

si π/2 ≤ x ≤ π.
π
En déduire la série de Fourier de la fonction paire, 2π-périodique g définie sur [0, π] par

 1 − 2x si 0 ≤ x ≤ π/2
g(x) = π

0 si π/2 ≤ x ≤ π.

Exercice .2 Calculer la série de Fourier de la fonction 2π-périodique f définie sur


[−π, π] par f (x) = | x |. En déduire

X∞
sin2 (2n + 1) x π2 2 π 3 π
= x − x pour x ∈ [0, ].
n=0
(2n + 1)4 8 6 2

Exercice .3 Calculer la série de Fourier de la fonction paire, 2-périodique f définie


sur [0, 1] par : f (x) = −2x + 1. En déduire la série de Fourier de la fonction impaire,
2-périodique g définie sur [0, 1] par : g(x) = −x2 + x.

Exercice .4 Calculer les séries de Fourier des fonctions 2π-périodiques, f et g


définies sur ] − π, π] par : f (x) = exp ax, g(x) = ch ax. En déduire :

X∞ ∞
X ∞
X
(−1)n a a2
.
n=0
n2 + 1 n=0
n + a2
2
n=0
(n2 + a2 )2

Exercice .5 Calculer la série de Fourier de la fonction f définie par :


3
∀x ∈ IR, f (x) = | sin x | .
En déduire :


256 4608 X 1
π2 = + .
45 5 n=1 (4n − 9) (4n2 − 1)2
2 2
24 Séries de Fourier
2m+1
Exercice .6 Pour m ∈ IN, on note fm (x) = | sin x | .
◦ ∗
1 . Montrer que ∀ n ∈
Z IN , bn (fm ) = 0 et a2n−1 (fm ) = 0.
π
(m)
2◦ . On note An = sin2m+1 x cos(2nx) dx. Montrer que
0

2
∀ n ∈ IN, A(0)
n = , et que
1 − 4n2
(2m + 1)2m
A(m)
n = A(m−1) , pour m ≥ 1.
(2m + 1) − 4n n
2 2

3◦ . Écrire la série de Fourier de fm . En déduire une expression de π comme somme


d’une série numérique.

Exercice .7 Soit la fonction 2π-périodique, f définie sur [0, 2π] par :

3(x − π)2 − π 2
f (x) = .
12
1◦ . Représenter graphiquement f et développer f en série de Fourier.
X∞
1
2◦ . En déduire la valeur de : 4
.
n=1
n
R∞
3◦ . Soient a > 0 et F (a) = 0 e−ax f (x)dx. Montrer que :
Z 2π
1
F (a) = e−ax f (x)dx.
1 − e−2aπ 0

et calculer F (a).

X
◦ 1
4 . Montrer que : F (a) = a .
n=1
n2 (n2 + a2 )

Exercice .8
1◦ . Soit la fonction 2π-périodique, g définie sur [−π, π] par : g(x) = a2 x2 /2.
a. Calculer la série de Fourier de g.
b. Montrer qu’elle converge uniformément sur IR vers g ; calculer :

X∞ X∞ X∞
1 (−1)n 1
2
, 2
, 4
.
n=1
n n=1
n n=1
n

X∞
◦ (−1)n cos nx
2
2 . On pose, pour a ∈ IR+ , x ∈ IR : f (x) = 2a .
n=1
(n2 + a2 )
Montrer que f est définie, continue, 2π-périodique sur IR.
3◦ .
X∞
a2 π 2 4 (−1)n cos nx
a. Montrer que : g(x) − f (x) = + 2a 2 (n2 + a2 )
.
6 n=1
n
Exercices 25

b. En déduire que g − f est de classe C 2 sur IR et calculer (g − f )00 .


c. Montrer que sur ] − π, π[, f est solution d’une équation différentielle linéaire
du second ordre que l’on précisera.
4◦ .
a. Montrer que sur ] − π, π[, f (x) = A ch ax + B sh ax + C. Calculer A, B et C.
X∞ X∞
1 (−1)n
b. En déduire : , .
n=1
1 + n2 n=1
1 + n2

Exercice .9 Calculer la série de Fourier de la fonction f : ∀x ∈ IR f (x) =


sup (sin x, 0). La série de Fourier converge-t-elle ? Quelle est sa somme ? En déduire

X∞
(−1)n
2−1
.
n=1
4n

Exercice .10 Soit Θ la fonction 2π-périodique définie par


 π π

x si x ∈ [− , ]
2 2
Θ(x) =

π − x π 3π
si x ∈ [ , ]
2 2
1◦ . Montrer que pour tout x de IR on a

2 X (−1)k 4 X 1
Θ(x) = ei(2k+1)x , et que = 1.
iπ (2k + 1)2 π 2 (2k + 1)2
k∈ZZ k∈ZZ

2◦ . En déduire que, pour n ∈ IN∗ ,

4n X (−1)k i 2k+1
∀ x ∈ [−n, n], x= e 2n πx .
iπ 2 (2k + 1)2
k∈ZZ

P
n
3◦ . Soit Pn (t) = cr eirt un polynôme trigonométrique de degré n. Montrer que
−n

µ ¶
4n X (−1)k 2k + 1
Pn0 (t) = 2 Pn t + π .
π (2k + 1)2 2n
k∈ZZ

4◦ . (Inégalité de Bernstein ) En déduire que, pour tout polynôme trigonométrique P


de degré n, l’on a k P 0 k∞ ≤ n k P k∞ . (Où k Q k∞ = sup {| Q(t) | : t ∈ IR} pour un
polynôme trigonométrique Q). Est- ce que l’on peut remplacer n dans l’inégalité
précédente par une constante plus petite ?
26 Séries de Fourier

Exercice .11
1◦ . On fixe dans cette question, r ∈ [0, 1[.
X∞
r sin t
a. Montrer ∀ t, rn sin(nt) = 2 − 2r cos t
, la série étant convergente nor-
n=1
1 + r
malement.
X∞
rn 1
b. En déduire que, ∀ x, − cos(nx) = Log (1 + r2 − 2r cos x).
n 2
n=1 Z π
2
c. Calculer, pour tout n ≥ 0, l’intégrale Log (1 + r2 − 2r cos x) cos(nx) dx,
π 0
et en déduire, pour tout n ≥ 1, la valeur de
Z π
2
sin xLog (1 + r2 − 2r cos x) sin(nx) dx (∗)
π 0

2◦ . On pose 
 sin xLog (1 − cos x) si x 6∈ 2πZZ
ϕ(x) =

0 si x ∈ 2πZZ
a. Montrer que ϕ est une fonction continue, 2π-périodique et impaire. Est-ce que
ϕ est dérivable en 0 ?
b. En faisant tendre r vers 1 dans (∗), calculer, pour tout n ≥ 1
Z π
2
ϕ(x) sin(nx) dx
π 0

c. Déduire de ce qui précède le développement en série de Fourier de ϕ. Est-ce


que la série de Fourier de ϕ converge vers ϕ ?

Exercice .12 Pour 0 < a ≤ π/2, on désigne par fa la fonction 2π-périodique définie
par  µ ¶

 π | x |
1− si x ∈ [−2a, 2a]
fa (x) = a 2a


0 si x ∈ [−π, −2a[∪]2a, π]
1◦ .
a. Ecrire la série de Fourier de fa . Cette série converge-t-elle vers fa ?
X∞ µ ¶2
n sin(na)
b. Calculer la valeur de la somme (−1) .
n=1
na
X∞
sin2 (na) sin2 (nb)
c. Calculer, pour 0 < a ≤ b ≤ π/2, la valeur de la somme .
n=1
n4
Quelle est la valeur de la somme précédente si 0 < b ≤ a ≤ π/2.
2◦ . Pour λ > 0, on désigne par gλ la fonction 2π-périodique définie par ∀ x ∈
[−π, π[, gλ (x) = ch (λx).
Exercices 27

Calculer les coefficients de Fourier exponentiels Cn (gλ ) (n ∈ ZZ). En déduire la série


de Fourier de gλ . Cette série converge-t-elle vers gλ ?
3◦ . Z π
1
a. En calculant fa (x)gλ (x) dx de deux façons différentes, montrer
2π −π

X∞
(−1)n sin2 (an) π sh 2 (λa) − a2 λ sh (λπ)
∀ a ∈]0, π/2], ∀λ ∈ IR∗+ , =
n=1
n2 (λ2 + n2 ) 2λ3 sh (λπ)

X∞
(−1)n sin2 (an)
b. En déduire la valeur de la somme 4
.
n=1
n
Xn
sin(kx)
Exercice .13 On pose, pour n ∈ IN∗ , Sn (x) = .
k
k=1
1◦ . On pose f (x) = (π − x)/2 pour x ∈ [0, 2π[. Montrer que

∀ x ∈]0, 2π[, lim Sn (x) = f (x).


n→∞

sin(n + 1/2)x
2◦ . On pose pour x ∈ IR \ 2πZZ, Dn (x) = . Montrer
sin(x/2)
Z x
1 x
∀ x ∈]0, 2π[, Sn (x) = Dn (t) dt − .
2 0 2

3◦ . On définit la fonction g sur [0, π] par


 1 1
 − si x ∈]0, π]
g(x) = 2 sin(x/2) x

0 si x=0

a. Montrer que g est continue sur [0, π].


b. Montrer que, pour tout x ∈ [0, π],
Z x Z x
2n + 1 sin(n + 1/2)t x
Sn (x) = g(t) sin( t) dt + dt − .
0 2 0 t 2
x Z
2n + 1
c. Montrer que lim g(t) sin( t) dt = 0.
Zn→∞ 0 2

sin t π
d. Déduire que dt = .
Z π 0 t 2
sin t ³π ´

4 . Montrer que dt = lim Sn .
0 t ³ πn→∞
´ n
5◦ . En déduire que lim Sn > lim f (x). (Phénomène de Gibbs).
n→∞ n >
x→0
28 Séries de Fourier

SOLUTIONS

Solution .1 En effet, pour n 6= 0,


Z π
1
Cn (f ) = f (x)e−inx dx
2π −π
Zπ Z 0
1 −inx 1
= f (x)e dx + f (x)e−inx dx
2π 0 2π −π
Z π Z π
1 −inx 1
= f (x)e dx + f (−x)einx dx
2π 0 2π 0
Z
1 π
= f (x) cos nx dx car f est paire
π 0
Z Z
1 π/2 1 π 2(π − x)
= cos nx dx + cos nx dx
π 0 π π/2 π
· ¸π/2 · ¸π Z π
sin nx 2(π − x) sin nx 2
= + + 2 sin nx dx
nπ 0 π2 n π/2 π n π/2
· ¸π
sin(nπ/2) sin(nπ/2) 2 cos nx
= − −
nπ nπ n2 π 2 π/2
2(cos(nπ/2) − cos nπ)
=
π 2 n2

D’autre part, un calcul immédiat montre que C0 (f ) = 3/4.


En notant que Cn (f ) = C−n (f ) nous avons

3 X
S(f )(x) = + 2Cn (f ) cos nx
4 n=1

ou bien,
∞ ∞
3 4 X cos(2n + 1)x 2 X cos(4n + 2)x
S(f )(x) = + 2 − 2 .
4 π n=0 (2n + 1)2 π n=0 (2n + 1)2

Notons que la série de Fourier de f est normalement convergente, la continuité de f


montre alors que
∞ ∞
3 4 X cos(2n + 1)x 2 X cos(4n + 2)x
∀ x ∈ IR, f (x) = + 2 − 2 .
4 π n=0 (2n + 1)2 π n=0 (2n + 1)2


X 1 π2
En particulier en prenant la valeur en x = 0, nous trouvons 2
= .
n=0
(2n + 1) 8
Solutions 29

Remarquons d’autre part que g(x) = 1 − f (x − π) pour tout x ∈ IR. Alors


∞ ∞
1 4 X cos(2n + 1)x 2 X cos(4n + 2)x
∀ x ∈ IR, g(x) = + 2 + 2 .
4 π n=0 (2n + 1)2 π n=0 (2n + 1)2

et la convergence normale de cette série montre que


∞ ∞
1 4 X cos(2n + 1)x 2 X cos(4n + 2)x
∀ x ∈ IR, S(g)(x) = + 2 + 2 .
4 π n=0 (2n + 1)2 π n=0 (2n + 1)2

Solution .2 En effet, pour n 6= 0,


Z π
1
Cn (f ) = | x | e−inx dx
2π −π
Z π Z 0
1 −inx 1
= xe dx − x e−inx dx
2π 0 2π −π
Z π Z π
1 −inx 1
= xe dx + x einx dx
2π 0 2π
Z π · 0 ¸π Z π
1 x sin nx 1
= x cos nx dx = − sin nx dx
π 0 nπ 0 nπ 0
h cos nx iπ (−1)n − 1
= = .
n2 π 0 n2 π

D’autre part, un calcul simple montre que C0 (f ) = π/2.


En notant que Cn (f ) = C−n (f ) nous avons

π X
S(f )(x) = + 2Cn (f ) cos nx
2 n=1

ou bien,

π 4 X cos(2n + 1)x
S(f )(x) = − .
2 π n=0 (2n + 1)2

Notons que la série de Fourier de f est normalement convergente. La continuité de f


montre alors, que la série de Fourier de f converge uniformément vers f , en particulier

π 4 X cos(2n + 1)x
∀ x ∈ [0, π], x= − . (1)
2 π n=0 (2n + 1)2

La convergence uniforme sur [0, π] nous permet d’intégrer terme à terme entre 0 et x :

πx x2 4 X sin(2n + 1)x
∀ x ∈ [0, π], − = . (2)
2 2 π n=0 (2n + 1)3
30 Séries de Fourier

La série précédente est aussi uniformément convergente sur [0, π]. Alors en intégrant
terme à terme entre 0 et x :

πx2 x3 4 X 1 − cos(2n + 1)x
∀ x ∈ [0, π], − = . (3)
4 6 π n=0 (2n + 1)4

En substituant x par 2x nous obtenons

X∞
π π 2 x2 πx3 sin2 (2n + 1)x
∀ x ∈ [0, ], − = . (4)
2 8 6 n=0
(2n + 1)4


X 1 π4
En particulier si x = π/2 nous trouvons = . Puis en remplaçant dans
n=0
(2n + 1)4 96
(3)

X
πx3 π 2 x2 π4 cos(2n + 1)x
∀ x ∈ [0, π], − + = .
24 16 96 n=0 (2n + 1)4

Solution .3 Soit fe la fonction 2π-périodique définie par fe(x) = f (x/π). La fonction


fe est paire et coı̈ncide avec x 7→ 1 − 2x/π sur [0, π].
Pour n 6= 0, Z π
1
Cn (fe) = fe(x)e−inx dx
2π −π
Z π Z 0
1 1
= fe(x)e −inx
dx + fe(x)e−inx dx
2π 0 2π −π
Z π Z π
1 e 1
= f (x)e −inx
dx + fe(−x)einx dx
2π 0 2π 0
Z
1 π e
= f (x) cos nx dx car f est paire
π 0
Z µ ¶
1 π 2x
= 1− cos nx dx
π 0 π
·µ ¶ ¸π Z π
2x sin nx 2
= 1− + sin nx dx
π nπ 0 π 2 n 0
· ¸π
2 cos nx 2(1 − (−1)n )
= − 2 2 =
n π 0 π 2 n2

D’autre part, un calcul immédiat montre que C0 (fe) = 0.


En notant que Cn (fe) = C−n (fe) nous avons


X ∞
8 X cos(2n + 1)x
S(fe)(x) = e
2Cn (f ) cos nx = 2
n=1
π n=0 (2n + 1)2
Solutions 31

Mais S(fe)(x) = S(f )(πx), alors



8 X cos(2n + 1)πx
S(f )(x) = .
π 2 n=0 (2n + 1)2

La série S(f ) est normalement convergente, alors la continuité de f montre que



8 X cos(2n + 1)πx
∀ x ∈ IR, f (x) = 2 .
π n=0 (2n + 1)2

La convergence normale de la série sur [0, 1] permet d’intégrer terme à terme, d’où,

8 X sin(2n + 1)πx
∀ x ∈ [0, 1], x − x2 = .
π 3 n=0 (2n + 1)3


8 X sin(2n + 1)πx
La fonction x 7→ 3 est une fonction impaire 2-périodique qui
π n=0 (2n + 1)3
coı̈ncide avec x 7→ x − x2 sur [0, 1], donc c’est la fonction g. D’où

8 X sin(2n + 1)πx
∀ x ∈ IR, g(x) = 3 ,
π n=0 (2n + 1)3

et c’est le développement en série de Fourier de g car la série trigonométrique précédente


converge normalement.

Solution .4 Nous supposons que a ∈ C


| \ iZ
Z, et nous écrivons fa pour désigner la
fonction 2π-périodique qui vérifie fa (x) = eax sur ] − π, π]. Remarquons que
Z π · ¸π
1 ax −inx e(a−in)x sh aπ (−1)n
Cn (fa ) = e e dx = =
2π −π 2π(a − in) −π π a − in
n
sh aπ (−1) (a + in)
= .
π a 2 + n2

Alors, la série de Fourier de fa est


∞ µ ¶
sh πa 2 sh πa X (−1)n a (−1)n n
S(fa )(x) = + cos nx − 2 sin nx .
πa π n=1
n2 + a2 n + a2

D’après le théorème de Dirichlet, la série S(fa ) converge pour tout x et en particulier


nous avons
∞ µ ¶
ax sh πa 2 sh πa X (−1)n a (−1)n n
∀ x ∈] − π, π[, e = + 2 + a2
cos nx − 2 2
sin nx .
πa π n=1
n n + a
32 Séries de Fourier

D’autre part, la fonction 2π-périodique ga qui coı̈ncide avec x 7→ ch ax sur ] − π, π]


fa + f−a
vérifie ga = . Alors
2
sh πa (−1)n a
Cn (ga ) =
π n2 + a 2
et la série de Fourier de ga est

sh πa 2 sh πa X (−1)n a
S(ga )(x) = + 2 + a2
cos nx.
πa π n=1
n
La série de Fourier de ga est normalement convergente et ga est continue sur IR, alors

sh πa sh πa X (−1)n 2a
∀ x ∈ IR, ga (x) = + cos nx.
πa π n=1 n2 + a2
En particulier,
à ∞
!
sh πa X 2a2
n
∀ x ∈ [−π, π], ch ax = −1 + (−1) 2 cos nx . (1)
πa n=0
n + a2
En remplaçant x par 0 nous obtenons

X
π 1 (−1)n a
+ = .
2 sh πa 2a n=0 n2 + a2
En particulier si a = 1

π 1 X (−1)n
+ = .
2 sh π 2 n=0 n2 + 1
D’autre part, en remplaçant x par π dans (1)
X∞
π 1 a
+ = .
2 th πa 2a n=0 n2 + a2
D’autre part,
sh 2 πa a2
Cn (ga ∗ ga ) = (Cn (ga ))2 = .
π 2 (n2 + a2 )2
La série de Fourier de ga ∗ ga est normalement convergente et de somme ga ∗ ga . Alors
à ∞
!
sh 2 πa X a4
∀ x ∈ IR, ga ∗ ga (x) = 1+2 cos nx .
π 2 a2 n=1
(a2 + n2 )2
On en déduit en prenant x = 0,
à ∞
!
sh 2 πa X a4
ga ∗ ga (0) = −1 + 2 .
π 2 a2 n=0
(a2 + n2 )2
Z π
1 1 sh 2πa
Mais ga ∗ ga (0) = ch 2 ax dx = + , ce qui donne
2π −π 2 4πa
X∞
π2 π 1 a2
+ + = .
4 sh 2 πa 4a th πa 2a2 n=0
(n 2 + a2 )2
Solutions 33

3
Solution .5 La fonction f (x) = | sin x | est paire donc sa série de Fourier est une
série de cosinus. Donc

X
S(f )(x) = C0 (f ) + an (f ) cos nx.
n=1

Mais la fonction f est π-périodique donc a2n−1 (f ) = 0 pour tout n. Il en résulte que

X
S(f )(x) = C0 (f ) + a2n (f ) cos 2nx.
n=1

D’autre part, si n ∈ IN ,
Z Z
1 π 3 2 π 3
a2n (f ) = | sin x | cos 2nx dx = sin x cos 2nx dx
π −π π 0
Z
2 π 3 sin x − sin 3x
= cos 2nx dx
π 0 4
Z π Z π
3 1
= sin x cos 2nx dx − sin 3x cos 2nx dx
2π 0 2π 0
Z π
3
= (sin(2n + 1)x − sin(2n − 1)x) dx−
4π 0
Z π
1
(sin(2n + 3)x − sin(2n − 3)x) dx
4π 0
µ ¶ µ ¶
3 1 1 1 1 1
= − − −
2π 2n + 1 2n − 1 2π 2n + 3 2n − 3
24
=
π(4n2 − 9)(4n2 − 1)
et Z π Z π
1 3 1 4
C0 (f ) = | sin x | dx = sin3 x dx = .
2π −π π 0 3π
1
La fonction f est de classe C par morceaux donc sa série de Fourier S(f ) converge
simplement vers f . Alors,

3 4 24 X cos 2nx
∀ x ∈ IR, | sin x | = + .
3π π n=1 (4n − 1)(4n2 − 9)
2

La formule de Parseval montre que


Z π ∞
1 6 16 288 X 1
sin x dx = 2
+ 2
2π −π 9π π n=1 (4n − 1) (4n2 − 9)2
2 2

Z π
1 5
Mais sin6 x dx = comme le montre un calcul simple. Alors
2π −π 16

256 4609 X
2 1
π = + .
45 5 n=1 (4n2 − 1)2 (4n2 − 9)2
34 Séries de Fourier

Solution .6 C’est une généralisation de l’exercice précédent.


1◦ . La fonction fm est paire, la série de Fourier de fm est une série en cosinus, et
bn (fm ) = 0 pour tout n. D’autre part fm est une fonction π-périodique donc pour tout
n, C2n−1 (fm ) = 0. Il en résulte que a2n−1 (fm ) = 0 pour tout n ≥ 1.
2◦ . C’est un calcul assez subtil. En effet,
Z π Z
(0) 1 π
An = sin x cos 2nx dx = (sin(2n + 1)x − sin(2n − 1)x) dx
0 2 0
· ¸π
1 cos(2n + 1)x cos(2n − 1)x
= − +
2 2n + 1 2n − 1 0
1 1 2
= − =
2n + 1 2n − 1 1 − 4n2
et si m ≥ 1,
Z π
A(m−1)
n − A(m)
n = sin2m−1 x cos2 x cos 2nx dx
0
· ¸π Z π
sin2m x 1
= cos x cos 2nx + sin2m x(cos x cos 2nx)0 dx
2m 0 2m 0
Z π
1
= sin2m x(2n cos x sin 2nx + sin x cos 2nx) dx
2m 0
Z Z π
n π 2m 1
= sin x cos x sin 2nx dx + sin2m+1 x cos 2nx dx
m 0 2m 0
· 2m+1 ¸π
1 (m) n sin x
= An + sin 2nx −
2m m 2m + 1 0
2 Z π
2n
sin2m+1 x cos 2nx dx
m(2m + 1) 0
1 (m) 4n2
= An − A(m)
2m 2m(2m + 1) n
µ ¶
(m−1) 1 4n2
ce qui démontre que An = 1+ − A(m)
n , ou bien
2m 2m(2m + 1)

(2m + 1)2m
A(m)
n = A(m−1) .
(2m + 1)2 − 4n2 n
3◦ . Le calcul précédent montre que
Z π
2 2(2m + 1)!
∀ (n, m) ∈ IN , (m)
An = sin2m+1 x cos 2nx dx = m .
0 Y
2 2
((2k + 1) − 4n )
k=0

2 (m) 1 (m)
Mais, a2n (fm ) = An pour n ≥ 1, et C0 (fm ) = A0 . La série de Fourier de fm
π π
converge simplement vers fm en tout point de IR car fm est de classe C 1 par morceaux.
Solutions 35

Alors

à m
!
2m+1 22m+1
(m!) 4(2m + 1)! X
2 Y 1
∀ x ∈ IR, | sin x | = + cos 2nx.
π(2m + 1)! π n=1
(2k + 1)2 − 4n2
k=0

En particulier, en prenant x = π/2,

X∞ m
Y
22m+1 (m!)2 n 1
π= + 4(2m + 1)! (−1) .
(2m + 1)! n=1
(2k + 1)2 − 4n2
k=0

La formule de Parseval donne aussi



à m
!
28m+4 (m!)4 24m+5 [(2m + 1)!]2 X Y 1
π2 = + .
(4m + 2)! (4m + 2)! n=1
[4n2 − (2k + 1)2 ]2
k=0

Solution .7 1◦ . La figure ci-contre montre la


représentation graphique de la fonction 2π-périodique
π2
6
f définie sur [0, 2π] par f
π
0
3(x − π)2 − π 2 2
− π12

f (x) =
12

D’autre part, si n 6= 0,
Z 2π
1 3(x − π)2 − π 2 −inx
Cn (f ) = e dx
2π 0 12
· ¸2π Z 2π
3(x − π)2 − π 2 −inx 1
= − e + (x − π) e−inx dx
24inπ 0 4inπ 0
· ¸2π Z 2π
x − π −inx 1 1
= 2
e − 2 e−inx dx = 2
4n π 0 4n π 0 2n

et un calcul simple montre que C0 (f ) = 0.


Comme la série de Fourier de f est normalement convergente et la fonction f est continue
alors
X∞
cos nx
∀ x ∈ IR, f (x) = .
n=1
n2

2◦ . L’égalité de Bessel s’écrit


Z ∞
1 2π
2 1X 1
f (x) dx = .
2π 0 2 n=1 n4
36 Séries de Fourier
Z 2π ∞
X
1 2 π4 1 π4
Mais f (x) dx = , donc = .
2π 0 180 n=1
n4 90
3◦ . La fonction f est bornée sur IR donc l’intégrale définissant F est convergente.
Z ∞ ∞ Z
X 2(n+1)π
−ax
F (a) = f (x) e dx = f (x) e−ax dx
0 n=0 2nπ
∞ Z 2π
X ∞
X Z 2π
−a(x+2nπ) −2naπ
= f (x + 2nπ) e dx = e f (x) e−ax dx
n=0 0 n=0 0
Z 2π
1
= f (x) e−ax dx
1 − e−2aπ 0
Deux intégrations par parties successives montrent que
Z 2π Z 2π
−ax 3(x − π)2 − π 2 −ax
f (x) e dx = e dx
0 0 12
· ¸2π Z 2π
3(x − π)2 − π 2 −ax 1
= − e + (x − π) e−ax dx
12a 0 2a 0
2 −2aπ
· ¸2π Z 2π
π (1 − e ) π − x −ax 1
= + e + 2 e−ax dx
6a 2a2 0 2a 0
π 2 a2 + 3 π
= 3
(1 − e−2aπ ) − 2 (1 + e−2aπ )
6a 2a
Il en résulte que
µ ¶
1 π 2 a2 πa
F (a) = 3 +1− , pour a > 0.
2a 3 th πa

4◦ . La série de Fourier de f converge uniformément sur [0, 2π] donc


Z 2π X∞ Z 2π
−ax 1
f (x)e dx = 2
e−ax cos nx dx.
0 n=1
n 0

Mais
Z 2π Z 2π ³ ´
−ax 1 (−a+in)x −(a+in)x a(1 − e−2aπ )
e cos nx dx = e +e dx = .
0 2 0 n2 + a2
Alors Z 2π ∞
X
−ax −2aπ a
f (x)e dx = (1 − e ) .
0 n=1
n2 (n2 + a2 )
On conclut enfin que
µ ¶ ∞
X
1 π 2 a2 πa a
∀ a > 0, F (a) = 3 +1− = .
2a 3 th πa n=1
n2 (n2 + a2 )
Solutions 37

Solution .8 1◦ .a. Pour n 6= 0,


Z · 2 2 ¸π Z π
a2 π 2 −inx a x −inx a2
Cn (g) = x e dx = − e + x e−inx dx
4π −π 4πin −π 2πin −π
· 2 ¸π 2 Z π 2 n
a x −inx a a (−1)
= 2
e − 2
e−inx dx =
2πn −π 2πn −π n2
Z π
a2 a2 π 2
D’autre part, C0 (g) = x2 dx = . La série de Fourier de g est alors
4π −π 6

X∞
a2 π 2 a2 (−1)n
S(g)(x) = +2 cos nx.
6 n=1
n2

1◦ .b. La série précédente converge normalement donc uniformément sur IR. La


fonction g est continue donc la série S(g) converge vers g. En prenant x = 0 et x = π
nous obtenons
X∞ X∞
(−1)n π2 1 π2
2
= − , 2
= .
n=1
n 12 n=1
n 6
L’égalité de Bessel permet d’écrire que
X∞ Z π 4
π4 1 1 x π4
+2 = dx = ,
36 n=1
n4 2π −π 4 20

X∞
1 π4
donc 4
= .
n=1
n 90
2◦ . La série définissant f est normalement convegente, donc f est une fonction
définie et continue sur IR. La 2π-périodicité de f est immédiate.
3◦ .a. Il est clair que, pour tout x ∈ IR,
∞ µ ¶
a2 π 2 X 2 1 1
g(x) − f (x) = + 2a (−1)n − cos nx
6 n=1
n2 n2 + a2
X∞
a2 π 2 (−1)n
= + 2a4 2 (n2 + a2 )
cos nx.
6 n=1
n

3◦ .b. Les deux séries


X∞ X∞
4 (−1)n−1 sin nx 4 (−1)n−1 cos nx
2a 2 + a2 )
, et 2a
n=1
n(n n=1
n2 + a 2

sont normalement, donc uniformément, convegentes. Il en résulte que g − f est de classe


C 2 sur IR et que
X∞
00 (−1)n−1 cos nx
4
(g − f ) (x) = 2a 2 + a2
= −a2 f (x).
n=1
n
38 Séries de Fourier

3◦ .c. Sur ] − π, π[ la fonction g est de classe C 2 et g 00 (x) = a2 pour tout x ∈] − π, π[.


Il en résulte que f = (f − g) + g est aussi de classe C 2 sur ] − π, π[ et que

∀ x ∈] − π, π[ f 00 (x) − a2 f (x) = a2 .

4◦ .a. La fonction constante −1 est evidemment une solution particulière de


l’équation précédente, et la solution générale de l’équation homogène associée est x 7→
A ch ax + B sh ax. On conclut qu’il existe A et B tels que f (x) = A ch ax + B sh ax − 1
pour tout x ∈] − π, π[.
La parité de f sur ] − π, π[ montre que B = 0. D’autre part, (g − f )0 est 2π-périodique
et continue sur IR, donc

a2 π − aA sh aπ = lim (g − f )0 (x) = lim (g − f )0 (x)


< >
x→π x→π
= lim (g − f )0 (x) = −a π + aA sh aπ.
2
>
x → −π

π
Ce qui donne A = . Nous arrivons à la conclusion
sh aπ

X∞
2 (−1)n cos nx π ch ax
∀ x ∈ [−π, π], f (x) = 2a 2 2
= − 1.
n=1
n +a sh aπ

4◦ .b. En prenant (a, x) = (1, π) puis (a, x) = (1, 0) nous obtenons


X X∞
1 e2π + 1 1 (−1)n πeπ 1
= π − , = − .
n=1
n2 + 1 e2π − 1 2 n=1
2
n +1 2π
e −1 2

1
Solution .9 Notons que f (x) = (sin x + | sin x |). L’exercice 6 avec m = 0 permet
2
alors d’écrire

1 sin x 2 X cos 2nx
∀ x ∈ IR, max(0, sin x) = + − .
π 2 π n=1 4n2 − 1

π
En prenant x = , nous obtenons
2

X∞
(−1)n 2−π
2
= .
n=1
4n − 1 4
Solutions 39

Solution .10 1◦ . Il est facile de voir que Θ est de classe C 1 par morceaux donc
d’après le théorème de Dirichlet, la série de Fourier de Θ converge partout vers Θ.
Z π/2 Z 3π/2
1 −inx 1
Cn (Θ) = xe dx + (π − x) e−inx dx
2π −π/2 2π π/2
Z π/2 Z
1 (−1)n π/2
−inx
= xe dx + x einx dx
2π−π/2 2π −π/2
Z π/2
1
Il en résulte que si n est pair alors Cn (Θ) = x cos nx dx = 0, et si n est impair,
π −π/2
n = 2k + 1, alors
Z Z π/2
−i π/2 2
Cn (Θ) = x sin nx dx = x sin nx dx
π −π/2 iπ 0
· ¸π/2 Z π/2
2x cos nx 2
= − + cos nx dx
inπ 0 inπ 0
· ¸π/2
2 sin nx 2(−1)k
= =
in2 π 0 iπ(2k + 1)2
Il en résulte que, pour tout x ∈ IR,
2 X (−1)k
Θ(x) = ei(2k+1)x .
iπ (2k + 1)2
k∈ZZ

π 2 X 1
En prenant x = π/2 nous obtenons, = , d’où le résultat.
2 π (2k + 1)2
k∈ZZ
◦ πx π π
2 . Si x ∈ [−n, n] alors ∈ [− , ] et
2n 2 2
πx πx 2 X (−1)k 2k+1
= Θ( ) = 2
ei 2n πx ,
2n 2n iπ (2k + 1)
k∈ZZ

ce qui démontre que


4n X (−1)k i 2k+1
∀ x ∈ [−n, n], x= 2 e 2n πx .
iπ (2k + 1)2
k∈ZZ

3◦ . Il est immédiat que


n n
à !
X X4n X (−1)k i 2k+1
Pn0 (t) = ir cr e = irt
e 2n πr cr eirt
π 2 (2k + 1)2
r=−n r=−n k∈ZZ
à n !
4n X (−1)k X 2k + 1
= 2 cr exp(ir(t + π))
π (2k + 1)2 r=−n 2n
k∈ZZ
µ ¶
4n X (−1)k 2k + 1
= 2 Pn t + π .
π (2k + 1)2 2n
k∈ZZ
40 Séries de Fourier

4◦ . Il en résulte que pour tout t ∈ IR,

4n X 1
| Pn0 (t) | ≤ 2
k P n k∞ = n k P n k∞ .
π (2k + 1)2
k∈ZZ

La dernière égalité vient de 1◦ . Ce qui démontre que k P 0 k∞ ≤ n k P k∞ pour tout


polynôme trigonométrique P de degré au plus n. De plus c’est la meilleure inégalité
possible comme le montre l’exemple P (x) = cos nx.

Solution .11 1◦ .a. En effet, la série considérée est normalement convergente et


à ∞
! µ ¶
X X reit r sin t
n it n
r sin(nt) = Im (re ) = Im = .
n=1 n=1
1 − reit r2
1 + − 2r cos t

1◦ .b. La convergence normale, en t, de la série précédente nous permet d’intégrer


terme à terme entre 0 et x ;

X∞
rn 1
(1 − cos nx) = Log (1 + r2 − 2r cos x) − Log (1 − r).
n=1
n 2

X∞
rn
Mais = −Log (1 − r) alors
n=1
n

X∞
rn 1
∀ x ∈ IR, − cos nx = Log (1 + r2 − 2r cos x).
n=1
n 2


X
◦ rk
1 .c. La convergence normale de la série cos(kx) cos(nx) et le résultat de
k
k=1
1◦ .b permettent d’écrire
Z ∞ Z
2 π
2 4 X rk π
Log (1 + r − 2r cos x) cos(nx) dx = − cos(kx) cos(nx) dx
π 0 π k 0
k=1
 2rn
− si n 6= 0
= n

0 si n = 0

Mais 2 sin(x) sin(nx) = cos(n − 1)x − cos(n + 1)x, alors nous obtenons immédiatement
 n+1

 r rn−1
π Z  − si n>1
2 n+1 n−1
In (r) = sin x Log ((1 + r2 − 2r cos x) sin(nx) dx =
π 0 
 r2
 si n=1
2
Solutions 41

2◦ .a. Il est immédiat que ϕ est continue 2π-périodique et impaire. ϕ n’est pas
dérivable en 0. Z π
◦ 2
2 .b. Posons Jn = sin xLog (2(1 − cos x)) sin(nx) dx. En remarquant que pour
π 0
x ∈ [0, π] nous avons
¡ ¢
Log (1 + r2 − 2r cos x) = Log (1 + r)2 sin2 (x/2) + (1 − r)2 cos2 (x/2)

nous pouvons écrire, pour x ∈]0, π],


µ ¶ Ã µ ¶2 !
1 + r2 − 2r cos x 2 1−r 1
Log = −2Log ( ) + Log 1 + .
2(1 − cos x) 1+r 1+r tg 2 (x/2)

Donc, pour x ∈]0, π] nous avons,


¯ µ ¶¯ µ ¶2
¯ 1 + r 2
− 2r cos x ¯ 2 1 − r 4
¯ Log ¯ ≤ 2Log +
¯ 2(1 − cos x) ¯ 1+r 1+r x2
car Log (1 + x) ≤ x et 2tg (x/2) ≥ x pour x ∈ [0, π]. Il en résulte, en notant que
| sin(x) sin(nx) | ≤ min(1, nx2 ),
µ ¶2
2 1−r
∀ r ∈ [0, 1[, | In (r) − Jn | ≤ 4Log + 8n ,
1+r 1+r
puis que 
 −2
 si n>1
n2 −1
Jn = lim In (r) =
<
r→1

 1
si n=1
2
Z π Z π
2 2Log 2
Mais Jn = ϕ(x) sin(nx) dx + sin(x) sin(nx) dx, alors
π 0 π 0
 −2
Z π 
 si n>1
2 n2−1
bn (ϕ) = ϕ(x) sin(nx) dx =
π 0 
 1 − Log 2 si n=1
2
2◦ .c. La série de Fourier de ϕ est
µ ¶ X∞
1 2
S(ϕ)(x) = − Log 2 sin x − 2
sin(nx).
2 n=2
n − 1

La fonction ϕ est de classe C 1 au voisinage de chaque x ∈


/ 2πZZ, alors la série de Fourier
de ϕ converge vers ϕ sur IR \ 2πZZ. La convergence de S(ϕ)(x) vers ϕ(x) pour tout
x ∈ 2πZZ est évidente. Alors
µ ¶ X∞
1 2
∀ x ∈ IR, sin(x)Log (1 − cos x) = − Log 2 sin x − 2
sin(nx).
2 n=2
n −1
42 Séries de Fourier

Solution .12 1◦ .a. Il est immédiat que fa est de classe C 1 par morceaux donc la
série de Fourier de fa converge vers fa .
D’autre part, pour n 6= 0,
Z 2a µ ¶ Z 2a
1 | x | −inx 1
Cn (fa ) = 1− e dx = 2 (2a − x) cos nx dx
2a −2a 2a 2a 0
· ¸2a Z 2a
(2a − x) sin nx 1
= + 2 sin nx dx
2a2 n 0 2a n 0
h cos nx i2a sin2 na
= − 2 2 = 2 2
2a n 0 a n

De plus C0 (fa ) = 1. Nous concluons

∞ µ
X ¶2
sin na
∀ x ∈ IR, fa (x) = 1 + 2 cos nx.
n=1
na


X µ ¶2
n sin na 1
1◦ .b. En prenant x = π, nous obtenons (−1) =− .
n=1
na 2
◦ sin (na) sin2 (nb)
2
1 .c. Remarquons que Cn (fa ∗ fb ) = . La série de Fourier de fa ∗fb
a2 b2 n4
converge normalement vers f a ∗ fb . Donc

X∞
sin2 (na) sin2 (nb)
fa ∗ fb (0) = 1 + 2 2 b2 n4
.
n=1
a

Mais, Z π
1
fa ∗ fb (0) = fa (x)fb (−x) dx
2π −π
µ
Z 2a ¶µ ¶
π |x| |x|
= 1− 1− dx
2ab−2a 2a 2b
Z 2a
π π(3b − a)
= 2 2 (2a − x)(2b − x) dx =
4a b 0 3b2
Alors, si 0 < a ≤ b ≤ π/2,

X∞
sin2 (na) sin2 (nb) π π 1
4
= a2 b − a3 − a2 b2
n=1
n 2 6 2

et si 0 < b ≤ a ≤ π/2,

X∞
sin2 (na) sin2 (nb) π π 1
4
= b2 a − b3 − a2 b2 .
n=1
n 2 6 2
Solutions 43

2◦ . Les coefficients de Fourier de gλ ont été calculés dans l’exercice 4. On trouve


sh πλ (−1)n λ
Cn (gλ ) =
π n 2 + λ2
et la série de Fourier de gλ est

sh πλ 2 sh πλ X (−1)n λ
S(gλ )(x) = + cos nx.
πλ π n=1
n2 + λ2

La série de Fourier de gλ est normalement convergente et gλ est continue sur IR alors



sh πλ sh πλ X (−1)n 2λ
∀ x ∈ IR, gλ (x) = + cos nx.
πλ π n=1 n2 + λ2

3◦ .a. Notons que


Z π X
1
fa (x) gλ (x) dx = fa ∗ gλ (0) = Cn (fa )Cn (gλ )
2π −π
n∈ZZ

Un calcul direct montre, en utilisant la parité, que


Z 2a
1
fa ∗ gλ (0) = 2 (2a − x) ch (λx) dx
2a 0
· ¸2a Z 2a
(2a − x) sh (λx) 1
= + 2 sh (λx) dx
2a2 λ 0 2a λ 0
ch (2aλ) − 1 sh 2 λa
= = 2 2
2a2 λ2 λ a
D’autre part,
λ sh (λπ) (−1)n sin2 (an) sh (λπ)
Cn (fa )Cn (gλ ) = si n 6= 0 et C0 (fa )C0 (gλ ) =
a2 π n2 (λ2 + n2 ) λπ
Donc,

sh 2 λa sh (λπ) 2λ sh (λπ) X (−1)n sin2 (an)
= +
λ2 a 2 λπ a2 π n=1
n2 (λ2 + n2 )
ou bien
X∞
(−1)n sin2 (an) π sh 2 (λa) a2
2 (n2 + λ2 )
= 3 sh (λπ)
− 2
.
n=1
n 2λ 2λ

◦ (−1)n sin2 (na) P


3 .b. Si, pour n ≥ 1 et λ ∈ IR, hn (λ) = , alors la série hn converge
n2 (n2 + λ2 )

X
normalement sur IR et toute fonction hn est continue sur IR, donc la somme hn est
n=1
continue sur IR. D’où
X∞ µ ¶
(−1)n sin2 (na) π sh 2 (λa) a2 2a4 − a2 π 2
= lim − = .
n=1
n4 >
λ→0
2λ3 sh (λπ) 2λ2 12
44 Séries de Fourier

Solution .13 1◦ . Soit fe la fonction 2π-périodique qui coı̈ncide sur [0, 2π[ avec f .
Un calcul simple montre que (Sn (x))n est la suite des sommes partielles de la série de
Fourier de fe. Comme fe est de classe C 1 sur ]0, 2π[, alors la suite (Sn (x))n converge vers
f (x) pour tout x ∈]0, 2π[.
n
X

2 . Il est bien connu que Dn (x) = 1 + 2 cos(kx). Il est alors immédiat de voir
k=1
que Z x
1 x
∀ x ∈ [0, 2π[, Sn (x) = Dn (t) dt − .
2 0 2
3◦ .a. Un développement limité simple montre que lim g(x) = 0, d’où la continuité
>
x→0
de g sur [0, π].
3◦ .b. En utilisant 2◦ nous obtenons immédiatement, pour tout x ∈ [0, π],
Z x Z x
2n + 1 sin(n + 1/2)t x
Sn (x) = g(t) sin( t) dt + dt − .
0 2 0 t 2
3◦ .c. C’est le lemme bien connu de Riemann.
3◦ .d. D’après 1◦ nous avons lim Sn (π) = 0, il en résulte que
n→∞
Z π
sin(n + 1/2)t π
lim dt =
n→∞ 0 t 2
Z (n+1/2)π Z ∞
sin t π sin t
ou bien, lim = . Mais l’intégrale dt est convergente, alors
n→∞ 0 t 2 0 t
Z ∞
sin t π
dt = .
0 t 2
4◦ . En effet,
³π ´ n
π X sin(kπ/n)
Sn = .
n n kπ/n
k=1

Alors Sn (π/n) est uneZsomme de Riemann qui converge, lorsque n tend vers l’infini,
π
sin t
vers l’intégrale définie dt.
0 t
5◦ . Notons que
Z ∞ Z π ∞ Z (2k+1)π
X
sin t sin t sin t
dt = dt + dt,
0 t 0 t (2k−1)π t
k=1

mais,
Z (2k+1)π Z π
sin t sin u
dt = du
(2k−1)π t −π 2πk + u
Z π Z π
sin u sin u
= du − du
0 2πk + u 0 2πk − u
Z π
u sin u
=−2 2 2 2
du.
0 4π k − u
Solutions 45

Alors,
Z ∞X Z ∞
X Z ∞
π
sin t π π
u sin u π
u sin u 1 X 1 π
dt − = 2 du > 2 du = =
0 t 2 0 4π 2 k 2 − u2 0
2
4π k 2 2π k 2 12
k=1 k=1 k=1

³π ´ π
On conclut que lim Sn > + lim f (x).
n→∞ n 12 x →>
0

OKMRAN
OUBA
ÉQUATIONS DIFFÉRENTIELLES LINÉAIRES

I. Généralités

Dans ce chapitre E désigne un espace vectoriel normé de dimension finie, L(E) l’espace
vectoriel des applications linéaires de E dans E et J un intervalle non réduit à un point
de IR. On considère deux applications continues sur J, la première notée b est à valeurs
dans E et la seconde notée A est à valeurs dans L(E):

b : J −→ E, et A : J −→ L(E).

On appelle équation diférentielle linéaire toute équation de la forme

y 0 = A(t)y + b(t). (1L )

On appelle équation diférentielle linéaire homogène associée à (1L ), l’équation

y 0 = A(t)y. (1H )

Une application ϕ : J −→ E est dite une solution de (1L ) si, et seulement si, elle est
dérivable et ∀ t ∈ J, ϕ0 (t) = A(t)ϕ(t) + b(t).

Exemple : En prenant, E = IR2 et


· ¸
1 1
b : IR −→E : t 7→ .
1 + t2 t
· ¸
1 1 −t
A : IR −→M2 (IR) : t 7→ .
1 + t2 t 1
L’équation différentielle linéaire Z 0 = A(t)·Z+b(t) est équivalente, en posant Z = t [x, y],
à 
 x ty 1
 x0 = 2
− 2
+ .
1+t 1+t 1 + t2

 y0 = tx y t
2
+ 2
+ .
1+t 1+t 1 + t2
et admet  √ 
exp(Arctg t) cos(Log 1 + t2 ) − 1
ϕ : IR −→ E : t 7→  √
.
exp(Arctg t) sin(Log 1 + t ) 2

comme solution.
2 Équations différentielles linéaires

II. La résolvante

Lemme II.1 Soit (gn )n≥0 une suite de fonctions continues sur un intervalle compact
[a, b] de IR à valeurs dans IR+ . Notons, pour k ∈ IN, Mk = sup{gk (x) : x ∈ [a, b]}, et
supposons qu’il existe t0 ∈ [a, b], et λ ∈ IR∗+ tels que
¯Z t ¯
¯ ¯
∀ n ≥ 1, ∀ t ∈ [a, b], gn (t) ≤ λ ¯¯ gn−1 (s) ds ¯¯ .
t0

λn (b − a)n
Alors, ∀ n ∈ IN, Mn ≤ M0 .
n!

Preuve : En effet, démontrons par récurrence sur n que


n
| t − t0 |
∀ n ≥ 0, ∀ t ∈ [a, b], gn (t) ≤ λn M0 . (∗)
n!

C’est immédiat si n = 0. Supposons l’assertion (∗) vraie pour n. Alors d’après


l’hypothèse, si t ∈ [t0 , b] on a,
Z t Z t
λn+1 M0 λn+1 M0
gn+1 (t) ≤ λ gn (s) ds ≤ (s − t0 )n ds = (t − t0 )n+1
t0 n! t0 (n + 1)!

et si t ∈ [a, t0 ] on a,
Z t0 Z t0
λn+1 M0 λn+1 M0
gn+1 (t) ≤ λ gn (s) ds ≤ (t0 − s)n ds = (t0 − t)n+1 .
t n! t (n + 1)!

Ce qui démontre (∗) pour n + 1 et par suite le lemme.

Théorème II.2. Soit E un espace vectoriel normé de dimension finie, L(E) l’espace
vectoriel des applications linéaires de E dans E et J un intervalle non réduit à un point
de IR. On considère une application continue A : J −→ L(E), et on fixe t0 ∈ J. Alors
il existe une, et une seule, application dérivable Xt0 : J −→ L(E) telle que

Xt0 (t0 ) = IE , ∀ t ∈ J, Xt00 (t) = A(t)◦Xt0 (t). (R)

On appelle la résolvante en t0 de l’équation différentielle y 0 = A(t)y, l’application

R : J × J −→ L(E) : (t, t0 ) 7→ R(t, t0 ) = Xt0 (t).


La résolvante 3

0
Preuve : Dans cette preuve nous notons k · k la norme de E et k · k la norme
correspondante des endomorphismes continues sur E.
– L’unicité: Soient X et Y deux applications dérivables sur J et vérifiant (R). Posons
Z = X − Y . Alors Z t
Z(t) = A(s)◦Z(s) ds. (†)
t0

Soit [a, b] un intérvalle compact contenu dans J et contenant t0 . La continuité des


0 0
applications t 7→ k A(t) k et t 7→ k Z(t) k sur [a, b] justifie l’existence de

0 0
λ = sup k A(t) k , et M0 = sup k Z(t) k .
t∈[a,b] t∈[a,b]

La relation (†) montre que


¯Z t ¯
¯ ¯
∀ t ∈ [a, b], ¯
k Z(t) k ≤ λ ¯ k Z(s) k ds ¯¯ .
0 0

t0

Donc en appliquant le lemme II.1 à la suite constante (gn )n∈IN définie par la relation
0
gn (t) = k Z(t) k , on obtient

λn (b − a)n
∀ n ∈ IN, M0 ≤ M0 ,
n!

et en faisant tendre n vers +∞ on trouve M0 = 0. Comme [a, b] est un sous-intervalle


arbitraire de J contenant t0 alors Z(t) = 0 pour tout t ∈ J. Ce qui prouve l’unicité.
– L’existence: Considérons la suite de fonctions (Zn )n∈IN de J dans E définie par
Z t
Z0 = 0, et ∀ t ∈ J, Zn (t) = IE + A(s)Zn−1 (s) ds. (‡)
t0

Soit [a, b] un intérvalle compact contenu dans J et contenant t0 . Nous allons démontrer
que la suite (Zn )n∈IN converge uniformément sur [a, b].
0
Posons, pour t ∈ [a, b], gn (t) = k Zn+1 (t) − Zn (t) k . La continuité des applications
0
t 7→ gn (t) et t 7→ k A(t) k sur [a, b] nous permet de poser

0
λ = sup k A(t) k , et Mn = sup gn (t).
t∈[a,b] t∈[a,b]

La relation (‡) démontre aussitôt que


¯Z t ¯
¯ ¯
∀ n ≥ 1, ∀ t ∈ [a, b], ¯
gn (t) ≤ λ ¯ gn−1 (s) ds ¯¯ .
t0
4 Équations différentielles linéaires

λn (b − a)n
et le lemme II.1 montre que ∀ n ∈ IN, Mn ≤ . Donc la série d’applications
X n!
(Zn+1 − Zn ) converge normalement sur [a, b], ce qui implique la convergence uni-
n≥0
forme de la suite (Zn )n∈IN sur [a, b].
Ce qui précède démontre que (Zn )n∈IN converge uniformément sur tout compact de
J. Notons sa limite Xt0 . Il est immédiat que Xt0 est continue sur J car les Zn ’s sont
continues sur J.
Nous allons démontrer que Xt0 est la solution demandée de (R). Posons, pour t ∈ J,
Z t
∆(t) = Xt0 (t) − IE − A(s)Xt0 (s) ds.
t0

0
Soit, de nouveau, un intevalle compact [a, b] de J contenant t0 . Si λ = sup k A(t) k
t∈[a,b]
alors nous avons vu que

0 λn (b − a)n
∀ n ∈ IN, ∀ t ∈ [a, b], k Zn+1 (t) − Zn (t) k ≤ .
n!
X
En utilisant le fait que Xt0 − Zn = (Zk+1 − Zk ), nous obtenons
k≥n

0 λn (b − a)n λ(b−a)
∀ n ∈ IN, ∀ t ∈ [a, b], k Xt0 (t) − Zn (t) k ≤ e . ($)
n!

Il est immédiat de voir que, pour t ∈ J,


Z t
∆(t) = Xt0 (t) − Zn (t) − A(s)[Xt0 (s) − Zn−1 (s)] ds,
t0

et d’après ($), pour tout n ≥ 1 et tout t ∈ [a, b],

0 (n + 1)λn (b − a)n λ(b−a)


k ∆(t) k ≤ e .
n!

En faisant tendre n vers l’infini nous voyons que ∆(t) = 0 pour tout t ∈ [a, b].Comme
[a, b] est un sous-intervalle arbitraire de J contenant t0 alors ∆(t) = 0 pour tout t ∈ J.
On conclut enfin que
Z t
∀ t ∈ J, Xt0 (t) = IE + A(s)Xt0 (s) ds.
t0

Ce qui démontre que Xt0 est solution de (R).


La résolvante 5

Définition : Soit E un espace vectoriel normé de dimension finie, L(E) l’espace


vectoriel des applications linéaires de E dans E et J un intervalle non réduit à un
point de IR. On considère deux applications continues A : J −→ L(E) et, b : J −→ E.
Pour (t0 , x0 ) ∈ J × E, on appelle problème de Cauchy Pt0 ,x0 , le problème de la
recherche des solutions de l’équation différentielle y 0 = A(t)y +b(t) vérifiant la condition
y(t0 ) = x0 .

Proposition II.3. On conserve les notations de la définition précédente avec b ≡ 0.


Pour t0 ∈ J on note t 7→ R(t, t0 ) la résolvante en t0 de l’équation différntielle
y 0 = A(t)y. (C’est à dire l’unique application de J dans L(E) védifiant R(t0 , t0 ) = IE ,
d
et R(t, t0 ) = A(t)◦R(t, t0 ) ). Alors
dt
1◦ . Pour tout x0 ∈ E, le problème de Cauchy

Pt0 ,x0 : y 0 = A(t)y, y(t0 ) = x0

admet une, et une seule, solution définie sur J par t 7→ R(t, t0 )x0 .
2◦ . Pour tout (t, u, v) ∈ J 3 on a

R(t, u)◦R(u, v) = R(t, v).

3◦ . Pour tout (t, u) ∈ J 2 , R(t, u) est inversible et [R(t, u)]−1 = R(u, t).
4◦ . Si S est l’espace des solutions de l’équation différentielle homogène y 0 = A(t)y:
½ ¾
0
S = y : J −→ E| y est dérivable et y = A(t)y ,

alors S est un espace vectoriel de dimension n = dim E. Plus précisément

T : E −→ S : x 7→ R(·, t0 )x,

est un isomorphisme.
5◦ . L’unique solution du problème de Cauchy

Pt0 ,x0 : y 0 = A(t)y + b(t), y(t0 ) = x0

est donnée par


Z t
∀ t ∈ J, y(t) = R(t, t0 )x0 + R(t, s) b(s) ds
t0
6 Équations différentielles linéaires

Preuve : 1◦ . – L’unicité: Soient z1 et z2 deux solutions de Pt0 ,x0 définies sur J. On


pose z = z1 − z2 , il est clair que z(t0 ) = 0 et z 0 = A(t)z, ce qui est équivalent à
Z t
∀ t ∈ J, z(t) = A(s)z(s) ds.
0

Ceci implique que z(t) = 0 pout tout t ∈ J. Voir la démonstration de l’unicité dans le
théorème II.2 qui utilise le lemme II.1.
– L’existence: Posons y(t) = R(t, t0 )x0 . Un calcul immédiat montre que y est une
solution de Pt0 ,x0 .
2◦ . Soient x ∈ E et (u, v) ∈ J 2 . Posons ϕ(t) = R(t, u)◦R(u, v)x et ψ(t) = R(t, v)x.
Il est immédiat que ϕ et ψ sont deux solutions du problème de Cauchy Pu,R(u,v)x .
L’unicité démontre que ϕ = ψ et par conséquent,

∀ t ∈ J, R(t, u)◦R(u, v)x = R(t, v)x,

Mais x est arbitraire dans E, d’où l’égalité demandée.


3◦ . en prenant v = t dans l’égalité de 2◦ on trouve IE = R(t, u)◦R(u, t). Ce qui
donne le résultat car on est en dimension finie.
Remarquons qu’en particulier, si t0 ∈ J,

∀ (u, v) ∈ J 2 , R(u, v) = R(u, t0 )◦[R(v, t0 )]−1 .

4◦ . La linéarité de T est évidente, et l’application T −1 est tout simplement

S −→ E : y 7→ y(t0 ).

5◦ . L’unicité est evidente. Il suffit de vérifier que y est bien une solution de Pt0 ,x0
ce qui est immédiat car
Z t
y(t) = R(t, t0 )x0 + R(t, t0 ) R(t0 , s) b(s) ds.
t0

et il suffit de dériver et de remplacer.

Remarque : D’après la proposition précédente nous constatons que l’ensemble des


solutions SL de l’équation différentielle linéaire y 0 = A(t)y + b(t), (L), est un
espace affine de dimension n. Si y0 est une solution particulière de l’équation (L) alors
SL = y0 + SH où SH est l’espace vectoriel de dimension n formé des solutions de
l’équation homogène y 0 = A(t)y, (H), associée.
Le wronskien 7

Une autre manière pour trouver la solution particulière de l’équation non homogène
est connue sous le nom de la méthode de variation des constantes:
Soit (ϕ1 , ϕ2 , . . . , ϕn ) une base de SH , l’espace vectoriel de dimension n formé des
solutions de l’équation homogène y 0 = A(t)y. Pour chercher une solution particulière
Xn
0
de y = A(t)y + b(t), on la cherche sous la forme z(t) = ck (t)ϕk (t) où les ck ’s sont
k=1
des fonctions dérivables inconnues à déterminer. En substituant dans l’équation non
homogène on trouve
n
X
c0k (t)ϕk (t) = b(t). (∗)
k=1

Or nous verrons plus loin que, pour tout t ∈ J, (ϕ1 (t), . . . , ϕn (t)) est une base de E et
par conséquent (∗) détermine uniquement les dérivées (c01 , . . . , c0n ). Nous obtenons alors
les ck ’s par intégration.

III. Le wronskien

Définition : Soit E un espace vectoriel normé de dimension finie et E = (e1 , . . . , en )


une base de E, J un intervalle non réduit à un point de IR et A : J −→ L(E) une
application continue. Pour (ϕ1 , ϕ2 , . . . , ϕn ) des éléments de SH , l’espace vectoriel de
dimension n formé des solutions de l’équation homogène y 0 = A(t)y, on pose

W : J −→ IK : t −→ detE (ϕ1 (t), . . . , ϕn (t)).

cette application W s’appelle le wronskien du système (ϕ1 , ϕ2 , . . . , ϕn ).

Proposition III.1. Avec les notations de la définition précédente on a, pour tout


t0 ∈ J, µZ ¶
t
∀ t ∈ J, W (t) = W (t0 ) exp Tr (A(s)) ds .
t0

En particulier, (ϕ1 , ϕ2 , . . . , ϕn ) est une base de SH si, et seulement s’il existe t0 ∈ J tel
que (ϕ1 (t0 ), ϕ2 (t0 ), . . . , ϕn (t0 )) soit une base de E. Enfin,
µZ t ¶
∀ t ∈ J, det R(t, t0 ) = exp Tr (A(s)) ds .
t0
8 Équations différentielles linéaires

Preuve : Nous avons besoin du lemme algèbrique suivant:

Lemme. Soit u un endomorphisme d’un espace vectoriel E de dimension n et E une


base de E. Alors, pour (x1 , x2 , . . . , xn ) ∈ E n , on a
n
X
detE (x1 , . . . , xk−1 , u(xk ), xk+1 , . . . , xn ) = Tr (u)detE (x1 , . . . , xn ).
k=1

En effet, notons f (x1 , . . . , xn ) le premier membre de l’égalité précédente. Nous


vérifions immédiatement que f est une forme n-linéaire alternée. Donc f est propor-
tionnelle à detE . C’est à dire il existe une constante λ ∈ IK telle que f = λ detE . La
constante λ est déterminée en testant l’égalité précédente sur la base E, ce qui donne
λ = f (e1 , . . . , en ) = Tr (u).

Revenons à la démonstration de la proposition. On a


n
X
0
W (t) = detE (ϕ1 (t), . . . , ϕk−1 (t), ϕ0k (t), ϕk+1 (t), . . . , ϕn (t))
k=1
Xn
= detE (ϕ1 (t), . . . , ϕk−1 (t), A(t)ϕk (t), ϕk+1 (t), . . . , ϕn (t))
k=1
=Tr (A(t)) W (t)
Ce qui prouve que
µZ t ¶
∀ t ∈ J, W (t) = W (t0 ) exp Tr (A(s)) ds .
t0

Le reste de la proposition est évident.

IV. Équations différentielles linéaires scalaires d’ordre n

Dans ce paragraphe IK désigne le corps des nombres réels ou complexes, et J un


intervalle de IR non réduit à un point. Pour n ≥ 1, on considère l’équation différentielle
linéaire scalaire d’ordre n:

y (n) + an−1 (t) y (n−1) + · · · + a1 (t) y 0 + a0 (t) y = f (t) (2L )

où (f, a0 , . . . , an−1 ) sont des applications continues de J dans IK. On considère aussi
l’équation différentielle homogène associée

y (n) + an−1 (t) y (n−1) + · · · + a1 (t) y 0 + a0 (t) y = 0. (2H )


Équations différentielles linéaires scalaires d’ordre n 9

Une application ϕ : J −→ IK est une solution de (2L ) si, et seulement si, elle est n-fois
dérivable sur J et vérifie

∀ t ∈ J, ϕ(n) (t) + an−1 (t) ϕ(n−1) (t) + · · · + a1 (t) ϕ0 (t) + a0 (t) ϕ(t) = f (t).

Ce qui démontre que la fonction Φ : J −→ IKn définie par


 ϕ(t) 
 ϕ0 (t) 
Φ(t) = 
 .. ,

.
ϕ(n−1) (t)
est solution de Y 0 = A(t)Y + b(t) avec
   
0 1 0 ··· 0 0
 .. ..
.
..
.
..
.
..  0
 . .  .
 
A =  ... ..
.
..
. 0 ; b= .
 . .
  0
 0 ··· ··· 0 1 
−a0 ··· ··· −an−2 −an−1 f

Inversement, la première composante de toute solution de Y 0 = A(t)Y + b(t) est


solution de (2L ). Les résultats suivants sont donc des corollaires de l’étude générale des
paragraphes précédents:
♣ L’ensemble des solutions de (2H ) est un IK-espace vectoriel de dimension n.
♣ Pour tout (t0 , x0 , . . . , xn−1 ) ∈ J × IKn , il existe une et une seule solution
ϕ : J −→ IK de (2L ) telle que ϕ(k) (t0 ) = xk pour tout k vérifiant 0 ≤ k ≤ n − 1.
♣ Si (ϕ1 , . . . , ϕn ) est une base de l’espace des solutions de (2H ), alors pour obtenir
une solution particulière de (2L ), on peut appliquer la méthode de variation
X
des constantes en cherchant cette solution sous la forme ck ϕk , où les ck ’s
k=1
sont des applications dérivables sur J déterminées par les conditions
 n

 X (k)

 c0j ϕj =0, 0≤k <n−1

 j=1
 n
X

 (n−1)

 c0j ϕj =f

j=1

♣ Si (ϕ1 , . . . , ϕn ) est un système de n solutions de (2H ) alors le wronskien de ce


système est
 ϕ1 (t) ··· ··· ϕn (t) 
 ϕ01 (t) ··· ··· ϕ0n (t) 
W (t) = det 
 .. .. .

. .
(n−1) (n−1)
ϕ1 (t) · · · ··· ϕn (t)
10 Équations différentielles linéaires

et vérifie µ Z t ¶
W (t) = W (t0 ) exp − an−1 (s) ds .
t0

Remarque : Considérons le cas particulier important d’une équation différentielle


linéaire scalaire d’ordre 2:

y 00 + a1 (t) y 0 + a0 (t) y = f (3L )

où a0 , a1 et f sont des fonctions continues de l’intervalle J dans IK.


Supposons que l’on connaı̂sse une solution particulière ϕ, ne s’annulant pas sur
J, de l’équation homogène associée à (3L ). Alors on peut trouver la solution générale
de (3L ) en la cherchant sous la forme ψ(t) = z(t) ϕ(t), où z est une fonction inconnue
à déterminer. En remplaçant dans (3L ) nous voyons que z 0 est solution de l’équation
différentielle linéaire d’ordre 1 suivant
µ ¶
00 2ϕ0 (t) f (t)
z + a1 (t) + z0 = .
ϕ(t) ϕ(t)

Une quadrature permet alors de trouver z puis ψ.

Exemple : Nous allons intégrer l’équation différentielle

(1 + x)y 00 − 2y 0 + (1 − x)y = xe−x . (4L )

Remarquons que J est l’un des deux intervalles ] − ∞, −1[ et ] − 1, ∞[. Nous voyons
immédiatement que x 7→ ex est une solution particulière de l’équation homogène asociée
à (4L ). Cherchons alors la solution générale de (4L ) sous la forme y = zex . ce qui donne

y 0 = (z + z 0 ) ex et y 00 = (z + 2z 0 + z 00 )ex .

Par conséquent z est solution de

(1 + x)z 00 + 2xz 0 = xe−2x . (5L )

L’équation homogène associée à (5L ) admet pour solution générale z 0 = λ(1 + x)2 e−2x .
La méthode de variation de la constante conduit à poser dans (5L ),

z 0 = u(x) (1 + x)2 e−2x


Systèmes différentiels linéaires à coefficients constants 11

x 1 + 2x
avec u(x) est donnée par u0 (x) = 3
, soit u(x) = − + µ, et
(1 + x) 2(1 + x)2

1
z 0 (x) = −( + x) e−2x + µ(1 + x)2 e−2x .
2

Enfin,
1 + x −2x
z(x) = e + c1 (5 + 6x + 2x2 ) e−2x + c2 .
2
La solution générale de (4L ) est

1 + x −x
y(x) = e + c1 (5 + 6x + 2x2 ) e−x + c2 ex .
2

V. Systèmes différentiels linéaires coefficients constants

Dans ce paragraphe, n est un entier naturel non nul, IK est le corps des nombres réels
ou celui des nombres complexes et E désigne IKn . Soient A une matrice carrée d’ordre
n à coefficients dans IK et b : J −→ E une application continue sur un intervalle J non
réduit à un point. On se propose d’étudier le système différentiel linéaire à coefficients
constants
X 0 = AX + b(t). (6L )

Rappelons que la résolvante R(t, 0) est la matrice obtenue comme limite de la suite
(Zm )m∈IN définie par
Z t
Z0 = 0, Zm+1 (t) = IE + AZm (s) ds.
0

m k
X t
Une récurrence simple montre que Zm+1 (t) = Ak , donc
k!
k=0

∞ k
X t
R(t, 0) = Ak .
k!
k=0

Définition : Soit A une matrice carrée d’ordre n à coefficients dans IK. On appelle
exponentielle de A la matrice notée eA ou exp(A) définie comme la somme de la série
X∞
1 k
normalement convergente A .
k!
k=0
12 Équations différentielles linéaires

Le théorème suivant résume quelques propriétés de l’exponentielle d’une matrice.

Théorème V.1. Notons Mn (IK) l’espace vectoriel des matrices carrées d’ordre n à
coefficients dans IK.
1◦ . Si 0 est la matrice nulle de Mn (IK) alors exp(0) = IE .
2◦ . Si A et B sont deux matrices de Mn (IK) qui commutent alors A eB = eB A.
3◦ . Si A est une matrice de Mn (IK), alors l’application

IR −→ Mn (IK) : t 7→ etA

est la solution unique du problème différentiel Z(0) = I, Z 0 (t) = AZ(t).


4◦ . Si A et B sont deux matrices de Mn (IK) qui commutent alors eA+B = eA eB .
5◦ . Si A est une matrice de Mn (IK) alors eA est inversible et [eA ]−1 = e−A .
6◦ . Si A est une matrice de Mn (IK) alors det(eA ) = exp(Tr (A).

Preuve : 1◦ . C’est evident.


2◦ . Car pour tout m ∈ IN on a
à m
! Ã m
!
X 1 k X 1 k
A B = B A.
k! k!
k=0 k=0

Il suffit de faire tendre m vers l’infini.


3◦ . Car t 7→ etA est la résolvante R(t, 0). Voir théorème II.2.
4◦ . En effet, Si Z(t) = etA etB on a Z(0) = IE et

Z 0 (t) = A etA etB + etA B etB = (A + B) Z(t).

Alors, d’après 3◦ ., Z(t) = et(A+B) . En particulier, au point t = 1 on trouve le résultat.


Remarquons qu’en général eA+B 6= eA eB , comme le montre l’exemple suivant:
¸ · · ¸
0 1 0 0
A= , B= .
0 0 1 0

D’où, · ¸ · ¸ · ¸
A 1 1 B 1 0 A+B ch 1 sh 1
e = , e = , ,e = .
0 1 1 1 sh 1 ch 1

5◦ . Les matrices A et −A commutent alors eA e−A = eA−A = I.


6◦ . La proposition III.1 et le résultat de 3◦ montrent que det R(t, 0) = exp(tTr (A)).
Ce qui prouve le résultat.
Systèmes différentiels linéaires à coefficients constants 13

L’étude du cas général nous permet d’écrire:


♣ Si SH est l’espace des solutions de l’équation différentielle homogène y 0 = Ay,
alors SH est un espace vectoriel de dimension n.
♣ Pour tout (t0 , x0 ) ∈ J × E, l’unique solution du problème de Cauchy

Pt0 ,x0 : y 0 = Ay + b(t), y(t0 ) = x0

est donnée par


Z t
(t−t0 )A
∀ t ∈ J, y(t) = e x0 + e(t−s)A b(s) ds.
t0

Le théorème suivant nous montre comment on peut calculer la fonction t 7→ etA .

Théorème V.2. Soit A ∈ Mn ( C)


| une matrice carrée d’ordre n. On suppose qu’il existe
p nombres complexes distincts λ1 , . . . , λp , et (m1 , . . . , mp ) ∈ (IN∗ )p tels que le polynôme
p
X
P de degré µ = mk défini par
k=1

p
Y
P (X) = (X − λj )mj ,
j=1

annule A. Alors, etA = Et (A) où Et (X) est l’unique polynôme de C[X]
| qui vŕifie
(
deg Et (X) ≤ µ − 1;
($) (k)
∀ ` ∈ {1, . . . , p}, ∀ k ∈ {0, . . . , m` − 1}, Et (λ` ) = tk eλ` t .

Preuve : Notons ∆ = {(i, j) : 1 ≤ i ≤ p, 0 ≤ j < mi }. D’après l’hypothèse Card∆ = µ.


| ∆ l’espace vectoriel des familles de C
– Soient C | indexées par ∆, et E
µ−1 l’espace

vectoriel des polynômes complexes de degré ne dépassant pas µ − 1. Notons

| ∆
ϕ : Eµ−1 −→ C : P 7→ (P (k) (λ` ))(`,k)∈∆ .

Clairement ϕ est linéaire. De plus si Q ∈ Ker ϕ alors, pour tout k ∈ {1, 2, . . . , p}, λk est
une racine d’ordre mk de Q ; c’est à dire (λk −X)mk divise Q pour tout k ∈ {1, 2, . . . , p}.
p
Y
Comme λ1 , . . . , λp sont distincts, alors Q est divisible par P = (λk − X)mk . Mais
k=1
deg P = µ et deg Q < µ. Il en résulte que Q = 0 et par conséquent ϕ est injective.
| ∆ . D’où ϕ est un isomorphisme d’espaces
D’autre part dim Eµ−1 = µ = Card∆ = dim C
14 Équations différentielles linéaires

vectoriels. Ceci démontre qu’il y a au plus un polynôme vérifiant les conditions ($) du
théorème.
| ∆ par
Soit (Pij )(i,j)∈∆ la base de Eµ−1 qui est l’image de la base canonique de C
l’isomorphisme ϕ−1 .
– Le reste Rm de la division euclidienne de X m par P est un élément de Eµ−1 et
donc s’exprime sur la base (Pij )(i,j)∈∆ de Eµ−1 . On peut alors écrire

p m
X Xi −1
m (m)
X = Qm (X)P (X) + Rm (X) = Qm (X)P (X) + dij Pij . (∗)
i=1 j=0

En utilisant le fait que λk est une racine d’ordre mk de Qm (X)P (X) nous obtenons,
pour 0 ≤ ` < mk ,

p m
X X i −1
(m) (`) (m)
m(m − 1) · · · (m − ` + 1)λm−`
k = dij Pij (λk ) = dk` .
i=1 j=0

Ce qui démontre que

p m
X Xi −1

Rm = m(m − 1) · · · (m − j + 1)λm−j
i Pij .
i=1 j=0

En utilisant (∗) et le fait que P (A) = 0 nous arrivons à Am = Rm (A) ou bien,

p m
X X i −1

∀ m ∈ IN A m
= m(m − 1) · · · (m − j + 1)λm−j
i Pij (A).
i=1 j=0

Nous pouvons donc écrire, pour t ∈ C,


|

X∞ p mi −1
tm m X X
exp(tA) = A = tj eλi t Pij (A) = Et (A).
m=0
m! i=1 j=0

Avec
p m
X X i −1

Et (X) = tj eλi t Pij (X).


i=1 j=0

Par conséquent, Et (X) est l’unique polynôme de degré inférieur ou égal à µ−1 déterminé
(k)
par les conditions Et (λ` ) = tk eλ` t pour tout (`, k) ∈ ∆.

Remarque : Le polynôme caractéristique XA (X) de A est un candidat toujours valable


pour le polynôme P du théorème précédent.
Systèmes différentiels linéaires à coefficients constants 15

Corollaire V.3. Si A ∈ Mn ( C)
| est une matrice diagonalisable, alors
X
etA = eλt Pλ (A)
λ∈Sp(A)

Y X −ν
Avec Pλ (X) = .
λ−ν
ν∈Sp(A)\{λ}
Corollaire V.4. Si A ∈ Mn ( C)
| est une matrice diagonalisable, et si V = (v1 , . . . , vn )
| n qui diagonalise A, (Av = λ v , pour 1 ≤ i ≤ n), alors les applications
est une base de C i i i

(t 7→ eλi t vi )1≤i≤n forment une base de l’espace des solutions du système différentiel
homogène Y 0 = AY .

Pour A ∈ Mn ( C),
| le théorème V.2 nous fournit un moyen pratique pour obtenir
une base de l’espace des solutions du système différentiel homogène Y 0 = AY en
considérant les colonnes de t 7→ etA .

Une autre méthode consiste à changer de fonction inconnue de la manière suivante:


| n qui contient un maximum de vecteurs
On considère une base V = (v1 , . . . , vn ) de C
propres de A. Toute solution Y du système différentiel homogène Y 0 = AY , s’écrit
Xn
Y = e (qu’on
zk vk où Z = t [z1 , . . . , zk ] est solution d’un nouveau système Z 0 = AZ,
k=1
obtient en remplaçant), et qui est en général beaucoup plus simple que le système
différentiel original.

Si le système différentiel n’est pas homogène alors nous pouvons, soit utiliser la
méthode exposée précédement en écrivant aussi le second membre dans la nouvelle base
V, ou bien utiliser la méthode de variation des constantes après avoir trouvé une base
de solutions du système différentiel homogène.

| n alors l’étude de l’équation différentielle linéaire ho-


Si (a0 , a1 , . . . , an−1 ) ∈ C
mogène d’ordre n à coefficients constants

y (n) + an−1 y (n−1) + · · · + a1 y 0 + a0 y = 0. (7H )

se ramène à l’étude du système Y 0 = AY où


 
0 1 0 ··· 0
 . . . .. .. .. .. 
 . . . . 
 . .. .. 
A =  .. . . 0  ∈ Mn ( C)
|
 
 0 ··· ··· 0 1 
−a0 · · · ··· −an−2 −an−1
16 Équations différentielles linéaires

Un calcul immédiat montre que


à n−1
! p
X Y
n n k
XA (λ) = det(A − λIn ) = (−1) λ + ak λ = (λi − λ)mi
k=0 i=1

avec (λ1 , . . . , λp ) les valeurs propres distinctes de A† et mi la multiplicité de λi .


L’étude générale et la forme de etA donnée par le théorème V.2 montrent qu’une
base de l’espace vectoriel des solutions de (H) est fournie par les applications t 7→ tk eλ` t
avec ` ∈ {1, . . . , p} et k ∈ {0, . . . , m` − 1}.
Pour une équation différentielle linéaire non homogène d’ordre n à coefficients
constants, on peut utiliser la méthode de variation des constantes. La méthode des
coefficients indéterminés est aussi d’une certaine utilité dans des cas particulier, la voici
résumée dans la proposition suivante:

| n , R(X) ∈ C[X]
Proposition V.5. Soient (a0 , a1 , . . . , an−1 ) ∈ C | et λ ∈ C.
| Alors
l’équation différentielle linéaire d’ordre n à coefficients constants

y (n) (t) + an−1 y (n−1) (t) + · · · + a1 y 0 (t) + a0 y(t) = R(t) eλt . (7L )

admet une solution particulière de la forme t 7→ Q(t) tr eλt où Q est une fonction
polynomiale de même degré que R, r est égal à la multiplicité de λ si λ est une valeur
caractéristique de l’équation différentielle, et r = 0 dans le cas contraire.

VI. Équations différentielles. “ Prélude à la théorie générale ”

Soient I un intervalle ouvert non vide de IR, U un ouvert non vide de IKn , (n ≥ 1),
et f une application continue de I × U dans IKn . On appelle équation différentielle
toute équation de la forme
dy
= f (t, y). (D)
dt
On appelle solution de (D) toute application ϕ : J −→ IKn telle que
 ◦

 1 .J est un sous-intervalle ouvert de I.

2◦ .ϕ(J) ⊂ U.


 ◦
3 .ϕ est dérivable sur J et ∀ t ∈ J, ϕ0 (t) = f (t, ϕ(t)).

λ1 , . . . , λp s’appellent les valeurs caractéristiques de l’équation différentielle, et
n−1
X
n
λ + ak λk = 0 s’appelle l’équation caractéristique associée à l’équation différentielle.
k=0
Équations différentielles. “ Prélude à la théorie générale ” 17

Pour insister sur le rôle joué par l’intervalle de définition d’une solution de (D)
nous noterons (J, ϕ) pour désigner une solution ϕ de (D) définie sur J.
Notons SD l’ensemble des solutions (J, ϕ) de (D). Nous avons vu que dans le cas
des équations différentielles linéaires† l’ensemble des solutions SD est un espace affine
et que toutes les solutions étaient définies sur l’intervalle I tout entier. Dans le cadre
général que nous avons décrit, l’ensemble SD n’a aucune raison d’avoir une structure
algébrique simple et peut ne contenir aucune solution définie sur I tout entier.
On peut définir sur SD une relation d’ordre ≺ en posant

 J1 ⊂ J2
(J1 , ϕ1 ) ≺ (J2 , ϕ2 ) ⇐⇒

∀ t ∈ J1 , ϕ1 (t) = ϕ2 (t).

On dit que (J0 , ϕ0 ) est une solution maximale de (D) si, et seulement si, (J0 , ϕ0 )
est un élément maximal dans (SD , ≺) ; i.e.

∀ (J, ϕ) ∈ SD , (J0 , ϕ0 ) ≺ (J, ϕ) =⇒ (J = J0 , ϕ = ϕ0 ).

Enfin, si (t0 , y0 ) ∈ I × U, on appelle problème de Cauchy Pt0 ,y0 la recherche des


solutions (J, ϕ) de D qui vérifient t0 ∈ J et ϕ(t0 ) = y0 .
p
Exemple : Considérons l’équation différentielle (D) y 0 = 3 3 y 2 . (Donc I = IR et
p
f (t, y) = 3 3 y 2 ).
L’application identiquement nulle y ≡ 0 est une solution de (D).
Si y est une solution de (D) qui ne s’annule pas sur un intervalle ouvert J de IR
¡ √ ¢0
alors, sur cet intervalle, (D) est équivalente à 3 y = 1 et donc

∃ c, ∀ t ∈ J, y(t) = (t + c)3 .

Soit (t0 , y0 ) ∈ IR × IR∗+ . On pose c0 = t0 − 3 y0 ,
et pour tout λ < c0 on considère
 ϕλ :
 (t − c0 )3 si t ∈ [c0 , ∞[


 y0
ϕλ (t) = 0 si t ∈ [λ, c0 [ λ



 c0 x0
(t − λ)3 si t ∈ [−∞, λ[

On vérifie immédiatement que, pour tout λ < c0 ,


(IR, ϕλ ) est une solution maximale de (D) qui vérifie
ϕ(t0 ) = y0 .

c’est à dire f (t, y) = A(t)y + b(t).
18 Équations différentielles linéaires

Donc le problème de Cauchy Pt0 ,y0 admet une infinité de solutions définies sur IR.
Ce résultat subsiste si (t0 , y0 ) ∈ IR × IR− , nous laissons au lecteur la tache de construire
ces solutions.
Remarquons que, dans le cas (t0 , y0 ) ∈ IR × IR∗+ , toutes les solutions de Pt0 ,y0

coı̈ncident sur le voisinage ]t0 − ε, t0 + ε[ de t0 avec ε = 3 y0 . Il en est de même pour
Pt0 ,y0 lorsque y0 < 0. Nous disons que les points (t0 , y0 ) ∈ IR × IR∗ sont des points
d’unicité locale pour l’équation différentielle (D).
Par contre le point (t0 , 0) n’est pas un point d’unicité locale pour (D), car pour
tout ε > 0 il y a au moins deux solutions distinctes définies sur ]t0 − ε, t0 + ε[, à savoir
ϕ1 (t) = 0 et ϕ2 (t) = (t − t0 )3 , au problème de Cauchy Pt0 ,0 .

Définition : Soient I un intervalle ouvert non vide de IR, U un ouvert non vide de IKn ,
et f une application continue de I × U dans IKn . On dit que le point (t0 , y0 ) ∈ I × U
est un point d’unicité locale pour l’équation différentielle y 0 = f (t, y) si, et seulement
si, il existe ε > 0 tel que toutes les solutions du problème de Cauchy Pt0 ,y0 coı̈ncident
sur ]t0 − ε, t0 + ε[.

Exemple : Considérons l’équation différentielle

(D) y 0 = −y 2

(Donc I = IR et f (t, y) = −y 2 ).
L’application identiquement nulle y ≡ 0 est une solution de (D).
Si y est une solution de (D) qui ne s’annule pas sur un intervalle ouvert J de IR
µ ¶0
1
alors, sur cet intervalle, (D) est équivalente à = 1 et donc
y

1
∃ c, ∀ t ∈ J, y(t) = .
t+c

On conclut que, pour tout (t0 , y0 ) ∈ IR2 il existe une, et une seule, solution du problème
y0
de Cauchy Pt0 ,y0 , c’est la fonction t 7→ . Remarquons que tout point de
1 + y0 (t − t0 )
IR2 est un point d’unicité locale (même globale) pour (D). Mais la seule solution de (D)
définie sur IR est la solution identiquement nulle.
Équations différentielles. “ Prélude à la théorie générale ” 19

Définition : Soient I un intervalle ouvert non vide de IR, U un ouvert non vide de
IKn , et f une application continue de I × U dans IKn . On dit que f est localement
lipschitzienne par rapport à la seconde variable si, et seulement si,

∀ (t0 , y0 ) ∈ I × U , ∃(ε, r, K) ∈ (IR∗+ )3 ,

 
 t ∈]t0 − ε, t0 + ε[∩I 

 

 
y ∈ B(y0 , r) ∩ U =⇒ k f (t, y) − f (t, z) k ≤ K k y − z k .

 


 

z ∈ B(y0 , r) ∩ U

Remarque : Soient I un intervalle ouvert non vide de IR, U un ouvert non vide de IRn ,
et
f : I × U −→ IRn , (t, y1 , . . . , yn ) 7→ f (t, y1 , . . . , yn ).
µ ¶
∂f
une fonction continue. Si les dérivées partielles existent et sont continues
∂yk 1≤k≤n
sur I × U alors f est localement lipschitzienne par rapport à la variable (y1 , . . . , yn ). En
effet c’est une conséquence du théorème des accroissements finis.

Les deux théorèmes suivants sont admis sans démonstration:

Théorème VI.1. (Existence et unicité locale) Soient I un intervalle ouvert non


vide de IR, U un ouvert non vide de IKn , et f une application de I × U dans IKn ,
continue et localement lipschitzienne par rapport à la seconde variable. Alors, pour
tout (t0 , y0 ) ∈ I × U , il existe ε > 0, et une solution (]t0 − ε, t0 + ε[, ϕ) de l’équation
différentielle y 0 = f (t, y) vérifiant ϕ(t0 ) = y0 . De plus, si (J, ψ) est aussi une solution
du problème de Cauchy Pt0 ,y0 alors ψ et ϕ coı̈ncident sur J∩]t0 − ε, t0 + ε[.

Théorème VI.2. (Existence et unicité globale) Soient I un intervalle ouvert non


vide de IR, U un ouvert non vide de IKn , et f une application de I × U dans IKn ,
continue et localement lipschitzienne par rapport à la seconde variable. Alors, pour tout
(t0 , y0 ) ∈ I × U , le problème de Cauchy Pt0 ,y0 pour l’équation différentielle y 0 = f (t, y)
admet une, et une seule, solution maximale.
20 Équations différentielles linéaires

EXERCICES

Exercice .1 Calculer etA lorsque A est l’une des matrices suivantes:


     
5 −1 −3 0 1 −1 2 −3 −1
1 1 −1  ,  −1 0 2,  1 −2 −1  ,
1 0 0 1 −2 0 −2 6 3
 
  2 1 0 0 0
1 −1 2 −2
 −1 0 0 0 0
0 0 1 −1   
 ,  1 2 3 1 0.
1 −1 1 0  
−1 −2 −1 2 1
1 −1 1 0
1 2 1 0 1

Exercice .2 Soit A une matrice de Mn ( C).


| On suppose A3 = 3A − 2I, calculer
etA .

Exercice .3 Calculer etA lorsque


 
m−1 −2m − 1 2m − 2
A= 1 m+1 1 .
1 2m + 1 2−m

Exercice .4 Donner la solution générale du système différentiel


 0

 x =−y+1

y0 = − z


 0
z = − m2 x + m(m + 2)y − (1 + 2m)z + 1

On discutera suivant les valeurs du paramètre m.

Exercice .5 Trouver la solution générale sur ]0, +∞[ du système différentiel


 0
x = −6x + 5y + 3z +1/t
y0 = −8x + 7y + 4z
 0
z = −2x + y + z +2/t
Exercices 21

Exercice .6 Trouver la solution du système différentiel


 0
x = −4x + y + z +3t2 e−2t
y0 = x − y − 2z
 0
z = −2x + y − z
vérifiant la condition initiale x(0) = 0, y(0) = 3, et z(0) = 0.

Exercice .7 Soient Ω et Θ deux vecteurs de IR3 . Résoudre



 dV =Ω ∧ V + Θ
dt

V (0) =0

Exercice .8 Déterminer (a0 , a1 , a2 , a3 ) ∈ IR4 tels que l’équation différentielle


x(4) = a3 x000 + a2 x00 + +a1 x0 + a0 x admette t 7→ tet sin t comme solution.

Exercice .9 Trouver la solution de l’équation différentielle y 00 + 2y 0 + y = | x |,


vérifiant y(0) = y 0 (0) = 0.

Exercice .10 Soit (E) l’équation différentielle t(t − 1)x00 + 3x0 − 6x = 0.


1◦ . Chercher les solutions de (E) qui s’écrivent comme somme de séries entières.
Préciser leur intervalle de convergence.

2 . Déterminer l’espace des solutions de (E) sur ]0, 1[, sur ] − ∞, 0[ et sur ]1, ∞[. Quelle
est la dimension de chacun de ces espaces ?
3◦ . Donner la dimension des espaces de solutions sur ] − ∞, 1[, ]0, ∞[ et IR.

Exercice .11 Soient t 7→ a(t) et t 7→ b(t) deux fonctions définies sur un intervalle
J, telles que a soit dérivable et b soit continue. On suppose qu’il existe ϕ1 : J −→ IR∗+
une solution de y 00 + ay 0 + by = 0 telle que ϕ2 (t) = tϕ1 (t) soit aussi une solution de
cette équation.
1◦ . Quelle relation existe-t-il entre a et b ?
1 3
2◦ . Résoudre y 00 + tg t y 0 + ( + tg 2 t) y = (cos t)3/2 , pour t ∈] − π/2, π/2[.
2 4
Exercice .12 Soient t 7→ a(t) et t 7→ b(t) deux fonctions définies sur un intervalle J,
telles que b soit dérivable et a soit continue. On suppose qu’il existe θ : J −→ IR telle
que t 7→ cos θ(t) et t 7→ sin θ(t) soient des solutions de y 00 + ay 0 + by = 0.
1◦ . Quelle relation existe-t-il entre a et b ?
2◦ . Résoudre
cos t y 00 + sin t y 0 + cos3 t y = 0, pour t ∈] − π/2, π/2[.
t y 00 − y 0 + t3 y = t3 sin(t2 /2), pour t ∈ IR∗+ .
22 Équations différentielles linéaires

Exercice .13 Résoudre (x2 −1) y 00 +x y 0 −y = 0. (On pourrait chercher une solution
particulière polynomiale).

Exercice .14 Résoudre x y 00 + 2 y 0 + ω 2 x y = 0. (ω 6= 0). (On pourrait chercher une


solution sous forme d’une série entière).

Exercice .15 L’équation différentielle x2 y 00 + x y 0 + (x2 − 4) y = 1 possède-t-elle des


solutions développables en série entière ?

Exercice .16 Résoudre le système différentiel


 00
 2x + 3y 00 + 2x0 + y0 + x + y = 0

x00 + 3y 00 + 4x0 + 2y 0 − x − y = 0

Exercice .17 Intǵrer les équations différentielles suivantes:

x2 y 00 + xy 0 + y = cos(αLog x); (α ∈ IR∗+ ).


x2 y 00 + 4xy 0 + 2y = Log (1 + x2 ).

Exercice .18 Soit l’équation différentielle x2 y 00 + xy 0 − (x2 + x + 1)y = 0.


1◦ . On suppose que cette équation admet une solution φ développable en série entière

X
telle que φ0 (0) = 1, on note φ(x) = an xn . Montrer qu’alors φ est unique.
n=0
Montrer aussi que, pour tout n ∈ IN∗ , | an | ≤ 1/(n − 1)!. En déduire que l’équation
considérée admet une et une seule solution φ développable en série entière telle que
φ0 (0) = 1.
2◦ . Résoudre l’équation en posant y = ze−x /x et reconnaı̂tre la solution trouvée en 1◦ .

Exercice .19 Soient E un e.v.n de dimension finie, I un intervalle


Z t ouvert non vide
de IR, et G : I −→ L(E) une application continue. On note G(t) = G(s) ds, où t0 ∈ I
t0
est fixé.
Montrer que si, pour tout t ∈ I, G(t).G(t) = G(t).G(t), alors

d
exp(G(t)) = G(t) exp(G(t)).
dt

En déduire que l’unique solution de l’équation différentielle X 0 (t) = G(t)X(t)


vérifiant X(t0 ) = x0 ∈ E est donnée par X(t) = exp(G(t))x0 .
Exercices 23

Exercice .20 Résoudre le système différentiel suivant



 y(t)

 x0 (t) = 2tx(t) −
1 + t2

 y 0 (t) = − x(t)
 + 2ty(t)
1 + t2

Exercice .21 On considère sur IR∗+ le système différentiel suivant



 0 −x(t) y(t) 1

 x (t) = + +
t(t2 + 1) t2 (t2
+ 1) t

 −t2 x(t) 2
(2t + 1)y(t)
 y 0 (t) = + + 1
t2 + 1 t(t2 + 1)

1◦ . Montrer que (x, y) = (1, t) est une solution du système homogène associé.
2◦ . Trouver la solution générale du système.

Exercice .22 On considère le système différentiel suivant


 0
 x (t) = tx(t) + (1 − t2 )y(t) + (1 − t2 )2

y 0 (t) = x(t) − ty(t) + t(1 + t2 )

1◦ . Rechercher une solution du système homogène associé sous forme d’un système de
deux polynômes de degré au plus deux.
2◦ . Trouver la solution générale du système.
24 Équations différentielles linéaires

SOLUTIONS

Solution .1 Rappelons le théorème suivant:


Théorème: Soit A ∈ Mn ( C)
| une matrice d’ordre n. On suppose qu’il existe une
| × IN∗ telle que λ , . . . , λ
suite finie ((λi , mi ))1≤i≤p de C 1 p soient distincts,
Yp
et le polynôme P (X) = (X − λi )mi est un polynôme qui annule A
i=1
(i.e. P (A) = 0). Alors etA = Rt (A) où Rt (X) est l’unique polynôme de
p
X
degré strictement inférieur à ` = mi qui vérifie:
i=1

dj Rt
j
(λi ) = tj eλi t , 1 ≤ i ≤ p, 0 ≤ j < mi .
dX
 
5 −1 −3
a. Calcul de etA avec A =  1 1 −1 .
1 0 0
Un calcul immédiat montre que le polynôme caractéristique de la matrice A est donné
par XA (X) = −(X−1)2 (X−4). Cherchons le polynôme Rt (X) = α+β(X−1)+γ(X−1)2
qui vérifie Rt (1) = et , Rt0 (1) = tet et Rt (2) = e2t . Une substitution simple montre alors
e4t −et −3tet
que α = et , β = tet et γ = 9 . Ce qui donne
e4t − et − 3tet
etA = et I + tet (A − I) + ( )(A − I)2
9
ce qui, tout calcul fait, s’écrit
 
−3et + 12e4t (4 + 3t)et − 4e4t (8 − 3t)et − 8e4t
1
etA =  −3et + 3e4t (10 + 3t)et − e4t (2 − 3t)et − 2e4t t  .
9
−3et + 3e4t (1 + 3t)et − e4t (11 − 3t)et − 2e4t
 
0 1 −1
b. Calcul de etA avec A =  −1 0 2 .
1 −2 0
Un calcul immédiat montre que
   
−2 2 2 0 −6 6
A2 =  2 −5 1, A3 =  6 0 −12  = −6A.
2 1 −5 −6 12 0
Une recurrence simple montre alors que A2n+1 = (−6)n A et A2n+2 = (−6)n A2 pour
tout n ≥ 0. Alors
∞ n
à ∞
! Ã ∞
!
X t n X (−6)n t2n+1 X (−6)n t2n+2
etA = A =I+ A+ A2
n=0
n! n=0
(2n + 1)! n=0
(2n + 2)!
Solutions 25

soit, √ √
tA sin( 6t) 1 − cos( 6t) 2
e =I+ √ A+ A .
6 6
 
2 −3 −1
c. Calcul de etA avec A =  1 −2 −1 .
−2 6 3
Un calcul immédiat montre que
   
1 −3 −1 1 −3 −1
(A − I)2 =  1 −3 −1  .  1 −3 −1  = 0.
−2 6 2 −2 6 2

Alors etA = etI+t(A−I) = et (I + t(A − I)), donc


 
(1 + t)et −3tet −tet
etA =  tet (1 − 3t)et −tet  .
−2tet 6tet (1 + 2t)et
 
1 −1 2 −2
0 0 1 −1 
d. Calcul de etA avec A =  .
1 −1 1 0
1 −1 1 0
Un calcul simple montre que le polynôme caractéristique de A est XA (X) = X 2 (X −1)2 .
Cherchons le polynôme Rt (X) = α + βX + γX 2 + δX 3 qui vérifie Rt (0) = 1, Rt0 (0) = t,
Rt (1) = et et Rt0 (1) = tet . Une substitution simple montre alors que

α = 1, β = t, γ + δ = et − 1 − t, 2γ + 3δ = tet − t.

Ce qui donne α = 1, β = t, γ = −tet + 3et − 2t − 3, δ = tet − 2et + t + 2.

etA = I + tA + (−tet + 3et − 2t − 3) A2 + (et − 2et + t + 2) A3

ce qui, tout calcul fait, s’écrit


 
et 1 − et −1 + t + et 1 − t − et
 0 1 t −t 
etA = t t.
te −tet 1 + tet t
−1 + e − te
tet −tet tet et − tet
 
2 1 0 0 0
 −1 0 0 0 0
 
e. Calcul de etA avec A =  1 2 3 1 0 .
 
−1 −2 −1 2 1
1 2 1 0 1
26 Équations différentielles linéaires

Le polynôme caractéristique de A est XA (X) = −(X − 1)2 (X − 2)3 . Cherchons le


polynôme Rt (X) = α + β(X − 2) + γ(X − 2)2 + δ(X − 2)3 + ²(X − 1)(X − 2)3 qui vérifie
Rt (2) = e2t , Rt0 (2) = te2t Rt00 (2) = t2 e2t , Rt (1) = et et Rt0 (1) = tet . Une substitution
simple montre alors que

t2 2t
α = e2t , β = te2t , γ= e ,
2

t2 2t t2 2t
δ = (1 − t + )e − et , ² = (3 − 2t + )e − (t + 3)et
2 2
ce qui, tout calcul fait, s’écrit
 
1+t t 0 0 0
 −t 1 − t 0 0 0
 
etA =et  −2 + t −3 + t 0 0 0 
 
4−t 5−t 0 0 0
−8 − t −7 − t 0 0 0
 
0 0 0 0 0
 0 0 0 0 0 
 2
t2 
2t  2 − 2t + 2t2
+e  3 − 2t + t2 1 + t t + t2 2 .
2 2 
 −4 + 6t − 2t2 −5 + 4t − t2 −t 1 − t2 t − t2 
t2 t2
8 − 6t + 2t2 7 − 4t + t2 t 2 1−t+ 2

Solution .2 Le polynôme P (X) = X 3 − 3X + 2 = (X − 1)2 (X + 2) annule A.


Cherchons alors l’unique polynôme Rt (X) = a+b(X −1)+c(X −1)2 tel que Rt (1) = et ,
Rt0 (1) = tet et Rt (−2) = e−2t .
1 −2t
Une substitution simple montre que a = et , b = tet et a = (e − et + 3tet ). Ce qui
9
donne
1
etA = Rt (A) = et I + tet (A − I) + (e−2t − et + 3tet ) (A − I)2 .
9
 
m − 1 −2m − 1 2m − 2
Solution .3 Posons Am =  1 m+1 1 .
1 2m + 1 2−m
Un calcul immédiat montre que le polynôme caractéristique de la matrice Am est

XAm (X) = −(X − m)(X − m − 1)(X + m − 1).

Supposons m ∈
/ {0, 1/2} pour que les valeurs propres de Am soient distinctes. Cherchons
alors l’unique polynôme

Rt (X) = a(X − m)(X − m − 1) + b(X − m − 1)(X + m − 1) + c(X − m)(X + m − 1).


Solutions 27

qui vérifie Rt (m) = emt , Rt (m + 1) = e(m+1)t , et Rt (1 − m) = e(1−m)t . Une substitution


simple montre que

e(1−m)t emt e(1+m)t


a= , b= , c= .
2m(2m − 1) 1 − 2m 2m

Un calcul simple mais long montre que

etAm = αm (t)I + βm (t)Am + γm (t)A2m .

avec
m + 1 mt sh (m − 1/2)t 1 − m (1+m)t
αm (t) = e − (m + 1) et/2 + e
2 2m − 1 2
sh mt sh (m − 1/2)t
βm (t) = − et + 4 et/2
m 2m − 1
sh mt sh (m − 1/2)t
γm (t) =et − 2 et/2 .
m 2m − 1
Notons que la fonction m 7→ Am est continue sur IR donc, pour tout t la fonction
m 7→ etAm est aussi continue sur IR. Il en résulte que pour étudier les cas m = 0 et
m = 1/2, il suffit de passer à la limite dans les expressions de αm , βm et de γm .

α0 (t) = lim αm (t) = 1,


m→0
β0 (t) = lim βm (t) = (2 − t)et − 2,
m→0
γ0 (t) = lim γm (t) = (t − 1)et + 1.
m→0

et
1 3t/2
α1/2 (t) = lim αm (t) = (e + 3(1 − t)et/2 ),
m→1/2 4
β1/2 (t) = lim βm (t) = −e3t/2 + (1 + 2t)et/2 ,
m→1/2

γ1/2 (t) = lim γm (t) = e3t/2 − (1 + t)et/2 .


m→1/2

Ce qui donne le résultat.

Solution .4 Le système est équivalent à X 0 = Am X + B, avec


   
0 −1 0 1
Am = 0 0 −1 , B = 0.
−m2 m(m + 2) −(1 + 2m) 1
28 Équations différentielles linéaires

Le polynôme caractéristique de Am est

XAm (λ) = −(λ + 1)(λ + m)2 .

Supposons que m 6= 1. Cherchons le polynôme Rt (λ) = a + b(λ + m) + c(λ + m)2 qui


vérifie Rt (−1) = e−t , Rt (−m) = e−mt et Rt0 (−m) = te−mt . Une substitution simple
montre que
µ ¶
−mt e(m−1)t − 1 − (m − 1)t 2
Rt (λ) = e 1 + t(λ + m) + (λ + m) .
(m − 1)2
Alors,
µ ¶
tAm −mt e(m−1)t − 1 − (m − 1)t 2
e =e 1 + t(Am + mI) + (Am + mI) .
(m − 1)2
t m+1 2
Si m 6= 0, la matrice Am est inversible et le vecteur Z = −A−1
m B = [( m ) , 1, 0]
est une solution particulière de l’équation non homogène. Il en résulte que la solution
générale du système est X(t) = etAm X0 + Z si m ∈
/ {0, 1}, ce qui, tout calcul fait, donne
µ ¶
2 e(m−1)t − 1 − (m − 1)t −mt (m + 1)2
x(t) = α + (αm − β)t + (αm − 2βm + γ) e +
(m − 1)2 m2
µ ¶
e(m−1)t − 1 − (m − 1)t
y(t) =1 + e−mt β + (βm − γ)t + (αm2 − 2βm + γ)
(m − 1)2
µ
z(t) =e−mt γ + (−αm2 + βm(m + 2) − γ(m + 1))t +

2 e(m−1)t − 1 − (m − 1)t
(αm − 2βm + γ)
(m − 1)2
La fonction m 7→ Am est continue sur IR, alors en faisant tendre m vers 1 nous obtenons
la solution générale du système dans le cas m = 1 à savoir
µ ¶
−t t2
x(t) =4 + e α + (α − β)t + (α − 2β + γ)
2
µ ¶
−t t2
y(t) =1 + e β + (β − γ)t + (α − 2β + γ)
2
µ ¶
−t t2
z(t) =e γ + (−α + 3β − 2γ)t + (α − 2β + γ)
2
Il nous reste à étudier le cas m = 0. Dans ce cas le système s’intègre immédiatement et
nous donne la solution générale
t2
x(t) =α − βt + γe−t + t + ,
2
y(t) =β + γe−t − t,
z(t) =γe−t + 1.
Solutions 29

Solution .5 Le système considéré est équivalent à X 0 = AX + B, avec


   
−6 5 3 1
 1 
A = −8 7 4, B(t) = 0 .
t
−2 1 1 2

Le polynôme caractéristique de A est XA (λ) = −λ(λ − 1)2 . Un calcul élémentaire nous


permet de trouver des vecteurs (v1 , v2 , v3 ) de IR3 qui vérifient Av1 = 0, Av2 = v2 et
Av3 = v3 + v2 .
     
1 1 0
v1 =  0  , v2 =  2  , v3 =  −1  .
2 −1 2

Les vecteurs (v1 , v2 , v3 ) forment une base de IR3 , la solution cherchée s’exprime alors
sur cette base: X(t) = x1 (t)v1 + x2 (t)v2 + x3 (t)v3 . En remplaçant dans X 0 = AX + B
et en identifiant les coefficients de v1 , v2 et v3 , nous arrivons au système
 0
 x1 = 1/t
x0 = x2 + x3
 20
x3 = x3

Ce système est très facile à résoudre et nous obtenons

x1 (t) = α + Log t, x2 (t) = (β + γt)et , x3 (t) = γet .

En revenant à X,

x(t) =α + (β + γt) et + Log t,


y(t) =(2β − γ + 2γt) et ,
z(t) =2α + (2γ − β − γt) et + 2Log t.

Solution .6 Le système considéré est équivalent à X 0 = AX + B, avec


   
−4 1 1 1
A =  1 −1 −2  , 2 −2t  
B(t) = 3t e 0 .
−2 1 −1 0
30 Équations différentielles linéaires

Le polynôme caractéristique de A est XA (λ) = −(λ + 2)3 . Un calcul élémentaire


nous permet de trouver des vecteurs (v1 , v2 , v3 ) de IR3 qui vérifient Av1 = −2v1 ,
Av2 = −2v2 + v1 et Av3 = −2v3 + v2 .
     
1 −1 1
1
v1 =  1  , v2 =  0  , v3 =  −1  .
3
1 −1 0
Les vecteurs (v1 , v2 , v3 ) forment une base de IR3 . Le vecteur B(t) et la condition initiale
X(0) s’expriment sur cette base,

B(t) = 3t2 e−2t (v1 + v2 + 3v3 ), X(0) = 3(v1 + v2 ),

enfin la solution cherchée s’exprime aussi sur cette base: X(t) = x1 (t)v1 + x2 (t)v2 +
x3 (t)v3 . En remplaçant dans X 0 = AX + B et en identifiant les coefficients de v1 , v2 et
v3 , nous arrivons au système
 0
 x1 = −2x1 + x2 + 3t2 e−2t
x0 = − 2x2 + x3 + 3t2 e−2t
 02
x3 = − 2x3 + 9t2 e−2t

avec la condition initiale x1 (0) = 3, x2 (0) = 3 et x3 (0) = 0. Ce système est très facile à
résoudre et nous obtenons
1 3
x1 (t) =(3 + 3t + t3 + t4 + t5 ) e−2t ,
4 20
3
x2 (t) =(3 + t3 + t4 )e−2t ,
4
3 −2t
x3 (t) =3t e .

En revenant à X,
1 3
x(t) =(3t + t3 − t4 + t5 )e−2t ,
2 20
1 4 3 5 −2t
y(t) =(3 + 3t + t + t )e ,
4 20
1 3
z(t) =(3t − t4 + t5 )e−2t .
2 20

Solution .7 Posons ω = k Ω k et supposons que ω 6= 0 car le cas ω = 0 est trivial.


1
Posons e1 = Ω, et choisissons e2 un vecteur unitaire orthogonal à e1 . Posons enfin
ω
e3 = e1 ∧ e2 .
E = (e1 , e2 , e3 ) est une base orthonormée directe de IR3 . On note (θ1 , θ2 , θ3 ) les
composantes de Θ sur la base E.
Solutions 31

La solution cherchée s’exprime sur la base E: V (t) = x1 (t)e1 + x2 (t)e2 + x3 (t)e3 , et le


système est équivalent à

x01 = θ1 , x02 = −ωx3 + θ2 , x03 = ωx2 + θ3 .

avec les conditions initiales x1 (0) = x2 (0) = x3 (0) = 0.


θ2 − x02
Donc x1 (t) = θ1 t, et comme x3 = , alors l’équation x03 = ωx2 + θ3 avec les
ω
conditions initiales sont équivalentes à

x002 + ω 2 x02 + ωθ3 = 0, x2 (0) = 0, x02 (0) = θ2

ce qui donne
1
x2 (t) = (θ2 sin(ωt) − θ3 (1 − cos(ωt))
ω
1
x3 (t) = (θ3 sin(ωt) + θ2 (1 − cos(ωt)).
ω
Enfin,
sin(ωt) 1 − cos(ωt)
V (t) = θ1 te1 + (θ2 e2 + θ3 e3 ) + (θ2 e3 − θ3 e2 ).
ω ω
hΘ, Ωi hΘ, Ωi 1
Mais, θ1 e1 = 2
Ω, θ2 e2 + θ3 e3 = Θ − 2
Ω, et θ2 e3 − θ3 e2 = Ω ∧ Θ. Alors
ω ω ω
µ ¶
sin(ωt) hΘ, Ωi sin(ωt) 1 − cos(ωt)
V (t) = t − 2
Ω+ Θ+ Ω ∧ Θ.
ω ω ω ω2

Solution .8 Posons P (λ) = λ4 − a3 λ3 − a2 λ2 − a1 λ − a0 ∈ IR[X]. La fonction


t 7→ tet sin t est solution de x(4) = a3 x000 + a2 x00 + a1 x0 + a0 x si, et seulement si, 1 + i
est une racine double de P . Mais P est à coefficients réels, donc si 1 + i est une racine
double de P , il en sera de même pour 1 − i, et donc (λ − 1 − i)2 (λ − 1 + i)2 divise P . En
comparant les coefficients de X 4 nous obtenons P (λ) = (λ − 1 − i)2 (λ − 1 + i)2 =
λ4 − 4λ3 + 8λ2 − 8λ + 4.
Alors, a0 = −4, a1 = 8, a2 = −8, a3 = 4.

Solution .9 La solution générale de y 00 +2y 0 +y = 0 est de la forme x 7→ axe−x +be−x .


La fonction x 7→ x − 2 est une solution particulière de y 00 + 2y 0 + y = x sur IR+ . En
tenant compte des conditions initiales nous obtenons

y(x) = (x + 2)e−x + x − 2, pour x ≥ 0.


32 Équations différentielles linéaires

D’autre part, la fonction x 7→ 2 − x est une solution particulière de y 00 + 2y 0 + y = −x


sur IR− . En tenant compte des conditions initiales nous obtenons

y(x) = −(x + 2)e−x − x + 2, pour x ≤ 0.

D’où, en récapitulant,
µ ¶
−x |x|
y(x) = (1 + e ) | x | − 2 th , pour x ∈ IR.
2

P
Solution .10 1◦ . Supposons que an tn soit une série entière dont la somme est
solution de (E). Alors

X ∞
X ∞
X ∞
X
n(n − 1)an tn − (n + 1)nan+1 tn + 3 (n + 1)an+1 tn − 6 a n tn = 0
n=0 n=0 n=0 n=0

ou bien,

X
(n − 3)((n + 2)an − (n + 1)an+1 )tn = 0.
n=0

On conclut,
an+1 an
= , pour n ∈ IN \ {3}.
n+2 n+1
Donc an = (n + 1)a0 si n ≤ 3 et an = (n + 1)a4 /5 si n ≥ 4. Alors (E) admet
3
X
deux solutions développables en série entière ; la première ϕ(t) = (k + 1)tk de rayon
t=0

X
de convergence +∞, et la seconde Ψ(t) = (k + 1)tk de rayon de convergence 1.
t=0
1
Remarquons que pour t ∈] − 1, 1[ nous avons Ψ(t) = .
(1 − t)2
1
Dans la suite nous posons ψ(t) = pour t ∈ IR \ {1}.
(1 − t)2
D’autre part, si I est un intervalle de IR alors SI (E) désigne l’espace vectoriel des
solutions de (E) sur I.
2◦ . Sur chacun des intervalles I1 =] − ∞, 0[, I2 =]0, 1[ et I3 =]1, +∞[ l’équation
3 6
(E) s’écrit x00 + x0 − = 0. Les résultats généraux montrent que, pour
t(t − 1) t(t − 1)
k ∈ {1, 2, 3}, SIk (E) est de dimension 2 et admet {ϕ|Ik , ψ|Ik } pour base.
3◦ . Soit J1 =] − ∞, 1[= I1 ∪ {0} ∪ I2 . Définissons

 0 si t ∈]0, 1[
ϕ1 = ϕ|J1 , ϕ2 = ψ|J1 , ϕ3 =

ψ(t) − ϕ(t) si t≤0
Solutions 33

Il est immédiat de vérifier que (ϕ1 , ϕ2 , ϕ3 ) sont des solutions linéairement indépendantes
de SJ1 (E). Inversement, si x ∈ SJ1 (E) alors x|I1 ∈ SI1 (E) et x|I2 ∈ SI2 (E), donc pour
t ∈ J1 \ {0}, 
 αϕ(t) + βψ(t) si t ∈]0, 1[
x(t) =

(α + δ)ϕ(t) + (β + γ)ψ(t) si t<0
Mais x doit être de classe C 2 sur J1 ce qui est équivalent à la condition γ + δ = 0, donc
x = αϕ1 + βϕ2 + γϕ3 . Alors {ϕ1 , ϕ2 , ϕ3 } est une base de SJ1 (E) et dim SJ1 (E) = 3.
Soit J2 =]0, +∞[= I2 ∪ {1} ∪ I3 . Il est immédiat de vérifier que ϕ4 = ϕ|J2 est une
solution de SJ2 (E). Inversement, si x ∈ SJ2 (E) alors x|I2 ∈ SI2 (E) et x|I3 ∈ SI3 (E),
donc pour t ∈ J2 \ {1},

 αϕ(t) + βψ(t) si t ∈]0, 1[
x(t) =

δϕ(t) + γψ(t) si t>1

Mais x doit être de classe C 2 sur J2 ce qui est équivalent à la condition γ = β = 0 et


α = δ donc x = αϕ4 . Alors
½ {ϕ4 } est une base de SJ2 (E) et dim SJ2 (E) = 1.
0 si t > 0
Enfin, posons ϕ5 (t) = . Il est immédiat de vérifier que (ϕ, ϕ5 )
ψ(t) − ϕ(t) si t ≤ 0
sont des solutions linéairement indépendantes de SIR (E). Inversement, si x ∈ SIR (E)
alors x|I1 ∈ SI1 (E) et x|J2 ∈ SJ2 (E), donc pour t ∈ IR \ {0},

 αϕ(t) si t > 0
x(t) =

(α − β)ϕ(t) + γψ(t) si t < 0

Mais x doit être de classe C 2 sur IR ce qui est équivalent à la condition γ = β, donc
x = αϕ + βϕ5 . Alors {ϕ, ϕ5 } est une base de SIR (E) et dim SIR (E) = 2.

Solution .11 Appelons (E) l’équation différentielle y 00 + ay 0 + by = 0.


1◦ . Remarquons que ϕ2 est solution de (E) si, et seulement si, pour tout t ∈ J,

(tϕ001 (t) + 2ϕ01 (t)) + a(t)(ϕ1 (t) + tϕ01 (t)) + b(t)tϕ1 (t) = 0

ce qui est équivalent à 2ϕ01 + aϕ1 = 0.


En dérivant la relation précédente nous obtenons 2ϕ001 + a0 ϕ1 + aϕ01 = 0. Alors
a 1 2
ϕ01 = − ϕ1 , ϕ001 = (a − 2a0 )ϕ1 .
2 4
En utilisant le fait que ϕ1 est solution de (E) nous obtenons

2a0 + a2 = 4b. (∗)


34 Équations différentielles linéaires

Inversement, supposons (∗) vérifiée. Soit A une primitive de a. Nous définissons alors
1
ϕ1 (t) = exp(− A(t)) et ϕ2 (t) = tϕ1 (t), et nous vérifions directement que ϕ1 et ϕ2 sont
2
des solutions de (E).
2◦ . Nous avons a(t) = tg t et b(t) = 21 + 34 tg 2 t pour t ∈] − π/2, π/2[. Clairement
√ √
(∗) est vérifiée, donc ϕ1 (t) = cos t et ϕ2 (t) = t cos t sont deux solutions linéairement
1 3
indépendantes de l’équation homogène y 00 + tg ty 0 + ( + tg 2 t)y = 0 sur ] − π/2, π/2[.
2 4
Utilisons la méthode de variation de la constante pour chercher une solution particulière
ϕ(t) = c(t)ϕ1 (t) + d(t)ϕ2 (t) de l’équation non homogène. Nous avons

c0 (t) + td0 (t) = 0, −c0 (t) sin t + d0 (t)(2 cos t − t sin t) = 2 cos2 t

donc c0 (t) = −t cos t, d0 (t) = cos t. Ce qui donne, pour t ∈] − π/2, π/2[,

c(t) = −t sin t − cos t, d(t) = sin t.

On en déduit la solution particulière

ϕ(t) = c(t)ϕ1 (t) + d(t)ϕ2 (t) = −(cos t)3/2 .

La solution générale de l’équation est



y(t) = (c + dt − cos t) cos t.

Solution .12 Appelons (E) l’équation différentielle y 00 + ay 0 + by = 0.


1◦ . La fonction t 7→ ψ(t) = exp(iθ(t)) est, d’après l’hypothèse, solution de (E),
donc
(b − θ02 ) + i(θ00 + aθ0 ) = 0

soit b = θ02 , et θ00 + aθ0 = 0. En dérivant la première relation et en utilisant la seconde


nous obtenons
b0 + 2ab = 0
b ≥ 0.et (∗)
p
Inversement, En prenant θ une primitive de t 7→ b(t), nous vérifions que t 7→ cos(θ(t))
et t 7→ sin(θ(t)) sont deux solutions linéairement indépendantes de (E).
2◦ . – Pour l’équation (E): cos ty 00 + sin ty 0 + cos3 ty = 0 sur ] − π/2, π/2[, nous
avons a(t) = tg t, b(t) = cos2 t, et la relation (∗) est vérifiée. Un calcul simple montre
que θ(t) = sin t et puis ϕ1 (t) = cos(sin t), ϕ2 (t) = sin(sin t) sont deux solutions qui
forment une base de l’espace des solutions de (E).
Solutions 35

– Pour l’équation (E): ty 00 − y 0 + t3 y = t3 sin(t2 /2) sur IR∗+ , nous avons a(t) = 1/t,
b(t) = t2 , et la relation (∗) est vérifiée. Un calcul simple montre que θ(t) = t2 /2 et
puis ϕ1 (t) = cos(t2 /2), ϕ2 (t) = sin(t2 /2) sont deux solutions qui forment une base de
l’espace des solutions de l’équation homogène associée à (E).
Utilisons la méthode de variation de la constante pour chercher une solution particulière
ϕ(t) = c(t)ϕ1 (t) + d(t)ϕ2 (t) de l’équation non homogène. Nous avons

c0 (t) cos(t2 /2) + d0 (t) sin(t2 /2) = 0, −c0 (t) sin(t2 /2) + d0 (t) cos(t2 /2) = t sin(t2 /2)

t cos t2 − t 0 t sin t2
donc c0 (t) = , d (t) = . Ce qui donne, pour t ∈ IR∗+ ,
2 2

2 sin t2 − t2 1 − cos t2
c(t) = , d(t) = .
4 2

On en déduit la solution particulière

t2
ϕ(t) = c(t)ϕ1 (t) + d(t)ϕ2 (t) = − cos(t2 /2).
4

La solution générale de l’équation (E) est

t2 t2 t2
y(t) = c sin + (d − ) cos .
2 4 2

Solution .13 Soit I l’un des intervalles ]−∞, −1[, ]−1, 1[ et ]1, +∞[. Il est immédiat
que y1 (x) = x est une solution particulière de l’équation sur I. On cherche l’autre
solution sous la forme x 7→ y(x) = xψ(x). En remplaçant dans l’équation nous arrivons

∀ x ∈ I, x(x2 − 1)ψ 00 (x) − (2 − 3x2 )ψ 0 (x) = 0.
1
Ce qui donne en intégrant ψ 0 (x) = p , pour tout x ∈ I différent de 0.
x2 | x2 − 1 |
Une deuxième intégration nous permet de trouver ψ et puis y. D’où sur ] − ∞, −1[ les

fonctions {x 7→ x, x 7→ x2 − 1} forment une base de l’espace des solutions.

Sur ] − 1, +1[ les fonctions {x 7→ x, x 7→ 1 − x2 } forment une base de l’espace des
solutions.

Sur ]1, +∞[ les fonctions {x 7→ x, x 7→ x2 − 1} forment une base de l’espace des
solutions.
36 Équations différentielles linéaires

Solution .14 Une méthode plus simple que celle suggérée par l’énoncé pour
résoudre l’équation différentielle xy 00 + 2y 0 + ω 2 xy = 0 consiste à remarquer que
(xy(x))00 = xy 00 + 2y 0 . Si l’on pose z(x) = xy(x), alors z est solution de l’équation
z 00 + ω 2 z = 0. Alors une base de l’espace des solutions de xy 00 + 2y 0 + ω 2 xy = 0 sur tout
cos ωx sin ωx
intervalle I ne contenant pas zéro est donnée par {x 7→ , x 7→ }.
x x

Solution .15 Supposons que l’équation x2 y 00 + xy 0 + (x2 − 4)y = 1 admette une


X∞
solution développable en série entière au voisinage de 0: S(x) = an xn alors en
n=0
remplaçant,

X ∞
X
2 n
(n − 4)an x + an−2 xn = 1.
n=0 n=2

En comparant le coefficient constant nous obtenons 1 = −4a0 et en faisant de même


pour le coefficient de x2 nous obtenons a0 = 0. C’est une contradiction qui démontre
que l’équation x2 y 00 + xy 0 + (x2 − 4)y = 1 n’admet pas de solution développable en série
entière au voisinage de 0.

Solution .16 En résolvant par rapport à x00 et y 00 le système considéré, nous arrivons
au système équivalent
½
x00 = 2x0 + y0 − 2x − 2y
(S) :
y 00 = −2x0 − y0 + x + y

Nous remarquons alors que les combinaisons u = x + y et v = 2x + y jouent un rôle


particulier dans le système précédent. En notant que x = v − u et y = 2u − v nous
obtenons le système suivant, qui est équivalent à (S),

u00 + u = 0, v 00 − v 0 = −3u.

Ce qui donne
u(t) = A cos t + B sin t.

En reportant dans la seconde équation et en intégrant une fois nous obtenons

v 0 (t) − v(t) = −3A sin t + 3B cos t + C

ou bien,
(v(t)e−t )0 = (C + 3B cos t − 3A sin t)e−t .

Enfin,
3 3
v(t) = Det − C + (A + B) sin t + (A − B) cos t.
2 2
Solutions 37

Revenant à x et y, nous obtenons,

3A + B A − 3B
x(t) = Det − C + sin t + cos t
2 2
B − 3A A + 3B
y(t) = − Det + C + sin t + cos t.
2 2

Solution .17 Les équations considérées s’appellent des équations d’Euler. Le


changement de variable x = et ou x = −et les transforme en équations linéaires à
coefficients constants.
– L’équation x2 y 00 + xy 0 + y = cos(αLog x) sur IR∗+ .
Effectuons le changement de variable x(t) = et et posons Y (t) = y(et ). Nous avons

dY d2 Y
Ẏ (t) = (t) = x(t)y 0 (x(t)), Ÿ (t) = 2
(t) = Ẏ (t) + x2 (t)y 00 (x(t)).
dt dt

L’équation considérée est équivalente à

Ÿ + Y = cos(αt),

ce qui donne
cos(αt)
Y (t) = A cos t + B sin t + si α 6= 1
1 − α2
t
Y (t) = A cos t + B sin t − sin t si α = 1
2
En revenant à x nous obtenons

cos(αLog x)
y(x) = A cos(Log x) + B sin(Log x) + si α 6= 1
1 − α2
Log x
y(x) = A cos(Log x) + B sin(Log x) − sin(Log x) si α = 1
2

– L’équation x2 y 00 + 4xy 0 + 2y = Log (1 + x2 ).


si I est l’un des deux intervalles ] − ∞, 0[ et ]0, +∞[, alors nous remarquons que les
deux fonctions définies sur I par x 7→ 1/x et x 7→ 1/x2 sont deux solutions linéairement
indépendantes de l’équation homogène associée. En utilisant la méthode de variation
de la constante nous obtenons la solution suivante sur I,

A B 3 Arctg x x2 − 1
y(x) = + 2 − +2 + Log (1 + x2 ).
x x 2 x 2x2
38 Équations différentielles linéaires

Solution .18 1◦ . En substituant la série entière définissant φ dans l’équation nous


obtenons

X ∞
X ∞
X ∞
X ∞
X
n n n n
n(n − 1)an x + nan x − an x − an−1 x − an−2 xn = 0
n=0 n=0 n=0 n=1 n=2

ce qui donne

X ¡ 2 ¢
−a0 − a0 x + (n − 1)an − an−1 − an−2 xn = 0.
n=2

et en tenant compte de la condition a1 = φ0 (0) = 1, nous obtenons

an−1 + an−2
a0 = 0, a1 = 1, an = , pour n ≥ 2. (∗)
n2 − 1

La suite (an )n∈IN est uniquement déterminée par les conditions précédentes, d’où
l’unicité de φ si elle existe.
Considérons la suite déterminée par (∗), et remarquons que a1 = 1 et a2 = 1/3, donc
1 1
| an | ≤ pour n = 1 et n = 2. Soit n ≥ 3, supposons l’inégalité | ak | ≤
(n − 1)! (k − 1)!
vraie pour tout k < n alors
µ ¶
1 1 1
| an | ≤ 2 +
n − 1 (n − 2)! (n − 3)!
µ ¶
1 1 (n − 1)(n − 2)
= +
(n − 1)! n + 1 n2 − 1
n−1 1 1
= < .
n + 1 (n − 1)! (n − 1)!

1
La suite (an )n∈IN déterminée par (∗) vérifie | an | ≤ pour tout n ≥ 1, alors le
P (n − 1)!
rayon de convergence de la série entière an xn est +∞. On vérifie immédiatement
X∞
que la fonction définie sur IR par φ(x) = an xn est la solution développable en série
n=0
entière qui vérifie φ0 (0) = 1 de l’équation considérée.
2◦ . En effectuant le changement de fonction inconnue y = ze−x /x, nous obtenons

xz 00 = (2x + 1)z 0

une intégration simple montre que z 0 (x) = 4Axe2x puis z(x) = A(2x − 1)e2x + B.
La solution générale de l’équation considérée sur tout intervalle ne contenant pas 0 est
donnée par
ch x sh x
y(x) = 2Aex + (B − A) − (A + B) .
x x
Solutions 39

On conclut qu’il existe (A1 , B1 , A2 , B2 ) ∈ IR4 tel que


 ch x sh x
 x
 2A1 e + (B1 − A1 ) − (A1 + B1 ) si x > 0
x x
φ(x) =

 2A ex + (B − A ) ch x − (A + B ) sh x
2 2 2 2 2 si x < 0
x x

La continuité de φ en 0 montre que A1 = B1 et A2 = B2 . D’autre part, la dérivabilité de


φ en zéro montre aussi que A1 = A2 . Enfin, le fait que φ0 (0) = 1 montre que 2A1 = 1.
sh x
Il en résulte que φ(x) = ex − .
x

Solution .19 Notons que, par une récurrence simple sur n, nous avons [G n ]0 =
X 1
nGG n−1 . La série [G n ]0 converge alors normalement sur tout intervalle compact
n!
contenu dans I. Par conséquent,
̰ !0
d X 1
n
exp(G(t)) = G (t)
dt n=0
n!
X∞ ∞
1 n 0 X n
= [G (t)] = G(t)G n−1 (t)
n=0
n! n=1
n!
X∞
1 n
=G(t) G (t) = G(t) exp(G(t)) (∗)
n=10
n!

L’unicité de la solution de X 0 (t) = G(t)X(t) vérifiant X(t0 ) = x0 résulte des théorèmes


généraux. Le fait que X(t) = exp(G(t))x0 est une solution du problème est immédiat
d’après (∗).

Solution .20 Posons Z = x + y et U = x − y. Le système considéré est équivalent à

1 1
Z 0 (t) = (2t − )Z(t), U 0 (t) = (2t + )U (t)
1 + t2 1 + t2

Alors, Z(t) = 2A exp(t2 − Arctg t), et U (t) = 2B exp(t2 + Arctg t).


En revenant à x et y nous obtenons,

2
x(t) = et (A exp(−Arctg t) + B exp(Arctg t)),

2
y(t) = et (A exp(−Arctg t) − B exp(Arctg t)).
40 Équations différentielles linéaires

Solution .21 1◦ . C’est trivial.


2◦ . Si (x, y) est une solution linéairement indépendante de (1, t) de l’équation
homogène, alors en considérant le Wronskien, nous avons, quitte à bien choisir la
condition initiale,
µZ ¶
2t
y(t) − tx(t) = exp dt = 1 + t2 .
1 + t2

En remplaçant, dans la deuxième équation du système homogène nous obtenons


x0 (t) = 1/t2 donc nous pouvons prendre x(t) = −1/t et puis y(t) = t2 . Une base
de l’espace des solutions du système homogène est alors donnée par
· ¸ · ¸
1 −1/t
Φ(t) = , Ψ(t) = .
t t2

La méthode de variation de la constante permet de · trouver


¸ une solution particulière du
1
système avec second membre, à savoir t 7→ Log t. . La solution générale du système
t
est donc

B
x(t) =A − + Log t
t
y(t) =At + Bt2 + t Log t

Solution .22 1◦ . Cherchons les solutions du système homogène sous la forme


x(t) = at2 + bt + c et y(t) = dt2 + et + f . Une substitution dans le système et une
identification nous donnent d = 0, b = f, a = e = c. Alors, une base de l’espace des
solutions du système homogène est alors donnée par
· ¸ · ¸
t 1 + t2
Φ(t) = , Ψ(t) = .
1 t

La méthode de variation de la constante permet


· ¸de trouver une solution particulière du
t
système avec second membre, à savoir t 7→ 2 . La solution générale du système est
t
donc

x(t) =A(1 + t2 ) + Bt + t
y(t) =At + B + t2

OKMRAN
OUBA
FONCTIONS DE PLUSIEURS VARIABLES

Dans ce chapitre, pour chaque entier naturel non nul n, l’espace IRn est considéré comme
un espace vectoriel normé pour une certaine norme k · k. L’équivalence des normes sur
IRn nous permet de ne pas préciser cette norme que dans quelques rares cas.
Nous appelons fonction de n variables une application définie sur une partie non
vide de IRn et nous disons qu’une fonction est numérique si elle prend ses valeurs dans
IR. Sauf mention contraire, toutes les fonctions sont à valeurs dans un espace IRm . Si
A est une partie non vide de IRn et f : A −→ IRm est une fonction de n variables,
nous appellerons fonctions coordonnées de f , les fonctions f1 = p1 ◦f ,..., fm = pm ◦f
où p1 , . . . , pm sont les projections canoniques de IRm sur IR. On a alors, pour chaque
élément x de A, f (x) = (f1 (x), . . . , fm (x)).

I. Continuité et limites

Commençons par rappeler les propriétés des fonctions continues de plusieurs variables,
déjà démontrées dans le chapitre sur les espaces vectoriels normés.
♣ Si f est définie sur une partie non vide A de IRn , et si a ∈ A, alors f est
continue au point a si, et seulement si x→a
lim f (x) existe.
x∈A
♣ La composée de deux fonctions continues est continue.
♣ Une fonction est continue en un point si, et seulement si, toutes ses fonctions
coordonnées sont continues en ce point.
♣ Si f est une fonction numérique, définie et continue sur une partie compacte
(i.e. fermée et bornée) A de IRn , alors f est bornée sur A et y atteint ses bornes.
♣ Si f est une fonction, définie et continue sur une partie compacte (i.e. fermée
et bornée) A de IRn , alors f est uniformément continue sur A.
2 Fonctions de plusieurs variables

Définition : Soit f : A −→ IRm une fonction de n variables et soit a = (a1 , . . . , an )


un élément de A. Pour i ∈ {1, . . . , n}, on pose

Aai = {xi ∈ IR : (a1 , . . . , ai−1 , xi , ai+1 , . . . , an ) ∈ A} ,

et on définit l’application partielle

ϕai : Aai −→ IRm , xi 7→ ϕai (xi ) = f (a1 , . . . , ai−1 , xi , ai+1 , . . . , an ).

On dit que f est continue par rapport à sa iième variable, ou par rapport à xi , au point
a si, et seulement si, la fonction ϕai est continue au point ai .
La proposition suivante est immédiate:

Proposition I.1 Si une fonction de n variables est continue en un point, alors elle est
continue par rapport à chacune de ses variables en ce point.

5
! Remarque : La réciproque de la proposition I.1 est fausse. Soit par exemple
 3 3
 xy + x y si (x, y) 6= (0, 0)

f : IR2 −→ IR, f (x, y) = x4 + y 4


0 si (x, y) = (0, 0).

La fonction f est continue par rapport à chacune de ses variables au point a = (0, 0), car
on a ϕa1 (x) = f (x, 0) = 0 pour x ∈ IR∗ et ϕa2 (y) = f (0, y) = 0 pour y ∈ IR∗ . Cependant,
la fonction f n’est pas continue au point (0, 0) car
1 1
lim f ( , ) = 1 6= f (0, 0).
n→∞ n n

II. Différentiabilité

Lemme II.1. Soit A un ouvert non vide de IRn , f une application de A dans IRm et
a ∈ A. On suppose qu’il existe une application linéaire u : IRn −→ IRm telle que
k f (a + h) − f (a) − u(h) k
lim = 0. (¶)
h→0 khk
Alors u est unique.
Différentiabilité 3

Preuve : Car si h ∈ IRn alors la relation (¶) montre que


1
u(h) = lim [f (a + th) − f (a)] .
>
t→0
t

Ce qui démontre l’unicité.

Le lemme précédent nous permet de donner la définition suivante:

Définition : Soient A un ouvert non vide de IRn , f une application de A dans IRm et
a ∈ A. On dit que f est différentiable au point a si, et seulement si, il existe une
application linéaire u : IRn −→ IRm telle que
k f (a + h) − f (a) − u(h) k
lim = 0.
h→0 khk
L’unique application linéaire u qui vérifie cette condition est appelée la différentielle
de f au point a et on la note dfa .
Si f est différentiable au point a, il existe donc une fonction ε définie dans un
voisinage W de 0 ∈ IRn , à valeurs dans IRm , telle que

∀ h ∈ W, f (a + h) = f (a) + dfa (h) + k h k ε(h),

et lim ε(h) = 0.
h→0
Si f est différentiable en tout point de A on dit qu’elle différentiable sur A. On
note alors df l’application de A dans l’espace vectoriel L(IRn , IRm ), des applications
linéaires de IRn dans IRm , qui à x fait correspondre l’application linéaire dfx .

Exemples :
♣ Si f : A −→ IRm est une fonction constante sur un ouvert A de IRn . Alors f
est différentiable sur A et dfx = 0 pour tout x ∈ A.
♣ Si I est un intervalle ouvert non vide de IR et f : I −→ IR est une fonction
dérivable au point a ∈ I. Alors elle est différentiable en a et dfa (h) = f 0 (a)h.
♣ Si f : IRn −→ IRm est une application linéaire alors, pour tout a ∈ IRn , on a
dfa = f . Car f (a + h) − f (a) − f (h) = 0 = k h k ε(h).
♣ Si B : IRn × IRn −→ IRm est une application bilinéaire alors,pour tout
(a, b) ∈ IRn × IRn , on a dB(a,b) (h, k) = B(a, k) + B(h, b). En effet, si

εe(h, k) = B(a + h, b + k) − B(a, b) − B(a, k) − B(h, b)


4 Fonctions de plusieurs variables

alors d’après la bilinéarité de B nous avons εe(h, k) = B(h, k) et en utilisant la


continuité de B, nous obtenons

k εe(h, k) k ≤ α k h k k k k ≤ α(k h k + k k k)2 .

Ce qui démontre que εe(h, k) = k (h, k) k ε(h, k) avec lim ε(h, k) = 0. D’où
(h,k)→(0,0)
le résultat.
♣ Si h·, ·i : IRn × IRn −→ IR est un produit scalaire sur IRn et si Q : IRn −→ IR
est la forme quadratique associée alors Q est différentiable sur IRn et pour tout
a ∈ IRn on a
∀ h ∈ IRn , dQa (h) = 2hh, ai.

Les deux propositions suivantes sont immédiates

Proposition II.2. Soient A un ouvert non vide de IRn et f une application de A dans
IRm , différentiable en a ∈ A. Alors elle est continue en a.

Proposition II.3. Soient A un ouvert non vide de IRn , f et g deux applications de A


dans IRm , différentiables en a ∈ A. Alors
♣ f + g est différentiable au point a et d(f + g)a = dfa + dga .
♣ Si λ ∈ IR, λf est différentiable au point a et d(λf )a = λdfa .

Théorème II.4. Soient A un ouvert non vide de IRn , B un ouvert non vide de IRm ,
g : A −→ IRm et f : B −→ IRp deux applications avec g(A) ⊂ B. On suppose que g
est différentiable au point a ∈ A et que f est différentiable au point g(a). Alors f ◦g est
différentiable au point a et d(f ◦g)a = dfg(a) ◦dga .

Preuve : En effet, il existe une fonction ϕ définie sur un voisinage V de 0 ∈ IRn , telle
que
∀ h ∈ V, g(a + h) = g(a) + dga (h) + k h k ϕ(h), et lim ϕ(h) = 0
h=0

et une fonction ψ définie sur un voisinage W de 0 ∈ IRm , telle que

∀ k ∈ W, f (g(a) + k) = f (g(a)) + dfg(a) (k) + k k k ψ(k), et lim ψ(k) = 0


k=0

Posons k(h) = dga (h) + k h k ϕ(h) pour h ∈ V . Nous avons

k k(h) k ≤ k h k (k dga k + k ϕ(h) k) .


Différentiabilité 5

Nous concluons qu’il existe V 0 un nouveau voisinage de 0 ∈ IRn tel que

h ∈ V 0 =⇒ k(h) ∈ W.

Mais, pour h ∈ V 0 , g(a + h) = g(a) + k(h) et par conséquent

∀ h ∈ V 0 \ {0}, f (g(a + h)) =f (g(a)) + dfg(a) (k(h)) + k k(h) k ψ(k(h))

=f (g(a)) + dfg(a) (dga (h)) + k h k ε(h)

k k(h) k
avec ε(h) = dfg(a) (ϕ(h)) + ψ(k(h)). D’où
khk
° °
k ε(h) k ≤ ° dfg(a) ° k ϕ(h) k + (k dga k + k ϕ(h) k) k ψ(k(h)) k ,

et lim ε(h) = 0. Nous concluouns que f ◦g est différentiable en a et que


h→0

d(f ◦g)a = dfg(a) ◦dga .

La proposition suivante est facile à démontrer. Nous laissons sa démonstration en


exercice au lecteur.

Proposition II.5. Soient A un ouvert non vide de IRn , f : A −→ IRm et g : A −→ IRp


deux applications. On pose

Ψ : A −→ IRm+p , x 7→ (f (x), g(x)).

Alors Ψ est différentiable au point a ∈ A si, et seulment si, f et g sont différentiables


au point a. et dΨa (h) = (dfa (h), dga (h)).

Corollaire II.6. Soient A un ouvert non vide de IRn et f : A −→ IRm une application.
Alors f est différentiable au point a ∈ A si, et seulment si, les fonctions coordonnées
f1 , . . . , fm de f sont différentiables au point a, et

dfa (h) = (d(f1 )a (h), . . . , d(fm )a (h)).


6 Fonctions de plusieurs variables

Théorème II.7. Soient f un homéomorphisme d’une partie ouverte A de IRn sur une
partie ouverte B de IRn et a un élément de A. On suppose que f est différentiable au
point a et que dfa est un automorphisme de IRn . Alors l’application réciproque f −1 est
différentiable au point f (a) ∈ B et on a

d(f −1 )f (a) = (dfa )−1 .

Preuve : Étudions d’abord le cas particulier suivant:


– On suppose que 0 ∈ A, f (0) = 0 et df0 = I où I est l’application identité de IRn .
Soit ε ∈]0, 1[ et δ = ε/(1 + ε). La différentiabilité de f en 0 montre qu’il existe
η1 > 0 tel que
k h k < η1 =⇒ k f (h) − h k ≤ δ k h k .

La continuité de f −1 en 0 montre qu’il existe η2 > 0 tel que

° °
k k k < η2 =⇒ ° f −1 (k) ° ≤ η1 .

On conclut que

° ° ° °
k k k ≤ η2 =⇒ ° f (f −1 (k)) − f −1 (k) ° ≤ δ ° f −1 (k) ° .

soit, ° ° ° °
k k k ≤ η2 =⇒ ° f −1 (k) − k ° ≤ δ ° f −1 (k) − k + k °
° −1 ° ° °
° f (k) − k ° ≤ δ ° f −1 (k) − k ° + δ k k k
° −1 ° δ
° f (k) − k ° ≤ kkk = εkkk.
1−δ
Ce qui démontre, ° −1 °
° f (k) − f −1 (0) − k °
lim = 0.
k→0 kkk
C’est à dire que f −1 est différentiable en 0 et que d(f −1 )0 = I.
e = A − {a}, B
– Venons au cas général. Notons A e = (dfa )−1 (B − {f (a)}) et posons,
e fe(x) = (dfa )−1 (f (x + a) − f (a)). Il est immédiat de voir que fe est un
pour x ∈ A,
e de IRn sur la partie ouverte B
homéomorpfisme de la partie ouverte A e de IRn . De plus,
e fe(0) = 0 et fe est différentiable en 0 avec dfe0 = I. En appliquant le cas paticulier
0 ∈ A,
à fe nous voyons que fe−1 est différentiable en 0 avec d(fe−1 )0 = I.
Dérivées partielles 7

D’autre part, si λ(x) = (dfa )−1 (x − f (a)) pour x ∈ IRn , alors λ est différentiable
sur IRn et dλx = (dfa )−1 pour tout x ∈ IRn .
La différentiabilité de fe−1 en 0 implique alors la différentiabilité de la fonction
h = fe−1 ◦λ, qui est définie seulement dans un voisinage de f (a), en f (a), avec

dhf (a) = d(fe−1 )0 ◦dλf (a) = (dfa )−1 .

Mais h(x) = f −1 (x) − a. Donc f −1 est différentiable en f (a) et d(f −1 )f (a) = (dfa )−1 .

III. Dérivées partielles

Soient A un ouvert non vide de IRn , et f : A −→ IR une fonction numérique de n


variables et soit a = (a1 , . . . , an ) un élément de A. Pour i ∈ {1, . . . , n}, nous rappelons
la notation

Ai = {xi ∈ IR : (a1 , . . . , ai−1 , xi , ai+1 , . . . , an ) ∈ A} ,

et

ϕi : Ai −→ IR, xi 7→ ϕi (xi ) = f (a1 , . . . , ai−1 , xi , ai+1 , . . . , an ).

Supposons que f est différentiable au point a, alors il existe un voisinage V de 0 ∈ IRn


tel que

∀ h ∈ V, f (a + h) = f (a) + dfa (h) + k h k ε(h),

et lim ε(h) = 0. Soit (e1 , . . . , en ) la base canonique de IRn . En prenant h = tei nous
h→0
déduisons de ce qui précède que, pour | t | assez petit,

ϕi (ai + t) = ϕi (ai ) + tdfa (ei ) + | t | εe(t),

avec lim εe(t) = 0. Ce qui démontre que ϕi est dérivable au point ai ∈ Ai et que
t→0
0
ϕi (ai ) = dfa (ei ).
8 Fonctions de plusieurs variables

Définition : Soit f une fonction numérique définie sur une partie ouverte non vide
A de IRn et a ∈ A. Si j est un entier tel que 1 ≤ j ≤ n, on dit que f est dérivable
par rapport à sa j ième variable au point a si, et seulement si, la fonction ϕj définie
précédemment est dérivable au point aj . Le nombre ϕ0j (aj ), lorsqu’il existe, est appelé
∂f
la j ième dérivée partielle de f au point a et on le note Dj f (a) ou (a), ou encore
∂xj
fx0 j (a).
Si f admet une dérivée partielle par rapport à sa j ième variable en chaque point de
∂f
A, on note Dj f ou la fonction de A dans IR qui à a = (a1 , . . . , an ) associe ϕ0j (aj ).
∂xj

Notation : Si j ∈ {1, . . . , n}, on note dxj l’application linéaire

dxj : IRn −→ IR, h = (h1 , . . . , hn ) 7→ dxj (h) = hj .

En fait, (dx1 , . . . , dxn ) est une base de (IRn )∗† qui est la base duale de la base canonique
de IRn .

Proposition III.1. Soit f une fonction numérique définie sur une partie ouverte non
vide A de IRn et a ∈ A. Si f est différentiable au point a, alors elle admet des dérivées
partielles par rapport à chacune de ses variables au point a, et on a
n
X ∂f
dfa = (a)dxk .
∂xk
k=1

Preuve : En effet, nous avons vu que si (e1 , . . . , en ) est la base canonique de IRn alors,
Xn
∂f n
pour tout j ∈ {1, . . . , n}, on a dfa (ej ) = (a). Mais si h ∈ IR on a h = dxk (h) ek
∂xj
k=1
et par conséquent
X n
dfa (h) = dxk (h) dfa (ek ),
k=1

ou bien,
n
X ∂f
dfa (h) = (a)dxk (h).
∂xk
k=1

ce qui prouve le résultat.


(IRn )∗ est le dual de IRn , c’est-à-dire l’espace vectoriel des formes linéaires sur IRn
Dérivées partielles 9

5
! Remarque : La réciproque de la proposition III.1 est fausse. C’est à dire qu’une fonction
numérique de n variables admettant des dérivées partielles par rapport à chacune de
ses variables en un point peut ne pas être différentiable en ce point. Soit par exemple
 3 3
 xy + x y

si (x, y) 6= (0, 0)
f : IR2 −→ IR, f (x, y) = x4 + y 4


0 si (x, y) = (0, 0).

Nous avons vu qu’au point a = (0, 0) on a ϕ1 (x) = f (x, 0) = 0 pour x ∈ IR∗ et


ϕ2 (y) = f (0, y) = 0 pour y ∈ IR∗ . Donc

∂f ∂f
(0, 0) = 0, et (0, 0) = 0.
∂x ∂y

Cependant, la fonction f n’est pas différentiable au point (0, 0) car elle n’est même pas
continue en ce point.

Mais nous avons le théorème suivant:


Théorème III.2. Soit f une fonction numérique définie sur une partie ouverte non
vide A de IRn et a ∈ A. On suppose que f admet, sur A, des dérivées partielles par
rapport à chacune de ses variables et que ces dérivées partielles sont continues au point
a. Alors f est différentiable au point a, et on a

n
X ∂f
dfa = (a)dxk .
∂xk
k=1

Preuve : Nous allons, pour simplifier, supposer n = 2 ce qui ne change pas l’idée de
la démonstration.
Soit ε > 0, en utilisant la continuité des dérivées partielles de f au point
a = (a1 , a2 ), on trouve η > 0 tel que
 ¯¯ ¯
¯
 ∂f ∂f
 ¯¯ ∂x (a1 + α, a2 + β) −

∂x1
(a1 , a2 ) ¯¯ <ε
1
∀ (α, β) ∈ IR2 , | α | + | β | < η =⇒ ¯ ¯

 ¯ ∂f ∂f ¯
 ¯¯ (a1 + α, a2 + β) − (a1 , a2 ) ¯¯ <ε
∂x2 ∂x2
10 Fonctions de plusieurs variables

Soit h = (h1 , h2 ) ∈ IR2 tel que k h k = | h1 | + | h2 | < η. En utilisant le théorème des


accroissements finis nous trouvons θ1 et θ2 de ]0, 1[ tels que

f (a1 + h1 , a2 + h2 ) − f (a1 , a2 ) =f (a1 + h1 , a2 + h2 ) − f (a1 + h1 , a2 )

+ f (a1 + h1 , a2 ) − f (a1 , a2 ),
∂f ∂f
=h2 (a1 + h1 , a2 + θ2 h2 ) + h1 (a1 + θ1 h1 , a2 ).
∂x2 ∂x1

Donc
¯ ¯ ¯ ¯
¯ ∂f ∂f ¯ ¯ ∂f ∂f ¯
¯ f (a + h) − f (a) − h2 (a) − h (a) ¯ ≤ | h1 | ¯ (a + θ h , a ) − (a) ¯
¯ ∂x2
1
∂x1 ¯ ¯ ∂x1 1 1 1 2
∂x1 ¯
¯ ¯
¯ ∂f ∂f ¯
¯
+ | h2 | ¯ (a1 + h1 , a2 + θ2 h2 ) − (a) ¯¯ ,
∂x2 ∂x2
≤ εkhk.

Ce qui prouve que f est différentiable au point a et que

∂f ∂f
dfa (h) = h1 (a) + h2 (a).
∂x1 ∂x2

Corollaire III.3. Soit f une fonction numérique définie sur une partie ouverte non
vide A de IRn . On suppose que f admet, sur A, des dérivées partielles par rapport à
chacune de ses variables et que ces dérivées partielles sont continues sur A. Alors f est
continuement différentiable sur A, et on a

n
X ∂f
df = dxk .
∂xk
k=1

Corollaire III.4. Soit f une fonction définie sur une partie ouverte non vide A de IRn
à valeurs dans IRmµ, on note
¶ f1 , . . . , fm ses fonctions coordonnées. On suppose que les
∂fi
dérivées partielles existent et sont continues sur A. Alors f est continuement
∂xj i,j
différentiable sur A.
Dérivées partielles 11

5
! Remarque : Les hypothèses du théorème III.2 ne sont pas nécesaires pour qu’une
fonction soit différentiable. Soit par exemple

 (x2 + y 2 ) sin p 1 si (x, y) 6= (0, 0)
f : IR2 −→ IR, f (x, y) = x2 + y 2

0 si (x, y) = (0, 0).
En munissant IR2 de la norme euclidienne usuelle, nous avons clairement,
¯ ¯
| f (x, y) − f (0, 0) | p 2 ¯ 1 ¯
¯ ¯
= x + y 2 ¯ sin p ¯ ≤ k (x, y) k .
k (x, y) k ¯ x +y ¯
2 2

Alors f est différentiable en (0, 0) et df(0,0) = 0.


Par contre, si (x, y) 6= (0, 0) on a
∂f 1 x 1
(x, y) =2x sin p −p cos p
∂x x2 + y 2 x2 + y 2 x2 + y 2
∂f 1 y 1
(x, y) =2y sin p −p cos p
∂y x2 + y 2 x2 + y 2 x2 + y 2
∂f ∂f
Les fonctions et ne sont pas continues au point (0, 0), car si t > 0 on a
∂x ∂y
∂f ∂f 1 1 1
(t, t) = (t, t) = 2t sin √ − √ cos √
∂x ∂y 2t 2 2t
et cette quantité n’a pas de limite au point 0.

Définition : Soit f une fonction numérique définie sur une partie ouverte non vide
A de IRn et a ∈ A. Soit aussi e ∈ IRn \ {0}. Si la fonction réelle de la variable réelle
f (a + te) − f (a)
t 7→ possède une limite lorsque t tend vers 0, on dit que f possède une
t
dérivée dans la direction e au point a, et on pose
f (a + te) − f (a)
De f (a) = lim .
t→0 t
Notons que si f : A −→ IR est différentiable au point a alors f admet une dérivée
dans toute direction au point a avec De f (a) = dfa (e).
D’autre part, si (e1 , . . . , en ) est la base canonique de IRn , et si f est une fonction
numérique définie sur A, alors f admet une dérivée partielle par rapport à la j ième
variable en a si, et seulement si, f admet une dérivée dans la direction ej au point a,
∂f
et (a) = Dej f (a).
∂xj
12 Fonctions de plusieurs variables

5
! Remarques :
– Si f admet des dérivées partielles par rapport à chacune de ses variables au point
a, alors cela n’implique pas qu’elle admet des dérivées dans toutes les directions au
point a. Soit par exemple
 3 3
 xy + x y

si (x, y) 6= (0, 0)
f : IR2 −→ IR, f (x, y) = x4 + y 4


0 si (x, y) = (0, 0).

∂f ∂f
Nous avons vu que (0, 0) = 0 et que (0, 0) = 0. Mais si e = (1, 1) on a
∂x ∂y

f (te) − f (0) 1
= ,
t t

et donc ceci n’a pas de limite quand t tend vers 0. f n’est pas dérivable dans la
direction e = (1, 1).
– Si f admet des dérivées dans toutes les directions au point a, alors cela n’implique
pas qu’elle est différentiable en ce point.Soit par exemple
 2
 xy

si (x, y) 6= (0, 0)
f : IR2 −→ IR, f (x, y) = x2 + y 4


0 si (x, y) = (0, 0).

Si e = (α, β) est un vecteur tel que α 6= 0, on a

f (te) − f (0) αβ 2 β2
lim = lim 2 = .
t→0 t t→0 α + t2 β 4 α

et si e = (0, β) avec β 6= 0 on a

f (te) − f (0)
lim = 0.
t→0 t

On conclut que f admet des dérivées dans toutes les directions au point (0, 0).
Mais f n’est pas différentiable au point (0, 0), car elle n’est même pas continue en
ce point:
1
lim f (y 2 , y) = 6= f (0, 0).
>
y→0
2
Dérivées partielles 13

Définition : Soit f une fonction définie sur une partie ouverte non vide A de IRn à
valeurs dans IRm et soient f1 , . . . , fm ses fonctions coordonnées. Supposons que f est
différentiable au point a ∈ A. Une généralisation immédiate du corollaire II.6 montre
que les fonctions coordonnées f1 , . . . , fm sont aussi différentiables au point a et que,
pour h = (h1 , . . . , hn ),
 
d(f1 )a (h)
 .. 
 . 
dfa (h) =  . 
 .. 
d(fm )a (h)
Xn
∂fi
mais, pour tout i ∈ {1, . . . , m}, on a d(fi )a (h) = (a) hj . Donc,
j=1
∂x j

  
∂f1 ∂f1 h1
 ∂x1 (a) ···
∂xn
(a)   
  . 
dfa (h) =  .. ..   ..  .
  
 ∂fm. ∂fm
.  
(a) ··· (a)
∂x1 ∂xn hn
On appelle matrice·jacobienne¸ de f au point a, et on note Ja (f ) la matrice à m
∂fi
lignes et n colonnes (a) . On note symboliquement
∂xj 1≤i≤m
1≤j≤n

 
dx1
.
dfa = Ja (f )  ..  .
dxn
Enfin, si f est à valeurs dans IRn et différentiable au point a, alors la matrice jacobienne
de f en a est une matrice carrée. On appelle jacobien de f au point a, et on note
∂(f1 , . . . , fn )
, le déterminant de la matrice Ja (f ).
∂(x1 , . . . , xn )
∂(f1 , . . . , fn )
det Ja (f ) = .
∂(x1 , . . . , xn )

Remarques :
– Soient A un ouvert non vide de IRn , B un ouvert non vide de IRm , g : A −→ IRm et
f : B −→ IRp deux applications avec g(A) ⊂ B. On suppose que g est différentiable
au point a et que f est différentiable au point g(a). Alors, d’après le théorème II.4
nous savons que f ◦g est différentiable au point a et que

Ja (f ◦g) = Jg(a) (f )Ja (g).


14 Fonctions de plusieurs variables

En particulier, si f est à valeurs dans IR, et h = f ◦g on a


m
X ∂f
∂h ∂gi
(a) = (g(a)) (a),
∂xj i=1
∂y i ∂x j

avec des notations evidentes.


– Soient f un homéomorphisme d’une partie ouverte A de IRn sur une partie ouverte
B de IRn et a un élément de A. On suppose que f est différentiable au point a et
que det Ja (f ) 6= 0. Alors, d’après le théorème II.7, l’application réciproque f −1 est
différentiable au point f (a) ∈ B et on a

Jf (a) (f −1 ) = (Ja (f ))−1 .

Exemple : Soit f : IR3 −→ IR3 l’application définie par


 
r cos θ cos ϕ
f (r, θ, ϕ) =  r sin θ cos ϕ  .
r sin ϕ

On a donc
 
cos θ cos ϕ −r sin θ cos ϕ −r cos θ sin ϕ
J(r,θ,ϕ) (f ) =  sin θ cos ϕ r cos θ cos ϕ −r sin θ sin ϕ 
sin ϕ 0 r cos ϕ

et le jacobien de f est det J(r,θ,ϕ) (f ) = r2 cos ϕ.

Définition : Soit f une fonction numérique définie sur une partie ouverte non vide A
de IRn . Si µ continue on dit que f est de classe C 0 sur A, et si f admet des dérivées
f est ¶
∂f
partielles qui sont continues sur A on dit que f est de classe C 1 . Plus
∂xi 1≤i≤n
p
généralement,
µ on ¶ dit que f est de classe C , (p ≥ 1), sur A si f admet des dérivées
∂f
partielles qui sont de classe C p−1 sur A. Enfin, f est dite de classe C ∞ si
∂xi 1≤i≤n
elle est de classe C p pour tout p ∈ IN.

Notons que f est de classe C 1 sur A si, et seulement si, f est différentiable en tout
point de A et si la différentielle df : A −→ L(IRn , IR) est une application continue sur
A. (Voir corollaire III.3).
Dérivées partielles 15

∂2f
Notation : Nous notons (a) pour désigner la dérivée partielle par rapport à
∂xj ∂xi
∂f ∂pf
xj , au point a, de la fonction . Nous définissons de même la notation .
∂xi ∂xi1 · · · ∂xip
∂2f ∂2f
Pour simplifier, on note au lieu de .
∂x2i ∂xi ∂xi

Le théorème suivant est simple à démontrer par récurrence. La démonstration est


laissée en exercice au lecteur.

Théorème III.5. Soient A un ouvert non vide de IRn , B un ouvert non vide de IRm ,
g : A −→ IRm et f : B −→ IRp deux applications avec g(A) ⊂ B. On suppose que g et
f sont de classe C k , alors f ◦g est aussi de classe C k .

Théorème III.6. (de Schwarz) Soit f une fonction numérique définie sur un ouvert
2 ∂2f ∂2f
non vide A de IR et possédant sur A les dérivées partielles et . Si les
∂x ∂y ∂y ∂x
∂2f ∂2f
fonctions et sont continues au point (a, b) ∈ A, alors
∂x ∂y ∂y ∂x

∂2f ∂2f
(a, b) = (a, b).
∂x ∂y ∂y ∂x

Preuve : Soit V un voisinage de (0, 0) tel que, pour (h, k) ∈ V , on ait (a+h, b+k) ∈ A.
On définit sur V la fonction F par

F (h, k) = f (a + h, b + k) − f (a + h, b) − f (a, b + k) + f (a, b).

Pour k fixé, soit ϕ la fonction réelle de la variable réelle définie par

ϕ(x) = f (x, b + k) − f (x, b).

La fonction ϕ est dérivable et on a

∂f ∂f
ϕ0 (x) = (x, b + k) − (x, b).
∂x ∂x

Mais F (h, k) = ϕ(a + h) − ϕ(a), il résulte du théorème des accroissements finis qu’il
existe θ1 ∈]0, 1[ tel que
µ ¶
0 ∂f ∂f
F (h, k) = hϕ (a + θ1 h) = h (a + θ1 h, b + k) − (a + θ1 h, b) .
∂x ∂x
16 Fonctions de plusieurs variables

∂f
Mais la fonction y 7→ (a + θ1 h, y) est dérivable et une nouvelle application du
∂x
théorème des accroissements finis montre qu’il existe θ2 ∈]0, 1[ tel que

∂2f
F (h, k) = hk (a + θ1 h, b + θ2 k).
∂y ∂x

∂2f
Comme est continue au point (a, b), on a
∂y ∂x

F (h, k) ∂2f ∂2f


lim = lim (a + θ1 h, b + θ2 k) = (a, b). (1)
(h,k)→(0,0) hk (h,k)→(0,0) ∂y ∂x ∂y ∂x

Soit maintenant ψ la fonction réelle de la variable réelle définie par

ψ(y) = f (a + h, y) − f (a, y).

Comme précédemment il existe θ1 ∈]0, 1[ tel que

F (h, k) =ψ(b + k) − ψ(b) = kψ 0 (b + θ1 k)


µ ¶
∂f ∂f
=k (a + h, b + θ1 k) − (a, b + θ1 k) .
∂y ∂x

et il existe θ2 ∈]0, 1[ tel que

∂2f
F (h, k) = hk (a + θ2 h, b + θ1 k).
∂x ∂y

∂2f
Comme est continue au point (a, b), on a
∂x ∂y

F (h, k) ∂2f
lim = (a, b). (2)
(h,k)→(0,0) hk ∂x ∂y

En combinant (1) et (2) nous obtenons le résultat.

Corollaire III.6. Soient f une fonction numérique de classe C p sur un ouvert non
vide A de IRn et i1 , . . . , in des entiers naturels tels que i1 + · · · + in = p. Alors toutes
les dérivées partielles d’ordre p où l’on a dérivé ij fois par rapport à xj , (1 ≤ j ≤ n),
sont égales.
Dérivées partielles 17

5
! Remarques :
∂2f ∂2f
– Si les fonctions et existent seulement au point (a, b), ou si elles existent
∂x ∂y ∂y ∂x
sur A mais ne sont pas continues au point (a, b), on peut avoir
∂2f ∂2f
(a, b) 6= (a, b).
∂x ∂y ∂y ∂x
Soit f la fonction numérique définie sur IR2 par
 2 2
 xy(x − y )

si (x, y) 6= (0, 0)
f : IR2 −→ IR, f (x, y) = x2 + y 2


0 si (x, y) = (0, 0).
on a
∂f x4 y + 4x2 y 3 − y 5 ∂f
(x, y) = si (x, y) 6= (0, 0) et (0, 0) = 0
∂x (x2 + y 2 )2 ∂x
∂f x5 − 4x3 y 2 − xy 4 ∂f
(x, y) = si (x, y) 6= (0, 0) et (0, 0) = 0.
∂y (x2 + y 2 )2 ∂y
Par suite, on a
µ ¶
∂2f 1 ∂f ∂f
(0, 0) = lim (0, y) − (0, 0) = −1,
∂y ∂x y→0 y ∂x ∂x
µ ¶
∂2f 1 ∂f ∂f
(0, 0) = lim (x, 0) − (0, 0) = 1.
∂x ∂y x→0 x ∂y ∂y

∂2f ∂2f
– Même si les fonctions et existent et sont continues sur un voisinage
∂x ∂y ∂y ∂x
∂2f ∂2f
de (a, b), il se peut que ou n’existe pas au point (a, b). C’est le cas par
∂x2 ∂y 2
exemple, au point (0, 0), de la fonction f définie sur IR2 par
 1 1

 x2 sin + y 2 sin si x 6= 0 et y 6= 0

 x y



 1
 x2 sin si x 6= 0 et y = 0
f (x, y) = x
 1

 y 2 sin si x = 0 et y 6= 0

 y



0 si x = 0 et y = 0

Définition : Soient A et B deux ouverts non vides de IRn , et f : A −→ B. On dit que f


est un C k -difféomorphisme si, et seulement si, f est bijective et les deux applications
f et f −1 sont de classe C k .
18 Fonctions de plusieurs variables

Le théorème suivant sera admis sans démonstaration.

Théorème III.7. Soit A un ouvert non vide de IRn et f : A −→ IRn . On suppose que f
est une application injective et de classe C p , (p ≥ 1), sur A avec ∀ a ∈ A, det Ja (f ) 6= 0.
Alors B = f (A) est un ouvert de IRn et l’application

fe : A −→ B : x 7→ f (x)

est un C p -difféomorphisme.

IV. Inégalité des accroissements finis, et théorème de Taylor

Dans la suite, si (x, y) ∈ IRn , nous notons [x, y] pour désigner l’ensemble
© ª
tx + (1 − t)y : t ∈ [0, 1] .

Théorème IV.1. Soit A un ouvert convexe non vide de IRn . Nous munissons IRn et
IRm des normes k · kIRn et k · kIRm respectivement, et nous munissons L(IRn , IRm ) de la
norme k · kL des applications linéaires continues correspondante. Si f : A −→ IRm est
une application de classe C 1 sur A alors

∀ (x, y) ∈ A2 , k f (x) − f (y) kIRm ≤ k x − y kIRn sup k dfz kL .


z∈[x,y]

Cette inégalité s’appelle l’inégalité des accroissements finis.

Preuve : Soit (x, y) ∈ A2 . Pour t ∈ [0, 1] nous posons ϕ(t) = f (x + t(y − x)),
c’est une application de la variable réelle, de classe C 1 sur [0, 1]. Clairement ϕ0 (t) =
dfx+t(y−x) (y − x). Donc
° °
∀ t ∈ [0, 1], k ϕ0 (t) kIRm ≤ ° dfx+t(y−x) °L k y − x kIRn ≤ k y − x kIRn sup k dfz kL .
z∈[x,y]

Mais Z 1
f (y) − f (x) = ϕ(1) − ϕ(0) = ϕ0 (t) dt,
0
donc
Z 1
k f (y) − f (x) kIRm ≤ k ϕ0 (t) kIRm dt ≤ k y − x kIRn sup k dfz kL .
0 z∈[x,y]

Ce qui démontre le résultat.


Inégalité des accroissements finis, et théorème de Taylor 19

Corollaire IV.2. Soit A un ouvert convexe non vide de IRn . Si f : A −→ IRm est
une application de classe C 1 sur A telle que df = 0 alors f est constante sur A.

Théorème IV.3. Soit A une partie ouverte non vide de IRn , et f : A −→ IR une
fonction numérique de classe C 2 sur A. Soient a ∈ A et r > 0 tel que la boule de centre
a et de rayon r, B(a, r) soit contenue dans A. Alors si h = (h1 , . . . , hn ) ∈ IRn vérifie
k h k < r, on a
Xn X
∂f 1 ∂2f 2
f (a + h) = f (a) + (a) hj + (a) hi hj + ε(h) k h k .
j=1
∂xj 2 ∂xi ∂xj
1≤i,j≤n

avec lim ε(h) = 0.


h→0

Preuve : Pour h ∈ B(0, r) fixé, soit ϕ la fonction numérique définie sur [0, 1] par
Xn
∂f
ϕ(t) = f (a + th) − f (a) + (1 − t) (a + th) hj .
j=1
∂x j

Un calcul immédiat montre que


X ∂2f
0
ϕ (t) = (1 − t) (a + th) hi hj .
∂xi ∂xj
1≤i,j≤n

Posons
Xn X
∂f 1 ∂2f
R(a, h) = f (a + h) − f (a) − (a) hj − (a) hi hj .
j=1
∂xj 2 ∂xi ∂xj
1≤i,j≤n

Z 1
La relation ϕ(1) − ϕ(0) = ϕ0 (t) dt permet d’écrire
0

X Z 1 µ ¶
∂2f ∂2f
R(a, h) = hi hj (1 − t) (a + th) − (a) dt.
0 ∂xi ∂xj ∂xi ∂xj
1≤i,j≤n

Soit ε > 0, en utilisant la continuité des dérivées partielles d’ordre 2 de f , il existe η > 0
tel que si k h k < η on a
¯ ¯
¯ ∂2f ∂2f ¯
2 ¯
∀ (i, j) ∈ {1, . . . , n} , ∀ t ∈ [0, 1], ¯ (a + th) − (a) ¯¯ ≤ ε.
∂xi ∂xj ∂xi ∂xj

On conclut que
 2
Xn
2 2
k h k < η =⇒ | R(a, h) | ≤ ε  | hi | = ε k h k1 ≤ αε k h k
j=1
20 Fonctions de plusieurs variables

(Nous avons utilisé l’équivalence des normes k · k1 et k · k). Alors

1
lim 2 R(a, h) = 0.
h→0 khk

1
Il suffit alors de poser ε(h) = 2 R(a, h).
khk

V. Recherche des extremums d’une fonction numérique

Soit f une fonction numérique définie sur une partie A de IRn . On dit que f admet
un maximum (resp. minimum) en un point a de A si, et seulement si, il existe un
voisinage V de a dans A tel que, pour x ∈ V , on ait f (x) ≤ f (a) (resp. f (x) ≥ f (a)).
On dira que f admet un extremum au point a si f admet un maximum ou un
minimum au point a. Enfin, un extremum est dit strict si les inégalités ci-dessus sont
strictes pour x 6= a.

Définition : Soit A un ouvert non vide de IRn , et f une fonction numérique définie
sur A. On dit que µ le point ¶a ∈ A est un point critique de f si, et seulement si, les
∂f
dérivées partielles (a) existent et valent 0.
∂xi 1≤i≤n

Proposition V.1. Soit A un ouvert non vide de IRn . Si f admet un extremum au point
a ∈ A, et si elle admet des dérivées partielles par rapport à toute ses variables, alors a
est un point critique de f .

Preuve : En effet, si f admet un extremum au point a = (a1 , . . . , an ), alors pour tout


j ∈ {1, . . . , n}, la fonction

ϕj : xj 7→ f (a1 , . . . , aj−1 , xj , aj+1 , . . . , an )

admet un extremum au point aj et elle est dérivable en ce point, par conséquent


∂f
(a) = ϕ0j (aj ) = 0. Ce qui prouve le résultat.
∂xj

Rappelons que être point critique est une condition nécessaire pour être un
extremum mais elle n’est pas suffisante comme le montre l’exemple de la fonction
x 7→ x3 .
Recherche des extremums d’une fonction numérique 21

Le lemme suivant est une conséquence immédiate de notre étude des endomor-
phismes symétriques.
· ¸
r s
Lemme V.2. Soit M = une matrice carrée d’ordre 2. Alors l’équation
s t
X 2 − (r + t)X + rt − s2 = 0 admet deux racines réelles λ ≤ Λ et on a

hM h, hi hM h, hi
Λ = sup 2 , λ = inf 2 .
h6=0 k h k2 h6=0 k h k2

De plus la borne supérieure est atteinte en hΛ un vecteur propre associé à Λ et la


borne inférieure est atteinte en hλ un vecteur propre associé à λ.

Soit f une fonction numérique de classe C 2 sur un ouvert non vide A de IR2 . On
∂f ∂f
suppose que a ∈ A est un point critique pour f , c’est à dire (a) = 0 et (a) = 0.
∂x ∂y
Notons · ¸
∂2f ∂2f ∂2f r s
r= (a), s = (a), t = (a), et M =
∂x2 ∂x ∂y ∂y 2 s t
Si la boule B(a, r) de centre a et de rayon r est contenue dans A alors d’après le théorème
IV.3 on peut écrire, pour h = (α, β) ∈ B(0, r),

1 2 1 2
f (a + h) − f (a) = (rα2 + 2sαβ + tβ 2 ) + k h k2 ε(h) = hM h, hi + k h k2 ε(h). (¶)
2 2

Dans la suite nous supposons rt−s2 6= 0. Alors l’équation X 2 −(r+t)X +rt−s2 = 0


1
admet deux racines non nulles λ ≤ Λ. On choisit ² ∈]0, min(| λ | , | Λ |)[, alors il existe
4
η > 0 tel que k h k < η =⇒ | ε(h) | < ². En utilisant (¶) on obtient

λ 2 Λ 2
k h k2 < η =⇒ ( − ²) k h k2 ≤ f (a + h) − f (a) ≤ ( + ²) k h k2 . (§)
2 2

Nous distinguons plusieurs cas:

– Si λ > 0 ( ⇐⇒ rt − s2 > 0 et r > 0). Dans ce cas (§) montre que

λ 2
∀ h ∈ B(0, η) \ {0}, f (a + h) − f (a) ≥ k h k2 > 0,
4

et f admet un minimum strict en a.


22 Fonctions de plusieurs variables

– Si Λ < 0 ( ⇐⇒ rt − s2 > 0 et r < 0). Dans ce cas (§) montre que

Λ 2
∀ h ∈ B(0, η) \ {0}, f (a + h) − f (a) ≤ k h k2 < 0,
4

et f admet un maximum strict en a.


– Si λ < 0 < Λ ( ⇐⇒ rt − s2 < 0). Dans ce cas on considère hλ (resp. hΛ ) un vecteur
propre de M , de norme euclidienne égale à 1 et associé à λ (resp. Λ). En utilisant
(¶) on obtient

Λ
∀ t ∈]0, η[, f (a + thΛ ) − f (a) = ( + ε(thΛ ))t2 > 0
2
λ
∀ t ∈]0, η[, f (a + thλ ) − f (a) = ( + ε(thλ ))t2 < 0.
2
Donc f n’admet pas un extremum en a. On dit que a et un point selle.

Nous avons démontré le résultat suivant

Théorème V.3 Soit f une fonction numérique de classe C 2 sur un ouvert non vide A
de IR2 . On suppose que a ∈ A est un point critique pour f . Notons

∂2f ∂2f ∂2f


r= (a), s= (a), t= (a).
∂x2 ∂x ∂y ∂y 2

♣ Si rt − s2 > 0 et r > 0 alors f admet un minimum strict en a.


♣ Si rt − s2 > 0 et r < 0 alors f admet un maximum strict en a.
♣ Si rt − s2 < 0 alors f n’admet pas un extremum en a.

Notons que le cas rt−s2 = 0 ne permet pas de conclure concernant le point critique
a, une étude plus précise de f est alors nécessaire.
Théorèmes des fonctions implicites 23

VI. Théorèmes des fonctions implicites

Les deux théorèmes suivants sont des cas particuliers d’un résultat plus général, et
seront admis sans démonstration.

Théorème VI.1. Soit A un ouvert non vide de IR2 et f : (x, y) 7→ f (x, y) une fonction
numérique de classe C k , (k ≥ 1), sur A. On suppose qu’il existe (a, b) ∈ A tel que

∂f
f (a, b) = 0, (a, b) 6= 0.
∂y

Alors, il existe (α, β) ∈ (IR∗+ )2 tel que, pour tout x ∈]a − α, a + α[, l’équation f (x, y) = 0
admette une seule solution ϕ(x) ∈]b − β, b + β[. La fonction ϕ ainsi définie est de
classe C k sur ]a − α, a + α[. De plus ϕ(a) = b, et pour tout x ∈]a − α, a + α[, on a
∂f
(x, ϕ(x)) 6= 0 et
∂y
∂f
(x, ϕ(x))
ϕ0 (x) = − ∂x .
∂f
(x, ϕ(x))
∂y

Exemple : Considérons la fonction f définie sur IR2 par f (x, y) = ex+y + y − 1. Nous
∂f
avons f (0, 0) = 0 et (0, 0) = 2, donc d’après le théorème des fonctions implicites
∂y
l’équation f (x, y) = 0 définie une seule application ϕ :] − ε, ε[−→ IR telle que ϕ(0) = 0
et, pour tout x ∈] − ε, ε[, f (x, ϕ(x)) = 0. La fonction ϕ est de classe C ∞ car f est de
classe C ∞ sur A. On conclut que ϕ admet un développement limité à tout ordre au
voisinage de 0. Donc

ϕ(x) = a0 + a1 x + a2 x2 + a3 x3 + O(x4 )

avec a0 = ϕ(0) = 0 et a1 = ϕ0 (0) = −1/2. Les deux coefficients qui restent peuvent être
calculés par la méthode des coefficients indéterminés:

µ ¶³ ´
−x x2 x3 4 x 2 3 4
−e (ϕ(x) − 1) = 1 − x + − + O(x ) 1 + − a2 x − a3 x + O(x )
2 6 2
µ ¶
x 1
=1 − − a2 x2 + + a2 − a3 x3 + O(x4 )
2 12
24 Fonctions de plusieurs variables

et µ ¶ µ ¶
ϕ(x) x 1 2 1 a2
e = 1 − + a2 + x + − − + a3 x3 + O(x4 ).
2 8 48 2
Comme eϕ(x) = −e−x (ϕ(x) − 1) pour tout x ∈] − ε, ε[ alors nous en déduisons, par
1 1
identification, que a2 = − et a3 = . Enfin
16 192
x x2 x3
ϕ(x) = − − + + O(x4 ).
2 16 192

Théorème VI.2. Soit A un ouvert non vide de IR3 et f : (x, y, z) 7→ f (x, y, z) une
fonction numérique de classe C k , (k ≥ 1), sur A. On suppose qu’il existe (a, b, c) ∈ A
tel que
∂f
f (a, b, c) = 0, (a, b, c) 6= 0.
∂z
Alors, il existe (ε, η) ∈ (IR∗+ )2 tel que, pour tout (x, y) ∈]a − ε, a + ε[×]b − ε, b + ε[,
l’équation f (x, y, z) = 0 admette une seule solution ϕ(x, y) ∈]c − η, c + η[. La fonction
ϕ ainsi définie est de classe C k sur ]a − ε, a + ε[×]b − ε, b + ε[. De plus ϕ(a, b) = c, et
∂f
pour tout (x, y) ∈]a − ε, a + ε[×]b − ε, b + ε[, on a (x, y, ϕ(x, y)) 6= 0 et
∂z

∂f ∂f
(x, y, ϕ(x, y)) (x, y, ϕ(x, y))
∂ϕ ∂ϕ ∂y
(x, y) = − ∂x , (x, y) = − .
∂x ∂f ∂y ∂f
(x, y, ϕ(x, y)) (x, y, ϕ(x, y))
∂z ∂z

VII. Formes différentielles du premier degré

Définition : Soit A une partie non vide de IRn . On appelle forme différentielle du
premier degré sur A une application ω de A dans L(IRn , IR). La forme différentielle
ω associe donc à chaque élément x de A une forme linéaire ωx : IRn −→ IR. Une telle
forme se décompose sur la base canonique (dx1 , . . . , dxn ) de L(IRn , IR), (rappelons que
(dx1 , . . . , dxn ) est la base duale de la base canonique de IRn ), d’où
n
X
ωx = ωi (x)dxi .
i=1
Formes différentielles du premier degré 25

Définition : Soit A une partie non vide de IRn . On appelle champ de vecteurs sur
A une application V de A dans IRn .

Remarquons que la donnée d’une forme différentielle du premier degré ω sur A


permet de définir le champ de vecteurs x 7→ (ω1 (x), . . . , ωn (x)). Réciproquement, si V
est un champ de vecteurs sur A et si V1 , . . . , Vn sont les fonctions coordonnées de V , on
Xn
peut associer au champ V la forme différentielle ω = Vi dxi . Ceci montre que l’on
i=1
peut étudier en même temps les formes différentielles du premier degré et les champs
de vecteurs.

Définition : Soit f une fonction numérique définie sur un ouvert non vide A de IRn . Si
f est différentiable sur A, l’application df qui associe à chaque x de A la forme linéaire
dfx est une forme différentielle du premier degré sur A. Le champ de vecteurs associé
à df est appelé le gradient de f , on le note gradf . On a donc
µ ¶
∂f ∂f
gradf (x) = (x), · · · , (x) .
∂x1 ∂xn

Définition : Une forme différentielle ω du premier degré sur un ouvert A de IRn est dite
exacte si, et seulement si, il existe f : A −→ IR, différentiable sur A et telle que df = ω.
La fonction f , si elle existe, n’est évidemment pas unique, car ∀ α ∈ IR, d(f + α) = df .

n
X
Définition : Une forme différentielle ω = ωi dxi du premier degré sur un ouvert
i=1
A de IRn , (n ≥ 2), est dite fermée si, et seulement si, les fonctions ω1 , . . . , ωn sont de
classe C 1 sur A et
∂ωi ∂ωj
∀ (i, j) ∈ {1, . . . , n}2 , = .
∂xj ∂xi

n
X
Proposition VII.1. Soient n un entier supérieur ou égal à 2 et ω = ωi dxi une
i=1
forme différentielle du premier degré sur un ouvert A de IRn . On suppose que ω est
exacte et que les fonctions ω1 , . . . , ωn sont de classe C 1 sur A. Alors, ω est fermée sur
A.
26 Fonctions de plusieurs variables

Preuve : Comme ω est exacte, il existe une fonction f : A −→ IR telle que df = ω,


∂f
c’est à dire, telle que ωi = pour 1 ≤ i ≤ n. Les fonctions ω1 , . . . , ωn étant de classe
∂xi
C 1 , la fonction f admet des dérivées partielles d’ordre 2 continues sur A et, en vertu
du théorème III.6, on a

∂ωi ∂2f ∂2f ∂ωj


= = = , pour (i, j) ∈ {1, . . . , n}2 .
∂xj ∂xj ∂xi ∂xi ∂xj ∂xi

Proposition VII.2. Soient n un entier supérieur ou égal à 2, A un ouvert étoilé † de


n
X
IRn , et ω = ωi dxi une forme différentielle du premier degré fermée sur A. Alors, il
i=1
existe une fonction f : A −→ IR telle que df = ω.

Preuve : Soit a ∈ A tel que ∀ x ∈ A, [a, x] ⊂ A. Pour x = (x1 , . . . , xn ) ∈ A, nous


posons
n µZ
X 1 ¶
f (x) = ωi (a + t(x − a)) dt (xi − ai ).
i=1 0

Il est immédiat que


Z 1 n µZ
X 1 ¶
∂f ∂ωi
(x) = ωj (a + t(x − a)) dt + t (a + t(x − a)) dt (xi − ai )
∂xj 0 i=1 0 ∂xj
Z 1 Z 1 ÃXn
!
∂ωj
= ωj (a + t(x − a)) dt + t (xi − ai ) (a + t(x − a)) dt
0 0 i=1
∂xi
Z 1µ ¶
d
= ωj (a + t(x − a)) + t [ωj (a + t(x − a))] dt
0 dt
Z 1
d
= [t ωj (a + t(x − a))] dt = ωj (x)
0 dt

Ce qui prouve que dfx = ω(x) pour tout x ∈ A.

Corollaire VII.3. Soient n un entier supérieur ou égal à 2, A un ouvert non vide de


IRn , et V un champ de vecteurs sur A de classe C 1 et dont les applications coordonnées
sont V1 , . . . , Vn .
∂Vi ∂Vj
i. Si V est le gradient d’une fonction f : A −→ IR, alors = pour
∂xj ∂xi
1 ≤ i ≤ n et 1 ≤ j ≤ n.
∂Vi ∂Vj
ii. Si A est étoilé et si = pour 1 ≤ i ≤ n et 1 ≤ j ≤ n, alors V est le
∂xj ∂xi
gradient d’une fonction f : A −→ IR.

c’est à dire qu’il existe a ∈ A vérifiant ∀ x ∈ A, [a, x] ⊂ A.
Formes différentielles du premier degré 27

5
! Remarque : Il est faux en général qu’une forme différentielle ω fermée sur un ouvert
A de IRn soit exacte sur A. Par exemple, soit ω la forme différentielle définie sur
A = IR2 \ {(0, 0)} par
−y x
ω(x,y) = dx + 2 dy.
x2 +y 2 x + y2
On vérifie facilement que ω est une forme fermée sur A.
Supposons qu’il existe f : A −→ IR de classe C 1 telle que
∂f y ∂f x
(x, y) = − 2 , (x, y) = 2 .
∂x x + y2 ∂y x + y2
On considère alors,
g : IR −→ IR, θ 7→ f (cos θ, sin θ).

La fonction g est de classe C 1 et, pour tout θ ∈ IR, on a


∂f ∂f
g 0 (θ) = − sin θ (cos θ, sin θ) + cos θ (cos θ, sin θ) = 1.
∂x ∂y
Il en résulte que Z 2π
2π = g 0 (θ) dθ = g(2π) − g(0) = 0.
0
Ce qui est absurde, et ω n’est pas exacte sur A.
n
X
Définition : Soit ω = ωi dxi une forme différentielle du premier degré sur un ouvert
i=1
A de IRn , et γ : [a, b] −→ IRn , t 7→ (γ1 (t), . . . , γn (t)), un arc paramétré de classe C 1 tel
que γ([a, b]) ⊂ A. On appelle l’intégrale de ω le long de γ l’intégrale
Z Z b ÃX n
!
0
ω= ωi (γ(t)) γi (t) dt.
γ a i=1

Remarquons que si ϕ : [α, β] −→ [a, b] est un C 1 -difféomorphisme croissante et si


γ
e = γ ◦ϕ alors Z Z
ω= ω.
γ e
γ
Z
Donc ω ne dépend que de l’arc géométrique orienté défini par la paramétrisation γ.
γ
La définition précédente peut être généralisée d’une manière évidente pour les arcs
paramétrés de classe C 1 par morceaux, nous laissons cette tâche comme exercice au
lecteur.
Enfin, la proposition suivante est immédiate:
28 Fonctions de plusieurs variables
n
X
Proposition VII.4. Soient ω = ωi dxi une forme différentielle du premier degré
i=1
exacte sur un ouvert A de IRn , et γ : [a, b] −→ IRn un arc paramétré de classe C 1 tel
que γ([a, b]) ⊂ A. Alors, si ω = df , on a
Z
ω = f (γ(b)) − f (γ(a)).
γ
Exercices 29

EXERCICES

Exercice .1 Etudier l’existence d’une limite en (0, 0) pour les fonctions f suivantes:

xy 1 − cos(xy) xα y β
f (x, y) = , f (x, y) = , f (x, y) = , (α, β) ∈ IR2 .
x+y y2 y − x2
xyz
Exercice .2 La fonction f : (x, y, z) 7→ a-t-elle une limite en (0, 0, 0) ?
x+y+z
x+y
Exercice .3 La fonction f : (x, y, z) 7→ 2 a-t-elle une limite en
x − y2 + z2
(2, −2, 0) ?
Exercice .4 Soit f : IR2 −→ IR définie par
 2 2
 xy(x − y ) si

(x, y) 6= (0, 0)
f (x, y) = x2 + y 2


0 si (x, y) = (0, 0)

Montrer que f est de classe C 1 sur IR2 . Est-t-elle de classe C 2 sur IR2 .
Exercice .5 Soit f : IR −→ IR de classe C 1 , telle qu’il existe k ∈]0, 1[ vérifiant
∀ t ∈ IR, | f 0 (t) | ≤ k. On considère alors

F : IR2 −→ IR2 : F (x, y) = (x − f (y), y − f (x)).

1◦ . Montrer que F est un difféomorphisme de IR2 sur F (IR2 ).


2◦ . Montrer que F (IR2 ) = IR2 . (On pourrait utiliser un théorème du point fixe.)
Exercice .6 Soit la fonction

1 − xy
f : IR2 −→ IR : f (x, y) = Arccos p .
1 + x2 + y 2 + x2 y 2

Préciser le domaine de définition de f ; donner un ouvert de IR2 sur lequel f est de


classe C 1 . Calculer alors df et simplifier l’écriture de f .
Exercice .7 Trouver le plus grand ouvert ∆ de (IR∗+ )3 sur lequel la fonction f
suivante est définie et différentiable,

x2 + y 2 − z 2 y 2 + z 2 − x2 z 2 + x2 − y 2
f (x, y, z) = Arccos + Arccos + Arccos .
2xy 2yz 2zx

Simplifier en suite l’expression de f .


30 Fonctions de plusieurs variables

Exercice .8 Soit U un ouvert de IRn . Pour g : U −→ IR de classe C 2 , le laplacien


Xn
∂2g
de g est donné par ∆g = . On dit que g est harmonique si ∆g = 0 sur U .
i=1
∂x2i
1◦ . Pour (x, y) ∈ IR2 , soient z = x + iy ∈ C
| et f (x, y) = Re (zez ). Montrer que f est

harmonique sur IR2 .


y z x
2◦ . Vérifier que f : (x, y, z) 7→ Arctg + Arctg + Arctg est harmonique sur IR∗3 .
x y µzq ¶
3◦ . Déterminer f : IR∗+ 2
−→ IR de classe C telle que f 2 2 2
x1 + x2 + · · · + xn soit
harmonique sur IR∗n .
4◦ . On considère f : (r, θ) 7→ (x, y) = (r cos θ, r sin θ) et g : IR2 −→ IR de classe C 2 . On
pose G = g ◦f . Calculer ∆g en fonction des dérivées partielles de G.

5 . Trouver tous les polynômes P ∈ IR[X, Y ] tels que la fonction polynomiale (x, y) 7→
P (x, y) soit harmonique.
Exercice .9 Soient (A, B, C) ∈ IR3 \ {(0, 0, 0)} et (E) l’équation aux dérivées
partielles:
∂2f ∂2f ∂2f
A + 2B + C =0
∂x2 ∂x∂y ∂y 2
où f : IR2 −→ IR est la fonction inconnue , de classe C 2 .
Effectuer le changement de variables X = x + αy, Y = x + βy, (α, β) ∈ IR2 , α 6= β,
et montrer qu’on peut choisir (α, β) pour ramener (E) à l’une des quatres équations
∂2F ∂2F ∂2F ∂2F ∂2F
i. = 0, ii. = 0, iii. = 0, iv. + = 0.
∂X∂Y ∂X 2 ∂Y 2 ∂X 2 ∂Y 2
Résoudre i, ii et iii
Exercice .10 Déterminer les extremums locaux des applications f suivantes:
f: IR2 −→ IR : f (x, y) = x2 + xy + y 2 + 2x + 3y,

f : IR∗2
+ −→ IR : f (x, y) = x3 + 3xy 2 − 15x − 12y,

f: IR2 −→ IR : f (x, y) = x4 + y 4 − 4xy,


xy
f : IR∗2
+ −→ IR : f (x, y) = .
(1 + x)(1 + y)(x + y)
Exercice .11 Montrer que l’égalité f (x, y) = x3 + y 3 − 3xy − 1 = 0 définit au
voisinage de 0 une fonction implicite ϕ : x 7→ ϕ(x) = y telle que ϕ(0) = 1 ; donner un
développement limité de ϕ à l’ordre 3 au voisinage de 0.

Exercice .12 Montrer que l’égalité f (x, y) = ex+y +y −1 = 0 définit au voisinage de


0 une fonction implicite ϕ : x 7→ ϕ(x) = y telle que ϕ(0) = 0 ; donner un développement
limité de ϕ à l’ordre 3 au voisinage de 0.
Exercices 31

Exercice .13 Montrer, dans chacun des cas suivants, que la relation proposée définit
implicitement y en fonction de x sur IR, et que l’application ϕ : x 7→ ϕ(x) = y est de
classe C ∞ sur IR:

i. y 3 + y + x = 0, ii. y 3 + ex y + x2 = 0 iii. y 5 + (1 + x2 )y + 1 = 0.

Exercice .14 Soit n ∈ IN. Montrer que la relation y 2n+1 + y − x = 0 définit


implicitement y en fonction de x sur IR, et que l’application
Z x ϕ : x 7→ ϕ(x) = y est
de classe C ∞ sur IR. Calculer ensuite, pour x ∈ IR, ϕ(t) dt en fonction de n, x, et
0
ϕ(x).

Exercice .15 Montrer que la relation x3 + y 3 + z 3 − zx − x + y − 2z + 1 = 0 définit


au voisinage de (0, 0) une fonction implicite, de classe C ∞ , ϕ : (x, y) 7→ z = ϕ(x, y) telle
que ϕ(0, 0) = 1.Calculer ensuite, les dérivées d’ordre 2 de ϕ en (0, 0).

Exercice .16 Montrer que les formes différentielles ω suivantes sont exactes et
calculer une primitive dans chaque cas:
2xy dx + (x2 − y 2 ) dy,
(2 − 9xy 2 )x dx + (4y 2 − 6x3 )y dy,
(1 + y 2 sin 2x) dx − 2y cos2 x dy,
(x2 + 2xy + 2xz) dx + (x2 + y 2 ) dy + (x2 + z 2 ) dz.

Exercice .17 Si ω est une forme différentielle non fermée, on appelle facteur
intégrant une fonction µ non nulle telle que ω1 = µω soit fermée.
Dans chacun des cas suivants chercher un facteur integrant µ de ω, puis déterminer
une primitive de ω1 = µω.

1. y 2 dx + x2 dy, µ(x, y) = λ(x + y)

2. (x2 + y 2 − 1) dx − 2y dy, µ(x, y) = λ(x)

3. y dx − x dy, µ(x, y) = λ(xy)

4. (x2 + y 2 − 1) dx − 2xy dy, µ(x, y) = λ(x2 − y 2 )

Exercice .18 Déterminer f , g de IR dans IR de classe C 1 pour que la forme


y2
différentielle ω définie par: ω(x, y, z) = 2xz dx + f (y)g(z) dy + (x2 + ) dz, soit fermée
2
sur IR3 . En calculer alors les primitives.
32 Fonctions de plusieurs variables
Z
Exercice .19 Calculer les intégrales curvilignes ω suivantes:
Γ
a. ω(x, y) = (x − y 3 ) dx + x3 dy, et Γ est le cercle d’équation x2 + y 2 = 1 parcouru
une fois dans le sens positif.
1
b. ω(x, y, z) = xyz dz, et Γ est le cercle paramétré par x = cos t, y = √ sin t,
2
1
z = √ sin t, t allant de 0 à 2π.
2
c. ω(x, y, z) = z dx + x dy + y dz, et Γ est le cercle d’équations x2 + y 2 + z 2 = 1,
x + z = 1, parcouru une fois dans le sens positif, le plan x + z = 1 étant orienté par le
vecteur normal (1, 0, 1).
(x − y) dx + (x + y) dy
d. ω(x, y) = , et Γ est le carré ABCD de sommets A(a, a),
x2 + y 2
B(−a, a), C(−a, −a) et D(a, −a) avec a > 0 parcouru une fois dans le sens positif.
Solutions 33

SOLUTIONS

xy
Solution .1 – Étude de la limite en (0, 0) de la fonction f (x, y) =
.
x+y
Remarquons d’abord que l’ensemble Df de définition de f est IR2 \ {(x, −x) : x ∈ IR},
et (0, 0) ∈ Df . D’autre part, nous avons

lim f (x, 0) = 0 et lim f (x, x2 − x) = −1


> >
x→0 x→0

donc f n’a pas de limite en (0, 0).


1 − cos(xy)
– Étude de la limite en (0, 0) de la fonction f (x, y) = .
y2
L’ensemble Df de définition de f est IR2 \ {(x, 0) : x ∈ IR}, et (0, 0) ∈ Df . Nous
utilisons l’inégalité simple ∀ t ∈ IR, 0 ≤ 1 − cos t ≤ t2 /2. Alors,
x2
∀ (x, y) ∈ Df , 0 ≤ f (x, y) ≤
2
Il en résulte que lim f (x, y) = 0 .
(x,y)→(0,0)
xα y β
– Étude de la limite en (0, 0) de la fonction f (x, y) = , (α, β) ∈ IR2 .
y − x2
L’ensemble Df de définition de f est (IR∗+ )2 \ {(x, x2 ) : x ∈ IR∗+ }, et (0, 0) ∈ Df .
Remarquons que, si γ > max(2, α + 2β) alors

f (x, x2 + xγ ) = xα+2β−γ (1 + xγ−2 )β

Il en résulte que lim f (x, x2 + xγ ) = +∞, et f n’a pas de limite en (0, 0).
>
x→0

xyz
Solution .2 Étude de la limite en (0, 0, 0) de la fonction f (x, y, z) = .
x+y+z
L’ensemble Df de définition de f est IR3 \ {(x, y, z) : x + y + z =
6 0}, et (0, 0, 0) ∈ Df .
Remarquons que lim f (x, x, x4 − 2x) = −∞. Donc f n’a pas de limite en (0, 0, 0).
>
x→0

x+y
Solution .3 Étude de la limite en (2, −2, 0) de la fonction f (x, y, z) = .
x2
− y2 + z2
L’ensemble Df de définition de f est IR3 \ {(x, y, z) : x2 + z 2 =
6 y 2 }, et (2, −2, 0) ∈ Df .
Remarquons que
2 1
lim f (2 + ε, ε − 2, ε) = lim = , et lim f (2 + ε, −ε − 2, ε) = 0,
>
ε→0
>
ε→0
8+ε 4 >
ε→0

f n’a pas de limite en (2, −2, 0).


34 Fonctions de plusieurs variables

Solution .4 Notons que


 4 2 3 5
∂f  x y + 4x y − y

si (x, y) 6= 0
(x, y) = (x2 + y 2 )2
∂x 

0 si (x, y) = 0

et  5

 x − 4x3 y 2 − xy 4
∂f si (x, y) 6= 0
(x, y) = (x2 + y 2 )2
∂y 

0 si (x, y) = 0
En remarquant que
¯ 4 ¯ ¯ 5 ¯
¯ x + 4x2 y 3 − y 5 ¯ ≤ 2 | y | (x2 + y 2 )2 , et ¯ x − 4x3 y 2 − xy 4 ¯ ≤ 2 | x | (x2 + y 2 )2 ,

nous obtenons
¯ ¯ ¯ ¯
¯ ∂f ¯ ¯ ∂f ¯
∀ (x, y) 6= (0, 0), ¯ (x, y) ¯ ≤ 2|y| et ¯ (x, y) ¯ ≤ 2|x|.
¯ ∂x ¯ ¯ ∂y ¯

∂f ∂f
Les fonctions et sont continues sur IR2 et f est de classe C 1 sur IR2 .
∂x ∂y
Par contre f n’est pas de classe C 2 car
µ ¶
∂2f 1 ∂f ∂f
(0, 0) = lim (x, 0) − (0, 0) = 1.
∂x∂y x→0 x ∂y ∂y
µ ¶
∂2f 1 ∂f ∂f
(0, 0) = lim (0, y) − (0, 0) = −1.
∂y∂x y→0 y ∂x ∂x

Solution .5 1◦ . Montrons que F est injective. En effet, en utilisant l’inégalité des


accroissements finis,

F (x, y) = F (z, t) =⇒ (x − f (y) = z − f (t)) et (y − f (x) = t − f (z))


=⇒ (x − z = f (y) − f (t)) et (y − t = f (x) − f (z))
=⇒ (| x − z | ≤ k | y − t |) et (| y − t | ≤ k | x − z |)
=⇒ (| x − z | ≤ k 2 | x − z |) et (| y − t | ≤ k 2 | y − t |)
=⇒ (x = z) et (y = t)

D’autre part, la matrice jacobienne de F est


· ¸
1 −f 0 (y)
J(F )(x, y) = ,
−f 0 (x) 1
Solutions 35

donc F est de classe C 1 sur IR2 , et pour tout (x, y) de IR2 ,

det(J(f )(x, y)) = 1 − f 0 (x)f 0 (y) ≥ 1 − k 2 > 0

Alors F est un C 1 -difféomorphisme de IR2 sur F (IR2 ).


2◦ . Soit (a, b) ∈ IR2 . Posons , pour y ∈ IR, G(y) = b + f (a + f (y)). En utilisant le
théorème des accroissements finis nous obtenons

∀ (y, z) ∈ IR2 , | G(y) − G(z) | ≤ k 2 | y − z | .

Donc G est une application k 2 -contractante avec k 2 < 1. Comme IR2 est complet alors
le théorème du point fixe nous montre qu’il existe un réel et un seul y tel que G(y) = y.
On pose alors, x = a + f (y) et nous obtenons F (x, y) = (a, b).
On conclut que F (IR2 ) = IR2 et F est un C 1 -difféomorphisme de IR2 sur IR2 .

Solution .6 Notons d’abord que


p
| 1 − xy | ≤ 1 + x2 + y 2 + x2 y 2 ⇐⇒ (x + y)2 ≥ 0

donc f est définie sur IR2 .


Posons ∆+ = {(x, y) ∈ IR2 : x + y > 0} et ∆− = {(x, y) ∈ IR2 : x + y < 0}, ce sont
deux ouverts convexes de IR2 .
Pour tout (x, y) ∈ ∆+ ,
∂f 1 ∂f 1
(x, y) = et (x, y) = .
∂x 1 + x2 ∂y 1 + y2
∂f ∂f
Donc et sont continues sur ∆+ et f est de classe C 1 sur ∆+ et, pour tout
∂x ∂y
(x, y) ∈ ∆+ ,
dx dy
df(x,y) = 2
+ .
1+x 1 + y2
Mais ∆+ est un demi-espace donc convexe, il en résulte qu’il existe une constante c telle
que
∀ (x, y) ∈ ∆+ , f (x, y) = c + Arctg (x) + Arctg (y).

En testant le point (0, 1) nous trouvons c = 0 et par conséquent,

∀ (x, y) ∈ ∆+ , f (x, y) = Arctg (x) + Arctg (y).

En utilisant le fait que f (x, y) = f (−x, −y) nous voyons aussi que

∀ (x, y) ∈ ∆− , f (x, y) = −Arctg (x) − Arctg (y).


36 Fonctions de plusieurs variables

Solution .7 La fonction t 7→ Arccos t est définie sur [−1, 1] et dérivable sur


] − 1, 1[, alors le plus grand ouvert de différentiabilité ∆ de f est l’ensemble des points
(x, y, z) ∈ (IR∗+ )3 qui vérifient
¯ 2 ¯ ¯ 2 ¯ ¯ 2 ¯
¯ x + y2 − z2 ¯ ¯ y + z 2 − x2 ¯ ¯ z + x2 − y 2 ¯
¯ ¯ < 1, ¯ ¯ < 1, et ¯ ¯<1
¯ 2xy ¯ ¯ 2yz ¯ ¯ 2zx ¯

Or chacune de ces trois conditions est équivalente à

x4 + y 4 + z 4 < 2(x2 y 2 + y 2 z 2 + z 2 x2 )

ou bien,
(x + y + z)(x + y − z)(x − y + z)(−x + y + z) > 0

Mais x + y + z > 0 donc


© ª
∆ = (x, y, z) ∈ (IR∗+ )3 : (x + y − z)(x − y + z)(−x + y + z) > 0 .

Notons que deux parenthèses du produit (x + y − z)(x − y + z)(−x + y + z) ne peuvent


pas être négatives si (x, y, z) ∈ (IR∗+ )3 , alors
© ª
∆ = (x, y, z) ∈ (IR∗+ )3 : x + y > z, x + z > y, y + z > x .

∂f
Un calcul simple montre qu’en tout point (x, y, z) ∈ ∆ nous avons (x, y, z) = 0
∂x
puis, le fait que x, y et z jouent des rôles symétriques nous permet d’écrire aussi,
∂f ∂f
(x, y, z) = 0 et (x, y, z) = 0. Alors df = 0 sur ∆ qui est un ensemble convexe.
∂y ∂z
Alors f est constante sur ∆. En testant avec (1, 1, 1) ∈ ∆ nous obtenons f (x, y, z) = π
pour tout (x, y, z) ∈ ∆.
Cet exercice exprime le fait que la somme des angles d’un triangle vaut π.

Solution .8 1◦ . Notons que zez = ex (x + iy)(cos y + i sin y), alors

∀ (x, y) ∈ IR2 , f (x, y) = x ex cos y − ex y sin y.

On trouve alors

∂f ∂2f
(x, y) = (1 + x)ex cos y − ex y sin y, (x, y) = (2 + x)ex cos y − ex y sin y.
∂x ∂x2
∂f ∂2f
(x, y) = − (1 + x)ex sin y − ex y cos y, (x, y) = − (2 + x)ex cos y + ex y sin y.
∂y ∂y 2

Alors ∆f = 0 sur IR2 .


Solutions 37

2◦ . Notons que

∂f y z ∂2f 2xy 2xz


(x, y, z) = − 2 2
+ 2 , (x, y, z) = 2 − 2 .
∂x x +y x + z2 ∂x 2 2
(x + y ) 2 (x + z 2 )2

De même,

∂2f 2yz 2xy ∂2f 2xz 2yz


2
(x, y, z) = 2 2 2
− 2 , et (x, y, z) = 2 − 2 .
∂y (y + z ) (x + y 2 )2 ∂z 2 2
(x + z ) 2 (y + z 2 )2

donc ∆f = 0 sur (IR∗ )3 .


p
3◦ . Posons, pour simplifier, x = (x1 , x2 , . . . , xn ), r = x21 + x22 + · · · + x2n , et enfin
p
posons g(x) = f ( x21 + x22 + · · · + x2n ).
Un calcul immédiat montre que, pour tout k ∈ {1, 2, . . . , n},
µ ¶0
∂g xk 0 ∂2g f 0 (r) f 0 (r) x2k
(x) = f (r), (x) = + .
∂xk r ∂x2k r r r

Alors,
n−1 0
∆g(x) = f 00 (r) + f (r).
r
On conclut que g est harmonique si, et seulement si,

 ar + b si n = 1,




f (r) = aLog r + b si n = 2,




 a + b si n ≥ 3.
rn−2
4◦ . Remarquons d’abord que

∂G ∂g ∂g
(r, θ) = cos θ (x, y) + sin θ (x, y),
∂r ∂x ∂y
∂G ∂g ∂g
(r, θ) = − r sin θ (x, y) + r cos θ (x, y)
∂θ ∂x ∂y

Ce qui s’écrit,
∂G ∂g ∂g ∂G ∂g ∂g
r (r, θ) = x (x, y) + y (x, y), (r, θ) = −y (x, y) + x (x, y).
∂r ∂x ∂y ∂θ ∂x ∂y
Donc,
µ ¶ µ ¶ µ ¶
∂ ∂G ∂ ∂g ∂g ∂ ∂g ∂g
r r (r, θ) = x x (x, y) + y (x, y) + y x (x, y) + y (x, y)
∂r ∂r ∂x ∂x ∂y ∂y ∂x ∂y
2 2 2
∂g ∂g ∂ g ∂ g ∂ g
=x (x, y) + y (x, y) + x2 2 (x, y) + 2xy (x, y) + y 2 2 (x, y)
∂x ∂y ∂x ∂x ∂y ∂y
38 Fonctions de plusieurs variables

et
µ ¶ µ ¶
∂2G ∂ ∂g ∂g ∂ ∂g ∂g
(r, θ) = − y −y (x, y) + x (x, y) + x −y (x, y) + x (x, y)
∂θ2 ∂x ∂x ∂y ∂y ∂x ∂y
2
∂g ∂g 2∂ g ∂2g 2
2∂ g
= − x (x, y) − y (x, y) + y (x, y) − 2xy (x, y) + x (x, y).
∂x ∂y ∂x2 ∂x ∂y ∂y 2
Alors,
µ ¶ µ ¶
∂ ∂G ∂2G ∂2g ∂2g
r r (r, θ) + (r, θ) = (x2 + y 2 ) (x, y) + 2 (x, y)
∂r ∂r ∂θ2 ∂x2 ∂y
Ce qui démontre que
∂2G 1 ∂G 1 ∂2G
(r, θ) + (r, θ) + (r, θ) = ∆g(x, y).
∂r2 r ∂r r2 ∂θ2
5◦ . Supposons que P ∈ IR[X, Y ] soit harmonique. Alors la fonction G définie
m
X
par G(r, θ) = P (r cos θ, r sin θ) s’écrit sous la forme G(r, θ) = rk hk (θ) avec hk un
k=0
polynôme trigonométrique de degré au plus k. D’après 4◦ nous avons
∂2G ∂G ∂2G
r2 (r, θ) + r (r, θ) + (r, θ) = 0
∂r2 ∂r ∂θ2
ce qui se traduit par
m
X
(k 2 hk (θ) + h00k (θ))rk = 0.
k=0

Par conséquent, h00k + k 2 hk = 0 pour tout k. Ce qui donne hk (θ) = Ak eikθ + Ak e−ikθ .
(Ici nous avons utilisé que hk est réel). En revenant à l’expression de G nous obtenons
Ãm !
X
iθ k
G(r, θ) = Re Ak (re )
k=0

ce qui donne à !
m
X
P (x, y) = Re Ak (x + iy)k .
k=0
m
X m
¡ ¢ X ¡ ¢
P (x, y) = ak Re (x + iy)k + bk Im (x + iy)k .
k=0 k=0

Si l’on pose, pour k ∈ IN,


1¡ ¢ 1 ¡ ¢
Pk (X, Y ) = (X + iY )k + (X − iY )k , Qk (X, Y ) = (X + iY )k − (X − iY )k
2 2i
alors l’espace des polynômes harmoniques de deux variables est engendré par {Qk }k≥1
et {Pk }k≥0 , (car la réciproque est immédiate).
Solutions 39

Solution .9 Notons d’abord que si deux des coefficients A, B et C sont nuls, alors
nous sommes déjà dans l’un des cas i, ii ou iii. Il en résulte, quitte à échanger A et C si
c’est nécessaire, que nous pouvons supposer C 6= 0. C’est ce que nous supposons dans
la suite.
Effectuons le changement de variables X = x + αy, Y = x + βy avec α 6= β. Définissons
F sur IR2 par F (X, Y ) = f (x, y).

∂f ∂F ∂F ∂f ∂F ∂F
(x, y) = (X, Y ) + (X, Y ), (x, y) = α (X, Y ) + β (X, Y )
∂x ∂X ∂Y ∂y ∂X ∂Y

et
∂2f ∂2F ∂2F ∂2F
(x, y) = (X, Y ) + 2 (X, Y ) + (X, Y )
∂x2 ∂X 2 ∂X ∂Y ∂Y 2
∂2f ∂2F ∂2F ∂2F
(x, y) =α (X, Y ) + 2(α + β) (X, Y ) + β (X, Y )
∂x ∂y ∂X 2 ∂X ∂Y ∂Y 2
∂2f 2
2∂ F ∂2F 2
2∂ F
(x, y) =α (X, Y ) + 2αβ (X, Y ) + β (X, Y )
∂y 2 ∂X 2 ∂X ∂Y ∂Y 2
∂2f ∂2f ∂2f
L’équation, A (x, y) + 2B (x, y) + C (x, y) = 0 est alors équivalente à
∂x2 ∂x ∂y ∂y 2

∂2F ∂2F ∂2F


a(α) (X, Y ) + 2b(α, β) (X, Y ) + a(β) (X, Y ) = 0 (∗)
∂X 2 ∂X ∂Y ∂Y 2

avec, a(t) = A + 2Bt + Ct2 et b(t, s) = A + B(t + s) + Cts. Distinguons alors les cas
suivants
• Cas B 2 − AC > 0. Alors nous choisissons (α, β) les deux racines réelles distinctes de
∂2F
l’équation a(t) = 0. Avec ce choix, (∗) devient (X, Y ) = 0.
∂X ∂Y
• Cas B 2 − AC = 0. Si B = 0 alors A = 0 (car C 6= 0) et par conséquent (∗) s’écrit
∂2f
déjà (x, y) = 0. Par contre, si B 6= 0 alors nous prenons β = −B/C = −A/B et α
∂y 2
∂2F
un réel quelconque différent de β. Avec ce choix (∗) devient (X, Y ) = 0.
∂X 2
2
• Cas B − AC < 0. Nous cherchons α et β distincts, qui vérifient a(α) = a(β) et
b(α, β) = 0 ce qui donne les deux conditions

2B 2B 2 − AC
α+β =− , αβ =
C C2
√ √
AC − B 2 − B AC − B 2 + B
Donc, avec le choix α = , et β = − , l’équation (∗)
C C
∂2F ∂2F
devient (X, Y ) + (X, Y ) = 0.
∂X 2 ∂Y 2
40 Fonctions de plusieurs variables

∂2F
Il est immédiat de voir que les solutions de (X, Y ) = 0 sont les fonctions
∂X ∂Y
F (X, Y ) = G(X) + H(Y ) où les fonctions G et H sont des fonctions quelconques
de classe C 2 .
∂2F
Les solutions de (X, Y ) = 0 sont les fonctions F (X, Y ) = XG(Y ) + H(Y ) où les
∂X 2
fonctions G et H sont des fonctions quelconques de classe C 2 .
∂2F
Enfin, les solutions de (X, Y ) = 0 sont les fonctions F (X, Y ) = Y G(X) + H(X) où
∂Y 2
les fonctions G et H sont des fonctions quelconques de classe C 2 .

Solution .10 – Les extrémums locaux de l’application:

f : IR2 −→ IR : f (x, y) = x2 + xy + y 2 + 2x + 3y.

Les points critiques de f sont les points (x, y) qui vérifient

∂f ∂f
(x, y) = 2x + y + 2 = 0, et (x, y) = x + 2y + 3 = 0
∂x ∂y

1 4
ce qui donne l’unique point critique (x0 , y0 ) = (− , − ) de f .
3 3

∂2f ∂2f ∂2f


r= (x0 , y0 ) = 2, s= (x0 , y0 ) = 1, t= (x0 , y0 ) = 2.
∂x2 ∂x ∂y ∂y 2

donc r > 0 et rt − s2 > 0. La fonction f admet un minimum local strict au point


1 4
(− , − ).
3 3
– Les extrémums locaux de l’application:

f : IR∗2 3 2
+ −→ IR : f (x, y) = x + 3xy − 15x − 12y.

Les points critiques de f sont les points (x, y) qui vérifient

∂f ∂f
(x, y) = 3(x2 + y 2 − 5) = 0, et (x, y) = 6(xy − 2) = 0.
∂x ∂y

Ce qui donne les deux points critiques (x1 , y1 ) = (1, 2) et (x2 , y2 ) = (2, 1) de f .

∂2f ∂2f ∂2f


r= (x, y) = 6x, s= (x, y) = 6y, t= (x, y) = 6x.
∂x2 ∂x ∂y ∂y 2

Donc rt − s2 = 36(x2 − y 2 ). Alors la fonction f n’admet pas un extrémum au point


(x1 , y1 ) = (1, 2), car en ce point rt−s2 < 0. Par contre la fonction f admet un minimum
local en (x2 , y2 ) car en ce point rt − s2 > 0 et r > 0.
Solutions 41

– Les extrémums locaux de l’application:


f : IR2 −→ IR : f (x, y) = x4 + y 4 − 4xy.
Les points critiques de f sont les points (x, y) qui vérifient
∂f ∂f
(x, y) = 4(x3 − y) = 0, et (x, y) = 4(y 3 − x) = 0
∂x ∂y
ce qui donne les trois points critiques (x1 , y1 ) = (0, 0), (x2 , y2 ) = (1, 1) et (x3 , y3 ) =
(−1, −1) de f .
∂2f 2 ∂2f ∂2f
r= (x, y) = 12x , s = (x, y) = −4, t = (x, y) = 12y 2 .
∂x2 ∂x ∂y ∂y 2
Donc rt − s2 = 16(9x2 y 2 − 1). Alors la fonction f n’admet pas un extrémum au point
(x1 , y1 ), car en ce point rt − s2 < 0. Par contre la fonction f admet un minimum local
en (x2 , y2 ) et en (x3 , y3 ) car en ces points rt − s2 > 0 et r > 0.
– Les extrémums locaux de l’application:
xy
f : IR∗2
+ −→ IR : f (x, y) = .
(1 + x)(1 + y)(x + y)
Les points critiques de f sont les points (x, y) qui vérifient
∂f y(y − x2 ) ∂f x(x − y 2 )
(x, y) = = 0, et (x, y) = =0
∂x (1 + y)(1 + x)2 (x + y)2 ∂y (1 + x)(1 + y)2 (x + y)2
ce qui donne l’unique point critique (x0 , y0 ) = (1, 1).
Un développement limité à l’ordre 2 nous montre que
1 t2 − st + s2
f (1 + t, 1 + s) = − + o(s2 + t2 ).
8 32
Alors, au voisinage de (1, 1), f (x, y) − 1/8 est strictement négative, et f présente un
maximum local en (1, 1).
Solution .11 La fonction f définie par f (x, y) = x3 + y 3 − 3xy − 1 est de classe
∂f
C ∞ sur IR2 . D’autre part, f (0, 1) = 0 et (0, 1) = 3 6= 0. Alors, d’après le théorème
∂y
des fonctions implicites, il existe une et une seule fonction ϕ : I −→ IR, définie sur un
voisinage ouvert de 0, de classe C ∞ telle que ϕ(0) = 1 et pour tout x ∈ I, f (x, ϕ(x)) = 0.
Comme ϕ est de classe C ∞ , alors elle admet un développement limité à tout ordre au
voisinage de 0.
En posant, ϕ(x) = 1 + ax + bx2 + cx3 + o(x3 ), et en remplaçant dans f (x, ϕ(x)) = 0
nous obtenons
3(a − 1)x + 3(a2 − a + b)x2 + (a3 + 6ab + 3c − 3b + 1)x3 + o(x3 ) = 0.
2
L’unicité du dévelppement montre que a = 1, b = 0, et c = − . D’où
3
2 3
ϕ(x) = 1 + x − x + o(x3 )
3
42 Fonctions de plusieurs variables

Solution .12 Il est immédiat que la fonction f définie par f (x, y) = ex+y + y − 1
∂f
est de classe C ∞ sur IR2 . D’autre part, f (0, 0) = 0 et (0, 0) = 2 6= 0. Alors, d’après
∂y
le théorème des fonctions implicites, il existe une et une seule fonction ϕ : I −→ IR,
définie sur un voisinage ouvert de 0, de classe C ∞ telle que ϕ(0) = 0 et pour tout x ∈ I,
f (x, ϕ(x)) = 0.
Comme ϕ est de classe C ∞ , alors elle admet un développement limité à tout ordre au
voisinage de 0.
En posant, ϕ(x) = ax + bx2 + cx3 + o(x3 ), et en remplaçant dans x + ϕ(x) − Log (1 −
ϕ(x)) = 0 nous obtenons

a2 2 a3
(2a + 1)x + (2b + )x + (2c + ab + )x3 + o(x3 ) = 0.
2 3
1 1 1
L’unicité du dévelppement montre que a = − , b = − , et c = . D’où
2 16 192
x x2 x3
ϕ(x) = − − + + o(x3 )
2 16 192

Solution .13 i. Étude de f (x, y) = y 3 + y + x = 0.


Pour x ∈ IR fixé, la fonction y 7→ y 3 + y + x est continue et strictement croissante sur IR
et tend vers −∞ en −∞, et vers +∞ en +∞. Alors il existe une et une seule solution
y = ϕ(x) de l’équation y 3 + y + x = 0. Il existe une unique application ϕ : IR −→ IR
telle que f (x, ϕ(x)) = 0.
∂f
Soit x0 ∈ IR, nous avons f (x0 , ϕ(x0 )) = 0 et (x0 , ϕ(x0 )) = 1 + 3ϕ2 (x0 ) 6= 0, alors
∂y
d’après le théorème des fonctions implicites l’équation f (x, y) = 0 définit au voisinage
Ix0 de x0 une seule fonction ψx0 de classe C ∞ , (car f est C ∞ sur IR2 ), telle que
ψx0 (x0 ) = ϕ(x0 ) et pour tout x ∈ Ix0 nous avons f (x, ψx0 (x)) = 0. Alors ψx0 est la
restriction de ϕ à Ix0 et par conséquent ϕ est de classe C ∞ sur Ix0 . On conclut que ϕ
est C ∞ sur IR car x0 est arbitraire dans IR.

ii. Étude de f (x, y) = y 3 + yex + x2 = 0.


Pour x ∈ IR fixé, la fonction y 7→ y 3 + yex + x2 est continue et strictement croissante sur
IR et tend vers −∞ en −∞, et vers +∞ en +∞. Alors il existe une et une seule solution
y = ϕ(x) de l’équation y 3 + yex + x2 = 0. Il existe une unique application ϕ : IR −→ IR
telle que f (x, ϕ(x)) = 0.
∂f
Soit x0 ∈ IR, nous avons f (x0 , ϕ(x0 )) = 0 et (x0 , ϕ(x0 )) = ex0 + 3ϕ2 (x0 ) 6= 0, alors
∂y
d’après le théorème des fonctions implicites l’équation f (x, y) = 0 définit au voisinage
Solutions 43

Ix0 de x0 une seule fonction ψx0 de classe C ∞ , (car f est C ∞ sur IR2 ), telle que
ψx0 (x0 ) = ϕ(x0 ) et pour tout x ∈ Ix0 nous avons f (x, ψx0 (x)) = 0. Alors ψx0 est la
restriction de ϕ à Ix0 et par conséquent ϕ est de classe C ∞ sur Ix0 . On conclut que ϕ
est C ∞ sur IR car x0 est arbitraire dans IR.

iii. Étude de f (x, y) = y 5 + y(1 + x2 ) + 1 = 0.


Pour x ∈ IR fixé, la fonction y 7→ y 5 +y(1+x2 )+1 est continue et strictement croissante
sur IR et tend vers −∞ en −∞, et vers +∞ en +∞. Alors il existe une et une seule
solution y = ϕ(x) de l’équation y 5 + y(1 + x2 ) + 1 = 0. Il existe une unique application
ϕ : IR −→ IR telle que f (x, ϕ(x)) = 0.
∂f
Soit x0 ∈ IR, nous avons f (x0 , ϕ(x0 )) = 0 et (x0 , ϕ(x0 )) = 1 + x20 + 5ϕ4 (x0 ) 6= 0,
∂y
alors d’après le théorème des fonctions implicites l’équation f (x, y) = 0 définit au
voisinage Ix0 de x0 une seule fonction ψx0 de classe C ∞ , (car f est C ∞ sur IR2 ),
telle que ψx0 (x0 ) = ϕ(x0 ) et pour tout x ∈ Ix0 nous avons f (x, ψx0 (x)) = 0. Alors ψx0
est la restriction de ϕ à Ix0 et par conséquent ϕ est de classe C ∞ sur Ix0 . On conclut
que ϕ est C ∞ sur IR car x0 est arbitraire dans IR.

Solution .14 La fonction g définie sur IR par g(y) = y 2n+1 + y est un


homéomorphisme strictement croissante de IR sur IR. De plus g est de classe C ∞ et
vérifie g 0 (y) = 1 + (2n + 1)y 2n > 0 sur IR. On conclut que g est un C ∞ -difféomorphisme
de IR sur IR. Nous remarquons enfin que ϕ(x) = g −1 (x) est bien de classe C ∞ sur IR.
En effectuant le changement de variable t = g(y) dans l’intégrale considérée, nous
obtenons
Z x Z ϕ(x)
ϕ2 (x) 2n + 1 2n+2
ϕ(t) dt = y(1 + (2n + 1)y 2n ) dy =
+ ϕ (x)
0 0 2 2n + 2
ϕ2 (x) 2n + 1 2n + 1 n
= + ϕ(x)(x − ϕ(x)) = xϕ(x) − ϕ2 (x)
2 2n + 2 2(n + 1) 2(n + 1)

Solution .15 La fonction f définie par f (x, y, z) = x3 +y 3 +z 3 −xz−x+y−2z+1 est


∂f
de classe C ∞ sur IR3 . D’autre part, f (0, 0, 1) = 0 et (0, 0, 1) = 1 6= 0. Alors, d’après
∂z
le théorème des fonctions implicites, il existe une et une seule fonction ϕ : V −→ IR,
définie sur un voisinage ouvert de (0, 0), de classe C ∞ telle que ϕ(0, 0) = 1 et pour tout
(x, y) ∈ V , f (x, y, ϕ(x, y)) = 0.
Comme ϕ est de classe C ∞ , alors elle admet un développement limité à tout ordre au
voisinage de (0, 0).
44 Fonctions de plusieurs variables

ϕ(x, y) = 1 + px + qy + rx2 + sxy + ty 2 + o(x2 + y 2 )

avec

∂ϕ ∂ϕ 1 ∂2ϕ ∂2f 1 ∂2ϕ


p= (0, 0), q= (0, 0), r= (0, 0), s= (0, 0), t= (0, 0).
∂x ∂y 2 ∂x2 ∂x ∂y 2 ∂y 2

En remplaçant dans f (x, y, ϕ(x, y)) = 0, nous obtenons

(p − 2)x + (q + 1)y + (r + 3p2 − p)x2 + (s + 6pq − q)xy + (t + 3q 2 )y 2 + o(x2 + y 2 ) = 0.

Donc
p = 2, q = −1, r = −10, s = 11, t = −3.

On conclut que

∂2ϕ ∂2ϕ ∂2ϕ


(0, 0) = −20, (0, 0) = 11, (0, 0) = −6.
∂x2 ∂x ∂y ∂y 2

Solution .16 Étude des formes différentielles ω = P (x, y) dx + Q(x, y) dy, avec
• P (x, y) = 2xy et Q(x, y) = x2 − y 2 . Nous avons, sur IR2 ,

∂P ∂Q
(x, y) = 2x, (x, y) = 2x.
∂y ∂x

Alors ω est une forme différentielle fermée. Les primitives de ω sont les fonctions f
y3
définies par f (x, y) = x2 y − + λ, avec λ ∈ IR.
3
• P (x, y) = (2 − 9xy )x et Q(x, y) = (4y 2 − 6x3 )y. Nous avons, sur IR2 ,
2

∂P ∂Q
(x, y) = 18x2 y, (x, y) = 18x2 y.
∂y ∂x

Alors ω est une forme différentielle fermée. Les primitives de ω sont les fonctions f
définies par f (x, y) = x2 − 3x3 y 2 + y 4 + λ, avec λ ∈ IR.
• P (x, y) = 1 + y 2 sin 2x et Q(x, y) = −2y cos2 x. Nous avons, sur IR2 ,

∂P ∂Q
(x, y) = 2y sin 2x, (x, y) = 2y sin 2x.
∂y ∂x

Alors ω est une forme différentielle fermée. Les primitives de ω sont les fonctions f
définies par f (x, y) = x − y 2 cos2 x + λ, avec λ ∈ IR.
• Étude de la forme différentielle ω = P (x, y, z) dx + Q(x, y, z) dy + R(x, y, z) dz,
avec
Solutions 45

P (x, y, z) = x2 + 2xy + 2xz, Q(x, y, z) = x2 + y 2 , R(x, y, z) = x2 + z 2 .

Nous avons, sur IR3 ,

∂P ∂Q ∂Q ∂R ∂R ∂P
(x, y, z) = (x, y, z), (x, y, z) = (x, y, z), (x, y, z) = (x, y, z).
∂y ∂x ∂z ∂y ∂x ∂z

Alors ω est une forme différentielle fermée. Les primitives de ω sont les fonctions f
1
définies par f (x, y, z) = (x3 + y 3 + z 3 ) + (y + z)x2 + λ, avec λ ∈ IR.
3

Solution .17 • La forme différentielle y 2 λ(x + y) dx + x2 λ(x + y) dy est fermée si,


et seulement si,
∂ ¡ 2 ¢ ∂ ¡ 2 ¢
y λ(x + y) = x λ(x + y) .
∂y ∂x
Ce qui est équivalent, après dérivation et simplification, à

(x + y)λ0 (x + y) + 2λ(x + y) = 0

Il suffit alors de choisir λ une solution de tλ0 (t) + 2λ(t) = 0, par exemple, λ(t) = 1/t2 .
Dans ce cas ω1 = µω s’écrit sous la forme

y2 x2
ω1 = dx + dy.
(x + y)2 (x + y)2

xy
Les primitives de ω1 sont les fonctions f (x, y) = + c avec c ∈ IR.
x+y
• La forme différentielle (x2 + y 2 − 1)λ(x) dx − 2yλ(x) dy est fermée si, et seulement
si,
∂ ¡ 2 ¢ ∂
(x + y 2 − 1)λ(x) = (−2yλ(x)) .
∂y ∂x
Ce qui est équivalent, après dérivation et simplification, à λ0 (x) + λ(x) = 0. Il suffit
alors de choisir λ(x) = e−x .
Dans ce cas ω1 = µω s’écrit comme

ω1 = (x2 + y 2 − 1)e−x dx − 2ye−x dy.

Les primitives de ω1 sont les fonctions f (x, y) = −(y 2 + (x + 1)2 ) e−x + c avec c ∈ IR.
• Il est immédiat que la forme diférentielle ω = y dx − x dy devient fermée si l’on
la divise par xy. Donc
dx dy
ω1 = − .
x y
46 Fonctions de plusieurs variables
¯ ¯
¯ x¯
Les primitives de ω1 sont les fonctions f (x, y) = Log ¯¯ ¯¯ + c avec c ∈ IR.
y
• La forme différentielle (x + y − 1)λ(x − y ) dx − 2xyλ(x2 − y 2 ) dy est fermée
2 2 2 2

si, et seulement si,


∂ ¡ 2 ¢ ∂ ¡ ¢
(x + y 2 − 1)λ(x2 − y 2 ) = −2xyλ(x2 − y 2 ) .
∂y ∂x
Ce qui est équivalent, après dérivation et simplification, à

(1 + x2 − y 2 )λ0 (x2 − y 2 ) + 2λ(x2 − y 2 ) = 0

Il suffit alors de choisir λ une solution de (1 + t)λ0 (t) + 2λ(t) = 0, par exemple,
λ(t) = 1/(1 + t)2 .
Dans ce cas ω1 = µω s’écrit sous la forme
x2 + y 2 − 1 2xy
ω1 = 2 2 2
dx − dy.
(1 + x − y ) (1 + x2 − y 2 )2
x
Les primitives de ω1 sont les fonctions f (x, y) = + c avec c ∈ IR.
y2 − x2 − 1

Solution .18 La forme considérée est fermée si, et seulement si, f (y)g 0 (z) = y pour
tout y et z. Ce qui démontre qu’il existe a ∈ IR∗ tel que f (y) = ay pour tout y ∈ IR et
que g(z) = (b + z)/a pour tout z ∈ IR. Alors f (y)g(z) = yz + by pour tout (y, z) ∈ IR2 .
La forme ω devient

y2
ω = 2xz dx + y(z + b) dy + (x2 + ) dz.
2
y2 b
Les primitives de ω sont les fonctions f (x, y, z) = (x2 + ) z + y 2 + c avec c ∈ IR.
2 2
Z
Solution .19 a. Calcul de l’intégrale curviligne ω, où ω(x, y) = (x − y 3 ) dx +
Γ
x3 dy, et Γ le cercle d’équation x2 + y 2 = 1 parcouru une fois
Z dans le sens positif.
Comme ω1 (x, y) = xdx est une forme fermée sur IR2 , alors ω1 = 0 et par conséquent
Γ
Z Z Z 2π ¡ 3 ¢
ω= 3 3
x dy − y dx = cos θd(sin θ) − sin3 θd(cos θ)
Γ Γ 0
Z 2π Z
4 4 1 2π 3π
= (cos θ + sin θ) dθ = (3 + cos 4θ) dθ = .
0 4 0 2
Z
b. Calcul de l’intégrale curviligne ω, où ω(x, y, z) = xyz dz, et Γ le cercle
Γ
sin t sin t
paramétré par x = cos t, y = √ , et z = √ , t allant de 0 à 2π.
2 2
Solutions 47

Z Z Z 2π Z 2π
1 2 1 2 π
ω = xyz dz = √ cos θ sin θ dθ = √ (1 − cos 4θ) dθ = √ .
Γ Γ 2 2 0 16 2 0 8 2
Z
c. Calcul de l’intégrale curviligne ω, où ω(x, y, z) = z dx + x dy + y dz, et Γ le
Γ
cercle d’équations x2 + y 2 + z 2 = 1, x + z = 1, parcouru dans le sens positif, le plan

→ 1 1
x + z = 1 étant orienté par le vecteur normal K = ( √ , 0, √ ).
2 2
1 1 √
Le cercle considéré est de centre ( , 0, ) et de rayon 1/ 2. Le plan du cercle Γ est
2 2

→ 1 1 −

engendré par les deux vecteurs orthonormaux I = ( √ , 0, − √ ), et J = (0, 1, 0), qui
2 2

→ −
→ −→ − →
forment avec K une base orthonormale directe ( I , J , K ) de IR3 . La paramétrisation
demandée de Γ est, par conséquent,
1 1 cos θ −
→ sin θ →

t 7→ ( , 0, ) + √ I + √ J
2 2 2 2
soit,
1 + cos θ sin θ 1 − cos θ
x(θ) = , y(θ) = √ , z(θ) = , (0 ≤ θ ≤ 2π).
2 2 2
Z Z 2π µ ¶
1 − cos θ 1 + cos θ sin2 θ
ω= − sin θ + √ cos θ + √ dθ
Γ 0 4 2 2 2 2
Z 2π µ ¶
1 cos θ sin θ sin 2θ π
= √ + √ − + dθ = √ .
0 2 2 2 2 4 8 2
Z
(x − y) dx + (x + y) dy
d. Calcul de l’intégrale curviligne ω, où ω(x, y) = , et Γ
Γ x2 + y 2
est le carré ABCD de sommets A(a, a), B(−a, a), C(−a, −a), et D(a, −a) avec a > 0
parcouru une fois dans le sens positif.
Notons que
Z Z 1
t+1
ω = dt, avec x(t) = −at, y(t) = a
AB t2 + 1
Z Z−1
1
t+1
ω = dt, avec x(t) = −a, y(t) = −at
BC t2 + 1
Z Z−1
1
t+1
ω = dt, avec x(t) = at, y(t) = −a
t2 + 1
ZCD Z−1
1
t+1
ω = dt, avec x(t) = a, y(t) = at.
DA −1 t2 + 1
Alors, Z Z Z Z
1 1 1
t+1 −t + 1 1
ω=4 dt = 4 dt = 4 dt = 2π.
Γ −1 t2 + 1 −1 t2 + 1 −1 t2 +1
OKMRAN
OUBA
SURFACES

I. Généralités

Nous rappelons qu’un domaine de IR2 est une partie ouverte et connexe.
Nous appelons une surface paramétriée tout couple (A, f ) constitué d’un domaine A
de IR2 et d’une application f continue de A dans IR3 .

− − → − →
f (A) se nomme le support de la surface paramétrée. Dans un repère (O, i , j , k ) de
IR3 , la fonction f est définie par ses coordonnées :
 
α(u, v)
f (u, v) =  β(u, v)  ,
γ(u, v)

où (u, v) ∈ IR2 est le couple de paramètres qui parcourt A, et {α, β, γ} étant trois
fonctions numériques continues sur A.
En particulier, la donnée d’une fonction numérique z = g(x, y) continue sur un domaine
A, définit une surface paramètrée (A, f ) où la fonction vectorielle f est définie par
f (x, y) = t [x, y, g(x, y)] dans le repère choisi. Une telle représentaion paramétrée sera
nommée cartésienne et notée (A, g).
Plus généralement, soit h une application continue de IR3 dans IR. On appelle surface
d’équation h(x, y, z) = 0 l’ensemble des points (x, y, z) ∈ IR3 solutions de l’équation
h(x, y, z) = 0.
Dans la suite nous allons étudier quelques exemples de surfaces.

II. Surfaces cylindriques

Définition : On se donne un arc Γ dans IR3 et une direction δ ∈ IR3 . L’ensemble S


des droites parallèles à δ et qui rencontrent Γ est dit surface cylindrique. Γ est une
directrice de S et les droites parallèles à δ et rencontrant Γ sont dites génératrices
de S.
Soient Γ un arc admettant la paramétrisation f : I −→ IR3 et δ ∈ IR3 . Pour tout t ∈ I,
f (t) est un point de Γ. Pour qu’un point M appartienne à la droite passant par f (t) et
2 Surfaces
−−−−→
parallèle à δ il faut, et il suffit, qu’il existe un scalaire u ∈ IR tel que f (t)M = uδ. On
obtient ainsi l’équation vectorielle de la surface cylindrique S.

Exemple : Soient, dans un repère orthonormal de IR3 , la direction δ = (0, 1, 1) et l’arc


Γ paramétré par  
3t2
t 7→  t3 − 3t  .
t3 + 3t
Les équations paramètriques de la surface cylindrique S, de directrice Γ et dont les
génératrices sont parallèles à δ sont dans le repère donné

 x(t, u) = 3t2
y(t, u) = t3 − 3t + u

z(t, u) = t3 + 3t + u

les pararamètres t et u parcourent IR.


Si l’on veut obtenir l’équation cartèsienne de cette surface S, on élimine les paramètres
t et u entre les trois équations précédentes. En éliminant d’abord u, on obtient

x = 3t2 , z − y = 6t

et l’élimination de t conduit à l’équation

(y − z)2 = 12x. (S)

Le plan d’équation z = 0 coupe S suivant la parabole d’équation y 2 − 12x = 0. On dit


que (S) est un cylindre parabolique.

On peut généraliser la définition d’une surface cylindrique de façon que la directrice de


la surface n’intervienne plus .

Définition : On appelle surface cylindrique toute surface engendrée par une famille à
un paramètre de droites ayant une direction fixe.
Si la famille de droites (Dt )t∈I possède la direction δ, L’équation vectorielle d’une
−−−−→
droite Dt est du type f (t)M = uδ, (u ∈ IR). Lorsque (t, u) parcourt I × IR, on a encore
l’équation vectorielle de S et l’on reconnaı̂t que t 7→ f (t) est une paramétrisation d’une
directrice de S.
Examinons le cas où la famille de droites dépend de deux paramètres u et v liés par
une relation de façons que toute droite Du,v de la famille puisse être considérée comme
l’intersection de deux plans Pu et Qv parallèles à deux plans fixes d’équations respectives
Surfaces cylindriques 3

P = 0 et Q = 0 (P ∩ Q définit la direction δ). Toute Du,v de la famille a alors des


équations du type ½
P (x, y, z) = u
Q(x, y, z) = v
les deux paramètres u et v étant liés par la relation f (u, v) = 0.
L’élimination des paramètres u et v est immédiate et donne l’équation cartésienne de
la surface cylindrique S:
f (P, Q) = 0. (S)

Réciproquement, une équation de ce type où P = 0 et Q = 0 sont les équations de deux


plans sécants représente une surface cylindrique S. En effet, pour toute solution réelle
(x, y, z) de l’équation (S), il existe deux nombres réels u et v tels que P (x, y, z) = u et
Q(x, y, z) = v. L’intersection Du,v des deux plans représentés par ces équations est une
droite parallèle à l’intersection des plans P = 0 et Q = 0, et les valeurs u et v vérifient
la condition f (u, v) = 0. Par suite, (S) représente une surface cylindrique.
Nous avons donc démontré le théorème suivant:

Théorème II.1: Toute équation du type f (P, Q) = 0 où P = 0 et Q = 0 sont les


équations de deux plans sécants suivant ue droite D, représente une surface cylindrique
dont les génératrices dont parallèles à D.

Définition : On se donne une surface Σ dans IR3 et une direction δ ∈ IR3 . L’ensemble
S des droites parallèles à δ et qui sont tangeantes à Σ est dit surface cylindrique
circonscrit à Σ parallèlement à δ.

Si Σ est une surface d’équation f (x, y, z) = 0, où f est une fonction polynôme à
trois variables, et si (α, β, γ) sont les coordonnées de la direction δ, alors un point
M = (X, Y, Z) appartient à la surface cylindrique S circonscrit à Σ parallèlement à δ
si, et seulement si, la droite Dt paramétrée par
 
X + αt
t 7→  Y + βt 
Z + γt

est tangente à Σ, c’est à dire si, et seulement si, l’équation

f (X + αt, Y + βt, Z + γt) = 0

admet une racine double en t.


4 Surfaces

Exemple : Soit Σ la surface d’équation

x2 y2 z2
+ + = 1.
a2 b2 c2

Déterminons le cylindre S circonscrit à Σ parallèlement à la direction δ = (1, 1, 1).


En effet, la droite Dt paramétrée par
 
X +t
t 7→  Y + t 
Z +t

est tangente à Σ, c’est à dire si, et seulement si, l’équation

(X + t)2 (Y + t)2 (Z + t)2


+ + =1
a2 b2 c2

admet une racine double en t, ce qui équivalent à dire que le discriminant de l’équation
de second degré suivante est nul.

µ ¶ µ ¶
1 1 1 2 X Y Z X2 Y2 Z2
+ + t +2 + + t+ + + − 1 = 0.
a2 b2 c2 a2 b2 c2 a2 b2 c2
i.e. µ ¶2 µ ¶µ ¶
X Y Z 1 1 1 X2 Y2 Z2
2
+ 2+ 2 = 2
+ 2+ 2 + 2 + 2 −1 ,
a b c a b c a2 b c
soit
a2 (Y − Z)2 + b2 (Z − X)2 + c2 (X − Y )2 = a2 b2 + b2 c2 + c2 a2 . (S)

III. Surfaces coniques

Définition : On se donne un arc Γ dans IR3 et un point s ∈ IR3 . L’ensemble S des


droites passant par s et qui rencontrent Γ est dit surface conique. s est le sommet de
S, Γ est une directrice de S et les droites passant par s et rencontrant Γ sont dites
génératrices de S.
Soient Γ un arc admettant la paramétrisation f : I −→ IR3 et s ∈ IR3 . Pour tout t ∈ I,
f (t) est un point de Γ. Pour qu’un point M appartienne à la droite passant par f (t)
−→ −−−→
et s il faut, et il suffit, qu’il existe un scalaire u ∈ IR tel que sM = usf (t). On obtient
ainsi l’équation vectorielle de la surface conique S.
Surfaces coniques 5

Exemple : Soient, dans un repère orthonormal de IR3 , le sommet s = (a, b, c), (abc 6= 0),
et l’ellipse Γ paramétrée par  
a cos t
t 7→  b sin t  .
0

Les équations paramètriques de la surface conique S, de directrice Γ et dont les


génératrices passent par s sont dans le repère donné

 x(t, u) = a + u(a cos t − a)
y(t, u) = b + u(b sin t − b)

z(t, u) = c + u(0 − c)

le couple de pararamètres (t, u) parcourant [0, 2π[×IR.


Si l’on veut obtenir l’équation cartèsienne de cette surface S, on élimine les paramètres
t et u entre les trois équations précédentes. En éliminant d’abord u, on obtient

x z ³ z´ y z ³ z´
− = 1− cos t, − = 1− sin t
a c c b c c

et l’élimination de t conduit à l’équation


³x z ´2 ³ y z ´2 ³ z ´2
− + − = 1− . (S)
a c b c c

On peut généraliser la définition d’une surface conique de façon que la directrice de la


surface n’intervienne plus .

Définition : On appelle surface conique toute surface engendrée par une famille à un
paramètre de droites passant par un point fixe.
Si les droites de la famille de droites (Dt )t∈I passe par s, L’équation vectorielle de Dt
−→
est du type sM = uf (t), (u ∈ IR). Lorsque (t, u) parcourt I × IR, on a encore l’équation
vectorielle de S.
Examinons le cas où la famille de droites dépend de deux paramètres u et v liés par
une relation de façons que toute droite Du,v de la famille puisse être considérée comme
l’intersection de deux plans

P − uR = 0, et Q − vR = 0,

P = 0, Q = 0, R = 0 étant les équations de trois plans fixes ayant en commun un point


s, et un seul, les deux paramètres u et v étant liés par la relation f (u, v) = 0.
6 Surfaces

L’élimination des paramètres u et v est immédiate et donne l’équation cartésienne de


la surface conique S engendrée par la famille proposée:
P Q
f ( , ) = 0. (S)
R R
Réciproquement, une équation de ce type où P = 0, Q = 0, R = 0 sont les équations
de trois plans ayant un, et un seul, point commun représente une surface conique S. En
effet, pour toute solution réelle (x, y, z) de l’équation (S), il existe deux nombres réels
u et v tels que

P (x, y, z) − uR(x, y, z) = 0 et Q(x, y, z) − vR(x, y, z) = 0.

L’intersection Du,v des deux plans représentés par ces deux équations appartient à une
famille de droites passant par s, les paramètres u et v vérifient la condition f (u, v) = 0.
Par suite, (S) représente une surface conique.
Nous avons donc démontré le théorème suivant:
µ ¶
P Q
Théorème III.1: Toute équation du type f , = 0 où P = 0, Q = 0, R = 0
R R
sont les équations de trois plans ayant un, et un seul, point commun s, représente une
surface conique de sommet s.

Corollaire III.2. Toute équation du type g(P, Q, R) = 0 où g est une fonction
homogène de IR3 dans IR et où P = 0, Q = 0 R = 0 sont les équations de trois
plans ayant un, et un seul, point commun s, représente une surface conique de sommet
s.

Définition : On se donne une surface Σ dans IR3 et un point s ∈ IR3 . L’ensemble S des
droites passant par s et qui sont tangeantes à Σ est dit surface conique circonscrit
à Σ de sommet s.

Si Σ est une surface d’équation f (x, y, z) = 0, où f est une fonction polynôme à trois
variables, et si (α, β, γ) sont les coordonnées du point s, alors un point M = (X, Y, Z)
appartient à la surface conique S circonscrit à Σ de sommet s si, et seulement si, la
droite Dt paramétrée par  
α + t(X − α)
t 7→  β + t(Y − β) 
γ + t(Z − γ)
est tangente à Σ, c’est à dire si, et seulement si, l’équation

f (α + t(X − α), β + t(Y − β), γ + t(Z − γ)) = 0

admet une racine double en t.


Surfaces de révolution 7

IV. Surfaces de révolution

Définition : On donne un arc Γ dans IR3 et une droite D. La surface S engendrée par
la famille des arcs (Γα )α∈IR déduits de Γ par rotations d’axe D et d’angles α est dite
surface de révolution d’axe D. L’arc Γ se nomme génératrice de la surface S.

Nous supposons l’espace euclidien IR3 rapporté à un



→ − → → − −

repère orthonormal (O, i , j , k ) avec O ∈ D et k
un vecteur unitaire de D. (D)
M
Soit (voir la figure ci-contre) un arc Γ ayant une α
f (t)
paramétrisation t 7→ f (t), le paramètre t décrivant
v
l’intervalle I de IR. Nous supposons que Γ est donnée ϕ(t) Γ(t)

en coordonnées cylindriques dans le repère précédent:



 x = u(t) cos ϕ(t)
f (t) : y = u(t) sin ϕ(t)

z = w(t)

u, ϕ et w étant trois fonctions numériques continues sur I. Pour tout t ∈ I, le point


M déduit du point f (t) par rotation d’axe D et d’angle α ∈ IR a pour coordonnées
cylindriques

x = u(t) cos(ϕ(t) + α)
M : y = u(t) sin(ϕ(t) + α)

z = w(t)

et par conséquent, appartient à la surface S de représentation paramétrique:



x = u(t) cos v
y = u(t) sin v (S)

z = w(t)

quand (t, v) parcourt I × IR.


Réciproquement, tout point M (t, v) de cette surface S est déduit du point corréspondant
f (t) ∈ Γ par rotation d’axe D et d’angle v − ϕ(t) ∈ IR. Par conséquent S est la surface
de révolution d’axe D et de génératrice Γ.
8 Surfaces

Pour tout point M de la surface de révolution S, le cercle CM d’axe D et contenant M


est inclu dans S. On le nomme parallèle de point M . Tout plan contenant l’axe D se
nomme plan méridien de S. Tout plan méridien coupe S suivant un arc plan (ou une
réunion ’arcs plans) admettant évidemment D pour axe de symétrie et est lui-même la
réunion γ ∪ γ 0 de deux arcs plans (ou de deux réunions d’arcs plans) symétriques par
rapport à D. On nomme γ (ou γ 0 ) méridienne de S. La méridienne de S joue aussi très
souvent le rôle de génératrice de la surface S.

On peut généraliser la définition d’une surface de révolution de façon que la génératrice


de la surface n’intervienne plus .

Définition : On appelle surface révolution d’axe D, toute surface engendrée par une
famille à un paramètre de cercles axés sur D.

− − → − →
Dans un repère orthonormal (O, i , j , k ), soit P = 0 l’équation d’un plan fixe
perpendiculaire à D et Σ = 0 l’équation d’une sphère fixe centrée sur D.
Tout cercle Cu,v axé sur D a des équations du type
½
Σ = u,
P = v.

où u et v sont deux parmètres réels. Cette famille de cercles dépendra en dernier ressort
d’un seul paramètre si u et v sont liés par une relation f (u, v) = 0. L’élimination des
paramètres u et v est immédiate et conduit à l’équation

f (Σ, P ) = 0, (S)

qui est l’équation de la surface de révolution engendrée par la famille de cercles.


Réciproquement, toute équation de ce type où P = 0 est l’équation d’un plan et Σ = 0
l’équation d’une sphère représente une surface de révolution. En effet, pour tout solution
réelle (x, y, z) de cette équation, il existe deux nombres réels u et v tels que

Σ(x, y, z) = u, P (x, y, z) = v, avec f (u, v) = 0.

Par le point M (x, y, z) de la surface représentée par (S) passe un cercle Cu,v axé sur
la perpendiculaire D menée du centre de Σ au plan P , les paramètres u et v étant liés
par la relation f (u, v) = 0. Par suite (S) représente une surface de révolution S d’axe
D. Nous avons donc démontré le théorème suivant:
Surfaces de révolution 9

Théorème IV.1: Toute équation du type f (Σ, P ) = 0 où Σ = 0, est léquation d’une
sphère et P = 0 l’équation d’un plan, représente une surface de révolution dont l’axe
est la perpendiculaire menée du centre de la sphère Σ au plan P .

Exemple : Étudions la surface de révolution d’axe D engendrée par une droite ∆ non
coplanaire à D.

→ − → → −
Choisissons un repère orthogonal (O, i , j , k ), la droite D coı̈ncidant avec l’axe Oz et
la perpendiculaire commune à D et ∆ étant l’axe Ox. Les équations de ∆ dans un tel
repère sont du type
x = a, z = ytg ϕ, avec atg ϕ 6= 0 (∆)

où a et ϕ sont deux nombres réels fixes.


Toute sphère Σu centrée en O ∈ D a pour équation

x2 + y 2 + z 2 = u. (1)

Tout plan Pv perpendiculaire à D a pour équation

z = v. (2)

Pour que le cercle Σu ∩ Pv rencontre ∆, il faut, et il suffit, que le système des quatre
équations (∆), (1), (2) ait une solution (x, y, z) qui nécessairement est

x = a, y = vcotg ϕ, z = v,

et cette solution existe effectivement si, et seulement si, elle vérifie (1):
v2
a2 + = u. (3)
sin2 ϕ
L’élimination de u et v entre (1), (2) et (3) donne l’équation de la surface de révolution
S étudiée:
x2 + y 2 − z 2 cotg 2 ϕ = a2 . (S)

Il s’agit d’une quadrique de révolution. La section de S par le plan méridien y = 0 a


pour équations
x2 − z 2 cotg 2 ϕ = a2 , y = 0.

C’est une hyperbole de centre O, d’axe focal Ox, d’axe non transverse D, les sommets
étant (a, 0, 0) et (−a, 0, 0), et les asymptotes faisant, comme la droite ∆ donnée au
π
départ, le même angle − ϕ avec l’axe D.
2
La surface S est engendrée par cette hyperbole en tournant autour de son axe non
transverse D. S est nommée hyperboloı̈de de révolution.
10 Surfaces


→ − → − →
Exemple : On se donne un repère orthogonal (O, i , j , k ) de IR3 . Déterminer la
surface de révolution S d’axe D(x = y = z) engendrée par le cercle Γ:

z = 0, x2 + y2 − 2ax = 0.

L’axe D passe par O ; on peut choisir une sphère Σ de centre O.


Toute sphère Σu centrée en O a pour équation

x2 + y 2 + z 2 = u. (1)

Tout plan Pv perpendiculaire à D a pour équation

x + y + z = v. (2)

Pour que le cercle Σu ∩ Pv rencontre Γ, il faut, et il suffit, que le système constitué


des deux équations de Γ et des équations (1) et (2) ait une solution (x, y, z) qui,
nécessairement, est
u u
x= , y=v− , z = 0,
2a 2a
et cette solution existe effectivement si, et seulement si, elle vérifie (1):

u2 ³ u ´2
+ v− = u,
4a2 2a

c’est à dire
u2 − 2auv + 2a2 (v 2 − u) = 0. (3)

L’élimination de u et v entre (1), (2) et (3) donne l’équation de la surface de révolution


S étudiée:

(x2 + y 2 + z 2 )2 − 2a(x2 + y 2 + z 2 )(x + y + z) + 4a2 (xy + yz + zx) = 0


Quadriques 11

V. Quadriques


→ − → − →
Dans ce paragraphe E désigne un espace euclidien de dimension 3, et (O, i , j , k ) un
repère orthonormal de E.
Définition : On appelle Quadrique la surface d’équation, dans un repère orthonormal
de E,
q(x, y, z) = 0

où q est une fonction polynôme de degré total 2.


L’équation d’une quadrique Q est donc du type

q(x, y, z) = ax2 + by 2 + cz 2 + 2dxy + 2eyz + 2f zx + 2rx + 2sy + 2tz + u = 0.

On note alors

ϕ(x, y, z) = ax2 + by 2 + cz 2 + 2dxy + 2eyz + 2f zx,

c’est le polynôme homogène de degré 2 de q, c’est à dire une forme quadratique à trois
variables dont la matrice est  
a d f
M = d b e .
f e c
Si X est la matrice colonne des coordonnées x, y, z de tout point p ∈ E dans le repère

− −→ − →
(O, i , j , k ), on a

ϕ(x, y, z) =t XM X, (1)
q(x, y, z) =ϕ(x, y, z) + 2rx + 2sy + 2tz + u = 0. (2)

→ → −
− − →
Nous allons passer du repère orthogonal (O, i , j , k ) à un autre repère orthogonal

→ → − − →
(O0 , I , J , K ). Ce changement est défini d’abord par la matrice colonne X0 des
coordonnées de la nouvelle origine O0 dans l’ancien repère:
 
x0 −−→ −
→ −
→ −

X0 =  y0  , avec OO0 = x0 i + y0 j + z0 k ,
z0


− −→ −→
ensuite par la matrice orthogonale P de passage de la base ( i , j , k ) à la base

− − → − →
( I , J , K ). Si l’on désigne par X et X 0 les matrices colonnes des coordonnées de tout
12 Surfaces

point p de l’espace E dans l’ancien et le nouveau repère respectivement, la relation


matricielle liant ces éléments est

X = X0 + P X 0 . (3)

L’équation de la quadrique Q dans le nouveau repère s’obtiendra en remplaçant, dans


(2), x, y, z par leurs valeurs obtenues en (3) en fonction de x0 , y 0 , z 0 :

q1 (x0 , y 0 , z 0 ) = ϕ1 (x0 , y 0 , z 0 ) + 2r1 x0 + 2s1 y 0 + 2t1 z 0 + u1 = 0, (4)

où ϕ1 est une forme quadratique à trois variables. Comme x, y, z s’expriment, dans (3),
en fonction de x0 , y 0 , z 0 par des polynômes du premier degré, ϕ1 se déduit de ϕ en posant
dans (1)
X = P X 0, d’où t
X = t X 0t P = t X 0 P −1 ,

puisque P est orthogonale, et l’on obtient

ϕ1 (x0 , y 0 , z 0 ) = t X 0 (P −1 M P )X 0 .

On sait que les matrices M et P −1 M P ont les mêmes valeurs propres et les mêmes
vecteurs propres. On peut donc énoncer le théorème suivant:

Théorème V.1: Les vecteurs propres de la matrice de la forme quadratique associée à


une quadrique Q de l’espace euclidien E sont indépendants du repère orthogonal choisi
pour représenter Q.

Définition : Les vecteurs propres de la matrice de la forme quadratique associée à une


quadrique Q de l’espace euclidien E sont nommés directions principales de Q.

La réduction de l’équation d’une quadrique Q, c’est trouver un repère orthonormal, dont


les directions des axes sont des directions principales de Q et dans lequel l’équation de
Q soit la plus simple possible.

→ − → → − −
→ −→ → −
Dans un repère orthonormal (O, I , J , K ), où I , J , K sont des vecteurs unitaires
propres de M (donc orthogonaux duex à deux par rapport à la forme quadratique ϕ),
l’équation de la quadrique Q est du type

q1 (x0 , y 0 , z 0 ) = λ1 x02 + λ2 y 02 + λ3 z 02 + 2αx0 + 2βy 0 + 2γz 0 + δ = 0. (5)

La réduction de l’équation va se poursuivre selon le nombre de valeurs propres nulles


qui existent dans le triplet (λ1 , λ2 , λ3 ).
Quadriques 13

1◦ . Aucune valeur propre n’est nulle: λ1 λ2 λ3 6= 0.


Nous décomposons alors q1 en carrés selon x, y et z respectivement:
µ ¶2 µ ¶2 µ ¶2
0 0 0 0 α 0 β 0 γ
q1 (x , y , z ) = λ1 x + + λ2 y + + λ3 z + + h = 0.
λ1 λ2 λ3
Nous effectuons ensuite une translation du repère pour amener l’origine O en un point
ω défini par
−→ α− → β− → γ →−
Oω = − I − J − K.
λ1 λ2 λ3

− − → − →
Dans le repère (ω, I , J , K ) obtenu, l’équation de Q est

λ1 x2 + λ2 y 2 + λ3 z 2 + h = 0. (6)

On voit facilement que ω est un centre de symétrie de Q, que les axes de coordonées
sont des axes de symétries et que les trois plans contenant respectivement deux de ces
axes sont plans de symétrie de Q.
Si h 6= 0, Q est une quadrique propre et si h = 0, Q est dégénérée en un cône de
sommet ω. Envisageons ces deux cas séparément.

1◦ .a. Q est une quadrique propre (h 6= 0).


Des trois nombres λ1 , λ2 , λ3 deux au moins ont le même signe. Supposons que λ1 λ2 > 0.
En changeant au besoin les signes des éléments de l’équation (6), on peut supposer
λ1 > 0 et λ2 > 0. Il existe alors trois nombres réels a, b, c positifs tels que
¯ ¯
|h| |h| ¯ h ¯
2
a = , 2
b = , c = ¯¯
2 ¯,
¯
λ1 λ2 λ3

et l’équation de la quadrique Q est

x2 y2 z2
+ + ε + ε0 = 0,
a2 b2 c2

avec (ε, ε0 ) ∈ {−1, 1}2 . Envisageons toutes les éventualités:


1◦ .a.i. Ellipsoı̈de : ε = −ε0 = 1.
L’équation de Q est
x2 y2 z2
+ + = 1.
a2 b2 c2
1◦ .a.ii. Ellipsoı̈de imaginaire : ε = ε0 = 1.
L’équation de Q est
x2 y2 z2
+ + + 1 = 0.
a2 b2 c2
14 Surfaces

Aucun point réel n’appartient à Q.


1◦ .a.iii. Hyperboloı̈de à deux nappes : ε = −ε0 = −1.
L’équation de Q est
x2 y2 z2
+ 2 − 2 + 1 = 0.
a2 b c
1◦ .a.iv. Hyperboloı̈de à une nappe : ε = −ε0 = −1.
L’équation de Q est
x2 y2 z2
+ 2 − 2 − 1 = 0.
a2 b c
1◦ .b. Q est dégénérée en un cône de sommet ω (h = 0).
Supposons λ1 et λ2 aient le même signe. Au besoin zn changeant les signes des élément
de l’équation (6), on peut supposer λ1 > 0 et λ2 > 0. Il existe alors trois nombres réels
positifs a, b, c tels que

1 1 1
a2 = , b2 = , c2 = ,
λ1 λ2 | λ3 |

et l’équation de la quadrique Q est

x2 y2 z2
+ + ε = 0,
a2 b2 c2

avec ε ∈ {−1, 1}.


Dans le cas où ε = +1, on a un cône imaginaire.
Dans le cas où ε = −1, on a un cône réel.
2◦ . Une seule valeur propre est nulle : λ3 = 0, λ1 λ2 6= 0.
Nous décomposons alors q1 , donné par (5), en carrés selon x et y:
µ ¶2 µ ¶2
0 0 0 0 α 0 β
q1 (x , y , z ) = λ1 x + + λ2 y + + 2γz 0 + h = 0.
λ1 λ2
Nous effectuons ensuite une translation du repère pour amener l’origine O en un point
ω défini par
−→ α− → β− →
Oω = − I − J.
λ1 λ2

− − → − →
Dans le repère (ω, I , J , K ) obtenu, l’équation de Q est

λ1 x2 + λ2 y 2 + 2γz 0 + h = 0. (7)

Q est une quadrique propre si γ 6= 0, et Q est dégénérée si γ = 0. Envisageons


séparément ces deux cas.
Quadriques 15

2◦ .a. Q est une quadrique propre (γ 6= 0).



→ → − − →
Dans le repère (s, I , J , K ) déduit du précédent par la translation

→=− h →

ωs

K,

l’équation de Q est
λ1 x2 + λ2 y 2 + 2γz = 0. (8)

2◦ .a.i. Paraboloı̈de elliptique. C’est le cas où λ1 λ2 > 0.


2◦ .a.ii. Paraboloı̈de hyperbolique. C’est le cas où λ1 λ2 < 0.
2◦ .b. Q est une quadrique dégénérée (γ = 0).
L’équation (7) s’écrit
λ1 x2 + λ2 y 2 + h = 0.


Elle représente une surface cylindrique dont le génératrices sonr parallèles à IR K , et
dont la section dans le plan z = 0 orthogonal à cet axe est une conique, propre ou
dégénérée. On obtient ainsi des cylindres elliptiques ou hyperboliques ou la réunion de
deux plans, ou enfin une surface imaginaire.
3◦ . Deux valeurs propres nulles : λ2 = λ3 = 0, λ1 6= 0.
Nous écrivons alors q1 , donné par (5), comme suit:
µ ¶2
0 0 0 0 α
q1 (x , y , z ) = λ1 x + + 2βy 0 + 2γz 0 + h = 0.
λ1
Nous effectuons ensuite une translation du repère pour amener l’origine O en un point
ω défini par
−→ α− →
Oω = − I.
λ1

− − → − →
Dans le repère (ω, I , J , K ) obtenu, l’équation de Q est

λ1 x2 + 2βy 0 + 2γz 0 + h = 0. (9)

– Si β = γ = 0, l’équation (9) devient λ1 x2 + h = 0, et Q est dégénérée en la réunion


de deux plans parallèles, distincts si λ1 h > 0, et elle n’a aucun point réel si λ1 h > 0.
Enfin Q est dégénérée en deux plans confondus si h = 0.

→ − → − →
– Si γ 2 + β 2 6= 0, alors dans le repère (ω, I 0 , J 0 , K 0 ) défini par

− −
→ −
→ −


−0 →− −
→0 β J + γK −
→0 −γ J + β K
I = I, J = p , K = p .
β2 + γ2 β2 + γ2
L’équation de Q devient
p
λ1 x2 + 2δy + h = 0. avec δ = β2 + γ2.
16 Surfaces

→=−h−
une translation du repère pour amener l’origine ω en un point s défini par −
ωs
→0
J ,

donne à Q l’équation
λ1 x2 + 2δy = 0.


Q est un cylindre parabolique de génératrices parallèles à IRK 0 .

VI. “Prélude” à la théorie générale des surfaces

Soit (A, f ) une surface paramétrée de classe C p (p ≥ 1), où les paramètres sont (u, v).
Un changement de paramètres est un homéomorphisme
ϕ : B −→ A
(B est un domaine de IR2 ) qui donne les paramètres (u, v) en fonction de deux nouveaux
paramètres (u1 , v1 ): ½
u = ξ(u1 , v1 ),
ϕ
v = η(u1 , v1 ).
Si l’on pose g = f ◦ϕ, on obtient une surface paramétrée (B, g) ayant évidemment le
même support que (A, f ). Notons que (B, g) est de classe C p si, et seulement si, ϕ est
de classe C p . Pour être admissible dans la théorie, ce changement de paramètres doit
être un difféomorphisme de classe C p , c’est-à-dire l’application ϕ et sa réciproque ϕ−1
sont toutes deux de classe C p .
Supposons l’application ϕ de classe C p . Son jacobien est
 ∂ξ ∂ξ 
 1 ∂v1 
∆ϕ (u1 , v1 ) = det  ∂u
∂η ∂η 
∂u1 ∂v1
p
et ϕ est un difféomorphisme de classe C , si, et seulement si, son jacobien ∆ϕ ne s’annule
pas sur B.
Définition : – Pour toute surface paramétrée (A, f ) de classe C p (p ≥ 1), un
changement de paramètres ϕ est dit admissible si ϕ est un difféomorphisme de même
classe C p .
– Deux surfaces paramétrées (A, f ) et (B, g) sont dites C p -équivalentes (p ≥ 1), si
elles sont de même classe C p et s’il existe un changement de paramètres admissibles
ϕ : B −→ A tel que g = f ◦ϕ.
La relation “ (A, f ) est C p -équivalent à (B, g)” est une relation d’équivalence sur
l’ensemble des surfaces paramétrées de classe C p . Une classe d’équivalence sera nommée
surface géomètrique de classe C p . Un élément quelconque (A, f ) de la classe est
une représentation paramétrée de la surface géométrique.
“Prélude” à la théorie générale des surfaces 17

Définition : – Soit une surface Σ de classe C 1 , de représentation paramétrée (A, f ).


On dira que le point (u, v) de la surface paramétrée (A, f ) est régulier, si, en ce point,
∂f ∂f
∧ 6= 0.
∂u ∂v
∂f ∂f
En un point régulier, les deux vecteurs et sont linéairement indépendants. Le
∂u ∂v
plan affine passant par m = f (u, v) et engendré par ces deux vecteurs se nomme plan
tangent en m à la surface paramètrée (A, f ). Nous pouvons démontrer, et nous laissons
cette tâche au lecteur, que les notions de point régulier et de plan tangent en un tel
point sont indépendant de la représentation paramétrique admissible choisie pour Σ.

Définition : On appelle Surface réglée toute surface engendrée par une famille de
droites (Dt )t∈I à un paramètre, que l’on nomme génératrices rectilignes de la surface
reglée.
Par exemple, les cylindres, les cônes sont des surfaces réglées. L’équation vectorielle
d’une surface réglée (S) est du type
m = f (t) + ug(t), (S)
f et g étant deux fonctions de classe C 1 sur I à valeurs dans IR3 , et le couple (t, u)
parcourant I × IR.
Pour tot t ∈ I, Dt est la droite passant par f (t) et dirigée par g(t). (si g(t) 6= 0, ce que
l’on supposera dans la suite pour tout t ∈ I).
Le plan tangent en un point régulier m(t, u) est défini par les deux vecteurs g(t) et
f 0 (t) + ug 0 (t). Le point est régulier si ces deux vecteurs ne sont pas colinéaires. La
proposition suivante est évidente:

Théorème VI.1: En tout point régulier d’une surface réglée, le plan tangent contient
la génératrice de ce point.

Cherchons à quelle condition le plan tangent en deux points distincts m1 et m2 d’une


génératrice Dt est le même. Ces deux points correspondent au même t ∈ I et se
distinguent par deux valeurs distinctes u1 et u2 du paramètre u. Les plans tangents P1 et
P2 en m1 et m2 contiennent tous deux g(t) et respectivement les vecteurs f 0 (t) + u1 g 0 (t)
et f 0 (t) + u2 g 0 (t). Pour que P1 = P2 , il faut, et il suffit, que la famille
{g(t), f 0 (t) + u1 g 0 (t), f 0 (t) + u2 g 0 (t)}
soit liée, c’est-à-dire que la famille {g(t), g 0 (t), f 0 (t)} soit liée.par Mais alors, cette
condition étant indépendante de u1 et u2 , si elle est réalisée, le plan tangent sera le
même pour tout les points d’une telle génératrice Dt .
18 Surfaces

Définition : La surface réglée m = f (t)+ug(t) ((t, u) ∈ I ×IR) est dite développable,


si la famille {g(t), g 0 (t), f 0 (t)} est liée pour tout t ∈ I.

Théorème VI.2: Pour toute surface développable, le plan tangent est le même en tout
point régulier d’une même génératrice rectiligne.

Les surfaces cylindriques et coniques sont développables.


Exercices 19

EXERCICES

Exercice .1 Déterminer une équation cartésienne des nappes paramétrées suivantes


et les reconnaı̂tre ( a, b, c sont des réels strictement positifs):

1◦ . x = a sh u cos v , y = b sh u sin v , z = c ch u , u ∈ IR , v ∈ [0, 2π[


2◦ . x = a ch u cos v , y = b ch u sin v , z = c sh u , u ∈ IR , v ∈ [0, 2π[
3◦ . x = a sin u cos v, y = b sin u sin v, z = c cos u, u ∈ [0, 2π[, v ∈ [0, 2π[
4◦ . x = au cos v , y = bu sin v , z = cu , u ∈ IR , v ∈ [0, 2π[

Exercice .2 Soit H l’hélice circulaire définie paramétriquement par

x = cos u, y = sin u, z = u, (u ∈ IR),

et soit S définie par la paramétrisation:

cos u + cos v sin u + sin v u+v


x= , y= , z= , (u, v) ∈ IR2 .
2 2 2

1◦ . Donner une définition géométrique de S à partir de H.


2◦ . Déterminer une équation du plan tangent à S au point M (u, v) lorsque M (u, v)
est un point régulier. Montrer que ce plan a une position limite lorsque v tend vers u
supposé fixé.
3◦ . Montrer que M (x, y, z) ∈ S si, et seulement si,

x sin z = y cos z, x2 + y 2 ≤ 1.

Que peut-on déduire concernant S ?


Exercice .3 Donner l’équation du cylindre dont les génératrices ont la direction
δ(1, 1, 0) et dont une directrice est la courbe Γ:

x(t) = sin t, y(t) = cos t, z(t) = sin t cos t, (t ∈ [0, 2π[).

Exercice .4 Donner l’équation du cône de sommet (0, 0, a) circonscrit à la surface

x3 + y 3 + z 3 − 3xyz − a3 = 0.

Exercice .5 Donner l’équation du cône de sommet (1, 1, 1) circonscrit à la surface

xyz = 1.
20 Surfaces

Exercice .6 Déterminer l’ensemble des sommets des cônes qui contiennent l’ellipse
x2 y2
z = 0, + −1=0
a2 b2
et qui coupent le plan yoz suivant un cercle.
Exercice .7 Soient ∆ la droite d’équations x = 0, y = 0, Q le plan d’équation z = y,
et le point A(a, 0, 0). À toute droite U incluse dans Q et passant par A, on associe la
perpendiculaire commune D à ∆ et U . Déterminer la surface engendrée par la famille
des droites D (conoı̈de Plüker). Déterminer le lieu du point de rencontre M de D et U .
Exercice .8 Déterminer l’équation du cylindre de révolution de rayon a ∈ IR∗+ et
qui a pour axe la droite D d’équations

x + 2y − z + a = 0, 2x − y + z + a = 0.

Exercice .9 Déterminer l’équation de la surface de révolution engendrée par la


rotation de Γ:

x(t) = cos t, y(t) = sin t, z(t) = sin t + cos t (t ∈ [0, 2π[).

en tournant autour de oz. Construire une méridienne.


Exercice .10 Montrer que la surface (Σ) d’équation

x3 + y 3 + z 3 − 3xyz − 1 = 0

est de révolution, et préciser son axe.


Exercice .11 Montrer que l’équation

(x2 − yz)2 + (y 2 − zx)2 + (z 2 − xy)2 = a4

représente une surface de révolution (Σ), et préciser son axe.


Exercice .12 Pour chacune des équations suivantes représentant une quadrique Q
dans un repère orthonormal, déterminer une équation réduite et la nature de Q.

x2 + y 2 + z 2 − 2xy + 2xz + 3x − y + z + 1 = 0,
(x − y)(y − z) + (y − z)(z − x) + (z − x)(x − y) + (x − y) = 0,
x2 + 9y 2 + 4z 2 − 12yz + 4zx − 6xy + 4x − 2y + z + 4 = 0.
Exercice .13 Soit la nappe paramétrée Σ:
cos θ sin θ ch v
x(v, θ) = , y(v, θ) = , z(v, θ) = .
sh v sh v sh v
Montrer qu’il existe une courbe γ tracée sur Σ telle que la tangente à γ en tout point
soit tangente à la sphère x2 + y 2 + z 2 = 1.
Solutions 21

SOLUTIONS

x2 y2 z2
Solution .1 1◦ . Un calcul immédiat montre que + − = −1. C’est l’une
a2 b2 c2
des deux nappes d’un hyperboloı̈de à deux nappes.
x2 y2 z2
2◦ . Un calcul immédiat montre que 2 + 2 − 2 = 1. C’est un hyperboloı̈de à une
a b c
nappe.
x2 y2 z2
3◦ . Un calcul simple montre que 2 + 2 + 2 = 1. C’est un ellipsoı̈de.
a b c
◦ x2 y2 z2
4 . On voit immédiatement que 2 + 2 − 2 = 0. C’est un cône.
a b c

Solution .2 1◦ . Un point M appartient à S si, et seulement si, il est le milieu d’une


corde de H.
2◦ . L’équation du plan Pu,v tangent à S au point M (u, v) est donnée par
 ∂x ∂x 
X − x(u, v) (u, v) (u, v)
 ∂u ∂v 
 ∂y ∂y 
det 
 Y − y(u, v) (u, v) (u, v) 
=0
 ∂u ∂v 
∂z ∂z
Z − z(u, v) (u, v) (u, v)
∂u ∂v
ce qui donne,

u+v u+v u−v u+v u−v


Pu,v : sin( ) X − cos( ) Y + cos( )Z = cos( ).
2 2 2 2 2

Il est immédiat de voir que lorsque v tend vers u, le plan Pu,v tend vers Pu avec

Pu : sin u X − cos u Y + Z = u.

C’est l’équation du plan osculateur de H au point de paramètre u.


3◦ . Notons que M (x, y, z) appartient à S si, et seulement si, il existe (u, v) tel que

u+v u−v u+v u−v u+v


x = cos( ) cos( ), y = sin( ) cos( ), z= .
2 2 2 2 2

ce qui est équivalent à l’existence de (u, v) tel que

u−v u−v u+v


x = cos z cos( ), y = sin z cos( ), z=
2 2 2

d’oú
x sin z = y cos z, x2 + y 2 ≤ 1. (∗)
22 Surfaces

Inversement, si (x, y, z) vérifie (∗), alors il suffit de choisir (u, v) solutions de u + v = z,


y x
u − v = 2Arccos ( ), si sin z 6= 0, ou de u + v = z, u − v = 2Arccos ( ), si
sin z cos z
sin z = 0, pour que M (u, v) soit égal à M (x, y, z). On conclut que

S = {(x, y, z) : x sin z = y cos z, x2 + y 2 ≤ 1}

Remarquons que S et la partie, se trouvant à l’intérieure du cylindre x2 + y 2 ≤ 1,


du conoı̈de droite, de plan directeur (oxy), d’axe (oz) et de courbe directrice l’hélice
circulaire H.

Solution .3 Le point M (X, Y, Z) appartient au cylindre (C) cherché si, et seulement


si, il existe un reel t tel que les deux vecteurs (X − sin t, Y − cos t, Z − sin t cos t) et δ
soient parallèles. Donc

X = sin t + u
M (X, Y, Z) ∈ (C) ⇐⇒ ∃(t, u) ∈ IR2 : Y = cos t + u

Z = sin t cos t
1 1
D’où, l’équation cartésienne correspondante (X − Y )2 + 2Z = 1 avec Z ∈ [− , ].
2 2

Solution .4 Le point M (X, Y, Z) appartient au cône (C) cherché si, et seulement


si, la droite de représentation paramétrée t 7→ (tX, tY, a + t(Z − a)) est tangente à la
surface (S) d’équation x3 + y 3 + z 3 − 3xyz − a3 = 0. Ce qui est équivalent à l’existence
d’une racine double de l’équation (en t):

X 3 t3 + Y 3 t3 + (a + t(Z − a))3 − 3XY (a + t(Z − a))t2 − a3 = 0

qui s’écrit
¡ 3 ¢ ¡ ¢
X + Y 3 + (Z − a)3 − 3XY (Z − a) t3 + 3a (Z − a)2 − XY t2 + 3a2 (Z − a)t = 0.

Soit t = 0 est la racine double de cette équation, ce qui donne le plan d’équation z = a
qui est le plan tangent à (S) au point (0, 0, a), soit la racine double est différante de 0,
c’est donc une racine double de
¡ ¢ ¡ ¢
X 3 + Y 3 + (Z − a)3 − 3XY (Z − a) t2 + 3a (Z − a)2 − XY t + 3a2 (Z − a) = 0.

Cette équation admet une racine double si, et seulement si, le discriminant est nul. Ce
qui donne

(C) : (Z − a)4 − 6XY (Z − a)2 + 4(X 3 + y 3 )(Z − a) − 3X 2 Y 2 = 0, ou Z = a.


Solutions 23

Solution .5 Le point M (X, Y, Z) appartient au cône (C) cherché si, et seulement


si, la droite de représentation paramétrée t 7→ (1 + t(X − 1), 1 + t(Y − 1), 1 + t(Z − 1))
est tangente à la surface (S) d’équation xyz = 1. Ce qui est équivalent à l’existence
d’une racine double de l’équation (en t):

(1 + t(X − 1))(1 + t(Y − 1))(1 + t(Z − 1)) = 1

qui s’écrit

(X − 1)(Y − 1)(Z − 1)t3 + (XY + Y Z + XZ − 2(X + Y + Z) + 3)t2 + (X + Y + Z − 3)t = 0.

Soit t = 0 est la racine double de cette équation, ce qui donne le plan d’équaion
x + y + z = 3 qui est le plan tangent à (S) au point (1, 1, 1), soit la racine double
est différante de 0, c’est donc une racine double de

(X − 1)(Y − 1)(Z − 1)t2 + (XY + Y Z + XZ − 2(X + Y + Z) + 3)t + (X + Y + Z − 3) = 0.

Cette équation admet une racine double si, et seulement si, le discriminant est nul. Ce
qui donne pour (C),
(XY + Y Z + ZX − 2(X + Y + Z) + 3)2 = 4(X − 1)(Y − 1)(Z − 1)(X + Y + Z − 3)
ou X +Y +Z =3

Solution .6 Supposons que S(α, β, γ) est un point de IR3 avec γ 6= 0. Cherchons


le cône CS de sommet S et circonscrit à l’ellipse qui admet la paramétrisation θ 7→
(a cos θ, b sin θ, 0). En effet

X = a cos θ + t(α − a cos θ)
2
M (X, Y, Z) ∈ (CS ) ⇐⇒ ∃(t, u) ∈ IR : Y = b sin θ + t(β − b sin θ)

Z = tγ
Ce qui nous donne l’équation cartésienne suivnte du cône CS ,
(γX − αZ)2 (γY − βZ)2
CS : + = (Z − γ)2 .
a2 b2
L’intersection avec le plan (yoz) s’obtient alors facilement en mettant X = 0, et nous
trouvons µ ¶
α2 β2 2 γ 2 2 2βγ
+ − 1 Z + Y − 2 Y Z + 2γZ − γ 2 = 0.
a2 b2 b2 b
α2 β2 γ2
C’est l’équation d’un cercle si, et seulement si, βγ = 0 et + − 1 = . Mais γ 6= 0
a2 b2 b2
donc
α2 γ2
β = 0, − =1
a2 b2
24 Surfaces

L’ensemble des sommets S(α, β, γ) des cônes qui contiennent l’ellipse d’équation
x2 y2
z = 0, + = 1 et qui coupent le plan (yoz) suivant un cercle est l’hyperbole
a2 b2
α2 γ2
d’équation β = 0, 2 − 2 = 1 auquel il faut enlever les deux points (a, 0, 0) et
a b
(−a, 0, 0).

Solution .7 La famille des droites U qui passent par A(a, 0, 0) et qui sont
incluses dans le plan Q d’équation z = y peut être représentée par (Uλ )λ∈IR avec
Uλ : y = z, x = a + λy.
Le plan d’équation z = 0 est orthogonal à ∆ et le plan d’équation λx + y = 0 est
orthogonal à Uλ alors la droite Dλ , la perpendiculaire commune à ∆ et Uλ , admet une
équation de la forme z = h, λx + y = v avec (h, v) déterminé par la condition que Dλ
−λa
rencontre ∆ et Uλ , ce qui donne v = 0, h = . Donc
1 + λ2

λa
Dλ : λx + y = 0, z=− .
1 + λ2

On conclut, en supprimant λ des deux équations, que l’équation de la surface cherchée


est
(S) : z(x2 + y 2 ) − axy = 0. (Conoı̈de de Plüker)
a aλ aλ
Le point d’intersection M de Dλ avec Uλ est M ( 2
,− 2
,− ). Lorsque λ
1+λ 1+λ 1 + λ2
varie dans IR le point M parcourt une ellipse d’équation

y = z, x2 + y 2 − ax = 0.

Solution .8 La droite D est la droite passant par le point A(−a, a, 2a) et dirigée

→ t
par le vecteur V = (1, −3, −5). Un point M (x, y, z) appartient au cylindre cherché si,

− −
→ −−→
et seulement si, la distance de M à D est a. Ce qui s’écrit a2 || V ||2 = || V ∧ AM ||2 , d’où

35a2 = (3x + y + 2a)2 + (z + 5x + 3a)2 + (3z − 5y − a)2

ou bien

34x2 + 6xy + 10xz + 26y 2 − 30yz + 10z 2 + 42ax − 14ay = 21a2 .


Solutions 25

Solution .9 Remarquons que Γ est engendrée par les cercles C(h, v) : z =


v, x2 + y 2 + z 2 = h avec h et v liés par le fait que le cercle C(h, v) doit rencontrer Γ.
Ce qui se traduit par l’existence de t tel que 2 + 2 sin t cos t = h et v = sin t + cos t, ces
√ √
deux conditions sont équivalentes à v 2 + 1 = h et v ∈ [− 2, 2]. Alors,
√ √
Γ: x2 + y 2 = 1, z ∈ [− 2, 2].
√ √
Une méridienne est le segment de droite joignant (0, 1, − 2) à (0, 1, 2).

Solution .10 Posons P = x + y + z et S = x2 + y 2 + z 2 . Alors


P S =x3 + y 3 + z 3 + xy(y + x) + yz(y + z) + zx(z + x)
=x3 + y 3 + z 3 + xy(P − z) + yz(P − x) + zx(P − x)
=x3 + y 3 + z 3 − 3xyz + P (xy + yz + zx)
1
=x3 + y 3 + z 3 − 3xyz + P (P 2 − S)
2
On conclut que l’équation de la surface considérée (Σ) s’écrit

(x + y + z)3 − 3(x + y + z)(x2 + y 2 + z 2 ) = 2

elle est de la forme f (P, S) = 0 avec P = λ l’équation d’un plan et S = µ l’équation


d’une sphère centrée en (0, 0, 0). Il en résulte que (Σ) est une surface de révolution d’axe
la droite D passant par l’origine et dirigée par le vecteur t (1, 1, 1).

Solution .11 Faisons la même démarche que dans l’exercice précédent. Posons
P = x + y + z et S = x2 + y 2 + z 2 . Notons enfin

F (x, y, z) = (x2 − yz)2 + (y 2 − zx)2 + (z 2 − xy)2 .

Alors,
F (x, y, z) =x4 + y 4 + z 4 + (xy)2 + (yz)2 + (xz)2 − 2xy.xz − 2xy.yz − 2xz.yz
¡ ¢
=S 2 − (xy)2 + (yz)2 + (zx)2 + 2xy.xz − 2xy.yz − 2xz.yz
1
=S 2 − (xy + yz + zx)2 = S 2 − ((x + y + z)2 − x2 − y 2 − z 2 )
4
1 3 1 1
=S 2 − (P 2 − S)2 = S 2 + SP 2 − P 4
4 4 2 4
l’équation de la surface (Σ) s’écrit sous la forme f (S, P ) = 3S 2 + 2SP 2 − P 4 = 4a4 avec
P = λ l’équation d’un plan et S = µ l’équation d’une sphère centrée en (0, 0, 0). Il en
résulte que (Σ) est une surface de révolution d’axe la droite D passant par l’origine et
dirigée par le vecteur t (1, 1, 1).
26 Surfaces

Solution .12 Dans chacun des trois cas notons A la matrice dans la base canonique

− − → − →
( i , j , k ) de IR3 , de la forme quadratique définie par la partie homogène de degré 2
de l’équation de (Q).
a. Étude de F (x, y, z) = x2 + y 2 + z 2 − 2xy + 2xz + 3x − y + z + 1 = 0.
La matrice A est donnée par
 
1 −1 1
A =  −1 1 0.
1 0 1

Le polynôme caractéristique de A est


√ √
XA (λ) = −(λ − 1)(λ − 1 − 2)(λ − 1 + 2).

Comme 0 n’est pas une valeur propre de A alors Q est une quadrique à centre. Le centre
C est obtenu en résolvant le système d’équations

∂F
(x, y, z) =2x − 2y + 2z + 3 = 0
∂x
∂F
(x, y, z) = − 2x + 2y − 1 = 0
∂y
∂F
(x, y, z) =2x + 2z + 1 = 0
∂z
1
ce qui donne C( , 1, −1).
2

→ → − − →
L’équation de Q dans le repère R1 = (C; i , j , k ) devient

3
X 2 + Y 2 + Z 2 − 2XY + 2XZ + = 0.
4

La matrice symétrique A se diagonalise dans le groupe orthogonal et, tout calcul fait,
nous obtenons A = P DP −1 où,
√  1 1 
  √
1− 2 0 0 −√ 0
 2 2 
 √   1 1 1 
   √ 
D= 0 1+ 2 0, P = .
   2 2 2
 1 1 1 
0 0 1 − − √
2 2 2

→ − → − → −
→ −→ → −
L’équation de Q dans le nouveau repère orthonormé R2 = (C; I , J , K ), (où I , J , K
sont les vecteurs colonnes de P


→ 1 −→ 1−→ 1→− −
→ 1 −→ 1−→ 1−→ −
→ 1 −→ 1 −→
I = √ i + j − k, J = −√ i + j − k , K = √ j + √ k .)
2 2 2 2 2 2 2 2
Solutions 27

s’écrit sous la forme réduite


√ √ 3
(1 − 2)α2 + (1 + 2)β 2 + γ 2 + =0
4

α2 β2 γ2
On conclut que Q admet l’équation réduite − − = 1 avec
a2 b2 c2
√ √
2 3(1 + 2) 2 3( 2 − 1) 3
a = , b = , c2 = .
4 4 4

Q est un hyperboloı̈de à deux nappes.


b. Étude de F (x, y, z) = (x − y)(y − z) + (y − z)(z − x) + (z − x)(x − y) + x − y = 0.
Notons d’abord que

F (x, y, z) = x2 + y 2 + z 2 − xy − xz − yz + y − x.

La matrice A est donnée par


 
2 −1 −1
1
A =  −1 2 −1  .
2
−1 −1 2

3
Le polynôme caractéristique de A est XA (λ) = −λ(λ − )2 . Comme 0 est une valeur
2
propre de A alors Q n’est pas une quadrique à centre.
La matrice symétrique A se diagonalise dans le groupe orthogonal et nous obtenons
A = P DP −1 où,
3   1 1 1 
0 0 √ √ √
 3
2


  12 1
6
1 
3  √ 
D=
0

0, P =  −√ √ .
 2   2 6 3
 2 1 
0 0 0 0 −√ √
6 3

→ − → → − −
→ →− − →
En considérant le repère R1 = (O; I , J , K ), où I , J , K sont les vecteurs colonnes de
P:


→ 1 →− −→ −
→ 1 −→ −→ →
− →
− 1 −→ −→ → −
I = √ ( i − j ), J = − √ ( i + j − 2 k ), K = √ ( i + j + k ).
2 6 3

3 2 3 2 √
L’équation de Q s’écrit dans ce nouveau repère sous la forme X + Y − 2X = 0,
2 2
ou encore √
9 2 2 9 2
(X − ) + Y = 1.
2 3 2
28 Surfaces

−−→ 2→

En posant enfin C(1/3, −1/3, 0) de telle manière que OC = I et en prenant pour
3

→ −→ →− 9
nouveau repère R2 = (C; I , J , K ), l’équation de Q devient (α2 + β 2 ) = 1. Q est
2 √

→ 2
alors le cylindre de révolution d’axe passant par C et dirigé par K , et de rayon .
3
c. Étude de F (x, y, z) = x2 + 9y 2 + 4z 2 − 6xy + 4xz − 12yz + 4x − 2y + z + 4 = 0.
La matrice A est donnée par
 
1 −3 2
A =  −3 9 −6  .
2 −6 4

Le polynôme caractéristique de A est XA (λ) = −λ2 (λ − 14). Comme 0 est une valeur
propre de A alors Q n’est pas une quadrique à centre.
La matrice symétrique A se diagonalise dans le groupe orthogonal et nous obtenons
A = P DP −1 où,
 1 1 22 
 
14 0 0 −√ √ − √
 14 5 6 5 21 
   3 7 4 
   √ √ − √ 
D= 0 0 0, P = .
   14 5 6 5 21 
 2 2 1 
0 0 0 −√ √ √
14 6 21

→ − → → − −
→ →− − →
En considérant le repère R1 = (O; I , J , K ), où I , J , K sont les vecteurs colonnes de
P:

→ 1 −
→ →
− −

I = √ (− i + 3 j − 2 k ),
14

→ 1 − → −
→ →

J = − √ ( i + 7 j + 10 k ),
5 6

→ 1 −
→ →
− −

K = √ (−22 i − 4 j + 5 k ).
5 21
L’équation de Q s’écrit dans ce nouveau repère sous la forme
12 15
14X 2 − √ X − √ Z + 4 = 0
14 21
ou encore µ ¶2 Ã √ !
3 15 187 21
X− √ − √ Z− = 0.
7 14 14 21 735

8453 821 38 −−→ 3 − → 187 21 → −
En posant C( ,− ,− ), de telle manière que OC = √ I + K
7350 7350 735 7 14 735

→ → − −→
et en prenant pour nouveau repère R2 = (C; I , J , K ), l’équation de Q devient
15
α2 − √ β = 0. Q est alors un cylindre parabolique.
14 21
Solutions 29

Solution .13 Notons


   cos θ 
x(v, θ)
   sh v 
   sin θ 
F : IR × IR −→ IR : (v, θ) 7→  y(v, θ)  = 
∗ 3 .
    sh v 

ch v
z(v, θ)
sh v
Pour toute courbe γ de classe C 1 tracée sur Σ, il existe Ψ : I −→ IR∗ × IR : t 7→
(v(t), θ(t)), définie sur un intervalle de IR, de classe C 1 , telle que la courbe γ soit
représentée par t 7→ M (t) = F ◦Ψ(t).
Remarquons que

∂x ∂y ∂z ∂x ∂y
= −xz, = −yz, = 1 − z2, = −y, = x.
∂v ∂v ∂v ∂θ ∂θ

Alors, en notant x, y, z pour x◦Ψ(t), y ◦Ψ(t) et z ◦Ψ(t),


 
−xzv 0 (t) − yθ0 (t)
 
0  0 0 
M (t) =  −yzv (t) + xθ (t)  .
 
(1 − z 2 )v 0 (t)

La droite D(t) tangente à γ au point M (t) est la droite passant par M (t) et dirigée
par M 0 (t). Cette droite est aussi tangente à la sphère (S) d’équation cartésienne
x2 + y 2 + z 2 = 1 si, et seulement si, la distance de l’origine O(0, 0, 0) à D(t) vaut
k M (t) ∧ M 0 (t) k
1. Mais d(O, D(t)) = . On conclut que D(t) est tangente à (S) si, et
k M 0 (t) k
seulement si,
k M (t) ∧ M 0 (t) k = k M 0 (t) k (∗)

Mais,
 
−yv 0 (t) + xzθ0 (t)
 
0  
M (t) ∧ M (t) =  xv 0 (t) + yzθ0 (t) 
 
(1 − z 2 )θ0 (t)
Alors (∗) est équivalente à

(−yv 0 (t) + xzθ0 (t))2 + (xv 0 (t) + yzθ0 (t))2 + (1 − z 2 )2 θ02 (t) =
(−xzv 0 (t) − yθ0 (t))2 + (−yzv 0 (t) + xθ0 (t))2 + (1 − z 2 )2 v 02 (t).

ce qui est équivalent à θ02 (t) = v 02 (t). Les solutions sont donc v = c + θ ou v = c − θ
où c est une constante. Comme nous cherchons seulement une solution, nous pouvons
30 Surfaces
View publication stats

prendre pour γ la courbe représentée paramétriquement par


 
cos θ
 sh θ 
 
 sin θ 
M : IR+ −→ IR , θ 7→ 
∗ 3
 sh θ
.

 
 ch θ 
sh θ

OKMRAN
OUBA

Vous aimerez peut-être aussi